Download as pdf or txt
Download as pdf or txt
You are on page 1of 384

AL ish

GU g l
LIN n
BI E
h&

C L A S S E S
fgan

TOPPER’S HANDBOOK
FOR
QUICK
REVISION

HIGHLY USEFUL FOR :


SSC-CGL, CPO, CHSL, STENO, MTS, GD, JEN,
RRB NTPC, GROUP D & Other Competitive Exams.

Best Concepts
Thought Process
Elimination Methods

FOR VIDEO SOLUTIONS : Raja Sir


Join Neon Classes Income Tax Inspector
17 Years’ teaching experience
YouTube channels.
CAT GMAT UPSC SSC BANK
Copyright © Neon Classes
All right reserved. No part of this book may be reproduced, stored in or introduced
into a retrieval system, or transmitted, in any form, or by any means (electronic,
mechanical, photocopying, recording or otherwise) without the prior written
permission of the author. Any person who does any unauthorised act in relation to
this production may be liable to criminal prosecution and civil claims for damages.

Rs.700/-
13 17

13
Roma Invicta Arya
Best App for Govt. Jobs : Neonclasses (Download Now)

1. Alphabet Test ;fn ds vUnj nksuksa ck,¡ ;k nksuksa nk,¡ gS rc ge la[;k,¡ ?kVk nsras gS
vkSj vyx&vyx vkus ij tkM+ nsrs gSa vkSj gekjk LFkku tks igys
 Concept - fn;k gS ogk¡ ls gksrk gS tSls bl ls le>us dh dksf’k’k djrs gSaA
vaxzsth o.kZekyk esa 26 v{kj gksrs gSA izFke v)kZ’k esa 13 v{kj gksrs gSa Type:- 1
Which letter will be 6th to the right of 4th from left end in the
tks A ls M gSA ckfd 13 v{kj N ls Z rd f}rh; v)kZ’k esa gksaxsA
English alphabet series?
A ls Z ¼ck,¡ ls nk;k¡½=> vxzxkeh
When we look at this type of question, there are 4th ways in
Z ls A ¼ck,¡ ls nk;k¡½=> vxzxkeh
which the left and right can be placed in the question.
vFkkZr~ ck,¡ Nksj ls igyk v{kj % A Right, Right = - Right
nk,¡ Nksj ls igyk v{kj % Z Left, Left = - Left
bl v/;k; ds vUnj ge o.kZekyk ij vk/kkfjr J`a[kykvksa dk v/;;u Right, Left = + Left
djrs gSa tks fd dqN bl izdkj gSa%& Left, Right = + Right
There are 26 letters in the English alphabetical serise 13 are If there are both left and both right position in a question, then
in the first half. Which are from A to M and other 13 are in the we difference and if both positions are different, then we will
second half, which are from N to Z. add them. Finally our answer will be from that end which is
A to Z (left to right) ⇒ Forward order given at last in the question.
Z to A (right to left to) ⇒ Reverse order 1- vaxzsth o.kZekyk esa dkSu lk v{kj ck,¡ ls 5 ds nk,¡ 6 gksxk\
i.e. 1st letter from left end: A Which letter will be 6th to the right of 5th from left end in the
1st letter from right end: Z English alphabet?
If we rembere the sequence of English letters, we will be able (a) K (b) G (c) R (d) O
to solve these types of questions very quicly: Sol. (a)
1. A,B,C,………….X,Y,Z bues igys ck,¡ gS vkSj ckn esa nk,¡ vFkkZr~ nksuksa vyx&vyx gS] rks
2. Z,Y,X,…………...C,B,A ge tksM+ nsxsa vkSj gekjk mÙkj igys vFkkZr~ ck,¡ ls gksxkA
3. B,D,F,…………... V, X, Z, (Even series) It has given first right and then left which means both are
4. A,C,E,……………... U, W, Y (Odd series) different positions, therefore we will add them. And our
5. A________MZ________N answer will be form the last positions which is left end in the
6. M________AN________Z question.
7. A________MZ________N Sol. (a) A, B, C, D, E, F, G, H, I, J ____ Z
8. ,YQkU;wesfjd J`a[kyk Left=5
Right = 6
Alphanumeric series 2-
Then Left = 5+6=11(K)
vaxzsth o.kZekyk esa dkSu lk v{kj ck,¡ ls 14 osa ds ck,¡ 5 ok¡ gksxk \
;fn ge bl v/;k; esa fdlh Hkh dk mÙkj dqN gh lsd.M esa nsuk
Which letter will be 5th to the left of 14th from left end in
pkgrs gSa rks lcls igys gesa o.kZekyk esa gj v{kj ds LFkku dks ;kn
English alphabetical series?
djuk iM+sxkA bl v/;k; esa nks izdkj ds iz’k curs gSa tks fd dqN (a) I (b) J (c) R (d) L
bl izdkj gSa Sol. (a)
If we want to answer the questions in this chapter within a Left 14
few seconds, then first we have to remember the place value Left = 5
off every letter in the English alphabet. ;gk¡ ij nksuksa ck,¡ gS vFkkZr~ leku gSa blfy, ge la[;k,¡ ?kVk nsxsa
There are two types of questions in this chapter which are as
Here both are left positions i.e. same positions, hence find the
follows
difference between the two.
1- vaxzsth o.kZekyk esa dkSu lk v{kj ck,¡ ls lkrosa ds nk,¡ rhljk gksxk\ Left 14 / Left = 5
Which letter will be 3rd to the right of 7th letter from the left
ck,¡&ck,¡ ¾ 09/ Left –Left = 14-5 = 09
end in English alphabet?
ck,¡ ls ¾ 09 ¼I½ mÙkj
2- vaxzsth o.kZekyk esa dkSu lk v{kj ck,¡ ls pkSFks vkSj ck,¡ ls 10 osa v{kj
From left end = 09 (I)
ds Bhd chp esa gksxk\
3- vaxzsth o.kZekyk esa dkSu lk v{kj ck,¡ ls 9 osa ds ck,¡ 19 ok¡ gksxk\
Which letter will be exactly between the 4th letter from the
Which letter will be 9th to the left of 19th from the left end in
left and the 10th letter from right in English alphabetical
English alphabetical series?
series?
(a) S (b) G (c) J (d) O
 Concept - I Sol: (c)
When given series A B C ……. XYZ. Left =9
vaxzsth o.kZekyk esa dkSu lk v{kj ck,¡ ls 4 ds nk,¡ 6 gksxk\ Left = 19
tc ge bl izdkj ds dks ns[krs gSa rks ;gk¡ ij pkj rjhds ls ds vanj Left= 19-9=10(J)
ck,¡ vkSj nk,¡ dk LFkku vk ldrk gSA 4- vaxzsth o.kZekyk esa dkSu lk v{kj ck,¡ ls 6 osa ds ck,¡ 16 ok¡ gksxk\
ck,¡] ck,¡ = - ck,¡ Which letter will be 16th to the left of 6th from the left end in
ck,¡] nk,¡ = + ck,¡ English alphabetical series?
nk,¡] ck,¡ = + nk,¡ (a) S (b) O (c) P (d) Q
Sol: (c)
nk,¡] nk,¡ = - nk,¡

1
Download Free PDFs & e-Books from Neon Classes App

बाएँ = 6 Left = 6 दाएँ = 4 Right = 4


=
बाएँ =16 Left =16 =
दाएँ −7 Right 7

बाएँ=−10 Left =−10
दाएँ=−3 Right =−3

uksV % vxj la[;k _.kkRed gS rc ge bldksa 26 esa ls ?kVk nsxsa ck,¡¾ 27&¼&3½ ¾30
Note: If the positions value from left end is negative then Left = 27-30
subtract it from 26. Left = -3
ck,¡ ¾ &10 / left = - 10 Left = 26 – 3 = 23 (W) Ans.
ck,¡ ¾ 26&10 ¾ 16 ¼P½ mÙkj
 Concept - II
From left end = 26 – 10 = 16 (P) Ans.
X, Y, Z A, B, C, D ------- When questions asked midway (Bhd chp okys iz’u)
&2] &1] 0] 1] 2] 3] 4 8- vaxzsth o.kZekyk esa dkSu lk v{kj ck,¡ ls 5 vkSj ck,¡ ls 17 osa v{kj
5- vaxzsth o.kZekyk esa dkSu lk v{kj nk,¡ ls 3osa ds ck,¡ 8 gksxk\ ds Bhd chp esa gksxk\
Which letter will be 8th to the left 3th from the right end in Which letter will be exactly between the 5th letter from left
English alphabetical series? end and 17th letter from left end in English alphabetical
(a) S (b) P (c) R (d) O series?
Sol. (b) (a) J (b) K (c) R (d) O
दाएँ = 3 Right = 3 Sol. (b) ck,¡ = 5(E)
= = ck,¡ = 17(Q)
बाएँ 8 Left 8
+
+
दाएँ=−11 Rig ht=−11 left = 5(E)
Left = 17 (Q)
uksV %v{kj dk LFkku nk,¡ ls irk gksus ij ml LFkku dks ck,a esa cny DEFGHIJKLMNOPQR
ns] ftrus v{kj gSa mlls ,d T;knk esa ls ?kVkdj uksV% tc Hkh ge Bhd chp esa v{kj irk djrs gSa rks ge nksuksa
Note: if we know the position value of a letter from the right la[;kvksa dk vkSlr fudky ysxa ysfdu mlds fy, nksuksa la[;k,¡
end. Then convert it into the left end. For this subtract the leku Nksj ls gks vFkkZr~ ;k rks nksuksa ck,¡ ls ;k nksuksa nk,¡ ls
position value from the (Total letters +1). Note: Whenever we find the exact middle letter, we will
extract the average of both the numbers but for that they
26 letters
A B should be from the same end i.e. either both from the left or
Left 11 Right both from the right
? ck,¡ ¾ 5
ck,¡ ¾ 17

nk,¡ ¾ 11 / Right ck,¡ = ⇒11(K) mÙkj


ok,¡ ¾ 27&11 ¾ 16 ¼P½ mÙkj
From left end = 27 – 11 = 16 (P) Ans.
6- vaxzsth o.kZekyk esa dkSu lk v{kj nk,¡ ls 15 osa ds nk,¡ 5 ok¡ gksxk\ Left = 5
Which letter will be 5th to the right of 15th from the right end Ledt = 17
in English alphabetical series?
(a) Q (b) H (c) R (d) P Left = ⇒11(K) Ans.
Sol: (a)
Right = 15
दाएँ = 15
दाएँ =5 Right =5
− 9- vaxzsth o.kZekyk esa dkSu lk v{kj ck,¡ ls 6 vkSj ck,¡ ls 18 osa v{kj

दाएँ =10 Right =10
ds Bhd chp esa gksxk\
Which letter will be exactly between the 6th letter from left
ok,¡ ¾ 27&10 ¾ 17 ¼Q½ mÙkj
end and 18th letter from left end in English alphabetical
From left end = 27-10 = 17(Q) Ans
series?
A,B Y,Z (a) S (b) O (c) L (d) N
Left Right Sol. (c)
10 Left = 6
(Q) Left = 18

7- vaxzsth o.kZekyk esa dkSu lk v{kj nk,¡ ls 4 ds nk,¡ 7ok¡ gksxk \ Left = =12(L)
Which letter will be 7th to the right 4th from the right end in
English alphabetical series? 10- vaxzsth o.kZekyk esa dkSu lk v{kj nk,¡ ls 4 vkSj nk,¡ ls 16 osa v{kj
(a) S (b) E (c) W (d) M ds Bhd chp esa gksxk\
Sol. (c) Which letter will be exactly between the 4th letter from right
end and 16th letter from right end in English alphabetical
series?
(a) S (b) G (c) R (d) Q
Sol: (d)

2
Best App for Govt. Jobs : Neonclasses (Download Now)

nk,¡ ¾ 4
nk,¡ ¾ 16
Left = 3
nk,¡ = =10 Left = 18

Right = 4
Right = 16 Left 10, 11 (J,K)
 Concept - III
Right = =10
When more than two position are given.
13- vaxzsth o.kZekyk esa dkSu lk v{kj ck,¡ ls 7 ds nk,¡ 5 ds ck,a 5
gksxk\
nk,¡ ¾ 10 Which letter is 5th to the left of 5th to the right of 7th from the
ck,¡ ¾ 27&10 = (Q) mÙkj left end?
Right = 10 (a) S (b) G (c) R (d) O
Left = 27 – 10 = (Q) Ans. Sol: (b) bl ç'u esa gesa nks ls T;knk LFkku fn, x, gSa ,sls ç'uksa dks gy
11- vaxzsth o.kZekyk esa dkSu lk v{kj ck,¡ ls rhljs vkSj nk,¡ ls pkSFks djus ds fy, lcls igys gesa igyk LFkku ysuk gksxkA
v{kj ds Bhd chp esa gksxk\ bl igys LFkku esa foijhr vkus okys LFkkuksa dks tksM+ nsaxs rFkk leku
Which letter will be exactly between the 3rd letter from left vkus okyks dks ?kVk nsaxsA tSls bl ç'u esa gekjk igyk LFkku gS ok;-
end and 4th letter from right end in English alphabetical blesa ge lkjs ck,¡ ?kVk nsaxs rFkk lkjs nk,¡ tksM+ nsaxsA
series?
In this question we are given more than 2 positions. To solve
(a) M (b) J (c) R (d) O
such questions, first of all we have to take last position.
Sol: (a)
In this last position, we will add all the oppositions and will
ck,¡ ¾ 3 subtract the correspondings same positions. As the last
nk,¡ ¾ 4 position in this question is from left end, from this we will
Left = 3 subtract all the left position and add all the right positions in it.
Right = 4 Left = 7+5-5=7 (G) Ans.
uksV% ge vkSlr rHkh fudky ldrs gS tc ;g nksuksa leku Nksj ls gks 14- vaxzsth o.kZekyk esa dkSulk v{kj ck,a ls 7 osa ds nk,¡ 6 ds nk,a 4
vr% igys budks leku Nksj ls cuk,¡xsA gksxk\
Note: We can extract the average only when both letters are Which letter is 4th to the right of 6th to the right of 7th from
from the same end. Therefore convert them into same ends. the left end in English alphabetical series?
ck,¡ ¾ 3 % ck,¡ ¾ 3 (a) X (b) V (c) R (d) Q
nk,¡ ¾ 4 % ck,¡ ¾ 27 & 4 ¾ 23 Sol: (d) ck,¡ = 7+6+4 = 17 ¼Q½ mÙkj
Left = 3 : Left = 3 Left = 7+6+4 = 17 (Q) Ans.
Right = 4 : Left = 27-4=23 15- vaxzsth o.kZekyk esa dkSulk v{kj nk,¡ ls 11 os ds nk,¡ 7 ds ck,¡ 5 os
ds ck,¡ 4 gksxk\
ck,¡ = =13(M) mÙkj
Which letter is 4th to the left of 5th to the left of 7th to the right
of 11th from the right end in English alphabetical series?
(a) X (b) N (c) R (d) O
Sol: (b) nk,¡ 11-7+5+4 = 13
Left = =13(M) Ans. vc gesa bls ck,¡ esa cnyuk gksxkA
ck;k¡ ¼26+1½&13 = 14 ¼N½ mÙkj
Right = 11-7+5+4 = 13
12- vaxzsth o.kZekyk esa dkSu lk v{kj ck,¡ ls 3 vkSj ls ck,¡ 18osa v{kj ds Now we will convert it into the left end.
Bhd chp esa gksxk\ Left = (26-1)-13 = 14 (N) Ans.
Which letter will be exactly between the 3rd letter from left  Concept - IV
end and 18th letter from left end in English alphabetical
series? ZYX______CBA
(a) J,K (b) L,K (c) J,L (d) J To solve this series, we convert all right positions into left and
Sol: (a) ck,¡ ¾ 3 all left positions into right.
ck,¡ ¾ 18 bl lhfjt dks gy djus ds fy, gesa lHkh nkbZ LFkfr dks cka;h vkSj
Left-3
lHkh cka;h dks nk;h esa cnyuk iMsxkA
Left- 18 16- ;fn vaxzsth o.kZekyk dks foijhr Øe esa fy[kk tk, rks dkSulk v{kj
uksV% tc vkSlr n’keyo esa fudys rks ,d ugh nks v{kj Bhd chp esa nk,¡ ls 4 ds ck,¡ 5 gksxk\
gksrs gS ,d ckj la[;k esa ,d de djds vk/kk vkSj ,d ckj la[;k esa If English alphabetical series is written in the reverse order
,d T;knk djds vk/kk dj nsA then which letter will be 5th to the left of 4th from the right
Note:When the average turns out to be decimal, but two letters end?
(a) I (b) V (c) R (d) O
are exactly in the middle. For letter add 1 into the sum and
Sol: (a) ∵ o.kZekyk foijhr Øe esa gS blfy,
then do the half Similary for the second letter, subtract 1 from
sum and then do the half. ck;k¡ → nk;k¡

3
Download Free PDFs & e-Books from Neon Classes App

nk;k¡ → ck;k¡  Concept - V


vc ge igys dh rjg gh ç'u dks gy djsaxsA
ck,¡ = 4 BDF..........VXZ (Even series)
nk;k¡ ¾ 5 This is called the even series because this series has all the
even numbered position letters. It consists of 13 letters.
ck;k¡ ¾ 4 $ 5 ¾ 9 ¼l½ mÙkj
According to this series the value of B,D,F...V,X,Z will be
Since the series is in the reverse order. Therefore
respectively 1,2,3.....11,12,13.
Left → Right
While in A to Z series
Right → Left
A B C D E F G H ........ X Y Z
Now we will solve the question as before.
2 4 6 8 24 26
Right = 5
On comparing the two series, we find that to obtain the
Left=4
original A to Z series, we have to double the position obtained
Left=5+4=9 (I) Ans.
but keep in mind that 2 times will always be done from the
17- ;fn vaxzsth o.kZekyk dks foijhr Øe esa fy[kk tk, rks dkSu lk v{kj
left end.
ck,¡ ls 3 ds nk,¡ 7 gksxk\ 21- ;fn A ls 'kq: djrs gq, çR;sd nwljk v{kj gVk fn;k tk, rc dkSu
If English alphabetical series is written in the reverse order
lk v{kj ck,¡ ls 4osa ds nk,¡ 6 gksxk\
then which letter is 7th to the right of 3rd from the left end?
If every alternative letter starting from A is deleted then
(a) X (b) V (c) R (d) Q
which letter will be 6th to the right of 4th from the left end?
Sol: (d) ∵ o.kZekyk foijhr Øe esa gS blfy,
(a) X (b) V (c) T (d) O
ck;k¡ → nk;k¡ Sol: (c)
nk;k¡ → ck;k¡ ck;k¡ ¾4 nk;k¡+=6
nk,¡ ¾ 3 nk;k¡ ¾ 4$6 ¾ 10
ck,¡ ¾ 7 Right = 6
nk,¡ ¾ 3+7 ¾ 10 Left = 4
blfy, ck,¡ ¼26+1½- 10=17¼Q½ mÙkj Left 4+6=10
Since the series is in the reverse order. Therefore left is Right ewy Js.kh (A-Z) ds vuqlkj 10×2 = 20 ¼T½ mÙkj
& Right is Left According to the original series (A-Z) = 10×2 = 20 (T) Ans.
Left=7 22- ;fn A ls 'kq: djrs gq, çR;sd nwljk v{kj gVk fn;k tk, rc dkSu
Right=3 lk v{kj nk,¡ ls 10 osa ds nk,¡ 4 gksxk\
Right=7+3 =10 If every alternative letter starting from A is deleted then
Hence Left = (26+1)-10=17 (Q) Ans. which letter will be 5th to the right of 5th from the left end?
18- ;fn vaxzsth o.kZekyk dks foijhr Øe esa fy[kk tk, rks dkSu lk v{kj (a) P (b) V (c) R (d) O
ck,¡ 3 ds nk,¡ 5 ds ck,¡ 2 gksxk \ Sol: (a) nk,¡ ¾4
If English alphabetical series is written in the re-verse order ck,¡ = 10
then which letter will be 3rd to the left of 5th to the right of ck,¡ ¾ 10–4 ¾ 6
3rd from the left end?
ck,¡ ¼ 13+1½&6 = 8
(a) T (b) U (c) V (d) O
ewy Js.kh ds vuqlkj 8×2 = 16 ¼P½ mÙkj
Sol:• (c)
Right = 4
∵ o.kZekyk foijhr Øe esa gS blfy,
Left = 10
ck;k¡ → nk;k¡
Left = 10-4 =6
nk;k¡ → ck;k¡ Left = (13 + 1)-6=8
nk,¡ ¾ 3++5&3 ¾ 5 According to the original series (A-Z) = 8×2 = 16 (P) Ans.
ck,¡ ¾ ¼26+1½&5 ¾ 22 ¼V½ mÙkj
 Concept - VI
Sol: Since the series is in the reverse order. Therefore
Left → Right A C E G ------- U W Y (Odd seires)
Right → Left This is called the odd series because it has all the odd
Right = 3+5-3 = 5 numbered position letters. It also has 13 letters.
Left = (26+1)-5 = 22 (V) Ans. A C E G ------- U W Y
19. ;fn vaxzsth o.kZekyk ds v{kjksa dks mYVs Øe esa fy[kk tkrk gS rks 1234 11 12 13
dkSu lk v{kj nk;sa ls 12osa v{kj ds nk;sa ls 5oka gS\ While in A to Z series.
If letters of the English alphabet are written in reverse order A B C D E F G H ………. W X Y Z
then which letter is 5th to the right of 12th letter from the 1357 23 25
right? On comparing the two series, we find that to obtain the
(a) X (b) G (c) R (d) O original A to Z series, we have to double the position obtained
7th and then subtract 1. But keep in mind that all calculations will
Sol. Z Y X_ _ _ L _ _ _ G C B A Right be made from left end.

23- ;fn B ls 'kq: djrs gq, çR;sd nwljk v{kj gVk fn;k tk, rks QkS
Right to Right = 12–5 =7 v{kj ck,¡ ls 5 ds nk,¡ 6 gksxk\
Right to 7th position = G If every alternative letter starting from B is deleted then
Option (b) is correct which letter will be 5th to the right of 4th from the left end?

4
Best App for Govt. Jobs : Neonclasses (Download Now)

(a) X (b) U (c) R (d) O If first half and second half both are written in the reverse
Sol: (b) A B C D E F G H ……… X Y Z order respectively then which letter will be 10th to the left of
ç'u ds vuqlkj ges B ls 'kq: djrs gq, v{kjks dks g;uk blfy, B, 10th from the right end?
D, F, H -------- X, Z vkfn gVk, x, gSA bl rjg çIr Js.kh fo"ke (a) S (b) G (c) H (d) P
Js.kh gSA Sol. (b) tc ge igyk v)kZa'k foijhr Øe esa fy[ksaxs rks M ls A çkIr
According to question we have to start deleting letter from B, a s rks Z ls N çkIr
gksxk rFkk tc f}rh; v)ka'k foijhr Øe esa fy[ksx
therefore B, D, F, H ……… X, Z etc. deleted. Thus we will get an gksxkA bl rjg gesa fuEu J`a[kyk çkIr gksxh&
odd series. We will get M to A when we write the first half in the reverse
6 8 order and we will get Z to N when we write the second half in
1 4
the reverse order. Thus we will obtain the following series:
×2-1A ×2-1C ×2-1E ×2-1G ……(fo"ke) M L ------- B A Z Y -------- O N
ck,¡ Nksj ls igyk v{kj M gksxk rFkk nk,¡ Nksj ls igyk v{kj N
1 2 3 4
gksxkA vc bl ç'u dks fuEu rjhds ls gy fd;k tk,xkA
The first letter from the left end is M and the first letter from
1 4 6 8 the right end will be N. Now we will solve the question in the
following way:
×2-1A ×2-1C ×2-1E ×2-1G ……(Odd) nk,¡ ¾ 10
1 3 4 ck,¡ ¾10
2
nk,¡ ¾ 10+10 ¾ 20
gesa v{kjksa dk ewy LFkku çkIr djus ds fy, ×2-1 djuk gksxkA Left = 10
Right = 10
To get original position of letters, we have to double the
Right = 10 + 10 = 20
position obtained and then subtract 1 i.e.
Right = 6
nk,¡ ls 20 ok¡ v{kj Kkr djrs le; 13 v{kj gekjs f}rh; v)ka'k esa
Left = 5 vk tk,x ckfd cps 20 - 13 = 7 ok¡ v{kj A ls ns[ksaxsA A ls 7 ok¡
Left = 6 + 5 = 11 v{kj ¼G½ mÙkj
According to the original series (A –Z)=11×2-1=21(U) Ans. While finding the 20th letters from the right end, the 13 letters
24- ;fn B ls 'kq: djrs gq, çR;sd nwljk v{kj gVk fn;k tk, oks dkSu will come in the second hand. The remaining (20-13)=7th letter
lk v{kj ck,¡ ls 10osa ds ck,¡ 5 gksxk\ will be seen from A. now 7th letter from A = (G) Ans.
If every alternative letter starting from B is deleted then 7 13
which letter will be 5th to the left of 10th from the left end? M----------A, Z----------N
(a) D (b) I (c) R (d) O 20
Sol: (b) ck,¡ ¾ 10 27- ;fn çFke v)kZa'k o f}rh; v)ka'k nksuks Øe'k% foijhr Øe esa fy[ks
ck,¡ 5 vk,dks dkSu lk v{kj nk,¡ ls 16osa ds nk,¡ 7 gksxk\
ck,¡ ¾ 10 & 5 ¾ 5 If first half and second half both are written in the reverse
ewy Js.kh ¼A-Z½ ds vuqlkj 5×2&1¾9 ¼I½ mÙkj order respectively then which letter will be 7th to the right of
Right = 5 16th from the right end?
Left = 10 (a) X (b) V (c) R (d) O
Left = 10-5=5 Sol: (b)
According to the original series (A –Z)=5×2-1=9(I) Ans. nk,¡ ¾7
25- ;fn B ls 'kq: djrs gq, izR;sd nwljk v{kj gVk fn;k tk, rks nk,¡ ¾ 16
dkSulk v{kj nk,¡ ls 10 ds nk,¡ 8 gksxk \ nk,¡ ¾16&7 ¾ 9
If every alternative letter starting from B is deleted then Right = 7
which letter will be 8th to the right of 10th from the right Right = 16
end? Right = 16-7=9
(a) Z (b) M (c) W (d) Q 9
Sol: (c) nk,¡ ¾ 10 M-----------A, Z--------N
nk,¡ ¾ 8 bls ewy Js.kh esa ge bl rjg fn[kk ldrs gSA
nk,¡ ¾ 10 & 8 ¾ 2 We can show it as follows in the original; series:
ck;k¡ ¾¼13 $ 1½ & 2 ¾ 12 M----------A, Z----------N
ewy Js.kh ¼A-Z½ ds vuqlkj 12×2&1¾23 ¼W½ mÙkj 13 9
Right = 8
bl rjg ges A ls tks v{kj pkfg, og 13 $ 9 ¾ 22ok¡ gksxkA
Left = 10
A ls 22ok¡ v{kj ¾ ¼V½ mÙkj
Left = 10-8=2
In this way we have to find the (13+9) =22nd letters fromA.
Left = (13+1)-2 = 12
22nd letter from A = (V) Ans.
According to the original series (A –Z)=12×2-1=23(W) Ans.
28- ;fn izFke v)kZa’k rFkk f}rh; v)kZa’k nksuksa dks dze’k% foijhr dze esa
 Concept - VII fy[kk tk, rks dkSulk v{kj ck,¡ ls 16osa ds ck,¡ 7ok¡ gksxk \
M L ____ B, A, Z, Y _____O N (Both halfs are in reverse order) If first half and second half both are written in the reverse
26- ;fn çFke v)ka’kk o f}rh; v)ka'k nksuksa Øe'k% foijhr Øe esa fy[ks order respectively then which letter will be 7th to the left of
tk, rks dkSulk v{kj nk,¡ ls 10osa ds ck,¡ 10ok¡ gksxk\ 16th from the left end?

5
Download Free PDFs & e-Books from Neon Classes App

(a) D (b) V (c) R (d) E nk,¡ ¾14


Sol: (d) ck,¡ ¾ 9
ck,¡ ¾ 16 nk,¡ =14+9 = 23
ck,¡ ¾ 7 Left = 9
ck,¡ ¾ 16 & 7 ¾ 9 Right = 14
Left = 16 Right = 14 + 9 = 23
Left = 7 10 13
Left = 16-7=9 M----------A, Z----------N
M----------A, Z----------N 23
95
23&13 ¾ 10 ¼J½ mÙkj
ck,¡ ls 9 ok¡ v{kj igys v)kZa’k esa jgsxkA
23-13 = 10 (J) Ans.
igys v)kZa’k esa M ls 9ok¡ v{kj tks gksxk og A ls ¼13 $ 1½ & 9 ¾
31- ;fn vaxt sz h o.kZekyk ds igys rsjg v{kj dks mYVs Øe esa fy[kk tkrk
5 ok¡ v{kj gksxkA
gS vkSj fQj A ds ckn 'ks"k 13 v{kjksa dks Øe esa fy[kk tkrk gS] rks
A ls 5ok¡ v{kj ¾ (E) mÙkj
vkids nk;sa ls 17 osa LFkku ij dkSu lk v{kj fn[kkbZ nsxk\
The 9th letter from the left end will come in the first held.
If the first thirteen letter of English alphabet is written in the
The 9th letter from M in the first held – 5th(13+1-9) letter
reverse order and then after A, the remaining 13 letters are
fromA
written in order, which letter will appear at the 17th position
5th letter from A = (E) Ans.
from your right?
 Concept - VIII
M________AN________Z ( when given half in reverse order) (a) D (b) E (c) C (d) F
29- ;fn çFke v)kZa'k dks foijhr Øe esa fy[kk tk, rc dkSu lk v{kj Sol. (a)
nk,¡ ls 10 osa ds ck,¡ 9 gksxk\ M L K_ _ _D C B A N O_ _ X Y Z
If first half and second half both are written in the reverse 17
order respectively then which letter will be 9th to the left of Right
10th from the right end? Left
(a) D (b) F (c) G (d) Q 17th position from right = D option (a) is correct
Sol: (b) ç'u ds vuqlkj dsoy çFke v)kZa'k dks foijhr Øe esa fy[ksaxs rks 32- ;fn çFke v)kZa'k dks foijhr Øe esa fy[kk tk, rc dkSu lk v{kj
ges fuEu J`[a kyk çkIr gksxh& nk,¡ 16osa ds nk,¡ 13 ok¡ gksxk \
According to the question we will write only first half in the If the written in the reverse order then which letter will be
reverse order and we will obtain the following series: 13th to the right of 16th from the right end?
M L ------ B A N---------Z (a) X (b) W (c) R (d) Q
ck,¡ Nksj ls igyk v{kj M rFkk nk,¡ Nksj ls igyk v{kj Z gksxkA Sol: (a)
vc ç'u dks fuEu rjhds ls gy fd;k tk,xkA nk,¡ ¾ 16
The first letter from the end is M and the first letter from the nk,¡ ¾ 13
right end will be Z. Now we will solve the question in the nk,¡ ¾ 16&13 ¾ 3
following way. Right = 13
nk,¡ ¾10 Right = 16
ck,¡ ¾ 9 Right = 16 – 13 = 3
nk,¡ 10+9 = 19 Z ls rhljk v{kj A ls ¼26+1½&3=24 ¼X½ mÙkj
Left = 9 3rd letter from Z = (26 + 1 – 3) = 24th letter from A i.e. (X) Ans.
Right = 10 33- ;fn izFke v}kZa’k dks foijhr dze esa fy[kk tk, rc dkSulk v{kj ck,¡
Right = 10 + 9 = 19 ls 20ok¡ ds ck,¡ 11ok¡ gksxk\
6 13 If first half written in the reverse order then which letter will
M----------A, Z----------N be 11th to the left of 20th from the left end?
19 (a) D (b) W (c) R (d) E
nk,¡ ls 19 ok¡ v{kj Kkr djrs le; 13 v{kj gekjs f}rh; v)kZa'k esa Sol: (d)
vk tk,xsA ckfd cps 19&13 ¾ NVk v{kj A ls ns[ksx
a sA ck,¡ ¾ 20
While finding the 19thleeter from the right end, the 13 letter ck,¡ ¾ 11
will come in the second half. The remaining (19-13)=6th letter ck,¡ ¾ 20 & 11 ¾ 9
will be seen from A. Left = 11
A ls NBk v{kj ¼F½ mÙkj Left = 20
Now the 6th letter will be seen from A. Left = 20-11 = 9
30- ;fn çFke v)kZa'k dks foijhr Øe esa fy[kk tk, rc dkSulk v{kj nk,¡ M----------A, Z----------N
ls 14 osa ds ck,¡ 9 ok¡ gksxk\ 9
If the written in the reverse order then which letter will be 7th ck,¡ ls 9 ok¡ v{kj igys c)kZa’k esa gksxk
to the left of 16th from the right end? igys v)kaZa’k esa M ls 9ok¡ v{kj tks gksxk og A ls ¼13$1½&9¾5ok¡
(a) D (b) H (c) J (d) Q v{kj gksxkA
Sol: (c) A ls 5ok¡ v{kj (E) mÙkj

6
Best App for Govt. Jobs : Neonclasses (Download Now)

The 9th letter from left end will come in the first half. M----------A, Z----------N
The 9th letter from M in the first half =(13 +1-9)= 5th letter 5
fromA N ls 5 ok¡ v{kj A ls 13+5 = 18 ok¡ v{kj gksxkA
5th letter from A = E Ans. A ls 18 ok¡ v{kj ¾ ¼R½ mÙkj
 Concept - IX 5th letter from N = (13+5) = 18 letter from A
18th letter from A = (R) Ans.
A________MZ________N
(when given second half in reverse order)
37- ;fn f}rh; v)kZa'k dks foijhr czke esa fy[kk tk, rks dkSulk v{kj
34- ;fn f}rh; v)kZa'k dks foijhr Øe esa fy[kk tk, rc dkSulk v{kj nk,¡ ls 18osa ds nk,¡ 8oka gksxk\
ck,¡ ls 18 osa ds ck,¡ 14 ok¡ gksxk\ If second half written in the reverse order then which letter
will be 8th to the right of 18th from the right end?
If second half written in the reverse order then which letter
(a) D (b) W (c) H (d) I
will be 14th to the left of 18th from the left end?
Sal: (b) nk,¡ ¾ 18
(a) D (b) W (c) R (d) Q
Sol: (a) ç'u ds vuqlkj f}rh; v)kZa’k dks foijhr Øe esa fy[ksaxs gesa fuEu nk,¡ ¾ 8
J`[a kyk çkIr gksxh& nk,¡ ¾ 18&8 ¾ 10
According to question we will write ony second half in the Right = 8
reverse order and we will obtain the following series: Right = 18
A B …..L M Z Y--------- O N Right = 18-8=10
ok,¡ Nksj ls igyk v{kj A rFkk nk,¡ Nksj ls igyk v{kj N gksxkA vc A-----------M, Z-----------N
ç'u dks fuEu rjhds ls gy fd;k tk,xkA 10
The first letter from left end is A and the first letter from right
N ls 10ok¡ v{kj ¾ A ls
end will be N. Now we will solve the question in the following
13+10 ¾23ok¡ v{kj gksxkA
way:
A ls 23ok¡ v{kj ¼W½ mÙkj
ck,¡ ¾ 18
10th letter from N = (13+10) = 23rd letter from A 23rd letter
ck,¡ ¾ 14
from A = (W) Ans.
ck,¡ ¾ 18&14 ¾ 4
38- ;fn f}rh; vnkZa’k dks foijhr Øe esa fy[kk tk, rc dk v{kj nk,¡
Left = 14
ls 13osa ds ck,¡ 10ok¡ gksxk\
Left = 18
If second half written in the reverse order then which letter
Left = 18-14=4
will be 13th to the left of 10th from the right end?
M----------A, Z----------N
4 (a) D (b) W (c) H (d) I
Sol: (a) nk,¡ ¾ 13
A ls pkSFkk v{kj ¾ ¼D½ mÙkj ck,¡ ¾ 10
4th letter from A = (D) Ans. nk,¡ = 13+10 ¾ 23
35- ;fn f}rh; v)kZa’k dks foijhr Øe esa fy[kk tk, rc dkSu lk v{kj Left = 10
ck,a ls 18 ls ds ck,¡ 9 gksxk\ Right = 13
If second half written in the reverse order then which letter Right = 13+ 10 = 23
will be 9th to the left of 18th from the left end?
10 13
(a) T (b) W (c) R (d) I
A----------M, Z----------N
Sol: (d)
23
ck,¡ ¾ 18
ck,¡ ¾ 9 N ls 23 ok¡ ¾ A ls ¼26+1½&23 ¾ pkSFkk v{kj gksxk
ck,¡¾ 18&9=9 A ls pkSFkk v{kj ¼D½ mÙkj
Left = 9 23rd letter from N = (26+1-23) = 4th letter from A
Left = 18 4th letter from A= (D) Ans.
Left = 18-9 = 9  Concept - X
M----------A, Z----------N
9 Alphanumeric series
39- C 3 4 @ D 5 6 © E F G I J 7 % U * $ Z # 8 9 T S V
A ls 9 ¼l½ mÙkj bl Js.kh ls lacaf/kr fuEu ik¡p iz’uksa dks gy dhft,A
9th letter from A = (I) Ans. Now solve the following question related to this series.
36- ;fn f}rh; v)kZa'k dks foijhr Øe esa fy[kk tk, rks dkSulk v{kj nk,¡ (i) dkSulk rRo ck,¡ ls 8 osa ds nk,¡ 7ok¡ gksxk\
ls 15 esa ds nk,¡ 10 ok¡ gksxk\ Which element will be 7th to right of 8th from the left end?
If second half written in the reverse order then which letter (a) % (b) U (c)7 (d) *
will be 10th to the right of 15th from the right end? (ii) dkSulk rRo ck,¡ ls 3 vkSj nk,¡ ls 3 ds Bhd chp esa gksxk\
(a) D (b) W (c) R (d) Q Which element will be exactly between the 3rd element from
Sol: (c) nk,¡ ¾ 15 the left end and 3rd element from the right end?
nk,¡ 10 (a) J (b) I (c) Q (d) 7
nk,¡ ¾ 15&10¾5 (iii) fuEu esa ls fo"ke Nk¡fV;sa &
Right = 10 Find the odd one out:
Right = 15 (A) 3 4 C (B) 5 6 ©
Right = 15-10=5

7
Download Free PDFs & e-Books from Neon Classes App

(C) J 7 I (D) $ z * fn, x, v{kj] la[;k] çrhd J`[a kyk dk lanHkZ ysa vkSj uhps fn, x,
Sol. ç'u dk mÙkj nsAa
(i) (a) (Left) K L % Y & 4 E 2 * 3 M & 7 S W # 8 2 H * L (Right)
ck,¡ ¾ 8 If all the numbers are dropped from the series, which of the
nk,¡ ¾ 7 following will be eight from the right?
ck,¡ ¾ 8 $ 7 ¾ 15ok¡ rRo =%¼mÙkj½ ;fn Ja[kyk esa ls lHkh la[;kvksa dks gVk fn;k tk,] rks fuEufyf[kr esa
Right = 7 ls dkSu lk nk;sa ls vkBosa LFkku ij gksxk\
Left = 8 (a) M (b) & (c) * (d) E
Left = 8 + 7 = 15th element = (%) Ans. Sol. (a)
(ii) (a) Right to 8th position when numbers are dropped new series is
ck;k¡ ¾ 3
KL%Y&E*M&SW#H*L
nk;k ¾ 3 Right
ck;k ¾ 25 $ 1 & 3 ¾ 23 Left 18th
3  23
ck,¡ ¾ ¾ 13ok¡ rRo ¾ ¼J½ mÙkj So correct option (a).
2 43. Study the following arrangement of letters and symbols (from
Left = 3 Left to Right) and answer the question that follows;
Right = 3 v{kjksa vkSj çrhdksa dh fuEufyf[kr O;oLFkk dk vè;;u djsa ¼ck,a ls
Left = (25 + 1 = 3) = 23
nk,a½ vkSj uhps fn, x, ç'u dk mÙkj ns(a
3  23 (Left) H S % V ^ E R U ; F J # S A U & G * G H @ D C ! I N O ? S
Left = = 13th element = (J) Ans.
2 (Right)
How many consonants are there in this arrangement which
(iii) C 3 4 @ D 5 6 © E F G I J 7 % U * $ Z # 8 9 T S V
are immediately preceded by a vowel?
bl O;oLFkk esa ,sls fdrus O;atu gSa ftuds Bhd igys ,d Loj gS\
C34@D56©EFGIJ7%U*$Z#89TSV (a) 4 (b) 5 (c) 2 (d) 3
→ → → → →
bl Js.kh esa fodYi (A), Sol. (c)
(C), (D), ds iSVuZ leku gSA fodYi (B) mÙkj gksxkA
Vowel Consonant
In this series the pattern for options (A), (C), (D) is same.
Hence Ans (B) Only Two pairs are present in this series.
40. Study the given arrangement of letters, symbols and numbers, (1) E R
and answer the question that follows. (2) I N
v{kjks]a çrhdksa vkSj la[;kvksa dh nh xbZ O;oLFkk dk vè;;u djsa vkSj So correct option (c)
uhps fn, x, ç'u dk mÙkj nsAa 44. Reading the given cluster from left to right, how many
(Left) G 6 & $ 4 Y 8 9 # @ W H 5 % 6 & K 9 @ R 9 4 T & % # U Y consonants are their that are immediately followed by a
8 $ # Q Y 8 4 $ (Right) vowel?
Which of the following is ninth to the right of the fourteenth fn, x, lewg dks ck,a ls nk,a i<+rs gq,] vki fdrus O;atu gSa ftuds
from the right end of the given arrangement? Bhd ckn ,d Loj gS\
fuEufyf[kr esa ls dkSu nh xbZ O;oLFkk ds nk;sa Nksj ls pkSngosa ds nk;sa (Left) ATGH@7DOGKI7#PE41D&GOH4@3IKMBUCD (Right)
ls ukSosa LFkku ij gS\ (a) 4 (b) 5 (c) 3 (d) 6
(a) # (b) T (c) Y (d) Q Sol. (b)
Right to Right = 14 – 9 = 5 Consonant Vowel
Right to 5th position = Q
option (d) is correct. Only Five pairs are present in this series.
41. Study the given digit – letter – symbol sequence carefully and (1) D O (2) K I (3) P E (4) G O (5) B U
answer the question that follows. So correct option (b)
fn, x, vad & v{kj & izrhd vuqØe dk /;kuiowZd v/;;u djs
vkSj iwNs x, dk mÙkj nsAa 2. Alphabetical series
(Left/ck,a) 5 Q + Sr 8 B @ A 3 ? 6 c < Z % 6 d & G $ 2 (Right/ nk,a)
If the given sequence is written in the reverse order, then  Concept -
which element will be 7th to the right of the 10th element from
In alphabet series section, a string of alphabets, either in a
the right end?
single file or in combination form a sequence. This sequence
;fn fn, x, vuqØe dks mYVs Øe esa fy[kk tk,] rks dkSu lk ?kVd comes together follow a define rule. The candidate is expected
nk,a fljs ls 10osa ?kVd ds nkbZa vksj 7osa LFkku ij gksxk\ to defect this rule and answer the questions at the end.
(a) B (b) 8 (c) % (d) +
 Concept - 1
Sol. (d)
Right to Right = 10 – 7 = 3 (When given single alphabet/Letter.)
Right to 3rd position but series is reverse order so now left to Fix addition
3rd position + , so option (d) is correct. Alternate addition/subtraction
42. Refer to the given letter, number, symbol series and answer Series (addition/ subtraction)
the question that follows. Series (odd , even, prime number)

8
Best App for Govt. Jobs : Neonclasses (Download Now)

Vowel/Consonant based 4. Select the number from among the given options that can
 Concept - 2 replace the question mark (?) in the following series.
fn, fodYiksa esa ls og la[;k pqfu, tks fuEufyf[kr J`s.kh esa
DOUBLE ALPHABET iz’uokpd fpUg (?) ds LFkku ij vk ldrh gSA
Fix addition E, F, I, N, U, ?
Alternate addition/subtraction (a) A (b) D (c) N (d) Z
Series (addition/ subtraction) Sol. (b)
 Concept - 3 Pattern :- Odd number series addition
TRIPLE ALPHABET E F I N U D
Fix addition 5 6 9 14 21 29
Alternate addition/subtraction
Series (addition/ subtraction) +1 +3 +5 +7 +9
Concept:- 4, FOUR ALPHABET 5. Select the letter from among the given options that can
Fix addition replace the question mark (?) in the following series.
Alternate addition/subtraction fn, x, fodYiksa esa ls ml v{kj dk p;u djsa tks fuEufyf[kr
Series (addition/ subtraction) J`a[kyk esa ç'u fpà ¼\½ dks çfrLFkkfir dj ldrk gSA
Opposite letter based B, E, G, J, L, ?
 Concept - 5 (a) Q (b) P (c) N (d) O
Sol. (d)
FIVE OR MORE ALPHABET/LETTER Pattern :- Alternate series addition
 Concept - 6
B E G J L O
ALPHANUMERIC SERIES 2 5 7 10 12 15
1. Select the letter that can replace the question mark (?) in
the following series. +3 +2 +3 +2 +3
ml v{kj dk p;u djsa tks fuEufyf[kr J`[a kyk esa ç'u fpà ¼\½ dks 6. Select the letter from among the given options that can
çfrLFkkfir dj ldrk gSA replace the question mark (?) in the following series.
J, M, P, ? , V, Y fn, x, fodYiksa esa ls ml v{kj dk p;u djsa tks fuEufyf[kr
(a) T (b) O (c) R (d) S J`a[kyk esa ç'u fpà ¼\½ dks çfrLFkkfir dj ldrk gSA
Sol. (d) P, N, J, H, ?
Type : - Single Alphabet (a) B (b) D (c) E (d) C
Fixed Addition Sol. (b)
Pattern :- Alternate series substraction
J M P S V Y
10 13 16 19 22 25 P N J H D
16 14 10 8 4
+3 +3 +3 +3 +3
2. Select the letter that can replace the question mark in the –2 –4 –2 –4
following series. 7. Select the letter from among the given options that can
ml v{kj dk p;u djsa tks fuEufyf[kr J`[a kyk esa ç'uokpd fpUg replace the question mark (?) in the following series.
dks çfrLFkkfir dj ldrk gSA fn, x, fodYiksa esa ls ml v{kj dk p;u djsa tks fuEufyf[kr
A, E, I, ?, Q, U J`a[kyk esa ç'u fpà ¼\½ dks çfrLFkkfir dj ldrk gSA
(a) N (b) K (c) M (d) O Y, S, N, J, ?
Sol. (c) (a) F (b) E (c) G (d) H
(Letter +4) Fixed Addition Sol. (c)
Pattern :- Subtraction series, decreasing order
A E I M Q U
1 5 9 13 17 21 Y S N J G
25 19 14 10 7
+4 +4 +4 +4 +4
3. Which letter will replace the question mark (?) in the –6 –5 –4 –3
following letter series? 8. Select the letter that can replace the question mark (?) in
fuEufyf[kr v{kj Ja[kyk esa ç'uokpd fpUg ¼\½ ds LFkku ij dkSu the following series.
lk v{kj vk,xk\ fuEufyf[kr J`a[kyk esa iz’u fpUg (?) dh txg ij vkus okys v{kj
E, J, N, Q, S, ? dk p;u djsAa
(a) T (b) V (c) U (d) S O, B, L, C, I, D, F, E, ?
Sol. (a) (a) K (b) C (c) H (d) N
Pattern :- Addition Series Sol. (b)
Pattern :- Double series ¼/kM+du½
E J N Q S T
5 10 14 17 19 20

+5 +4 +3 +2 +1

9
Download Free PDFs & e-Books from Neon Classes App

–3 –3 –3 –3 (c) WR (d) WS
Sol. (c)
O B L C I D F E C Pattern :- (1st letter +4), (2nd letter +3)
15 2 12 3 9 4 6 5 3 +4 +4 +4 +4

+1 +1 +1 GF KI OL SO W R
9. Select the letter that can replace the question mark (?) in
the following series. +3 +3 +3 +3
ml v{kj dk p;u djsa tks fuEufyf[kr J`[a kyk esa ç'u fpà ¼\½ dks Elimination : - Option (b) & (d) eliminated because last letter S.
çfrLFkkfir dj ldrk gSA Option (a) eliminated because 1st letter X wrong.
A, E, J, ? , W 13. Select the letter-cluster from among the given options that
(a) P (b) R (c) Q (d) S can replace the question mark (?) in the following series.
Sol. (a) fn, x, fodYiksa esa ls ml v{kj&lewg dk p;u djsa tks fuEufyf[kr
Pattern :- Addition in series J`a[kyk esa ç'u fpà ¼\½ dks çfrLFkkfir dj ldrk gSA
AY, BX, ?, DV, EU, FT
A E J P W
(a) CW (b) CV
1 5 10 16 23 (c) CU (d) CE
Sol. (a)
+4 +5 +6 +7
Pattern :- (1st letter +1), (2nd letter –1)
10. Select the letter will replace the question mark (?) in the
+1 +1 +1 +1 +1
following series.
fuEufyf[kr J`a[kyk esa ç'u fpà ¼\½ ds LFkku ij v{kj dk p;u AY BX CW DV E U FT
djsAa
C, B, B, C, Z, E, W, H, S, ?, N –1 –1 –1 –1 –1
(a) L (b) K (c) P (d) I Elimination:– In option (b), (c) & (d) last letter is wrong so
Sol. (a) gets eliminated.
Pattern :- Double series ¼/kM+du½ 14. Select the letter-cluster that can replace the question mark (?)
in the given letter cluster series.
C B B C Z E W H S L N
ml v{kj&lewg dk p;u djs]a tks nh xbZ v{kj&lewg dh Js.kh esa
3 2 2 3 26 5 23 8 19 12 14
iz’uokpd fpgu (?) ds LFkku ij vk,xkA
+1 +2 +3 +4 AE, KQ, EI, LR, IO, ?, OU, NT
(a) MR (b) NR
W U S Q (c) MS (d) NS
51. , , , ,?
S O K G Sol. (c)
(a) P/R (b) C/O (c) R/J(d) O/C Pattern :- /kM+du
W –2 U –2 S –2 Q -2 O +1 +1 +1
-4 -4 -4 -4 AE KQ EI LR IO MS OU N T
S O K G C
Sol. (d) +1 +1 +1
11. Select the letter cluster that can replace the question
Elimination: - 1st
letter wrong in option (b) & (d) and 2nd letter
mark (?) in the following series.
wrong (a) so eliminated.
fn, x, fodYiksa esa ls ml v{kj & lewg dk p;u djs tks
AB ZY DE WV GH
fuEufyf[kr Js.kh esa iz’uokpd (?) ds LFkku ij vk ldrh gSA 53. , , , , ,?
C X F U I
AR, CP, ?, GL, IJ, KH
(a) DM (b) DN SR TS ST RS
(a) (b) (c) (d)
(c) EN (d) EM Q R R Q
Sol. (c) Sol. (b)
Pattern :- (1st letter +2), (2nd letter –2)
+2 +2 +2 +2 +2 AB ZY DE WV GH TS
C X F U I R

AR CP EN GL I J KH
55. HK, ?, PQ, TT, XW
–2 –2 –2 –2 –2
(a) LN (b) NO (c) LK (d) NM
Elimination : - Option (a), (b) eliminated because 1st letter D Sol. (a)
wrong. Option (d) eliminated because last letter M wrong. +4 +4 +4 +4
12. Select the letter-cluster from among the given options that H L P T X
can replace the question mark (?) in the following series. +3 +3 +3 +3
fn, x, fodYiksa esa ls ml v{kj&lewg dk p;u djsa tks fuEufyf[kr K N Q T W
J`a[kyk esa ç'u fpà ¼\½ dks çfrLFkkfir dj ldrk gSA
15. Select the letter-cluster from among the given options that
GF, KI, OL, SO, ?
can replace the question mark (?) in the following series.
(a) XR (b) YS

10
Best App for Govt. Jobs : Neonclasses (Download Now)

fn, x, fodYiksa esa ls ml v{kj&lewg dk p;u djsa tks fuEufyf[kr B –2 Z –2 X –2 V –2 T –2 R


J`a[kyk esa ç'u fpà ¼\½ dks çfrLFkkfir dj ldrk gSA
TSF, RPJ, PMN, NJR, ? +3 +3 +3 +3 +3
R U X A D G
(a) JFV (b) KGU
(c) LGN (d) LGV –4 –4 –4 –4 –4
H D Z V R N
Sol. (d)
Elimination:– Last letter wrong in option (a) & 2nd letter wrong
Pattern :- /kM+du
in option (b) & (d) so eliminated.
T –2 R –2 P –2 N –2 L 19. Select the letter-cluster that can replace the question
mark (?) in the following series.
–3 –3 –3 –3
S P M J G ml v{kj&lewg dk p;u djsa tks fuEufyf[kr J`[a kyk esa ç'u fpà
+4 +4 +4 +4 ¼\½ dks çfrLFkkfir dj ldrk gSA
F J N R V eBd, hXi, kTn, nPs,?
Elimination: - 1st
letter wrong in option (a) & (b) and last letter (a) qKx (b) Qlv
wrong in option (c) so these are eliminated. (c) qLx (d) pLw
16. Select the letter-cluster from among the given options that Sol. (c)
can replace the question mark (?) in the following series. Pattern :- (Capital letter –4), (1st Small +3) (3rd Small +5)
fn, x, fodYiksa esa ls ml v{kj&lewg dk p;u djsa tks fuEufyf[kr e +3 h +3 k +3 n +3 q
J`a[kyk esa ç'u fpà ¼\½ dks çfrLFkkfir dj ldrk gSA
GCB, JHE, MMH, PRK, ? –4 –4 –4 –4
B X T P L
(a) SWN (b) TXO
+5 +5 +5 +5
(c) RVM (d) SXM d i n s x
Sol. (a) Elimination:– Middle letter is not capital in option (b) middle
Pattern :- Addition (1st letter +3), (2nd letter +5) & (3rd letter is wrong in option (a) and last letter wrong in option (d)
letter +3) so these option gets eliminated.
G +3 J +3 M +3 P +3 S 20. Which letter-cluster will replace the question mark (?) in
the following series?
+5 +5 +5 +5
C H M R W fuEufyf[kr J`a[kyk esa dkSu&lk v{kj&lewg ç'uokpd fpUg ¼\½ dk
+3 +3 +3 +3 LFkku ysxk\
B E H K N PSV, UXA, ZCF, EHK. ?
Elimination: – From last letter option (b), (c), (d) gets (a) JMO (b) IMP
eliminated. (c) JMP (d) JNP
17. Select the letter-cluster from among the given options that Sol. (c)
can replace the question mark (?) in the following series. Pattern :- Addition of 5 in every letter.
fn, x, fodYiksa esa ls ml v{kj&lewg dk p;u djsa tks fuEufyf[kr +5 +5 +5 +5
P U Z E J
J`a[kyk esa ç'u fpà ¼\½ dks çfrLFkkfir dj ldrk gSA
NZK, MBH, LDE, KFB, ? +5 +5 +5 +5
S X C H M
(a) JHX (b) JHY
+5 +5 +5 +5
(c) JHZ (d) HIY V A F K P
Sol. (b) Elimination:– 1st letter wrong in option (b), last letter wrong in
Pattern :- (1st letter –1), (2nd letter +2) & (3rd letter –3) option (a) and middle letter wrong in option (d) so eliminated.
N –1 M –1 L –1 K –1 J 21. Select the letter-cluster from among the given options that
can replace the question mark (?) in the following series.
+2 +2 +2 +2
Z B D F H fn, x, fodYiksa esa ls ml v{kj&lewg dk p;u djsa tks fuEufyf[kr
–3 –3 –3 –3 J`a[kyk esa ç'u fpà ¼\½ dks çfrLFkkfir dj ldrk gSA
K H E B Y AAI, AEI, AII, AOI, ?
Elimination: – 1st
letter wrong in option (d) and last letter (a) AUI (b) IAA
wrong in option (a) & (c) so these are eliminated. (c) IIA (d) EAI
18. Select the letter-cluster that can replace the question mark (?) Sol. (a)
in the following series. Pattern :- 1st and last letter as it is and middle letter in
ml v{kj&lewg dk p;u djsa tks fuEufyf[kr J`[a kyk esa ç'u fpà Vowel sequence A, E, I , O , U
¼\½ dks çfrLFkkfir dj ldrk gSA A A A A A
BRH, ZUD, ?. VAV, TDR, RGN
(a) XZY (b) XZZ A E I O U
(c) XXZ (d) XYZ
Sol. (c) I I I I I
Pattern :- (1st letter –2), (2nd letter +3) & (3rd letter –4) Elimination:– Because of middle letter wrong options (b), (c),
(d) eliminated.
22. Select the correct option that will fill in the blank and
complete the series.

11
Download Free PDFs & e-Books from Neon Classes App

lgh fodYi dk p;u djsa tks fjä LFkku dks Hkjsxk vkSj J`a[kyk dks Elimination:– From 1st letter option (b), (c), (d) gets
iwjk djsxkA eliminated.
ZCC, XEE, _______, TII, RKK 26. Find the missing term to complete the given pattern.
(a) RGG (b) SGG fn, x, iSVuZ dks iwjk djus ds fy, yqIr in Kkr dhft,A
(c) UGG (d) VGG DPJ, LJK, FL?
Sol. (d) (a) Y (b) N
Pattern :- (1st –1), (2nd +2), (3rd +2) (c) I (d) M
Sol. (c)
Z –2 X –2 V –2 T–2 R
Pattern :
+2 +2 +2 +2 1st letter code+ 2nd letter code rd
C E G I K = 3 letter code
2
+2 +2 +2 +2
C E G I K 4 16 10
Elimination:– From first letter all options are wrong except D P J
option (b)
23. Select the letter-cluster from among the given options that can 12 10 11
replace the question mark (?) in the following series. L J K
fn, x, fodYiksa esa ls ml v{kj&lewg dk p;u djsa tks fuEufyf[kr
J`[a kyk esa ç'u fpà ¼\½ dks çfrLFkkfir dj ldrk gSA 6 12 9
FCC, UFH, OIS, LLX, FOC, ? F L I
(a) TRF (b) VSG Elimination:– From last letter option (b), (c), (d) eliminated.
(c) URH (d) WTH 27. Select the letter cluster that will replace the question
Sol. (c) mark (?) in the following series.
Pattern :- Addition and opposite mix ml v{kj lewg dk p;u djsa tks fuEufyf[kr J`a[kyk esa ç'uokpd
F opp. U –6 O opp. L –6 F opp. U fpUg ¼\½ dks çfrLFkkfir djsxkA
DZP, SCG, JFV, YIM, PLB, ?
+3 +3 +3 +3 +3
C F I L O R (a) FOR (b) EOS
(c) DOS (d) ESP
+5 opp. +5 opp. +5
C H S X C H Sol. (b)
Elimination:– From middle letter option (b) & (d) eliminated. Pattern :- Letter +3 in cross order
24. Select the letter-cluster from among the given options that 4 26 16 10 6 22 16 12 2
can replace the question mark (?) in the following series. D Z P J F V P L B
fn, x, fodYiksa esa ls ml v{kj&lewg dk p;u djsa tks fuEufyf[kr +3 +3 +3 +3 +3 +3 +3 +3 +3
J`a[kyk esa ç'u fpà ¼\½ dks çfrLFkkfir dj ldrk gSA
LGG, PJL, KMO, OPT, LSG, ? S C G Y I M E O S
(a) PVL (b) OVK 19 3 7 25 9 13 5 15 19
(c) NUL (d) PUK Elimination:– From 1st letter option (a) & (c) eliminated from
Sol.(a) last letter option (d) eliminated.
Pattern :- Opposite and Addition mix 28. Select the letter-cluster from among the given options that
L +4 P opp. K +4 O opp. L +4 P can replace the question mark (?) in the following series.
fn, x, fodYiksa esa ls ml v{kj&lewg dk p;u djsa tks fuEufyf[kr
+3 +3 +3 +3 +3
G J M P S V J`a[kyk esa ç'u fpà ¼\½ dks çfrLFkkfir dj ldrk gSA
+5 opp. +5 opp. +5 KYBA, FTEE, AOHI, VJKO, ?, LZQA
G L O T G L (a) PFNU (b) QENU
Elimination:– From the letter option (b), (c), (d) gets (c) QEMI (d) QDNI
eliminated. Sol. (b)
25. Select the letter-cluster that can replace the question Pattern :- (1st letter–5) (2nd letter–5) (3rd letter+3)(4th
mark (?) in the following series. letter Vowel sequence)
ml v{kj&lewg dk p;u djsa tks fuEufyf[kr J`a[kyk esa ç'uokpd K –5 F –5 A –5 V –5 Q –5 L
fpà ¼\½ dks çfrLFkkfir dj ldrk gSA
GHB, LMG, PQK, ? –5 –5 –5 –5 –5
Y T O J E Z
(a) STN (b) TSM
+3 +3 +3 +3 +3
(c) PQE (d) ABX B E H K N Q
Sol. (a)
Pattern :- Addition in series. A E I O U A
G +5 L +4 P +3 S Elimination:– From 1st letter option (a) and from last letter
+5 +4 +3 option (c) & (d) eliminated.
H M Q T 29. Select the letter-cluster from among the given options that
+5 +4 +3 can replace the question mark (?) in the following series.
B G K N

12
Best App for Govt. Jobs : Neonclasses (Download Now)

fn, x, fodYiksa esa ls ml v{kj&lewg dk p;u djsa tks fuEufyf[kr (c) jres, LUIX (d) jrfq, LUIW
J`a[kyk esa ç'u fpà ¼\½ dks çfrLFkkfir dj ldrk gSA Sol. (b)
KMTC, EVOM, OQXG, IZSQ, ? Pattern :- Addition in increasing series order. Capital
(a) SUCL (b) SVCK alphabet and small alphabets in alternate order.
(c) TVBK (d) SUBK +2 +2 +2 +2 +2 +2
Z b D f H j L
Sol. (d)
Pattern :- Letter +2 in cross order +3 +3 +3 +3 +3 +3
C f I l O r U
11 13 20 3 +4 +4 +4 +4 +4 +4
K M T C K o S w A e I
+5 +5 +5 +5 +5 +5
R w B g L q V
E V O M Elimination:– Capital last letter wrong in option (c) & (d) and
5 22 15 13 in option (a) small and capital sequence wrong so eliminated.
31. Select the letter-cluster from among the given options that
can replace the question mark (?) in the following series.
O Q X G fn, x, fodYiksa esa ls ml v{kj&lewg dk p;u djsa tks fuEufyf[kr
15 17 24 7 J`a[kyk esa ç'u fpà ¼\½ dks çfrLFkkfir dj ldrk gSA
TALT, YAGT, YFGO, DFBO, DKBJ, ?
(a) IJKW (b) DPBE
I Z S Q (c) IKWJ (d) BDEP
9 26 19 17 Sol. (c)
Pattern: - addition, subtraction mix
T +5 Y +0 Y +5 D +0 D +5 I
S U B K
19 21 2 11 +0 +5 +0 +5 +0
A A F F K K
Elimination:– From 1st letter option (c) from last letter option
–5 +0 –5 +0 –5
(a) & from 2nd letter option (b) eliminated. L G G B B W
29. Select the letter-cluster from among the given options that can +0 –5 +0 –5 +0
replace the question mark (?) in the following series. T T O O J J
fn, x, fodYiksa esa ls ml v{kj & lewg dk p;u djs tks Elimination: – From last letter all option eliminated except
fuEufyf[kr Js.kh esa iz’uokpd (?) ds LFkku ij vk ldrh gSA option (c).
FJNR, CGKO, IMQU, ? 32. Select the letter-cluster from among the given options that
(a) DHLU (b) FJMR can replace the question mark (?) in the following series.
(c) KOSX (d) LPTX fn, x, fodYiksa esa ls ml v{kj lewg dk p;u djs] tks fuEufyf[kr
Sol. (d) J`a[kyk esa iz’u fpUg (?) ds LFkku ij vk ldrk gSA
Pattern :- Opposite letter in cross order CBDD, EBBD, ?, IBXD, KBVD
6 10 14 18 (a) HCYE (b) GCZD
F J N R (c) GBZD (d) GCZF
Sol. (c)
Opposite Pattern: - addition, subtraction mix
I M Q U C +2 E +2 G +2 I +2 K
9 13 17 21
+0 +0 +0 +0
B B B B B
3 7 11 15
–2 –2 –2 –2
C G K O D B Z X V
+0 +0 +0 +0
Opposite D D D D D
L P T X Elimination: – From 2nd letter option (a), (b), (d) gets
12 16 20 24 eliminated.
Elimination :– From 1st letter option (a), (b), (c) gets 33. Select the option that will fill in the blank and complete
eliminated. the given series.
30. Select the letter-cluster from among the given options that ml fodYi dk p;u djsa tks fjä LFkku dks Hkjsxk vkSj nh xbZ
can sequentially replace both the question mark (?) in the J`a[kyk dks iwjk djsxkA
following series. TMKB, CLNU, VOMD, _________, XQOF
fn, x, fodYiksa esa ls mu v{kj&lewgksa dk p;u djsa tks (a) ENPW (b) BQEA
fuEufyf[kr J`a[kyk esa nksuksa iz’u fpUgksa (?) dks Øfed :i ls (c) ETRW (d) BQUR
izfrLFkkfir dj ldrs gSaA Sol. (a)
ZCKR, bfow, DISB, flwg, HOAL, ?, ? Pattern: - Addition of 2 in alternate word.
(a) JREQ, luiw (b) jreq, LUIV

13
Download Free PDFs & e-Books from Neon Classes App

+2 Sol. (c)
+2
Pattern :- Interchanging in cross order.
+2
+2 S T G M Q
T M K B, C L N U, V O M D, E N P W, X Q O F
+2
+2
+2
+2 S G Q T M
Elimination: – From 2nd
letter options (b), (c), (d) gets
eliminated.
34. Select the letter-cluster from among the given options that S Q M G T
can replace the question mark (?) in the following series.
fn, x, fodYiksa esa ls ml v{kj&lewg dk p;u djsa tks fuEufyf[kr
J`a[kyk esa ç'u fpà ¼\½ dks çfrLFkkfir dj ldrk gSA S M T Q G
BOARD, BOARW, BOAIW, ?, BLZIW, YLZIW Elimination:– From last letter option (b), from 2nd last letter
(a) BOZIW (b) BOZRW option (a), (d) eliminated.
(c) BOAIW (d) BLZRW 37. Select the letter-cluster from among the given options that
Sol. (a) can replace the question mark (?) in the following series.
Pattern : - letter starting from the last letter of the word fn, x, fodYiksa esa ls ml v{kj&lewg dks pqfu, tks fuEufyf[kr
changed to its opposite letter in each step. Ja[kyk esa iz’uokpd fpUg (?) ds LFkku ij vk ldrk gSA
B O A R D ORGANS, SORGAN, SNORGA, ?, SNAGOR, SNAGRO
(a) SNAROG (b) SNAORG
B O A R W (c) SNOARG (d) SNAOGR
Sol. (b)
B O A I W Pattern : - Last letter is shifted to 1st position then again
last letter is shifted to 2nd position in next step, then again
B O Z I W last letter is shifted to 3rd position in next step and so on.
O R G A N S
B L Z I W
S O R G A N
Y L Z I W
S N O R G A
Elimination : - From 2nd last letter and 3rd letter options (b),
(c), (d) gets eliminated.
(S N A O R G)
35. Select the letter – cluster from among the given options
that can replace the question mark (?) in the following
series. S N A G O R
fn, x, fodYiksa esa ls v{kj & lewg dk p;u djsa tks fuEufyf[kr
J`a[kyk esa ç'u fpà ¼\½ dks çfrLFkkfir dj ldrk gSA S N A G R O
AEJOT, XBGLQ, UYDIN? Elimination : - From last letter option (d), from 2nd last letter
(a) RVBGJ (b) RVBFJ option (a) and from 3rd letter option (c) eliminated.
(c) RUBDG (d) RVAFK 38. Select the letter-cluster from among the given options that
Sol. (d) can replace the question mark (?) in the following series.
Pattern :- Fix subtraction of 3 every letter. fn, x, fodYiksa esa ls ml v{kj&lewg dk p;u djsa tks fuEufyf[kr
–3 –3 –3 J`a[kyk esa ç'u fpà ¼\½ dks çfrLFkkfir dj ldrk gSA
A X U R
CUPBOARD, DVPBOARD, DVQCOARD, ?, DVQCPBSE
–3 –3 –3 (a) DVQCPBSD (b) DVQCPBRD
E B Y V
(c) DVPBPBRD (d) DVQCOASE
–3 –3 –3 Sol. (b)
J G D A
Pattern : - 1st & 2nd letter +1, then in next 3rd & 4th letter +1,
–3 –3 –3
O L I F and so on.
C U P B O A R D
T –3 Q –3 N –3 K +1 +1
Elimination:– From last letter options (a), (b), (c) gets D V P B O A R D
+1 +1
eliminated.
36. Select the letter-cluster from among the given options that D V Q C O A R D
+1 +1
can replace the question mark (?) in the following series.
fn, x, fodYiksa esa ls ml v{kj & lewg dk p;u djs tks (D V Q C P B R D)
+1 +1
fuEufyf[kr Js.kh esa iz’uokpd (?) ds LFkku ij vk ldrh gSA
STGMQ, SGQTM, SQMGT, ? D V Q C P B S E
(a) STQMG (b) SGTQM 39. Select the correct option that will fill in the blank and
(c) SMTQG (d) SMQTG complete the series.

14
Best App for Govt. Jobs : Neonclasses (Download Now)

lgh fodYi dk p;u djsa tks fjä LFkku dks Hkjsxk vkSj J`a[kyk dks I (9)+ K (11) = T (20)
iwjk djsxkA 43. Select the letter-cluster from among the given options that
MIXTURE, IXTURE, IXTUR, _________, XTU can replace the question mark (?) in the following series.
(a) XTRU (b) XTUR ml v{kjkafdd&lewg dh igpku djs]a tks fuEufyf[kr J`a[kyk ls
(c) RTUX (d) TURE lacaf/kr ugha gSA
Sol. (b) F6, H12, J18, L24, ?
Pattern : - 1st letter removed, then in next last letter (a) M30 (b) M12
removed, then in next again 1st letter removed also in next (c) N30 (d) N12
again last letter removed and so on. Sol. (c)
M I X T U R E Pattern : - (Letter +2), (Number +6)
+2 +2 +2 +2
I X T U R E F 6 H 12 J 18 L 24 N 30
+6 +6 +6 +6
I X T U R
Elimination :- Because of letter option (a), (b) & because of
X T U R
number option (d) eliminated.
X T U 44. If the letters in each the following strings are omitted and
Elimination:– From 1st letter option (c) & (d) and from last then the remaining number are arranged in increasing
letter option (a) eliminated. sequence, which number will come in the middle?
40. Select the letter-cluster from among the given options that ;fn fuEufyf[kr çR;sd fLVªax esa v{kjksa dks NksM+ fn;k tkrk gS vkSj
can replace the question mark (?) in the following series. fQj 'ks"k la[;kvksa dks c<+rs Øe esa O;ofLFkr fd;k tkrk gS] rks chp
fn, x, fodYiksa esa ls ml v{kj&lewg dk p;u djsa tks fuEufyf[kr esa dkSu lh la[;k vk,xh\
J`a[kyk esa ç'u fpà ¼\½ dks çfrLFkkfir dj ldrk gSA 33A, 44R, 41V, 54D, 21S, C56, D46, V68, F36, G32, 61C, 47G,
YZ, BCD, FGHI, ?, QRSTUV N55
(a) JKLM (b) LMNOP (a) 46 (b) 44
(c) KLMNO (d) JKLMN (c) 54 (d) 41
Sol. (c) Sol. (a)
Pattern : - (Last letter +2) and word length increased by 1 Accroding to question letters are removed.
in every step. 33, 44, 41, 54, 21, 56, 46, 68, 36, 32, 61, 47, 55
Y Z, B C D, F G H I, K L M N O, Q R S T U V Now numbers are arranged in increasing order.
+2 +2 +2 +2 21, 32, 33, 36, 41, 44, 46, 47, 54, 55, 56, 61, 68
Elimination:– First letter
wrong in option (a), (b) & (d) so eliminated. So 46 is in the middle of the sequence.
41. Select the letter that can replace the question mark (?) in 45. Select the letter-cluster from among the given options that
the following series. can replace the question mark (?) in the following series.
fuEufyf[kr J`a[kyk esa iz’u fpUg (?) dh txg ij vkus okys v{kj ml v{kjkafdd&lewg dh igpku djs]a tks fuEufyf[kr J`a[kyk ls
dk p;u djsAa lacaf/kr ugha gSA
7N9, 14D18, 21G28, 28K39, ?
A, T, E, U, I, V, O, W, ?, X
(a) R (b) M (a) 35P51 (b) 33P51
(c) 35P49 (d) 32P49
(c) U (d) Z
Sol. (a)
Sol. (c)
Pattern : - Vowel series, /kM+du pattern. Pattern :- (1st letter +7) (2nd Number addition in increase
series order +3, +10, +11 ………….
+4 +4 +6 +6
7 14 21 28 35
+7 +7 +7 +7
A T E U I V O W U X
+1 +1 +1 +1 9 18 28 39 51
+9 +10 +11 +12
42. Study the given pattern carefully and select the letter that
(35 P 51)
can replace the question mark (?) in it.
Elimination:– From 1st number options (b) & (d) from 2nd
fn, x, iSVuZ dk /;kuiwoZd v/;;u djsa vkSj ml v{kj dk p;u
number option (c) eliminated.
djsa tks mlesa ç'uokpd fpà (?) dks çfrLFkkfir dj ldsA
46. Select the correct option that will fill in the blank and
complete the series.
lgh fodYi dk p;u djsa tks fjä LFkku dks Hkjsxk vkSj J`a[kyk dks
iwjk djsxkA
P2CT, _________, PCT4, P5CT, PC6T
(a) PC3T (b) P2C3T
(a) T (b) V (c) P2C2T (d) PCT7
(c) U (d) S Sol. (a)
Sol. (a) Pattern :- Number are increased in series order and
Pattern :- (1st letter code + 3rd letter code = 2nd letter code) shifted in next position in every step.
B (2) + H (8) = J (10) P2CT, PC3T, PCT4, P5CT, PC6T
M (13) + E (5) = R (18)

15
Download Free PDFs & e-Books from Neon Classes App

Elimination:– Position of number is right only in option (a) so An inference is something which we get or interpret from the
other three options are eliminated. given statement.
47. Direction:- What should come in place of the question ,d vuqeku ,d ,slh pht gS ftls ge fn, x, dFku ls çkIr djrs
mark(?) in the following letter series? gSa ;k mldh O;k[;k djrs gSaA
fuEufyf[kr v{kj J`[a kyk esa iz’ufpUg ds LFkku ij D;k vk,xk\ Strategy to handle CR questions/lhvkj ç'uksa dks laHkkyus dh
KKMLLNMMONNPOOQP ? j.kuhfr:
(a) PQ (b) ST (c) QV (d) PR 1. Break the CR passage into parts/lhvkj ekxZ dks Hkkxksa esa rksMa+s
+1 +1 +1 +1 +1 If you are having difficulty in understanding the passage, break
the passage into various parts. Identify the conclusion, facts,
KKMLLNMMONNPOOQPPR
and assumptions. This will help in understanding of the
passage using the structural approach for solving the passage.
Sol. (d) +1 +1 +1 +1 +1
;fn vkidks x|ka'k dks le>us esa dfBukbZ gks jgh gS] rks x|ka'k dks
48. ZbYa XdWc VfUe ?
fofHkUu Hkkxksa esa rksM+ nsAa fu"d"kZ] rF;ksa vkSj ekU;rkvksa dks igpkusAa ;g
(a) TgSh (b) ThSg
iSlst dks gy djus ds fy, lajpukRed –f"Vdks.k dk mi;ksx djds
(c) ThSi (d) ShTg
iSlst dks le>us esa enn djsxkA
Sol.
2. Beware of answer choices, which are contradictory to the
Z –2 X –2 V–2 T main idea given in the question
+2 +2 +2 mÙkj ds fodYiksa ls lko/kku jgsa] tks ç'u esa fn, x, eq[; fopkj ds
b d f h foijhr gSa
-2 -2 -2 Always read the question carefully, and be careful of what it is
Y W U S
asking. For example, if the question asks which of the
+2 +2 +2 following weakens the argument, then definitely there will be
a c e g
one option which will be strengthening the argument. Beware
49. NOABOPBCPQCD ? ? ? ? of such answers, these are specially written to confuse the
(a) QRDE (b) RTEF students. If you are careful about this trick, you will be able to
(c) QSDE (D) QRGI save yourself from a lot of incorrect answers, and thereby
Sol. (a) increasing your score.
+1 +1 +1 +1 +1 +1 +1 ç'u dks ges'kk /;ku ls i<+s]a vkSj tks iwNk tk jgk gS mlls lko/kku
NO AB OP BC PQ CD QR DE jgsAa mnkgj.k ds fy,] ;fn ç'u iwNs fd fuEu esa ls dkSu rdZ dks
+1 +1 +1 +1 +1 +1 +1 detksj djrk gS] rks fuf'pr :i ls ,d fodYi gksxk tks rdZ dks
etcwr djsxkA ,sls mÙkjksa ls lko/kku jgs]a ;s fo'ks"k :i ls Nk=ksa dks
Hkzfer djus ds fy, fy[ks x, gSaA ;fn vki bl fVªd ds ckjs esa
lko/kku gSa] rks vki cgqr lkjs xyr mÙkjksa ls [kqn dks cpk ik,axs] vkSj
3. Analyzing argument bl rjg viuk Ldksj c<+k ik,axsA
3. Re-phrase passages into simpler words
x|ka'k dks ljy 'kCnksa esa fQj ls fy[ksa
 Concept - The language given in critical reasoning passages is always
tough and confusing. Hence for your own benefit it is better to
Common Terms used in Critical Reasoning/vkykspukRed rdZ
simplify the language. That ways you will understand the
esa ç;qä lkekU; 'kCn:
question better and will also be able to break the question and
Argument/rdZ:
get rid of the irrelevant data, which might hinder the process
An argument is a set of statements, out of which, one is
of finding the correct answer.
conclusion, another is facts and third is assumptions and the
vkykspukRed rdZ&fordZ esa nh xbZ Hkk"kk ges'kk dfBu vkSj Hkzfer
facts support the conclusion.
djus okyh gksrh gSA blfy, vius ykHk ds fy, Hkk"kk dks ljy cukuk
,d rdZ c;kuksa dk ,d lewg gS] ftlesa ls ,d fu"d"kZ gS] nwljk rF;
csgrj gSA bl rjg vki ç'u dks csgrj <ax ls le> ik,axs vkSj ç'u
gS vkSj rhljk vuqeku gS vkSj rF; fu"d"kZ dk leFkZu djrs gSaA
dks rksM+ Hkh ik,axs vkSj vçklafxd MsVk ls NqVdkjk ik ldsaxs] tks lgh
Conclusion/fu"d"kZ:
mÙkj [kkstus dh çfØ;k esa ck/kk cu ldrk gSA
Conclusion is the main point of an argument, and it is based on
4. Understand what is being asked/tks iwNk tk jgk gS mls le>sa
facts.
A lot of times, the students get really confused about what is
fu"d"kZ rdZ dk eq[; fcanq gksrk gS] vkSj ;g rF;ksa ij vk/kkfjr gksrk
being asked in the question, thus it is imperative to
gSA
understand the question, because of the lack of understanding,
Facts or premises: a premise or fact is a stated evidence that
we tend to mark the wrong answer. Let's discuss a few
supports the conclusion.
examples to understand the Concept better.
rF; ;k ifjlj ,d vk/kkj ;k rF; ,d ?kksf"kr lk{; gS tks fu"d"kZ dk
dbZ ckj] ç'u esa D;k iwNk tk jgk gS] bl ckjs esa Nk= okLro esa Hkzfer
leFkZu djrk gSA
gks tkrs gSa] bl çdkj ç'u dks le>uk vfuok;Z gS] le> dh deh ds
Assumption//kkj.kk:
dkj.k] ge xyr mÙkj dks fpfUgr dj nsrs gSaA vo/kkj.kk dks csgrj
an unstated / hidden fact which supports the conclusion.
<ax ls le>us ds fy, vkb, dqN mnkgj.kksa ij ppkZ djsAa
,d vudgk fNik gqvk rF; tks fu"d"kZ dk leFkZu djrk gSA
Critical Reasoning Questions/fØfVdy jhtfuax ç'u.
Inference/vuqeku:
When the questions become more difficult, you can save time
by remembering that it is not your job to consider whether the

16
Best App for Govt. Jobs : Neonclasses (Download Now)

options are true or not when the question is written this way. lHkh] gj] lc dqN] lc vkfn tSls 'kCnksa dk ç;ksx
The answer to ‘why are you saying this’ will be justified if the 3. Statement/dFku: Jade plant has thick leaves and it requires
‘why’ is based on relevant and logical statem little water.
tc ç'u vf/kd dfBu gks tkrs gSa] rks vki ;g ;kn djds le; cpk tsM ikS/ks dh ifÙk;ka eksVh gksrh gSa vkSj bls de ikuh dh vko';drk
ldrs gSa fd tc ç'u bl çdkj fy[kk tkrk gS rks fodYi lgh gSa ;k gksrh gSA
ugha] bl ij fopkj djuk vkidk dke ugha gSA vki ,slk D;ksa dg jgs Conclusions/Inference:
gSa fd mÙkj mfpr gksxk ;fn D;ksa çklafxd vkSj rkfdZd dFku ij fu"d"kZ/vuqeku
vk/kkfjr gSA I. all plants, with thick leaves, require little water
Short Statement and strengthening argument is asked eksVh ifÙk;ksa okys lHkh ikS/kksa dks de ikuh dh vko';drk gksrh gSA .
laf{kIr oäO; vkSj rdZ dks etcwr djus ds fy, dgk tkrk gS II. Jade plant may be grown in places where water is not in
1. Statement/Conclusion: Chandigarh is a pleasant abundance.
city/dFku/fu"d"k:Z paMhx<+ ,d lq[kn 'kgj gSA. tsM IykaV dks mu txgksa ij mxk;k tk ldrk gS tgka ikuh cgqrk;r
Which of the following, is true, would most strengthen the esa ugha gSA
above conclusion?” (a) Only conclusion I follows/dsoy fu"d"kZ I vuqlj.k djrk gS
fuEufyf[kr esa ls dkSu lk lR; gS] mijksä fu"d"kZ dks lcls vf/kd (b) Only conclusion II follows/dsoy fu"d"kZ II vuqlj.k djrk gS
etcwr djsxk\ (c) Either I or II follows/;k rks I ;k II vuqlj.k djrk gS
I. There are many great galleries in Chandigarh. (d) Neither I nor II follows/u rks I u gh II vuqlj.k djrk gS
paMhx<+ esa dbZ csgrjhu xSyjh gSaA (e) Both I and II follow/I vkSj II nksuksa vuqlj.k djrs gSa
II. Patiala has many great restaurants. Sol. (b)
ifV;kyk esa dbZ csgrjhu jsLVksjVsa gSaA The answer should be II only as I talks about all the plants
(a) Only conclusion I follows/dsoy fu"d"kZ I vuqlj.k djrk gS whereas in the given statement only Jade plant is being talked
(b) Only conclusion II follows/dsoy fu"d"kZ II vuqlj.k djrk gS about.
(c) Either I or II follows/;k rks I ;k II vuqlj.k djrk gS Driven out of statement
(d) Neither I nor II follows/u rks I u gh II vuqlj.k djrk gS 4. Statement/dFku: The best evidence of India’s glorious past
(e) Both I and II follow/I vkSj II nksuksa vuqlj.k djrs gSa is the growing popularity of Ayurvedic medicines in the
Sol. (a) west.
Sometimes people are confused as to whether they have to Hkkjr ds xkSjo'kkyh vrhr dk lcls vPNk çek.k if'pe esa vk;qosZfnd
consider if the options are true or not. You don’t have to do nokvksa dh c<+rh yksdfç;rk gSA
this. You can assume all the options are true. You don’t have to Conclusions/Inference:
ask yourself whether it is actually true that Patiala has many fu"d"kZ/vuqeku
great galleries. It’s clear that Option I would be the right I. Ayurvedic medicines are not popular in India.
answer, because it's the only answer that actually applies to vk;qoZfs nd nok,a Hkkjr esa yksdfç; ugha gSaA
Chandigarh II. Allopathic medicines are more popular in India.
Pointing key words from statement in given ,yksifS Fkd nok,a Hkkjr esa vf/kd yksdfç; gSaA
options/arguments (a) Only conclusion I follows/dsoy fu"d"kZ I vuqlj.k djrk gS
fn, x, fodYiksa esa dFku ls eq[; 'kCnksa dks bafxr djuk (b) Only conclusion II follows/dsoy fu"d"kZ II vuqlj.k djrk gS
2. Statement/dFku: The serious accident in which a person (c) Either I or II follows/;k rks I ;k II vuqlj.k djrk gS
was run down by a car yesterday has again focused (d) Neither I nor II follows/u rks I u gh II vuqlj.k djrk gS
attention on the most unsatisfactory state of roads. (e) Both I and II follow/I vkSj II nksuksa vuqlj.k djrs gSa
xaHkhj nq?kZVuk ftlesa ,d O;fä dy ,d dkj ls uhps fxj x;k Fkk] us Sol. (d)
fQj ls lM+dksa dh lcls vlarks"ktud fLFkfr ij /;ku dsfa ær fd;k The popularity of Ayurvedic or allopathic medicines in India is
gSA not being talked about in the statement. So, neither I nor II
Conclusions/Inference/fu"d"kZ/vuqeku: follows.
I. The accident that occurred was fatal/tks nq?kZVuk gqbZ og
?kkrd FkhA. Weaken the Statement / dFku dks detksj
II. Several accidents have so far taken place because of
unsatisfactory state of roads. djsa
lM+dksa dh vlarks"ktud fLFkfr ds dkj.k vc rd dbZ nq?kZVuk,a gks 5. Diamonds are a more popular investment among the
pqdh gSaA extremely rich than are yachts. Ten thousand times as
(a) Only conclusion I follows/dsoy fu"d"kZ I vuqlj.k djrk gS many people, who can afford both, buy a diamond as a
(b) Only conclusion II follows/dsoy fu"d"kZ II vuqlj.k djrk gS yacht.
(c) Either I or II follows/;k rks I ;k II vuqlj.k djrk gS ukSdkvksa dh rqyuk esa vR;fèkd èkuh yksxksa ds chp ghjs vfèkd yksdfç;
(d) Neither I nor II follows/u rks I u gh II vuqlj.k djrk gS fuos'k gSaA nl gtkj xquk vfèkd yksx] tks nksuksa dk [kpZ mBk ldrsA
(e) Both I and II follow/I vkSj II nksuksa vuqlj.k djrs gSa ,d ghjk dks ,d ;‚V ds :i esa [kjhnrs gSaA
Sol. (e) The answer should be both I and II as both can be Which of the following, if true, would most weaken the
concluded or driven from the given statement. For the first argument above?
conclusion the key words are ‘run down’ and for the second fuEufyf[kr esa ls dkSu lk] ;fn lR; gS] rks mijksä rdZ dks lcls
conclusion the key words are ‘unsatisfactory state of roads’. vfèkd detksj djsxk\
Use of words like All, every, Everything,Everyone etc

17
Download Free PDFs & e-Books from Neon Classes App

(a) Owning a private yacht is more prestigious than owning a djuh pkfg,A ;fn vki ,d xgu ,jksfcd dljr dh ryk'k dj jgs
diamond. gSa] rks ,d ;ksx d{kk vkidh vko';drkvksa dks iwjk ugha djsxhA
,d futh ukSdk dk ekfyd gksuk ghjs ds ekfyd gksus ls T;knk This paragraph supports which of the following
çfrf"Br gSA statements;
(b) There are fewer yacht brokers than there are diamond ;g vuqPNsn fuEufyf[kr esa ls fdl dFku dk leFkZu djrk gSA
brokers. (a) Yoga provides both mental and health benefits
ghjk nykyksa dh rqyuk esa de ukSdk nyky gSaA ;ksx ekufld vkSj LokLF; ykHk çnku djrk gSA
(c) It is harder to resell a yacht than to resell a diamond. (b) Yoga is less popular than aerobics
,d ghjs dks fQj ls cspus dh rqyuk esa ,d ukSdk dks fQj ls cspuk ;ksx ,jksfcDl dh rqyuk esa de yksdfç; gSA
dfBu gSA (c) Understand your needs before joining a fitness class
(d) The ratio of people who can afford diamonds to the people fQVusl Dykl esa 'kkfey gksus ls igys viuh vko';drkvksa dks le>sAa
who can afford yachts is a million to one. (d) Yoga is not a rigorous form of exercise
tks yksx ghjs [kjhn ldrs gSa vkSj tks ukSdk [kjhn ldrs gSa] mudk ;ksx O;k;ke dk dBksj :i ugha gSA
vuqikr ,d fefy;u ls ,d gSA Sol. (c)
Sol. (d) In this paragraph, the author is saying that we should evaluate
If more people are buying diamonds then there will be our fitness needs before starting a fitness class. He is not
oversupply of the diamonds in the market. This in turn will discussing the benefits of yoga, not comparing yoga with
decrease the value (in terms of selling) of the diamonds in the aerobics, also not telling what kind of exercise yoga is.
market. Hence there is no point in spending money on the Weaken the Author’s contention
diamonds. On the other hand the availability of 'yachts' is less 8. French cuisine is highly regarded all over the world. Yet in
in the market so its value (in terms of buying/selling) is more Paris there are more American restaurants selling
in the market. Therefore if money is invested in yacht then this burgers and fries (which many people now class as junk
will increase/multiply the invested money. This is clearly food) than there are in any other European capital city.
shown by option 4. Obviously the French are very fond of junk food, and are
Supports the Statement not too proud to eat it.
6. If you are a fitness walker, you don't need to go to a health Ýspa O;atu dks iwjh nqfu;k esa vR;fèkd ekuk tkrk gSA fQj Hkh isfjl esa
gym. You also don't need any fitness equipment for fdlh Hkh vU; ;wjksih; jktèkkuh 'kgj dh rqyuk esa vfèkd vesfjdh
workout. All you need is a pair of comfortable athletic jsLrjka cxZj vkSj Ýkbt+ ¼ftUgsa vc dbZ yksx tad QwM ds :i esa
shoes. oxÊ—r djrs gSa½ csprs gSaA tkfgj gS] Ýsp
a tad QwM ds cgqr 'kkSdhu gSa]
;fn vki ,d fQVusl o‚dj gSa] rks vkidks LokLF; fte tkus dh vkSj bls [kkus esa cgqr xoZ ugha gSA
vko';drk ugha gSA vkidks dljr ds fy, fdlh fQVusl midj.k Which of the following, if true, would most weaken the
dh Hkh vko';drk ugha gSA vkidks cl vkjkenk;d ,FkysfVd twrksa dh author’s contention?
,d tksM+h pkfg,A fuEufyf[kr esa ls dkSu lk] ;fn lR; gS] rks ys[kd ds rdZ dks lcls
This paragraph supports which of the following vfèkd detksj djsxk\
statements; (a) There are also a larger number of Lebanese restaurants in Paris
;g vuqPNsn fuEufyf[kr esa ls fdl dFku dk leFkZu djrk gSA than there are in other European capital cities.
(a) Fitness walking is better than weight lifting. isfjl esa vU; ;wjksih; jktèkkuh 'kgjksa dh rqyuk esa cM+h la[;k esa
osV fyf¶Vax dh rqyuk esa fQVusl ?kweuk csgrj gSA yscukuh jsLrjka gSaA
(b) Walking outdoors provides more health benefits than walking (b) French Cordon Bleu cuisine is very expensive.
indoors. Ýsp
a d‚MZu Cyw O;atu cgqr egaxk gSA
?kj ds ckgj pyus ls ?kj ds vanj pyus dh rqyuk esa vf/kd LokLF; (c) The number of French tourists eating in New York burger
ykHk feyrk gS A restaurants is very low.
(c) Fitness walking is an effective and convenient form of exercise. U;w ;‚dZ cxZj jsLrjka esa [kkus okys Ýkalhlh i;ZVdksa dh la[;k cgqr
fQVusl pyuk O;k;ke dk ,d çHkkoh vkSj lqfo/kktud rjhdk gSA de gSA
(d) Poorly designed shoes can cause foot injuries. (d) Junk food is actually has high nutritional value when eaten in
[kjkc fMtkbu okys twrs iSjksa dh pksV dk dkj.k cu ldrs gSaA moderation.
Sol. (c) tad QwM dk okLro esa mPp iks"k.k ewY; gksrk gS tc bls de ek=k esa
In this paragraph, the author is talking about the convenience [kk;k tkrk gSA
of fitness walking. He is saying that fitness walking is a (e) There are an unusually large number of American tourists in
workout that does not need equipment and membership of a Paris who eat at burger joints.
health gym. isfjl esa vlkekU; :i ls cM+h la[;k esa vesfjdh i;ZVd gSa tks cxZj
7. Yoga is a popular form of exercise, but it is not for tksM+ksa esa [kkrs gSaA
everyone. Before you enroll yourself for a yoga class, you Sol. (e)
should examine what you want from your fitness routine. The author’s contention (argument) is that the French are very
If you are looking for an intense aerobic workout, a yoga fond of junk food because there are so many American
class will not fulfill your needs. restaurants in Paris. The best way to defeat this argument is to
;ksx O;k;ke dk ,d yksdfç; :i gS] ysfdu ;g gj fdlh ds fy, show, if possible, that the French do not eat in those American
ugha gSA blls igys fd vki [kqn dks ,d ;ksxk Dykl ds fy, restaurants. The closest to that is answer E which suggests
nkf[kyk ys]a vkidks viuh fQVusl :Vhu ls D;k pkfg,] bldh tkap that the American tourists are the ones who eat at those
restaurants.

18
Best App for Govt. Jobs : Neonclasses (Download Now)

Most Suitable conclusion Qly Hkwfe dh mRikndrk dks de djrk gS vkSj tylzksrksa] vknZzHkwfe;ksa
9. India is a home to a number of varied species of flora and vkSj >hyksa ds çnw"k.k dk dkj.k curk gSA
fauna. Lots of measures are taken to conserve wildlife in Which among the following options does not show the
India. Wildlife conservation is the practice of protecting step taken to prevent this?
wild plant and animal species and their habitat. Wildlife fuEufyf[kr esa ls dkSu lk fodYi bls jksdus ds fy, mBk, x, dne
plays an important role in balancing the environment and dks ugha n'kkZrk gS\
provides stability to different natural processes of nature. (a) Using of contour ploughing will help reduce soil erosion.
Government of India has taken several measure for leksPp tqrkbZ ds mi;ksx ls feêh ds dVko dks de djus esa enn
conservation of wildlife. feysxhA
Hkkjr ouLifr;ksa vkSj thoksa dh dbZ fofo/k çtkfr;ksa dk ?kj gSA Hkkjr (b) Soil erosion is a result of mankind's unwise actions such as
esa oU;thoksa ds laj{k.k ds fy, cgqr lkjs mik; fd, tkrs gSaA oU;tho overgrazing or unsuitable cultivating practices.
laj{k.k taxyh ikS/kksa vkSj tkuojksa dh çtkfr;ksa vkSj muds vkokl dh e`nk vijnu ekuo tkfr ds ukle> dk;ksZa tSls vfrpkj.k ;k vuqi;qä
j{kk djus dh çFkk gSA oU;tho i;kZoj.k dks larqfyr djus esa egRoiw.kZ [ksrh çFkkvksa dk ifj.kke gSA
Hkwfedk fuHkkrs gSa vkSj ç—fr dh fofHkUu çk—frd çfØ;kvksa dks fLFkjrk (c) Planting provides a protective cover on the land and slow
çnku djrs gSaA Hkkjr ljdkj us oU;thoksa ds laj{k.k ds fy, dbZ down water as it flows over the land and this allows much of
mik; fd, gSaA the rain water to soak into the ground.
Which among the following options shows the reason jksi.k Hkwfe ij ,d lqj{kkRed vkoj.k çnku djrk gS vkSj Hkwfe ds Åij
why we need to save wildlife? cgus okys ikuh dks /khek dj nsrk gS vkSj blls ckfj'k dk vf/kdka'k
fuEufyf[kr fodYiksa esa ls dkSu lk dkj.k crkrk gS fd gesa oU; thou ikuh tehu esa lek tkrk gSA
dks cpkus dh vko';drk D;ksa gS\ (d) Use of proper drainage canals.
(a) Saving wildlife plays a great role in ensuring a check on the ty fudklh ugjksa dk mfpr mi;ksxA
ecological imbalance. (e) Terracing of the sloping fields.
ikfjfLFkfrd vlarqyu dh tkap lqfuf'pr djus esa oU;thoksa dks cpkuk <yku okys [ksrksa dh NrA
,d cM+h Hkwfedk fuHkkrk gSA Sol. (b)
(b) Scientists are working to preserve plants and animals through Option (b) would be the correct solution as only that option
'Gene banks'. does not talk about ways on how to prevent soil erosion. It
oSKkfud thu cSad ds tfj, ikS/kksa vkSj tkuojksa dks lajf{kr djus dk talks about one of the reasons which causes it. There are
dke dj jgs gSaA natural ways as well as some human activities that will result
(c) People are highly dependent on wildlife industry for fruits, in soil erosion. Human activities include over grazing the fields
vegetables, grains, wood, clothes, and such. and such.
yksx Qy] lfCt;ka] vukt] ydM+h] diM+s vkfn ds fy, oU;tho m|ksx Other 4 options talks about methods by which soil erosion can
ij vR;f/kd fuHkZj gSaA be reduced or prevented.
(d) Wildlife beautifies the Earth in many ways. Challenges faced against the
oU;tho dbZ rjg ls i`Foh dks lq'kksfHkr djrs gSaA Theme
(e) Measures are taken so that various species don't become 11. The critical reasoning question is based on a short
extinct. argument, a set of statements, or a plan of action. For each
mik; fd, tkrs gSa rkfd fofHkUu çtkfr;ka foyqIr u gksAa question, select the best answer of the choices given.
Sol. (a) fØfVdy jhtfuax ç'u ,d laf{kIr rdZ] c;kuksa ds ,d lsV ;k dk;Z
Option (a) would be the correct answer as it gives the main ;kstuk ij vk/kkfjr gSA çR;sd ç'u ds fy, fn, x, fodYiksa esa ls
reason on why wildlife should be saved and why the loZJs"B mÙkj dk p;u djsAa
government should take actions regarding preserving them. It Till recently only cricket was emphasized in India. But
tells us that wildlife does play a great and significant role in now several other sports got attention and are widely
ensuring a check on the ecological imbalance as it offers the spread. Globally, sports is one of the largest industries. In
earth a lot of things in many ways. India, it is in the path to become one. As India has more
Other options are not relevant to the question asked thus number of young people, there is more potential to utilize
cannot be considered. sports talent in youth and to become one of the sports
Steps taken to prevent superpowers.
something vHkh rd Hkkjr esa dsoy fØdsV ij tksj fn;k tkrk FkkA ysfdu vc
10. Soil erosion is a naturally occurring process that affects all dbZ vU; [ksyksa us /;ku vkdf"kZr fd;k vkSj O;kid :i ls QSy x,A
land forms. It refers to the wearing away of a fields topsoil fo'o Lrj ij] [ksy lcls cM+s m|ksxksa esa ls ,d gSA Hkkjr esa ;g ,d
by natural physical forces of water and wind. Topsoil cuus dh jkg ij gSA pwafd Hkkjr esa ;qokvksa dh la[;k vf/kd gS]
which is high in organic matter, fertility and soil life which blfy, ;qokvksa esa [ksy çfrHkk dk mi;ksx djus vkSj [ksy egk'kfä;ksa
majorly contributes on the growing of crops. Soil erosion esa ls ,d cuus dh vf/kd laHkkouk gSA
reduces cropland productivity and causes the pollution of Future of sports in India faces a lot of challenges which
watercourses, wetlands and lakes. are listed below. Select the one which doesn't talk about
e`nk vijnu ,d LokHkkfod :i ls gksus okyh çfØ;k gS tks lHkh Hkwfe the challenges faced by sports in the country.
:iksa dks çHkkfor djrh gSA ;g ikuh vkSj gok dh çk—frd HkkSfrd Hkkjr esa [ksyksa dk Hkfo"; cgqr lkjh pqukSfr;ksa dk lkeuk djrk gS tks
'kfä;ksa }kjk ,d [ksr dh Åijh feêh ds f?klus dks lanfHkZr djrk gSA uhps lwphc) gSaA mls pqusa tks ns'k esa [ksyksa ds lkeus vkus okyh
Åijh feêh tks dkcZfud inkFkZ] moZjrk vkSj feêh ds thou esa mPp pqukSfr;ksa ds ckjs esa ckr ugha djrk gSA
gksrh gS tks Qlyksa ds c<+us esa çeq[k ;ksxnku nsrh gSA e`nk vijnu (a) Lack of infrastructures.

19
Download Free PDFs & e-Books from Neon Classes App

cqfu;knh <kaps dk vHkkoA (c) It helped in spreading important mass movements like Swacha
(b) Lack of Sports schools, especially in rural areas. Bharat.
fo'ks"k :i ls xzkeh.k {ks=ksa esa [ksy fo|ky;ksa dk vHkkoA blus LoPN Hkkjr tSls egRoiw.kZ tu vkanksyuksa dks QSykus esa enn
(c) Low salaries of Sport Coaches and staff in sports filed. dhA
[ksy çf'k{kdksa vkSj [ksy dfeZ;ksa dk de osru nkf[kyA (d) Because of it becoming successful "Mann Ki Baat" now has its
(d) Indian government gives cash prizes and awards to the own mobile application that promotes its new topics and
winners of national sports events. issues faced by the country.
Hkkjr ljdkj jk"Vªh; [ksy vk;kstuksa ds fotsrkvksa dks udn iqjLdkj ^eu dh ckr* ds lQy gksus ds dkj.k vc bldk viuk eksckby
vkSj iqjLdkj nsrh gSA ,fIyds'ku gS tks ns'k ds lkeus vkus okys vius u, fo"k;ksa vkSj eqíksa
(e) As it is difficult to succeed in making a career in sports in India, dks c<+kok nsrk gSA
most of the parents are not encouraging their children to take (e) Narendra Modi tried uplifting the employment rate within the
so. lower sections of the society.
pwafd Hkkjr esa [ksyksa esa dfj;j cukus esa lQy gksuk eqf'dy gS] ujsæa eksnh us lekt ds fupys oxksZa ds Hkhrj jkstxkj nj dks Åij
vf/kdka'k ekrk&firk vius cPpksa dks ,slk djus ds fy, çksRlkfgr ugha mBkus dh dksf'k'k dhA
dj jgs gSaA Sol. (e)
Sol. (d) Option (e) would be the correct solution which does not
Option 4 would be the correct answer as it is the only option showcase the impacts of this campaign. It talks about the PM
which does not talk about the challenges faced by sports in the trying to help the lower sections of the society by uplifting the
country. It talks about how the government of the country employment rate but it does not show if this was because of
felicitates the winners of different sports by giving them cash "Mann ki Baat campaign".
prizes and awards. Options 1 to 4 show us the impacts of this campaign such as
Options 1 to 3 and 5 talk about the challenges faced by the bridging the gap between the old and new aspects of the
future of sports in the country such as it lacking in country, and Swacha Bharat, etc.
infrastructure, lack of sports schools, coaches and staff getting 13. The critical reasoning question is based on a short
low salaries, and such. argument, a set of statements, or a plan of action. For each
Negative Impact question, select the best answer of the choices given.
12. The critical reasoning question is based on a short fØfVdy jhtfuax ç'u ,d laf{kIr rdZ] c;kuksa ds ,d lsV ;k dk;Z
argument, a set of statements, or a plan of action. For each ;kstuk ij vk/kkfjr gSA çR;sd ç'u ds fy, fn, x, fodYiksa esa ls
question, select the best answer of the choices given. loZJs"B mÙkj dk p;u djsAa
fØfVdy jhtfuax ç'u ,d laf{kIr rdZ] c;kuksa ds ,d lsV ;k dk;Z The banking industry in India is rapidly progressing with
;kstuk ij vk/kkfjr gSA çR;sd ç'u ds fy, fn, x, fodYiksa esa ls increased customer base and due to newly improved and
loZJs"B mÙkj dk p;u djsAa innovative facilities offered by technology. The impact of
Prime Minister Narendra Modi started a campaign called technology on the Indian banking sector has both
"Mann ki Baat" on 3rd October 2014. It turned out to be a positive and negative impacts. The risks are high, as it
popular show on the Indian radio stations. This show was can be very risky that the data can be hacked.But this risk
broadcasted for one hour once a month as the Prime can be minimized and controlled by taking some
Minister addresses the masses and encourages them to important measures that will ensure to protect the data.
work to the benefit of the country. Some of the people Hkkjr esa cSafdax m|ksx rsth ls c<+rs xzkgd vk/kkj ds lkFk vkSj
from many villages would write letters to this radio çkS|ksfxdh }kjk is'k dh xbZ ubZ mUur vkSj uohu lqfo/kkvksa ds dkj.k
station from which the PM would select and provide a rsth ls çxfr dj jgk gSA Hkkjrh; cSafdax {ks= ij çkS|ksfxdh ds çHkko
solid solution to his listener. ds ldkjkRed vkSj udkjkRed nksuksa çHkko gSaA tksf[ke vf/kd gSa]
ç/kku ea=h ujsæa eksnh us 3 vDVwcj 2014 dks "eu dh ckr" uked D;ksfa d ;g cgqr tksf[ke Hkjk gks ldrk gS fd MsVk gSd fd;k tk
,d vfHk;ku 'kq: fd;kA ;g Hkkjrh; jsfM;ks LVs'kuksa ij ,d yksdfç; ldrk gSA ysfdu dqN egRoiw.kZ mik; djds bl tksf[ke dks de
'kks cu x;kA ;g 'kks eghus esa ,d ckj ,d ?kaVs ds fy, çlkfjr fd;k fd;k tk ldrk gS vkSj fu;af=r fd;k tk ldrk gS tks MsVk dh
tkrk Fkk D;ksfa d ç/kku ea=h turk dks lacksf/kr djrs gSa vkSj mUgsa ns'k lqj{kk lqfuf'pr djsxkA
ds ykHk ds fy, dke djus ds fy, çksRlkfgr djrs gSaA dbZ xk¡oksa ds Given below are few of the impacts of technology on the
dqN yksx bl jsfM;ks LVs'ku dks i= fy[krs Fks] tgk¡ ls ç/kkuea=h pqus banking sector, select the negative impact among them.
tkrs vkSj vius Jksrk dks ,d Bksl lek/kku çnku djrsA cSafdax {ks= ij çkS|ksfxdh ds dqN çHkko uhps fn, x, gSa] muesa ls
Listed below are few impacts of this campaign. Select the udkjkRed çHkko dk p;u djsAa
option which does not showcase one of the impacts. (a) Banking processes are faster than before.
uhps lwphc) dqN fodYi bl vfHk;ku ds dqN çHkko gSaA ml fodYi cSafdax çfØ;k,a igys dh rqyuk esa rst gSaA
dk p;u djsa tks fdlh ,d çHkko dks çnf'kZr ugha djrk gSA (b) Maintenance of documents have become very easy and faster
(a) It played a vital role in bridging the gap between the Old India to access them.
and today's New India. nLrkostksa dk j[kj[kko mu rd igqapuk cgqr vklku vkSj rst gks x;k
blus iqjkus Hkkjr vkSj vkt ds u, Hkkjr ds chp dh [kkbZ dks ikVus esa gSA
egRoiw.kZ Hkwfedk fuHkkbZA (c) Increase in unemployment as people are losing their jobs due
(b) This turned out to be one of the most successful initiatives to automation taking their positions.
taken by Narendra Modi. csjkstxkjh c<+ jgh gS D;ksfa d v‚Vkse's ku ds dkj.k yksxksa dh ukSdfj;ka
;g ujsæa eksnh }kjk dh xbZ lcls lQy igyksa esa ls ,d lkfcr gqbZA tk jgh gSaA
(d) The scope of frauds in banks is minimized.

20
Best App for Govt. Jobs : Neonclasses (Download Now)

cSadksa esa /kks[kk/kM+h dk nk;jk de ls de gksA introduction of new technologies, better trades and got
(e) With introduction of mobile banking everything is one quick positive changes on the political and cultural domains.
tap away. Disadvantage
eksckby cSafdax dh 'kq#vkr ds lkFk lc dqN ,d Rofjr VSi nwj gSA 15. The critical reasoning question is based on a short
Sol. (c) argument, a set of statements, or a plan of action. For each
Option (c) would be the correct answer as it is the only option question, select the best answer of the choices given.
which talks about the negative impact of technology being fØfVdy jhtfuax ç'u ,d laf{kIr rdZ] c;kuksa ds ,d lsV ;k dk;Z
used in banking sectors. It is true that as automation are taking ;kstuk ij vk/kkfjr gSA çR;sd ç'u ds fy, fn, x, fodYiksa esa ls
the place many people tend to lose their jobs and thus loZJs"B mÙkj dk p;u djsAa
unemployment increases. The importance of education and learning has been
Technology being used in the banking sectors has both recognized since the beginning of time. Education means
advantages as well as disadvantages. The other options talk a lot in everyone's life as it facilitates our learning,
about the advantages, hence are not correct. knowledge and skill. Education must be given more
14. The critical reasoning question is based on a short importance than any other targets as it is the main thing
argument, a set of statements, or a plan of action. For each which shapes one's career. Education has a lot of
question, select the best answer of the choices given. advantages. Education is a very good tool which benefits
fØfVdy jhtfuax ç'u ,d laf{kIr rdZ] c;kuksa ds ,d lsV ;k dk;Z all throughout life.
;kstuk ij vk/kkfjr gSA çR;sd ç'u ds fy, fn, x, fodYiksa esa ls f'k{kk vkSj f'k{kk ds egRo dks vkfndky ls gh ekU;rk nh xbZ gSA f'k{kk
loZJs"B mÙkj dk p;u djsAa gj fdlh ds thou esa cgqr ek;us j[krh gS D;ksfa d ;g gekjs lh[kus]
Globalization consists of the social, political and Kku vkSj dkS'ky dks lqxe cukrh gSA fdlh Hkh vU; y{; dh rqyuk
economic changes that we all adapt to. The strong esa f'k{kk dks vf/kd egRo fn;k tkuk pkfg, D;ksfa d ;g eq[; pht gS
currency rates, constructions, trading are all tks fdlh ds dfj;j dks vkdkj nsrh gSA f'k{kk ds cgqr Qk;ns gSaA
consequences of globalization. There are both positive f'k{kk ,d cgqr vPNk lk/ku gS tks thou Hkj ykHkkfUor djrh gSA
and negative impacts of this globalization on the Few options are listed below, select the one which isn't
economy and it leads to the interconnection between the one of the advantages of education on humans.
countries of different cultures, caste and heritage. dqN fodYi uhps lwphc) gSa] muesa ls ,d dk p;u djsa tks euq";ksa ij
Globalization leads to the usage of new innovative ideas f'k{kk ds ykHkksa esa ls ,d ugha gSA
of new technology that people tend to start using. (a) It develops confidence and helps building personality of a
oS'ohdj.k esa lkekftd] jktuhfrd vkSj vkfFkZd ifjorZu gksrs gSa ftUgsa person.
ge lHkh vuqdwfyr djrs gSaA etcwr eqæk njs]a fuekZ.k] O;kikj lHkh ;g vkRefo'okl fodflr djrk gS vkSj O;fä ds O;fäRo ds fuekZ.k esa
oS'ohdj.k ds ifj.kke gSaA vFkZO;oLFkk ij bl oS'ohdj.k ds ldkjkRed enn djrk gSA
vkSj udkjkRed nksuksa çHkko gSa vkSj ;g fofHkUu laL—fr;ks]a tkfr vkSj (b) It makes us strong mentally, socially and intellectually.
fojklr ds ns'kksa ds chp ijLij laca/k dh vksj ys tkrk gSA oS'ohdj.k ;g gesa ekufld] lkekftd vkSj ckSf)d :i ls etcwr cukrk gSA
ubZ rduhd ds u, uohu fopkjksa ds mi;ksx dh vksj ys tkrk gS (c) It brings feeling of equality among all people in the society.
ftldk yksx mi;ksx djuk 'kq: dj nsrs gSaA ;g lekt esa lHkh yksxksa ds chp lekurk dh Hkkouk ykrk gSA
Given below are the impacts of Globalization, select the (d) It is been listen that every human has a right to education.
negative impact among them. lquus esa vk;k gS fd gj balku dks f'k{kk dk vf/kdkj gSA
oS'ohdj.k ds çHkko uhps fn, x, gSa] muesa ls udkjkRed çHkko dk (e) Education makes a person capable of making him stand
p;u djsAa strong in the society.
(a) It introduces new technologies. f'k{kk O;fä dks lekt esa etcwr cukus esa l{ke cukrh gSA
;g ubZ rduhdksa dk ifjp; nsrk gSA Sol. (d)
(b) It has brought an impact on political and cultural domain. Option (d) would be the correct answer as it does not talk
blus jktuhfrd vkSj lkaL—frd {ks= ij çHkko Mkyk gSA about how education has an impact on the humans. It talks
(c) Because of too much exchange of trade, money etc about the government having a rule (i.e.,) right to education,
independent domestic policies are lost. which says that any child whether from a very low class of the
O;kikj] /ku vkfn ds cgqr vf/kd vknku&çnku ds dkj.k Lora= ?kjsyw society has right to education.
uhfr;ka [kks tkrh gSaA Other options show how by being educated one is being able
(d) Globalization has introduced many different ideologies to stand strong in the society. By education one individual
among people. gains confidence, helps building up his personality, makes him
oS'ohdj.k us yksxksa ds chp dbZ vyx&vyx fopkj/kkjkvksa dks is'k mentally strong and such.
fd;k gSA 16. The critical reasoning question is based on a short
(e) It introduces better trade. argument, a set of statements, or a plan of action. For each
;g csgrj O;kikj dk ifjp; nsrk gSA question, select the best answer of the choices given.
Sol. (c) fØfVdy jhtfuax ç'u ,d laf{kIr rdZ] c;kuksa ds ,d lsV ;k dk;Z
Option (c) is the correct answer as it is the only option which ;kstuk ij vk/kkfjr gSA çR;sd ç'u ds fy, fn, x, fodYiksa esa ls
shows the negative impact of globalization. Due to excess loZJs"B mÙkj dk p;u djsAa
exchange of trade and money, it has effected the independent Pesticides are used to control various insects and disease
domestic policies. Other options talk about only the positive carriers such as rats, ticks and mice. They are used in
impacts that are caused because of globalization such as the agriculture to control weeds and fungi too. A pesticide is
a chemical or biological agent that kills pests. Although

21
Download Free PDFs & e-Books from Neon Classes App

pesticides have benefits, some have drawbacks too if they II: Home Affairs Minister Rajnath Singh has directed the
are used in larger quantities. states to identify Rohingyas, collect their biometrics to
dhVuk'kdksa dk mi;ksx fofHkUu dhVksa vkSj jksx okgd tSls pwgks]a fVDl Myanmar.
vkSj pwgksa dks fu;af=r djus ds fy, fd;k tkrk gSA budk mi;ksx — x`g ekeyksa ds ea=h jktukFk flag us jkT;ksa dks jksfgaX;kvksa dh igpku
f"k esa [kjirokj vkSj dod dks fu;af=r djus ds fy, Hkh fd;k tkrk djus] E;kaekj esa muds ck;ksefs VªDl ,d= djus dk funsZ'k fn;k gSA
gSA dhVuk'kd ,d jklk;fud ;k tSfod ,tsVa gS tks dhVksa dks ekjrk (a) Only I/dsoy I
gSA ;|fi dhVuk'kdksa ds ykHk gSa] dqN esa dfe;ka Hkh gSa ;fn mudk (b) Only II/dsoy II
vf/kd ek=k esa mi;ksx fd;k tkrk gSA (c) None of the above/mijksDr esa ls dksbZ ugha
Given below are some options, select the one which is not (d) Either I or II/;k rks I ;k II
one of the the advantages of pesticides. (e) Both I and II/ I vkSj II nksuksa
uhps dqN fodYi fn, x, gSa] muesa ls ,d dk p;u djsa tks Sol. (b)
dhVuk'kdksa ds ykHkksa esa ls ,d ugha gSA Only II. Because as given in the statement, the Home Affairs
(a) They are used to kill mosquitoes that can transmit potentially Minister has directed all the states to identify Rohingyas,
deadly diseases like malaria and fever and such. collect their biometrics and submit the reports to the Centre
mudk mi;ksx ePNjksa dks ekjus ds fy, fd;k tkrk gS tks eysfj;k (and not Myanmar), and then India will resolve the issue with
vkSj cq[kkj tSlh laHkkfor ?kkrd chekfj;ksa dks çlkfjr dj ldrs gSaA Myanmar. So, the option 2 is the answer.
(b) They can also used to kill wasps or ants which causes allergic 18. NASA plans for Americans to orbit the Moon and send
reactions. astronauts to Mars. It has unveiled its plan for building
os rrS;k ;k phafV;ksa dks Hkh ekj ldrs gSa tks ,ythZ dk dkj.k curh commercial spaceflight partnerships, long-term human
gSaA deployment on and around the Moon.
(c) They are used to kill weeds that may cause environmental uklk us vesfjfd;ksa ds fy, paæek dh ifjØek djus vkSj varfj{k
damage. ;kf=;ksa dks eaxy xzg ij Hkstus dh ;kstuk cukbZ gSA blus okf.kfT;d
mudk mi;ksx mu [kjirokjksa dks ekjus ds fy, fd;k tkrk gS tks Lisl¶ykbV lk>snkjh] paæek ij vkSj mlds vklikl yach vof/k ds
i;kZoj.kh; {kfr dk dkj.k cu ldrs gSaA ekuo rSukrh ds fuekZ.k ds fy, viuh ;kstuk dk vukoj.k fd;k gSA
(d) Pesticides are poisons and can cause a number of health Which of the following do not support the statement?
effects on humans. fuEufyf[kr esa ls dkSu bl dFku dk leFkZu ugha djrk gS\
dhVuk'kd tgj gSa vkSj euq";ksa ij dbZ LokLF; çHkko iSnk dj ldrs I: NASA has plans for building commercial spaceflight
gSaA partnerships.
(e) DDT, sprayed on the walls of houses that fights malaria. uklk dh okf.kfT;d varfj{k&mM+ku lk>snkjh cukus dh ;kstuk gSA
eysfj;k ls yM+us okys ?kjksa dh nhokjksa ij MhMhVh dk fNM+dko fd;k II: NASA plans for Americans to orbit the Moon.
tkrk gSA uklk us vesfjfd;ksa ds fy, paæek dh ifjØek djus dh ;kstuk cukbZ
Sol. (d) gSA
Option (d) is the correct answer as it talks about how III: NASA has unveiled its plan for long-term human deployment
pesticides affect the humans. Pesticides are toxic and when on and around the Mars.
used more then a certain amount can cause a lot of issues to uklk us eaxy xzg ij vkSj mlds vklikl yach vof/k ds ekuo rSukrh
humans. If pesticides are used in more quantity in agriculture ds fy, viuh ;kstuk dk vukoj.k fd;k gSA
then the crops consumed by humans will have a very bad (a) Only I/dsoy I
effect on their health. (b) Only II/dsoy II
Other options talk about the advantages of using pesticides, (c) Only III/dsoy III
such as they are used to kill insects as wasps, ants or (d) All of these/;s lHkh
mosquitoes. They are also used to kill weeds. A common (e) None of these/buesa ls dksbZ ugha
pesticide DDT is sprayed on the house walls which prevents Sol. (c)
malaria. Only III as NASA has unveiled its plan for long-term human
Against the statement deployment on and around the Moon. Hence, option 3 is the
17. Home Affairs Minister Rajnath Singh has directed all the answer.
states to identify Rohingyas, collect their biometrics and Not-in-Line with Statement
submit the reports to the Centre. India will then take 19. The UN Secretary-General Antonio Guterres said that
action through diplomatic channels with Myanmar to India’s initiatives such as ‘Save the Girl Child, Educate the
resolve the issue. Girl Child’ are emblematic of the types of actions needed
x`g ea=h jktukFk flag us lHkh jkT;ksa dks jksfgaX;kvksa dh igpku djus] to achieve Sustainable Development Goals.
muds ck;ksefs VªDl ,d= djus vkSj dsæa dks fjiksVZ tek djus dk la;qä jk"Vª egklfpo ,aVksfu;ks xqVsjsl us dgk fd Hkkjr dh igy tSls
funs'Z k fn;k gSA blds ckn Hkkjr bl eqís dks lqy>kus ds fy, E;kaekj fd lso n xyZ pkbYM] ,tqdsV n xyZ pkbYM lrr fodkl y{;ksa dks
ds lkFk jktuf;d ek/;eksa ls dkjZokbZ djsxkA çkIr djus ds fy, vko';d dk;ksaZ ds çdkj ds çrhd gSaA
Which of the following do not support the statement? Which of the following is not-in-line with the given
fuEufyf[kr esa ls dkSu bl dFku dk leFkZu ugha djrk gS\ statement?
I: India will resolve the issue of Rohingyas through diplomatic fuEufyf[kr esa ls dkSu fn, x, dFku ds vuq:i ugha gS\
channels with Myanmar. I: The UN Secretary-General is criticizing India’s initiatives
Hkkjr E;kaekj ds lkFk jktuf;d ek/;eksa ls jksfgaX;kvksa ds eqís dk such as ‘Save the Girl Child, Educate the Girl Child'.
lek/kku djsxkA la;qä jk"Vª egklfpo Hkkjr dh igy dh vkykspuk dj jgs gSa tSls
^^ckfydk cpkvks] ckfydk dks f'kf{kr djks**A

22
Best App for Govt. Jobs : Neonclasses (Download Now)

II: India’s initiatives such as ‘Save the Girl Child, Educate the  Concept -
Girl Child’ are the types of actions which are bringing
intricacies in achieving Sustainable Development Goals.
Hkkjr dh igy tSls ^^ckfydk cpkvks] ckfydk dks f'kf{kr djks** ,slh 1. Consider the given argument an decide which of the given
fØ;k,a gSa tks lrr fodkl y{;ksa dks çkIr djus esa isphnfx;ka yk jgh assumptions is (are) implicit.
gSaA fn, x, rdZ ij fopkj djsa vkSj r; djsa nh xbZ ekU;rkvksa esa ls
III: India's initiatives such as ‘Save the Girl Child, Educate the dkSulh fufgr gS¼gSa½A
Girl Child’ are symbolic of the types of actions. Argument: /rdZ%
Hkkjr dh igy tSls ^^ckfydk cpkvks] ckfydk dks f'kf{kr djks** dk;ksaZ All the faculty members are instructed to reach the
ds çdkjksa dk çrhd gSaA college by 9.00 A.M.
(a) Only I/dsoy I lHkh f’k{kdksa dks lqcg 9-00 cts rd dkWyst igqapus dk funs’Z k fn;k
(b) Only III/dsoy III tkrk gSA
(c) Only II and III/dsoy II vkSj III Assuptions:/ekU;rk,¡%
(d) Only I and II/dsoy I vkSj II 1. Some faculty members do not come on time.
(e) All of the above/mijksDr lHkh
dqN f’k{kd le; ij ugha vkrs gSaA
2. Faculty members will follow all the instructions in the
Sol. (d)
notice.
Only I and II. The UN Secretary-General is appreciating India's
initiatives stating they are symbolic of the types of actions
f'k{kd lwpuk&i= esa fn, lHkh funs’Z kksa dk ikyu djsaxsA
needed to achieve Sustainable Development Goals. So, I and II (a) Only assumption 2 implicit
are not-in-line with the statement. Option 4 is the answer. dsoy ekU;rk 2 fufgr gSA
Inferred from the statement (b) Only assumption 1 implicit
20. Following the Supreme Court verdict allowing entry of dsoy ekU;rk 1 fufgr gSA
women in Sabarimala Temple, the Kerala government will (c) Both 1 and 2 are implicit
reserve 25% seats in buses and make separate toilets for 1 vkSj 2 nksuksa fufgr gSaA
women pilgrims. The existing bathing Ghats for women (d) Neither 1 nor 2 is implicit
will be expanded to accommodate more pilgrims. 1 vkSj 2 nksuksa fufgr ugha gSaA
lcjhekyk eafnj esa efgykvksa ds ços'k dh vuqefr nsus okys lqçhe dksVZ Sol. (c)
ds QSlys ds ckn] dsjy ljdkj clksa esa 25% lhVsa vkjf{kr djsxh vkSj Obviously, some faculty members are not punctual that is why
efgyk rhFkZ;kf=;ksa ds fy, vyx 'kkSpky; cuk,xhA vf/kd rhFkZ;kf=;ksa such instructions have been provided. Faculty members listen
dks lek;ksftr djus ds fy, efgykvksa ds fy, ekStwnk Luku ?kkVksa dk and follow the instructions given to them. If they do not listen
foLrkj fd;k tk,xkA or follow them , it will have an impact on their professional
life.
What can be inferred from the above statement?
Hence, Both 1 and 2 are implicit.
mijksä dFku ls D;k vuqeku yxk;k tk ldrk gS\
2. Read the given question and decide which of the following
I: The Supreme Court verdict has banned the entry of
arguments is (are) strong.
women in Sabarimala Temple.
fuEufyf[kr iz’u i<+sa vkSj r; djsa fd fn, x, rdZ esa ls dkSu lk
lqçhe dksVZ ds QSlys us lcjhekyk eafnj esa efgykvksa ds ços'k ij
rdZ izHkko’kkyh gSA
çfrca/k yxk fn;k gSA
Question : Are the best things in life free?
II: Honoring the Supreme Court verdict, the Kerala
iz'u% D;k thou esa lcls vPNh phtsa eq¶r feyrh gSa\
government will reserve 25% seats in buses and make
Argument 1: Life gives us so many opportunities for free
separate toilets for women pilgrims.
and yet most of us waste it.
lqçhe dksVZ ds QSlys dk lEeku djrs gq,] dsjy ljdkj clksa esa 25%
rdZ1% thou gesa cgqr ls voljksa dks eq¶r esa nsrk gS vkSj fQj Hkh ge
lhVsa vkjf{kr djsxh vkSj efgyk rhFkZ;kf=;ksa ds fy, vyx 'kkSpky;
buesa ls vf/kdka’k dks O;FkZ esa tkus nsrs gSaA
cuk,xhA
Argument 2: Smile, friends, family, good talk, good sleep
III: The Kerala government will build new ghats to
and happy memories are all free.
accommodate more pilgrims.
rdZ 2 % eqLdku] nksLr] ifjokj] vPNh ckrs]a vPNh uhan vkSj vPNh ;knsa
dsjy ljdkj vf/kd rhFkZ;kf=;ksa dks lek;ksftr djus ds fy, u,
lHkh eq¶r feyrh gSaA
?kkVksa dk fuekZ.k djsxhA
(a) Only argument 2 is strong
(a) Only I/dsoy I (b) Only III/dsoy III
dsoy rdZ 2 izHkko’kkyh gSA
(c) Only II/dsoy II (d) All of the above/mijksDr lHkh
(b) Both arguments 1 and 2 are strong
(e) None of the above/mijksDr esa ls dksbZ ugha
1 vkSj 2 nksuksa izHkko’kkyh gSA
Sol. (c)
(c) Neither argument 1 nor 2 is strong
Only II is correct. Honoring the Supreme Court verdict, the
1 vkSj 2 nksuksa izHkko’kkyh ugha gSaA
Kerala government will reserve 25% seats in buses and make
(d) Only argument 1 is strong
separate toilets for women pilgrims. Further, the state
dsoy rdZ 1 izHkko’kkyh gSA
government will expand the existing ghats but not build new
Sol. (b)
ones. Hence, III is wrong. So, option 3 is correct.
The question is asking if the best things in life are free.
According to the first argument, life gives us many
4. Argument & Assumption opportunities for free, and yet most of us waste it.

23
Download Free PDFs & e-Books from Neon Classes App

Now, the question is that what are those opportunities? These Hence, only assumption II is implicit.
opportunities are explained in argument 2. That is smiles,
friends, family, good talk, good sleep, and happy memories. 5. Read the question and answer from the given information:
These are given by life and are free. Ikz’u dks i<sa+ vkSj nh xbZ tkudkjh ds vk/kkj ij mRrj nsAa
Therefore, both arguments 1 and 2 are strong. Should the govt stop giving permission to new industries.
3. Consider the Argument and decide which of the given D;k ljdkj dks u, m|ksxksa dks vuqefr nsuk can dj nsuk pkfg,A
assumptions is (are) implicit. Argument/ rdZ 1: Yes, new industries create
rdZ ij /;kuiwoZd fopkj djsa vkSj fu.kZ; djsa fd blesa uhps fn;k dkSu environmental pollution.
lk vuqeku fufgr gS@gSaA gk¡] u, m|ksx i;kZoj.k iznw"k.k iSnk djrs gSaA
Argument:/rdZ Argument/ rdZ 2: No, New industries improves
The present government had reduced the base price for employment and takes part in economic growth of
auctioning for the 2G spectrum. country.
orZeku ljdkj us 2G LisDVªe dh uhykeh ds fy, vk/kkj ewY; de ugha] u, m|ksx jkstxkj esa lq/kkj ykrs gSa vkSj ns’k ds vkfFkZd fodkl
dj fn;k FkkA esa lgk;d gksrs gSaA
Assumptions:/vuqeku% (a) only II is correct
1. Next time price will reduce further. dsoy 2 lgh gSA
vxyh ckj dher vkSj de gksxhA (b) Neither I nor II is correct
2. Last time the auction was not good. Uk rks 1 vkSj u gh 2 lgh gSA
fiNyh ckj uhykeh vPNh ugha FkhA (c) Either I or II is correct
(a) Neither 1 nor 2 is implicit ;k rks 1 ;k 2 lgh gSA
u rks 1 vkSj u gh 2 fufgr gSA (d) only I is correct
(b) Only assumption 2 is implicit dsoy 1 lgh gSA
dsoy vuqeku 2 fufgr gSA Sol. (a)
(C) Both 1 and 2 are implicit The argument that government should not permit new
1 vkSj 2 nksuksa vuqeku fufgr gS industries because they create pollution in the environment is
(d) Only assumption 1 is implicit not a strong argument because it is not necessary that all new
dsoy vuqeku 1 fufgr gSA industries create environmental pollution So argument I is not
Sol. (b) correct.
As the auction price has been reduced by the government so New industries will generate employement and will contribute
we can assume that this decision has been taken as last time to the economic growth of the country. So argument II is
the auction was not that good due to the high base price. But it correct.
can not be assumed that the base price will be reduced further 6. Read the question and answer from the given information:
in future because it depends on the consequences of the iz'u dks if<+, vkSj nh xbZ tkudkjh ds vk/kkj ij mRrj nhft,&
current auction. Pravalika is studying 10th class, is Pravalika elder than
Hence, only assumption II is implicit. pranitha who is her classmate?
4. “Gain your weight by 5 kgs in 3 sessions” – an izofydk nloha d{kk esa i<+rh gSA D;k izofydk viuh lgikBh izf.krk
advertisement. ls cM+h gS\
rhu l=ksa esa viuk otu 5 fdyksxzke c<k,a* & ,d foKkiu Argument 1: Yes, as she is her classmate Pranitha is
Assumption 1: “if you join in 6 sessions your weight will Younger to her.
be lost by 10 kgs. rdZ% gk¡] pwafd og mldh lgikBh gS] izf.krk mlls NksVh gSA
iwoZ/kkj.kk 1% ^;fn vki 6 l=ksa esa Hkkx ysrs gSa rks vkidk otu 10 Argument 2: No, as she is classmate pranitha is elder to
fdyksxzke rd de gks tk,xkA* her.
Assumption 2. “the company is dealing with weight gain rdZ% gk¡] pwafd og mldh lgikBh gS] izf.krk mlls cM+h gSA
programs”. (a) Neither I nor II follow
iwoZ/kkj.kk 2% ^daiuh otu c<+kus okys dk;ZØe pykrh gSA* u rks 1 vkSj u gh 2 vuqlj.k djrk gSA
(a) Neither I nor II is implicit. (b) Either I or II follow
Uk rks I vkSj u gh II varfuZfgr gS ;k rks 1 ;k 2 vuqlj.k djrk gSA
(b) II only is implicit (c) Only II follow
dsoy II varfuZfgr gSA dsoy 2 vuqlj.k djrk gSA
(c) I only is implicit (d) Only I follow
dsoy I varfuZfgr gSA dsoy 1 vuqlj.k djrk gSA
(d) Either I or II is implicit Sol. (a)
;k rks I ;k II varfuZfgr gSA The statement, Pravalika is studying is class 10th, describes
Sol. (b) only about the class in which Pravalika is studying and it is
The statement says that “Gain your weight by 5 kgs in 3 also mentioned that Pranitha is her classmate.
sessions” – an advertisement. That means the company is As both are fellow mates, it means that any one of them can be
dealing with weight gain programs that is why they have elder or younger than the other or they can be of same age.
published the advertisement. Studying only in same class has got nothing to deal with their
In the given advertisement, there is no information given age.
about a weight loss program, so assumption I is not implicit. Hence, Neither I nor II follow

24
Best App for Govt. Jobs : Neonclasses (Download Now)

7. Consider the argument and decide which of the given A top television company announced up to 50% discount
assumptions is / are implicit. on their new launch of LED TV.
rdZ ij /;kuiwoZd fopkj djsa vkSj mlds vk/kkj ij r; djsa fd uhps 'kh"kZ dh ,d Vsyhfotu daiuh us vius u, ysM (LED) Vhoh mRikn
fn, x, vuqekuksa esa ls dkSu lk@ls mlesa fufgr gS@gSaA ij 50 izfr’kr rd dh NwV nsus dk ,yku fd;k gSA
Argument:/ rdZ% Assumptions:/iwo/Z kkj.kk;s%a
A father advised his son that the shortest route between 1. The sales of the LED TV may increase.
Chennai and Mangalore is via Bengaluru. ysM (LED) Vhoh dh fcØh c<+ ldrh gSA
,d firk us vius csVs dks lykg nh dh psUubZ ls eaxykSj rd dk 2. The company will become the top seller of the LED TV.
lcls NksVk :V csx
a yq# ls gksdj gSA daiuh ysM (LED) Vhoh dh 'kh"kZ foØsrk cu tk,xkA
Assumptions:/vuqeku% (a) Both 1 and 2 are implicit.
1. Son wishes to go to Mangalore. nksuksa iwoZ/kkj.kk;sa vUrfuZfgr gSaA
csVk eaxykSj tkuk pkgrk gSA (b) Only assumption 1 is implicit.
2. Father gives advice to everyone. flQZ iwoZ/kkj.kk 1 vUrfuZfgr gSA
firk gj fdlh dks lykg nsrk gSA (c) Neither 1 nor 2 is implicit.
(a) Only assumption 2 is implicit Ukk rks iwoZ/kkj.kk 1 vkSj uk gh iwoZ/kkj.kk 2 vUrfuZfgr gSA
dsoy vuqeku 2 fufgr gSA (d) Only assumption 2 is implicit.
(b) Both 1 and 2 are implicit flQZ iwoZ/kkj.kk 2 vUrfuZfgr gSA
1 vkSj 2 nksuksa fufgr gSaA Sol. (b)
(c) Only assumption 1 is implicit According to the given angument, A top television company
dsoy vuqeku 1 fufgr gSA announced up to 50% discount on their new launch of LED TV.
(d) Neither 1 nor 2 implicit Now we can see that the discount offered might increase the
Uk rks 1 vkSj u gh 2 fufgr gSaA sales of the company. So , assumption I is implicit.
Sol. (c) And now it’s not necessary that the company will become the
Father is advising his son about the shortest route to top seller of the LED TV. So assumption II is not implicit.
manglore. It implies that the son is planning or wishes to go to 10. Consider the given argument and decide which of the
manglore. So, assumption 1 is implicit. given assumptions is (are) implicit.
And here, the father is giving advice to his son but nothing is nh xbZ ;qfDr ij fopkj djsa vkSj r; djsa fd nh gqbZ iwoZ/kkj.kkvksa esa ls
mationedabout wheather he gives advice top anyone else. So, dkSu lh varfuZfgr gSA
assumption 2 is not implicit. Argument:/;qfDr%
8. Consider the given argument and decide which of the The year 2020 is a leap year.
given assumptions is (are) implicit. Ok"kZ 2020 ,d vf/ko"kZ gSA
rdZ ij /;kuiwoZd fopkj djsa vkSj mlds vk/kkj ij r; djsa fd uhps Assumptions:/iwo/Z kkj.kkZ%
fn, x, vuqekuksa esa ls dkSu lk@ls mlesa fufgr gS@gSaA 1. February will have 29 days in 2020.
Argument:/rdZ% 2020 esa Qjojh esa 29 fnu gksx
a As
An army general said ‘Shoot Now’ to the soldier. 2. The year 2017 is not a leap year.
,d lsuk tujy us lSfud dks dgk ^vc xksyh pykvks*A Ok"kZ 2017 ,d vf/ko"kZ ugha gSA
Assumptions:/vuqeku% (a) Both 1 and 2 are implicit.
1. The soldier knows where and what to shoot. 1 vkSj 2 nksuksa iwoZ/kkj.kk,a varfuZfgr gSaA
lSfud dks irk gS fd dgka vkSj fdl ij xksyh pykuh gSA (b) Only assumption 2 is implicit.
2. The army general doesn’t know to shoot. dsoy iwoZ/kkj.kk 2 varfuZfgr gSA
lsuk tujy dks xksyh pykuk ugha vkrk gSA (c) Neither 1 nor 2 is implicit.
(a) Only assumption 2 is implicit. Uk rks 1 vkSj u gh 2 varfuZfgr gSA
dsoy vuqeku 2 fufgr gSA (d) Only assumption 1 is implicit.
(b) Both 1 and 2 are implicit. dsoy iwoZ/kkj.kk 1 varfuZfgr gSA
1 vkSj 2 nksuksa fufgr gSaA Sol. (a)
(c) Neither 1 nor 2 is implicit. According to the given argument , the year 2020 is a leap year
Uk rks 1] u gh 2 fufgr gSA which means the year containing 366 days with february 29 as
(d) Only assumption 1 is implicit. the extra day. So assmption I is implicit.
dsoy vuqeku 1 fufgr gSA The year 2017 is not a leap year, 2017 is not divisible by 4, so
Sol. (d) we can say that 2017 is not a leap year. So assumption II is also
In the army when a general says ‘shoot now’ soldier follows implicit.
the order and does the shooting. Soldiers already know how to 11. Consider the given argument and decide which of the
shoot and where to shoot because they are trained already given assumptions is (are) implicit.
about this. And that doesn’t means that general doesn’t know nh xbZ ;qfDr ij fopkj djsa vkSj r; djsa fd nh gqbZ iwoZ/kkj.kkvksa esa ls
how to shoot. So conclusion 1 is implicit. dkSulh varfuZfgr gSA
9. Consider the given argument and decide which of the Argument:/;qfDr%
given assumptions is (are) implicit. Government had put advertisements and issued
;gk¡ izLrqr rdZ@dFku ij fopkj djsa vkSj bl vk/kkj ij crk,a fd advisories to public not to eat food from the roadside, as
uhps nh xbZ iwoZ/kkj.kkvksa esa ls dkSu blesa vUrfuZfgr gS\ they are unhygienic.
Argument:/rdZ%

25
Download Free PDFs & e-Books from Neon Classes App

ljdkj us foKkiu fn, gSaS vkSj turk dks lykg nh gS fd os lM+dksa Due to potholes in the city a large number of people are
ij feyus okyk Hkkstu u djsa D;ksfa d og vLokLF;dj gksrk gSA unable to reach the airport on time.
Assumptions:/iwo/Z kkj.kk% 'kgj esa xM~<+ksa ds dkj.k cgqr ls yksx le; ij gokbZ vM~Ms rd ugha
1. Government wants to stop roadside vendors business. igq¡p ikrs gSaA
ljdkj lM+d fdukjs ds nqdkunkjksa dk O;kikj can djuk pkgrh gSA Assumptions:/vuqeku%
2. Government wants to increase the profit of hotel owners. 1. Reaching the airport on time is not necessary.
ljdkj gksVy ekfydksa ds ykHk dks c<+kuk pkgrh gSA le; ij gokbZ vM~Ms rd igqapuk vko’;d ugha gSA
(a) Neither 1 nor 2 is implicit. 2. There is no other route to the airport.
Uk rks 1 vkSj u gh 2 varfuZfgr gSA gokbZ vM~Ms ds fy, dksbZ vU; ekxZ ugha gSA
(b) Both 1 and 2 are implicit. (a) Neither 1 nor 2 is implicit
1 vkSj 2 nksuksa iwoZ/kkj.kk,a varfuZfgr gSA Uk rks vuqeku 1 u 2 gh fufgr gSA
(c) Only assumption 1 is implicit. (b) Only assumption 2 is implicit
dsoy iwoZ/kkj.kk 1 varfuZfgr gSA dsoy vuqeku 2 fufgr gSA
(d) Only assumption 2 is implicit. (c) Only assumption 1 is implicit
dsoy iwoZ/kkj.kk 2 varfuZfgr gSA dsoy vuqeku 1 fufgr gSA
Sol. (a) (d) Both 1 and 2 are implicit
The argument represents that the government is trying to stop vuqeku 1 o 2 nksuksa fufgr gSaA
people from eating from roadside stalls as the food is not Sol. (b)
hygienic. This is just because the government is thinking about Assumption 1 is not implicit because there is nothing
the public's health. They neither want to increase the profit of mentioned in the statement which we can relate to reaching
the hotel owners nor are they thinking of stopping the on time to the airort. Assumption 2 is implicit because there is
roadside vendor's business. Hence, we can conclude that no alternate route is available so people are taking the same
neither assumption 1 nor 2 is implicit. route and hence getting delayed.
12. Read the following question and decide which of the given 14. Consider the given argument and decide which of the
arguments is/are strong. given assumptions is (are) implicit.
fuEufyf[kr iz’u dks i<+sa vkSj fu.khZr djsa fd fuEufyf[kr esa ls dkSu nh xbZ ;qfDr ij fopkj djsa vkSj r; djsa fd nh gqbZ iwoZ/kkj.kkvksa esa ls
lk@ls rdZ l’kDr gS@gSaA dkSu lh varfuZfgr gSA
Question: Should CBSE introduce the retention system till Argument:/;qfDr%
Class 10? A government servant sitting ilde tells a visitor to come
iz'u% D;k lh]ch],l]bZ] dks d{kk 10 rd vuqRrh.kZ djus dh izfØ;k after 2 weeks to get this certificate attested, as he is very
’kq: djuh pkfg,\ busy and have a lot of work.
Argument 1: The foundations laid in lower classes are fuBYyk cSBk ,d ljdkjh deZpkjh ;g dgrs gq, fd og cgqr O;Lr gSa
important for higher education. vkSj cgqr lk dke iM+k gqvk gS] vkxarqd ds izek.k i=ksa dks izekf.kr
rdZ 1% mPp f’k{kk ds fy,] fuEure d{kkvksa esa LFkkfir fd, tkus okys djokus ds fy, 2 lIrkg ckn vkus ds fy, dgrk gSA
ewy fl}kar vR;f/kd egRoiw.kZ gksrs gSaA Assumptions:/iwo/Z kkj.kk%
Arguent 2: It is boring for a student to repeat the same 1. The government servant wants to help the visitor.
class again. Lkjdkjh deZpkjh vkxarqd dh enn djuk pkgrk gSA
rdZ 2% fon~;kFkhZ dks iqu% mlh d{kk dk v/;;u djuk v#fpdj 2. The government servant is expecting a favour from the
yxrk gSA visitor.
(a) Only argument 2 is strong. Lkjdkjh deZpkjh vkaxrqd ls dqN ikus dh vk’kk j[krk gSA
dsoy rdZ 2 gh l’kDr gSA (a) Only assumption 2 is implicit.
(b) Both arguments 1 and 2 are strong. dsoy iwoZ/kkj.kk 2 varfuZfgr gSA
rdZ 1 vkSj 2 nksuksa l’kDr gSaA (b) Only assumption 1 is implicit.
(c) Neither argument 1 nor 2 is strong. dsoy iwoZ/kkj.kk 1 varfuZfgr gSA
Uk rks rdZ 1 vkSj u gh 2 l’kDr gSA (c) Both 1 and 2 are implicit.
(d) Only argument 1 is strong. 1 vkSj 2 nksuksa iwoZ/kkj.kk,a varfuZfgr gSaA
dsoy rdZ 1 gh l’kDr gSA (d) Neither 1 nor 2 is implicit.
Sol. (d) Uk rks 1 vkSj u gh 2 varfuZfgr gSA
Argument 1 tells that the retention system should not be Sol. (a)
introduced till class 10 as it will weaken the foundation of The government servant makes up the excuse of being busy
learning in the students, which can cause problems in higher despite sitting idle. Therefore it is safe to assume that he is
education. expecting a favour from the visitor. This act of the government
Argument 2 tells that the retention system should be servant nowhere resembles his intention to help the visitor
introduced till class 10 but the reason given is not strong. So Hence only assumption 2 is implicit.
only argument 1 is strong. 15. Consider the argument and decide which of the given
13. Consider the argument and decide which of the given assumptions is/are implicit.
assumptions is /are implicit. r; djsa fd dFku esa nh xbZ tkudkjh ls izkIr dkSu&lk vuqeku
rdZ ij /;kuiwoZd fopkj djsa vkSj fu.kZ; djsa fd blesa uhps fn;k dkSu rdZlaxr :i ls vuqlj.k djrk gSA
lk vuqeku fufgr gS@gSaA Argument:/dFku%
Argument:/rdZ%

26
Best App for Govt. Jobs : Neonclasses (Download Now)

A mother is crying in the hospital as her only son died in a The Supreme court has decide that all rapists will be
road accident while driving his bike. hanged till death.
,d ek¡ vLirky esa jks jgh gS D;ksfa d mldk ,dek= csVs k rdZ%
lM+d&nq?kZVuk esa rc ej x;k tc og ckbd pyk jgk FkkA loksPZ p U;k;ky; us fu.kZ; fy;k gS fd lHkh cykRdkfj;ksa dks vkej.k
Assumptions:/vuqeku% dks Qkalh ij yVdk fn;k tk,A
1. The mother thinks it’s her son’s fault. Assumption:/ekU;rk %
og ek¡ lksprh fd ;g mlds csVs dh xyrh FkhA 1. Women will get protection.
2. The mother thinks that God is not with her. Ekfgykvksa dks lqj{kk feysxhA
ek¡ lksprh gS fd Hkxoku mlds lkFk ugha gSA 2. The number of rapes cases can be reduced.
(a) Only assumption 2 is implicit cykRdkj ds ekeyksa dks de fd;k tk ldrk gSA
dsoy vuqeku 2 vuqlj.k djrk gSA (a) Only assumption 2 is implicit.
(b) Both 1 an 2 are implicit dsoy ekU;rk 2 Li"V gSA
vuqeku 1 vkSj 2 nksuksa gh vuqlj.k djrs gSaA (b) Neither 1 nor 2 is implicit
(c) Only assumption 1 implicit. Uk rks 1 vkSj u gh 2 Li"V gSA
dsoy vuqeku 1 vuqlj.k djrk gSA (c) Both 1 and 2 are implicit
(d) Neither 1 nor 2 implicit. 1 vkSj 2 nksuksa Li"V gSA
u rks vuqeku 1 vuqlj.k djrk gS vkSj u gh 2 (d) Only assumption 1 is implicit.
Sol. (d) dsoy ekU;rk 1 Li"V gS
There is no information given about whose fault it is in the Sol. (a)
accident so, assumption 1 is not implicit. And also we can not The supreme court has decided that all rapists will be hanged
predict her intentions towards god. So, assumption 2 also not till death. So here it is not going to contribute in the protection
implicit. of women. So, Assumption 1 is not implicit.
Hence 1 nor 2 is implicit. But due to such harsh punishment rapists will be afraid of
16. Consider the argument and decide which of the given death and that will result in the less number of rape cases. So ,
assumptions is/are implicit. assumption 2 is implicit.
rdZ ij /;kuiwoZd fopkj djsa vkSj fu.kZ; djsa fd blesa uhps fn;k x;k 18. Argument:
dkSu lk vuqeku fufgr gSA During an election, the opposition party said, “look at the
Argument/rdZ% price rise in the last five years”.
A top university has reduced the syllabus of Mathematics ,d pquko ds nkSjku foi{kh ny us dgk] **fiNys ikap lky esa egaxkÃ
in class 12. dks nsf[k,-**
,d 'kh"kZ fo’ofon~;ky; us d{kk 12 ds xf.kr ds ikB~;Øe esa dVkSrh Assumptions:/ekU;rk,%¡&
dh gSA 1. This time, give a chance to our party for power.
Assumptions:/vuqeku% bl ckj] gekjs ny dks lRrk esa ekSdk nsAa
1. Students can score more marks than before in 2. Prices will still rise if you choose the same party to be in
mathematics. power
Nk= igys dh rqyuk esa xf.kr esa vf/kd vad yk ldrs gSaA ;fn vki lRrk ds fy, blh ny dks nqckjk pqusaxs rks dhersa c<+rh
2. This has been done to reduce the stress and burden of the jgsx
a hA
students in Mathematics. (a) Both 1 and 2 are implicit.
;g Nk=ks esa xf.kr ds ruko vkSj cks> dks de djus ds fy, fd;k 1 vkSj 2 nksuksa fufgr gSaA
x;k gSA (b) Only assumption 2 is implicit.
(a) Only assumption 2 is implicit dsoy vuqeku 2 fufgr gSA
dsoy vuqeku 2 fufgr gSA (c) Neither 1 nor is 2 implicit.
(b) Both 1 and 2 are implicit 1 vkSj 2 nksuksa fufgr ugha gSaA
1 vkSj 2 nksuksa fufgr gSaA (d) Only assumption 1 is implicit.
(c) Only assumption 1 is implicit dsoy vuqeku 1 fufgr gSA
dsoy vuqeku 2 fufgr gSA Sol. (a)
(d) Neither 1 and 2 is implicit The argument is about the opposition criticizing the
Uk rks 1 vkSj u gh 2 fufgr gSA government about increased prices. In assumption 1, pointing
Sol. (a) the failure of current government, opposition asking people
As per the argument , we can assume that this has been to for votes their party in upcoming elections and give their party
reduce the burden and stress of students . So assumption 2 is a chance to come into power. So, assumption 1 is implicit. In
implicit. assumption 2, the opposition party is triying to tell the
While assumption 1 is irrelevant to the argument and thus, disadvantage of choosing the party in the current government
assumption 1 does not implicit. over them. So assumption 2 is implicit, Hence, both assmptions
17. Consider the argument and decide Which of the given 1 and 2 are implicit.
assumptions is /are implicit. 19. Consider the argument and decide which of the given
fuEu rdZ ij fopkj djsa vkSj fu.kZ; ysa fd fn;k x;k dkSu&lk assumptions is/are implicit.
vuqeku Li"V gSA dFku ij /;kuiwoZd fopkj djsa vkSj fu.kZ; djsa fd blesa uhps fn;k
Argument: dkSu lk vuqeku fufgr gS@gSaA
Argument:/dFku%

27
Download Free PDFs & e-Books from Neon Classes App

Akbar said to his queen, “Birbal is the wisest man in my


kingdom.”
5. Assertion & Reason
vdcj us viuh jkuh ls dgk] ^chjcy esjs jkT; dk lcls cqf}eku
O;fDr gSA*  Concept -
Argument:/vuqeku%
1. A: We feel comfortable in hot and humid climate.
1. Akbar is not as wise as Birbal. A: ge xeZ vkSj vknzZ tyok;q esa lgt eglwl djrs gSA
vdcj chjcy ftruk cqf}eku ugha gSaA R: Sweat evaporates faster in humid climate.
2. Akbar wants Birbal to become the next king. R: vknZz tyok;q esa ilhuk rsth ls okf"ir gksrk gSA
vdcj chjcy dks vxyk jktk cukuk pkgrs gSaA (a) Both A and R are true and R is the correct explanation of A.
(a) Only assumption 2 is implicit. A vkSj R nksuksa lR; gSa vkSj R] A dh lgh O;k[;k gSA
dsoy vuqeku 2 fufgr gSA (b) Both A and R are true, but R is not the correct explanation of A.
(b) Neither 1 nor 2 is implicit A vkSj R nksuksa lgh gSa] ysfdu R] A dh lgh O;k[;k ugÈ gSA
u rks 1 vkSj u gh 2 fufgr gS
(c) A is true, but R is false.
(c) Both 1 and 2 are implicit A lR; gS] ijUrq R vlR; gSA
1 vkSj 2 nksuksa fufgr gSaA
(d) A is false, but R is true.
(d) Only assumption 1 is implicit
A >wBk gS] ysfdu R lp gSA
dsoy vuqeku 1 fufgr gSA
(e) Both A and R are false.
Sol. (d)
A vkSj R nksuksa >wBs gSaA
Birbal is the wisert man in the kingdom implies that no one,
Sol. (e)
not even Akbar is as wise as birbal. Hence assumption 1 is
Our body sweats more in hot and humid climate, and sweat
implicit.
does not evaporate quickly in hot and humid climate.
In the argument it is not indicated anywhere that Akbar wish
2. Assertion: Anil Kumble lost his place in the Indian ODI
to make Birbal the next king. Therefore, assumption 2 is not
squad sometime back
implicit.
nkok% vfuy dqacys us dqN le; igys Hkkjrh; ouMs Vhe esa viuh
20. Consider the argument and decide which of the given
txg [kks nh Fkh
assumptions is/are implicit.
Reason: In spite of being India`s best spinner he was
rdZ ij /;kuiwoZd fopkj djsa vkSj fu.kZ; djsa fd blesa uhps nh xbZ
considered to be too old for ODIs.
dkSu lk@ls vuqeku fufgr gS@gSaA
dkj.k% Hkkjr ds loZJs"B fLiuj gksus ds ckotwn mUgsa ,dfnolh; eSpksa
Argument:/rdZ%
ds fy, cgqr iqjkuk ekuk tkrk FkkA
The highway authority had announced that it will be
(a) Both A and R are true and R is the correct explanation of A.
undertaking a major road repair work for three days
A vkSj R nksuksa lR; gSa vkSj R] A dh lgh O;k[;k gSA
beginning Friday between Bangalore and Tumkur.
(b) Both A and R are true, but R is not the correct explanation of A.
jktekxZ izkf/kdj.k us ?kks"k.kk dh Fkh fd og 'kqØokj ls rhu fnu ds
A vkSj R nksuksa lgh gSa] ysfdu R] A dh lgh O;k[;k ugÈ gSA
fy, caxykSj vkSj rqedqj ds chp lM+d ejEer dk cM+k dke 'kq#
(c) A is true, but R is false.
djsxhA
A lR; gS] ijUrq R vlR; gSA
Assumption:/vuqeku%
(d) A is false, but R is true.
1. People need to plan accordingly for their trip between
A >wBk gS] ysfdu R lp gSA
Bangalore and Tumkur.
(e) Both A and R are false.
Ykksxksa dks caxykSj vkSj rqedqj ds chp viuh ;k=k dh ;kstuk blds
A vkSj R nksuksa >wBs gSaA
vuq:i cukus dh t:jr gSA
Sol. (a)
2. The authority is troubling citizens with unnecessary
Both Assertion and Reason are true and Reason is the correct
repair work.
explanation of Assertion.
izkf/kdj.k vuko’;d ejEer dk;Z ls ukxfjdksa dks ijs’kku dj jgh gSA
3. A: Shimla is colder than Delhi.
(a) Neither 1 nor 2 is implicit
A: f’keyk fnYyh dh rqyuk esa BaMk gSA
u rks 1 vkSj u gh 2 fufgr gS
R: Shimla is at a higher altitude as compated to Delhi.
(b) Both 1 and 2 are implicit
R: fnYyh dh rqyuk esa f’keyk vf/kd ÅpkabZ ij gSA
1 vkSj 2 nksuksa fufgr gSa
(a) Both A and R are true and R is the correct explanation of A.
(c) Only assumption 1 is implicit
A vkSj R nksuksa lR; gSa vkSj R] A dh lgh O;k[;k gSA
dsoy vuqeku 1 fufgr gS
(b) Both A and R are true, but R is not the correct explanation of A.
(d) Only assumption 2 is implicit
A vkSj R nksuksa lgh gSa] ysfdu R] A dh lgh O;k[;k ugÈ gSA
dsoy vuqeku 2 fufgr gS
(c) A is true, but R is false.
Sol. (c)
A lR; gS] ijUrq R vlR; gSA
Considering statement, if the authority is planning for a three
(d) A is false, but R is true.
days road repair work between Banglore and Tumkur and if
A >wBk gS] ysfdu R lp gSA
they are informing citizens early beacase they want their
(e) Both A and R are false.
citizens to plan their trip accordingly, they do not want them
A vkSj R nksuksa >wBs gSaA
to face any trouble, The repair is done to assist people in
transportation not to trouble them. Hence assumptions 1 is Sol. (a)
implicit while 2 is not.

28
Best App for Govt. Jobs : Neonclasses (Download Now)

Simla is colder than Delhi because it is situated at a higher The Assertion is false because Kerala is not the largest state in
altitude and temperature decreases by 10C for every 165 India. R is true but it is not related to Assertion. Hence, option
metres of ascent. (d) is correct.
Hence, option A is correct 7. A: India is facing the problem of inflation.
4. Assertion: Over the past few decades, there has been A: Hkkjr egaxkbZ dh leL;k ls tw> jgk gSaA
drastic global warming. R: We have failed to check the growth of black money.
vfHkdFku% fiNys dqN n'kdksa es]a vR;fèkd Xykscy okÉex gqà gSA R: ge dkys /ku dh o`f} dh tkap djus esa foQy jgs gSaA
Reason: The push for economic development has (a) Both A and R are true and R is the correct explanation of A.
increased to dangerous proportions the emission of A vkSj R nksuksa lR; gSa vkSj R] A dh lgh O;k[;k gSA
greenhouse gases into the atmosphere. (b) Both A and R are true, but R is not the correct explanation of A.
dkj.k% vkÆFkd fodkl ds fy, èkôk [krjukd vuqikr esa c<+ x;k gS A vkSj R nksuksa lgh gSa] ysfdu R] A dh lgh O;k[;k ugÈ gSA
ftlls okrkoj.k esa xzhugkml xSlksa dk mRltZu gksrk gSA (c) A is true, but R is false.
(a) Both A and R are true and R is the correct explanation of A. A lR; gS] ijUrq R vlR; gSA
A vkSj R nksuksa lR; gSa vkSj R] A dh lgh O;k[;k gSA (d) A is false, but R is true.
(b) Both A and R are true, but R is not the correct explanation of A. A >wBk gS] ysfdu R lp gSA
A vkSj R nksuksa lgh gSa] ysfdu R] A dh lgh O;k[;k ugÈ gSA (e) Both A and R are false.
(c) A is true, but R is false. A vkSj R nksuksa >wBs gSaA
A lR; gS] ijUrq R vlR; gSA Sol. (a)
(d) A is false, but R is true. Inflation in India is caused by unrestricted growth of black
A >wBk gS] ysfdu R lp gSA money.
(e) Both A and R are false. Hence, Both Assertion and reason are true & Reason is the
A vkSj R nksuksa >wBs gSaA correct explanation of Assertion.
Sol. (a) 8. Assertion: The filament inside a bulb is made of copper.
Green house gases are the root cause of global warming. Hence vfHkdFku% ,d cYc ds vanj dk fQykesVa rkacs dk cuk gksrk gSA
both Assertion and Reason are true and Reason explains Reason: Usually there is vacuum in the bulb
Assertion. Hence, option (a) is correct. dkj.k% çk;% cYc esa fuokZr gksrk gS
5. A: Leakages in household gas cylinders can be detected. (a) Both A and R are true and R is the correct explanation of A.
A: ?kjsyw xSl flysaMj esa fjlko dk irk] yxk;k tk ldrk gSA A vkSj R nksuksa lR; gSa vkSj R] A dh lgh O;k[;k gSA
R: LPG has a strong smell. (b) Both A and R are true, but R is not the correct explanation of A.
R: ,yihth esa rst xa/k gksrh gSA A vkSj R nksuksa lgh gSa] ysfdu R] A dh lgh O;k[;k ugÈ gSA
(a) Both A and R are true and R is the correct explanation of A. (c) A is true, but R is false.
A vkSj R nksuksa lR; gSa vkSj R] A dh lgh O;k[;k gSA A lR; gS] ijUrq R vlR; gSA
(b) Both A and R are true, but R is not the correct explanation of A. (d) A is false, but R is true.
A vkSj R nksuksa lgh gSa] ysfdu R] A dh lgh O;k[;k ugÈ gSA A >wBk gS] ysfdu R lp gSA
(c) A is true, but R is false. (e) Both A and R are false.
A lR; gS] ijUrq R vlR; gSA A vkSj R nksuksa >wBs gSaA
(d) A is false, but R is true. Sol. (e)
A >wBk gS] ysfdu R lp gSA A is false because a bulb filament is made of Tungsten and not
(e) Both A and R are false. copper. R is also false because inert gases or nitrogen are filled
A vkSj R nksuksa >wBs gSaA inside the bulb and there is no vacuum in the bulb. Hence,
Sol. (c) option (e) is correct.
Leakages in household gas cylinders can be detected because 9. A: Plaster of Paris is used by doctors for setting fractured
of the strong smell of ethyl mercaptan mixed with LPG. bones.
Hence, A is true but R is false. MkWDVjksa }kjk VwVh gqbZ gfMM~;ksa dks lgh djus ds fy, IykLVj vkWQ
6. Assertion: Kerala is the largest state in India isfjl dk mi;ksx fd;k tkrk gSaA
dFku% dsjy Hkkjr dk lcls cM+k jkT; gS R: When Plaster of Paris is mixed with water & applied
Reason: Some of the leading car manufacturing companies around the fractured limbs, it sets into a hard mass.
of the world are Japanese tc IykLVj vkWQ isfjl dks ikuh dslkFk feyk;k tkrk gS vkSj [kafMr
dkj.k% nqfu;k dh dqN çeq[k dkj fuekZrk daifu;ka tkikuh gS vaxksa ds pkjksa vksj yxk;k tkrk gS] rks ;g ,d dBksj nzO;eku esa cny
(a) Both A and R are true and R is the correct explanation of A. tkrk gSaA
A vkSj R nksuksa lR; gSa vkSj R] A dh lgh O;k[;k gSA (a) Both A and R are true and R is the correct explanation of A.
(b) Both A and R are true, but R is not the correct explanation of A. A vkSj R nksuksa lR; gSa vkSj R] A dh lgh O;k[;k gSA
A vkSj R nksuksa lgh gSa] ysfdu R] A dh lgh O;k[;k ugÈ gSA (b) Both A and R are true, but R is not the correct explanation of A.
(c) A is true, but R is false. A vkSj R nksuksa lgh gSa] ysfdu R] A dh lgh O;k[;k ugÈ gSA
A lR; gS] ijUrq R vlR; gSA (c) A is true, but R is false.
(d) A is false, but R is true. A lR; gS] ijUrq R vlR; gSA
A >wBk gS] ysfdu R lp gSA (d) A is false, but R is true.
(e) Both A and R are false. A >wBk gS] ysfdu R lp gSA
A vkSj R nksuksa >wBs gSaA (e) Both A and R are false.
Sol. (d) A vkSj R nksuksa >wBs gSaA

29
Download Free PDFs & e-Books from Neon Classes App

Sol. (a) A lR; gS] ijUrq R vlR; gSA


Plaster of Paris when mixed with water applied around the (d) A is false, but R is true.
fractured limbs, it sets into a hard mass and keeps the bone A >wBk gS] ysfdu R lp gSA
joints in a fixed position. (e) Both A and R are false.
So, it can be used for setting fractured bones. Hence, Both A A vkSj R nksuksa >wBs gSaA
and R are true and R is the correct explanation of A. Sol. (d)
10. A: We prefer to wear white clothes in winter. Banking soda, being alkaline, neutralises the acidity in the
ge lfnZ;ksa esa lQsn diMs+ iguuk ilan djrs gSA stomach and removes it.
R: White clothes are good reflectors of hear. Hence, A is false, but R is true.
lQsn diM+s xehZ ds vPNs ijkorZd gksrs gSA 13. Assertion: Mahatma Gandhi is considered to be the father
(a) Both A and R are true and R is the correct explanation of A. of the Indian Nation.
A vkSj R nksuksa lR; gSa vkSj R] A dh lgh O;k[;k gSA dFku% egkRek xkaèkh dks Hkkjrh; jk"Vªfirk ekuk tkrk gSA
(b) Both A and R are true, but R is not the correct explanation of A. Reason: The name India was coined by Mahatma Gandhi.
A vkSj R nksuksa lgh gSa] ysfdu R] A dh lgh O;k[;k ugÈ gSA dkj.k% Hkkjr uke egkRek xkaèkh }kjk x<+k x;k FkkA
(c) A is true, but R is false. (a) Both A and R are true and R is the correct explanation of A.
A lR; gS] ijUrq R vlR; gSA A vkSj R nksuksa lR; gSa vkSj R] A dh lgh O;k[;k gSA
(d) A is false, but R is true. (b) Both A and R are true, but R is not the correct explanation of A.
A >wBk gS] ysfdu R lp gSA A vkSj R nksuksa lgh gSa] ysfdu R] A dh lgh O;k[;k ugÈ gSA
(e) Both A and R are false. (c) A is true, but R is false.
A vkSj R nksuksa >wBs gSaA A lR; gS] ijUrq R vlR; gSA
Sol. (d) (d) A is false, but R is true.
We prefer to wear dark clothes in winter because they absorb A >wBk gS] ysfdu R lp gSA
the heat and keep the body warm. However, white clothes are (e) Both A and R are false.
good reflectors of heat and are worn in summer. Hence, option A vkSj R nksuksa >wBs gSaA
D is correct. Sol. (c)
11. Assertion The Ozone layer present in the upper Assertion: Mahatma Gandhi is considered to be the father of
atmosphere of earth is beneficial for sustaining life on the Indian Nation.
earth Reason: The name 'India’ was coined by Mahatma Gandhi
vfHkdFku i`Foh ds Åijh ok;qeaMy esa ekStwn vkstksu ijr i`Foh ij So, A is true but R is false.
thou dks cuk, j[kus ds fy, Qk;nsean gS 14. A: When common salt is kept open, it absorbs moisture
Reason The Ozone layer blocks the harmful ultraviolet from the air.
rays from the Sun and thus protects life on earth tc vke ued dks [kqyk j[kk tkrk gS] rks ;g gok ls ueh dks
dkj.k vkstksu ijr lw;Z ls vkus okyh gkfudkjd ijkcSaxuh fdj.kksa dks vo’kksf"kr djrk gSA
jksdrh gS vkSj bl çdkj i`Foh ij thou dh j{kk djrh gS R: Common salt contains magnesium chloride.
(a) Both A and R are true and R is the correct explanation of A. vke ued esa eSXuhf’k;e DyksjkbM gksrk gSA
A vkSj R nksuksa lR; gSa vkSj R] A dh lgh O;k[;k gSA (a) Both A and R are true and R is the correct explanation of A.
(b) Both A and R are true, but R is not the correct explanation of A. A vkSj R nksuksa lR; gSa vkSj R] A dh lgh O;k[;k gSA
A vkSj R nksuksa lgh gSa] ysfdu R] A dh lgh O;k[;k ugÈ gSA (b) Both A and R are true, but R is not the correct explanation of A.
(c) A is true, but R is false. A vkSj R nksuksa lgh gSa] ysfdu R] A dh lgh O;k[;k ugÈ gSA
A lR; gS] ijUrq R vlR; gSA (c) A is true, but R is false.
(d) A is false, but R is true. A lR; gS] ijUrq R vlR; gSA
A >wBk gS] ysfdu R lp gSA (d) A is false, but R is true.
(e) Both A and R are false. A >wBk gS] ysfdu R lp gSA
A vkSj R nksuksa >wBs gSaA (e) Both A and R are false.
Sol. (b) A vkSj R nksuksa >wBs gSaA
Ozone is a powerful oxidizing agent compared to oxygen as it Sol. (a)
is unstable. Magnesium chloride present in common salt is a deliquescent
Ozone layer prevent UV radiations from reaching the earth, substance i.e. it absorbs moisture from the air when kept in
hence ozone layer acts as a reactive blanket to earth. open.
Hence, Both A and R are true, but R is not the correct Hence, option A is correct.
explanation of A.
12. A: Baking soda creates acidity in the stomach. 15. A: Increase in carbon dioxide would melt polar ice.
csfdax lksM+k isV esa ,flfMVh iSnk djrk gSA dkcZu MkbvkWDlkbM esa o`f} ls /kqzoh; cQZ fi?ky tk,xhA
R: Baking soda is alkaline. R: Global temperature would rise.
csfdax lksMk {kkjh; gSA oS’fod rkieku esa o`f} gksxhA
(a) Both A and R are true and R is the correct explanation of A. (a) Both A and R are true and R is the correct explanation of A.
A vkSj R nksuksa lR; gSa vkSj R] A dh lgh O;k[;k gSA A vkSj R nksuksa lR; gSa vkSj R] A dh lgh O;k[;k gSA
(b) Both A and R are true, but R is not the correct explanation of A. (b) Both A and R are true, but R is not the correct explanation of A.
A vkSj R nksuksa lgh gSa] ysfdu R] A dh lgh O;k[;k ugÈ gSA A vkSj R nksuksa lgh gSa] ysfdu R] A dh lgh O;k[;k ugÈ gSA
(c) A is true, but R is false. (c) A is true, but R is false.

30
Best App for Govt. Jobs : Neonclasses (Download Now)

A lR; gS] ijUrq R vlR; gSA A vkSj R nksuksa lR; gSa vkSj R] A dh lgh O;k[;k gSA
(d) A is false, but R is true. (b) Both A and R are true, but R is not the correct explanation of A.
A >wBk gS] ysfdu R lp gSA A vkSj R nksuksa lgh gSa] ysfdu R] A dh lgh O;k[;k ugÈ gSA
(e) Both A and R are false. (c) A is true, but R is false.
A vkSj R nksuksa >wBs gSaA A lR; gS] ijUrq R vlR; gSA
Sol. (a) (d) A is false, but R is true.
The carbon dioxide envelope in earth's atmosphere traps the A >wBk gS] ysfdu R lp gSA
heat. (e) Both A and R are false.
With increase in the proportion of carbon dioxide, therefore, A vkSj R nksuksa >wBs gSaA
the global temperature would rise, thus causing the polar ice Sol. (c)
to melt. The use of chlorofluoro carbons is banned nowadays because
Hence, Both A and R are true and R is the correct explanation these cause holes in the ozone layer through which ultraviolet
of A. rays penetrate and may cause skin cancer.
16. A: When a body is dipped in liquid fully or partially, there Hence, A is true, but R is false.
is a decrease in its weight. 19. Assertion/ nkok (a):
tc dksbZ 'kjhj iwjh rjg ;k vkaf’kd :i ls rjy esa Mwck gksrk gS] rks IIT students are very industrious.
mlds otuesa deh gksrh gSA IIT ds Nk= cgqr esgurh gksrs gSaA
R: The decrease in weight is due to the higher density of Reason/ dkj.k (R):
the displaced liquid. IIT students got high package salaries.
otu esa deh foLFkkfir rjy ds mPp ?kuRo ds dkj.k gSA IIT ds Nk=ksa dks mPp iSdst osru feykA
(a) Both A and R are true and R is the correct explanation of A. (a) Both A and R are true and R is the correct explanation of A.
A vkSj R nksuksa lR; gSa vkSj R] A dh lgh O;k[;k gSA A vkSj R nksuksa lR; gSa vkSj R] A dh lgh O;k[;k gSA
(b) Both A and R are true, but R is not the correct explanation of A. (b) Both A and R are true, but R is not the correct explanation of A.
A vkSj R nksuksa lgh gSa] ysfdu R] A dh lgh O;k[;k ugÈ gSA A vkSj R nksuksa lgh gSa] ysfdu R] A dh lgh O;k[;k ugÈ gSA
(c) A is true, but R is false. (c) A is true, but R is false.
A lR; gS] ijUrq R vlR; gSA A lR; gS] ijUrq R vlR; gSA
(d) A is false, but R is true. (d) A is false, but R is true.
A >wBk gS] ysfdu R lp gSA A >wBk gS] ysfdu R lp gSA
(e) Both A and R are false. (e) Both A and R are false.
A vkSj R nksuksa >wBs gSaA A vkSj R nksuksa >wBs gSaA
Sol. (c) Sol. (b)
When a body is dipped in a liquid, there is a decrease in weight Both A and R are true, but R is not the correct explanation of A.
due to the upward thrust exerted on it by the water. 20. Assertion/ nkok (a):
Hence, A is true, but R is false. TATA steel has opened another industry in Odisha.
17. Assertion: Crude oil is abundantly found in nature. VkVk LVhy us vksfM'kk esa ,d vkSj m|ksx [kksyk gSA
dFku dPpk rsy ç—fr esa çpqj ek=k esa ik;k tkrk gSA Reason/ dkj.k (R):
Reason: It is the main raw material for all automobiles. Odisha is rich in mine.
dkj.k ;g lHkh v‚Vkseksckby ds fy, eq[; dPpk eky gSA vksfM'kk [knku ls le`) gSA
(a) Both A and R are true and R is the correct explanation of A. (a) Both A and R are true and R is the correct explanation of A.
A vkSj R nksuksa lR; gSa vkSj R] A dh lgh O;k[;k gSA A vkSj R nksuksa lR; gSa vkSj R] A dh lgh O;k[;k gSA
(b) Both A and R are true, but R is not the correct explanation of A. (b) Both A and R are true, but R is not the correct explanation of A.
A vkSj R nksuksa lgh gSa] ysfdu R] A dh lgh O;k[;k ugÈ gSA A vkSj R nksuksa lgh gSa] ysfdu R] A dh lgh O;k[;k ugÈ gSA
(c) A is true, but R is false. (c) A is true, but R is false.
A lR; gS] ijUrq R vlR; gSA A lR; gS] ijUrq R vlR; gSA
(d) A is false, but R is true. (d) A is false, but R is true.
A >wBk gS] ysfdu R lp gSA A >wBk gS] ysfdu R lp gSA
(e) Both A and R are false. (e) Both A and R are false.
A vkSj R nksuksa >wBs gSaA A vkSj R nksuksa >wBs gSaA
Sol. (b) Sol. (a)
Both Assertion and Reason are true but the Reason does not Both A and R are true and R is the correct explanation of A.
explain the Assertion. Hence, (b) is the answer
18. Assertion/ nkok (a): 6. Blood Relation
The use of chlorofluoro carbons is banned throughout the
world nowdays.  Concept -
Dyksjks¶yksjks dkcZu ds mi;ksx ij vktdy iwjh nqfu;k esa çfrca/k yxk
gqvk gSA Questions on Blood Relationship are related to our day to day
Reason/dkj.k (R): life. We are bound by our kith and kin through a chain of
These chemicals cause skin cancer. relationships. The examiner defines the simple relationship by
;s jlk;u Ropk ds dSalj dk dkj.k curs gSaA using rather complicated set of definitions and expects from us
to comprehend these definitions rather quickly. In order to
(a) Both A and R are true and R is the correct explanation of A.

31
Download Free PDFs & e-Books from Neon Classes App

solve these problems, analyses the given statements carefully bl izdkj ge ns[krs gS fd fdlh O;fDr ds fyax dh tkudkjh cgqr
and systematically. For examples: egRoiw.kZ gS nks O;fDr;ksa ds chp esa laca/k fudkyus ds fy,A
jDr laca/k ls tqMs+ iz’u gekjh nSfud fnup;kZ ls tqMs+ gq, gSA ge Some Important Tips:
fj’rs dh ,d J`[a kyk ds ek/;e ls vius ifjtuksa ls ca/ks gSAa ijh{kd (i) First of all choose the two persons, between whom
ifjHkk"kkvksa ds tfVy lsV dk mi;ksx djds ljy laca/kksa dks ifjHkkf"kr relationship is to be established.
djrk gS vkSj gels vis{kk djrk gS fd ge bu ifjHkk"kkkvksa dks 'kh?kzrk (ii) Next, pin-point the intermediate relationship i.e., such
ls le>sAa bu leL;kvksa dks gy djus ds fy, O;ofLFkr #i ls fn, relationship through which long drawn relationship can be
x, dFku dk fo’ys"k.k djsAa mnkgj.k ds fy,& established between the required persons.
(i) My father’s only child means I (Myself) (iii) Finally, conclude the relationship directly between the two
esjs firk dk bdykSrk cPpk eryc ^^eSa** [kqnA persons as per the requirement of the question.
(ii) Ritu’s husband’s father-in-law’s only daughter means Ritu (iv) From a particular name we cannot ascertain the sex (gender)
(herself) of that person. The name does not always show the gender of
fjrq ds ifr ds llqj ds vdsyh csVh eryc fjrq [kqn gh gSA that person. The name does not always show the gender
Type of questions:- beyond reasonable doubt.
(1) Introduction type ¼ifjp; djokuk½ For example, we often hear the same name for male and
(2) Pointing ¼b'kkjk djuk½ female in the Punjabi community.
The names, Harvinder, Sukhwinder, Gurinder etc. are used for
(3) Photograph ¼rLohj lEcf/kr½
both the sexes in the Punjabi community.
(4) Puzzle form/grouping ¼igsyh :i esa@lewg esa½
There are certain other names which are used for both the
(5) coded blood relation ¼dksMhÑr jDRk lEc/k½
sexes all over the country. For example, Suman, Kamal etc.
Pay particular attention to the information given in the dqN eq[; ckrs&a
question itself without your personal biases and preconceived
(i) igyk mu nks O;fDr;ksa dh igpku djsa ftuds chp laca/k fudkyuk
notions and assumptions coming to the fore questions on
gSA
Blood relationship can be solved by any of the following
(ii) fQj lcls utnhdh laca/k dks bafxr djsa ftldh enn ls ge ml yacs
methods:
iz’u esa nh x;h tkudkjh dks cMs+ /;ku ls ns[ksa vkSj fcuk fdlh rjQ fj’rs ds chp lac/a k LFkkfir dj ldsaA
(iii) vkf[kj esa] iz’u dh vko’;drk vuqlkj mu nks O;fDr;ksa ds
>qdko ds ,oa iwoZ /kkj.kk ds fdlh fu"d"kZ ij igq¡psA jDr laca/k ds
iz’u ge fuEu nks izdkj ls gy dj ldrs gSA chp dk fu"d"kZ fudkysAa
(iv) fdlh fo’ks"k uke ls ge mldk fyax ugah crk ldrs gSA
(i) Deduction Method
(ii) Pictorial Method fdlh fo’ks"k uke ls ge mlds fyax ds ckjs esa ugha crk ldrs gSA
(i) dVkSrh fof/k mnkgj.k ds rkSj] iatkch leqnk; esa iq#"k vkSj L=h ds uke leku gksrs
(ii) fp=kRed fof/k gS] tSls fd gjfoanj] lq[kfoanj] xqfjanj vkfnA
While attempting questions on Blood Relationship, first read dqN uke gj txg nksuksa fyax ds lkFk iz;ksx fd, tkrs gSa tSls fd
all the pieces of information as quickly as possible relationship lqeu] dey bR;kfnA
is to be established. Finally, try to co-relate the given How to make family tree ¼ifjokj o`{k dSls cuk;s½%&
relationship. While concluding relationship between two (1) male/female ¼iq:"k@L=h½
person be careful about the sexes of the persons involved. male (+), Female (-)
Majority of the students tend to define or derive relationship (2) A, B dk HkkbZ gSA A is brother of B.
without caring for sex of the persons. Is it possible to define
relationship between two persons without knowing their sex ? (3) A, B dh cgu gSA A is sister of B.
(Consider the following illustration:
iz'u dks gy djrs le; lcls igys tkudkjh ds lHkh fgLlksa dks <ax (4) A, B dk ifr gSA A is husband of B.
ls i<sa vkSj fQj mu nks O;fDr;ksa dks vafdr djsa ftuds chp esa laca/k
crkuk gSA vkf[kj esa fj’rs dks tksM+us dk iz;kl djsAa O;fDr;ksa ds
(5) A, B dh iRuh gSA A is wife of B.
laca/k dks crkrs le; muds fyax dk vo’; /;ku j[ksAa T;knkrj
fo|kFkhZ fcuk fyax dk /;ku fn, gh fj’rksa dk lqy>kus esa yx tkrs
gSA fuEufyf[kr mnkgj.k ij /;ku nsa& (6) A, B dk firk gSA A is father of B.
A is the child of P and Q.
‘A’ P vkSj Q dk cPpk gSA
From this statement can we conclude that P is the father of A.
No, it is not possible. Without knowing the sex of either P or Q,
it is not possible to conclude that P is the father of A. What we
(7) A, B dh ekrk gSA A is mother of B.
can conclude from the above statement is that ‘P and Q are
parents of A.
Thus, we see that the knowledge about the sex of persons is
necessary to conclude relationship between the two persons.
bl dFku ls ge fu"d"kZ fudkyrs gS fd P, A dk firk gS rks ;g Hkh
(8) A, B dk iq= gSA A is son of B.
xyr gS ;k laHko ugha gSA fcuk P ;k Q ds fyax dh tkudkjh ds ge
fu"d"kZ ugha fudky ldrs fd P, ‘A’ dk firk gS ge flQZ bruk
fu"d"kZ fudky ldrs gS fd ‘P’ vkSj ‘Q’, ‘A’ ds ekrk&firk gSA

32
Best App for Govt. Jobs : Neonclasses (Download Now)

HkkbZ ;k cgu dk csVk Hkrhtk@Hkkutk


HkkbZ dh csVh Hkrhth
cqvk ;k pkpk dk csVk ;k dftu
csVh
(9) A, B dh iq=h gSA A is daughter of B. cgu dk ifr thtk
HkkbZ dh iRuh HkkHkh
nknk ;k nknh dk csVk firk ;k pkpk
nknk ;k nknh dh bdykSrh ek¡
(10) A, B dk nkekn gSA A is son-in-law of B. cgq
nknk ;k nknh dh cgq ek¡ ;k pkph
nknk dk bdykSRkk iq= firk
ukuk dh bdykSrh iq=h ek¡
(11) A, B dh lkl gSA A is mother-in-law of B.
pkpk@ekek dh iRuh pkph@ekeh
ekrk ;k firk ds firk nknk@ukuk
firk ;k ekrk dh ek¡ nknh@ukuh
Father’s mother’s son Brother nknk ;k nknh ds firk iM+nknk
Father’s mother’s daughter Sister nknk ;k nknh dh ek¡ iM+nknh
Father’s or mother’s brother Uncle firk dh iRuh ek¡
Father’s or mother’s sister Aunt * ek¡ ds ifr firk
Father’s or mother’s mother Grandmother The questions on Blood relationship are asked in various
Father’s or mother’s father Grandfather formats but substantially there is no difference between
Son’s wife Daughter-in-law them.
Husband’s or wife’s sister Sister-in-law jDr lac/a k ds iz’u vyx&vyx QksjesV esa iwNs tkrs gSa ijarq muesa
Brother’s or Sister’s son Nephew eq[;r% dksbZ varj ugha gksrkA
Brothers daughter Niece Concept- Puzzle based question
Aunt’s or Uncle’s son or daughter Cousin 1. B's mother R has only one daughter. A is the only son of R.
Sister’s husband Brother-in-law How is B related to A?
Brother’s wife Sister-in-law B dh ekW R dh dsoy ,d iq=h gSA A, R dk bdykSrk iq= gSaA B dk A
Grandfather’s or Grandmother’s son Father or Uncle ls D;k lac/a k gS\
Grandfather’s or Grandmother’s only (a) sister/cgu
Mother (b) Father/ firk
daughter-in-law
Grandfather’s or Grandmother’s (c) Brother/ HkkbZ
Mother or Aunt (d) Aunt/Mother's Sister/ekSlh@ekW dh cgu
daughter-in-law
Only son of Grandfather (Paternal) Father Sol. (a)
Only daughter of Grandfather R
Mother
(Maternal)
B– A+
Uncle’s wife Aunt
Father of father or mother Grandfather (B and A is in same generation and gender of B is female so
Mother of father or mother Grandmother option (B), (C), (D) wrong so B is the sister of A.
Father of Grandfather or Grandmother GreatGrandfather 2. Kapil is the nephew of Rajat. Nisha is Rajat's mother. Ramesh
Great has only two children Rajat and Nilesh. How is Nilesh related
Mother of Grandfather or Grandmother to Kapil?
Grandmother
Only Daughter-in-law of Grandfather dfiy] jtr dk Hkrhtk gSA fu'kk] jtr dh eka gSA jes'k ds dsoy nks
Mother cPps jtr vkSj uhys'k gSaA uhys'k dk dfiy ls D;k lacaèk gS\
(Paternal)
Wife of Father Mother (a) Father (b) Brother
Husband of Mother Father (c) Nephew (d) Uncle
fuEufyf[kr fyLV ls vizR;{k laca/k dh igpku dh tkudkjh fey Sol. (a)
ldrh gSA Nisha – Ramesh+

Rajat+ Nilesh+
firk ;k ek¡ dk csVk HkkbZ
firk ;k ek¡ dh csVh cgu Kapil
firk ;k ek¡ dk HkkbZ pkpk@ekek So according to question Nilesh is father/Mother of Kapil. But
firk ;k ek¡ dh cgu cqvk@ekSlh in options only father given so option (A) correct. Nilesh is the
firk ;k ek¡ dh ek¡ nknh@ukuh father of Kapil.
firk ;k ek¡ ds firk nknk@ukuk 3. There are six members, P, Q, R, S, T and U, in a family. T is the
csVs dh iRuh cgw brother of P’s husband. U is the mother of T. Q is the daughter
ifr ;k iRuh dh cgu uun@lkyh of S and P and the granddaughter of R. How is R related to T?

33
Download Free PDFs & e-Books from Neon Classes App

,d ifjokj esa Ng lnL; gSa] P, Q, R, S, T vkSj U. T, P ds ifr dk Sol. (b)


HkkbZ gSA U] T dh ekrk gSA Q] S vkSj P dh iq=h gS vkSj R dh iksrh Krishna+ Reshma–
gSA R] T ls fdl çdkj lacfa èkr gS\ +
Arvind
(a) Father (b) Son
(c) Brother (d) Uncle Rohit+ Rubi– Juhi –
Sol. (a)
(Krishna is father’s father of Rohit I.e. grandfather)
U– R+ 8. G, M, P, V, Z and D are six members of a family. Z is the son of V.
P– S + T + M is the brother oif V. D and V are a married couple. P is the
daughter of D. D is the sister of G. How is Z related to P?
Q– G] M] P, V] Z vkSj D ,d ifjokj ds Ng lnL; gSaA Z, V dk iq= gSA M]
st nd
(From family tree R is in 1 and T is in 2 generation so option V dk HkkbZ gSA D vkSj V ,d fookfgr ;qxy gSaA P, D dk iq=h gSA D, G
(B) & (C) wrong) (R is the father of T) dh cgu gSA Z] P ls fdl çdkj lacafèkr gS\
4. Mahesh is the father of Riva and paternal grandfather of (a) Mother (b) Sister
Vansh. Aakash is the brother of Riva and father of Vansh and (c) Brother (d) Father
Ritu. Ritu is the only daughter of Aakash and Maya. How is Sol. (c)
Maya related to Mahesh? M+ V + D– G
egs'k jhok ds firk vkSj oa'k ds nknk gSaA vkdk'k jhok dk HkkbZ vkSj
oa'k vkSj fjrq dk firk gSA fjrq] vkdk'k vkSj ek;k dh bdykSrh csVh Z+ P–
(Z is the brother of P)
gSA ek;k egs'k ls fdl çdkj lacafèkr gS\
9. There are 8 members P, Q, R, S, T, U, V and W in a joint family.
(a) Daughter-in-law (b) Aunt
Q is brother of S. V is son of Q and R. S is brother of U. W is
(c) Sister (d) Daughter
daughter of S and T. P is the wife of V. How is R related to P?
Sol. (a)
,d la;qä fijokj esa 8 lnL; P] Q] R] S] T] U] V vkSj W gSaA Q] S
Mahesh+
dk HkkbZ gSA V] Q vkSj R dk csVk gSA S] U dk HkkbZ gSA W] S vkSj T
Riva Aakash+ Maya– dh csVh gSA P] V dh iRuh gSA R dk lacaèk P ds lkFk fdl :i esa
gS\
Vansh+ Ritu– (a) Aunt/pkph
From Family tree, Maya is the daughter in law of Mahesh. (b) Mother-in-law/lkl
5. Priyank is Akshay's brother. Sonia is Sakshant’s sister. Akshay (c) Sister/cgu
is Sonia’s son. How is Priyank related to Sonia? (d) Mother/ekW
fç;kad v{k; ds HkkbZ gSaA lksfu;k] lk{kr dh cgu gSA v{k; lksfu;k ds Sol. (b)
csVs gSaA fç;kad] lksfu;k ls fdl çdkj lacafèkr gS\ R– Q+ U S+ T –
(a) Son/iq= (b) Nephew/Hkrhts
(c) Father/firk (d) Brother/HkkbZ V+ P– W
Sol. (a) (R is the mother in law of P)
Sakshant Sonia – 10. Ram, Rajat and Ramesh are the three sons of Kavita. Mukul is
the son of Ram. Mukul’s paternal grandfather is Jay. Jay has
Akshay+ Priyank+ one daughter, Nikita. Rajat’s son, Abhiram is married to Rani
and their daughter is Ina. Nirmala is the daughter of Nikita.
(Priyank is the son of Sonia)
How is Jay related to Ina?
6. C is the mother of K, K is the wife of H.R is the brother of K. P is
the father of R. How is C related to P?
jke] jtr vkSj jes'k] dfork ds rhu iq= gSaA eqdqy] jke dk iq= gSA
C] K dh ekrk gS] K] H dh iRuh gSA R] K dk HkkbZ gSA P] R dk eqdqy ds nknk t; gSaA t; dh ,d csVh fufdrk gSA jtr ds csV]s
firk gSA C] P ls fdl çdkj lacafèkr gS\ vfHkjke dh 'kknh jkuh ls gqbZ gS vkSj mudh csVh buk gSA fueZyk]
(a) Brother’ wife (b) Wife fufdrk dh iq=h gSA t;] buk ls dSls lacafèkr gS\
(c) Daughter (d) Sister (a) Father-in-law/ llqj
Sol. (b) (b) Paternal uncle/ iSr`d pkpk
(c) Great grandfather/ ijnknk
P+ C –
(d) Grandfather/ nknkth
R+ K– H + Sol. (c)
(C is the wife of P) Jay+ Kavita
7. Rubi and Juhi are sisters. Krishna is Juhi’s father’s father;
Ram+ Rajat+ Ramesh+ Nikita–
Reshma is the mother of Arvind. Arvind is the father of Rohit,
who is the only brother of Rubi. How is Krishna related to Mukul+ Abhiram+ Rani Nirmala–
Rohit?
:ch vkSj twgh cgusa gSaA —".kk twgh ds firk ds firk gSa] js'kek vjfoan Ina–
dh eka gSaA vjfoan] jksfgr dk firk gS] tks :ch dk bdykSrk HkkbZ gSA (Jay is the great grand father Ina)
—".kk dk jksfgr ls D;k lacaèk gS\ 11. Jahnavi is the sister of Taruni. Taruni is married to Dilip. Dilip
is the father of Raghu. Manoj is the son of Hema. Taruni is the
(a) Mother’s brother(b) Father’s father
mother-in-law of Hema. Dilip has only one son and no
(c) Father (d) Mother’s father

34
Best App for Govt. Jobs : Neonclasses (Download Now)

daughter. Jahnavi is married to Barun. Lekha is the daughter of os O;fDr ftuds chp laca/k iz’u gS
Barun. How is Jahnavi’s sister related to Manoj’s paternal 2. Direction of Relation.
grandfather? lac/k dh fn’kk
tkàoh r#uh dh cgu gSaA r#.kh dh 'kknh fnyhi ls gqbZ gSA fnyhi We need to solve complete question just focus on those two
j?kq dk firk gSA eukst gsek dk iq= gSA r#uh gsek dh lkl gSaA family members between which relation is asked.
fnyhi dk ,d gh csVk gS vkSj dksbZ csVh ugha gSA tkàoh dh 'kknh gesa iwjs iz’u dks gy djus dh vko’;drk ugha gS dsoy mu nks
c#.k ls gqbZ gSA ys[kk c#u dh iq=h gSA tkàoh dh cgu dk eukst ds ifjokj lnL;ksa ij /;ku dsfa nzr djssa ftuds chp laca/k iwNk x;k gSA
nknk ls D;k lacaèk gS\ Direction of Relation
(a) Sister (b) Wife
laca/k dh fn’kk
(c) Mother-in-law (d) Mother
Sol. (b)
Barun Jahnavi Taruni– Dilip+
Forward Relation/vkxs dh vksj
Lekha Raghu+ Hema–

Manoj
A is father of B(A, B dk firk gS )
(Jahnavi’s sister Taruni is wife of manoj’s Paternal grandfather
Dilip) A is mother of B (A, B dh ek¡ gS )
12. Each child in a family has at least four brothers and three
sisters. Then what will be the minimum number of children in Direction of Relation
the family? laca/k dh fn’kk
,d ifjokj esa izR;sd cPpsa ds de ls de 5 HkkbZ vkSj 4 cgus gSA rc
ifjokj esa cPpksa dh U;wure la[;k D;k gksxh\
(a) 9 (b) 10 (c) 11 (d) 8 Forward+ Backword Relation
Sol. (c)
/vkxs $ i'porhZ dh vksj
For every boy to have at least four brothers, it is necessary to
have five boys in the family. Similarly, for each girl to have at
least three sisters, it is necessary to have four girls. A is father of B(A, B dk firk gS )
Hence, the minimum number of children izR;sd yM+ds ds de ls
de 5 HkkbZ gksus ds fy, ifjokj esa 6 yM+ds gksuk vko’;d gS blh B is mother of A (B, A dh ek¡ gS )
izdkj izR;sd yM+dh dh de ls de 4 cgus gksus ds fy, 5 yM+fd;k¡ Here we have to move in alphabetical order for ease of
gksuk vko’;d gSA question
vr% cPpksa dh u;wure la[;k ;gka gesa iz’uksa esa vklkuh ds fy, vaxszth o.kkZuqØe esa tkuk gksxkA
= 6+5=11 14. If ‘A#B’ means ‘A is the father of B’, ‘A $ B’ means ‘A is the
ΔΔΔΔΔΔ mother of B’, ‘A @B’ means ‘A is the husband of B’, ‘A % B’
13. In a family there are one grand father and one grand mother. means ‘A is the wife of B’ ‘A = B’ means ‘A is the brother of B’,
There are two fathers and two mothers. There are four then how is P related to V in the following expression?
children and three grand children in the family. ;fn ‘A#B’ dk vFkZ gS A, B dk firk gS'] 'A $ B' dk vFkZ gS ‘A, B
There are one brother, two sisters, two sons, two daughters, dh eka gS'] ‘A@B' dk vFkZ gS A, B dk ifr gS'] ‘A%B' dk vFkZ gS 'A]
one father-in-law, one mother-in-law and one daughter-in-law.
B dh iRuh gS' 'A = B' dk vFkZ gS 'A] B dk HkkbZ gS'] rks fuEu O;atd esa
How many members are there in the family?
P, V ls fdl çdkj lacafèkr gS\
,d ifjokj esa ,d nknk vkSSj nknh gSA nks firk vkSj nks ekrk,¡ gSA
P@Q$U=R%S#V
ifjokj esa pkj cPps vkSj rhu iksrs&iksrh gSA ,d HkkbZ vkSj nks cgusa gSA
(a) Mother’s brother
nks csVs vkSj nks csfV;k¡ gSA ,d lkl vkSj ,d llqj gS vkSj , iq=o/kw (b) Paternal grandfather
gSA rc ifjokj esa dqy fdrus lnL; gS\ (c) Father’s brother
(a) 7 (b) 15 (c) 22 (d) 23 (d) Maternal grandfather
Sol. (a) Sol. (d)
The same person has different relationships with different
individuals in the family. For e.g. a boy is the son of his father Q
and is the father of his son.Similarly, a girl is the daughter of P
her parents while for grand parents she is the grand-daughter.
So, in this way the following diagram will be obtained by
connecting all the relations:
P R S
Δ+
+Δ
Δ V
Hence, there are 7 members in the family.
Concept- Coded equations Clearly from the above family tree we can canclude that ‘p’ is
In such questions check two things maternal grand father of V.
,sls iz’uksa esa nks phtksa dh tkap djsa 15. A – B means ‘A is the sister of B’
1. Persons between which relation is asked. A - B dk vFkZ gS 'A] B dh cgu gS

35
Download Free PDFs & e-Books from Neon Classes App

A × B means ‘A is the husband of B’; (a) Husband/ ifr


A × B dk vFkZ gS 'A] B dk ifr gS (b) Son/ csVk
A ÷ B ‘A is the son of B’ (c) Brother/ HkkbZ
A ÷ B dk vFkZ gS 'A] B dk csVk gS (d) Father/firk
If, Y × M – T – Z ÷ L × U, then how is U related to Y? Sol. (a)
;fn] Y × M – T – Z ÷ L × U] rks U] Y ls fdl çdkj lacafèkr gS\ Mathod 1 : Family Tree
(a) Father-in-law/llqj
A@L#F*E×G$C
(b) Mother-in-law/lkl
A+ C–
(c) Father/firk
(d) Mother/eka –
Sol. (b) L– F+ E G+
(A is the husband of C)
U
L
Mathod 2 : Elimination mathod
A @ L # F * E × G $ C
(+1) (+0) (+0) (+0) (–1)
Y M T Z st
(b) A and C both are in 1 generation so option eliminated.
Clearly from the st
(c) A & C both are in 1 generation but relation between them
above family tree we can canclude that ‘V’ is mother-in-law of
is husband – wife so option (c) eliminated.
Y. st
16. A # B means ‘A is the mother of B’ (d) There is no generation gap between A & C both are in 1
A @ B means ‘A is the sister of B’ generation so option eliminated.
A & B means ‘A is the husband of B’ 18. A + B means ‘B is the daughter of A’;
A % B means ‘A is the wife of B’ A + B dk vFkZ gS 'A, B dh csVh gS
A # B dk vFkZ gS ‘A, B dh eka gS' A - B means ‘B is the sister of A’;
A @ B dk vFkZ gS ‘A, B dh cgu gS' A - B dk vFkZ gS 'A, B dh cfgu gS
A & B dk vFkZ gS ‘A, B dk ifr gS' A × B means ‘B is the husband of A’;
A % B dk vFkZ gS ‘A, B dh iRuh gS' A × B dk vFkZ gS 'A, B dk ifr gS
If W & Q @ T # Y @ M % K, then how is K related to T? A ÷ B means ‘A is the father of B’.
;fn W & Q @ T # Y @ M % K, rks K, T ls fdl çdkj lacfa /kr gS\ A ÷ B dk vFkZ gS 'A, B dk firk gS
(a) Brother/ HkkbZ If, P ÷ R × T + Q - S × U + Z, then how is R related to Z?
(b) Wife’s brother/ iRuh dk HkkbZ ;fn] P ÷ R × T + Q - S × U + Z, rks R] Z ls fdl çdkj lacfa èkr gS\
(c) Daughter’s husband/ iq=h dk ifr (a) Maternal grandfather/ukuk
(d) Son/ csVk (b) Maternal grandmother/ekr` nknh
(c) Paternal grandfather/iSr`d nknk
Sol. (c)
According to question- (d) Paternal grandmother/iSr`d nknh
Sol. (b)
Q T Mathod 1 : Family Tree
W
P+

R– T+
Y M K
– +
Clearly from the Q– S U
above family tree we can conclude that ‘K’ is daughter’s
Z–
husband of T.
(R is the maternal grandmother of Z)
17. ‘C @ D’ means ‘C is the father of D’.
‘C * D’ means ‘C is the brother of D’. Mathod 2 : Elimination mathod
'C × D’ means ‘D is the husband of C’. P ÷ R × T + Q – S
‘C # D’ means ‘C is the sister of D’. (+1) (+0) (+1) (+0)
‘C $ D’ means ‘C is the son-in-law of D’. × U + Z
‘C = D’ means ‘C is the mother of D’. (+1) (+1)
'C @ D' dk vFkZ gS 'C, D dk firk gS' nd th
(From given symbol R is in 2 generation and Z is in 4
'C * D' dk vFkZ gS 'C, D dk HkkbZ gS'
generation so the generation gap between them 2)
'C × D' dk vFkZ gS 'C, D dk ifr gS'
(a) (×) symbol gender of R is female so gets eliminated.
'C # D' dk vFkZ gS 'C, D dk cgu gS'
(c) Again gender of R is male so option gets eliminated because
'C $ D' dk vFkZ gS 'C, D dk nkekn gS'
gender of R must be female according to answer.
'C = D' dk vFkZ gS 'C, D dk ekrk gS' (d) From (×) symbol S is female and Z is her daughter so R is
How is A related to C in the following expression? not the paternal grandmother so gets eliminated.
fuEufyf[kr O;atd esa A] C ls fdl çdkj lacaf/kr gS\ 19. If ‘A # B’ means A is the father of B,
A@L#F*E×G$C ;fn ‘A # B’ dk vFkZ gS fd A] B dk firk gS]

36
Best App for Govt. Jobs : Neonclasses (Download Now)

‘A & B’ means A is the brother of B, (c) Beacause gender of Y is male so option (c) gets eliminated.
‘A & B’ dk vFkZ gS A] B dk HkkbZ gS] 21. ‘A + B’ means ‘A is the daughter of B’.
‘A @ B’ means A is the mother of B, 'A $ B' dk vFkZ gS 'A, B dh csVh gS'A
‘A @ B’ dk vFkZ gS A] B dh eka gS] ‘A $ B’ means ‘A is the husband of B’.
Then, which of the following is definitely true about C @ A # B 'A $ B' dk vFkZ gS 'A, B dk ifr gS'A
& D? ‘A @ B’ means ‘A is the brother of B’.
rks] fuEufyf[kr esa ls dkSu C @ A # B & D ds ckjs esa fuf'pr :i 'A @ B' dk vFkZ gS 'A, B dk HkkbZ gS'A
ls lR; gS\ ‘A & B’ means ‘A is the mother of B’.
(a) C is paternal grandmother of B and D/C, B vkSj D dh iSr`d nknh 'A & B' dk vFkZ gS 'A, B dh eka gS'A
gS ‘A % B’ means ‘A is the son of B’.
(b) D is granddaughter of C./D, C dh iksrh gSA 'A % B' dk vFkZ gS 'A, B dk csVk gS'A
(c) C is mother of A and D./C, A vkSj D dh eka gSA If ‘W @ S % K $ G & U & T @ R + C’, then which of the following
(d) D is daughter of A./D, A dh csVh gSA statements is correct?
Sol. (a) ;fn ‘W @ S % K $ G & U & T @ R + C’, rks fuEufyf[kr esa ls dkSu
lk dFku lgh gS\
Mathod 1 : Family Tree
(a) G is the maternal grandmother of R./ G] R dh ukuh gSA
C@A#B&D
(b) G is the father of W./ G] W dk firk gSA
C–
(c) C is the father-in-law of K./ C] K dk llqj gSA
(d) C is the wife of U./ C, U dh iRuh gSA
A+ Sol. (a)
Mathod 1 : Family Tree
B+ D K+ G–
(C is the paternal grandmother of B and D)
+
Mathod 2 : Elimination mathod W S+ U – C+
C @ A # B & D T+ R–
(+1) (+1) (+0) (G is the maternal grandmother of R)
(b) Gender of D not spacified so option (b) gets eliminated. Mathod 2 : Elimination mathod
st rd
(c) C is in 1 generation while D is in 3 generation so there is W @ S % K $ G & U
two generation gap so option (c) gets eliminated. (+0) (–1) (+0) (+1)
(d) Gender of D not known so option (d) gets eliminated. & T @ R + C
20. ‘A & B’ means ‘A is the sister of B’. (+1) (+0) (–1)
‘A % B’ means ‘A is the brother of B’.
(b) Gender of G is female so gets eliminated
‘A $ B’ means ‘A is the mother of B’. st nd
‘A # B’ means ‘A is the wife of B’. (c) K is in 1 gemeration and C is in 2 generation so k is older
If U $ Q & Y % D $ S # K, then which of the following than C so eliminated.
statements is NOT correct? (d) Gender of C is male so gets eliminated.
‘A & B’ dk vFkZ gS ‘A, B dh cgu gS’A 22. B % D’ means ‘B is the brother of D’.
‘A % B’ dk vFkZ gS ‘A, B dk HkkbZ gS’A ‘B & D’ means ‘B is the mother of D’.
'B × D’ means ‘B is the husband of D’.
‘A $ B’ dk vFkZ gS ‘A, B dh eka gS’A
‘B # D’ means ‘B is the sister of D’.
‘A # B’ dk vFkZ gS ‘A, B dh iRuh gS’A
‘B $ D’ means ‘B is the son of D’.
;fn U $ Q & Y% D $ S # K, rks fuEufyf[kr esa ls dkSulk dFku lgh
ugha gS\ ‘B @ D’ means ‘B is the father of D’.
(a) D is the mother-in-law of K./ D, K dh lkl gSA 'B % D' dk vFkZ gS 'B, D dk HkkbZ gS'
(b) Q is the sister of D./ Q, D dh cgu gSA 'B & D' dk vFkZ gS 'B, D dk eka gS'
(c) Y is the daughter of U./ Y, U dh iq=h gSA 'B × D' dk vFkZ gS 'B, D dk ifr gS'
(d) U is the mother of D./ U, D dh ekrk gSA 'B # D' dk vFkZ gS 'B, D dk cgu gS'
Sol. (c) 'B $ D' dk vFkZ gS 'B, D dk iq= gS'
'B @ D' dk vFkZ gS 'B, D dk firk gS'
Mathod 1 : Family Tree
Which of the following statements is correct regarding the
U– given expression?
nh xbZ vfHkO;fä ds laca/k esa fuEufyf[kr esa ls dkSu lk dFku lgh
Q– Y+ D– gS\
S– K+ P%Q$S#R%U×T
(Y not daughter of U) (a) P is the son of T.
(b) S is the brother-in-law of T.
Mathod 2 : Elimination mathod (c) U is the mother’s brother of P.
U $ Q & Y % D $ S # K (d) P is the daughter of T.
(+1) (+0) (+0) (+1) (+0) Sol. (c)

37
Download Free PDFs & e-Books from Neon Classes App

Mathod 1 : Family Tree 24. ‘A×B’ means ‘A is the brother of B’. ‘A−B’ means ‘A is the
mother of B’.‘A÷B’ means ‘A is the father of B’.Which of the
S – R+ U+ T –
following means ‘M is the son of Q’?
+ ‘A×B’ dk vFkZ gS 'A, B dk HkkbZ gS'A 'A & B' dk vFkZ gS 'A, B dh
P Q+ eka gS'A ‘A ÷ B’ dk vFkZ gS 'A B dk firk gS'A buesa ls dkSu lk
(U is the mother’s brother of P) fuEufyf[kr dk vFkZ gS 'M] Q dk iq= gS'\
Mathod 2 : Elimination mathod (a) M×R÷Q (b) Q÷M×R
P % Q $ S # R % U×T (c) M÷R×Q (d) M−R÷T
(+0) (–1) (+0) (+0) (+0) Sol. (b)
nd st Mathod 1 : Family Tree
(a) P is in 2 generation, T is in 1 generation but P is nephew M+ R+
so gets eliminated. (a)
(b) Gender of S is female so gets eliminated. _ Q
(d) Gender of P is male so gets eliminated. st
(M is in 1 generation so eliminated)
23. If ‘A # B’ means ‘A is the sister of B’ and ‘A @ B’ meeans ‘A is
Q+
the mother of B’, then which of the following expressions (b)
means ‘C is the mother of F’? _ M+ R
;fn ‘A # B’ dk vFkZ gS 'A, B dh cgu gS' vkSj ‘A@B’ dk vFkZ gS 'A,
(M is the son of Q)
B dh eka gS'] rks fuEufyf[kr esa ls fdl vfHkO;fä dk vFkZ gS ‘C, F
dh eka gS'\ M+
(c)
(a) C @ H # M # F
_ R+ Q
(b) C @ H # M @ F st
(c) H @ C # M # F (M is in 1 generation so gets eliminated)
(d) F @ H # M # C M+
Sol. (a) (d)
R+
Mathod 1 : Family Tree
C– T
(a)
st
_ (M is in 1 generation so gets eliminated)
H– M– F Mathod 2 : Elimination mathod
(C is the mother’s brother of F) (a) M × R ÷ Q
C– (+0) (+1)
nd st
(b) H – M – (Q is in 2 generation and M is in 1 generation so gets
_ eliminated)
F
M ÷ R ÷ T
rd
(F is in 3 generation so gets eliminated) (c)
(+1) (+0)
H– st nd
(c) (M is in 1 generation and Q is in 2 generation so gets
_ C – M– F eliminated)
(C & F in same generation so gets eliminated) M – R ÷ T
(d)
F– (+1) (+1)
(d)_ (M’s gender is female & Q is not available in equation so
H– M – C
eliminated)
st
(F is in 1 generation so gets eliminated) 25. ‘A # B’ means ‘A is the sister of B’;
‘A $ B’ means ‘A is the father of B’;
Mathod 2 : Elimination mathod
(b) ‘A @ B’ means ‘A is the wife of B’;
C @ H # M@ F
‘A % B’ means ‘A is the brother of B’.
(+1) (+0) (+1)
Which of the following options means ‘J is the father of R’?
rd
(F is in 3 generation here generation gap between C & F so ‘A # B’ dk vFkZ gS fd ‘A, B dh cgu gS’A
gets eliminated) ‘A $ B’ dk vFkZ gS fd ‘A, B dk firk gS’A
H @ C # M# F ‘A @ B’ dk vFkZ gS fd ‘A, B dh iRuh gS’A
(c)
(+1) (+0) (+0) ‘A % B’ dk vFkZ gS fd ‘A, B dk HkkbZ gS’A
fuEufyf[kr esa ls fdl fodYi dk vFkZ gS fd ‘J, R dk firk gS*\
(C & F is in same generation so gets eliminated)
(a) C @ J % K $ M # R (b) C @ R $ K % M # J
F @ H # M # C (c) C @ J $ K % M # R (d) J @ C $ K % M # R
(d)
(+1) (+0) (+0) Sol. (c)
(C’s gender is not defined & C is in 2
nd
generation so gets Mathod 1 : Family Tree
C – J + K+
eliminated) (a)
_ Q M–

38
Best App for Govt. Jobs : Neonclasses (Download Now)

(J is uncle of R so eliminated) A– B+
C – R+ (c)
(b) _ C– E+
_ K + M– J
D+
nd
(J is in 2 generation so gets eliminated)
(J is father of R)
C– J +
(c) E
(d)
_ K + M– R –
_ B C– D+
(J is father of R)
A+
J – C+
(d)
(A is E’s daughter’s son)
_ K + M– R
(Gender of J is female so eliminated) Mathod 2 : Elimination mathod
(a) A ∩ B ∩ C / D + E
Mathod 2 : Elimination mathod
(–1) (–1) (+0) (+1)
(a) C @ J % K $ M # R nd
(+0) (+0) (+1) (+0) (E is in 2 generation & gender of A is female so eliminated)
st
(J is in 1 generation and R is in 2
nd
but because of % J is uncle A / B ∪ C ∪ D / E
(b) (+0) (–1) (–1) (+0)
of R so gets eliminated)
C @ R $ K % M # J (Gender of A is female so gets eliminated)
(b) (+0) (+1) (+0) (+0) A / B + C – D ∪ R
st nd (c) (+0) (+1) (+1) (–1)
(R is in 1 generation and J is in 2 and also J’s gender is not
known so gets eliminated) st
(A is in 1 generation & gender of A is female so gets
J @ C $ K % M # R eliminated)
(d) (+0) (+1) (+0) (+0) 27. ‘Y ① Z’ means ‘Y is the sister of Z’
‘Y ② Z’ means ‘Y is the husband of Z’
(Gender of J is female so gets eliminated) ‘Y ③ Z’ means ‘Y is the daughter of Z’
26. If /;fn ‘Y ④ Z’ means ‘Y is the mother of Z’
‘P + Q’ means ‘P is the father of Q, ‘Y ⑤ Z’ means ‘Y is the brother of Z’
'P $ Q' dk vFkZ gS 'P] Q dk firk gS] In the following expression, select the option that can replace
‘P - Q’ means ‘P is the mother of Q’, the question mark (?) to establish that ‘C is the child of L’
'P - Q' dk vFkZ gS 'P] Q dh ekW gS] U④L②G④M①O?C
‘P / Q’ means ‘P is the sister of Q’. ‘Y ① Z’dk vFkZ gS ‘Y, Z dh cgu gS
'P / Q' dk vFkZ gS 'P] Q dh cfgu gS] ‘Y ② Z’dk vFkZ gS ‘Y, Z dk ifr gS
‘P ∪ Q’ means ‘P is the son of Q’, ‘Y ③ Z’ dk vFkZ gS ‘Y, Z dh iq=h gS
'P ∪ Q' dk vFkZ gS 'P] Q dk iq= gS] ‘Y ④ Z’ dk vFkZ gS ‘Y, Z dh ekW gS
‘P ∩ Q’ means ‘P is the daughter of Q’, ‘Y ⑤ Z’ dk vFkZ gS ‘Y,Z dk HkkbZ gS
'P ∩ Q' dk vFkZ gS 'P] Q dh iq=h gS] In the following expression, select the option that can replace
Then which of the following means that ‘A is E’s daughter’s the question mark (?) to establish that ‘C is the child of L’
son’? ml fodYi dk p;u djs]a tks fuEufyf[kr O;atd esa iz’u fpUg ds
rks fuEu esa ls fdldk vFkZ gS fd 'A] E dh iq=h dk iq= gS'\ LFkku ij vkdkj ;g LFkkfir dj ldrk gS & ‘C, L dh larku gSA
(a) A ∩ B ∩ C / D + E U④L②G④M①O?C
(b) A/B ∪ C ∪ D / E (a) Only ‘②’ (b) Only ‘④’
(c) A/B + C – D ∪ E (c) Either ‘⑤’ or ‘④’ (d) Either ‘①’ or ‘⑤’
(d) A ∪ B / C /D ∪ E Sol. (d)
Sol. (d)
Mathod 1 : Family Tree
Mathod 1 : Family Tree U–
C – D+ (a)
(a) _ L+ G–
_ B– E
+
M– O C
A–
(C is daughter in law of L so wrong)
(gender of A is female)
U–
D– E
(b) L+ –
G
(b) C+ _
_ M– O–
B+ A
C
(Gender of A is female) (C is grand child of L so wrong)

39
Download Free PDFs & e-Books from Neon Classes App

U– W%O–B$C+D
(d) L+ – (c) C+
G
_ _
O– B + D+
M– O– C +
W –
(C is the child of L )
(D is the son of C so wrong)
U–
W%O–B+C#D

L+ G (d) O– B+
_
M– O– C –
W+ D C+
(C is the child of L ) (D is the wife of W so true)
Mathod 2 : Elimination mathod
2 Mathod 2 : Elimination mathod
(a) (a)
U 4 L 2 G 4 M 1 O ? C W % O – B + C – D
(+1) (+0) (+1) (+0) (+0) (-1) (+0) (+1) (+0)
(So from code (2) between O & C, denotes C is daughter in law (No relation shown between W & D so gets eliminated)
of L so gets eliminated) W% O – B % C – D
4 (b) (-1) (+0) (-1) (+0)
U 4 L 2 G 4 M 1 O ? C
(b) rd st
(+1) (+0) (+1) (+0) (+0) (W is in 3 generation and D is in 1 generation so gets
(So from code (4) C is the child of O so eliminated) eliminated)
(c) Because in option (c) code (4) angain appear between O & W% O – B $ C + D
C so gets eliminated from option (B). (c) (-1) (+0) (-1) (+1)
28. ‘Z + Y’ means ‘Y is the son of Z’. rd nd
(W is in 3 generation and D is in 2 generation and gender of
‘Z $ Y’ means ‘Y is the father of Z’.
D male so eliminated)
‘Z % Y’ means ‘Y is the son-in-law of Z’.
29. Given:/fn;k gS%
‘Z – Y’ means ‘Y is the wife of Z’.
‘Z * Y’ means ‘Z is the brother of Y’. M + N meanseM is the brother of N.
‘Z # Y’ means ‘Z is the only sister of Y’. M + N dk vFkZ gS] ÞM, N dk HkkbZ gSÞA
Which two symbols can sequentially replace the question M × N meanseM is the husband of N.
marks (?) in the following expression to show that ‘D is the M × N dk vFkZ gS] ÞM, N dk ifr gSÞA
wife of W’? M ÷ N meanseM is the daughter of N.
W%O–B?C?D M ÷ N dk vFkZ gS] ÞM, N dh iq=h gSÞA
‘Z + Y’ dk vFkZ gS fd ‘Y, Z dk iq= gS*A M @ N meanseM is the mother of N.
‘Z $ Y’ dk vFkZ gS fd ‘Y, Z dk firk gS*A M @ N dk vFkZ gS] ÞM, N dh ek¡ gSÞA
‘Z % Y’ dk vFkZ gS fd ‘Y, Z dk nkekn gS*A (i) If P×R÷T+S÷U
‘Z – Y’ dk vFkZ gS fd ‘Y, Z dk iRuh gS*A ;fn P×R÷T+S÷U rks U dk P ls D;k laca/k gS\
‘Z * Y’ dk vFkZ gS fd ‘Z, Y dk HkkbZ gS*A (a) Grandfather-in-law@nknk&llqj
‘Z # Y’ dk vFkZ gS fd ‘Z, Y dk bdykSrh cgu gS*A (b) Grandfather@nknk
fuEufyf[kr O;atd esa iz’uokpd fpUgksa (?) ds LFkku ij Øfed :i (c) Grandmother-in-law@nknh&lkl
ls vk ldus okys dkSu ls nks fpUg ;g n’kkZ ldrs gSa fd ‘D, W dh (c) Grandfather-in-law/Grandmother-in-law@nknk&llqj ;k
iRuh gS*\ nknh lkl
W%O–B?C?D (ii) Which of the following expressions means A is the daughter-
(a) + and – (b) % and – (c) $ and + (d) + and # in-law of B?
Sol. (d) fuEufyf[kr esa ls dkSulh vfHkO;fDr dk vFkZ gS fd A, B dh iq=o/kw
Mathod 1 : Family Tree gS\
W%O–B+C –D (a) A@C×D@B
(a) O– B + (b) B@C×A@D
_ (c) A@C×B@D
W C + D–
(d) A÷C÷D÷
(D is the wife of C so wrong) (e) None of these
W%O–B%C–D Sol. (i) (D)
(b) D– C + U
_
O– B + T S
W+
R
(D is the wife of c so wrong) P
Here, U is the grandfather/grandmother of P’s wife. Therefore,
U will be the grandfather-in-law/grandmother-in-law of P.

40
Best App for Govt. Jobs : Neonclasses (Download Now)

;gk¡ U, P dh iRuh ds nknk@nknh gS] vr% U, P ds nknk&llqj ;k Grand father/Grand mother = +2


nknh&lkl gksx
a As Her/Mother = +1
(ii) Here, A is female. Therefore eliminate the options in which A is Brother/Sister/Wife/Husband = 0
a male. Daughter/Son=-1
;gk¡ A efgyk gS] vr% ftu fodYiksa esa A iq:"k gS mUgsa gVk fn;k Grandson/Grand daughter = -2
tk,xkA Concept-
Option (A): A@C = A is the mother (female) of C. Pointing form/ikWbafVax QkWe
fodYi (A): A@C = A, C dh ek¡ ¼efgyk½ gSA (i) Direct form questions: In such questions we give the
Option (B): A@D = A is the mother (female) of D. introduction of the same person whom we point to.
fodYi (B): A@D = A, D dh ek¡ ¼efgyk½ gSA izR;{k :i okys iz’u% ,sls iz’uksa esa ftl O;fDr dh vksj b’kkjk fd;k
Option (C): A@C = A is the mother (female) of C. tkrk gS] mlh O;fDr dk ifjp; fn;k tkrk gS vFkkZr~ mlh O;fDr ds
fodYi (C): A@C = A, C dh ek¡ ¼efgyk½ gSA ckjs esa crk;k tkrk gSA
Option (D): A÷C = A is the daughter (female) of C. (ii) Indirect form questions: In such questions we don’t give the
fodYi (C): A@C = A, C dh iq=h ¼efgyk½ gSA introduction of the same person whom we point to. Instead, a
A is female in all the options. Therefore, no option can be person directly or indirectly related to that person is
eliminated. In such a situation, there ae two methods of solve introduced.
the questions. There are two methods to solve both types of questions. Let’s
;gk¡ lHkh fodYiksa esa A efgyk gS] vr% fdlh Hkh fodYi dks ugha see the following examples that will better illustrate our point.
gVk;k tk ldrkA ,slh fLFkfr esa iz’u dks gy djus ds nks rjhds gS& vizR;{k :i okys iz’u% ,sls iz’uksa esa ftl O;fDr dh vksj b’kkjk fd;k
Method-I: In this method we will find the correct option by tkrk gS] ge ml O;fDr dk ifjp; ugha nsrsA blds ctk; ml O;fDr
making diagrams for each option. ls izR;{k ;k vizR;{k :i ls lacaf/kr O;fDr dk ifjp; fn;k tkrk gSA
igyk rjhdk% blesa izR;sd fodYi ds fy, vkjs[k dk fuekZ.k dj lgh 30. Introducing Reshma to a guest, a lady, Karishma, said, “She is
fodYi Kkr djsaxsA the daughter of my father-in-law’s daughter”. How is Reshma
Option (A) : A@C×D@B – related to Karishma?
,d vfrfFk ls js’kek dk ifjp; djkrs gq,] ,d efgyk] dfj’ek us
A
dgk] ^^og esjs llqj dh csVh dh csVh gS**A js’kek dk dfj’ek ls D;k
laca/k gSA
C D
(a) sister-in-law/HkkHkh
B (b) Sister/cgu
(c) Cousin/ppsjh cgu
Here, A is the grandmother of B. Therefore, it is incorrect. (d) Niece/Hkkath
;gk¡ A, B dh nknh gS] vr% xyr gSA Sol. (d)
Option (B): B@C×A@D- Method – 1
B Speaker ¼oDrk½ = Karishma ¼dfj’ek½
(About whom the speaker is talking) = Reshma
C A fdlds ckjs esa ckr dh xbZ
The statement (Deduction Approach)
D “ She is the daughter of my father-in-law’s daughter”.

Here, A is the daughter-in-law of B. She is the daughter of sister-in-law


;gk¡ A, B dh iq=o/kw gSA
(If the option (B) would not be correct, then we had to create She is my Niece

diagrams for other options ass well.) Method – 2


¼;fn fodYi (B) lgh ugha gksrk rc v
fodYiksa ds fy, Hkh vkjs[k cukus iM+rsA½ Karishma’s father-in-law
Method-II: It is called generation gap trick. If a person is one +
Karishma– husband(Karishma) Sister –
generation above us, he will be given ‘+1’. Similarly if a person
is two generation above us, he will be given ‘+2’ and a person Reshma–
of the same generation will be given ‘0’. 31. Introducing Rajat to a guest, Ajay said, "He is the only son of
nwljk rjhdk%& bls ih<+h ds varj dh fVªd dgrs gSA ;fn dksbZ O;fDr my father's daughter-in law". How is Ajay related to Rajat?
gels ,d ih<+h Åijh gS rks mls *$1^ fn;k tk,xkA blhizdkj ;fn ,d vfrfFk ls jtr dk ifjp; djkrs gq,] vt; us dgk] "og esjs
dksbZ O;fDr gels nks ih<+h;k¡ Åij gS rks mldks ‘+2’ fn;k tk;sxk vkSj firk dh iq=oèkw dk bdykSrk iq= gS"A vt;] jtr ls fdl çdkj
leku ih<+h ds O;fDr dks ^0^ fn;k tk,xkA lacafèkr gS\
;fn dksbZ O;fDr gels ,d ih<+h uhps gS rks (a) Maternal grandfather/ ukuk
A person is one generation below us, he will be given ‘-1’. (b) Paternal grandfather/ iSr`d nknk
Similarly, if a person is two generation below us, he will be (c) Brother/ HkkbZ
given ‘-2’. (d) Either father or uncle/;k rks firk ;k pkpk
mls ^&1^ fn;k tk,xkA blh izdkj dksbZ O;fDr gels nks ih<+h uhps gS Sol. (d)
rks mls ^&2^ fn;k tk,xkA Method – 1
In conclusion: Deduction method (dVkSrh fof/k)

41
Download Free PDFs & e-Books from Neon Classes App

Speaker (oDrk) = Ajay (vt;) ,d efgyk dh vksj b'kkjk djrs gq, jhuk us dgk] "og esjs llqj dh
About whom the speaker talking = Rajat bdykSrh csVh gSA" efgyk jhuk ls fdl çdkj lacafèkr gS\
Statement (a) Sister (b) Mother
“ He is the only son of my father’s daughter-in-law”. (c) Daughter (d) Sister-in-law
Sol. (d)
He is the only son of my wife/sister-in-law.
Speaker = Reena
He is my Son/Nephew Statement
Rajat Ajay “She is the only daughter of my father-in-law”.
It is asked, how is Ajay related to Rajat.
She is my Sister-in-law
Ajay is Son/Nephew of Rajat
Rajat is uncle/ father of Ajay. 34. Pointing towards a man, a lady said, “His brother is my
Method – 2 mother’s only daughter’s husband’s son’s father”. How is the
man related to the lady’s husband?
Ajay’s father ,d vkneh dh vksj b'kkjk djrs gq,] ,d efgyk us dgk] mldk HkkbZ
esjh ek¡ dh bdykSrh csVh ds csVs dk firk gSA iq#"k efgyk ds ifr ls
Ajay + Ajay’s wife dSls lacfa èkr gS\
Rajat + (a) Paternal uncle/iSr`d pkpk
(b) Brother/HkkbZ
Ajay’s father (c) Father/ firk
(d) Brother-in-law/cguksbZ
Ajay+ Brother of Ajay ()–
Sol. (b)
Rajat Speaker = lady
32. Arunima told Sahil, “Your brother-in-law’s only sister's son is Statement
my grandson.” How is Sahil related to Arunima? His brother is my mother’s only daughter’s son’s father

v#f.kek us lkfgy ls dgk] "rqEgkjs lkys dh bdykSrh cgu dk csVk His brother is my son’s father
esjk ukrh gSA" lkfgy dk v#f.kek ls D;k laca/k gS\
His brother is my husband
(a) Son-in-law/nkekn man lady
(b) Uncle/pkpk
Relation is, man is lady’s husband’s brother.
(c) Son/csVk
Method – II
(d) Father-in-law/llqj
Mother–
Sol. (a)
Method – 1 Man Brother + Lady –
Speaker = Arunima
To whom the speaker talking = Sahil Both are brothers Son
Statement – I
35. Pointing towards the photograph of a lady, Varun said, “She is
Your brother-in-law’s on sister’s son my father-in-law’s son’ only sister’s mother-in-law”. How is
Varun related to that lady?
Your wife’s son ,d efgyk dh rLohj dh vksj b’kkjk djrs gq,] o:.k us dgk] ^^og
esjs llqj ds csVs dh bdykSrh cgu dh lkl gS**A o:.k dk ml
Your son efgyk ls D;k laca/k gS\
Statement – II (a) Brother/HkkbZ
“ Is my grandson” (b) Son/csVk
Here ‘my’ came for speaker Arunima (c) Husband/ifr
Connecting both statements we get Arunima can be mother or (d) Son-in-law/nkekn
mother-in-law of Sahil. Sol. (b)
It is asked that how is Sahil related to Arunima Sahil is either Statement
son or son-in-law of Arunima. She is my father-in-law’s son’s only siter’s mother-in-law
Arunima
She is my brother-in-law’s only sister’s mother-in-law
Sahil Sahil’s wife Sahil’s
brother-in-law She is my wife’s mother-in-law

Sahil’s son She is my mother


Lady Varun
Arunima Method – II
Sahil Sahil’s wife Sahil’s Lady – Varun’s father-in-law +
brother-in-law
+
Sahil’s son Varun+ Varun’s wife– Varun’s
brother-in-law
33. Pointing towards a woman, Reena said, “She is the only
36. Pointing to the photograph of a man Gopesh, a lady said, “his
daughter of my father-in-law.” How is the woman related to
daughter’s father’s mother’s only daughter’s husband’s
Reena?

42
Best App for Govt. Jobs : Neonclasses (Download Now)

father’s only son is the husband of my husband’s sister.” If A leap year is a year which has 366 days (52 week + 2 days).
Gopesh is the only son of his parents, how is the lady related to Such years are exactly divisible by 4. Eg. 2004, 2008, 2012 etc.
Gopesh? Leap year in the form of a Century are exactly divisible by 400
,d iq#"k xksis’k dh rLohj dh vksj b’kkjk djrs gq,] ,d efgyk us eg. 400, 1200 etc.
dgk] ^^mldh csVh ds firk dh eka dh bdykSrh csVh ds ifr ds firk yhi o"kZ og o"kZ gksrk gS ftlesa 366 fnu ¼52 lIrkg + 1 fnu½ gksrs
dk bdykSrk csVk esjs ifr dh cgu dk ifr gSA** ;fn xksi’s k vius gSA bl izdkj ds o"kZ pkj ls iwjh rjg foHkkftr gksrs gSA eg. 2004,
ekrk&firk dk bdykSrk iq= gS] rks efgyk dk xksis’k ls D;k laca/k gS\ 2008, 2012 etc. yhi o"kZ] ’krkCnh ds #i esa 400 ls iwjh rjg
(a) Niece (b) Wife (c) Sister (d) Mother foHkkftr gksrs gSA tSls 400] 1200 etc.
Sol. (b) Note:- Generally there is a time gap of 4 years between any 2
His (daughter’s father’s) years. But as we know that every century year is not a leap
mother’s only daughter’s husband’s year. Therefore, some times the time gap between 2 leap years,
father’s only son is the husband of my can be of 8 years. For e.g. The next leap year after the leap year
husband’s sister. 1896 will be 1904 which will come after a time gap of 8 years.
Because in the middle 1900 is a normal century year.
(His mohter’s only daughter’s) husband’s lkekU;r% nks vf/ko"kksaZ ds chp 4 o"kZ dk le;kUrjky gksrk gSA ysfdu
father’s only son is the husband of tSlk fd ge tkurs gS fd izR;sd 'krkCnh o"kZ] vf/ko"kZ ugha gksrkA vr%
my husband’s sister. nks vf/ko"kksaZ ds chp 8 o"kksaZ dk le;kUrjky Hkh gks ldrk gSA ;fn chp
esa lk/kkj.k 'krkCnh o"kZ vk,A tSls & 1896¼vf/ko"kZ½ ds ckn vxyk
(His sister’s husband’s) father’s only vf/ko"kZ 1904 gksxk tks fd 8 o"kZ ds le;kUrjky ls vk;k gS D;ksfa d
son is the husband of my husband’s sister buds chp esa 1900 ,d lk/kkj.k 'krkCnh o"kZ gSA
years Number of leap years
His (brother-in-law’s father’s only son)
First 5 year 1 [4th]
is the husband of (my husband’s sister)
First 10 year 2 [4th, 8th]
First 15 year 3 [4th, 8th, 12th]
His brother-in-law is the husband of my
sister-in-law
First 25 year 6
Gopesh lady First 50 year 12
h The number of leap years will be equal to the question
obtained when we divide the given years by 4. But keep in
7. Calender mind that if the number of leap years is asked between any
two particular years, then we have to count such leap years i.e.
 Theory:- we cannot find the number of leap years by dividing by 4
directly in this situation for e.g.
The record of all the days of the year is given in the calendar. It fn, x, o"kksZa dks 4 ls foHkkftr djus ij tks HkkxQy izkIr gksrk gS
shows the months, weeks and days in the year. mrus gh vf/ko"kZ gksx
a sA ysfdu ;g /;ku j[ksa fd fdUgh nks fo’ks"k o"kksZa
o"kZ ds lHkh fnuksa dk fjdkMZ] dSys.Mj esa fn;k tkrk gSA ;g o"kZ esa ds chp ;fn vf/ko"kksZa dh la[;k iwNh tk, rc gesa ,sls vf/ko"kksZa dh
ekg lIrkg rFkk fnuks dks fn[kkrk gSA fxurh djuh gksxh vFkkZr~ lh/ks rkSj ij 4 ls Hkkx nsdj vf/ko"kksZa dh
Days (fnu) la[;k ugha Kkr dj ldrs gSA tSls &
There are seven days in a week. A week starts with Monday If the number of leap years is asked between 2011 and 2021,
and ends at Sunday. After which again comes Monday. The we have to count such leap years one by one. Here, 2012, 2016
Same cycle goes on and on. and 2020 are three leap years.
,d lIrkg esa 7 fnu gksrs gSA lIrkg lkseokj ls ’kq# gksrk gS rFkk 2011 vkSj 2021 ds chp vf/ko"kksaZ dh la[;k iwNh tk, rc
jfookj ij [kRe gksrk gSA blds ckn fQj ls lkseokj vkrk gSA blh gesa ,d&,d djds vf/ko"kZ Kkr djus gksaxsA ;gk¡ 2012] 2016 vkSj
izdkj ;g pØ pyrk jgrk gSA 2020 rhu vf/ko"kZ gksx
a sA
Century (’krkCnh) :- Years number of leap years
A block of 100 yr is called century. A Century has, thus a total (1) First 100 year (25-1) = 24
of 100 yr. Thus, each one of the years 1100, 1800, 2000, 2100 [100÷4 = but 100th year is a normal year therefore we have to
is a Century. subtract 1.]
100 o"kksZ ds lewgks dks ’krkCnh dgrs gSA ,d ’krkCnh esa bl izdkj (2) First 200 year (50-2) = 48
100 o"kZ gksrs gSA 1100] 1800] 2000] 2100 ;s lHkh o"kZ] ’krkCnh gSA [100 the and 200th years are not leap years. Therefore, we
Ordinary year (lk/kkj.k o"kZ) :- have to subtract 2.]
An ordinary year is a year which has 365 days (52 week + 1 (3) Firest 300 years (75-3) = 72
day). Such year are not divisible by 4. Eg. 2001, 2002, [100th, 200th and 300th years are not leap years Therefore, we
2005, etc. Ordinary years in the form of Century are not have to subtract 3.]
exactly divisible by 400 eg. 100, 200, 500 etc. (4) First years (100-3)=97
,d lk/kkj.k o"kZ] og o"kZ gksrk gS ftlesa 365 fnu ¼52 lIrkg + 1 [100th, 200th and 300th year are not leap years but 400th year is
fnu½ gksrs gSA bl izdkj ds o"kZ 4 ls iwjh rjg foHkkftr ugha gksrs gSA a leap years. Therefore, we heave to subtract 3.]
eg. 2001, 2002, 2005 etc. lk/kkj.k o"kZ tks fd ’krkCnh ds #i esa Concept – 1
gksrs gS 400 ls foHkkftr ugha gksrs gSA eg. 100, 200, 500 etc. Number of leap years (o"kksaZ esa vf/ko"kksaZ dh la[;k )
Leap year (yhi o"kZ):- 1. Find the odd one out –
fuEufyf[kr esa ls fo"ke Nk¡fV, &

43
Download Free PDFs & e-Books from Neon Classes App

(a) 1992 Number of leap years in 400 years = 97


(b) 1900 Thus, 29th February will come 97 times.
(c) 1800 tSlk fd ge tkurs gS fd ,d vf/ko"kZ esa 29 Qjojh vkrh gS vr%
(d) 1882 400 o"kksZa esa ftrus vf/ko"kZ gksaxsA mruh gh ckj 29 Qjojh vk,xhA
Sol: 1900th and 1800th years are normal years. 400 o"kksZa esa vf/ko"kZ ¾ 97
[Because not fully divisibile by 400.] vr% 29 Qjojh 97 ckj vk,xhA
Year 1882 is a normal year.[Because not fully divisible by 4.] 6. How many time does 29th date come in 400 years?
Thus, the years given in the option (B), (C) and (D) are normal 400 o"kksZa esa dqy fdruh ckj 29 rkjh[k vkrh gS\
years while there is a leap year in option (A). (a) 96
1900 vkSj 1800 osa o"kZ lkekU; o"kZ gSA ¼D;ksfa d 400 ls iw.kZr% foHkkftr (b) 97
ugha gSA½ (c) 4800
o"kZ 1882 ,d lkekU; o"kZ gSA ¼D;ksfa d 4 ls iw.kZr% foHkkftr gSA½ (d) 4497
bl izdkj fodDi (b), (c) vkSj (d) esa fn, x, o"kZ lk/kkj.k o"kZ gS Sol: 29th date comes at least 11 times in every year.
yfdu fodYi (a) ,d vf/ko"kZ gSA If any year is a leap, 29th date will come 12 times.
2. How many leap years are there between year 2009 and 2019? 29th date comes in one year = 11 times
o"kZ 2009 vkSj 2019 ds chp vf/ko"kksZa dh la[;k crkb,\ 29th date come in 400 years = 11×400 = 440 times
(a) 2 izR;sd o"kZ esa 29 rkjh[k de ls de 11 ckj vkrh gS vkSj ;fn dksbZ
(b) 3 o"kZ vf/ko"kZ gS rc 29 rkjh[k 12 ckj vkrh gSA
(c) 4 1 o"kZ esa 29 rkjh[k vk,xh & 11 ckj
(d) 1 400 o"ksZa esa 29 rkjh[k vk,xh - 11×400 = 440 ckj
Sol: Leap years between 2009 and 2019 = 2012, 2016 (2 leap years) Add: The additional 29th date in every leap year = 97 times
2009 vkSj 2019 ds chp vf/ko"kZ ¾ 2012]2016 ¼2 vf/ko"kZ½ ∴ 4400+97 = 4497 times
3. How many leap years were there in the first century? Note:- There are 97 leap years in 400 years. Therefore, we
igyh 'krkCnh esa dqy fdrus vf/ko"kZ Fks \ have added 97.
(a) 25 400 o"ksZa esa 97 vf/ko"kZ gksrs gS blfy, 97 tksM+k x;kA
(b) 23 Odd days: The remainder obtained by dividing the number of
(c) 24 days by 7, will be called odd days.
(d) 26 As we know that there are 7 days in week and after 7 days this
Sol: (100÷4)=25 but 100th year is not a leap year. cycle repeats itself. Since odd days are the remainder of 7,
Thus leap years in the first century = 25-1 = 24 therefore odd days cannot be 7 or more than 7.
(100÷4)=25] ysfdu 100ok¡ o"kZ vf/ko"kZ ugha gksrkA Further, odd days will neither be negative nor will be decimal.
izFke 'krkCnh esa vf/ko"kZ = 25-1 = 24 Odd days can be: [0,1,2,3,4,5,6]
4. How many leap years were there in the fourth century? fo"ke fnu@vfrfjDr fnu%& fnuksa dh la[;k 7 ls Hkkx nsus ij tks
pkSFkh 'krkCnh esa dqy fdrus vf/ko"kZ Fks\ 'ks"kQy izkIr gksrk gS mls gh fo"ke fnu dgrs gSA tSlk fd ge tkurs
(a) 24 gS ,d lIrkg esa 7 fnu gksrs gS vkSj 7 7 fdlh ds ckn ;g pdz okil
(b) 25 'kq: gks tkrk gSA pwfa d fo"ke fnu 7 dk 'ks"kQy gksrs gS vr% fo"ke
(c) 97 fnuksa dh la[;k dHkh Hkh 7 ;k 7 ls T;knk ugha gks ldrhA blds lkFk
(d) 72 fo"ke fnu u rks _.kkRed gksaxs vkSj u gh n’keyo esa vk,axsA fo"ke
400 fnu gks ldrs gS
Sol: Number of leap years in the first 400 years =  3  97
4 =[0,1,2,3,4,5,6]
400 15
=  3  97 Fore e.g. – number of odd days in 15 days in 15 days = →1
izFke 400 o"kksZa esa vf/ko"kksaZ dh la[;k 4 7
300 23
Number of leap years in the first 300 years =  3  72 Number of odd days in 23 = →2
4 7
300 Total days Odd days
=  3  72 15 - 1
izFke 300 o"kksZa esa vf/ko"kZ 4
28 - 0
Thus, the number of leap years in the fourth century (301- Months 29 - 1
400) 30 - 2
= 97-72 = 25 31 - 3
vr% pkSFkh 'krkCnh (301-400) esa vf/ko"kksaZ dh la[;k = 97-72 = 25 (Normal years)365 - 1
5. How many time does 29th February come in 400 years? (Leap year) 366 - 2
400 o"kksZa esa dqy fdruh ckj 29 Qjojh vkrh gS\ 50 weeks - 0
(a) 400 Note:- A week is of 7 days. Therefore, when number of weeks
(b) 100 is given, there will be 0 odd days.
(c) 97 25  7
For e.g. – number of odd days in 25 weeks = =0 odd
(d) 96 7
Sol: As we know that 29th February comes in a leap year. Therefore, days
in 400 years, 29 February will come as manytimes as many
leap years are there in 400 years.

44
Best App for Govt. Jobs : Neonclasses (Download Now)

uksV% 1 lIrkg 7 fnu ls curk gS] blfy, tc lIrkgksa dh la[;k nks Therefore, the number of odd days will be subtracted.
gks rc muls 'kwU; ¼0½ fo"ke fnu gksaxsA tSls 25 lIrkgksa esa fo"ke fnu The day on 1st date will be: Sunday – 1 = Saturday
25  7 30 &1 ¾ 29 fnu → 1 fo"ke fnu
¾ ¾ 0 fo"ke fnu dSyM
sa j esa ge ihNs tk jgs gS blfy, fo"ke fnuksa dks ?kVk,axsA
7
Odd days 1 rkjh[k dks fnu ¾ jfookj &1 ¾ 'kfuokj
7. If todas is Tuesday then what day will it be after 37 days? 11. If today is Friday then what day will it be after 60 weeks?
;fn vkt eaxyokj gS rks vkt ls 37 fnu ckn dkSu lk fnu gksxk\ vkt 'kqdzokj gS rks vkt ls 60 lIrkg ckn dkSulk fnu gksxk\
(a) Wednesday (a) Monday
(b) Thursday (b) Tuesday
(c) Tuesday (c) Thursday
(d) Friday (d) Friday
Sol: Number of odd days in 37 days: Remainder (37÷7) = 2 Sol: Number of odd days in 60 weeks = 0
Here we are moving forward in the calendar. ∴The day after 60 weeks will be : Friday+0 = Friday
Therefore the number of odd days will be added. Tuesday +2 = 60 lIrkgksa esa fo"ke fnuksa dh la[;k ¾ 0
Thursday 60 lIrkg ckn fnu gksxk & 'kqdzokj $0 ¾ 'kqdzokj
37 fnuksa esa fo"ke fnu 'ks"kQy (37÷7) = 2 12. If 3 days before yesterday was Sunday then what will be 3
;gk¡ dSyM
sa j esa vkxs vkrk gS blfy, fo"ke fnuksa dh l[;k tksM+h days after tomorrow?
tk,xhA ;fn chrs gq, dy ls 3 fnu igys jfookj Fkk rc vkus okys 3 fnu
eaxyokj $2 ¾ xq:okj ckn dkSulk fnu gksxk\
8. If todas is Tuesday then what was the day before 22 days? (a) Sunday
;fn vkt xq:okj gS rks vkt ls 22 fnu igys dkSu lk fnu gksxk\ (b) Monday
(a) Wednesday (c) Tuesday
(b) Thursday (d) Wednesday
(c) Monday Sol: We take 1 odd day for yesterday; similarly, we will take 1 odd
(d) Tuesday day for tomorrow. But no odd days will be taken for today.
Sol: Number of odd days in 22 days: Remainder (22÷7)=1 Odd days = 1+3+1+3 = 8 →1 odd day
Here in the calendar we are going backwards. Here, we are moving forward in the calendar
Therefore the number of odd days will be subtracted. chrs gq, dy ds fy, 1 fo"ke fnu ysaxs vkSj blh izdkj vkus okys
Thursday -1 = Wenesday. fnu ds fy, Hkh 1 fnu ysx
a sA ysfdu vkt ds fnu dks fy, ugha fy;k
22 fnuksa esa fo"ke fnu 'ks"kQy (22÷7) = 1 tk,xkA
;gk¡ dSyM
sa j esa ge ihNs tk jgs gS blfy, fo"ke fnuksa dh l[;k ?kVk fo"ke fnu = 1+3+1+3 = 8 →1 fo"ke fnu
nh tk,xhA ∴ Sunday+1 = Monday
xq:okj &1¾cq/kokj Yesterday Today Tomorrow
Thus, odd days will be added when moving forward in the
calendar i.e. ‘+’. Sunday Thursday Monday
And odd days will be subtracted when moving backwards in
the calendar i.e. ‘-’. 13. 09 April 2023 is Friday. What will be 30 April 2023?
dSyM
sa j esa vkxs tkus ij fo"ke fnuksa dks tksM+k tk,xk vkSj dSyM
sa j esa 09 vçSy 2023 'kqØokj gSA 30 vçSy 2023 D;k gksxk\
ihNs tkus ds fy, fo"ke fnuksa dks ?kVk;k tk,xkA (a) Monday (b) Saturday
9. Today on 4th August it is Tuesday. What day will it be on the (c) Tuesday (d) Friday
26th of this month? Sol:
vkt 4 vxLr eaxyokj dk fnu gS rc blh ekg dh 26 rkjh[k dks 9 April 2023  'kqdzokj
dkSulk fnu gksxk\ 21
(a) Thursday (+) (30-9)= = R(0) + 0
7
(b) Tuesday
(c) Wednesday 30 April 2023  ? 'kqdo
z kj
(d) Monday Correct answer is option (d)
Sol: 4 August – Tuesday 14. A person says that the 26th day of a month is a Friday. Which of
(26 August -4 August) = 22 days → 1 odd day the following was the 5th day of the same month?
∴ 26 August : Tuesday+1 = Wednesday. ,d O;fä dgrk gS fd eghus dk 26oka fnu 'kqØokj gSA fuEufyf[kr
10. If it is Sunday on the 30th of a month, then what day was it on esa ls dkSu lk ,d gh eghus dk 5oka fnu Fkk\
the 1st of that month? (a) Thursday (b) Tuesday (c) Saturday (d)
fdlh eghus dh 30 rkjh[k dks jfookj gS rks ml eghus dh 1 rkjh[k Friday
dks dkSulk fnu Fkk\ Sol:
(a) Saturday
(b) Friday
(c) Sunday
(d) Monday
Sol: 30-1 = 29 days → 1 odd day
Here, in the calendar we are moving backwards.

45
Download Free PDFs & e-Books from Neon Classes App

fdlh eghus dk 26 okW fnu  'kqdoz kj 2009


Year gap (6) + leap year (2) 2004, 2008 = 6+2=8
21
() (26  5) = = R(0) 0 8
fo"ke fnu odd days = =1 fo"ke fnu@odd days.
7 7
mlh eghus dk 5 okW fnu  \ ¼'kqdzokj½ Correct answr is Concept:
Reference calendar ¼lanHkZ dSyM
sa j½
option (d)
Case-1 When month and days are same but year are different
(i) After ‘n’ days from some day
Note: The 29th February of whichever leap years are used in
Some day + n days
the question, count only those years as leap years. Similarly,
For e.g. (1) After 3 days from 10th → 10 + 3 = 13
the 29th February of whichever leap eyears are not used in the
(2) After 3 days from Sunday → Sunday+3 = Wednesday.
questions, never include those years as leap years.
(ii) nth day from some day
Someday + (n-1) days
uksV% ftl vf/ko"kZ dh 29 Qjojh dk iz;ksx gekjs iz’u esa gksxk dsoy
For e.g. (1) Third day from 10th →10+2 = 12
mlh vf/ko"kZ dks vf/ko"kksaZ dh la[;k esa fxusaxsA blh izdkj fdlh
(2) Third day from Sunday → Sunday + 2 = Tuesday. vf/ko"kZ dh 29 Qjojh dk iz;ksx ge ugha dj jgs gS rc mls vf/ko"kZ
15. If 10 days before yesterday was Monday then what will be 5th ds tSls ugha fxusaxsA
day from today? For e.g. In this question the 29th February off year 2004 has
(a) Monday been used, therefore year 2004 will be counted has leap year.
(b) Tuesday But the 29th February of year 2020 has not been used,
(c) Saturday therefore year 2020 will not be counted as leap year.
(d) Sunday tSls & 20 Qjojh 2004 ls 20 Qjojh 2020 ij tkrs le; geus o"kZ
Sol: Odd days = 1 + 10 + (5-1) 2004 dh 29 Qjojh dk iz;ksx fd;k vr% o"kZ 2004 dks vf/ko"kZ ekusaxsA
= 1 + 10 + 4 = 15 →+1 odd day ysfdu geus 2020 dh 29 Qjojh dk iz;ksx ugha fd;k vr% o"kZ 2020
∴ Monday + 1 = Tuesday dks vf/ko"kZ ugha ekusx
a sA
16. If 15th June falls on 3 days after tomorrow which is Friday then 18. If 1st January 2012 is Tuesday them what Day of the week lies
what day of week will fall on last date of month? on 1st January 2013 ?
;fn 15 twu dy, tks fd 'kqØokj gS] ds 3 fnu ckn iM+rk gS] rks ;fn 1 tuojh 2012 eaxyokj gS] rks 1 tuojh 2013 dks lIrkg dk
lIrkg dk dkSu lk fnu eghus dh vafre rkjh[k dks iM+sxk\ dkSu lk fnu gS\
(a) Sunday (a) Thursday
(b) Monday (b) Monday
(c) Tuesday (c) Tuesday
(d) Thursday (d) Sunday
Sol: Tomorrow = Friday Sol: 2012 is Leap year
3 days after tomorrow = In a leap year last Day = 1st Day + 1 odd day
15th June = Friday + 3 odd days = Monday Tuesday + 1 odd day = 31st December → Wednesday
15 1st January 2013 →
Days from 15th to 30th june = = 1 odd day Wednesday + 1 odd day = Thursday
7
19. If 1st January 2010 is Tuesday, then what day of the week lies
30th June = Monday + 1 odd day = Tuesday
on 1st January 2014 ?
17. If in only month 5th day is after two days from Monday. What
;fn 1 tuojh 2010 dks eaxyokj gS] rks 1 tuojh 2014 dks lIrkg
day will be 18th in the month ?
;fn fdlh eghus esa 5oka fnu lkseokj ls nks fnu ds ckn gSA eghus esa dk dkSu lk fnu gksxk\
(a) Friday
18 oka fnu dkSu lk gksxk\
(b) Saturday
(a) Tuesday
(c) Sunday
(b) Monday
(d) Monday
(c) Sunday
Sol: 1st January 2010 = Tuesday
(d) Friday
→ Extra day →1st January 2011 = + 1
Sol: 3rd day – Monday
1st January 2012 = + 1
5th day – Wednesday
1st January 2013 = + 2
12th day – Wednesday
1st January 2014 = + 1
19th day – Wednesday
Then 18th day will be Tuesday
+5
Find odd days between years (o"kksaZ esa Odd fudkyuk) Tuesday
Ex. How many odd between 2003 to 2009? +5
2003 ls 2009 ds chp fdrus fo"ke fnu gksx
a s\ Sunday
Sol: First we find year gap between given year and than we find So → January 2014 = Sunday
leap year between both given year. 20. In the year 1996 the republic day was celebrated on Friday. On
lcls igys ge nksuksa o"kksZa dk vUrj Kkr djsaxs fQj nksuksa o"kksaZ ds chp which day was the republic day celebrated in the year 2000 ?
esa vkus okys yhi o"kZ Kkr djsx a s vkSj vUr esa nksuksa dks tksM+dj fo"ke o"kZ 1996 esa 'kqØokj dks x.kra= fnol euk;k x;kA o"kZ 2000 esa
fnu fudky ysaxsA x.kra= fnol fdl fnu euk;k x;k Fkk\
2003 (a) Wednesday

46
Best App for Govt. Jobs : Neonclasses (Download Now)

(b) Sunday 25. If 27 March, 2004 was a Monday, then what day of
(c) Monday the week was 1 November, 2004?
(d) Friday ;fn 27 ekpZ] 2004 dks lkseokj gS rks 1 uoEcj] 2004 dks lIrkg
Sol: 26 January 1996 = Friday dk dkSulk fnu gS\
Odd Days → 1996 → 1997 = 2 (a) Sunday
1997 → 1998 = 1 (b) Monday
1998 → 1999 = 1 (c) Tuesday
1999 → 2000 = 1 = 5 (d) Wednesday
26 January 2000 = Friday + 5 = Sol: Calculation of extra days
Wednesday from 27 March, 2004 to 1 Nov. 2004
21. It was Monday on Jan 1, 2006, what was the day of the week 23
Jan 1, 2015? 4+2+3+2+3+3+2+3+1 = or 2 days
7
Tkuojh 1] 2006 dks lkseokj Fkk rks mlh lIrkg ds 1 tuojh] 2015 26. if 15th Aug, 2011 was Monday, what day of the week was on
dks D;k fnu gksxk\ 17th Septmber, 2011?
(a) Monday ;fn 15 vxLr] 2011 dks lkseokj Fkk rks 17 flrEcj] 2011 dks D;k
(b) Tuesday fnu Fkk\
(c) Saturday
(a) Monday
(d) Friday
(b) Tuesday
Sol: calculation of extra day from Jan 2006 to Jan 2015 =
(c) Friday
(1+2+1+1+1+2+1+1+1)
(d) Saturday
11 Sol: calculation of extra days from 15th Aug, 2011 to 17th Sept, 2011
= =4 days, Monday + 4 i.e. Friday
7 for Aug = 16 days i.e. 2 days
22. It was Sunday on Jan 1, 2006. What was the day of the week for Sep = 17 days i.e. 3 days
Jan 1, 2010? Total extra days = 5 days
1 tuojh] 2006 dks jfookj Fkk rks 1 tuojh] 2010 dks dkSulk fnu (Monday + 5 days) = Saturday
gksxk\ 27. If Saturday falls on 14 Aug, 1992, what was the day on 29 Aug,
(a) Sunday 1992?
(b) Friday ;fn 14 vxLr 1992 dks 'kfuokj Fkk rks 29 vxLr] 1992 dks dkSulk
(c) Monday fnu gksxk\
(d) Tuesday (a) Monday
Sol: No. of odd days from the year 2006 to the year 2009 = (1 + 1 + (b) Sunday
2 + 1) = 5 day (c) Tuesday
Then 1st Jan 2010 it is Friday (d) Friday
23. If 28 February 2015 was Saturday, then what was the day of Sol: calculation of extra days
the week on 28 February 2021? From 14 Aug to 29 Aug
;fn 28 Qjojh 2015 dks 'kfuokj Fkk] rks 28 Qjojh 2021 dks Calculation of extra days = (29 – 14)
lIrkg dk dkSu lk fnu Fkk\ 15 days
(a) Thursday (b) Friday i.e. ¾ 1 day
7
(c) Saturday (d) Sunday So, the day (Saturday + 1 day) = Sunday
Sol: 28. If Friday fall on 15th sep 1992, what will be the day of 26 Dec
28 Feb 2015  'kfuokj 1992 ?
+8 +6(year) +7 ;fn 15 flracj 1992 dks ’kqØokj gSA rks 26 fnlEcj 1992 dks
+2 (L.Y.) dkSu&lk fnu gksxk \
(a) Tuesday/eaxyokj
28 Feb 2021  ? 'kfuokj +8 (b) Monday/lkseokj
+1 (c) Thursday/c`gLifrokj
jfookj (d) Wednesday/cq/kokj
Correct answer is option (d) Sol: 15 flrEcj 1992 ¾ 'kqdzokj
24. If it is Sunday on 20th March 2024, then what date was on 20 rks 26 fnlEcj 1992 ¾ \
March 2012? ;gk¡ ij lcls igys oks efguk ysaxs tks efguksa ds vuqlkj igys vkrk
;fn 20 ekpZ 2024 dks jfookj Fkk rks 20 ekpZ 2012 dks dkSulk fnu gS vr% flrEcj o fnlEcj esa ls flrEcj igys rFkk igys efgus esa
Fkk \ cps gq, fnu o vfUre eghus esa ¼fnlEcj½ esa nh gqbZ fnukad fy[ksaxs\
(a) Saturday rFkk ;gk¡ ij o"kZ rks leku gh gS
(b) Sunday So Sept Oct Nov Dec
(c) Monday
↓ ↓ ↓ ↓
(d) Tuesday
Sol: 2024-2012 = 12 30-15
Add: Leap years = 3 (2016, 2020, 2024)
Remaining days 15 31 30 26(Given date)
12+3=15 i.e. -1 odd day

47
Download Free PDFs & e-Books from Neon Classes App

11 12
Odd days → 1+3+2+5 =4 Odd days 0+2+3+2+5 = =5
7 7
efgus esa flrEcj dk okj irk gS fnlEcj dk iqNk x;k gS blfy, Saturday -5 /+2 Monday
fo"ke fnuksa dks tksM+ nsaxsA So correct option (b)
'kqdzokj $ 4 eaxyokj Case – 3 When North, date and years are different :-
29. If Friday fall on 26 January, 1904, what will be the day of 9 In this case solve the questions in 2 parts.
June 1904 ? blesa iz’u dks 2 Hkkxksa esa gy djsaxs &
;fn 26 tuojh 1904 dks ’kqØokj gSA rks 9 twu 1904 dks dkSu lk (1) The number of odd days for years will be calculated according
fnu gksxk \ to case –(1). If we are going forward in the calendar according
(a) Friday/’kqØokj to years, use ‘+’ sign. If we are moving backward in the
(b) Saturday/’kfuokj calendar according to years, use ‘-’ sign.
(c) Sunday/jfookj o"ksZa ds fy, fo"ke fnuksa dh la[;k fLFkfr&¼1½ ds vuqlkj Kkr djsx a sA
(d) Monday/lkseokj ;fn dSyM sa j esa o"kksZa ds vuqlkj vkxs tk jgs gS rc fo"ke fnu ^$^ gksaxs
Sol: 26 Jan 1904 = Friday vkSj ihNs tk jgs gS rks fo"ke fnu ^&^ gksx a sA
9 June 1904 = ? (2) Calculate odd days for days and months according to case –
fnuksa eghuksa esa ls tks igys vkrk gS igys fy[ksaxsA (2). If we are going forward in the calendar according to days
Jan Feb March April May June and months, use ‘+’ sign and if we are moving backwards in the
↓ ↓ ↓ ↓ ↓ ↓ calendar according to days and months, use ‘-‘ sign.
5 29 31 30 31 9 In the end, add both odd days calculated according to case- (1)
Odd days 5 1 3 2 3 2 and class-(2).
Odd days 5+1+3+2+3+2 = 16
16 Note: In case- (3), we will always take 28 days in February
Odd days = =2
7 while calculating odd days for days and months. Because if
50 Friday +2 = Sunday such a year is leap year, then we have included it earlier when
Correct option (c) calculating odd days for years.
30. If Tuesday fall on 19 July 2019, what will be the day of 16 Feb, Now, see the examples given below:
2019 ? 32. If it was Sunday on 15 August 2003 then what day would it be
;fn 19 tqykbZ 2019 dks eaxyokj gSA rks 16 Qjojh 2019 dks on 27 May 2015?
dkSu&lk fnu gksxk \ (a) Friday
(a) Tuesday/eaxyokj (b) Saturday
(b) Wednesday/cq/kokj (c) Thursday
(d) Monday
(c) Thursday/c`gLifrokj
Sol: (1) Number of odd days for years:
(d) Monday/lkseokj
2015-2003 = 12
Sol: 19 July 2019 = Tuesday
Add : Leap years = 3 (2004, 2008, 2012)
16 Feb 2019 = ?
12 + 3 = 15 i.e. 1 odd day
Feb March April May June July
↓ ↓ ↓ ↓ ↓ ↓ In the calendar we are going from 2003 to 2015 i.e. going
12 31 30 31 30 19 forward.
Odd days 5 3 2 3 2 5 Hence, odd days = +1
20 (2) Number days (4) +2 +3 + Given date (15) = 24 i.e. 3 odd
Odd days 5+3+2+3+2+5 = =6 days
7
In the calendar we are going from 15 August to 27 Hence, Odd
Tuesday – 6/+1 Wednesday
days = -3
Note: Here July month days in given and Feb month is asked so
Total odd days = + 1 -3 = -2
odd is negative
27 May 2015: Sunday – 2 = Friday
So correct option (b)
33. If 11th Feb 2005 was a Friday. What was the day the 30th March
31. It was Saturday on 12 December, 1342. What was the day of
2010?
week on 24 August 1342 ?
12 fnlEcj 1342 dks ’kfuokj dk fnu Fkk rks 24 vxLr 1342 dks ;fn 11 Qjojh 2005 dks 'kqØokj FkkA 30 ekpZ 2010 dk fnu D;k Fkk\
(a) Friday
lIrkg dks dkSu&lk fnu gksxk \
(b) Sunday
(a) Tuesday/eaxyokj
(c) Tuesday
(b) Monday/lkseokj
(d) Monday
(c) Sunday/jfookj
Sol: 11 Feb 2005 to 30 March 2010
(d) Friday/’kqØokj Friday + (odd Days of ordinary year + odd days of Leap year
Sol: 12 December 1342 = Saturday from 2005 to 2010) ÷ 7
24 August 1342 = ? Friday + (5 + 1)
Aug. Sept. Oct. Nov. Dec.
Friday + 6 = Thursday
↓ ↓ ↓ ↓ ↓
7 30 31 30 12 Thursday + (17 + 30) =
Odd days 0 2 3 2 5 47 ÷ 7 = 5
Thursday + 5 = Tuesday
34. What will be the day of the week 15th August, 2015?

48
Best App for Govt. Jobs : Neonclasses (Download Now)

15 vxLr] 2015 dks lIrkg dk dkSulk fnu vkrk gS\ ;fn 13 ebZ] 1947 dks 'kfuokj iM+rk gS rks 26 tuojh] 1950 dks
(a) Monday D;k okj gksxk\
(b) Sunday (a) Tuesday
(c) Saturday (b) Monday
(d) Tuesday (c) Thursday
Sol: calculation of extra days till 15th Aug, 2015 (d) Saturday
For first 2000 years = Nil Sol: calculation of extra days from 13 May, 1947 to 26 Jan 1950
14 for May 1947 = 198 i.e. 4 days
For next 2001 to 2014 = = (3×2+11×1)
4  2332323
for June to Dec    4 days
17  7 
= days i.e. 3 days
7 1948 – 1949 = 3
For 2015 = (3+3+2+3+2+3+1) = 3 days 26
26 Jan = 5 days
Total extra days = 6 i.e. Saturday 7
Total extra days = 16 or 2 days
35. The calendar of the year 2006 was used before in the year?
(Saturday + 2 days) = Monday
dyS.Mj o"kZ 2006 blls igys dc leku jgk gksxk\
39. If Monday falls on 12 Aug, 1942, that will be the day 15 Jan
(a) 1990
1947?
(b) 1995
;fn lkseokj] 12 vxLr 1942 dks Fkk] rks 15 tuojh] 1947 dks D;k
(c) 2002
fnu gksxk\
(d) 2000
(a) Monday
Sol: 2006 when divided by 4 it leaves 2 as remainder so, it will be
(b) Tuesday
deducted by 11
(c) Wednesday
i.e. (2006 – 11) = 1995
(d) Sunday
36. On what dates of Feb, 2014 did Monday fall?
Sol: Monday
Qjojh] 2014 esa fdl&fdl fnu lkseokj gksxk\
Concept – To find day with with given date ¼nh xbZ fnukad ls
(a) 3th, 10th, 17th, 24th
okj Kkr djuk½
(b) 2th, 9th, 16th, 23th
Type:-1. To find day with given date (nh x;h fnukad ls okj
(c) 1th, 8th, 15th, 22th
(d) 4th, 11th, 18th, 25th Kkr djuk):-
Sol: Step I firstly we should calculate day on 1 Feb, 2014 Remainder Formula ;-
date + Month Code + Century code + Year + no. of leap year
Calculation of extra days for 1 Feb, 2014 (fnukad) (ekg dksM) (’krkCnh dksM) (o"kZ) (yhi o"kksZ dh la[;k)
For 2000 year = Nil 7 * Month code (ekg
13  3  2  1  10  dksM)
For next 2001 to 2013 =  = 2 days
4  7  Jan feb March April May June July Aug Sept Oct Nov Dec.
For Feb 2014 = 4 days 1 4 4 0 2 5 0 3 6 1 4 6

Total extra days = 6 i.e. Saturday


12² 05² 06² 12²+2
On 1 Feb, 2014 is Saturday? * For leap year, code of Jan and Feb are respectively
3 Feb 2014 is Monday 0 and 3. yhi o"kZ ds fy, tuojh rFkk Qjojh ds dksM Øe’k% 0
Next Monday on 10th, 17th, 24th rFkk 3 gSA
37. If Monday falls on 15 Aug, 1906, then what will be the day on 9 * Centuary code (’krkCnh dksM)
June 1909?
1600 1700 1800 1900 2000 2100 2200 2300
;fn 15 vxLr] 1906 dks lkseokj Fkk] rks 9 twu] 1909 dks D;k fnu
6 4 2 0 6 4 2 0
gksxk\ * for no. of leap year, we divide 4 in the last two digit, and the
(a) Tuesday Quotient is the same.
(b) Monday yhi o"kksZ dh la[;k ds fy,] vafre nks vadks esa 4 dk Hkkx yxkrs gS
(c) Thursday
rFkk HkkxQy gh yhi o"kksZ dh la[;k gksrh gSA
(d) Saturday
* In formula, after divide by 7. According to the remaining
Sol: 15 Aug 1906 = Monday
part, the day will be fix.
9 June 1909 = ?
lw= esa] 7 ls foHkkftr djus ds ckn] ’ks"kQy fnu dks fuf’pr djsxkA
Calculation of extra days
* Remainders :-
for Aug 1906 = 2 days
1234560
for Sep to Dec (2+3+2+3) = 10 days
Sunday Monday Tuesday Wednesday Thrusday Friday
for 1907 & 1908 (1+2) = 3 days
Saturday
 303232 (jfookj) (lkseokj) (eaxyokj) (cq/kokj) (xq#okj) (’kqØokj) (’kfuokj)
for 1909   = 6 days
 7  40. What will the day on 20 may, 2018.
Total extra days = 0 days i.e. Monday 20 ebZ] 2018 dks D;k okj gksxk \
38. If Saturday falls on 13 May, 1947, what will be the day 26 Jan, Sol: ebZ dksM & 2] ’krkCnh (2000) dksM & 6
1950? yhi o"kksZ dh la[;k = = 4

49
Download Free PDFs & e-Books from Neon Classes App

= (Remainder) ’ks"k 1 ¼jfookj½ Total extra = 2 days i.e. Tuesday


41. What will be the day on 26 Jan, 2016 ? 47. What was of the week on 15 Aug, 1949?
26 jan, 2016 dks D;k okj gksxk \ 15 vxLr] 1949 dks lIrkg dk dkSulk fnu Fkk\
Sol: tuojh dksM & 0] ’krkCnh ¼2000½ dksM & 6] (a) Monday
yhi o"kksZ dh la[;k = = 4 (b) Tuesday
(c) Thursday
= = 3 ’ks"kQy ¼eaxyokj½
(d) Saturday
42. What was the day on 20th January 1940?
Sol: No. of extra days for first 1900 year = 1 day
20 tuojh 1940 dk fnu D;k Fkk\
for 1901 to 1948 is
(a) Monday
48 12  2  36  1
(b) Saturday = =  4 days
(c) Sunday 4 7
(d) Tuesday for 15 Aug, 1949
39  9  1  20 69 3323231
Sol:  =6 =  3 days
7
7 7
6 cod → Saturday 8
Total extra days = or 1 day i.e. Monday
43. What was the Day of week on 10th January 2003 ? 7
10 tuojh 2003 dks lIrkg dk fnu D;k Fkk\ Concept – Calendar repetition
(a) Friday It means that after how many years, a year repeats itself in the
(b) Sunday calendar. Any two given years are called equal years if the days
(c) Tuesday falling on all the dates of these two years are the same.
(d) Monday bldk vFkZ gS fd dSyM sa j esa dksbZ o"kZ fdrus o"kksZa ds ckn okil vkrk
02  0  0  10 12 gSA dksbZ nks o"kZ leku dgykrs gS ;fn nksuksa o"kksaZ dh lHkh rkjh[kksa ij
Sol: method → = = 5 ← odd days iM+us okys fnu ,d leku gS
7 7
A normal year and a leap year cannot be equal years.
Code → 5 = Friday
,d lk/kkj.k o"kZ vkSj vf/ko"kZ dHkh Hkh ,d leku o"kZ ughs gks ldrs
44. What was the day of the week on 28th May, 2006?
D;ksfa d vf/ko"kZ esa 29 Qjojh vkus ls vkxs ds lHkh fnu vyx&vyx
28 ebZ] 2006 dks lIrkg dk dkSulk fnu Fkk\
gksx
a sA
(a) Sunday
Because in the leap year, the days coming after 29th February
(b) Saturday
onwards will be different. If the odd days between any two
(c) Friday
years are zero, then such years will be called the equal years.
(d) Thursday
;fn dSyM sa j esa fdUgh nks o"kksZa ds chp fo"ke fnu 'kwU; gks rc ,sls o"kZ
Sol: calculation of extra days
For first 2000 years =Nil
leku o"kZ dgyk,axsA tSls & 1975 vkSj 1986 leku o"kZ gSA
for 2001 to 2005 = 6 days for 2006 = 1 day nksuksa
Total extra days For e.g. 1975 and 1986 are equal years.
nksuksa o"kksZa ds chp fo"ke fnu%
= 7or 0 days i.e.Sunday Odd days between 1975 and 1986: (1986-1975)+Leap years
45. What was the day of the week on 1 Jan, 2019? (1976, 1980, 1984)
1 tuojh] 2019 dks lIrkg dk dkSulk fnu gksxk\ 11+3=14 i.e. O odd day
(a) Monday Trick: (1) If a normal year comes just after a leap year than it
(b) Tuesday will repeat itself after 6 years.
(c) Wednesday For e.g.- The year 2017 comes just after the year 2016 (leap
(d) Sunday year), then it will repeat itself after 6 years.
Sol: calculation of extra days ;fn ,d lk/kkj.k o"kZ] vf/ko"kZ rqjar ckn vkrk gS rc ;g 6 o"kksaZ ds
for first 2000 year = Nil ckn nqckjk vk,xkA
for 2001 to 2018 = 1 day tSls & o"kZ 2017] vf/ko"kZ 2016 ds rqjar ckn vkrk gS vr% 6 o"kksaZ ckn
Jan 2019 = 1 day nqckjk vk,xkA
Total extra = 2days i.e. Tuesday 2017+6=2023 (equal year)
(2) Other normal years repeat themselves after 11 years. For
46. What was the day of the week on 4th June, 2002?
e.g. year 1987 will repeat itself after 11 years because there is
4 twu] 2002 dks lIrkg dk dkSulk fnu Fkk\
no leap year just before 1987.
(a) Tuesday vU; lk/kkj.k o"kZ dSyMsa j esa 11 o"kksZa ds ckn nqckjk vk,axsA tSls & o"kZ
(b) Monday
1987] 11 o"kksZa ckn nqckjk vk,xkA D;ksfa d 1987 ls rqjar igys ,d
(c) Friday
vf/ko"kZ ugha gSA
(d) Saturday
1987+11=1998 (equal year)
Sol: Calculation of extra days
(3) A leap year repeats itself after 28 years For eg.- year 2000
for first 2000 year = 0 day
is a leap year and it will repeat itself after 28 years.
for 2001 = 1 day
,d vf/ko"kZ dSysaMj esa 28 o"kksZa ckn nqckjk vkrk gSA tSls & o"kZ 2000
 3323 4 
for 2002 =    1 day ,d vf/ko"kZ gS vr% 28 o"kksaZ ckn nqckjk ck,xkA
 7  2000+28 = 2028 (equal year)

50
Best App for Govt. Jobs : Neonclasses (Download Now)

These three tricks can be used only if we are in the same Option (d): odd days between 2019 and 2030 = 11+3=14 i.e. 0
century or cross such a century which is a leap year. odd day
bu rhuksa fVªDl dk iz;ksx rHkh fd;k tk ldrk gS tc ge mlh Trick: Equal year of year 2019:
'krkCnh esa jgs vFkok ,slh 'krkCnh dks ikj djs tks 'krkCnh vf/ko"kZ gksA o"kZ 2019 dk leku o"kZ%
For e.g.-The equal year of 1990:1990+11-2001 2019+11 = 2030
(While going from 1990 to 2001, we have crossed the century 50. The calendar of the year 2005 will be equal to which of the
year 2000, which is a leap year. Therefore, we can use the following years?
trick.) o"kZ 2005 dk dSyM
sa j fuEu esa ls dkSuls o"kZ ds leku gksxk\
tSls& ¼1½ 1990 dk dSysaMj leku gksxk & 1990$11¾2001 (a) 2016
¼;gk¡ 1990 ls 2001 ij tkrs le; 'krkCnh o"kZ 2000 dks ikj fd;k gS (b) 2011
tks ,d vf/ko"kZ gSA vr% ;gk¡ fVªd dk iz;ksx dj ldrs gS½ (c) 2010
The equal year of 1890:1890+11-1901 (not equal year) (While (d) 2008
going from 1890 to 1901, we have crossed the century year Sol: year 2005 comes just after the leap 2004.
1900, which is not a leap year. Therefore, we cannot use the o"kZ 2005] vf/ko"kZ 2004 ds rqjar ckn vkrk gSA
trick.) Hence the equal year = 2005+6 = 2011
1890 dk dSyM sa j &1890$11¾1901 ds leku ugha gksxkA ¼D;ksfa d 1890 51. The calendar of the year 2017 will be equal to which of the
ls 1901 ij tkrs le; 'krkCnh o"kZ 1900 dks ikj fd;k gS tks fd ,d following year?
vf/ko"kZ ugha gSA vr% ;gk¡ fVªd dk iz;ksx ugha dj ldrsA½ o"kZ 2017 fuEu esa ls dkSuls o"kZ ds leku gksxk\
When we cannot use the tricks to find the equal years, we will (a) 2028
calculate odd days between two years to know whether they (b) 2024
are equal years or not. (c) 2023
tc leku o"kZ Kkr djus ds fy, fVªd dk iz;ksx ughs dj ldrs rc (d) 2025
2 o"kksZa ds chp fo"ke fnu Kkr djds ;g ekywe djsaxs fd nksuksa o"kZ Sol: Year 2017 comes just after the leap year 2016.
leku gS vFkok ughaA o"kZ 2017] vf/ko"kZ 2016 ds rqjar ckn vkrk gSA
48. The calendar of the year 1990 will be equal to which of the Hence the equal year = 2017+6=2023
following year? 52. The calendar of the year 1897 will be equal to which of the
o"kZ 1990 dk dSyM
sa j fuEu esa dkSu o"kZ leku gksxk\ following year?
(a) 1997 o"kZ 1897 dk dSyM
sa j fuEu esa ls dkSuls o"kZ ds leku gksxk\
(b) 1996 (a) 1908
(c) 2000 (b) 1903
(d) 2001 (c) 1904
Sol: A normal year and a leap year can not be equal years. (d) 1909
Therefore, option (b) and (c) are eliminated. Sol: here, we have crossed the century year 1900 which is not a
Option (a): odd days between 1990 and 1997-1998-1990 = 7 leap year. Therefore, we cannot use the trick to find equal year.
Add leap years = 2 ;gk¡ ge 'krkCnh o"kZ 1900 dks ikj dj jgs gSA tks fd ,d vf/ko"kZ ugh
∴ 7 + 2 = 9 i.e. 2 odd years gSA vr% leku o"kZ Kkr djus ds fy, fVªd dk iz;ksx ugha dj ldrsA
∴ Option (a) is eliminated Option (d): odd days between 1990 The calendar of a normal year is not equal to a leap year.
and 2001. 2001-1990 = 11 Hence, options (a) and (c) are eliminated.
Add: leap years = 3 lk/kkj.k o"kZ dk dSyMsa j vf/ko"kZ ds leku ugha gksrk vr% fodYi (a)
∴ 11 + 3 = 14 i.e. 0 odd day vkSj (c) gekjk mÙkj ugha gks ldrsA
Hence, the equal year of 1990 will be 2001 Option (b): Odd days between 1897 and 1903 = 6+0=6
Trick: Here in the question we have crossed the century year Hence, option (b) is eliminated.
2000 which is a leap year. Therefore we can use the trick to Option (d): odd days between 1897 and 1909.
find equal years. = 12 + 2 = 14 i.e. 0 odd day
;gk¡ iz’u esa ge 'krkCnh o"kZ 2000 dks ikj dj jgs gSA tks fd yhi Hence the calendar of year 1897 will be equal to the year
¼vf/kd½ o"kZ gS vr% fVªd ls Hkh leku o"kZ Kkr dj ldrs gSA 1909.
1990 + 11 = 2001 53. The calendar of the year 2016 will be equal to which of the
49. The calendar of the year 2019 will be equal to which of the following years?
following years? o"kZ 2016 dk dSyM
sa j fuEu esa ls dkSuls o"kZ ds leku gksxk\
o"kZ 2019 dk dSyM
sa j fuEu esa ls dkSuls o"kZ ds leku gksxk\ (a) 2044
(a) 2024 (b) 2024
(b) 2025 (c) 2032
(c) 2028 (d) 2035
(d) 2030 Sol: year 2016 is a leap year. Therefore, it will repeat itself after 28
Sol: Years 2024 and 2028 are leap years. Therefore options (a) and years.
(c) are eliminated. o"kZ 2016 ,d vf/ko"kZ gS vr% og 28 o"kksZ ckn nqckjk vk,xkA
Option (b): odd years between 2019 and 2025 =6 + 2 = 8 i.e. 1 ∴ 2016+28 = 20244
odd day Other Some important facts:
Hence, option (b) is eliminated. (1) In a normal year there is 1 odd day i.e. when there is a
difference of 1 year in the calendar, we move forward by 1 day.
For e.g.-

51
Download Free PDFs & e-Books from Neon Classes App

If 1 January 2001 was a Monday, then what day would it be on Hence, we can say that the first 2 day of a leap year come 53
1 January 2002? times in a year and all other days will come 52 times
,d lk/kkj.k o"kZ esa 1 fo"ke fnu gksrk gSA vFkkZr~ tc dSyMsa j esa 1 vr% ge dg ldrs gS fd ,d vf/ko"kZ ds izFke 2 fnu o"kZ esa 53 ckj
lky dk varj vkrk gS rks ge 1 fnu vkxs c<+rs gSA tSls& 1 tuojh vkrs gS vkSj vU; lHkh fnu 52 ckj ck,axsA
2001 dks lkseokj Fkk rks 1 tuojh 2002 dks dkSulk fnu gksxk\ (5) There can be 28/29/30/31 days in one month i.e. there will be
Odd days = 1+0=1 at least 28 days in one month. Hence, each day will come
∴ 1 January 2002 = Monday+1=Tuesday Apart from this, it will atleast 4 times in one month. Apart from this, which day will
be (Tuesday-1) =Monday on 31 December 2001. come 5 times in a month, we will know in the following way:
1 January 2001 : Monday ,d efgus esa 28@29@30@31 fnu gks ldrs gS vFkkZr~ ,d eghus esa
31 December 2001 : Monday de ls de 28 fnu vo’; gksaxsA vr% ,d ekg esa dksbZ Hkh fnu 4 ckj
Hence, we can say that the first and last days of a normal year fu’pr :i ls vk,xkA blds vfrfjDr dkSulk fnu 5 ckj vk,xk ;g
are the same. fuEu rjg ls Kkr djsx
a s&
vr% ge dg ldrs gS fd fdlh lk/kkj.k o"kZ dk izFke vkSj vkf[kjh Total days Days Coming 5 times
fnu ,d leku gksrs gSA in a month in a month
(2) A leap year consists of 2 odd days i.e. when there is a (1) 28 0 (No day will come 5 times.)
difference of 1 year in the calendar, we move forward by 2 (2) 29 1 (The first day of month will
days. For e.g. come 5 times)
1 January 2004 was a Monday, then what day would it be on 1 (3) 30 2 (The first two days of
January 2005? month will come 5 times)
,d vf/ko"kZ esa 2 fo"ke fnu gksrs gs vFkkZr~ tc dSysaMj esa 1 lky dk (4) 31 3 (The first three days of
varj vkrk gS rks ge 2 fnu vkxs c<+rsA tSls& ;fn 1 tuojh 2004 month will come 5 times)
dks lkseokj Fkk rks 1 tuojh 2005 dks dkSulk fnu gksxkA 54. How many times will Wednesday come in the year 2020?
Odd days = 1+1=2 o"kZ 2020 esa dqy fdruh ckj cq/kokj vk,xk\
∴ 1 January 2005 = Monday +2= Wedensday Also, it will be (a) 52 times
(Wednesday-1) = Tuesday on 31 December 2004. (c) 54 times
1 January 2004: Monday (b) 53 times
31 December 2004: Tuesday (d) 51 times
Thus, the last day of a leap year is the next day of the first day Sol: The first 2 days of a leap year come 53 times in a year. By
of the year. Comparing the todays date the reference calendar, or by using
bl izdkj fdlh vf/ko"kZ dk vkf[kjh fnu] izFke fnu dk vxyk fnu the standard formula we can find the day of Junary 1, 2020.
gksrk gSA fdlh vf/ko"kZ ds igys 2 fnu o"kZ esa 53 ckj vkrs gSA lanHkZ dSysaMj ds
(3) A normal year has 52 weeks and 1 day. vuqlkj vkt dh rkjh[k ls rqyuk djds 1 tuojh 2020 dk fnu Kkr
365 days = 52 weeks +the first day of 53rd week. djsxa s vFkok ekud lw= ls Hkh 1 tuojh 2020 dk fnu Kkr dj ldrs
(1 January-30 December) (31 December) gSA
Each day comes once in a week. Therefore in 52 weeks, 1 January 2020: Wednesday
Each day will come 52 times. Hence, we can say that each day 1 tuojh 2020 % cq/kokj
will definitely come 52 times in a year. The last day of a normal ∴Wednesday will come 53 times in year 2020.
year is the same as the first day and the day of 31st December ∴ cq/kokj o"kZ esa 53 ckj vk,xkA
will come 53 times in 1 year. 55. How many times will Sunday come in August 2020?
1 lIrkg esa izR;sd fnu 1 ckj vkrk gWsA vr% 52 lIrkg esa izR;sd fnu vxLr 1010 esa jfookj dqy fdruh ckj vk,xk\
52 ckj vk,xkA rc ge dg ldrs gS fd ,d lk/kkj.k o"kZ esa izR;sd (a) 4 times
fnu 52 ckj vo’; vk,xkA (b) 5 times
Hence, we can say that the first day or last day of a normal (c) 6 times
year comes 53 times in 1 year and all other days will come 52 (d) 3 times
times in 1 year. Sol: There are 31 days in August. Hence the first 3 days of month
vr% ge dg ldrs gS fd ,d lk/kkj.k o"kZ dk izFke fnu o"kZ esa 53 will come 5 times in a month.
ckj vkrk gS vkSj vU; lHkh fnu 52 ckj vk,axsA By comparing the today’s date in the reference calendar or by
(4) A leap year has 52 weeks and 2 days using the standard formula we can find the day of 1 August
,d vf/ko"kZ esa 52 lIrkg vkSj 2 fnu gksrs gSA 2020.
366 days = 52 weeks + The first two days of 53rd week vxLr esa 31 fnu gksrs gS vr% eghus ds igys 3 fnu 5 ckj vk,axsA
(1 January-29 December) (30 and 31 December) ge lanHkZ dSyM
sa j ds vuqlkj vkt ds fnu ls rqyuk djds ;k ekud
Here also each day will come 52 times in a leap year and the lw= ls Kkr djsaxs fd 1 vxLr 2020 dks dkSulk fnu FkkA
days of 30th December and 31st December will come 53 times 1 August 2020: Saturady
in 1 year. 2 August 2020: Sunday
;gk¡ Hkh ,d vf/ko"kZ esa izR;sd fnu 52 ckj vko’; vk,xk vkSj 30 Hence, Sunday will come 5 times in August 2020.
fnlacj o 31 fnlacj okys fnu ,d o"kZ esa 53 ckj ck,axsA Calculation of odd days for different centuries:

The last 2 days in a leap year are the same as the first 2 days.
,d vf/ko"kZ esa vkf[kjh fnu vkSj izFke nks fnuksa ds leku gksrs gSA

52
Best App for Govt. Jobs : Neonclasses (Download Now)

fofHkUu 'krkfCn;ksa ds fy, fo"ke fnuksa dh x.kuk Years Odd Last day century First day
days of century of century
Years Leap Odd days First 100 years 5 Friday First Monday
year’s First 200 years 3 Wednesday Second Saturday
First 100 years 24 (100-24) ÷7=5 odd days First 300 years 1 Monday Third Thursday
First 200 years 48 (200-48) ÷7=3 odd days First 400 years 0 Sunday Fourth Tuesday
First 300 years 72 (300-72) ÷7=1 odd day The first day of a century is the next day of the last day of the
First 400 years 97 (400-97) ÷7=0 odd day previous century.
The first day of the The last day of first century was Friday. Hence the first day of
calendar was 1 January, 1 and this day was Monday Similarly the second century will be (Friday +1) = Saturday
the second day of the calendar was Tuesday, the third day was The last day of the second century was Wednesday. Hence the
Wednesday, the fourth day was Thursday and so on. first day of the third century will be (Wednesday+1) =
dSyM
sa j dk izFke fnu 1 tuojh Fkk vkSj bl fnu lkseokj FkkA blh Thursday.
rjg dSyM sa j dk nwljk fnu eaxyokj Fkk] rhljk fnu cq/kokj] pkSFkk Similarly we can figure out the first day of other centuries.
fnu xq:okj vkSj vkxs Hkh ;gh dze tkjh jgsxkA As the entire calendar repeats itself after every 400 years.
Monday - 1 (First day of calendar) Hence, the fifth century will be equal to the first century, the
Tuesday – 2 (Second day) sixth century will be equal to the second century, the seventh
Wednesday – 3 (Third day)
century will be equal to the third century and so on.
Thursday – 4 (Fourth day)
Friday – 5 (Fifth day) As the entire calendar repeats itself after every 400 years.
Saturday - 6 (Sixth day) Hence, the fifth century will be equal to the first century, the
Sunday – 7 (Seventh day) The eighth day of sixth century will be equal to the second century, the seventh
the calendar will be Monday and thus these 7 days of week will century will be equal to the third century and so on.
repeat themselves in a cycle. For e.g. fdlh 'krkCnh dk tks vkf[kjh fnu gksrk gS mldk vxyk fnu nwljh
(1) Which was the 22nd day of the calendar? 'krkCnh dk igyk fnu gksrk gSA tSls&
dSyMsa j dk vkBok¡ fnu iqu% lkseokj gksxk vkSj bl izdkj ;s 7 fnu igyh 'krkCnh dk vkf[kjh fnu 'kqdzokj Fkk rks nwljh 'krkCnh dk
iqu% okil vk,axsA igyk fnu 'kqdzokj $1 ¾ 'kfuokj gksxkA
22 nwljh 'krkCnh dk vkf[kjh fnu cq/kokj Fkk rks rhljh 'krkCnh dk
= -1 odd day igyk fnu cq/kokj $1 ¾ xq:okj gksxkA
7
(Since Monday 11) blh rjg vU; 'krkfCn;ksa ds igys fnu Kkr djsaxsA
Hence, Monday will be the 22nd day tSlk fd dSyM sa j 400 o"kksZa ds ckn iqu% nksgjk;k tkrk gS vr% ik¡poha
blfy,22ok¡ fnu lkseokj gksxkA 'krkCnh] igyh 'krkCnh ds leku gksxh] NBh 'krkCnh nwljh 'krkCnh ds
(2) Which was the 100th day of the calendar? leku gksxh] lkooha 'krkCnh rhljh 'krkCnh ds leku gksxh vkSj blh
100 izdkj ;g dze pyrk jgsxkA
= -2 odd days i.e. Tuesday The last days of centuries:
7
(3) Which was the 91st day of the calendar? The last days of centuries can be- Friday, Wednesday, Monday
and Sunday
91
= -0 odd day i.e. Sunday Hence Tuesday, Thursday and Saturday will never be the last
7 days of any century. 'krkCnh ds vkf[kjh fnu% fdlh 'krkCnh ds
Here, we directly calculate the number of odd days Therefore,
vkf[kjh fnu gks ldrs gS &
we have not used ‘+’ or ‘-’sign.
'kqdzokj] 'kfuokj] lkseokj] jfookj
While in reference calendar, we calculate the day on a
The first days of centuries:
particular date with respect to some other date Therefore, we
The first days of centuries can be- Monday
use '+' or ‘-’ sign in the calculation of, odd days.
Saturday, Thursday and Tuesday
For e.g. If today is Monday, then what day will it be after 22
Hence Wednesday, Friday and Sunday will never be the first
days?
days of any century
;gk¡ ge lh/ks rkSj ij fo"ke fnuksa dh x.kuk djrs gS blfy, ^$^ ;k
'krkCnh ds izFke fnu% fdlh 'krkCnh ds izFke fnu gks ldrs gS&
^&^dk fpUg ugha yxkrsA tcfd lanHkZ dSyM sa j esa fdlh fnu dh rqyuk
lkseokj]] 'kfuokj] xq:okj] eaxyokj
esa nwljk fnu Kkr djrs gS rc ogk¡ fo"ke fnuksa dh x.kuk esa ^$^ ;k
vr% cq/kokj] 'kqdzokj vkSj jfookj dHkh Hkh 'krkCnh ds izFke fnu ugha
^&^ dk fpUg yxkrs gS
gksx
a sA
22 56. Which of the following days can never be the last day of any
= +1 odd day
7 century?
Here, we have to calculate the day after 22 days with respect buesa ls dkSulk fnu fdlh 'krkCnh dk vafre fnu ugha gks ldrk gS
to today. (a) Sunday
tSls& vkt lkseokj gS rks vkt ds 22 fnu ckn dkSu lk fnu gksxk \ (b) Monday
Monday+1 = Tuesday (c) Tuesday
The first and the last days of a century: (d) Wednesday
The entire calendar has been formed using 400 years i.e. the Sol: Tuesday, Tursday and Saturday can never be the last days of
whole calendar repeats itself after every 400 years. (Because any century.
there is. 0 Odd days in 400 years.) eaxyokj] xq:okj vkSj 'kfuokj dHkh Hkh 'krkCnh ds vafre fnu ugha gks
iwjk dSyM sa j 400 o"kksZa ls cuk;k x;k gS vFkkZr~ izR;sd 400 o"kksZa ds ckn ldrsA
dSyM sa j fQj ls nksgjk;k tkrk gSA ¼D;ksfa d 400 o"kksaZ esa 0 fo"ke fnu 57. Which of the following days can never be the first day of any
gksrs gS½ century?

53
Download Free PDFs & e-Books from Neon Classes App

buesa ls dkSulk fnu fdlh 'krkCnh dk izFke fnu ugha gks ldrk gS ckj dksM
(a) Sunday x + 300 + 350 = 2
(b) Monday x + 650 = 2
(c) Tuesday 650
(d) Saturda 650 fnuksa esa 'ks"kQy = =6
7
Sol: Wednesday, Friday and Sunday can never be the First days of
x+6=2
any century.
x=2–6
jfookj] cq/kokj vkSj 'kqdzokj dHkh Hkh 'krkCnh ds izFke fnu ugha gks
x=-4
ldrsA - 4 means (7 – 4) = 3
Other Important questions: - x = 3 → (Wednesday)
58. Suresh was born on 4th Oct 1999 Shashikant was born 6 days 61. If John celebrated his victory day on Tuesday, 5th January
before Suresh. The year independence day was a Sunday on 1965. When will he celebrated his next victory day on the
which day was Shashikant born ? same day ?
lqj's k dk tUe 4 vDVwcj 1999 dks gqvk Fkk 'kf'kdkar dk tUe lqjs'k ;fn t‚u eaxyokj] 5 tuojh 1965 dks viuk fot; fnol eukrk gSA
ls 6 fnu igys gqvk FkkA bl o"kZ Lora=rk fnol jfookj dks Fkk rks og mlh fnu viuk vxyk fot; fnol dc euk,xk\
crkb;s 'kf'kdkar dk tUe dc gqvk Fkk\ (a) 1973
(a) Monday (b) 1970
(b) Sunday (c) 1969
(c) Thursday (d) 1971
(d) Tuesday Sol: d)
Sol: Birthday of Suresh = 4th October 1994 1995 + 6 = 5th January 1971
So birthday of Shashikant = 28th September
No. of days between 1
44 So according to chart we add 6 year for next celebration on
15 August to 28 septe. = 16 + 28 =
7 same day
No. of odd days = 2 blfy, pkVZ ds vuqlkj ge mlh fnu vxys mRlo ds fy, 6 o"kZ
Required day = Sunday + 2 tksM+rs gSa
→ Tuesday Calculation of time b/w two dates:
59. Rakesh remembers that his father birthday is before 27th oct. 62. Ankit was born on 4 September 1996. Then how old will him
But after 23th Oct where as his sister remembers that her on 10th August 2020?
father birthday is before 29th Oct but after 24th Oct which Day vafdr dk tUe 4 flracj 1996 dks gqvkA rcog 10 vxLr 2020 dks
is his father birthday on which both agree? fdrus lky] ekg] vkSj fnu dk gksxk\
jkds'k dks ;kn gS fd muds firk dk tUefnu 27 vDVwcj ls igys gSA (a) 23 years 11 month 4 days
ysfdu 23 vDVwcj ds ckn tgka mudh cgu dks ;kn gS fd muds firk (b) 23 years 10 month 2 days
dk tUefnu 29 vDVwcj ls igys gS] ysfdu 24 vDVwcj ds ckn muds (c) 23 years 11 month 6 days
firk dk tUefnu dkSuls fnu gS] ftl ij nksuksa lger gSa\ (d) 24 years 10 month 4 days
(a) 26 Sol: Year - Month - day
(b) 25 2020 - 8 - 10
(c) 25, 26 1996 - 9 -4
(d) 24, 26 23 years 11 months 6days
Sol: According to Rakesh his father birthday on May be – 24 Oct, 25 Note: (1) 9 months cannot be subtracted from 8 months.
Oct, 26 Oct According to his Sister his father birthday on may Therefore, we will carry 1 year i.e. 12 months from the column
be 25 Oct 26 Oct 27 Oct and 28 Oct common Dates are 25, 26 of years. Now after subtracting 9 month from (8+12)=20
jkds'k ds vuqlkj mlds firk dk tUefnu gks ldrk gS & 24 vDVwcj] months, we will get 11 months.
25 vDVwcj] 26 vDVwcjA mldh cgu ds vuqlkj mlds firk dk 8 ekg esa ls 9 ?kVk,a ugh tk ldrs vr% o"kZ esa ls 1 o"kZ ¼12 ekg½
tUefnu 25 vDVwcj 26 vDVwcj 27 vDVwcj vkSj 28 vDVwcj rd gSA ysx
a sA rc ¼8$18½¾20 ekg esa ls 9 ?kVkus ij 11 ekg vk,xkA
blfy, vke frfFk 25 vkSj 26 gSA (B) On adding 12 months in the column of months now we
60. Anu is 300 days older them varun and sandeep is 50 weeks have to subtract 1 year from 2020 i.e. (2020 - 1)=2019.
older than anu. If sandeep was born on Tuesday. On which day After subtracting 1996 from 2019, we will get 23 years.
was varun born ? ekg esa 12 eghus tksM+us ij vc gesa 2020 o"kZ esa ls 1 o"kZ ?kVkuk
vuq , o#.k ls 300 fnuksa ls cM+h gSa] vkSj lanhi, vuq ls 50 lIrkg cM+s iM+x
s k vFkkZr~ ¼2020&9½¾2019
gSaA vxj eaxyokj dks lanhi dk tUe gqvk rks o#.k dk tUe fdl 2019 esa ls 1996 ?kVkus ij 23 o"kZ izkIr gq,A
fnu gqvk Fkk\ 63. Riya was born on 10 October 2000. Then how old will she be
(a) Wednesday on 5 July 2020?
(b) Monday fj;k dk tUe 10 vDVwcj 2000 dks gqvkA rc og 5 tqykbZ 2020 dks
(c) Tuesday fdrus lky] ekg] vkSj fnu dk gksxh\
(d) Sunday (a) 19 years 6 month 26 days
Sol: Varun Anu Sandeep (b) 19 years 8 month 26 days
x+ 300 + 50 weeks (c) 20 years 2 month 24 days
[7 × 50] → 350 = Tuesday (d) 20 years 8 month 26 days

54
Best App for Govt. Jobs : Neonclasses (Download Now)

Sol: Year - Month - day prescribed medicine is abundantly available in one of the cities
2020 - 7 -5 near ' Spatter' which can cure the disease, the Medical
2000 - 10 - 10 Association of the State has decided not to allow the medicine
23 years 8 months 26 days to be transported to 'Spatter' and is calling the infected people
Note: 10 days connot be subtracted from 5 days. Therefore, to the city to be injected with the medicine.
we have to carry 1 month i.e. 31 days in the column of months. X ' ' ' ' ' ' Which of the following can be a reason
Since there are 31 day in July that’s why we have taken 31 behind the decision of the Medical Association?
days. Thus the number of day taken forward will be equal to (a) The Association may not be able to earn high profits from
the days of that month which is being carried. selling the medicine to 'Spatter'.
5 fnu esa ls 10 fnu ugha ?kVk, tk ldrs vr% 1 ekg vFkkZr~ 31 fnu (b) Many of the villagers may not be able to afford the medicine as
fy,A D;ksfa d tqykbZ esa 31 gksrs gSA vr% 31 fnu fy, tk,axsA bl it is highly priced.
izdkj tks eghuk m/kkj ys jgs gS og ftrus fnu dk gksxk mrus gh fnu (c) The disease has started to spread in other villages also, making
ge fnuksa dh la[;k esa tksM+saxsA it challenging for the Association to provide the medicine
Hence (5+31)-4=26 everywhere.
(2) 12 months will be taken forward when 1 year is being (d) Reportedly, many of the villagers are trying home remedies to
carried from the column of years. reduce the effect of the disease and hence they may not
1 o"kZ dks ekg esa m/kkj ysus ij 12 ekg fy, tk,axsA require the medicine.
Hence (7-1+12)-10=8 months (e) If the medicine is not injected under the prescribed controlled
environments, it may have adverse effects.
Sol. (e)
8. Cause and Effect Obviously, option (e) is the reason behind the decision of the
Medical Association of the State 'X'.
1. The condition of the roads in the city has deteriorated e "X" 4. Statement / I. Many people visits the
considerably during the first two months of monsoon and religious places on week days and weekends to pray to
most of the roads have developed big pot holes. Mother Durga in Navratras.
Which of the following can be a possible effect of the II. Many religious people go on fasting during Navratras to
above cause. seek the blessings of Mother Durga.
1. The municipal corporation had repaired all the roads in the (a) If statement I is the cause and statement II is its effect:
city before onset of monsoon with good quality material. I II
2. A large number of people have developed spine related (b) If statement II is the cause and statement I is its effect:
injuries after regularly commuting long distances by road II I
within the city. (c) If both the statement I and II are independent causes;
3. The municipal corporation has been careful in choosing the I II
contractors for repairing roads in the past. (d) If both the statement I and II are effects of independent causes
4. People always complain about potholes roads during the I II (e) If both the statement I and II are effects of some
monsoon months. common cause.
5. None of these./ I II Sol. (d)
Sol. (b) ‘Blessing of Mother Durga’ is a common cause between two
Option (2) may be a possible effect of big pot holes developed statements. Hence, Both the statement are effects of some
on the roads. common cause.
2. There has been a spate of rail accidents in India in the recent 5. I. All India Truck owners association went on a nation
months killing large numbers of passengers and injuring wide strike due to hike in cost of diesel.
many more. This has raised serious doubts about the II. Government has withdrawn its proposal to levy additional
railway's capability of providing safety to travelers. taxes on diesel.
Which of the following statements substantiates the views (a) If statement I is the cause and statement II is its effect:
expressed in the above statement? I II
(a) Indian Railways has come to be known to provide best (b) If statement II is the cause and statement I is its effect:
passenger comfort in the recent years. II I
(b) People have no option other than travelling by rail over long (c) If both the statement I and II are independent causes;
distances. I II
(c) The railway tracks at many places have been found to be (d) If both the statement I and II are effects of independent causes
stressed due to wear and tear in the recent times. I II
(d) Local residents are always the first to provide a helping hand (e) If both the statement I and II are effects of some common
to the passengers in the event of such disasters. cause.
(e) None of these./ I II
Sol. (c) Sol. (a)
Option (3) substantiates the views expressed in the statement. Statement I is a cause and statement II is its effect, as in view
3. Study the following information carefully to answer the of strike of truck owners (possibly) Government has with
question given below: drawn proposal to levy additional taxes on diesel.
A fatal disease has spread across the 'Spatter' village of State X. I II 6. I. The price of aircraft fuel has risen during the past
Even through the prescribed medicine is though the few months.

55
Download Free PDFs & e-Books from Neon Classes App

II. Many passenger airlines in India have been forced to cut (a) If statement I is the cause and statement II is its effect:
their air fares by about 10%. I II
10% (b) If statement II is the cause and statement I is its effect:
(a) If statement I is the cause and statement II is its effect: II I
I II (c) If both the statement I and II are independent causes;
(b) If statement II is the cause and statement I is its effect: I II
II I (d) If both the statement I and II are effects of independent causes;
(c) If both the statement I and II are independent causes; and
I II I II
(d) If both the statement I and II are effects of independent causes; (e) If both the statement I and II are effects of some common
and I II cause.
(e) If both the statement I and II are effects of some common I II
cause. Sol. (a)
I II Since, the staff members have gone on strike, the help of
Sol. (d) outsiders has been sought.
I is usually the effect of hike in global petroleum prices. II is
usually the effect of competition in aviation. 10. I. The private medical colleges have increased the tuition fees
I II in the current year by 200% over the last year’s fee to meet the
7. I. There is sharp decline in the production of oil seeds this expenses.
year. 200%
II. The Government has decided to increase the import II. The Government medical colleges have not increased their
quantity of edible oil. fees inspite of price evaluation.
(a) If statement I is the cause and statement II is its effect: (a) If statement I is the cause and statement II is its effect:
I II I II
(b) If statement II is the cause and statement I is its effect: (b) If statement II is the cause and statement I is its effect:
II I II I
(c) If both the statement I and II are independent causes; I II (c) If both the statement I and II are independent causes;
(d) If both the statement I and II are effects of independent causes; I II
and I II (d) If both the statement I and II are effects of independent causes;
(e) If both the statement I and II are effects of some common and
cause. I II
I II (e) If both the statement I and II are effects of some common
Sol. (a) cause.
A sharp decline in oil seed production is bound to reduce oil I II
supply and import of oil is the only means to restore the Sol. (c)
essential supply. The increase in the fees of the private colleges and there being
no increase in the same in Government colleges seems to be
8. I. The Government has decided to increase the prices of LPG policy matters undertaken by the individual decisive boards at
cylinders with immediate effect. the two levels.
II. The Government has decided to increase the prices of 11. I. The Government has suspended several police officers in the
Kerosine with immediate effect. city.
(a) If statement I is the cause and statement II is its effect: II. Five persons carrying huge quantity of illicit liquor were
I II apprehended by police.
(b) If statement II is the cause and statement I is its effect: (a) If statement I is the cause and statement II is its effect:
II I I II
(c) If both the statement I and II are independent causes; (b) If statement II is the cause and statement I is its effect:
I II II I
(d) If both the statement I and II are effects of independent causes; (c) If both the statement I and II are independent causes;
and I II
I II (d) If both the statement I and II are effects of independent causes;
(e) If both the statement I and II are effects of some common and
cause. I II
I II (e) If both the statement I and II are effects of some common
Sol. (e) cause.
It seems the price of petroleum has increased in general. I II
Alternatively, subsidies may has been reduced, again a cause Sol. (d)
common to both the statements. A certainly cannot be the effect of II, if we believe in an ethical
9. I. Staff members of the university decided to go on strike in world. A is the effect of bad policing and II of good policing.
protest during the examinations. II A
II. The university administration made all the arrangements 12. I. This year, the Government has decided to procure foodgrains
for smooth conduct of examination with the help of outsiders. earlier than the stipulated date.

56
Best App for Govt. Jobs : Neonclasses (Download Now)

II. The year, the farmers have decided to sell their crops to looked into by the RBI. It seems to be a remedial step for the
private traders. same.
(a) If statement I is the cause and statement II is its effect: RBI 15. I. There is increase in water level of all the water
I II tanks supplying drinking water to the city during the last
(b) If statement II is the cause and statement I is its effect: fortnight.
II I II. Most of the trains were cancelled last week due to water
(c) If both the statement I and II are independent causes; logging on the tracks.
I II (a) If statement I is the cause and statement II is its effect:
(d) If both the statement I and II are effects of independent causes; I II
and (b) If statement II is the cause and statement I is its effect:
I II II I
(e) If both the statement I and II are effects of some common (c) If both the statement I and II are independent causes;
cause. I II
I II (d) If both the statement I and II are effects of independent causes;
Sol. (b) and
Government has decided to procure foodgrain earlier than the I II
stipulated data because farmers are planning to sell their (e) If both the statement I and II are effects of some common
crops to private traders. So, statement II is the cause and cause.
Statement A is its effect. I II
II I Sol. (e)
13. Cause → The Government has recently increased its taxes on The problems discussed in both the statements are clearly the
petrol and diesel by about 10%. result of heavy downpour in the area.
→ 10% Which of the following can be a possible effect of the 16. I. Majority of the students in the college expressed their
above cause? opinion against the college authority’s decision to break away
(a) The petroleum companies will reduce the prices of petrol and from the university and become autonomous.
diesel by about 10% . II. The university authorities have expressed their inability to
10% provide grants to its constituent colleges.
(b) The petroleum companies will increase the price of petrol and (a) If statement I is the cause and statement II is its effect:
diesel by about 10% I II
10% (b) If statement II is the cause and statement I is its effect:
(c) The petroleum companies will increase the prices of petrol II I
and diesel by about 5% (c) If both the statement I and II are independent causes;
5% I II
(d) The petrol pumps will stop selling petrol and diesel till the (d) If both the statement I and II are effects of independent causes;
taxes are rolled back by the Government and
(e) None of the above I II
Sol. (e) (e) If both the statement I and II are effects of some common
Companies may hike the price to cover the loss because of the cause.
increased tax. But increase in price is totally the matter of I II
company , we cannot make any guesses about it. Sol. (b)
14. I. The Reserve Bank of India has recently put restrictions on Clearly, the university’s decision to refuse grant to the colleges
few small banks in the country. must have triggered the college authority to become
II. The small banks in the private and cooperative sector in autonomous.
India are not in a position to withstand the competitions of the 17. I. A large number of employees could not report to the duty on
bigger in the public sector. time.
(a) If statement I is the cause and statement II is its effect: II. police had laid down barricades on the road to trap the
I II miscreants.
(b) If statement II is the cause and statement I is its effect: (a) If statement I is the cause and statement II is its effect:
II I I II
(c) If both the statement I and II are independent causes; (b) If statement II is the cause and statement I is its effect:
I II II I
(d) If both the statement I and II are effects of independent causes; (c) If both the statement I and II are independent causes;
and I II
I II (d) If both the statement I and II are effects of independent causes;
(e) If both the statement I and II are effects of some common and
cause. I II
I II (e) If both the statement I and II are effects of some common
Sol. (b) cause.
The inability of the small banks to compete with the bigger I II
ones shall not ensure security and good service to the Sol. (b)
customers, which is an essential concomitant that has to be

57
Download Free PDFs & e-Books from Neon Classes App

A large number of employees could not report to the duty on " "
time because police had laid down barricades on the road to
drop the miscreants. So, statement II is the cause while
9. Coding Decoding
Statement I is its effect.
II I 18. Statement / I. The prices of ‘silver’ have gone  Concept - 1
up from Rs. 27000 per kg to Rs. 50000 per kg in almost a
FIRST TO FIRST (LETTER TO LETTER)
year’s time.
Fix addition
' ' 27000 50000
01. In a certain code language, MONEY is written as PRQHB.In
II. Indian jewellers are receiving a lot of demand for the silver
the same code language, CREDIT will be written as:
ornaments from American and European clients.
,d fuf'pr dksM Hkk"kk esa] MONEY dks PRQHB ds :i esa fy[kk
(a) If statement I is the cause and statement II is its effect:
I II
tkrk gSA mlh dksM Hkk"kk es]a CREDIT dSls fy[kk tk,xk\
(b) If statement II is the cause and statement I is its effect: (a) FUGHLW (b) FUHGLW
II I (c) FUHGWL (d) FHGULW
(c) If both the statement I and II are independent causes; Sol. (b)
13 15 14 5 25 3 18 5 4 9 20
I II
M O N E Y C R E D I T
(d) If both the statement I and II are effects of independent causes;
and +3 +3 +3 +3 +3 ⇒ +3 +3 +3 +3 +3 +3
I II
(e) If both the statement I and II are effects of some common
P R Q H B F U H G L W
cause.
16 18 17 8 2 6 21 8 7 12 23
I II
Sol. (c) Alternate addition
The prices of ‘silver’ going up from Rs. 27000 per kg to over 02. In a certain code language, ‘COLOUR’ is written as
Rs. 50000 per kg and Indian jewellers receiving a lot of ‘FQOQXT’ and ‘VIOLET’ is written as ‘YKRNHV’. How will
demand for the silver ornament from American and European ‘PURPLE’ be written in that language?
clients are effects of independent causes. ,d fuf’pr dwV Hkk"kk es]a ‘COLOUR’ dks ‘FQOQXT’ fy[kk tkrk gS
27000 50000 vkSj ‘VIOLET’ dks ‘YKRNHV’ fy[kk tkrk gSA mlh dwV Hkk"kk esa]
19. Effect: Atleast twenty school children were seriously injured ‘PURPLE’ dks D;k fy[kk tk,xk\
while going for a school picnic during the weekend. (a) SXUSOG (b) RWUSPH
Which of the following can be a probable cause of the above (c) SWUROG (d) RXWSOH
effect? Sol. (c)
(a) The teacher accompanying the school children fell ill during 3 15 12 15 21 18 16 21 18 16 12 5
the journey. C O L O U R P U R P L E
(b) The bus in which the children were travelling met with an +3 +2 +3 +2 +3 +2 ⇒ +3 +2 +3 +2 +3 +2
accident while taking turn on the main highway.
(c) The driver of the bus in which the children were travelling did
not report after the break at the halting place on their journey. F Q O Q X T S W U R O G
(d) The school authority banned all school picnics for the next six 6 17 15 17 24 20 19 23 21 18 15 7
months with immediate effect.
(e) None of the above. Mix variation
Sol. (b) 03. In a certain code language. 'JUPITER' is written as '
The bus in which the children were travelling met with an JVOJSFR '. How will 'NEPTUNE' be written as in that
accident while taking turn on the main highway. language?
Therefore, it can be a possible effect of the cause. ,d fuf'pr dksM Hkk"kk esa 'JUPITER' dks 'JVOJSFR' fy[kk tkrk gSA
20. Statement/ Many persons in the locality are diagnosed mlh Hkk"kk esa 'NEPTUNE' dSls fy[kk tk,xk\
to be suffering from gastro – intestinal diseases after (a) NDPSVME (b) NFOSTOE
consuming poor quality drinking water supplied by the (c) NGOUTOE (d) NFOUTOE
municipal authority. Sol. (d)
10 21 16 9 20 5 18 14 5 16 20 21 14 5
Which of the following substantiates the facts stated in the
J U P I T E R N E P T U N E
above statement?
1. Gastro – intestinal diseases are not very severe in nature. +1 -1 +1 -1 +1 ⇒ +1 -1 +1 -1 +1
2. People suffering from gastrointestinal diseases need not take
medicines. J V O J S F R N F O U T O E
3. Many people suffer from gastrointestinal intestinal diseases 10 21 15 10 19 6 18 14 6 15 21 20 15 5
after consuming stale food. Series (addition)
4. Gastro – intestinal diseases are waterborne diseases. 04. In a certain language, CHHAPAK is coded as DJKEUGR.
5. None of these/ How will MALANGA be coded in that language?
Sol. (a) ,d fuf'pr Hkk"kk esa] CHHAPAK dks DJKEUGR ds :i esa dksfMr
The statement "Gastro-intestianl diseases are water-borne fd;k tkrk gSA mlh Hkk"kk esa MALANGA dks fdl çdkj dksfMr fd;k
diseases" substantiates the facts stated in the statement. tk,xk\

58
Best App for Govt. Jobs : Neonclasses (Download Now)

(a) NCEOSMC (b) NCOCSMC (c) 5291576 (d) 3611121


(c) NCOESMH (d) NCOCMSC Sol. (c)
Sol. (c) 7 21 13
3 8 8 1 16 1 11 13 1 12 1 14 7 1 G U M
C H H A P A K M A L A N G A

⇒49441169
+1 +2 +3 +4 +5 +6 +7 ⇒ +1 +2 +3 +4 +5 +6 +7

49 441 169
D J K E U G R N C O E S M H
4 10 11 5 21 7 18 14 3 15 5 19 13 8
23 1 24
Alternate addition/subtraction series
W A X
05. In a certain code language. WARDROBE is written as
YXVYXHJV. How will ACCURATE be written as in that ⇒5291576
language?
,d fuf'pr dksM Hkk"kk esaA WARDROBE dks YXVYXHJV fy[kk
529 1 576
tkrk gSA mlh Hkk"kk esa ACCURATE dks dSls fy[kk tk,xk\
Vowel/Consonant addition
(a) CZHPYTBV (b) BZHPXTBV
09. In a certain code language, ‘DARK’ is written as ‘CBQJ’ and
(c) DZGPXTBV (d) CZGPXTBV
‘SOUND’ is written as ‘RPVMC’. How will ‘TOXIC’ be written
Sol. (d)
1 3 3 21 18 1 20 5 in that language?
23 1 18 4 18 15 2 5
W A R D R O B E A C C U R A T E ,d fuf'pr dksM Hkk"kk esa] 'DARK' dks 'CBQJ' fy[kk tkrk gS vkSj
'SOUND' dks 'RPVMC' fy[kk tkrk gSA ml Hkk"kk esa 'TOXIC' dSls
+2 -3 +4 -5 +6 -7 +8 -9 ⇒ +2 -3 +4 -5 +6 -7 +8 -9
fy[kk tk,xk\
(a) SPWJB(b) SNWHB
Y X V Y X H J V C Z G P X T B V (c) UPYJD (d) PUYJD
25 24 22 25 24 8 10 22 3 26 7 16 24 20 2 22
Sol. (a)
Opposite relation
4 1 18 11
06. In a certain code language, ‘TULIPS’ is written as
D A R K
‘GFORKH’. How will ‘GARDEN’ be written in that language?
,d fuf’pr dwV Hkk"kk esa] ‘TULIPS’ dks ‘GFORKH’ ds :i esa fy[kk -1 +1 -1 -1
tkrk gSA mlh dwV Hkk"kk es]a ‘GARDEN’ dks fdl :i esa fy[kk
tk,xk\ C B Q J
(a) SZIXVM (b) TZIWUM 3 2 17 10
(c) TBIWVK (d) TZIWVM
Sol. (d)
T U L I P S G A R D E N 19 15 21 14 4
S O U N D
OPP OPP OPP OPP OPP OPP ⇒ OPP OPP OPP OPP OPP OPP
-1 +1 +1 -1 -1
G F O R K H T Z I W V M
Mix opposite R P V M C
07. In a certain code language, 'PERMIT' is written as 18 16 22 13 3
'VVLNOG'. How will 'INERTIA' be written in that language?
,d fuf'pr dwV Hkk"kk esa 'PERMIT' dks 'VVLNOG' fy[kk tkrk gSA Vowel +1
mlh Hkk"kk esa 'INERTIA' dSls fy[kk tk,xk\ Consonant -1
(a) OHYXZCU (b) XOYHCZU
(c) OMYIZRU (d) XYOHBCU 20 15 24 9 3
Sol. (c) T O X I C
16 5 18 13 9 20 9 14 5 18 20 9 1
- 1 +1 - 1 +1 - 1
P E R M I T I N E R T I A

+6 OPP- 6 OPP+6 OPP +6 OPP-6 OPP+6 OPP-6 S P W J B


19 16 23 10 2
V V L N O G O M Y I Z R U
22 22 12 14 15 7 15 13 25 9 26 18 21 Vowel/Consonant based on mix ops
10. In a certain code language, ‘FOREST’ is written as ‘GUSITU’,
Square and ‘MANGROVE’ is written as ‘NEOHSUWI’. How will
08. In a certain code language, 'GUM' is coded as '49441169'. ‘REINCARNATE’ be written in that language?
How will 'WAX' be coded as in that language? ,d fuf'pr dwV Hkk"kk esa] 'FOREST' dks 'GUSITU' fy[kk tkrk gS]
,d fuf'pr dwV Hkk"kk esa 'GUM' dks '49441169' fy[kk tkrk gSA vkSj 'MANGROVE' dks 'NEOHSUWI' fy[kk tkrk gSA ml Hkk"kk esa
mlh Hkk"kk esa 'WAX' dks fdl çdkj dksfMr fd;k tk,xk\ 'REINCARNATE' dSls fy[kk tk,xk\
(a) 2891400 (b) 8412525 (a) SIOODESOEUI (b) SFOFDESOBUI

59
Download Free PDFs & e-Books from Neon Classes App

(c) QIOQDESOEUF (d) SIOODESOEFF ⇓


Sol. (a) 2 21 20
6 15 18 5 19 20 B U T
13 1 14 7 18 15 22 5
F O R E S T
M A N G R O V E
-2 +2
+2 +2
+1 +1 +1 ⇒ +1 +1 +1 +1 +1

D S W V
G U S I T U N E O H S U W I 4 19 23 22
7 21 19 9 20 21 14 5 15 8 19 21 23 9
 Concept - 2
Vowel Next Vowel
Consonant +1 FIRST TO FIRST
(LETTER TO NUMBER)
Increasing/Decreasing order Fix addition
11. In a certain code language, ‘FIXED’ is written as ‘XIFED’ 13. In a certain code language, 'SPECIAL' is coded as
and ‘MOUSE’ is written as ‘USOME’. How will ‘GAMBIT’ be '20176410213'. How will 'MACHINE' be coded as in that
written in that language? language?
fdlh fuf’pr dwV Hkk"kk esa] ‘FIXED’ dks ‘XIFED’ fy[kk tkrk gS] vkSj ,d fuf'pr dwV Hkk"kk esa] 'SPECIAL' dks '20176410213' ds :i esa
‘MOUSE’ dks ‘USOME’ fy[kk tkrk gSA mlh Hkk"kk esa ‘GAMBIT’ dks dksfMr fd;k tkrk gSA mlh Hkk"kk esa 'MACHINE' dks fdl çdkj
dSls fy[kk tk,xk\ dksfMr fd;k tk,xk\
(a) TGMIBA (b) TMGIBA (a) 1324810155 (b) 1424901056
(c) TMIGBA (d) TIMGAB (c) 1424910156 (d) 1524910146
Sol. (c) Decreasing Order Sol. (c) 1424910156
19 16 5 3 9 10 12 13 1 3 8 9 14 5
S P E C I A L M A C H I N E
6 9 24 5 4
F I X E D +1 +1 +1 +1 +1 +1 +1 ⇒ +1 +1 +1 +1 +1 +1 +1
XI F ED
24 9 6 54 20 17 6 4 10 2 13 14 2 4 9 10 15 6
13 15 21 19 5
Series (addition)
M O U S E
14. In a code language, if PEN is written as 17717, then how
U S O M E
will CAP be written in the same language?
21 19 15 13 5
,d dwV Hkk"kk esa] ;fn PEN dks 17717 fy[kk tkrk gS] rks mlh Hkk"kk
esa CAP dSls fy[kk tk,xk\

(a) 4319 (b) 2320 (c) 4219 (d) 2319
7 1 13 2 9 20
Sol. (a) 4319
G A M B I T
16 5 14 3 1 16
T M I G B A
P E N C A P
20 13 9 7 2 1
One to many/Many to one +1 +2 +3 ⇒ +1 +2 +3
12. In a certain code language, ‘AND’ is written as ‘C-LP-F’ and
‘NOR’ is coded as ‘P-MQ-T’. How will ‘BUT’ be written in 17 7 17 4 3 19
that language?
Series (multipliaction)
,d fuf'pr dksM Hkk"kk esa] 'AND' dks 'C-LP-F' fy[kk tkrk gS vkSj
15. In a certain code language, ‘MANGO’ is coded as ‘13-2-42-
'NOR' dks 'P-MQ-T' ds :i esa dksfMr fd;k tkrk gSA ml Hkk"kk esa
28-75’and ‘GRAPE’ is coded as ‘7-36-3-64-25’. How will
'BUT' dSls fy[kk tk,xk\
‘APPLE’ be coded in that language?
(a) C-SU-V (b) D-SW-V ,d fuf'pr dwV Hkk"kk esa] 'MANGO' dks '13&2&42&28&75' ds :i
(c) C-TV-W (d) D-SW-U
esa dksfMr fd;k tkrk gS vkSj 'GRAPE' dks '7&36&3&64&25' ds
Sol. (b)
:i esa dksfMr fd;k tkrk gSA mlh Hkk"kk esa ^ APPLE* dks fdl çdkj
1 14 4 14 15 18
A N D N O R
dksfMr fd;k tk,xk\
(a) 2-16-48-21-25 (b) 1-32-48-48-25
-2 +2 +2 ⇒ -2 +2 +2 (c) 1-32-24-21-30 (d) 2-32-48-24-28
+2 +2
Sol. (b) 1-32-48-48-25
C L P F 13 1 14 7 15 7 18 1 16 5
P M Q T
3 12 16 6 M A N G O G R A P E
16 13 17 20
X1 X2 X3 X4 X5 ⇒ X1 X2 X3 X4 X5
2

13 2 42 28 75 7 36 3 64 25

60
Best App for Govt. Jobs : Neonclasses (Download Now)

⇓ ⇓

1 16 16 12 5 13 1 14 10 21
A P P L E M A N J U

X1 X2 X3 X4 X5 X2 X2 X2 X2 X2

1 32 48 48 25 26 2 28 20 42
Mix opposite Vowel/Consonant multiplication based
16. In a certain code language, ‘TURKEY’ is written as 19. In a certain code language, 'AROUND' is coded as
‘76911525’. How will ‘BREATH’ be written in that '52182412144' and 'FIX' is coded as '63624'. How will
language? 'PLASTIC' be coded in that language?
,d fuf'pr dwV Hkk"kk esa] 'TURKEY' dks '76911525' fy[kk tkrk ,d fuf'pr dwV Hkk"kk es]a 'AROUND' dks '52182412144' vkSj
gSA ml Hkk"kk esa 'BREATH' dSls fy[kk tk,xk\ 'FIX' dks '63624' ds :i esa dksfMr fd;k tkrk gSA mlh Hkk"kk esa
(a) 218526208 (b) 259221208 'PLASTIC' dks fdl çdkj dksfMr fd;k tk,xk\
(c) 259221197 (d) 251841208 (a) 1612521920363
Sol. (b) First Thre Opp. Last three same (b) 1612261920183
20 21 18 11 5 25 2 18 5 1 20 8 (c) 1612522021363
T U R K E Y B R E A T H (d) 1812521920383
Sol. (a) 1612521920363
OPP OPP OPP ⇒ OPP OPP OPP
Vowel → OPP × 2
Consonaut →Same
7 6 9 11 5 25 25 9 22 1 20 8
1 18 15 21 14 4 6 9 24
Mix ops with square
A R O U N D F I X
17. In a certain code language, 'PAGE' is coded as '161495'.
How will 'PART' be coded as in that language? OPP×2 OPP×2 OPP×2 ⇒ OPP×2
,d fuf'pr dwV Hkk"kk esa] 'PAGE' dks '161495' ds :i esa dwVc)
fd;k tkrk gSA mlh Hkk"kk esa 'PART' dks fdl çdkj dksfMr fd;k 52 18 24 12 14 4 6 36 24
tk,xk\ ⇓
(a) 16132420 (b) 16125020
16 12 1 19 20 9 3
(c) 16128620 (d) 16136220 P L A S T I C
Sol. (a) Pattern : Squre of third letter
OPP×2 OPP×2
1st,2nd , & 4th letter are same
16 1 7 5 16 1 18 20
P A G E P A R T 16 12 52 19 20 36 3
Vowel/Consonant based on mix ops
Square
⇒ Square
20. In a certain code language, LARVAE is coded as 15-1-9-5-1-
2. How will INSECT be coded as in that language?
16 1 49 3 16 1 324 20 ,d fuf'pr dwV Hkk"kk esa LARVAE dks 15&1&9&5&1&2 ds :i esa
Multiplication dwVc) fd;k tkrk gSA INSECT dks mlh Hkk"kk esa fdl çdkj dksfMr
18. In a certain code language, 'RAKHI' is coded as 36-2-22- fd;k tk,xk\
16-18 and 'SHALU' is coded as 38-16-2-24-42. How will (a) 3-13-8-2-24-8 (b) 9-13-8-22-24-7
'MANJU' be coded in that language? (c) 18-13-8-2-24-7 (d) 3-13-8-2-24-7
,d fuf'pr Hkk"kk esa] 'RAKHI' dks 36-2-22-16-18 vkSj 'SHALU' dks Sol. (d) 3-13-8-2-24-7
38-16-2-24-42 ds :i esa dksfMr fd;k tkrk gSA mlh Hkk"kk esa Vowel → Vowel Counting Forward
'MANJU' dks fdl çdkj dksfMr fd;k tk,xk\
(a) 13-2-28-10-24 (b) 13-2-14-10-24 A
Consonant E Opposite
→ I OU
(c) 26-1-14-20-42 (d) 26-2-28-20-42 12 1 1 18 22
2 1 35 4 5
Sol. (d) 26-2-28-20-42 L A R V A E
18 1 11 8 9 19 8 1 12 21
OPP OPP OPP ⇒
R A K H I S H A L U

X2 X2 X2 X2 X2 ⇒ X2 X2 X2 X2 X2
15 1 9 5 1 2
9 14 19 5 3 20
36 2 22 16 18 38 16 2 24 42 I N S E C T

3 13 8 2 24 7
Increasing/Decreasing order

61
Download Free PDFs & e-Books from Neon Classes App

21. In a certain code language, 'LOAD' is coded as '27241613' 24. In a certain code language, 'REFUSAL' is coded as
and 'CLEAN' is coded as '2725231614'. How will 'STRIKE' '2101239103' and 'POETRY' is coded as '1067927'. How
be coded in that language? will 'PROFIT' be coded in that language?
,d fuf'pr dwV Hkk"kk esa] 'LOAD' dks '27241613' vkSj 'CLEAN' ,d fuf'pr dwV Hkk"kk esa] 'REFUSAL' dks '2101239103' vkSj
dks '2725231614' ds :i esa dksfMr fd;k tkrk gSA mlh Hkk"kk esa 'POETRY' dks '1067927' ds :i esa dksfMr fd;k x;k gSA mlh
'STRIKE' dks fdl çdkj dksfMr fd;k tk,xk\ Hkk"kk esa 'PROFIT' dks fdl çdkj dksfMr fd;k tk,xk\
(a) 2317190189 (b) 2319171098 (a) 21671829 (b) 81217629
(c) 1231719089 (d) 1719102389 (c) 12186792 (d) 18216972
Sol. (b) 2319171098 Sol. (c) 12186792
Pattern : Increasing order Pattern : Increasing Order
Oppositive No. +1 Single digit X2
L O A D C L E A N Double digit → digit sum
1 4 12 15 1 3 5 12 14 R E F U S A L P O E T R Y
1 5 6 12 18 19 21 5 15 16 18 20 25
A D L O A C E L N A E F L R S U E O P R T Y

OPP+1 OPP+1 OPP+1 OPP+1 OPP+1 OPP+1 OPP+1 OPP+1 OPP+1


X2 X2 X2 digit
sum
digit digit
sum sum
digit
sum X2 digit
sum
digit digit
sum sum
digit digit
sum sum

27 24 16 13 27 25 23 16 14 2 10 12 3 9 10 3 10 6 7 9 2 7
⇓ ⇓
S T R I K E
P R O F I T
5 9 11 18 19 20
6 9 15 16 18 20
E I K R S T
F I O P R T

OPP+1 OPP+1 OPP+1 OPP+1 OPP+1 OPP+1


X2 X2 Digit digit digit digit
sum sum sum sum

23 19 17 10 9 8
12 18 6 7 9 2
First opposite then ops 25. In a certain code language, 'AUSTERE' is coded as
22. In a certain code language, ‘STREAM’ is coded as '13102595'. How will 'SETTING' be coded as in that
161418445228. How will ‘PERIOD’ be coded in that language?
language? ,d fuf'pr dwV Hkk"kk esa] 'AUSTERE' dks '13102595' ds :i esa
,d fuf’pr dwV Hkk"kk esa] ‘STREAM’ dks 161418445228 ds :i esa dwVc) fd;k tkrk gSA mlh Hkk"kk esa 'SETTING' dks fdl çdkj
dwVc) fd;k tkrk gSA ml Hkk"kk es]a ‘PERIOD’ dks dSls dwVc) fd;k dksfMr fd;k tk,xk\
tk,xk\ (a) 11544957 (b) 10522948
(a) 224418382448 (b) 224418362446 (c) 10522957 (d) 11522867
(c) 244418342446 (d) 224420363046 Sol. (c) 10522957
Sol. (b) 224418362446 Pattern : Digit Sum
Oppositive X2 1 21 19 20 5 18 5 19 5 20 20 9 14 7
19 20 18 5 1 13 16 5 18 9 15 4 A U S T E R E S E T T I N G

S T R E A M P E R I O D
Digit Digit Digit Digit Digit Digit Digit Digit Digit Digit Digit Digit Digit Digit
sum sum sum sum sum sum sum ⇒ sum sum sum sum sum sum sum
OPPX2 OPPX2 OPPX2 OPPX2 OPPX2 OPPX2 ⇒ OPPX2 OPPX2 OPPX2 OPPX2 OPPX2 OPPX2

1 3 10 2 5 9 5 10 5 2 2 9 5 7

16 14 18 44 52 28 22 44 18 36 24 46 Alphabetical order based


Fixed coding 26. In a certain code language, 'SERVANT' is coded as
23. In a certain code language, ‘PARK’ is coded as ‘3749’, and '195182211420'. How will 'MAGNIFY' be coded asin that
‘RACE’ is coded as ‘4762’.How will ‘CARE’ be coded in that language?
language? ,d fuf'pr dwV Hkk"kk esa] 'SERVANT' dks '195182211420' ds :i
,d fuf'pr Hkk"kk es]a ‘Park’ dks 3749 vkSj ‘RACE dks 4762 ds :i esa esa dwVc) fd;k tkrk gSA mlh Hkk"kk esa 'MAGNIFY' dks fdl çdkj
dksfMr fd;k tkrk gSA mlh Hkk"kk esa CARE dks fdl çdkj dksfMr dksfMr fd;k tk,xk\
fd;k tk,xk\ (a) 1426693625 (b) 1316143522
(a) 6724 (b) 7642 (c) 7624 (d) 6742 (c) 1317149625 (d) 1417139625
Sol. (d) 6742 Sol. (c) 1317149625
S E R V A N T M A G N I F Y
Pattern : Fix Coded
P AA C
R K R AA C E C A R E ⇒


19 5 18 22 1 14 20 13 1 7 14 9 6 25

3 77 4
6 9 4 77 6 2 6 7 4 2  Concept - 3
Digit sum FIRST TO LAST (LETTER TO LETTER)
Fix addition

62
Best App for Govt. Jobs : Neonclasses (Download Now)

27 In a certain code language, ‘COW’ is written as ‘ZRF’ and 16 5 14 3 9 12 5 18 1 19 5 18


P E N C I L E R A S E R
‘DOG’ is written as ‘JRG’. How will ‘RAT’ be written in that
language? ⇒
+1 +6 +1 +6
,d fuf'pr dwV Hkk"kk es]a 'COW' dks 'ZRF' fy[kk tkrk gS vkSj +2
M K F
+3 +4
R
+5
J V S G V
+2 +3 +4
E
+5
W K
'DOG' dks 'JRG' fy[kk tkrk gSA ml Hkk"kk esa 'RAT' dSls fy[kk 13 11 6 18 10 22 19 7 22 5 23 11

tk,xk\ ⇓
13 1 19 20 5 18
(a) UDW (b) WDU (c) VEU (d) XDV M A S T E R
Sol. (b) WDU
Pattern : Fix adding +3 +1 +6
+2 +3 +4 +5
3 15 23 4 15 7 18 1 20 S G W W F S
C O W D O G R A T 19 7 23 23 6 19
+3 +3 +3 +3 +3 +3 ⇒ +3 +3 +3 Opposite relation
J R G
Z R F
26 18 6 10 18 7
W D U
23 4 21
31. In a certain code language, ‘BLUES’ is written as ‘HVFOY’.
Fix subtraction How will ‘FORCED’ be written in the same language?
28. In a code language, TEMPLE is written as DKOLDS. How will ,d fuf’pr dwV Hkk"kk esa] ‘BLUES’ dks ‘HVFOY’ fy[kk tkrk gSA
WORSHIP be written as in that language? mlh dwV Hkk"kk esa] ‘FORCED’ dks D;k fy[kk tk,xk\
,d dksM Hkk"kk esa] TEMPLE dks DKOLDS ds :i esa fy[kk tkrk gSA (a) ULIXVW (b) XVWLIU
WORSHIP dks mlh Hkk"kk esa dSls fy[kk tk,xk\ (c) WVXILU (d) WVXJKU
(a) OHGRQNV (b) VNQGHOR Sol. (c) Pattern : Only Opposite
2 12 21 5 19 6 15 18 3 5 4
(c) QJITSPX (d) OGHQRVN B L U E S F O R C E D
Sol. (a) OHGRQNV
Pattern: Fix Substract -1 ⇒
OPP OPP OPP OPP
OPP OPP OPP OPP OPP OPP OPP
20 5 13 16 12 5 H V F O Y W V X I L U
T E M P L E 8 22 6 15 25 23 22 24 9 12 15

⇒ Mix opposite
-1
-1 -1 -1 -1 -1
32. In a certain code language, 'PRINT' is written as 'YMNIU'.
D K O L D S
4 11 15 12 4 19 How will 'MAGIC' be written in that language?
23 15 18 19 8 9 16 ,d fuf'pr dwV Hkk"kk esa 'PRINT' dks 'YMNIU' fy[kk tkrk gSA ml
W O R S H I P Hkk"kk esa 'MAGIC' dSls fy[kk tk,xk\
(a) HRLZR (b) HDLVR
-1
-1
-1 -1 -1 -1 -1 (c) HLDRV (d) HRLRZ
O H G R Q N V
15 8 7 18 17 14 22 Sol. (a) HRLZR
Alternate addition,subtraction Pattern : Mix With Oppositive Alternate +5, Oppositive
16 18 9 14 20 13 1 7 9 3
29. In a certain code language, 'COUNTRY' is written as P R I N T M A G I C
'BOWKXLF'. How will 'DESPAIR' be written in that
language? ⇒
+5 OPP +5 OPP +5 +5 OPP +5 OPP +5
,d fuf'pr dwV Hkk"kk esa 'COUNTRY' dks 'BOWKXLF' fy[kk tkrk Y M N I U H R L Z R
25 13 14 9 21 8 18 12 26 18
gSA mlh Hkk"kk esa 'DESPAIR' dSls fy[kk tk,xk\
(a) ULDSVHG (b) UFDMVBG 33. In a certain code language, 'TOPICS' is written as 'QAGONS'
(c) GBVMDFU (d) GBPSXIO and 'GREATH' is written as 'FRYDQF'. How will 'BANDEF'
Sol. (b) UFDMVBG be written in that language?
Pattern : Alternate Addition & Subtraction +3, -3 ,d fuf’pr dksM Hkk"kk esa] ‘TOPICS’ dks ‘QAGONS’ vkkSj ‘GREATH’
3 15 21 14 20 18 25 4 5 19 16 1 9 18 dks ‘FRYDQF’ fy[kk tkrk gSA mlh dksM Hkk"kk esa ‘BANDEF’ dks dSls
C O U N T R Y D E S P A I R
fy[kk tk,xk\
⇒ (a) DCCMZA (b) DCBMZA
+3 +3
-3
B O W
+3 -3

K
+3

X L F-3
+3

U F D
-3 +3 -3

M
+3

V B G
-3
+3
(c) DCBMYA (d) DCBNYA
2 15 23 11 24 12 6 21 6 4 13 22 2 7 Sol. (b) DCBMZA
Series (addition) Pattern : Section Wise
20 15 16 9 3 19 7 18 5 1 20 8
30. In a certain code language, ‘PENCIL’ is written as ‘MKFRJV’ T O P I C S G R E A T H
and ‘ERASER’ is written as ‘SGVEWK’. How will ‘MASTER’
be written in that language? ⇒
-2 -1 -2 -1
-2 -1 -2 -1
,d fuf’pr dwV Hkk"kk esa ‘PENCIL’ dks ‘MKFRJV’ fy[kk tkrk gS vkSj -2
Q A G
-1
O N S F R Y
-2 -1
D Q F
‘ERASER’ dks ‘SGVEWK’ fy[kk tkrk gSA mlh Hkk"kk esa] ‘MASTER’ 17 1 7 15 14 19 6 18 25 4 17 6

dSls fy[kk tk,xk\ 2 1 14 4 5 6
(a) REWWGT (b) SGWWFS B A N D E F
(c) RFWVES (d) SFWYFT
Sol. (b) SGWWFS -2
-2 -1 -1
-2 -1
Pattern : Decreasing Series +6, +5, +4, +3, +2, +1 D C B M Z A

 Concept - 4
FIRST TO LAST (LETTER TO NUMBER)

63
Download Free PDFs & e-Books from Neon Classes App

Fix addition Sol. (a) 326161424434


34. In a certain code language, 'HARVEST' is coded as ‘22-21- Pattern : Single digit → ×3
7-24-20-3-10’. How will ‘FARMER’ be coded as in that Double digit → OPP.×2
language? 8 1 14 9 6 10 9 7 25 1 19 2
H A N I F J I G Y A S A
,d fuf'pr dksM Hkk"kk esa] 'HARVEST' dks '22&21&7&24&20&3&10'
ds :i esa dksfMr fd;k tkrk gSA mlh Hkk"kk esa 'FARMER' dks fdl OPP OPP
X3 OPP X3 X3 X3 X3
çdkj dksfMr fd;k tk,xk\
+3 X3 OPP X2 X2
X2 X3
18 27 26 3 24 3 16 X23 4 21-3 27 34
(a) 19-7-15-19-3-8 (b) 20-7-15-20-3-8 ⇓
10 25 15 20 19 14 1
(c) 20-7-14-21-3-8 (d) 19-7-15-20-3-7
J Y O T S N A
Sol. (b) 20-7-15-20-3-8
Pattern : Fix added +2 OPP
OPP
8 1 18 22 5 19 20 6 1 18 13 5 18 X3
X3 X2
OPP
X2
OPP
X2
X2

3 26 16 14 24-3 4 34
H A R V E S T F A R M E R
Vowel/Consonant based

+2 +2 +2 +2
38. In a certain code language, ‘CROWD’ is coded as
+2 +2 +2 +2 +2 +2 +2 +2

234151124 and ‘TRHICK’ is coded as 162491997. How


+2
22 21 7 24 20 3 10 20 7 15 20 3 8
Series (addition) will ‘FRUGAL’ be coded in that language?
35. If in a certain coding language, ‘PRIMARY’ is written as ,d fuf'pr Hkk"kk esa] CROWD’ dks 234151124 vkSj ‘TRHICK’ dks
‘2620417142423’. How will ‘SECONDARY’ be written in 162491997 ds :i esa dksfMr fd;k tkrk gSA mlh Hkk"kk esa
that code language? FRUGAL’ dks fdl çdkj dksfMr fd;k tk,xk\
;fn ,d fuf'pr dksfMax Hkk"kk esa ‘PRIMARY’ dks (a) 1226761821 (b) 1521012291
‘2620417142423’ fy[kk tkrk gSA mlh dksM Hkk"kk esa (c) 1512021921 (d) 1221021186
‘SECONDARY’ dks dSls fy[kk tk,xk\ Sol. (c) 1512021921
(a) 2620481921101328 Pattern : Vowel → same
(b) 2621481921111328 Consonant → Oppositive
(c) 262048192191328 3 18 15 23 4 20 18 8 9 3 11
C P O W D T R H I C K
(d) 262048202191328
Sol. (a) 2620481921101328 ⇒
OPP
Pattern : Decreasing Series Depent on No. of letter
OPP sAME OPP OPP
OPP OPP OPP OPP SAME OPP
23 4 15 11 24 16 24 9 19 9 7
16 18 9 13 1 18 25 19 5 3 15 14 4 1 18 25
P R I M A R Y S E C O N D A R Y ⇓
6 18 21 7 1 12
⇒ F R U G A L
+1 +7
+9
+2 +3 +4 +5 +6 +6
+3 +5 +7 +8
+1 +2 +4
26 20 4 17 14 24 23 26 20 4 8 19 21 10 13 28

Alternate addition/subtraction series OPP


15 1 20
OPP OPP
21
SAME OPP
9
OPP

21
36. If in a certain coding language, ‘MANISH’ is written as
39. In a certain code language, 'REASON' is coded as '5410129'
‘1511184131’ and ‘SURPKHA’ is written as ‘8020629932’.
and 'HEALTHY' is coded as '716231216'.How will
How will ‘ENJOY’ be written in that code language?
'DIVULGE' be coded in that language?
;fn ,d fuf'pr dksfMax Hkk"kk esa ‘MANISH’ dks ‘1511184131’ vkSj ,d fuf'pr dwV Hkk"kk es]a 'REASON' dks '5410129' vkSj
‘SURPKHA’ dks ‘8020629932’ fy[kk tkrk gSA mlh dksM Hkk"kk esa
'HEALTHY' dks '716231216' ds :i esa dksfMr fd;k x;k gSA ml
‘ENJOY’ dks dSls fy[kk tk,xk\
Hkk"kk esa 'DIVULGE' dks dSls dksfMr fd;k tk,xk\
(a) 32791113 (b) 331973116
(a) 10733494 (b) 24134383
(c) 32719416 (d) 3072048
(c) 13704349 (d) 21435438
Sol. (c) 32719416
13 2 14 9 19 8 19 21 18 16 11 8 1
Sol. (d) 21435438
M A N I S H S U R P K H A Pattern : Vowel →Position Vowel
A E I O U
+7 -8 +9 -10 +11
-12
+7 -8 +9 -10 +11 -12
+13 1 2 3 4 5
15 11 18 4 12 1 8 0 20 6 29 9 32 Consonant → Single digit x2
5 14 10 15 25
E N J O Y
Double digit digit sum
18 5 1 19 15 14 8 5 1 12 20 8 25
R E A S O N H E A L T H Y
+7 -8 +9 -10 +11
32 7 19 4 16
Multiplication 5 4 10 1 2 9 7 16 2 3 1 2 16
37. If in a certain coding language, ‘HANIF’ is written as ⇓
‘182726324’ and ‘JIGYASA’ is written as ‘31634212734’. 4 9 22 21 12 7 5
D I V U L G E
How will ‘JYOTSNA’ be written in that code language?
;fn ,d fuf'pr dksfMax Hkk"kk esa ‘HANIF’ dks ‘182726324’ vkSj
‘JIGYASA’ dks ‘31634212734’ fy[kk tkrk gSA mlh dksM Hkk"kk esa 2 14 3 5 4 3 8
‘JYOTSNA’ dks dSls fy[kk tk,xk\ First opposite then ops
(a) 326161424434 (b) 316261424434 40. If in a certain coding language, ‘DEEPAK’ is written as
(c) 326162414430 (d) 326162414430 ‘203015262627’ and ‘RAJKUMAR’ is written as

64
Best App for Govt. Jobs : Neonclasses (Download Now)

‘1330181020213013’. How will ‘RAKESH’ be written in 4 18 15 14 5 8 15 21 19 5


D R O N E H O U S E
that code language?
;fn ,d fuf'pr dksfMax Hkk"kk esa ‘DEEPAK’ dks ‘203015262627’ ⇒
+1 +1 +1 +1 +1 +1
vkSj ‘RAJKUMAR’ dks ‘1330181020213013’ fy[kk tkrk gSA mlh +1 +1
S E R F O
+1 +1
P I X F T
dksM Hkk"kk esa ‘RAKESH’ dks dSls fy[kk tk,xk\ 19 5 18 6 15 16 9 24 6 20

(a) 231326201330 (b) 231226203013 Section wise


(c) 231226191817 (d) 231220262013 44. In a code language, VICTORY is written as CIVSYRO. How
Sol. (b) 231226203013 will TRAITOR be written as in that language?
Pattern : Opp. +4 ,d dksM Hkk"kk esa] VICTORY dks CIVSYRO ds :i esa fy[kk tkrk
4 5 5 16 1 11 18 1 10 11 21 13 1 18
R A J K U M A R
gSA TRAITOR dks mlh Hkk"kk esa dSls fy[kk tk,xk\
D E E P A K OPP
23 22 22 11 26
OPP OPP OPP OPP OPP

16 9 26 17 16 6 14 26 9
OPP OPP
(a) RTAJORT (b) RATHORT
OPP OPP OPP OPP OPP OPP
(c) ARTJOTR (d) ARTHROT
Sol. (d) ARTHROT
22 9 3 20 15 18 25 20 18 1 9 20 15 18
+4 +4 +4 +4 +4 +4 +4 +4 +4 +4 +4
+4 +4 +4
V I C T O R Y T R A I T O R
20 30 15 26 26 27 13 30 18 10 20 21 30 13
⇓ ⇒
18 1 11 5 19 8 -1 -1

R A K E S H C I V S Y R O A R T H R O T
9 26 16 22 8 19 3 9 22 19 25 18 15 1 18 20 8 18 15 20
OPP OPP OPP OPP OPP OPP

 Concept - 6

+4 +4 +4 +4 +4
+4 REARRANGEMENT
23 12 26 20 30 13 First to last
 Concept - 5 45. ACCIDENT is related to TNEDICCA in the same way as
PASSENGER is related to
CROSS/GROUPING (LETTER TO LETTER) ACCIDENT, TNEDICCA ls mlh rjg lacaf/kr gS tSls PASSENGER
Fix addition (2*2, 3*3, 4*4, 5*5)
lacaf/kr gS&
41. In a certain code language, 'MARGIN' is written as
(a) REGNSESAP (b) RGENESSAP
'SBNOJH'. How will 'PRAYER' be written in that language?
(c) REGNESSAP (d) REGMESSAP
,d fuf'pr dwV Hkk"kk esa 'MARGIN' dks 'SBNOJH' fy[kk tkrk gSA
Sol. (c) REGNESSAP
ml Hkk"kk esa 'PRAYER' dSls fy[kk tk,xk\ Pattern : Rearrangment
(a) BSQZFS (b) QSBSFZ A C C I D E N T P A S S E N G E R

(c) BSQQFZ (d) BSQSFZ


Rearrangment⇒
Sol. (d) BSQSFZ
13 1 18 7 9 14 16 18 1 25 5 18 T N E D I C C A R EG N E S SA P
M A R G I N P R A Y E R
Cross rearrangment
⇒ 46. In a certain code language, STRAIGHT is written as
+1 +1 +1 +1 +1 +1
+1 +1 +1
S B N O J H
+1 +1 +1
B S Q S F Z TSARGITH. How will THURSDAY be written as in that
19 2 14 15 10 8 2 19 17 19 6 26 language?
42. In a certain code language, ‘TRAVEL’ is written as ,d fuf'pr dwV Hkk"kk esa STRAIGHT dks TSARGITH fy[kk tkrk
‘SUWBMF’. How will ‘FROZEN’ be written in that language? gSA THURSDAY dks mlh Hkk"kk esa dSls fy[kk tk,xk\
,d fuf'pr dwV Hkk"kk es]a 'TRAVEL' dks 'SUWBMF' fy[kk tkrk gSA (a) UHTDRSYA (b) AYSDURTH
mlh Hkk"kk esa 'FROZEN' dSls fy[kk tk,xk\ (c) HTRUDSYA (d) HTRUDSAY
(a) SGAQPF (b) SGBPOG Sol. (c) HTRUDSYA
(c) SGAPOF (d) TGBPOF S T R A I G H T T H U R S D A Y

Sol. (c) ⇒
20 18 1 22 5 12 6 18 15 26 5 14
T R A V E L F R O Z E N T S A R G I T H H T R U D S Y A
Section wise

+1 +1 +1 +1 +1
+1
+1 +1 +1 +1 +1
+1
47. In a code language, INFORMATIVE is written as
S U W B M F S G A P O F
ROFNILEVITA. How will SUPERFICIAL be written as in that
19 21 23 2 13 6 19 7 1 16 15 6
language?
Mix variation
,d dwV Hkk"kk esa INFORMATIVE dks ROFNILEVITA fy[kk tkrk
43. In a certain code language, 'DRONE' is written as 'SERFO'.
How will 'HOUSE' be written as in that language?
gSA SUPERFICIAL dks mlh Hkk"kk esa dSls fy[kk tk,xk\
,d fuf'pr dwV Hkk"kk esa 'DRONE' dks 'SERFO' fy[kk tkrk gSA (a) LAICIGREPUS (b) REPUSGLAICI
(c) REPUSELAICI (d) USEPRFIC
mlh Hkk"kk esa 'HOUSE' dks dSls fy[kk tk,xk\
Sol. (c) REPUSELAICI
(a) QJXGB (b) PIXFT I NF OR M A T I V E S UP ER F I C I A L
(c) PQWGS (d) PIVFT

Sol. (b) PIXFT
R OF NI L E V I T A RE P US E L A I CI

 Concept - 7
LETTER TO COMPOSITE NUMBER BASED

65
Download Free PDFs & e-Books from Neon Classes App

Number of letters then multiplication 4+5+22+5+12+15+16+13+5+14+20


48. If CAB is coded as 6 and BED is coded as 40, then how will = 131
HAD be coded as? Sum of letters then addition
;fn CAB dks 6 ds :i esa vkSj BED dks 40 ds :i esa dksfMr fd;k 52. In a certain code language, 'SAFETY' is coded as '95' and
tkrk gS] rks HAD dks dSls dksfMr fd;k tk,xk\ 'EXPAND' is coded as '68'. How will 'GATHER' be coded in
(a) 52 (b) 46 (c) 32 (d) 16 that language?
Sol. (c) 32 ,d fuf'pr dwV Hkk"kk esa] 'SAFETY' dks '80' vkSj 'EXPAND' dks
Pattern : - Multiplication of letters '68' ds :i esa dksfMr fd;k tkrk gSA mlh Hkk"kk esa 'GATHER' dks
Follow : C A B fdl çdkj dksfMr fd;k tk,xk\
3×1×2=6 (a) 68 (b) 70 (c) 63 (d) 84
B E D Sol. (c) 63
2 × 5 × 4 = 40 Pattern : Sum of Letter + 4
H A D Follow :-
8 × 1 × 4 = 32 SAFET Y
Number of letters then multiply, addition mix 19+1+6+5+20+25= 76+4=80
49. If FLOWER is coded as 14 and DISTASTE is coded as 18, EXPAN D
then how will BUREAUCRAT be coded as? 5+24+16+1+14+4 =64+4 = 68
;fn FLOWER dks 14 ds :i esa vkSj DISTASTE dks 18 ds :i esa GAT HER
dksfMr fd;k tkrk gS] rks BUREAUCRAT dks fdl çdkj dksfMr 7+1+20+8+5+18 = 59+4 = 63
fd;k tk,xk\ So correct option = (c)
(a) 22 (b) 18 (c) 20 (d) 28 Sum of letters then multiplication
Sol. (a) 22 53. In a certain code language, ‘ONION’ is coded as 201 and
Pattern (No. of Letter)x2+2 ‘POTATO’ is coded as 261. How will ‘MANGO’ be coded in
FLOWER = No. of letter = 6 that language?
6 X 2 + 2 = 14 ,d fuf'pr Hkk"kk esa] ‘ONION’ dks 201 vkSj ‘POTATO’ dks 261 ds
DISTASTE = No. of Letter = 8 :i esa dksfMr fd;k tkrk gSA mlh Hkk"kk esa ‘MANGO’ dks fdl çdkj
8 X 2 + 2 = 18 dksfMr fd;k tk,xk\
B U R E A U C R A T = No. of letter = 10 (a) 208 (b) 178 (c) 150 (d) 228
10 X 2 + 2 = 22 Sol. (c) 150
Number of letters then multiply, subtraction mix Pattern : Sum of Letter multiplie by 3
Number of letters then squared Follow :-
50. In a certain code language, ‘NOSTALGIA’ is coded as ‘81’. ONION
How will ‘FRICTION’ be coded in that language? 15+14+9+15+14 = 67 × 3 = 201
,d fuf'pr dwV Hkk"kk esa] 'NOSTALGIA' dks '81' ds :i esa dksfMr POTATO
fd;k tkrk gSA mlh Hkk"kk esa ^FRICTION* dks fdl çdkj dksfMr 16+15+20+1+20+15 =87 × 3 = 261
fd;k tk,xk\ MANGO
(a) 105 (b) 64 (c) 85 (d) 36 13+1+14+7+15 = 50 × 3 = 150
Sol. (b) 64 Sum of letters then multiply with ops
Pattern : 54. In a certain code language, ‘ALPINE’ is coded as ‘171’ and
N O S T A L G I A = 81 ‘SPRING’ is coded as ‘83’. How will ‘CAPITAL’ be coded in
No. Of Letter = 9 that language?
Square of No. of Letter = = 81
,d fuf'pr dwV Hkk"kk esa] 'ALPINE' dks '171' vkSj 'SPRING' dks
'83' ds :i esa dksfMr fd;k tkrk gSA mlh Hkk"kk esa 'CAPITAL' dks
F R I C T I O N
No. of Letter = 8 fdl çdkj dksfMr fd;k tk,xk\
(a) 186 (b) 93 (c) 62 (d) 124
Square of No. of Letter = = 64
Sol. (a) 186
Sum of letters
Pattern : Sum of Letter multiplie by No. of vowel
51. If PROPERTY is coded as 133 and ESTATE is coded as 70
Follow : - A L P I N E
then DEVELOPMENT is coded as
1+12+16+9+14+5=57×3=171
;fn PROPERTY dks 133 ds :Ik esa vkSj ESTATE dks 70 ds :Ik esa
S P R I N G
dwVc) fd;k tkrk gS rks DEVELOPMENT dks fdl :Ik esa dwVc)
19+16+18+9+14+7 = 83×1 = 83
fd;k tk,xk\ C A P IT A L
(a) 129 (b) 130 3+1+16+9+20+1+12 = 62×3 = 186
(c) 131 (d) 139 Opposite Sum of letters then addition
Sol. (c) 131 55. In a certain code language, 'FRENCH' is coded as '114' and
Pattern : Sum of Letter 'LOSS' is coded as '47'. How will 'COURSE' be coded in that
Follow : P R O P E R T Y language?
16+18+15+16+5+18+20+25 = 133 ,d fuf'pr dwV Hkk"kk esa] 'FRENCH' dks '114' vkSj 'LOSS' dks '47'
ESTATE ds :i esa dksfMr fd;k tkrk gSA mlh Hkk"kk esa 'COURSE' dks fdl
5+19+20+1+20+5 = 70
çdkj dksfMr fd;k tk,xk\
DEVELOPMENT
(a) 103 (b) 120 (c) 87 (d) 81

66
Best App for Govt. Jobs : Neonclasses (Download Now)

Sol. (c) 87 PURSE


Pattern : Sum of Opp. Letters + No. of latters Opp :- 11+21+9+8+5 = 54
Follow : F R E N C H FIR ST
21+9+21+13+24+19= 108 Opp :- 21+9+9+8+7 = 54
108+6 = 114  Concept - 8
L O S S
OPP : - 15+12+8+8 = 43 SUBSTITUTION BASED
43+4 = 47 59. If in a code language, ‘Red’ is called ‘Green’, ‘Green’ is
C O U R S E called ‘Pink’, ‘Pink’ is called ‘Brown’, and ‘Brown’ is called
OPP: 24+12+6+9+8+22 = 81 ‘Violet’, then which coloured light is used to stop vehicles
81+6 = 87 on traffic Signal?
Opposite Sum of letters then subtraction ;fn fdlh lkadsfrd Hkk"kk esa 'yky' dks 'gjk'] 'gjs' dks 'xqykch']
56. In a certain code language ‘ROUBST’ is coded as 61. How 'xqykch' dks 'Hkwjk' vkSj 'Hkwjs' dks 'cSaxuh' dgk tkrk gS] VªSfQd flXuy
will ‘FORTUNATE’ be coded in that language? ij okguksa dks jksdus ds fy, fdl jax ds izdk’k dk bLrseky fd;k
,d fuf'pr Hkk"kk esa] ‘ROUBST dks 61 ds :i esa dksfMr fd;k tkrk tkrk gS\
gSA mlh Hkk"kk esa ‘FORTUNATE’ dks fdl çdkj dksfMr fd;k tk,xk\ (a) Brown/Hkwjk (b) Green/gjk
(a) 124 (b) 114 (c) 141 (d) 142 (c) Red/yky (d) Violet/cSaxuh
Sol. (b) 114 Sol. (b) Green/gjk
Pattern : Sum of Opp. Letters – No. of Latters Red colour is used to stop Vehicles on traffic Signal.
Follow : R O U B S T Here red is coded by Green
ILFYHG So correct option (b)
9+12+6+25+8+7 =67
 Concept - 09
67-6 = 61
F O R T U N A T E CHINESE CODING
Opp. Opp Opp Opp Opp Opp Opp Opp Opp Letter to letter
60. In a certain code language, 'do or die' is written as 'su ru
U L I G F M Z G V
ko', 'let us do it' is written as 'ko mo yo zu' and 'this or
21+12+9+7+6+13+26+7+22=
that' is written as 'ga tu ru'. How will 'die' be written in
Opp. Sum = 123 – 9 = 114
that language?
Opposite Sum of letters then multipliaction ,d fuf'pr dwV Hkk"kk esa 'do or die' dks ‘su ru ko’, fy[kk tkrk gS]
57. In a certain code language, ‘GLOVE’ is coded as 148. How ‘let us do it’ dks 'ko mo yo zu' fy[kk tkrk gS vkSj 'this or that'
will ‘OBDURATE’ be coded in that language? dks 'ga tu ru' fy[kk tkrk gSA mlh Hkk"kk esa ’die' dSls fy[kk tk,xk\
,d fuf’pr dwV Hkk"kk esa] ‘GLOVE’ dks 148 fy[kk tkrk gSA mlh (a) su (b) ru (c) ku (d) ko
Hkk"kk esa ‘OBDURATE’ dks dSls dwVc) fd;k tk,xk\ Sol. (a) su
(a) 602 (b) 402 (c) 520 (d) 502
do or die su ru ko
Sol. (c) 520
Pattern :- Opposite Sum of Letter multiplie by No. of Vowel let us do it ko mo yo zu
Follow :- G L O V E this or that ga ku ru
TOLEV
20+15+12+5+22 = 74 → die’s coded is present in option a,b,d
74×2 = 148 So option (d) eliminate.
→ Option (b) & option (d) are also eliminate because die word
OBDURATE not present in the other Sentence.
LYWFIZGV → So correct option is (a)
12+25+23+6+9+26+7+22= 130 → do’s code is present in option (a)
130 X 4 = 520 (c) & (d) so option (b) is eliminated
Opposite Sum of letters then division Option (a) & (d) also eliminated because these coded are
Sum of vowel + opposite sum of consonants present other Sentence.
58. In certain code language, 'BLINK'is coded as '78' and Because So word is not present other sentence.
'"PURSE'is coded as '54'. How will 'FIRST'be coded asin Letter to number
that language? 61. In a certain code language, ‘are you ready’ is written as
,d fuf'pr dwV Hkk"kk esa] 'BLINK' dks '78' ds :i esa dksfMr fd;k ‘541’, ‘we are going’ is written as ‘261’ and ‘she is ready’ is
tkrk gS vkSj '"PURSE' dks '54' ds :i esa dksfMr fd;k tkrk gSA written as ‘498’. How will ‘you’ be written in that
ml Hkk"kk esa 'FIRST' dks dSls dksfMr fd;k tk,xk\ language?
(a) 54 (b) 71 (c) 64 (d) 76
,d fuf’pr dwV Hkk"kk esa] ‘are you ready’ dks ‘541’ fy[kk tkrk gS]
Sol. (a) 54 ‘we are going’ dks ‘261’ fy[kk tkrk gS vkSj ‘she is ready’ dks
Pattern : sum of vowel actual values & opposite values of ‘498’ fy[kk tkrk gSA mlh Hkk"kk esa ‘you’ dks D;k fy[kk tk,xk\
consonants (a) 6 (b) 1 (c) 5 (d) 4
Follow : Sol. (c) 5
BLINK
25+15+9+13+16= 78

67
Download Free PDFs & e-Books from Neon Classes App

‘ are you ready ’ ‘5 4 1’


‘ we are going ’ 261
‘she is ready ’ ‘4 9 8’

10. Completion of figure


 Concept - Answer figure/ mÙkj vkd`fr

1. Question figure/iz’u vkd`fr

(a) (b) (c) (d )


Sol. (c) Clearly the figure in option (c) will complete the pattern
when placed in the blank space.
5. Question figure/iz’u vkd`fr

Answer figure/ mÙkj vkd`fr

?
(a) (b) (c) (d)
Sol. (b) Clearly the figure in option (b) will complete the pattern
Answer figure/ mÙkj vkd`fr
when placed in the blank space.

2. Question figure/iz’u vkd`fr

(a) (b) (c) (d )


Sol. (b) Clearly the figure in option (b) will complete the pattern
when placed in the blank space.
6. Question figure/iz’u vkd`fr

Answer figure/ mÙkj vkd`fr

(a) (b) (c) (d) Answer figure/ mÙkj vkd`fr


Sol. (b) Clearly the figure in option (b) will complete the pattern
when placed in the blank space.
3. Question figure/iz’u vkd`fr

(a) (b) (c) (d )


Sol. (d) Clearly the figure in option (d) will complete the pattern
when placed in the blank space.
7. Question figure/iz’u vkd`fr

Answer figure/ mÙkj vkd`fr

(a) (b) (c) (d)


Answer figure/ mÙkj vkd`fr
Sol. (a) Clearly the figure in option (a) will complete the pattern
when placed in the blank space.
4. Question figure/iz’u vkd`fr

68
Best App for Govt. Jobs : Neonclasses (Download Now)

fuEufyf[kr esa ls dkSulh mrj vkd`fr] iz’u vkd`fr dks iwjk djsxh?
Question figure/iz’u vkd`fr

(a) (b) (c) (d)


Sol. (a) Clearly the figure in option (a) will complete the pattern
when placed in the blank space.
8. Question figure/iz’u vkd`fr
Answer figure/ mÙkj vkd`fr

Sol. (b) Clearly the figure in option (b) will complete the pattern
when placed in the blank space.
12. Which one of the Answer figures shall complete the given
Answer figure/ mÙkj vkd`fr Question figure?
fuEufyf[kr esa ls dkSulh mrj vkd`fr] iz’u vkd`fr dks iwjk djsxh?
Question figure/iz’u vkd`fr

(a) (b) (c) (d )


Sol. (a) Clearly the figure in option (a) will complete the pattern
when placed in the blank space.
9. Question figure/iz’u vkd`fr

Answer figure/ mÙkj vkd`fr

Sol. (a) Clearly the figure in option (a) will complete the pattern
when placed in the blank space.
13. Question figure/iz’u vkd`fr
Answer figure/ mÙkj vkd`fr

(a) (b) (c) (d)


Sol. (c) Clearly the figure in option (c) will complete the pattern
when placed in the blank space.
10. Question figure/iz’u vkd`fr Answer figure/ mÙkj vkd`fr

Sol. (c) Clearly the figure in option (c) will complete the pattern
when placed in the blank space.
Answer figure/ mÙkj vkd`fr 14. Question figure/iz’u vkd`fr

(a) (b) (c) (d )


Sol. (d) Clearly the figure in option (d) will complete the pattern
when placed in the blank space. Answer figure/ mÙkj vkd`fr
11. which one of the Answer figures shall complete the given
Question figure?

69
Download Free PDFs & e-Books from Neon Classes App

Sol. (d) Clearly the figure in option (d) will complete the pattern
when placed in the blank space. Sol. (c) Clearly the figure in option (c) will complete the pattern
15. Question figure/iz’u vkd`fr when placed in the blank space.
19. Question figure/iz’u vkd`fr

Answer figure/ mÙkj vkd`fr

Answer figure/ mÙkj vkd`fr


Sol. (b) Clearly the figure in option (b) will complete the pattern
when placed in the blank space.
16. Question figure/iz’u vkd`fr

(a) (b) (c) (d )


Sol. (c) Clearly the figure in option (c) will complete the pattern
when placed in the blank space.
20. Question figure/iz’u vkd`fr

Answer figure/ mÙkj vkd`fr

Sol. (d) Clearly the figure in option (d) will complete the pattern
Answer figure/ mÙkj vkd`fr
when placed in the blank space.
17. Question figure/iz’u vkd`fr

(a) (b) (c) (d )


Sol. (d) Clearly the figure in option (d) will complete the pattern
when placed in the blank space.
Answer figure/ mÙkj vkd`fr
21. Question figure/iz’u vkd`fr

Sol. (c) Clearly the figure in option (c) will complete the pattern
when placed in the blank space.
18. Question figure/iz’u vkd`fr
Answer figure/ mÙkj vkd`fr

(a) (b) (c) (d)


Answer figure/ mÙkj vkd`fr Sol. (b) Clearly the figure in option (b) will complete the pattern
when placed in the blank space.
22. Question figure/iz’u vkd`fr

70
Best App for Govt. Jobs : Neonclasses (Download Now)

?
(a) (b) (c) (d )
Sol. (a) Clearly the figure in option (a) will complete the pattern
when placed in the blank space.
Answer figure/ mÙkj vkd`fr 26. Question figure/iz’u vkd`fr

(a) (b) (c) (d)


Sol. (a) Clearly the figure in option (a) will complete the pattern
when placed in the blank space.
23. Question figure/iz’u vkd`fr

Answer figure/ mÙkj vkd`fr

Answer figure/ mÙkj vkd`fr


(a) (b) (c) (d )
Sol. (b) Clearly the figure in option (b) will complete the pattern
when placed in the blank space.
27. Question figure/iz’u vkd`fr
(a) (b) (c) (d)
Sol. (c) Clearly the figure in option (c) will complete the pattern
when placed in the blank space.
24. Question figure/iz’u vkd`fr

?
Answer figure/ mÙkj vkd`fr

Answer figure/ mÙkj vkd`fr

(a) (b) (c) (d)


(a) (b) (c) (d) Sol. (c) Clearly the figure in option (c) will complete the pattern
when placed in the blank space.
Sol. (c) Clearly the figure in option (c) will complete the pattern
28. Question figure/iz’u vkd`fr
when placed in the blank space.
25. Question figure/iz’u vkd`fr

Answer figure/ mÙkj vkd`fr


Answer figure/ mÙkj vkd`fr

71
Download Free PDFs & e-Books from Neon Classes App

32. Question figure/iz’u vkd`fr

(a) (b) (c) (d)


Sol. (d) Clearly the figure in option (d) will complete the pattern
when placed in the blank space.
29. Question figure/iz’u vkd`fr
Answer figure/ mÙkj vkd`fr

(a) (b) (c) (d)


Sol. (c) Clearly the figure in option (c) will complete the pattern
when placed in the blank space.
Answer figure/ mÙkj vkd`fr 33. Question figure/iz’u vkd`fr

(a) (b) (c) (d )


Sol. (b) Clearly the figure in option (b) will complete the pattern
when placed in the blank space.
30. Question figure/iz’u vkd`fr
Answer figure/ mÙkj vkd`fr

(a) (b) (c) (d)


Sol. (a) Clearly the figure in option (a) will complete the pattern
Answer figure/ mÙkj vkd`fr when placed in the blank space.
34. Question figure/iz’u vkd`fr

(a) (b) (c) (d)


Sol. (b) Clearly the figure in option (b) will complete the pattern
when placed in the blank space.
Answer figure/ mÙkj vkd`fr
31. Question figure/iz’u vkd`fr

(a) (b) (c) (d)


Sol. (c) Clearly the figure in option (c) will complete the pattern
Answer figure/ mÙkj vkd`fr when placed in the blank space.
35. Question figure/iz’u vkd`fr

(a) (b) (c) (d )


Sol. (a) Clearly the figure in option (a) will complete the pattern
when placed in the blank space. Answer figure/ mÙkj vkd`fr

72
Best App for Govt. Jobs : Neonclasses (Download Now)

(a) (b) (c) (d)


Sol. (a) Clearly the figure in option (a) will complete the pattern
Answer figure/ mÙkj vkd`fr
when placed in the blank space.
36. Question figure/iz’u vkd`fr

(a) (b) (c) (d )


Sol. (d) Clearly the figure in option (d) will complete the pattern
when placed in the blank space.
40. Question figure/iz’u vkd`fr

Answer figure/ mÙkj vkd`fr

(a) (b) (c) (d)


?
Sol. (b) Clearly the figure in option (b) will complete the pattern Answer figure/ mÙkj vkd`fr
when placed in the blank space.
37. Question figure/iz’u vkd`fr

(a) (b) (c) (d )


Sol. (d) Clearly the figure in option (d) will complete the pattern
when placed in the blank space.

Answer figure/ mÙkj vkd`fr 11. Counting of figure


In this chapter, a diagram is given in the question and the
number of shapes given in that diagram is asked. Therefore,
we will learn to count the number of different types of shapes
like triangle, square, rectangle, circle, cube etc.
Sol. (d) Clearly the figure in option (d) will complete the pattern  Concept 1  Triangle :-
when placed in the blank space.
38. Question figure/iz’u vkd`fr Now, we will learn to count number of triangles in
different figures.
Case – I :- Basic type – I
1. Find out the number of triangles in the given figure ?

Answer figure/ mÙkj vkd`fr (a) 2 (b) 4 (c) 3 (d) 5


Sol. (c)
First we will number the individual triangles in the given
figure. After it we will find out the number of total triangles by
adding all such numbers.
Sol. (c) Clearly the figure in option (c) will complete the pattern
when placed in the blank space.
39. Question figure/iz’u vkd`fr 1 2

Total triangles = 1+2 = 3


2. Find out the number of triangles in the given figure ?

73
Download Free PDFs & e-Books from Neon Classes App

P 1 2Q 1 2
R S
1 2 3 4
L M
D 1 2 3
E
1 2 3 4 5
(a) 23 (b) 21 (c) 18 (d) 25 B
1
C
Sol. (b) (i) Number of triangles from point A on line BC = (1+2+3+4+5)
= 15
(ii) Number of triangles from point A on line DE = (1+2+3)=6
(iii) Number of triangles from point A on line LM = (1+2+3+4)
12 3 4 56 = 10
Total triangles = 1+2+3+4+5+6 = 21 (iv) Number of triangles from point A on line PQ = (1+2)= 3
3. Find out the number of triangles in the given figure ? (v) Number of triangles from point A on line RS = (1+2) = 3
Total triangles = 15+6+10+3+3 = 37
6. How many triangles are there in the given figure?
nh xbZ vk—fr esa fdrus f=Hkqt gSa\

(a) 18 (b) 22 (c) 20 (d) 25


Sol. (c)
First we will number the individual triangles in the given (a) 11 (b) 8 (c) 15 (d) 12
figure. After that add all such numbers and then multiply the (SSC CGL Pre. 2020, 13.08.2021 Shift-3)
value obtained with the number of horizontal lines in the Sol. (d)
figure.

 
D C
1 1+2 3+(1+2)=6
1 2 3 4
A B
1
There are two horizontal lines AB and CD in the given figure.  
Therefore, number of triangles = (1  2  3  4) 2  10  2  20
4. Find out the number of triangles in the given figure ? 6+3 9+3 = 12
7. How many triangles are there in the given figure?
nh xbZ vk—fr esa fdrus f=Hkqt gSa\

(a) 120 (b) 110 (c) 100 (d) 105


Sol. (d)
(a) 9 (b) 11 (c) 10 (d) 12
(SSC CGL Pre. 2020, 24.08.2021 Shift-2)
Sol. (a)
12 3 4 56
Number of horizontal lines = 5
 
Hence, total triangles
= (1  2  3  4  5  6) 5  21  5  105 1 1+2 3+2
5. Find out the number of triangles in the given figure ?

 
6+3 =9
(a) 37 (b) 40 (c) 35 (d) 42
Sol. (a) 8. Find out the number of triangles in the given figure?

74
Best App for Govt. Jobs : Neonclasses (Download Now)

(a) 7 (b) 8 (c) 9 (d) 6 A


Sol. (b) 1
A 2
3
4
F 3 4 5
E B 1 2
C

B D C Here, the number of lines drawn from point A and B is equal.


(A) From point A Therefore, the number of original triangles = 53 = 125
(i) Number of triangles on line BF = (1+2) = 3 11. How many triangles are there in the given figure?
(ii) Number of triangles on line BC = (1+2) = 3 nh xbZ vk—fr esa fdrus f=Hkqt gSa\
(B) From point B
(i) Number of triangles on line AD= (1+2) = 3
(ii) Number of triangles on line AC = (1+2) = 3
Total triangles = 3+3+3+3 = 12
There are some common triangles in it which have been
counted twice. Therefore, we have to subtract such triangles.
The number of common triangles will be equal to the number
of points (D,E,F and C) excluding point A and point B.
Hence, common triangles = 4 (a) 10
Therefore, total original triangles = 12-4 = 8 (b) 9
Trick : If the number of lines drawn from those points is equal, (c) 7
from which we have counted the number of triangles, then we (d) 8
can find out the number of original triangles in the given figure (SSC CGL Pre. 2020, 20.08.2021 Shift-1)
using the following trick. Sol. (d)
A
  
1 F 1 1+1 2+1 3+1
E
1 2  
B D C
4+2 6+2=8
Here one line AD is drawn from point A and similarly one line
BF is drawn from point B. Since the number of lines drawn Correct answer is option (d)
from both points are equal. 12. How many triangles are there in the given figure ?
Therefore, the number of original triangles 23 = 8
nh xbZ vk—fr esa fdrus f=Hkqt gSa\
9. Find out the number of triangles in the given figure ?

(a) 10 (b) 7 (c) 8 (d) 11


(a) 25 (b) 30 (c) 18 (d) 27 (SSC CHSL Pre. 2021, 10.06.2022 Shift-2)
Sol. (d) Sol. (b)
A
1 F
  
2 G

1 2 3 1+2 3+4 = 7
1
B D E C
Correct answer is option (b)
Two lines AD and AE have been drawn from point A. Similarly 13. Find out the number of triangles in the given figures ?
two lines BF and BG have been drawn from point B. Hence, the
number of lines from both points is equal.
Thus, number of original triangles = 33 = 27
10. Find out the number of triangles in the given figure ?

(a) 18 (b) 22 (c) 20 (d) 24


Sol. (c)
(a) 120 (b) 125 (c) 115 (d) 130
Sol. (b)

75
Download Free PDFs & e-Books from Neon Classes App

D B
A
1 1
2 A 2
3 3 123 E
D
4 4 1 1
E C 2 2
Number of triangles in ABC = (1+2+3+4) = 10 3 3
B C
Number of triangles in ADE = (1+2+3+4) = 10 F
Here, no new triangle is formed by connecting the two figures. Number of triangle in ΔADE = (1+2+3)=6
Total triangles = 10+10 =20 Similarly 6-6 triangle will be formed in Δ BDF and ΔCEF.
14. Find out the number of triangles in the given figures ? In addition 2 new triangles will be formed by connecting all
three figures = Δ DEF and ΔABC
Hence, total triangle = 18+2=20
17. Find out the number of triangles in the given figures?

(a) 16 (b) 18 (c) 20 (d) 14


Sol. (a)
A

F G E (a) 20 (b) 15 (c) 16 (d) 18


Sol. (b)

B D C A
Number of triangles in AGC = (1+2)=3
Number of triangles in AGB = (1+2)=3 1 2
D E
Number of triangles in BGC = (1+2)=3
The following 6 triangles other than these triangles 1 1
= ABD, ACD, BCE, BAE, CAF, and CBF 2 2
B F C
Finally one big triangle ABC is formed.
Hence, total triangles = 3+3+3+6+1 = 16 There are formed three triangle in each figure Δ ADE, Δ BDF
15. Find out the number of triangles in the given figure ? and Δ CEF.
There are 4 triangles in Δ DEF
In addition 2 triangles ΔDEF and Δ ABC will be formed.
Hence, total triangles = (3×3)+4+2
= 9+4+2=15
18. How many triangles are there in the given figure?
(a) 15 (b) 14 (c) 13 (d) 24 nh xbZ vkd`fr esa fdrus f=Hkqt gS\
Sol. (d)
A

F H
E
G (a) 11 (b) 13 (c) 10 (d) 12
B D C (SSC CGL Pre. 2020, 13.08.2021 Shift-1)
(i) If the line FE is not drawn in the given figure, number of Sol. (a)
total triangles = 16
  
[The new without line FE is similar to the figure in example
34.]
1 1+1 2+4
(ii) The new triangles formed by line FE = AFH, AHE, FHG,
EGH, AFE, GFE, BFE, and CFE
Hence, total triangles = 16+8 = 24 
16. Find out the number of triangles in the given figure ?
6+1 7+0

 

= 11
7+4
Correct answer is option (a)
(a) 20 (b) 18 (c) 22 (d)19 19. Find the number of triangles in the given figure.
Sol. (a)

76
Best App for Govt. Jobs : Neonclasses (Download Now)

nh xbZ vk—fr esa f=Hkqtksa dh la[;k Kkr dhft,A


  
0+8 8+4
0


12+2 14+1 =15
(a) 13 (b) 12 (c) 15 (d) 14 Correct answer is
(SSC CGL Pre. 2020, 13.08.2021 Shift-2) option (a)
Sol. (a) 22. How many triangles are there in the given figure?
Part-(A) Part-(B) nh xbZ vkd`fr esa fdrus f=Hkqt gS\

  
1 2
3 2


(a) 13 (b) 15 (c) 12 (d) 14
2+1 (SSC CGL Pre. 2019, 09.03.2020 Shift-2)
2+1+2 = 5
Part-(C) Combination Sol. (a)

 
5 2 2+1 = 3 3+2 = 5

=3+5+5 =13
Correct answer 4+4 = 8
is option (a)
20. How many triangles are there in the given figure?
nh xbZ vk—fr esa fdrus f=Hkqt gSa\


= 8+5 = 13

(a) 14 (b)12 (c) 11 (d) 13


Correct answer is option (a)
(SSC CGL Pre. 2020, 18.08.2021 Shift-1)
23. How many triangles are there in the given figure?
Sol. (b)
nh xbZ vk—fr esa fdrus f=Hkqt gSa\
3
3

3 3

3+3+3+3 = 12
Correct answer is option (b)
21. Find the number of triangles in the given figure. (a) 12 (b) 14 (c) 8 (d) 10
nh xbZ vk—fr esa f=Hkqtksa dh la[;k Kkr dhft,A (SSC CHSL Pre. 2021, 27.05.2022 Shift-2)
Sol. (d)

 
4+4
0 0+4


(a) 15
(b) 19
(c) 17 8+2 = 10
(d) 13 Correct answer is option (d)
(SSC CGL Pre. 2020, 23.08.2021 Shift-1) 24. Find the total number of inlaid triangles in the given figure.
Sol. (a)

77
Download Free PDFs & e-Books from Neon Classes App

nh xbZ vkd`fr esa vafuZfgr f=Hkqtksa dh dqy la[;k Kkr dhft,A

(a) 20 (b) 14 (c) 18 (d) 16


Sol. (d)
3 4
2 5
1 6
(a) 9 (b) 14 (c) 12 (d) 10 8 7
(SSC Constable GD 2021, 06.12.2021, Shift - 3) Here the figure given has eight different triangles.
Sol. (b) Hence, number of total triangles = 8×2 = 16
28. How many triangles are there in the following figure?
 
fuEufyf[kr vk—fr esa fdrus f=Hkqt gSa\

8 8+2 10+4 = 14

Correct answer is option (b)


25. Find the number triagnels in the given figure.
nh xbZ vk—fr esa la[;k f=Hkqt Kkr dhft,A
(a) 10 (b) 18 (c) 16 (d) 14
(SSC CPO 2018, 12.03.2019 Shift-2)
Sol. (b)
Part (A) Part (B)

(a) 16
(b) 18
(c) 14 8 8
(d) 20
(SSC CGL Pre. 2020, 18.08.2021 Shift-2)
Sol. (b) 
  

1 1+3 4+3 16+2 = 18

 Correct answer is option (b)


 29. How many triangles are present in the following figure?
7+3 10+4+1+1 16+2 = 18
fuEufyf[kr vk—fr esa fdrus f=Hkqt ekStwn gSa\

Correct answer is option (b)


Case-2
Basic Type-II
26. Find out the number of triangles in the given figures?

(a) 20 (b)17
(a) 8 (b) 10 (c) 7 (d) 6 (c) 18 (d)19
Sol. (a) (SSC CGL Pre. 2018, 12.06.2019 Shfit - 1)
Sol. (d)
2
1 3 
4

In such a pattern all the individual triangle in the given figure 18 18+1 = 19
will be numbered. Now, the total number of triangles in the Correct answer is option (d)
given figure will be equal to twice of those individual triangles. 30. Find the number of triangles in the given figure.
For e.g. There are four individual triangles in the given figure. nh xbZ vk—fr esa f=Hkqtksa dh la[;k Kkr dhft,A
Hence, total triangles = 4×2 = 8
27. Find out the number of triangles in the given figures?

78
Best App for Govt. Jobs : Neonclasses (Download Now)

Part (A)

8 8+2 = 10
(a) 17
(b) 21 Part (B)
(c) 23 
(d) 19
(SSC CGL Pre. 2020, 20.08.2021 Shift-3) 8 8+2 = 10
Sol. (d)

  

8 8+4 12+3 15+4 = 19


Correct answer is option (d)
31. How Many triangles are there in the given figure?
nh xbZ vk—fr esa fdrus f=Hkqt gSa\

20+2 = 22
Correct answer is
option (b)
33. How many triangles are there in the following figure?
(a) 20 (b) 15 (c) 17 (d) 19 fuEufyf[kr vk—fr esa fdrus f=Hkqt gSa\
(SSC CPO 2019, 12.12.2019 Shift-1)
Sol. (d)

16 16+3 = 19 (a) 14 (b) 24 (c) 22 (d) 18


Correct answer is option (d) (SSC CGL Pre. Exam. 10.06.2019 Shfit - 1)
32. How many triangles are present in the given figure? Sol. (c)
nh xbZ vk—fr esa fdrus f=Hkqt ekStwn gSa\ Part (A) Part (B)

8 8

(a) 30 (b) 22 Overlaping
(c) 28 (d) 26
(SSC CGL Pre. 2019. 04.03.2020 Shfit - 1)
Sol. (b)
16+2+2+2 = 22 Correct answer is
option(c)
34. Find out the number of triangles in the given figure?

(a) 35 (b) 31 (c) 32 (d) 39


Sol. (d)

79
Download Free PDFs & e-Books from Neon Classes App

A D E J

C F I
B
(a) 29 (b) 31 (c) 21 (d) 19
(SSC CGL Pre. 2019, 07.03.2020 Shift-3)
G H Sol. (a)
Total triangles in the figure ABCD = (4×2) = 8
Similarly, there will be 8-8 triangles in each figure CDEF, EFIJ 
and FGHI
Hence, total triangles = ¼4×8½ = 32
In addition some new triangles will be formed by connecting
2 2 + (3+3)
all the figures = Δ ACE, BDF, ΔDFJ, ΔCEI, ΔJFH and one big
triangle ΔDHJ. 2+6


Hence, total number of original triangles = 32 + 7 = 39
35. Find out the number of triangles in the given figure?

8+(2+1+3+2+2)+10+1
 8+20 + 1 = 29
Correct answer is option (a)
(a) 40 (b) 41 (c) 42 (d) 45
A
Sol. (b) D
B C
E I
F G H
A D E H J
K
2
1 3 L M N O P
4
Q R S
C K F G T
B
ABC,ACD,ABD,AEG,AGI,AEI,BEF,EFJ,GHK,ELN,
GIP,KOP,JMN,LMQ,JQR,OPS,PGI,QRT,RST,QST,
J LET,CDK,EBJ,PKS,LNT,QJT,ADK,ALP,DHI,PTL
I = 30

Here the figure given has 4 original figures given below: 36. Find the number of triangles in the given figure.
Figure (1) – ABCD nh xbZ vk—fr esa f=Hkqtksa dh la[;k Kkr dhft,A
Figure (2) – CDEF
Figure (3) – EFGH
Figure (4) – CFIJ
Number of original triangles in all the figures =
4×(4×2)=4×8=32
2 new triangles are formed by connecting figure (1) and (2) : Δ (a) 28 (b) 32 (c) 30 (d) 26
ACE and Δ BDF (SSC CGL Pre. 2020, 20.08.2021 Shift-2)
2 new triangles are formed by connecting figure (2) and (3) : Δ Sol. (b)
DFH and ΔCEG
2 new triangles are formed by connecting figure (2) and (4) : Δ
DFJ and Δ CEI
In addition one triangle ΔAEI is formed by connecting figure
(1), (2) and (4).
ΔDHJ is formed by connecting figure (2), (3) and (4). Finally
one triangle Δ AHK is formed by connecting all the figures.
Hence, total triangles = 32+9 = 41
36. How many triangles are there in the given figure?
nh xbZ vkd`fr esa fdrus f=Hkqt gS\

80
Best App for Govt. Jobs : Neonclasses (Download Now)

(SSC CPO 2019, 09.12.2019 Shift-1)


Sol. (b)

Part (A) Part (B) Part (C)

3
16 16+[(3+3)+6] 3
8 3 6


3 2
16 3+3=6 6+2 = 8


Combination
16
28+4 = 32 +1
+1 32+1 = 33
Correct answer is option(b)
=
37. How many triangles are there in the following figure?
18
fuEufyf[kr vk—fr esa fdrus f=Hkqt gSa\
)
Correct answer is option (b)
39. Find the number of triangles in the given figure.
nh xbZ vk—fr esa f=Hkqtksa dh la[;k Kkr dhft,A

(a) 33 (b) 18 (c) 31 (d) 29


(SSC CGL Pre. Exam. 10.06.2019 Shfit - 3) (a) 20 (b) 22 (c) 23 (d) 24
Sol. (c) (SSC CGL Pre. 2020, 17.08.2021 Shift-1)
Part (A) Sol. (d)
Part (B)

3 3+13 16+3+3

5
8+4 22+2 = 24

Part (C) Correct answer is option (d)


Combination 40. Find out the number of triangles in the given figure?
4

29+1+1 = 31
Correct answer is
(a) 20 (b) 22 (c) 28 (d) 24
option (C)
Sol. (c)
38. How many triangles are there in the given figure?
nh xbZ vk—fr esa fdrus f=Hkqt gSa\ C
D
2
1 3
1H 4 G
A F
2

B E

Here the figure given has 4 original figure given below:


Figure (1) – ΔABC
(a) 30 (b) 33 (c) 25 (d) 32
Figure (2) – ΔDEF

81
Download Free PDFs & e-Books from Neon Classes App

Figure (3) – BEGH Apart from this the figure is made up of five lines (AB, BC, CD,
Figure (4) – CDGH DE AND EA).
Number of triangle in figure (1) and (2) = 2×(1+2)=2×3=6 Each line will form one new triangle. Hence 5 new triangles
Number of triangles in figure (3) and (4) = 2(4×2) = 2×8=16 are formed which are as follows:
In addition the following new triangles will be formed. (1) Δ ABO (2) ΔBCP
Δ CGB and Δ DHE – by connecting will be formed 2 triangle. (3) ΔCDQ (4) ΔDER
Δ ACG – by connecting figure (1) and (4) 1 triangle formed. (5) Δ EAS
Δ ABG – by connecting figure (1) and (3) 1 triangle formed. 44. Find out the number of triangles in the given figure?
Δ HDF – by connecting figure (2) and (4) 1 triangle formed.
Δ EFH – by connecting figure (2) and (3) 1 triangle formed.
Hence, total triangles = 6+16+6=28
41. Find out the number of triangles in the given figure?

(a) 10 (b) 8 (c) 6 (d) 7


Sol. (b)
A

E 1 F
(a) 33 (b) 29 (c) 28 (d) 31 6 2
Sol. (b)
B F 5 3
B 4 C
D
I H
D There are 6 small triangles in the given figure. In addition
there are two big triangles:
A E G Δ ABC and ΔDEF
C
Hence, total triangles = 6+2=8
Here the figure given has 3 original figures: Case-3 Basic type-III
Figure (1) – ΔABC 45. Find out the number of triangles in the given figure?
Figure (2) – ΔEFG
Figure (3) – BCDEF
Number of triangles in figure (1) (2) = 2× 7 = 14
Number of triangles in figures (3) = 11
In addition the following new triangles will be formed.
(a) 5 (b) 7 (c) 6 (d) 4
Δ IBD and ΔICD – by connecting figure (1) and (3)
Sol. (a)
Δ DFH and Δ DEH-by connecting figure (3) and (2)
Hence, total triangles = 14+11+4 = 29 A
Note: The figure (1) and (2) are similar to the figure given in
example (51) and figure (3) is similar to the figure given in D E
example (50).
43. Find out the number of triangles in the given figure?
B C
In this pattern there are two formula to find out total number
of triangles:
(1) If the number or horizontal lines (n) is even:
(n(n  2)(2n  1)
Number of triangles =
8
(a) 8 (b) 7 (c) 10 (d) 12 n = number of horizontal lines
Sol. (c) (2) If the number of horizontal lines (n) is odd.
D (n(n  2)(2n  1)  1
Number of triangles =
8
P 2 S The figure given in the question has two horizontal lines (DE,
A B
1 3 BC).
R Q (n(n  2)(2n  1)
5 4 Hence, Number of triangles =
8
O
(2(2  2)(2  2  1)
C E 
8
There are 5 small triangles in the given figure. 2 4 5
 5
8

82
Best App for Govt. Jobs : Neonclasses (Download Now)

46. Find out the number of triangles in the given figure? A

D E

F G
(a) 27 (b) 26 (c) 25 (d) 30 H I

Sol. (a) J K
A
B C
D E
Number of horizontal lines (n) = 5
F G (DE, FG, HU, JK, BC)
H I
(n(n  2)(2n  1)
Hence, total triangles =
B C 8
Number of horizontal lines (n) = 4 [5( s  2)(2  5  1)]  1

(BC, HI, FG, DE) 8
(n(n  2)(2n  1) [5  7  11]  1
Number of triangles = 
8 8
(4(4  2)(2  4  1) 385  1
 
8 8
469 384
  27   48
8 8
Note: The number of triangles in this figure only, can be find Concept-2
out using the following trick: Square and rectangle and Rhombuses
1
48. Answer the questions related to the following figure:
3
2 4

6 8
5 7 9

11 13 15
10 12 14 16

First of all, we will number all the individual triangles given in


the figure. After it, we will find out the number of triangles as
given below: (i) Find out the number of total squares?
Total triangles = (number of triangles×2)-5 (a) 4 (b) 5 (c) 6 (d) 9
Hence, total triangles = (16×2)-5=27 (ii) Find out the total number of rectangles?
Note: (a) 6 (b) 7 (c) 8 (d) 9
(1) In this question we can find out number of triangles using (iii) Find out the number of only rectangles?
the following trick: (a) 4 (b) 5 (c) 6 (d)7
Number of triangles = (2×9)-5=13 Sol. First we will number the columns and rows in the given figure.
1 2
1
3
34
2
(i) Now take the largest numbers in rows and columns and
6 7 8 multiply them. After it subtract one from those numbers and
5 9
multiply them. Thus keep subtracting 1 from the numbers
obtained till one number out of the two numbers doesn’t
(2) Keep in mind that this trick can be used only for the figures
become 1. Now we will get total number of squares by adding
given in example-(61) and (62).
all these values obtained.
47. Find out the number of triangles in the given figure?
Number of squares = (2×2)+(1×1)
4+1 = 5
(ii) To find out the total number of rectangle add all the
numbers in the columns. Finally multiply both numbers
obtained.
Number of rectangles = (1+2)×(1+2)
= 3×3=9
(iii) Total rectangles include squared also because square is
also a rectangle.
(a) 41 (b) 50 (c) 43 (d) 48 Hence the number of only rectangles = total rectangles-Total
Sol. (d) squares. 9-5=4
49. Find out the number of total squares in the given figure?

83
Download Free PDFs & e-Books from Neon Classes App

(a) 14 (b) 13 (c) 12 (d) 16


(SSC CGL Pre. Exam 04.06.2019 Shift- 1)
Sol. (a)
(a) 55 (b) 60 (c) 50 (d) 70
    
Sol. (a)

1 1 1+4 5+4 9+4 13+0


2 3 4 5 13+1= 14

2 Correct answer is option (a)


53. How many squares are there in the given figure?
3 nh xbZ vk—fr esa fdrus oxZ gSa\
4
5
Total squares = (5×5)+(4×4)+(3×3)+(2×2)+(1×1)
= 25+16+9+4+1 = 55
50. How many squares are there in the following figure?
fuEufyf[kr vk—fr esa fdrus oxZ gSa\

(a) 37 (b) 21
(c) 36 (d) 20
(SSC CHSL Pre. 2021, 02.06.2022 Shift-2)
Sol. (d)
1+4=5
(a) 8 (b) 14 1+4=5
(c) 10 (d) 12  
(SSC CPO 2018, 16.03.2019 Shift-2)
Sol. (c) 1 1+4
1+4=5 1+4=5
Correct answr is option(d)
   54. How many squares are there in the given figure?
nh xbZ vk—fr esa fdrus oxZ gSa\
1 1+4 5+1 6+4 = 10
Correct answer is option (c)
51. Find the number of squares in the following figure.
fuEufyf[kr vk—fr esa oxksZa dh la[;k Kkr dhft,A

(a) 13 (b) 15 (c) 12 (d) 14


(SSC CHSL Pre. 2021, 31.05.2022 Shift-1)
(a) 6 (b) 4 Sol. (d)
(c) 9 (d) 7 1 2 3
(SSC CPO 2018, 16.03.2019 Shift-3)
Sol. (d) 1
3 39
2  2 24
1 11
    3 Total = 14

1 1+0 Correct answer is option (d)


1+1 2+1 3+4 = 7 55. How many rectangles are there in the given figure?
nh xbZ vk—fr esa fdrus vk;r gSa\
Correct answer is option (d)
52. How many squares are there in the following figure?
fuEufyf[kr vk—fr esa fdrus oxZ gSa\
(a) 10 (b) 14 (c) 8 (d) 12

84
Best App for Govt. Jobs : Neonclasses (Download Now)

(SSC CGL Pre. 2020, 17.08.2021 Shift-3) = (in the original figure)+(other rectangles)
Sol. (b) = 9+12=21
58. How many rectangles are there in the following figure?
1+2+3+4 =10
fuEufyf[kr vk—fr esa fdrus vk;r gSa\

10+1+1+1+1 =14

Correct answer is option (b)


56. Find out the number of squares in the given figure? (a) 7 (b) 8 (c) 6 (d) 9
(SSC Constable GD 2021, 08.12.2021, Shift-2)
Sol. (d)

 1+2+2+1

(a) 25 (b) 28 (c) 14 (d) 21  1


Sol. (d)

5
 1+1
5
5
Total = 9

5 Correct answer is option (d)


5 59. How many rectangles are there in the given figure?
nh xbZ vk—fr esa fdrus vk;r gSa\
Hence, we will get (5×5)=25 squares from such five figures
“⊞”.
But the four shaded squares which are common have been
counted twice. Therefore, we have to subtract such squares
one time.
Hence, total squares = 25-4 = 21 (a) 34 (b) 35 (c) 32 (d) 30
57. (SSC CGL Pre. Exam. 03.03.2020 Shit - 1)
Sol. (b)
Combination –
1 1 1
Part (A) Part (B) Part (C)
2 2 2
(i) Find out the number only rectangles? 3 3 3
(a) 14 (b) 16 (c) 20 (d) 12 4
6 6
(ii) Find out the number total rectangles? 5
(a) 21 (b) 20 (c) 18 (d) 24 6
Sol. In the figure “⊞”
 21
Total squares = 5
Total rectangles = 9 Total  6+21+6 = 33
Only rectangles = 9-5 = 4
Combination part-(A+B) Combination part-(B+C)
A E O B
A A A A
M N
A R AZ
H A F
A
A P A
A K
J
A
D
A
S G C
1 1
A A A A
There are 4 lines OP, JK, RS and MN in figure ABCD. We will get  33+1+1
3 new rectangles from line OP. 35
(i) EOPZ (i) OBFP (iii) AOPH
Similarly we will get 3-3 rectangles from other three lines. Correct answer is option (b)
Hence, rectangles from these 4 lines = 4×3 = 12 60. How many rectangles are there in the given figure?
(i) Only rectangles nh xbZ vk—fr esa fdrus vk;r gSa\
= (Number of only rectangles in the original figure)+(Other
rectangles)
= 4+12=16
(ii) Total rectangles

85
Download Free PDFs & e-Books from Neon Classes App

   
1 1+1
2+1
3+1 4+1 = 5
Correct answer is option (d)
. 63. Find the number of square in the given figure. (Rectangles
(a) 15 (b) 13 (c) 17 (d) 11 NOT to be included)
(SSC CGL Pre. Exam. 04.03.2020 Shit - 3) nh xbZ vk—fr esa oxksZa dh la[;k Kkr dhft,A ¼vk;rksa dks 'kkfey
Sol. (c) ugha fd;k tkuk pkfg,½
   

1 1+2 1+2+3 1+2+3+4

(a) 16 (b) 7 (c) 9 (d) 5


(SSC CGL Pre. 2020, 16.08.2021 Shift-2)
 Total= 17 Sol. (b)

10+2+3 15+2    

Correct answer is option (c)


61. How many squares are there in the given figure? 1 1+1 2+1 3+3 6+1 = 7
nh xbZ vk—fr esa fdrus oxZ gSa\
Correct answer is option (b)
64. Find out the number of rectangles in the given figures?

(a) 33 (b) 35 (c) 31 (d) 36


Sol. (a)
(a) 20 (b) 18 (c) 12 (d) 24
(SSC CHSL Pre. 2021, 26.05.2022 Shift-2) 21 rectangles
Sol. (b)
AE B
1 2 D1
H F
1 2
D
D G 3 D C 3 rectangles
2   
3 D D 4 T
P
4 5 D Q
3 rectangles W U
6
4 28 11+3 14+3 17+1 DR
S V
3 13 = 18 D
Total = 11 Here total triangleDis 21+3+3+3+3=33
65. How many triangles are there in the following figure?
Correct answer is option (b) fuEufyf[kr vk—fr esa fdrus f=Hkqt gSa\
62. How many squares are there in the given figure?
nh xbZ vk—fr esa fdrus oxZ gSa\

(a) 20 (b) 16
(c) 14 (d) 12
Sol. (b)

(a) 22 (b) 7 (c) 6 (d) 5


(SSC CHSL Pre. 2021, 01.06.2022 Shift-3)
Sol. (d)

86
Best App for Govt. Jobs : Neonclasses (Download Now)

=The simplest squares are ABED, BCFE, DEHG, EFIH, GHKJ and
HILK i.e., 6 in number.
 The square composed of four simple squares are ACIG and
DELJ i.e., 2 in number.
Thus, 6 + 2 = 8 squares will be formed.
10 10+6 = 16 B
A C
Correct answer is option (b)
D E F
66. Find the number of square in the given figure.
H
G I

J L
K
68. How many squares are there in the given figure.

(a) 22 (b) 20 (c) 18 (d) 14


Sol. (c)
W X
O P
E B F
V Y
I J
N Q
A M C
U R
L K
H G Z (a) 10 (b) 11 (c) 12 (d) 14
C1 D
T S Sol. (d)
B1 A1
The figure may be labeled as shown. B
A
The squares composed of two components each are BJMI,
I F
CKMJ, DLMK and AIML i.e., 4 in number.
The squares composed of three components each are EBMA, E L
C
BFCM, MCGD and AMDH i.e., 4 in number.
D
The squares composed of four components each are VWBA,
XYCB, ZA1DC and B1C1AD i.e., 4 in number.
The squares composed of seven components each are NOJL, H J G K
PQKI, RSLJ and TUIK i.e., 4 in number.
There are 10 small squares
There is only one square i.e., ABCD composed of eight
4 Squares : ABCD, EFGH, IJKL & ICJD
components.
 Total squares = 10 + 4 = 14
There is only one square i.e., EFGH composed of twelve
69. How many rhombuses are there in the given diagram ?
components.
∴ Total number of squares in the figure
= 4 + 4 + 4 + 4 + 1 + 1 = 18.

Directions (Q 67 to 69) : In each of the following questions,


count the number of triangles and squares in the given figure.
fuEufyf[kr çR;sd ç'u esa] fn, x, vkadM+s esa f=dks.k vkSj oxksaZ dh
la[;k dh x.kuk djsAa
67. In the adjoining figure, if the centres of all the circles are joined
by horizontal and vertical lines, then find the number of
squares that can be formed. (a) 4 (b) 1 (c) 5 (d) 6
fudVorÊ vk—fr es]a ;fn lHkh o`Ùkksa ds dsæa {kSfrt vkSj yacor js[kkvksa Sol. (C)
ls tqM+rs gSa] rks cu ldus okys oxksaZ dh la[;k Kkr dhft,A

(a) 6 (b) 7 (c) 8 (d) 1


Sol. (c)
We shall join the centres of all the circles by horizontal and
vertical lines and then label the resulting figure as shown.

87
Download Free PDFs & e-Books from Neon Classes App

B Correct answer is option (d)


Concept – 4 Cube
In this we have to find out the number of cubes in the given figure.
G Such cubes can be seen in the figure or can be hidden in the
F
figure. We have to find out both types of cubes.
I
73. Find out the number of cubes in the given figure ?
A C

E H

D
5 rhombuses : ABCD, AFIE, FBGI, IGCH, EIHD
Concept – 3 Circle
There is no trick to find out the number of circle in a given
figure. Also, Keep in mind that no new circle is formed by
adding two or more figures. Therefore, the number of circles
will be found out directly.
70. Find out the number of circle in the given figure ?
(a) 1 (b) 3 (c) 4 (d) 5
Sol : (c) 4

(a) 9 (b) 10 (c) 12 (d) 13


Sol. (b) 10
4 1 1
3
2

1
Number of Circles in the given figure = 4 + 3 + 2 + 1 = 10 Some cubes are visible in the figure given here, while some
cubes are hidden. We have to count both types f cubes to find
71. Find out the number of circle in the given figure ? out total number of cubes
(1) Cubes that can be seen :
If we divide the cubes on the basis of the floor in the given figure,
then two cubes are visible on the first floor and one cube is
visible on the second floor.
Hence, number of cubes which can be seen = 3
(a) 16 (b) 20 (c) 18 (d) 21 Cubes that are hidden :
Sol. There are 6 big circles in the given figure. In addition , each big (2) Cubes that are hidden
circle has 2 small circles in it. Here we see as many cubes on the second floor, the same number
Hence, Total circles = 6 + (6 × 2) of cubes will be hidden on the first floor as well.
= 6 + 12 = 18 Hence, number of cubes which are hidden = 1
72. How many circles are there in the given figure? Total cubes = 3 + 1 = 4 Ans. (c)
This question can also be solved as given below Total cubes :
On the top surfaces of the cubes shown , we will write the number
of the floor on which such a cube is placed. After it we will find
total number of cubes by adding all such number.
We have written the number on the top surfaces of cubes in the
above mentioned figure.
 Total cubes = 2 + 1 + 1 = 4
(a) 11 (b) 9
(c) 12 d) 10 74. Find out the number of cubes in the given figure ?
Sol. (d)
1
r=
10

88
Best App for Govt. Jobs : Neonclasses (Download Now)

(a) 10 (b) 12
(c) 8 d) 9
Sol. : (a) 10

3 (1) What is the number of total cubes ?


(a) 14 (b) 18 (c) 20 (d) 10
(2) How many cubes are there which can be seen ?
(a) 14 (b) 10 (c) 12 (d) 8
(3) How many cubes are there which are hidden ?
2 2 (a) 10 (b) 12 (c) 8 (d) 14
Sol : Here we have written the numbers on the top surfaces of the
cubes which are visible. These numbers are written on the
basis of visible cubes and hidden cubes.

1 1 1
4

The number of total cubes in the given figure will be found out 3 3
as follows : 4
There are 3 cubes on the first floor and there is no cube hidden
below them. So we will write “1” on the top surfaces f all the
cubes placed on first floor. There can be seen 2 cubes on the
second floor and there will be 1 – 1 cube hidden on the first 2 2 2
floor below them. So we will write “2” on the top surfaces of all
the cubes placed n second floor.
There can be seen 1 cube on the third floor and there will be 1
– 1 cube hidden on the second and the first floor below it. So
we will write “3” on the top surface of the cube placed on the
1 1 1
third floor. 1
Total cubes = 3 + 2 + 2 + 1 + 1 +1 = 10 Ans. (A)
Number of cubes that can be seen = 6
Hidden cubes = Total cubes – cubes that can be seen = 10 – 6 =
4
75. Answer the following question based on the given figure :
(1) Hence total cubes = 4 + 3 + 3 +2 + 2+ 2+ 1 + 1 +1 + 1 = 20
Or
Total cubes = (1×4)+(2×3)+(3×2)+(4×1)

89
Download Free PDFs & e-Books from Neon Classes App

= 4 + 6+ 6+ 4 = 20 Ans. (C) A
(2) Cubes that can be seen = 10 Ans. (B)
(3) Hidden cubes = Total cubes – cubes that can be seen = 20 – 10
= 10
Concept – 5 Straight Line
E H
When we have to find the number of straight lines in a figure, we
count three types of lines in it :
(1) Horizontal Line
(2) Vertical Line B D
(3) Slant line
76. Find out the number of straight lines in the given figure ?

F G

Number of straight lines in the given figure :


(1)Horizontal lines = 3 (EH,BD,FG)
(a) 7 (b) 8 (c) 6 (d) 5 (2)Vertical lines = 3 (EF,AC, HG)
Sol : (a) 7 (3)Slant lines = 8 (AE, AH, HD, DG, GC, CF, BE, BF)
Total Straight lines = 3 + 3 + 8 = 14 Ans. (d)
A D 78. Identify the number of straight lines in the given figure.

E F

B C
(a) 15 (b) 17 (c) 19 (d)20 (SSC CPO 2019,
Number of straight lines in the given figure : 12.12.2019 Shift-2)
(1)Horizontal lines = 3 (AD,EF,BC) Sol. (b)
(2)Vertical lines = 2 (AB,DC) (I) Vertical lines –  8
(3)Slant lines = 2 (AC, BD)
Total Straight lines = 3 + 2 + 2 = 7 Ans. (a) (II) Horizontal lines–  9
Total -17
77. Find out the number of straight lines in the given figure ?

P O

L K
D C H G
Q R
E F
A B
I J

M N

Toatl line  AB, BC,CD,DA,EF,FG,GH,HE,IJ,JK,KL,LI,QR,MN, NO, OP,


PM  17
Correct answer is option (b)
Concept – 6 Number of Diagonals
(a) 20 (b) 10 (c) 18 (d) 14
Sol. (d) 14 n(n  3)
The number of diagonals in a regular polygon =
2
79. Find out the number of diagonals in a square ?
(a) 1 (b) 0 (c) 2 (d) 3
Sol : (c) 2

90
Best App for Govt. Jobs : Neonclasses (Download Now)

A square has four sides (d) Statement II alone is sufficient, while Statement I alone is
Hence n = 4 not sufficient to answer the question
n(n  3) vdsys dFku II i;kZIr gS] tcfd vdsys dFku I vdsys ç'u dk mÙkj
Number of diagonals =
2 nsus ds fy, i;kZIr ugha gS
4(4  3) Sol. (c)
2
2
80. Find out the number of diagonals in a hexagon?
(a) 8 (b) 9 (c) 10 (d) 11
Sol : (b) 9
A hexagon has six sides n = 6
n(n  3)
Number of diagonals =
2
6(6  3)
=
2
63
= =9
2
81. Find out the number of diagonals in a triangle ?
(a) 1 (b) 0 (c) 2 (d) 3
Sol : (b) 0
A triangle has three sides.  Concept - 2
Hence n = 3 Ranking based
n(n  3) 2. A question is given, followed by three statements labeled
Number of diagonals =
2 I, II and III. Identify which of the statements is/are
3(3  3) sufficient to answer the question.
0
2 ,d ç'u fn;k x;k gS] ftlds ckn I] II vkSj III yscy okys rhu
dFku fn, x, gSaA igpkusa fd dkSu lk@ls dFku ç'u dk mÙkj nsus
ds fy, i;kZIr gS@gSaA
Question:
Who ranked first in the class?
Data sufficiency d{kk esa çFke LFkku ij dkSu jgk\
 Concept - 1 Statements:
I. Sreeja ranked 2nd
Coding based
Jhtk nwljs LFkku ij jgha
1. A question is given, followed by two statements labeled I
II. Aman rnaked one rank below Sreeja.
and II. Identify which of the statements is/are sufficient to
veu Jhtk ls ,d jSad uhps gSA
answer the question.
III. Rohit ranked better than Aman.
,d ç'u fn;k x;k gS] ftlds ckn I vkSj II yscy okys nks dFku fn,
jksfgr veu ls csgrj LFkku ij gSA
x, gSaA igpkusa fd dkSu lk@ls dFku ç'u dk mÙkj nsus ds fy,
(a) Each statement I, II, and III alone is sufficient
i;kZIr gS@gSaA
çR;sd dFku I] II vkSj III vdsys i;kZIr gS
Question:
(b) All statements I, II and III together are sufficient.
In a certain code language, ‘pu ma kas’ means ‘we eat
lHkh dFku I] II vkSj III ,d lkFk i;kZIr gSaA
mangoes’. What is the code of ‘mangoes’?
(c) Statement II alone is sufficient, while statement III alone is
,d fuf'pr dwV Hkk"kk esa] 'pu ma kas' dks ‘we eat mangoes’ dgk
not sufficient.
tkrk gSA ‘mangoes’ dk dwV D;k gS\
vdsys dFku II i;kZIr gS] tcfd vdsys dFku III i;kZIr ugha gSA
Statements :
(d) Statement I alone is sufficient, while statement II alone is
I. ‘la ti kas’’ means ‘we are graduate’.
not sufficient.
'la ti kas' dk vFkZ gS ‘we are graduate’
vdsys dFku I i;kZIr gS] tcfd vdsys dFku II i;kZIr ugha gSA
II. ‘hum tup pu’ means ‘they eat bananas’.
Sol. (b)
'hum tup pu' dk vFkZ gS ‘they eat bananas’A
(a) Statement I alone is sufficient, while Statement II alone is
not sufficient to answer the question
vdsys dFku I i;kZIr gS] tcfd vdsys dFku II ç'u dk mÙkj nsus ds
fy, i;kZIr ugha gS
(b) Statements I and II together are not sufficient to answer
the question.
dFku I vkSj II ,d lkFk ç'u dk mÙkj nsus ds fy, i;kZIr ugha gSaA
(c) Statement I and II together are sufficient to answer the
question.
dFku I vkSj II ,d lkFk ç'u dk mÙkj nsus ds fy, i;kZIr gSa

91
Download Free PDFs & e-Books from Neon Classes App

Given: D
Statement I: Sreeja’s rank is 2nd 
P-  L +
statement III: D is mother of S and T is the son-in-law of C.
Sreeja C D-
Statement II. Aman ranked one rank  
below Sreeja. It means his rank is 3rd -
 T+ S

The relation between P and C cannot be determined by using


Sreeja Aman any combination of the statements or all the statements
together. Hence, Data in statements I, II, and III together are
Statement III. Rohit ranked better
not sufficient to answer the question.
than Aman. It means his rank is 1st.
 Concept - 4
Comparision based
4. A question is given, followed by two statements labeled I
Rohit Sreeja Aman
and II. Identify which of the statements is/are
Hence, All statements I, II and III together are sufficient. sufficient/necessary to answer the question.
 Concept - 3 ,d ç'u fn;k x;k gS] ftlds ckn I vkSj II yscy okys nks dFku fn,
x, gSaA igpkusa fd dkSu&lk@ls dFku ç'u dk mÙkj nsus ds fy,
Blood relation
i;kZIr@vko';d gSa@gSaA
3. A question is given, followed by two statements labeled I
Question:
and II. Identify which of the statements is/are
Who is the shortest among Shamita, Tanya and Rekha?
sufficient/necessary to answer the question.
'kferk] rkU;k vkSj js[kk esa lcls NksVk dkSu gS\
,d ç'u fn;k x;k gS] ftlds ckn I vkSj II yscy okys nks dFku fn,
Statements :
x, gSaA igpkusa fd dkSu&lk@ls dFku ç'u dk mÙkj nsus ds fy,
I. Shamita is taller than Rekha. Rekha is shorter than Tanya.
i;kZIr@vko';d gSa@gSaA 'kferk js[kk ls yach gSA js[kk rkU;k ls NksVh gSA
Question:
II. Rekha is shorter than Surbhi. Surbhi is taller than
How is P related to C?
Shamita. Tanya is shorter than Surbhi.
P] C ls fdl çdkj lacfa èkr gS\
js[kk lqjfHk ls NksVh gSA lqjfHk 'kferk ls yach gSA rkU;k lqjfHk ls NksVh
Statements :
gSA
I. H is the only brother of S and P.
(a) Statement I alone is sufficient to answer the question,
H, S vkSj P dk bdykSrk HkkbZ gSSA
whereas statement II alone is not sufficient to answer the
II. P is the wife of L, who is the son – in law of D.
question
P, L dh iRuh gS] tks D dk nkekn gSA
dFku I vdsys ç'u dk mÙkj nsus ds fy, i;kZIr gS] tcfd dFku II
III. D is the mother of S and T is the son in law of C.
vdsys ç'u dk mÙkj nsus ds fy, i;kZIr ugha gSA
D, S dh ekrk gS vkSj T] C dk nkekn gSA
(b) Statement II alone is sufficient to answer the question,
(a) Data in statements I, II and III together are not sufficient to
whereas statement I alone is not sufficient to answer the
answer the question.
question
dFku I, II vkSj III esa MsVk ,d lkFk ç'u dk mÙkj nsus ds fy, i;kZIr vdsys dFku II ç'u dk mÙkj nsus ds fy, i;kZIr gS] tcfd dsoy
ugha gSaA dFku I vdsys ç'u dk mÙkj nsus ds fy, i;kZIr ugha gSA
(b) Data in statements II and III together are sufficient to
(c) Both statements I and II together are not sufficient to
answer the question.
answer the question
dFku II vkSj III esa MsVk ,d lkFk ç'u dk mÙkj nsus ds fy, i;kZIr gSaA nksuksa dFku I vkSj II ,d lkFk ç'u dk mÙkj nsus ds fy, i;kZIr ugha
(c) Data in statements I , II and III together are sufficient to
gSaA
answer the question.
(d) Both Statements I and II together are necessary to answer
dFku I, II vkSj III esa MsVk ,d lkFk ç'u dk mÙkj nsus ds fy, i;kZIr
the question
gSaA ç'u dk mÙkj nsus ds fy, dFku I vkSj II nksuksa ,d lkFk vko';d gSaA
(d) Data in statement I alone is sufficient to answer the
Sol. (a)
question
Statement 1: shamita is taller than Rekha. Rekha is shorter
dsoy dFku I esa MsVk ç'u dk mÙkj nsus ds fy, i;kZIr gSSA than Tanya.
Sol. (a) This implies Rekha < Tanya/Shamita.
statement I: H is the only brother of S and P. Statement 2:
H+  S-  P- Rekha is shorter than Surbhi.
statement II: P is the wife of L, who is son-in-law of D. Rekha < Surbhi
Surbhi is taller than Shamita.
Surbhi < Shamita
Tanya is shorter than Surbhi.

92
Best App for Govt. Jobs : Neonclasses (Download Now)

Tanya < Surbhi vdsys dFku I ;k dFku II esa fn, x, MsVk ç'u dk mÙkj nsus ds fy,
Combining all these, i;kZIr gSaA
Rekha/Tanya < Surbhi < Shamita Sol. (e)
The shortest person can’t be determined as per statement 2. From I, we conclude that 5th, 12th, and 26th of March, 2006
So, Statement 1 alone is sufficient to answer the question. were Sundays. So the last Sunday fell on 26th. From II, we
 Concept - 5 conclude that 31st March, 2006 was Friday. Thus, 26th March
2006 was the last Sunday of the month.
Sitting based
5. How many children are there between P and Q in a row of  Concept - 7
children? Number system
cPPkksa dh ,d iafDr esa P vkSj Q ds chp fdrus cPps gS \ 7. How many marks did Nigam get in Biology?
Statements: dFku % thofoKku esa fuxe dks fdrus vad feys\
(i) P is fifteenth from the left in the row. I. Nigam got 42 marks in English which were half the marks
P iafDr esa ck;s ls 15ok¡ gSA he got in Biology.
(ii) Q is exactly in the middle and there are ten children fuxe dks vaxzsth esa 42 vad feys tks fd tho foKku esa mls feys
towards his right. vadks ds vkèks vad FksA
Q iafDr ds Bhd chp esa gS rFkk mlds nk;sa 10 cPps gSA II. Nigam’s marks in Biology were 14% of the total marks he
(a) II alone is sufficient, while I alone is not sufficient got in all the subjects together.
vdsys II i;kZIr gS] tcfd I vdsyk i;kZIr ugha gS thofoKku esa fuxe ds vad dqy vadksa dk 14% Fks tks mls lHkh fo"k;ksa
(b) I alone is sufficient, while II alone is not sufficient esa ,d lkFk feys FksA
vdsys I i;kZIr gS] tcfd vdsys II i;kZIr ugha gS (a) The data given in Statement I alone are sufficient to answer
(c) Both I and II together are not sufficient the question whereas the data given in Statement II alone are
I vkSj II nksuksa ,d lkFk i;kZIr ugha gSa not sufficient to answer the question.
(d) Both I and II together are sufficient dFku I esa fn, x, vkadM+s vdsys ç'u dk mÙkj nsus ds fy, i;kZIr gSa
I vkSj II nksuksa ,d lkFk i;kZIr gSa tcfd dFku II esa fn, x, vkadM+s vdsys ç'u dk mÙkj nsus ds fy,
Sol. (d) i;kZIr ugha gSaA
From II, Q being in the middle, there are 10 children to his (b) The data given in Statement II alone are sufficient to
right as well as to his left. So, Q is 11th from the left. answer the question whereas the data given in Statement I
From I, P is 15th from the left. alone are not sufficient to answer the question.
Thus, from both I and II, we conclude that there are 3 children vdsys dFku II esa fn, x, MsVk vdsys ç'u dk mÙkj nsus ds fy,
between P and Q. i;kZIr gSa tcfd dFku I esa fn, x, MsVk vdsys ç'u dk mÙkj nsus ds
 Concept - 6 fy, i;kZIr ugha gSaA
(c) The data in either Statement I alone or in Statement II
Calendar based
alone are not sufficient to answer the question.
6. The last Sunday of March, 2006 fell on which date?
vdsys dFku I ;k dFku II esa fn, x, MsVk ç'u dk mÙkj nsus ds fy,
ekpZ] 2006 esa vafre jfookj dkSulh rkjh[k dks Fkk \
i;kZIr ugha gSaA
Statements: dFku %
(d) The data in both Statement I and II are not sufficient to
I. The first Sunday of that month fell on 5th.
answer the question.
ml efgus esa igyk jfookj 5 rkjh[k dks FkkA
dFku I vkSj II nksuksa dk MsVk ç'u dk mÙkj nsus ds fy, i;kZIr ugha gSA
II. The last day of that month was Friday.
(e) The data given in both the Statement I and II are necessary
ml efgus esa vafre fnu ’kqØokj FkkA
to answer the question.
(a) Statement I alone is sufficient to answer the question,
dFku I vkSj II nksuksa esa fn, x, MsVk ç'u dk mÙkj nsus ds fy,
whereas statement II alone is not sufficient to answer the
vko';d gSaA
question
Sol. (a)
dFku I vdsys ç'u dk mÙkj nsus ds fy, i;kZIr gS] tcfd dFku II
From Statement I,
vdsys ç'u dk mÙkj nsus ds fy, i;kZIr ugha gSA
Nigam’s marks in Biology = 2 × 42 = 84
(b) Statement II alone is sufficient to answer the question,
Hence, only Statement I is sufficient.
whereas statement I alone is not sufficient to answer the
question  Concept - 08
vdsys dFku II ç'u dk mÙkj nsus ds fy, i;kZIr gS] tcfd dsoy Numbers and digits
dFku I vdsys ç'u dk mÙkj nsus ds fy, i;kZIr ugha gSA 8. What is the two digits number?
(c) Both statements I and II together are not sufficient to nks vadksa dh la[;k D;k gS\
answer the question I. The difference between the two digits of the number is 0.
nksuksa dFku I vkSj II ,d lkFk ç'u dk mÙkj nsus ds fy, i;kZIr ugha la[;k ds nks vadksa ds chp dk varj 0 gSA
gSaA II. The sum of the two digits of the number is 18.
(d) Both Statements I and II together are necessary to answer la[;k ds nks vadksa dk ;ksx 18 gSA
the question (a) The data given in Statement I alone are sufficient to answer
ç'u dk mÙkj nsus ds fy, dFku I vkSj II nksuksa ,d lkFk vko';d gSaA the question whereas the data given in Statement II alone are
(e) The data in Statement I alone or in Statement II alone are not sufficient to answer the question.
sufficient to answer the question.

93
Download Free PDFs & e-Books from Neon Classes App

dFku I esa fn, x, vkadM+s vdsys ç'u dk mÙkj nsus ds fy, i;kZIr gSa (b) The Statement II alone is sufficient to answer the question,
tcfd dFku II esa fn, x, vkadM+s vdsys ç'u dk mÙkj nsus ds fy, but the Statement I alone is not sufficient.
i;kZIr ugha gSaA dFku II vdsys ç'u dk mÙkj nsus ds fy, i;kZIr gS] ysfdu dsoy
(b) The data given in Statement II alone are sufficient to dFku I i;kZIr ugha gSA
answer the question whereas the data given in Statement I (c) Both Statements I and II together are needed to answer the
alone are not sufficient to answer the question. question.
vdsys dFku II esa fn, x, MsVk vdsys ç'u dk mÙkj nsus ds fy, ç'u ds mÙkj ds fy, I vkSj II nksuksa dFku ,d lkFk vko';d gSaA
i;kZIr gSa tcfd dFku I esa fn, x, MsVk vdsys ç'u dk mÙkj nsus ds (d) Either the Statement I alone or Statement II alone is
fy, i;kZIr ugha gSaA sufficient to answer the question.
(c) The data in either Statement I alone or in Statement II ;k rks dFku I ;k dFku II vdsys gh ç'u dk mÙkj nsus ds fy, i;kZIr
alone are not sufficient to answer the question. gSA
vdsys dFku I ;k dFku II esa fn, x, MsVk vdsys ç'u dk mÙkj nsus ds (e) You cannot get the answer from the Statements I and II
fy, i;kZIr ugha gSaA together, but need even more data.
(d) The data in both Statement I and II are not sufficient to vkidks fooj.k I vkSj II dk mÙkj ,d lkFk ugha fey ldrk gS] ysfdu
answer the question. vkSj Hkh vfèkd MsVk dh vko';drk gSA
dFku I vkSj II nksuksa dk MsVk ç'u dk mÙkj nsus ds fy, i;kZIr ugha gSA Sol. (e)
(e) The data given in both the Statement I and II are necessary Since the ages of none of them is given, So no conclusion can
to answer the question. be drawn through both the statements.
dFku I vkSj II nksuksa esa fn, x, MsVk ç'u dk mÙkj nsus ds fy, 11. A question is given, followed by two statements labelled I
vko';d gSaA and II. Identify which of the statements sufficient to
Sol. (b) answer the question.
From Statement II, ,d ç'u fn;k x;k gS] ftlds ckn I vkSj II yscy okys nks dFku fn,
x + y = 18 x, gSaA igpkusa fd dkSu lk dFku ç'u dk mÙkj nsus ds fy, i;kZIr
It is possible only when x = y = 9. gSA
Hence, only Statement II is sufficient. Question:
In which year was Dheeraj born?
 Concept - 09
èkhjt dk tUe fdl o"kZ esa gqvk Fkk\
Count of members based Statements:
9. What is the staff strength of company ‘X’ ? I. Dheeraj at present is 35 years younger to his mother
daiuh ‘X’ ds deZpkfj;ksa dh la[;k fdruh gS\ Parvati.
I. Male and female employees are in the ratio of 2 : 3, èkhjt orZeku esa viuh ekrk ikoZrh ls 35 o"kZ NksVk gSA
respectively. II. Dheeraj's bother Shan, who was born in 1998, is 45 years
iq#"k vkSj efgyk deZpkjh Øe'k% 2: 3 ds vuqikr esa gSaA younger to his mother Parvati.
II. Of the officers employees 80% are males. 1998 esa iSnk gq, èkhjt ds Hkkà 'kku viuh eka ikoZrh ls 45 lky NksVs
vfèkdkjh deZpkfj;ksa esa ls 80% iq#"k gSaA gSaA
III. Total number of officers is 132. (a) I alone is sufficient, while II alone is not sufficient
vfèkdkfj;ksa dh dqy la[;k 132 gSA vdsys I i;kZIr gS] tcfd vdsys II i;kZIr ugha gS
(a) Only I and II/ dsoy I vkSj III (b) Both I and II together are sufficient
(b) Only II and either III or I/ dsoy II vkSj III ;k I I vkSj II nksuksa ,d lkFk i;kZIr gSa
(c) All I, II and III/lHkh I, II vkSj III (c) II alone is sufficient, while I alone is not sufficient
(d) Any two of the three/ rhu esa ls dksbZ nks vdsys II i;kZIr gS] tcfd I vdsyk i;kZIr ugha gS
(e) Question cannot be answered even with the information in (d) Either I alone or II alone is sufficient
all three statements/rhuksa dFkuksa esa tkudkjh ds lkFk Hkh ç’u dk ;k rks I vdsyk ;k vdsyk II i;kZIr gS
mÙkj ugha fn;k tk ldrk gS Sol. (b)
Sol. (e) Dheeraj’s brother Shan was born in 1998. Now according to
Question cannot be answered even with the information in all question,
three statements. Birth year of Dheeraj’s mother
 Concept - 10 = 1998 – 45 = 1953
Birth year of Dheeraj = 1953 + 35 = 1988
Age based Hence, both statements I and II are sufficient to answer the
10. What is Deepali’s age? question.
nhikyh dh mez D;k gS\
I. Deepali is two times younger than Nisha. Decision Making
nhikyh] fu'kk ls nks xquk NksVh gSA Decision Making Questions are part of Verbal Reasoning and a
II. Supriya is twice the age of Nisha. common topic for all major Government exams conducted in
lqfç;k] fu'kk ls nksxquh mez dh gSA the country.
(a) The Statements I alone is sufficient to answer the question, fu.kZ; ysus okys ç'u ocZy jhtfuax dk ,d fgLlk gSa vkSj ns'k esa
but the Statement II alone is not sufficient. vk;ksftr gksus okyh lHkh çeq[k ljdkjh ijh{kkvksa ds fy, ,d lkekU;
ç’u dk mÙkj nsus ds fy, dsoy dFku I gh i;kZIr gS] ysfdu dsoy fo"k; gSA
dFku II i;kZIr ugha gSA The approach:

94
Best App for Govt. Jobs : Neonclasses (Download Now)

Logical analysis and reviewing the questions carefully are the (b) (ii) but if he has more than five years of work experience, then
key factors to answer questions based on this topic. his case will be referred to the Chairman.
rkfdZd fo'ys"k.k vkSj ç'uksa dh lko/kkuhiwoZd leh{kk bl fo"k; ij mijksDr (ii), fdUrq mls ik¡p o"kZ ls vf/kd dk dk;Z vuqHko gS rks
vk/kkfjr ç'uksa dk mÙkj nsus ds fy, egRoiw.kZ dkjd gSaA mldk ekeyk v/;{k dks Hkstk tk,xkA
The Concept of Decision Making Show answer /mÙkj n’kkZb,
fu.kZ; ysus dh vo/kkj.kk (a) if the candidate is not to be admitted.
Under the decision making questions, candidates need to ;fn mEehnokj dh HkrhZ ugha dh tkuh gSA
analyze and review the given information and based on the (b) if the matter is to be referred to the Chairman.
same need to answer the question. ;fn ekeyk v/;{k dks Hkstk tkuk gSA
fu.kZ; ysus ds ç'uksa ds rgr] mEehnokjksa dks nh xbZ tkudkjh dk (c) The matter of the candidate is to be referred to the
fo'ys"k.k vkSj leh{kk djus dh vko';drk gksrh gS vkSj ç'u dk mÙkj Managing Director.
nsus dh mlh vko';drk ds vk/kkj ijA mEehnokj dk ekeyk izca/k funZs'kd ds ikl Hkstk tkuk gSA
There two types of questions which may be asked as a part of (d) If the candidate is to be recruited.
decision making. These include: ;fn mEehnokj dh HkrhZ dh tkuh gSA
nks çdkj ds ç'u tks fu.kZ; ysus ds ,d Hkkx ds :i esa iwNs tk ldrs (e) If available data is not sufficient for decision making.
gSaA bles 'kkfey gS ;fn miyC/k MkVk fu.kZ; ysus ds fy, i;kZIr ugha gSA
Direct Questions: In such questions, a condition may be given 1. Adesh Kulkarni has got 65% marks in his postgraduate level.
and based on the same the final decision has to be made He was born on 21 April 1976. He has scored 55% and 45%
çR;{k ç'u: ,sls ç'uksa es]a ,d 'krZ nh tk ldrh gS vkSj mlh ds marks in written competition test and interview respectively.
vk/kkj ij vafre fu.kZ; ysuk gksrk gS He has been working for the last five years after completing
Complex Questions: In such questions, a set of eligibility his postgraduate qualification.
criteria are given. Candidates need to analyse the eligibility vkns’k dqyd.khZ dks vius LukrdksÙkj Lrj ij 65% vad feys gSA
details given and based on it, answer what candidates can mldk tUe 21 vizSy 1976 dks gqvk FkkA mls fyf[kr izfr;ksfxrk
qualify/ appear/ register for an exam or a post ijh{kk vkSj lk{kkRdkj esa Øe’k% 55% vkSj 45% vad feys gSA og
tfVy ç'u ,sls ç'uksa esa] ik=rk ekunaM dk ,d lsV fn;k tkrk gSA viuh LukrdksÙkj ;ksX;rk iwjh dj fiNys ik¡p o"kksZa ls dk;Zjr gSA
mEehnokjksa dks nh xbZ vkSj mlds vk/kkj ij nh xbZ ik=rk fooj.kksa dk Sol. (d)
fo'ys"k.k djus dh vko';drk gS] mÙkj nsa fd mEehnokj fdlh ijh{kk (i) de ls de 60% vadks ds lkFk Lukrd gks ;k Lukrd
;k in ds fy, ;ksX; / çdV / iathdj.k dj ldrs gSa ;k ughaA vFkok LukrdksÙkj Lrj ij 60% vadksa ds lkFk
With the increased competition, the complex variety of mÙkh.kZ gksuk pkfg,A
questions have become more common. These types of mijksDr (i), fdUrq ih,p- Mh- (Ph.D.) fMxzh/kkjh gS] rks
questions are lengthy but if reviewed carefully, they are easy mldk ekeyk izc/ka funZs'kd dks Hkstk tk;sxkA
to answer. (ii) dh vk;q 01.06.2003 dks 23 o"kZ ls de ugha vkSj 30 o"kZ
c<+rh çfrLi/kkZ ds lkFk] fofHkUu çdkj ds ç'u vf/kd lkekU; gks x, ls vf/kd ugha gksuh pkfg,A
gSaA bl çdkj ds ç'u yacs gksrs gSa ysfdu ;fn lko/kkuhiwoZd leh{kk mijksDr (ii), fdUrq mls ik¡p o"kZ ls vf/kd dk dk;Z vuqHko
dh tk, rks os mÙkj nsuk vklku gksrs gSaA gS rks mldk ekeyk v/;{k dks Hkstk tk,xkA
Candidate /mEehnokj &
(iii) dks fyf[kr izfr;ksfxrk ijh{kk esa de ls de 50% vkSj
(i) Should be a graduate with at least 60% marks or passed with lk{kkRdkj esa de ls de 40% vad izkIr gq, gksus
60% marks at the undergraduate or postgraduate level.
pkfg,A
de ls de 60% vadks ds lkFk Lukrd gks ;k Lukrd vFkok
(iv) dks Lukrd ;k LukrdksÙkj ijh{kk ikl djus ds ckn de ls
LukrdksÙkj Lrj ij 60% vadksa ds lkFk mÙkh.kZ gksuk pkfg,A
de rhu o"kZ dk dk;Z vuqHko gksuk pkfg,A
(ii) Age should not be less than 23 years and not more than 30
Because the candidate fulfills all the required conditions.
years as on 01-06-2003.
dh vk;q 01.06.2003 dks 23 o"kZ ls de ugha vkSj 30 o"kZ ls vf/kd D;ksfa d mEehnokj fn, x, lHkh vko';d 'krksZ dks iwjk djrk gSA
2. Jayant Desai was 28 years old on 01-01-07. He has got 45%
ugha gksuh pkfg,A
and 60% marks respectively at the undergraduate and
(iii) Should have scored at least 50% marks in the written
postgraduate levels. After his postgraduate qualification, he
competition examination and at least 40% marks in the
has been working for the last three years. He got 60% marks in
interview.
the interview.
dks fyf[kr izfr;ksfxrk ijh{kk esa de ls de 50% vkSj lk{kkRdkj esa
01.07.2001 dks t;Ur nslkbZ 28 o"kZ dk FkkA mls Lukrd vkSj
de ls de 40% vad izkIr gq, gksus pkfg,A
LukrdksÙkj Lrj ij Øe’k% 45% vkSj 60% vad feys gSA viuh
(iv) Should have at least three years of work experience after
LukrdksÙkj ;ksX;rk ds ckn og fiNys rhu o"kZ ls dk;Zjr gSA mls
passing undergraduate or postgraduate examination.
dks Lukrd ;k LukrdksÙkj ijh{kk ikl djus ds ckn de ls de rhu lk{kkRdkj esa 60% vad feys gSA
Sol. (e)
o"kZ dk dk;Z vuqHko gksuk pkfg,A
If the candidate fulfills all other criteria except.
;fn mEehnokj vU; lHkh ekunaM iwjs djrk gks flok;& (i) de ls de 60% vadks ds lkFk
(a) (i) but he is a PhD. Degree holder his case will be referred to
Lukrd gks ;k Lukrd
the Managing Director. vFkok LukrdksÙkj Lrj ij
mijksDr (i), fdUrq ih,p- Mh- (Ph.D.) fMxzh/kkjh gS] rks mldk ekeyk 60% vadksa ds lkFk mÙkh.kZ
izc/ka funZs'kd dks Hkstk tk;sxkA gksuk pkfg,A
mijksDr (i), fdUrq ih,p- Mh-

95
Download Free PDFs & e-Books from Neon Classes App

(Ph.D.) fMxzh/kkjh gS] rks de rhu o"kZ dk dk;Z


mldk ekeyk izc/ka vuqHko gksuk pkfg,A
funZs'kd dks Hkstk tk;sxkA
(ii) dh vk;q 01.06.2003 dks 23 o"kZ Because the candidate has got 55% marks at postgraduate
ls de ugha vkSj 30 o"kZ ls level, but he has been selected and fulfills all the prerequisites.
vf/kd ugha gksuh pkfg,A D;ksfa d mEehnokj dks LukÙkdksÙkj Lrj ij 55% vad feys gS] ijUrq og
mijksDr (ii), fdUrq mls ik¡p o"kZ Ph. D fd;k gS rFkk lHkh 'krZ dks iwjk djrk gSA
ls vf/kd dk dk;Z vuqHko 4. Suresh Oberoi got 65% marks at postgraduate level at the age
gS rks mldk ekeyk v/;{k of 26 in 1997. He has been working since then. He has got 55%
dks Hkstk tk,xkA and 40% marks in written competition test and interview
(iii) dks fyf[kr izfr;ksfxrk ijh{kk esa respectively.
de ls de 50% vkSj lqj’s k vkscjks; dks o"kZ 1997 esa 26 o"kZ dh mez esa LukrdksÙkj Lrj ij
65% vad feys gSA rc ls og dk;Zjr gSaA mls fyf[kr izfr;ksfxrk
lk{kkRdkj esa de ls de
40% vad izkIr gq, gksus
ijh{kk vkSj lk{kkRdkj esa Øe’k% 55% vkSj 40% vad feys gSA
Sol. (b)
pkfg,A
(iv) dks Lukrd ;k LukrdksÙkj ijh{kk
(i) de ls de 60% vadks ds lkFk
ikl djus ds ckn de ls
Lukrd gks ;k Lukrd
de rhu o"kZ dk dk;Z
vFkok LukrdksÙkj Lrj ij
vuqHko gksuk pkfg,A
60% vadksa ds lkFk
mÙkh.kZ gksuk pkfg,A
Because the marks obtained in the written examination by the
mijksDr (i), fdUrq ih,p- Mh-
candidate have not been discussed.
(Ph.D.) fMxzh/kkjh gS] rks
D;ksfa d mEehnokj ds }kjk fyf[kr ijh{kk esa izkIr vadks dh ppkZ ugha
dh xbZ gSA mldk ekeyk izc/ka
3 Arun Bhosale was born on 2 January 1979. He has been
funZs'kd dks Hkstk tk;sxkA
working for the last three years after obtaining a PHD degree (ii) dh vk;q 01.06.2003 dks 23
in management. He has got 50% and 55% marks in written o"kZ ls de ugha vkSj 30
competition test and interview respectively. He has got 55% o"kZ ls vf/kd ugha gksuh
marks at postgraduate level. pkfg,A
v:.k Hkkslys dk tUe 2 tuojh 1979 dks gqvk FkkA izca/ku esa Ph.D. mijksDr (ii), fdUrq mls ik¡p
izkIr dj og fiNys rhu o"kksZ ls dk;Zjr gSA mls fyf[kr izfr;ksfxrk o"kZ ls vf/kd dk dk;Z
ijh{kk vkSj lk{kkRdkj esa Øe’k% 50% vkSj 55% vad feys gSaA mls vuqHko gS rks mldk
LukrdksÙkj Lrj ij 55% vad feys gSA ekeyk v/;{k dks Hkstk
Sol. (c) tk,xkA
(iii) dks fyf[kr izfr;ksfxrk ijh{kk esa
(i) de ls de 60% vadks ds lkFk de ls de 50% vkSj
Lukrd gks ;k Lukrd lk{kkRdkj esa de ls de
vFkok LukrdksÙkj Lrj ij 40% vad izkIr gq, gksus
60% vadksa ds lkFk mÙkh.kZ pkfg,A
gksuk pkfg,A (iv) dks Lukrd ;k LukrdksÙkj
mijksDr (i), fdUrq ih,p- Mh- ijh{kk ikl djus ds ckn
(Ph.D.) fMxzh/kkjh gS] rks de ls de rhu o"kZ dk
mldk ekeyk izc/ka dk;Z vuqHko gksuk pkfg,A
funZs'kd dks Hkstk tk;sxkA
(ii) dh vk;q 01.06.2003 dks 23 o"kZ Because the age of the candidate is more than 30 years, but he
ls de ugha vkSj 30 o"kZ ls has work experience of more than five years.
vf/kd ugha gksuh pkfg,A D;ksfa d mEehnokj dh vk;q 30 o"kZ ls vf/kd gS] ijUrq mlds ikl ik¡p
mijksDr (ii), fdUrq mls ik¡p o"kZ o"kZ ls vf/kd dk dk;kZuqHko gSA
ls vf/kd dk dk;Z vuqHko Candidate/mEehnokj
gS rks mldk ekeyk v/;{k (i) Should not be less than twenty years of age as on 1.09.04.
dks Hkstk tk,xkA ;Fkk 1.09.04 dks chl o"kZ ls de vk;q dk u gksA
(iii) dks fyf[kr izfr;ksfxrk ijh{kk esa (ii) Should be a graduate with at least 50% marks.
de ls de 50% vkSj de ls de 50 izfr’kr vadks ds lkFk Lukrd gksA
lk{kkRdkj esa de ls de (iii) Must have scored at least 60 percent marks in the entrance
40% vad izkIr gq, gksus examination.
pkfg,A us izos’k ijh{kk esa de ls de 60 izfr’kr vad izkIr fd, gksAa
(iv) Should be able to pay Rs. 50,000 at admission.
(iv) dks Lukrd ;k LukrdksÙkj ijh{kk
izos’k ds le; :- 50,000/- nsus esa leFkZ gksA
ikl djus ds ckn de ls
However, if a candidate fulfills all the criteria-

96
Best App for Govt. Jobs : Neonclasses (Download Now)

;fn dksbZ mEehnokj lHkh ekunaM iwjs djrk gks& vkSj 'ks"k jkf’k rhu fnu ckn vnk dj ldrk gSA mls xzstq,’ku esa 55
(a) Except for the above (iii) condition, but who has attained at izfr’kr vad feys gSaA
least 50 per cent marks, his case will be referred to the Sol. (d)
President. (i) ;Fkk 1.09.04 dks chl o"kZ ls de
flok,] mijksDr (iii) ds] ysfdu de ls de 50 izfr’kr vad izkIr fd, vk;q dk u gksA
gksa rks mldk ekeyk v/;{k izos’k dks Hkstk tk,xkA (ii) de ls de 50 izfr’kr vadks ds
(b) Except (iv) as mentioned above; but not less than Rs. 30.000 at lkFk Lukrd gksA
the time of admission and if can pay the remaining amount (iii) us izos’k ijh{kk esa de ls de 60
within a week, then he can be given admission provisionally. izfr’kr vad izkIr fd, gksaA
flok, mijksDr (iv) ds] ysfdu izos’k ds le; de ls de :- flok,] mijksDr (iii) ds] ysfdu de
30,000/- vkSj ckdh jde ,d lIrkg esa vnk dj ldrk gks rks mls
ls de 50 izfr’kr vad izkIr
vafre :i ls izos’k fn;k tk ldrk gSA fd, gksa rks mldk ekeyk
Answer (a) Dismiss is not to be given to the student.
v/;{k izos’k dks Hkstk tk,xkA
mÙkj (a) nhft, fo|kFkhZ dks izos’k ugha fn;k tkuk gSA
(iv) ;fn dksbZ mEehnokj lHkh ekunaM iwjs
Answer (b) If it is to be referred to the President.
djrk gks&
mÙkj (b) nhft, ;fn mls v/;{k izos’k dks lanfHkZr fd;k tkuk gSA
flok, mijksDr (iv) ds] ysfdu izos’k
Answer (c) If the student is to be entranced.
mÙkj (c) nhft, ;fn fo/kkFkhZ dks izos’k fn;k tkuk gSA ds le; de ls de :-
30,000/- vkSj ckdh jde
Answer (d) If the student is to be given provisional admission.
mÙkj (d) nhft, ;fn fo/kkFkhZ dks vafre :i ls izos’k fn;k tkuk gSaA ,d lIrkg esa vnk dj ldrk
Answer (e) if the available data is insufficient to make a
gks rks mls vuafre :i ls
decision. izos’k fn;k tk ldrk gSA
mÙkj (e) nhft, ;fn dksbZ fu.kZ; ysus ds fy, miyC/k MkVk vi;kZIr Because at the time of nomination the candidate can pay Rs.
gSA 35000 And can deposit the remaining amount after three days
5. Suresh Nadkarni passed the graduation examination in the and fulfills all the given conditions.
year 2002 with 56% marks. He was born on 15 July 1980. He D;kasfd vH;FkhZ ukekadu ds le; 35,000 :- rFkk 'ks"k jkf'k rhu fnu
has scored 65 percent marks in the entrance examination and ckn tek dj ldrk gS rFkk fn, x, lHkh 'krksZ dks iwjk djrk gSA
can give Rs. 40.000 at admission. Candidate/ mEehnokj
lqj’s k ukMd.khZ us Lukrd ijh{kk 56% vadks ds lkFk o"kZ 2002 esa (i) On 1.1.05 should be at least 21 years and not more than 30
mÙkh.kZ dh gSA mldk tUe 15 tqykbZ 1980 dks gqvk FkkA mlus izos’k years.
ijh{kk esa 65 izfr’kr vad fy, gS vkSj izos’k ds le; 40,000/- :i;s 1.1.05 dks de ls de 21 o"kZ vkSj vf/kdre 30 o"kZ dk gksuk pkfg,A
ns ldrk gSA (ii) Must be a graduate / engineer with specialized knowledge in
Sol. (e) computer science and IT.
(i) ;Fkk 1.09.04 dks chl o"kZ ls de daEI;wVj foKku ;k IT esa fo’ks"kKrk lfgr Lukrd@bathfu;j gksuk
vk;q dk u gksA pkfg,A
(iii) After graduation, should have at least three years work
(ii) de ls de 50 izfr’kr vadks ds
experience in the area of IT.
lkFk Lukrd gksA
LukÙkdLrj ds ckn IT ds {ks= esa de ls de rhu o"kZ dk dk;kZuqHko
(iii) us izos’k ijh{kk esa de ls de 60
gksuk pkfg,A
izfr’kr vad izkIr fd, gksAa
(iv) If he leaves his job within a period of two years from the date
flok,] mijksDr (iii) ds] ysfdu de of joining the office. then Should be ready to sign the bond of
ls de 50 izfr’kr vad izkIr Rs. 1,00,000.
fd, gksa rks mldk ekeyk ;fn dk;Zxzg.k dh rkjh[k ls nks o"kksZ dh vof/k ds Hkhrj viuh ukSdjh
v/;{k izos’k dks Hkstk tk,xkA NksM+ nsrk gS rks mls :- 1,00,000/- dk cakM gLrk{kfjr djus ds fy,
(iv) ;fn dksbZ mEehnokj lHkh ekunaM iwjs rS;kj jguk pkfg,A
djrk gks& If the candidate fulfills all other criteria except-
flok, mijksDr (iv) ds] ysfdu izos’k ;fn mEehnokj vU; lHkh ekunaM iwjs djrk gks flok;&
ds le; de ls de :- (a) (ii) but if the candidate is a postgraduate engineer in
30,000/- vkSj ckdh jde computer science, or IT then his case should be referred to GM
,d lIrkg esa vnk dj ldrk -IT provided that he completed his undergraduate degree with
gks rks mls vuafre :i ls at least 60% marks in any of the engineering streams.
izos’k fn;k tk ldrk gSA mijksDr (ii), fdUrq daI;wVj foKku ;k IT esa LukrdksÙkj bathfu;j gS
The period for depositing the remaining money is not known. rks mldk ekeyk GM –IT ds ikl Hkstk tkuk pkfg, c’krsZ fd mUgksua s
'ks"k :Ik;s tek djus dh vof/k Kkr ugha gSA viuh Lukrd fMxzh bathfu;fjax ds fdlh Hkh {ks= esa de ls de
6. Abhinav Samanta got 85% marks in Entrance Examination. He 60% vad lfgr iwjh dh gksA
was born on 5 August 1981. He can pay Rs. 35,000 at (b) (iii), but if he has at least one year experience of working as a
admission. And the remaining amount can be paid after three IT- Officer, then his case will be referred to the DGM -IT.
days. He has got 55% marks in graduation. mijksDr (iii) fdUrq ;fn mls crkSj IT vf/kdkjh de ls de ,d o"kZ
vfHkuo lkear dks izos’k ijh{kk esa 85 izfr’kr vad feys gSaA mldk dk dk;kZuqHko gS rks mldk ekeyk DGM-IT ds ikl Hkstk tk,xkA
tUe 5 vxLr] 1981 dks gqvk gSA og izos’k ds le; :- 35,000/- Answer /mÙkj&

97
Download Free PDFs & e-Books from Neon Classes App

(a) If the candidate is not to be select . ukSdjh NksM+us ij 1 yk[k


nhft, ;fn mEehnokj dk pu ugha fd;k tkuk gSA dk okaM
(b) The data is insufficient for decision making. If the candidate is to be selected.
nhft, ;fn fu.kZ; ysus gsrq MkVk vi;kZIr gSA ;fn mEehnokj dk p;u fd;k tkuk gSA
(c) If the case is to be sent to the GM-IT. Candidate /mEehnokj
nhft, ;fn ekeyk GM-IT dks Hkstk tkuk gSA (a) Should be a Bachelor of Commerce in the first class including
(d) If the candidate is to be selected. 65% marks.
nhft, ;fn mEehnokj dk p;u fd;k tkuk gSA 65% vad lfgr izFke Js.kh esa okf.kT; Lukrd gksuk pkfg,A
(e) If the case is to be sent to the DGM-IT. (b) Should have scored at least 70% marks in S.S.C.
nhft, ;fn ekeyk DGM –IT dks HksTkk tkuk gSA dks S.S.C. esa de ls de 70% vad izkIr gq, gksus pkfg,A
7. Samir Malhotra was born on 25 June 1982. If he leaves the job (c) On 1.08.2007, should not be more than 26 years and not less
two years ago, he is ready to sign a bond of Rs. 1,00,000. He than 21 years.
holds a master's degree in electronics with 65% marks. He has dh vk;q ;Fkk 1.8.2007, 26 o"kZ ls vf/kd vkSj 21 o"kZ ls de ugha
been working in an organization as an IT -Officer for two gksuh pkfg,A
years. (d) Should have scored at least 60% marks in the selection test.
lehj eYgks=k dk tUe 25 twu 1982 dks gqvk FkkA ;fn og nks o"kZ dks p;u ijh{kk esa de ls de 60% vad feys gksus pkfg,A
igys ukSdjh NksM+rk gS rks og :- 1 yk[k dk ckaM gLrk{kfjr djus ds (e) Should have scored at least 50% marks in the selection
fy, rS;kj gSA mlus 65% vad lfgr bysDVªksfud~l esa ekLVlZ fMxzh interview.
dh gSA og ,d laxBu esa crkSj IT vf/kdkjh nks o"kZ ls dk;Z dj jgk dks p;u lk{kkRdkj esa de ls de 50% vad feys gksus pkfg,A
gSA If the candidate fulfills all the above criteria, except-
Sol. (b) ;fn mEehnokj mijksDr lkjs ekunaM iwjs djrk gS] flok;&
(i) 1.1.05 dks mez 21 to 30 chp dh (i) Above (a) but economical graduate with at least 70% marks,
tUe frfFk gksuh pkfg,A his case should be referred to the organization's G.M.
(ii) dEI;wVj foKku ;k IT fo'ks"kKrk mijksDr (a) ds fdarq de ls de 70% vad lfgr vFkZ’kkL= Lukrd
Lukrd@bathfu;fjax gS rks mldk ekeyk laxBu ds GM ds ikl Hkstk tkuk pkfg,A
bathfu;fjax ;k IT esa Lukrdksrj (ii) Above (e), if the candidate has scored at least 40% in the
60% ds lkFk rks GM-IT ds selection interview and at least 70% in the selection test, then
ikl his / her case should be referred to the President of the
(iii) Lukrd Lrj ds ckn IT {ks= esa 3 organization.
o"kZ dk;Z vuqHko mijksDr (e) ds] fdarq p;u lk{kkRdkj esa de ls de 40% vkSj p;u
IT vf/kdkjh 1 o"kZ vuqHko DGM-IT ijh{kk esa de ls de 70% vad ik;s gS rks mldk ekeyk laxBu ds
ds ikl izslhMsUV ds ikl Hkstk tkuk pkfg,A
(iv) dk;Z xzg.k ls 2 o"kZ ds Hkhrj Answer/ mÙkj
ukSdjh NksM+us ij 1 yk[k dk Answer (a) If the candidate is to be selected.
okaM mÙkj (a) nhft, ;fn mEehnokj dk p;u fd;k tkuk gSA
Answer Give (b) if the candidate is not to be selected.
The data is insufficient for decision making.
nhft, ;fn fu.kZ; ysus gsrq MkVk vi;kZIr gSA mÙkj (b) nhft, ;fn mEehnokj dk p;u ugha fd;k tkuk gSA
Answer (c) If the matter is to be referred to GM.
8. Naveen Khosla has started working as a computer officer in an
mÙkj (c) nhft, ;fn ekeyk GM ds ikl Hkstk tkuk gSA
organization since June 2000, after completing his B.E in IT
with 70% marks at the age of 21 years. According to the terms Answer (d) if the matter is to be referred to the President.
and conditions, he is ready to sign A bond of 1 lakh.
mÙkj (d) nhft, ;fn ekeyk izslhMsUV ds ikl Hkstk tkuk gSA
21 o"kZ dh vk;q esa 70% vad lfgr IT esa viuk B.E. iwjk djus ds Answer Give (e) if the data is insufficient for decision making.
ckn uohu [kkslyk us twu 2000 ls ,d laxBu esa daEI;wVj vf/kdkjh mÙkj (e) nhft, ;fn fu.kZ; ysus ds fy, MkVk vi;kZIr gSA
ds :i esa dk;Z djuk vkjaHk fd;k gSA 'krksZ ds vuqca/k ds vuqlkj og 9. Amar has passed the degree examination with first class after
taking Economics in Commerce with 68% marks in 2006, in
:- 1 yk[k dk ckaM gLark{kfjr djus ds fy, rS;kj gSA
the age of 22 years. He scored 73% in S.S.C. He has passed the
Sol. (d)
selection exam with 64% marks and the selection interview
(i) 1.1.05 dks mez 21 to 30 chp dh
with 62 marks.
tUe frfFk gksuh pkfg,A vej us 22 o"kZ dh vk;q esa 2006 esa 68% vad lfgr okf.kT; esa ,d
(ii) dEI;wVj foKku ;k IT fo'ks"kKrk fo"k; vFkZ’kkL= ysrs gq, izFke Js.kh esa fMxzh ijh{kk ikl dh gSA S.S.C.
Lukrd@bathfu;fjax esa mlus 73% vad ik;s FksA mlus p;u ijh{kk 64% vad lfgr vkSj
bathfu;fjax ;k IT esa Lukrdksrj p;u lk{kkRdkj 62% vad lfgr ikl fd;k gSA
60% ds lkFk rks GM-IT ds Sol. (a)
ikl (a) 65% vad lfgr izFke Js.kh esa
(iii) Lukrd Lrj ds ckn IT {ks= esa 3 okf.kT; Lukrd gksuk pkfg,A
o"kZ dk;Z vuqHko mijksDr (a) ds fdarq de ls de
IT vf/kdkjh 1 o"kZ vuqHko DGM- 70% vad lfgr vFkZ’kkL=
IT ds ikl Lukrd gS rks mldk ekeyk
(iv) dk;Z xzg.k ls 2 o"kZ ds Hkhrj laxBu ds GM ds ikl Hkstk

98
Best App for Govt. Jobs : Neonclasses (Download Now)

tkuk pkfg,A Sol. (c)


(b) dks S.S.C. esa de ls de 70% (a) 65% vad lfgr izFke Js.kh esa okf.kT; Lukrd gksuk pkfg,A
vad izkIr gq, gksus pkfg,A mijksDr (a) ds fdarq de ls de 70% vad lfgr vFkZ’kkL=
(c) dh vk;q ;Fkk 1.8.2007, 26 o"kZ ls Lukrd gS rks mldk ekeyk laxBu ds GM ds ikl
vf/kd vkSj 21 o"kZ ls de Hkstk tkuk pkfg,A
ugha gksuh pkfg,A (b) dks S.S.C. esa de ls de 70% vad izkIr gq, gksus pkfg,A
(d) dks p;u ijh{kk esa de ls de (c) dh vk;q ;Fkk 1.8.2007, 26 o"kZ ls vf/kd vkSj 21 o"kZ ls
60% vad feys gksus de ugha gksuh pkfg,A
pkfg,A (d) dks p;u ijh{kk esa de ls de 60% vad feys gksus pkfg,A
(e) dks p;u lk{kkRdkj esa de ls de (e) dks p;u lk{kkRdkj esa de ls de 50% vad feys gksus
50% vad feys gksus pkfg,A
pkfg,A mijksDr (e) ds] fdarq p;u lk{kkRdkj esa de ls de 40%
mijksDr (e) ds] fdarq p;u vkSj p;u ijh{kk esa de ls de 70% vad ik;s gS rks
lk{kkRdkj esa de ls de mldk ekeyk laxBu ds izslhMsUV ds ikl Hkstk tkuk
40% vkSj p;u ijh{kk esa pkfg,A
de ls de 70% vad ik;s Rupali fulfills the (i), (b), (c), (d) and (e) criteria. Hence his
gS rks mldk ekeyk laxBu case should be referred to the GM of the organization.
ds izslhMsUV ds ikl Hkstk :Ikkyh ekunaM (i), (b), (c), (d) ,oa (e) iwjk djrh gSA vr% mldk
tkuk pkfg,A ekeyk laxBu ds GM ds ikl Hkstk tkuk pkfg,A
Amar fulfills all the criteria. Hence he can be selected. Candidate/mEehnokj
vej lHkh ekunaMks dks iwjk djrk gSA vr% mldk p;u gks ldrk gSA (i) should be at least 40 years old as on 01-05-2010 and not more
10. Animesha has done her graduation in arts with specialization than 50 years.
in Economics including 72% in S.S.C. and 85% in S.S.C. Her 01.05.2010 dks de ls de 40 o"kZ dk gks vkSj vf/kd ls vf/kd 50
birth date is 12. 06. 1985. She has scored 65% marks in both o"kZ dk gksA
the selection exam and other selection interview. (ii) Should be post graduate in executive management/ HR with
vfues"kk us 85% vad lfgr S.S.C. vkSj 72% vad lfgr vFkZ’kkL= esa 60% marks.
fo’ks"kKrk ds lkFk dyk esa Lukrd fd;k gSA mldh tUe rkjh[k de ls de 60 izfr’kr vadksa dks lkFk dkfeZd izca/ku@HR esa iksLV
12.06.1985 gSA mls p;u ijh{kk vkSj p;u lk{kkRdkj nksuksa esa 65% xzStq,V gSA
vad feys gSaA (iii) Should have at least fifteen years of work experience after
Sol. (c) attaining the requisite qualification, out of which he should be
(a) 65% vad lfgr izFke Js.kh esa okf.kT; Lukrd gksuk pkfg,A a Senior Manager HR in an institution for at least 5 years.
mijksDr (a) ds fdarq de ls de 70% vad lfgr dks ;ksX;rk izkfIr ds ckn de ls de ianzg o"kZ dk dk;Z vuqHko gks]
vFkZ’kkL= Lukrd gS rks mldk ekeyk laxBu ds GM ftlesa ls og de ls de 5 o"kZ rd fdlh laLFkk esa ofj"B izca/kd –
ds ikl Hkstk tkuk pkfg,A HR jgk gksA
(b) dks S.S.C. esa de ls de 70% vad izkIr gq, gksus pkfg,A (iv) Has successfully implemented the HR system in his
dh vk;q ;Fkk 1.8.2007, 26 o"kZ ls vf/kd vkSj 21 o"kZ ls organization during the last three years.
(c)
de ugha gksuh pkfg,A us fiNys rhu oa"kksZ ds nkSjku viuh laLFkk esa HR flLVe lQyrkiwoZd
ykxw fd;k gksA
(d) dks p;u ijh{kk esa de ls de 60% vad feys gksus pkfg,A
(v) Had scored at least 45 percent marks in the selection process.
(e) dks p;u lk{kkRdkj esa de ls de 50% vad feys gksus
(v) dks p;u izfØ;k esa dels de 45 izfr’kr vad feys gksAa
pkfg,A
If the candidate fulfills all other criteria except-
mijksDr (e) ds] fdarq p;u lk{kkRdkj esa de ls de 40% ;fn mEehnokj vU; lHkh ekunaM iwjs djrk gS flok;&
vkSj p;u ijh{kk esa de ls de 70% vad ik;s gS rks (a) Above (i) method, but it has got at least 50 percent marks in
mldk ekeyk laxBu ds izslhMsUV ds ikl Hkstk tkuk post-graduation and at least 65 percent marks in the selection
pkfg,A process, then the case will be sent to the executive director.
Completes (i), (b), (c), (d) anda (e). Hence his case should be mijksDr (i) ds] ysfdu mls iksLV xzstq,’ku esa de ls de 50 izfr’kr
referred to the GM of the organization. vkSj p;u izfØ;k esa de ls de 65 izfr’kr vad feys gks]a rks ekeyk
vfues"kk (i), (b), (c), (d) ,oa (e) iwjk djrk gSA vr% mldk ekeyk dk;Zikyd funZs'kd dks Hkstk tk,xkA
laxBu ds GM ds ikl Hkstk tkuk pkfg,A (b) Above (iii) at least twelve years of work experience after
11. Ruru Pali achieved 76% marks and first class in B.A. with attaining the qualification, of which he / she has be senior
special knowledge of economics. She came first in her class manager HR for atleast eight years in an institution, the matter
with 82% marks in S.S.C. On 25 May 2007, she has completed shall be sent to president.
24 years. He has passed the selection exam with 66% marks mijksDr (iii) ds] ysfdu ;ksX;rk&izkfIr ds ckn de ls de ckjg o"kZ
and the selection interview with 54% marks. dk dk;Z vuqHko gks] ftlesa ls og fdlh laLFkk esa de ls de vkB
vFkZ’kkL= esa fo’ks"kKrk lfgr 76% vad vkSj izFke Js.kh izkIr djrs o"kZ rd ofj"B izc/kad – HR jgk gks] rks ekeyk v/;{k dks Hkstk
gq, :ikyh us B.A. ijh{kk ikl dh gSA S.S.C. ijh{kk esa 82% vad tk,xkA
lfgr og viuh d{kk esa izFke vk;h FkhA 25 ebZ] 2007 dks mlus 24 Answer /mÙkj
o"kZ iwjs fd, gSA mlus 66% vad lfgr p;u ijh{kk vkSj 54% vad Answer (a) If the candidate is to be selected.
lfgr p;u lk{kkRdkj ikl fd;k gSA mÙkj (a) nhft, ;fn mEehnokj dk p;u fd;k tkuk gSA

99
Download Free PDFs & e-Books from Neon Classes App

Answer (b) If the candidate is not to be selected. mijksDr (i) ds] ysfdu mls iksLV
mÙkj (b) nhft, ;fn mEehnokj dk p;u ugha fd;k tkuk gSA xzstq,’ku esa de ls de 50
Answer (c) The data provided is not sufficient for decision izfr’kr vkSj p;u izfØ;k esa
making. de ls de 65 izfr’kr vad
mÙkj (c) nhft, ;fn fn;k miyC/k djk;k x;k MkVk fu.kZ; ysus ds feys gks]a rks ekeyk dk;Zikyd
fy, i;kZIr ugha gSA funZs'kd dks Hkstk tk,xkA
Answer (d) If the matter will be sent to excutive director. (ii) de ls de 60 izfr’kr vadksa dks
mÙkj (d) nhft, ;fn ekeyk dk;Zikyd funZs'kd dks Hkstk tk,xkA lkFk dkfeZd izca/ku@HR esa
Answer (e) If the case is to be sent to chairman. iksLV xzStq,V gSA
mÙkj (e) nhft, ;fn ekeyk v/;{k dks Hkstk tk,xkA (iii) dks ;ksX;rk izkfIr ds ckn de ls
12 Pranabhav Ghosh was born on March 8, 1968. After
de ianzg o"kZ dk dk;Z vuqHko
completing his post graduation with 68 percent marks, he is
gks] ftlesa ls og de ls de
working in an institution for the last eighteen years, out of
5 o"kZ rd fdlh laLFkk esa
which he is senior management HR from last year. He has
successfully implemented the HR system in his organization
ofj"B izca/kd – HR jgk gksA
last year. He has a score of 50% in the selection process. mijksDr (iii) ds] ysfdu
iz.ko ?kks"k dk tUe 8 ekpZ] 1968 dks gqvk FkkA 68 izfr’kr vadksa ds ;ksX;rk&izkfIr ds ckn de ls
lkFk HR esa iksLV xzstq,’ku djus ds ckn og fiNys vBkgj o"kksZ ls de ckjg o"kZ dk dk;Z vuqHko
,d laLFkk esa dke dj jgk gS ftlesa ls fiNys o"kZ ls ofj"B izc/kad - gks] ftlesa ls og fdlh laLFkk
HR gSA mlus fiNys o"kZ viuh laLFkk esa lQyrkiwod Z HR flLVe ykxw esa de ls de vkB o"kZ rd
fd;k gSA p;u izfØ;k esa mls 50 izfr’kr vad feys gSaA ofj"B izc/kad – HR jgk gks]
Sol.(a) rks ekeyk v/;{k dks Hkstk
(i) 01.05.2010 dks de ls de 40 o"kZ dk gks vkSj vf/kd ls tk,xkA
vf/kd 50 o"kZ dk gksA (iv) us fiNys rhu oa"kksZ ds nkSjku viuh
mijksDr (i) ds] ysfdu mls iksLV xzstq,’ku esa de ls de laLFkk esa HR flLVe
50 izfr’kr vkSj p;u izfØ;k esa de ls de 65 lQyrkiwoZd ykxw fd;k gksA
izfr’kr vad feys gks]a rks ekeyk dk;Zikyd funZs'kd (v) dks p;u izfØ;k esa de ls de 45
dks Hkstk tk,xkA izfr’kr vad feys gksAa
(ii) de ls de 60 izfr’kr vadksa dks lkFk dkfeZd Naveen Chopra does not fulfill condition (v). Therefore, he
izca/ku@HR esa iksLV xzStq,V gSA cannot be selected.
(iii) dks ;ksX;rk izkfIr ds ckn de ls de ianzg o"kZ dk dk;Z uohu pksiM+k 'krZ (v) iwjh ugha djrk gSA vr% mldk p;u ugha fd;k
vuqHko gks] ftlesa ls og de ls de 5 o"kZ rd tk ldrkA
Directions (14-15): Study the following information carefully
fdlh laLFkk esa ofj"B izca/kd – HR jgk gksA
and answer the questions given below.
mijksDr (iii) ds] ysfdu ;ksX;rk&izkfIr ds ckn de ls de
fuEufyf[kr tkudkjh dk è;kuiwod
Z vè;;u djsa vkSj uhps fn, x,
ckjg o"kZ dk dk;Z vuqHko gks] ftlesa ls og fdlh
ç'uksa ds mÙkj nsAa
laLFkk esa de ls de vkB o"kZ rd ofj"B izc/kad –
Following are the conditions for selecting Manager-HR in an
HR jgk gks] rks ekeyk v/;{k dks Hkstk tk,xkA
organisation.
(iv) us fiNys rhu oa"kksZ ds nkSjku viuh laLFkk esa HR flLVe ,d laxBu esa çcaèkd&,p- vkj- ds p;u ds fy, fuEufyf[kr 'krZsa
lQyrkiwoZd ykxw fd;k gksA gSaA
(v) dks p;u izfØ;k esa de ls de 45 izfr’kr vad feys gksAa (i) be atleast 30 year and not more than 35 year as on 1st March,
Pranav Ghosh fulfills all the conditions. Therefore, he can be 2012./
selected. 1 ekpZ] 2012 dks de ls de 30 o"kZ vkSj vf/kdre 35 o"kZ gksuk
iz.ko ?kks"k lHkh 'krsZ iwjh djrk gSA vr% mldk p;u fd;k tk ldrk pkfg,A
gSA (ii) have secured atleast 60% marks in graduation in any discipline.
13. Naveen Chopra was born on 12 June 1967. He is working from fdlh Hkh fo"k; esa Lukrd esa 60% vad çkIr fd, gSaA
the last sixteen years after completing post graduation in (iii) have secured atleast 65% marks in the post-graduation
Personnel Management with 75% marks and out of which he degree/diploma in personnel management/HR.
has been Senior Management Manager for last 7 years. He got dkfeZd çcaèku@ekuo lalkèku esa LukrdksÙkj fMxzh@fMIyksek esa de
44 percent marks in the selection process. He successfully ls de 65% vad çkIr fd, gksAa
implemented HR system in his organization last year. (iv) Have post qualification work experience of atleast five years in
uohu pksiM+k dk tUe 12 twu] 1967 dks gqvk FkkA 75 izfr’kr vad the personnel/HR department of an organisation.
ysdj dkfeZd izca/ku esa iksLV xzst,q ’ku djus ds ckn og fiNys lksyg ;ksX;rk ds ckn fdlh laxBu ds dkfeZd@ekuo lalkèku foHkkx esa
o"kZ ls dk;Zjr gS ftlesa ls og lkr o"kZ ofj"B izca/kd – HR jgk gSA de ls de ikap lky dk dk;Z vuqHko gSA
mls p;u izfØ;k esa 44 izfr’kr vad feys gSA mlus fiNys o"kZ viuh (v) Have secured atleast 50% marks in the selection process.
laLFkk esa lQyrkiwoZd HR – flLVe ykxw fd;k gSA p;u çfØ;k esa de ls de 50% vad çkIr fd, gSaA
Sol. (b) However, in the case of candidate who satisfies all the above
(i) 01.05.2010 dks de ls de 40 conditions, except
o"kZ dk gks vkSj vf/kd ls gkykafd] mEehnokj ds ekeys esa tks mijksä lHkh 'krksZa dks larq"V
vf/kd 50 o"kZ dk gksA djrk gS] flok;

100
Best App for Govt. Jobs : Neonclasses (Download Now)

(a) (ii) above but has secured atleast 55% marks in graduation in graduate diploma in personnel management with 70% marks.
any discipline and atleast 70% marks in post-graduate, She was born on 5th July, 1979. She has secured 65% marks in
degree/diploma in personnel management/HR, the case is to graduation and 50% marks in the selection process.
be referred to GM-HR. lhek cgy 70% vadksa ds lkFk dkfeZd çcaèku esa LukrdksÙkj fMIyksek
Åij ysfdu fdlh Hkh fo"k; esa Lukrd esa de ls de 55% vad iwjk djus ds ckn fiNys 7 o"kksZa ls ,d laxBu ds dkfeZd foHkkx esa
gkfly fd, gSa vkSj dfeZ;ksa ds çcaèku@ekuo lalkèku esa LukrdksÙkj] dk;Zjr gSaA mudk tUe 5 tqykbZ] 1979 dks gqvk FkkA mUgksua s Lukrd
fMxzh@fMIyksek esa 70% vad çkIr fd, gSa] bl ekeys dks th-,e- esa 65% vad vkSj p;u çfØ;k esa 50% vad çkIr fd, gSaA
&,p-vkj- dksas Hkstk tkuk gSA Sol. (e)
(b) (iv) above but has post qualification work experience of atleast Candidates (i) (ii) (a) (iii) (iv) (b) (v) Answers
four years out of which atleast two years as deputy manager-      b
HR, the case is to be referred to President-HR. Seema c
(iv) Åij ysfdu pkj lky dk de ls de ;ksX;rk ds ckn dk dk;Z a
vuqHko gS] ftlesa ls de ls de nks lky fMIVh eSustj&,p-vkj- ds d
:i esa gS] bl ekeys dks v/;{k&HR dks Hkstk tkuk gSA e
In each question below are given details of one candidate. You if the candidate is to be selected
have to take one of the following courses of actions based on mEehnokj dk p;u fd;k tkuk gS
the information provided and the conditions and
subconditions given above and mark the number of that
Dice
course of action as your answer. You are not to assume
anything other than the information provided in each ● Dice is a cube. In a cube there are 6 faces. Some important
question. All these cases are given to you as on 1st March, Points are given below:
2012. iklk ,d ?ku gSA ?ku esa dqy 6 lrg gksrh gSA dqN egRoiw.kZ fcanq
uhps fn, x, çR;sd ç'u esa ,d mEehnokj dk fooj.k fn;k x;k gSA uhps fn, x, gSa%
vkidks nh xbZ tkudkjh ds vkèkkj ij dkjZokbZ ds fuEufyf[kr B C
ikBîØeksa esa ls ,d ysuk gS vkSj Åij nh xbZ 'krksZa vkSj mi'krksaZ vkSj
dkjZokbZ ds ml uacj dks fpfàr djsa tks vkids mÙkj ds :i esa gSA
vki çR;sd ç'u esa nh xbZ tkudkjh ds vykok dqN Hkh ugha ekuuk
gSaA ;s lHkh ekeys vkidks 1 ekpZ] 2012 dks fn, x, gSaA E
F
Give answer/ tokc nks
(a) if the candidate is not to be selected
;fn mEehnokj dk p;u ugha fd;k tkuk gS
(b) if the data provided are not adequate to take a decision D
A
;fn çnku fd;k x;k MsVk fu.kZ; ysus ds fy, i;kZIr ugha gS
(c) if the case is to be referred to President-HR
;fn ekeyk v/;{k -HR dks Hkstk tkuk gS
(d) if the case is to be referred to GM-HR H G
;fn ekeyk th,e&,pvkj dks Hkstk tkuk gS ●There are five types of questions that are asked in this
(e) if the candidate is to be selected chapter:
;fn mEehnokj dk p;u fd;k tkuk gS bl v/;k; esa ik¡p izdkj ds iz’u iwNs tkrs gSa &
14. Rita Bhatt was born on 25th July, 1978. She has secured 62% 1. When only one position of a dice is given.
marks in graduation and 65% marks in post-graduate diploma tc ,d ikls dh dsoy ,d gh fLFkfr ¼fp=½ nh tkrh gSaA
in management. She has been working for the past 6 year in 2. When only two position of a dice is given.
the personnel department of an organisation after completing tc ,d ikls dh dsoy nks fLFkfr ¼fp=½ nh tkrh gSaA
her post-graduation. She has secured 55% marks in the 3. When more than two position of a dice is given.
selection process. tc ,d ikls dh nks ls vf/kd fLFkfr ¼fp=½ nh tkrh gSaA
jhrk Hkê dk tUe 25 tqykbZ] 1978 dks gqvk FkkA mUgksua s Lukrd esa 4. When dice is given in open form.
62% vad vkSj çcaèku esa LukrdksÙkj fMIyksek esa 65% vad çkIr fd, tc ikls dks [kksy fn;k tkrk gSA
gSaA og viuh iksLV&xzstq,'ku iwjh djus ds ckn ,d laxBu ds dkfeZd 5. Language based conditional questions
foHkkx esa fiNys 6 lky ls dke dj jgh gSA mlus p;u çfØ;k esa There are two types of dice-
55% vad çkIr fd, gSaA ikls nks izdkj ds gksrs gSa&
Sol. (e) 1. Standard Dice: If the sum of every two opposite surgaces is
equal to seven then the dice is called a standard dice.
Candidates (i) (ii) (a) (iii) (iv) (b) (v) Answers ekud iklk% ;fn izR;sd nks foijhr lrgksa dk ;ksx 7 gS rc ml ikls
Rita    dks ekud iklk dgrs gSA
 e
 1 is opposite to 6/1 ds foijhr 6
if the data provided are not adequate to take a decision 2 is opposite to 5/2 ds foijhr 5
fn;k x;k MsVk fu.kZ; ysus ds fy, i;kZIr ugha gS 3 is opposite to 4/3 ds foijhr 4
15. Seema Behl has been working in the personnel department of
an organisation for the past 7 years after completing her post-

101
Download Free PDFs & e-Books from Neon Classes App

2. General Dice: If the sum of any two adjacent surfaces is equal uhps fn;k x;k iklk ekud iklk gS] 2 ds vkl&ikl D;k gksxk\
to seven or if dice is not standard then the dice is called a
general dice. 2
lk/kkj.k iklk% ;fn fdlh Hkh nks fudVorhZ lrgksa dk ;ksx lkr gks ;k
iklk ekud u gks rks ikls dks lk/kkj.k iklk dgrs gSaA 3
6
Concept-I :When one position is given a single dice:-
1. If the following dice is standard. Then what will be the (a) 1, 3, 4, 6 (b) 5, 3, 4, 6
opposite of 2? (c) 1, 5, 6, 4 (d) 5, 4, 1, 3
;fn uhps fn;k x;k iklk ekud gS rks 2 ds foijhr D;k gksxk\ Sol. (a) opposite of 2 is 5
2 Then remaining number 1, 3, 4, 6 will be adjacent of 2
Concept-II :When two position is given of a single dice:
6
4 when only one Number / letter is common
6. What is opposite to a?
(a) 5 (b) 1 (c) 4 (d) 3
1 ds foijhr D;k gS\
Sol. (a) Trick – In standard dice the sum of opposite number is
7,hence opposite no of 2 is 5 5 3
2. If the following dice is standard, than what will be opposite of
6?
;fn uhps fn;k x;k iklk ekud gS rks 6 ds foijhr D;k gksxk\ 3 4
1 2
6

3 2

I II
(a) 3 (b) 2 (c) 1 (d) 4
Sol. (c) Trick – In standard dice the sum of opposite number is 7, (a) 2 (b) 3 (c) 4 (d) 5
hence opposite no of 6 is 1 Sol. (c)
3. In the following dice, what is the opposite of 3? 5 3
fuEu ikls esa 3 ds foijhr D;k gSa\
4
3 4
2 3
1 2

(a) 1/5/6 (b) 1/3/4


(c) 2/3/4 (d) 5/6/4 I II
Sol. (a) the sum of adjacent number 4 and 3 is 7, it is normal dice. Step (I): Take the
So, opposite number of 3 can be 1/5/6 common number from both figures i.e. 3.
4. Two different positions of the same dice are shown. What is pj.k ¼I½ % nksuksa fLFkfr;ksa ¼fp=ks½a esa ls leku la[;k dks ys vFkkZr~ 3
the number of dots on the face opposite to the one with four Step (II): Rotate in the clockwise direction along the common
dots? number. pj.k ¼II½ % leku la[;k ds lkFk ?kM+h dh fn’kk esa ?kwesA
,d gh ikls dh nks vyx&vyx fLFkfr;ksa dks n'kkZ;k x;k gSA pkj 3→5→1
fcanqvksa okys Qyd ds foijhr Qyd ij fcanqvksa dh la[;k D;k gS\ 6
3→2→4

Step (III):Here the missing number (6) will be opposite to the


common number (3).
pj.k ¼III½ % ;gk¡ fNih gqbZ la[;k ¼6½] leku la[;k ¼3½ ds foijhr
gksxhA
(a) 5 (b) 1 (c) 3 (d) 2 Thus 3 is opposite to 6.
Sol. (c) bl izdkj 3] 6 ds foijhr gSA
Both dice ‘: :’ is common 5 is opposite 2/5] 2 ds foijhr gSA
The left letter will be opposite each other writing then 1 is opposite to 4/1] 4 ds foijhr gSA
clockwise starting from I 8. Six letters and symbols, H, h, I, @, % and $, are written on the
Dots Opposite different faces of a dice. Two positions of this dice are shown.
Select the letter or symbol that will be on the face opposite to
4 dots 4 dots the one having ‘H’.
6 dots 1 dots ,d ikls ds fofHkUu Qydksa ij N% v{kj vkSj fpUg H] h] I] @] %
vkSj $ fy[ks x, gSaA bl ikls dh nks fLFkfr;k¡ fn[kkbZ xbZ gSaA ml
5 dots 2 dots v{kj ;k çrhd dk p;u djsa tks 'H' okys ds foijhr psgjs ij gksxkA
4 dots opposite  ‘3 dots’
The correct answer is option (c).
5. Dice given below is a standard dice, what will be adjacent to 2?

102
Best App for Govt. Jobs : Neonclasses (Download Now)

Number Opposite
I I
10  10 5
% H h @ 25 20

30 15
(a) $ (b) % (c) h (d) @
Sol. (d) 15 opposite  ‘30’
In first dice and second dice. The symbol ‘I’ is common The correct answer is option (b).
The left letter will be opposite each other writing then 11. Which of the following options is correct?
clockwise starting from I fuEufyf[kr esa dkSulk fodYi lgh gS \
Symbol From I 3 6

I $ 2 4
6 1
H @
I II
% h
5 4 2 3
Hence, the face opposite the H is ‘@’.
4 3 1 5
The correct answer is option (d). 6 5 3 2
9. Six letters A, B, C, D, E and F are written on different faces of a
dice. Two positions of the same dice are shown. Select the a b c d
letter that will be on the face opposite to the face having the Sol. (d)
letter ‘C’. In this question we have to answer that using two figures (I)
,d ikls ds fofHkUu Qydksa ij Ng v{kj A, B, C, D, E vkSj F fy[ks and (II) which figure can be made from the options.
x, gSaA ,d gh ikls dh nks fLFkfr;k¡ n'kkZbZ xbZ gSaA ml v{kj dk One common trick:
p;u djsa tks 'C' v{kj okys psgjs ds foijhr psgjs ij gksxkA bl iz’u esa gesa lg mÙkj nsuk gS fd nks vkd`fr (I) vkSj (III) dk
mi;ksx djds fodYiksa esa ls fdl vkd`fr dks cuk;k tk ldrk gSA
A A
6→2→3
5
C F E D 6→1→4

uksV% fodYi dks gVkus dh fVªd


(a) F (b) D (c) B (d) E Note: OPTION ELIMINATION TRICK
Sol. (d) nks ofijhr la[;kvksa dks dHkh ,d lkFk ugha ns[kk tk ldrk gS vkSj uk
In first dice and second dice. The symbol ‘A’ is common gh dHkh Hkh ,d lkFk fNik;k tk ldrk gSA
The left letter will be opposite each other writing then Two opposite surfaes can never be seen together and can
clockwise starting from I never be hidden together.
Letter Opposite fodYi (a) xyr gS D;ksfa d 5 vkSj 6 dks ,d lkFk ugha ns[kk tk
ldrk gSA
AA E Option (a) is incorrect as 5 & 6 cannot be seen together.
F D
fodYi (b) xyr gS D;ksfa d 3 vkSj 4 dks ,d lkFk ugha ns[kk tk
ldrk gSA
C E Option (b) is incorrect as 3 & 4 cannot be seen together.
fodYi (c) xyr gS D;ksfa d 1 vkSj 2 dks ,d lkFk ugha ns[kk tk
Hence, the face opposite the C is ‘E’.
ldrk gSA
The correct answer is option (d).
Option (c) is incorrect as 1 & 2 cannot be seen together.
10. Six numbers, 5, 10, 15, 20, 25 and 30, are written on different
faces of a dice. Two positions of this dice are shown. Select the ∴ Option (d) is correct
number that will be on the face opposite to the face having the 12. Which of the following options is correct?
number ‘15’. fuEufyf[kr esa dkSulk fodYi lgh gS \
,d ikls ds fofHkUu Qydksa ij N% la[;k,a 5, 10, 15, 20, 25 vkSj 30 T S
T
fy[kh xbZ gSaA bl ikls dh nks fLFkfr;ka fn[kkbZ xbZ gSaA ml la[;k dk
P T
p;u djsa tks la[;k ‘15’ okys Qyd ds foijhr Qyd ij gksxhA A X T Q

25 10
R T S T
10 30 15 20 T S Q P
S T R Q

a b c d
(a) 10 (b) 30 (c) 20 (d) 5
Sol. (b) Sol. (c)
Both dice ‘10’ is common

103
Download Free PDFs & e-Books from Neon Classes App

T→X→P (I) (II) × (III)


?
R
÷
T→S→Q = ÷

fodYi gVkus dh fVªd dk iz;ksx fd;k] vr% fodYi (c) lgh gSA
(a) × (b) = (c) + (d) 
From option elimination trick option (c) is correct.
Sol. (a)
13. What will in place of question mark?
Concept-III :When two position is given of a single dice:
iz’u fpà ¼\½ ds LFkku ij D;k vk,xk\
when only two Number / letter is common in two figure
3 1 4 tc nksuksa fLFkfr ¼fp=½ esa dsoy nks la[;k,¡ leku gks&
2 5 3 4 6 Type B: In this type the remaining numbers in each figure
?
other than the common numbers will be opposite to each
I II III other.
(a) 3 (b) 6 (c) 2 (d) 1 blesa izR;sd fLFkfr;ksa ¼fp=ks½a esa leku la[;kvksa ds vykok 'ks"k cph
Sol. gqbZ la[;k,¡ ,d&nwljs ds foijhr gksxhA
3 1
3→5→2
6 2 3
6 6
3→1→4
I II
fodYi (a) xyr gSA D;ksfa d 3] 6 ds foijhr gSA
Option (a) is incorrect as 3 is opposite to 6. Step (I): Remaining numbers other than common number in
fodYi (b) xyr gSA D;ksfa d 6 igys ls gh fp= (III) esa gSA both figure are 2 and 1, they will be opposite to each other.
Option (b) is incorrect as 6 is already there in figure (III). pj.k (I)% leku la[;kvksa ds cykok 'ks"k 2 vkSj 1 gS] os ,d nwljs ds
fodYi (c) xyr gSA D;ksafd 2] 4 ds foijhr gSA foijhr gksxhaA
Option (c) is incorrect as 2 is opposite to 4. Step (II): Common numbers (3/6) are opposite to hidden
fodYi (d) lgh gSA numbers (4/5).
Therefore option (d) is correct. pj.k (II): leku la[;kvksa ¼3@6½ ds foijhr fNih gqbZ la[;k,¡ ¼4@5½
14. What will in place of question mark? gksxhaA
iz'u fpà ¼\½ ds LFkku ij D;k vk,xk\ ∴ 2↔ 1
2 4 4 3/6 ↔ /5
1 3 3 5 6 2 is opposite to 1.
?

I
3 is opposite to 4/5.
II III
6 is opposite to 4/5.
(a) 1 (b) 4 (c) 5 (d) 6 4 is opposite to 3/6.
Sol. (d)
5 is opposite to 3/6.
3→1→2 16.
6
3→4→5 T X
P X P Q
nks ofijhr la[;k,¡ ,d lkFk ugha fn[k ldrh] bl vk/kkj ij fodYi
Q
(C) vkSj (D) gVk, x,A I II
vc fp= (III) esa la[;k,a 2 vkSj 3 gSaA ;s nks la[;k,¡ fp= (II) esa Hkh
ekStwn gSaA Sol. T is opposite to Q/ T ds foijhr Q gSA
ge tkap djsaxsA fd D;k nksuksa la[;kvksa dh fn’kk fp= (I) vkSj fp= P/X opposite to R/S/. P/X ds foijhr R/S gSA
(III) esa leku gSaA 17. What is the number opposite to 6?
vkd`fr (I) esa 2 vkSj 3 nf{k.kkorZ fn’kk esa gSa tcfd (III) esa 2 ls 3 6 ds foijhr la[;k D;k gS \
okekorZ fn’kk esa gSA If option are:
2 2 ;fn fodYi gSa%
1 1 2 3
3 ?
1 5 2 5
I II
I II
D;ksfa d fn’kk foijhr gS] blfy, iklk leku ugha gSaA bldk vFkZ gS
fd iz’u fpà ds LFkku ij 4 vk,xk ¼∴ 4] 1 ds foijhr gSA½ (I) (A) 1 (B) 3 (C) 5 (D) 4
Option (c) is incorrect as 2 is opposite to 4. (II) (A) 4 (B) 6 (C) 2/5 (D) CND
∴ Therefore option (b) is correct. (III) (A) 5 (B) 2 (C) CND (D) N.O.T.
∴ fodYi (b) lgh gSA Sol. 1 is opposite to 3.
15. Which symbol will appear in place of (?) 2/5 is opposite to 4/6
(?) ds LFkku ij dkSulk fpUg~ vk;sxk\ Thus number opposite to 2 is either 4 or 6.
∴ 2 ds foijhr la[;k 4 ;k 6 gSA
(I) Ans (C) 5

104
Best App for Govt. Jobs : Neonclasses (Download Now)

(II) Ans (C) 2/5 The correct answer is option (c).


(III) Ans © CND i.e. can’t determine 21. Which option is correct?
18. Two different positions of the same dice are shown, the six, dkSulk fodYi lgh gS\
faces of which are numbered from 1 to 6. Select the number 4 2
that will be on the face opposite to the face having the number 2 4
6 3
‘3’
,d gh ikls ds nks vyx&vyx LFkku fn[kk, x, gSa] ftuesa ls Ng ds I II
Qyd 1 ls 6 rd fxus x, gSaA ml la[;k dk p;u djsa tks 3 la[;k
okys Qyd ds foijhr Qyd ij gksxhA 6 1 5 1
3 2 1 3
5 4 4 5
5 5
a b c d
6 3 1 6
Sol. Two common trick
6 is opposite to 3.
2/4 is opposite to 1/5.
(a) 2 (b) 1 (c) 6 (d) 4
As 3 and 6 are opposite to each other, therefore eliminate the
Sol. (b)
option in which 3 & 6 are seen together and 3 & 6 are hidden
When two faces are common
together.
Third face are opposite to each other
pwafd 3 vkSj 6 ,d&nwljs ds foijhr gS] blfy, mu fodYiksa dks gVk,
6
Here (5, 6) are common ftuesa 3 vkSj 6 dks ,d lkFk ns[kk tkrk gS vkSj 3 vkSj 6 dks ,d
1
lkFk fNik;k tkrk gSA
The correct answer is option (b).
Option (A) eliminated (3 & 6 are togehter).
19. Two different positions of the same dice are shown. The six
faces of which are colured Red, Green, Blue, Yellow, Black,
fodYi (A) xyr gSA ¼3 vkSj 6 ,d lkFk gSa½
White. Select the colour that be on the face opposite to the one Option (B) & (C) eliminated (3 & 6 are togehter).
coloured White. fodYi (B) vkSj (C) xyr gSA ¼3 vkSj 6 ,d lkFk fNis gq, gSa½
,d gh ikls dh nks vyx&vyx fLFkfr;ksa dks n'kkZ;k x;k gSA ftuesa Option (D) is correct.
ls Ng Qyd yky] gjs] uhys] ihys] dkys] lQsn jax ds gSaA ml jax fodYi (D) lgh gSA
dk p;u djsa tks ,d jax ds lQsn ds foijhr Qyd ij gksA 22. Which option is correct?
dkSulk fodYi lgh gS\
B C
Red Green
A C A F
Yellow White Yellow Red
I II

E C C A
(a) Red (b) Green (c) Black (d) Blue B E
F D F E
Sol. (b) A E D
When two faces are common
a b c d
Third face is opposite to each other A/C/ is opposite to
white D/E.
Here (Red, yellow) are common so
green B is opposite to F.
The correct answer is option (b). Eliminate the option in which B & F are seen together or both
20. Two different positions of the same dice are shown, the six are hidden together.
faces of which are marked as A, B, C, D, E, F. select the letter mu fodYiksa dks gVk nsa ftuesa B vkSj F dks ,d lkFk ns[kk x;k gS ;k
that will be on the face opposite to the one showing ‘B’. nksuksa ,d lkFk fNis gq, gSaA
,d gh ikls ds nks vyx&vyx LFkku fn[kk, x, gSa] ftuesa ls Ng There fore option (C) is correct.
Qyd A] B] C] D] E] F ds :i esa fpfàr gSaA ml v{kj dk p;u ∴ fodYi (C) lgh gSA
djsa tks 'B' fn[kkus okys ds foijhr Qyd ij gksxkA 23. Two different positions of the same dice marked with the
numbers I, II, III, IV, V and VI are shown. Select the number
that will be on the face opposite to the face having the number
‘IV’.
la[;kvksa I] II] III] IV] V vkSj VI ls fpfàr ,d gh ikls dh nks
vyx&vyx fLFkfr;k¡ n'kkZbZ xbZ gSaA ml la[;k dk p;u djsa tks 'IV'
(a) A (b) D (c) C (d) E la[;k okys Qyd ds foijhr Qyd ij gksxhA
Sol. (c)
I VI
When two faces are common
Third face is opposite to each other III IV III I
B
Here (A, E) are common so
C (a) III (b) VI (c) II (d) V

105
Download Free PDFs & e-Books from Neon Classes App

Sol. (b) pj.k (II)% ;fn ,slh dksbZ nks vkd`fr ugha gSa ftuesa ,d leku la[;k
When two faces are common gS rks dksbZ Hkh nks vkd`fr pqusa ftlesa nks leku l[;k,¡ gks vkSj Vw
Third face is opposite to each other dksWeu fVªd dk iz;ksx djsAa
IV 26. What is the number opposite to 4?
Here (I, III) are common so
VI 4 ds foijhr la[;k D;k gS \
The correct answer is option (b). 5 5 3
Concept-IV :When two position is given of a single dice: 1 4 4
3 6 5
when No common number/ letter is common in two
figures of a single dice. b
a c

When no number is common in two figure, then we can not Sol. From (I) & (II)
determine exact opposite surfaces. We can solve this figure 3→1→2
only on the basis of possibility. 6
24. What is opposite to 2 ? 3→5→4
2 ds foijhr D;k gS\
∴ 2 is opposite 4.
27. Three different positions of the same non-standard dice are
2 6 shown. Which letter will be on the face opposite to the face
1 3 4 5 with the letter ‘Q’?
,d gh xSj&ekud ikls dh rhu vyx&vyx fLFkfr;k¡ fn[kkbZ xbZ gSaA
I II 'Q' v{kj okys psgjs ds foijhr psgjs ij dkSu lk v{kj gksxk\
(a) 4/5/6 (b) 4/3/2 T U S
(c) 6/1/2 (d) 1/2/4 S T U
Q R P
Sol. We can not determine exact opposite surfaces, we can solve
this figure only on the basis of possibility. Since 1, 2 and 3 are
adjacent to each other therefore these surfaces can never be (a) S (b) P (c) U (d) R
opposite to each other. There surfaces are opposite to hidden Sol. (c)
numbers. Figure 1 and 2, one faces are letter T is common
ge foijhr lrgksa dks fu/kkZfjr ugha dj ldrs gSa] bl ikls dks dsoy Then the opposite of other is achieved by checking
So direction rotation
laHkkouk ds vk/kkj ij gy dj ldrs gSaA pwafd 1] 2 vkSj 3 ,d&nwljs
ds fudVorhZ lrg gS blfy, ;s lrgsa ,d&nwljs ds foijhr dHkh ugha T S Q — Ist figure
gks ldrh gSaA ;s lrgs fNih gqbZ la[;kvksa ds foijhr gSA
Here 1/2/3 are opposite to 4/5/6. T R U — IInd figure
∴ 1 is opposite to 4/5/6. Q S
2 is opposite to 4/5/6. Here and are two pairs of opposite
U R
3 is opposite to 4/5/6.
Or
4 is opposite to 1/2/3.
Figure 1 and 2, letter T is common both figure
5 is opposite to 1/2/3.
So, T = T
6 is opposite to 1/2/3.
SR
25. In the following dice. What can be opposite of 2 ?
fuEu ikls esa 2 ds foijhr D;k gks ldrk gSa\ QU
Q
5 4 Then
U
The correct answer is option (c).
1 2 6 3
28. Three different positions of the same dice are shown. Select
the symbol that will be on the face opposite to the face having
(a) 4/3/6 (b) 4/3/2 ‘N’.
(c) 6/1/2 (d) 1/2/4 ,d gh ikls dh rhu vyx&vyx fLFkfr;ksa dks n'kkZ;k x;k gSA ml
fpUg dk p;u djsa tks 'N' okys Qyd ds foijhr Qyd ij gksxkA
Concept-V: When more than two figure given of a single
dice: S @ #
tc ,d ikls dh nks ls vf/kd fLFkfr;k¡ ¼fp=½ nh tkrh gSa& % N & N S &
In this case follow the steps given below:
blds fy, uhps fn, x, pj.kksa dk ikyu djs%a
(a) @ (b) & (c) % (d) #
Step (I) Try to select any two figures which have one common
Sol. (d)
number and apply one common trick.
Figure 2 and 3, symbol ‘&’ are common both figure
pj.k (I) fdUgh Hkh nks ,slh vkd`fr dk iz;ksx djsa ftlesa dsoy ,d gh
So, & = &
la[;k leku gks vkSj ou dkWeu fVªd dk iz;ksx djsAa
Step (II) If there are no such figures which have onr common @ S
number then select any two figures which have two common N #
numbers and applu two common trick.

106
Best App for Govt. Jobs : Neonclasses (Download Now)

N In the first and second dice 1 and 2 are common in both dice
Then
# 1 1
The correct answer is option (d).
3 2 2 5
29. Three different positions of a dice are shown below. Which
number appears on the face opposite number 3?
11
,d ikls dh rhu vyx&vyx fLFkfr;ksa dks uhps fn[kk;k x;k gSA
la[;k 3 ds foijhr Qyd ij dkSu lh la[;k fn[kkbZ nsrh gS\ 3  5 common
22
3
So,
5
In the 2 and 3 dice common in both dice 5 and 2.
(a) 2 (b) 5 (c) 4 (d) 6
Sol. (b) 1 6
Figure 2 and 3, number ‘6’ common both figure 2 5 5 2
So, 6 = 6
2 1 22
common
55
5 3
16
3 1
Then So, opposite face pair are
5 6
The correct answer is option (b). 3  5, 1  6, 2  4
30. Three different positions of the same dice are shown. Select
The correct answer is option (c).
the number/symbol that will be on the face opposite to the
32. Four different positions of the same dice are shown, the six
face showing the symbol ‘&’.
faces of which are numbered from 1 to 6 select the number
,d gh ikls dh rhu vyx&vyx fLFkfr;k¡ fn[kkbZ xbZ gSaA ml that will be one the face opposite to the one having the number
la[;k@fpà dk p;u djsa tks '&' fpà dks n'kkZus okys Qyd ds ‘1’
foijhr Qyd ij gksxkA ,d gh ikls dh pkj vyx&vyx fLFkfr;k¡ fn[kkbZ tkrh gSa] ftuesa ls
Ng Qydksa dks 1 ls 6 la[;kafdr fd;k tkrk gS rkfd og la[;k pqu
ldsa tks ‘1’ la[;k okys ds foijhr Qyd gksxhA
1 6
(a) 9 (b) $ (c) % (d) #
Sol. (c) 5 4 3 5
Figure 1 and 2, symbol ‘%’ common both figure
So, % = % & 2 1
# 9 4 3 4 2
$ *
(a) 6 (b) 2 (c) 4 (d) 3
&
Then Sol. (d)
%
Dice 1 and 2 are 5 numbers both dice common
The correct answer is option (c). So, 5 = 5
31. Three different positions of the same dice are shown. Study
1 3
the same and identify which of the following statements is
correct. 4 6
,d gh ikls dh rhu vyx&vyx fLFkfr;ksa dks fn[kk;k x;k gSA bld
v/;;u djsa vkSj igpkusa fd fuEufyf[kr esa ls dkSulk dFku lgh gSA 1
Then
3
1 1 6 The correct answer is option (d).
2 5 2 33. Four different positions of the same dice are shown. Select the
3 2 5
letter that will be on the face opposite to the face having the
(a) 1 is opposite to 4/ letter ‘B’.
1, 4 ds foijhr okys Qyd ij gS ,d gh ikls dh pkj vyx&vyx fLFkfr;k¡ n'kkZbZ xbZ gSaA ml v{kj
(b) 3 is opposite to 6/ dk p;u djsa tks 'B' v{kj okys psgjs ds foijhr psgjs ij gksxkA
3, 6 ds foijhr okys Qyd ij gS
(c) 6 is opposite to 1/
6, 1 ds foijhr okys Qyd ij gS
(d) 5 is opposite to 2/
5, 2 ds foijhr okys Qyd ij gS
Sol. (c) (a) E (b) C (c) D (d) F

107
Download Free PDFs & e-Books from Neon Classes App

Sol. (a) In this case each alternate surface will be opposite to each
Dice 3 and 4 are letter ‘C’ both dice common other. Alternate surfaces are taken horizontally or vertically.
So, C = C [kqys gq, ikls esa ,d nksM+ dj nwljh lrg ,d&nwljs ds foijhr gksrh
D A gSA oSdfYid lrgksa dks {kSfrt ;k yacor :i ls fy;k tkrk gSa
1 5
E B 4
2 1 2 4
4 2 5 3
B 5 3 6 3
Then 6
E
4 6
The correct answer is option (a).
34. Three different positions of a single dice are given, then what (I) (II) (III)
should come in place of question mark? 5↔6 1↔6 1↔4
,d gh iklk ds rhu vyx&vyx fp= fn, x, gSa rks iz’u fpà ds
1↔3 2↔3 5↔3
LFkku ij vkuk pkfg, \
4 5 ? 2↔4 4↔6 2↔6
2 3 1
3 1 6 Note: Two opposite surfaces can never touch each other either
by point or by any line, therefore if two surfaces have common
I II III point or common line then they can’t be opposite surfaces.
(a) 2 (b) 3 (c) 4 (d) 5 uksV% nks foijhr lrgsa ,d&nwljs dks dHkh Hkh fdlh js[kk ;k fcanq ls Nw
Sol. (d) ugha ldrh gS] blfy, ;fn nks lrgksa esa leku fcanq ;k leku js[kk gS
rks os foijhr lrg ugha gks ldrh gSaA
36. What is opposite to 5?
3→2→4
6 5 ds foijhr D;k gS\
3→5→1
4
Hence option (B) & (C) are eliminated. 5 2
Put ? = 2 and from figure (I) & (II)
nks foijhr lrg uk rks dHkh ,d lkFk fn[k lsrh gS vkSj u gh dHkh 3 1
fni ldrh gSA vr% fodYi (B) vkSj (C) xyr gSA 6
iz’u okpd fpà dh txg 2 j[kus ij
(a) 3 (b) 4 (c) 1 (d) 2
2→4→3 Sol. 3 ↔ 4.
5
2→6→1 2 ↔ 6 [Because 2 can’t be opposite to 5 & 1 as there are
common points]
Opposite numbers are not same as before, Hemce (A) is 2 ↔ 6 [D;ksfa d 2]5 vkSj 1 ds foijhr ugha gks ldrk tSlk fd leku
incorrect. Thus (D) in correct. fcanq gSa]
foijhr lrg igys tSlh ugha gS vr% fodYi (A) xyr gSA Therefore 1 is opposite to 5.
vr% fodYi (D) lgh gSA blfy, 1] 5 ds foijhr gSA
35. Three different positions of a single dice are given, then what 1 ↔ 5.
should come in place of question mark? 37. Select the dice that can be formed by folding the given sheet
,d gh iklk ds rhu vyx&vyx fp= fn, x, gSa rks iz’u fpà ds along the lines.
LFkku ij vkuk pkfg, \ ml ikls dk p;u djsa tks nh xbZ 'khV dks js[kkvksa ds vuqfn’k eksM+dj
B C ? cuk;k tk ldrk gSA
A A E 5
C D ?
6 1 4 2
I II III 3
(a) A,C (b) C,F (c) A,F (d) B,F
Sol. (d) 2 4
(a) (b)
A/C ↔ E/F
5 1 3 2
B↔D
Since B is opposite to D. Hence B & D can’t be seen together &
can’t be hidden together.
Therefore, option (D) is correct. 1 5
D;ksfa d B ds foijhr D gS vr% B vkSj D u rks dHkh ,d lkFk ns[ks tk (c) (c)
4 2 3 2
ldrs gSa vkSj u gh dHkh ,d lkFk fNi ldrs gSaA
vr% fodYi (D) lgh gSA
Sol. (b)
Concept- VI : When given dice is open dice
tc ikls dks [kksy dj fn;k tkrk gS&

108
Best App for Govt. Jobs : Neonclasses (Download Now)

5 Q #
6 1 4 2 (a) X The dice can be
3 R Q formed

Select the dice that can be formed by folding the given (b) X X and @ are opposite
opposite number @ Q so get eliminated
6 4
(c) * * and R are opposite
1 2 R @ to get eliminated

5 3 (d) 5 Q and # are opposite


3 2 so get eliminated.
(a) 2 1 and 2 the faces
5 1 opposite to each other The correct answer is option (a).
so get eliminated 39. Select the box that can be formed by folding the given sheet
along the lines.
(b) 4 The dice can be ml c‚Dl dk p;u djsa tks nh xbZ 'khV dks ykbuksa ds lkFk eksM+dj
3 2 formed cuk;k tk ldrk gSA

(c) 1 2 and 1 faces opposite


4 2 so get eliminated

(d) 5 5 and 3 the faces


3 2 opposite to each other
so get eliminated
Sol. (c)
The correct answer is option (b).
38. Select the cube that can be formed by folding the given sheet
along the lines.
ml ?ku dk p;u djsa tks nh xbZ 'khV dks js[kkvksa ds vuqfn'k eksM+dj
cuk;k tk ldrk gSA
Select the dice that can be formed by folding the given
X * opposite number
Q
R
# @

X X
(a) (b)
R Q @ Q (a) and are opposite
so get eliminated

* Q (b) and are opposite


(c) (d) so get eliminated
R @ R #
(c) The dice can be
Sol. (a) formed

X * (d) and are opposite


so get eliminated.
Q
The correct answer is option (c).
R
40. Select the box that can be formed by folding the given sheet
# @ along the lines.
ml c‚Dl dk p;u djsa tks nh xbZ 'khV dks ykbuksa ds lkFk eksM+dj
Select the dice that can be formed by folding the given
opposite number
cuk;k tk ldrk gSA
X @

* R

109
Download Free PDFs & e-Books from Neon Classes App

Select the dice that can be formed by folding the given


opposite number
Sol. (c) 6 1

3 5

4 2
(a) 6 The dice can be
3 4 formed
Select the dice that can be formed by folding the given
opposite number (b) 6 4 and 2 are opposite
4 2 so get eliminated

(c) 3 The dice can be


1 2 formed

(a) and are opposite (d) 3 5 and 3 are opposite


so get eliminated 5 4 so get eliminated.

The correct answer is option (b).


(b) and are opposite
42. Select the cube(s) that can be formed by folding the given
so get eliminated
sheet along the lines.
(c)
nh xbZ 'khV dks js[kkvksa ds vuqfn'k eksM+dj cuk, tk ldus okys ?ku
The dice can be formed
dk p;u dhft,A

(d) and are opposite


so get eliminated.

The correct answer is option (c).


41. Select the boxes that can be formed by folding the given sheet
(a) Only B and D
along the lines;
(b) Only A, B and C
mu cDlksa dk p;u djsa tks nh xbZ 'khV dks js[kkvksa ds vuqfn'k
(c) Only A
eksM+dj cuk, tk ldrs gSa(
(d) All A, B, C and D
Sol. (d)

Select the dice that can be formed by folding the given


opposite number
● ▭

◇ ▭

2 ▭
(a) ◇ The dice can be
(a) Only C and D (b) Only A and C
formed
(c) Only B and C (d) Only A, C and D
Sol. (b)
(b) The dice can be
◇ ● formed

110
Best App for Govt. Jobs : Neonclasses (Download Now)

(c) 2 The dice can be


● formed

(d) The dice can be


● 2 formed
Sol. (c)
All dice can be formed
The correct answer is option (d).
43. Select the dice that can be formed by folding the given sheet
along the lines.
Select the dice that can be formed by folding the given
ml ikls dk p;u djs]a ftls fuEukafdr i=d ('khV) dks js[kkvksa ij opposite number and symbol
eksM+us ij izkIr fd;k tk ldrk gSA
2 4

3 5

While unfolding the darker part of comes exactly above 3.


Which is correct in option (a) and (d).
Also option (b) represents correct combination in (c) we can
see 3 and does not follow the exact orientation.
(a) Only A, B and C (b) Only B and D The correct answer is option (c)
(c) Only A and B (d) A, B, C and D 45. Two orientations of the same box are shown. How will this box
Sol. (a) look when unfolded?
,d gh c‚Dl ds nks vksfj,aVs'ku fn[kk, x, gSaA [kqyk gksus ij ;g
c‚Dl dSlk fn[ksxk\

7 5
6 4 3 8

Select the dice that can be formed by folding the given 3 5


opposite number
P R 7 6 4 3 6 4

Q S 5 8

8 7
U T
(a) S The dice can be (a) (b)
P T formed

(b) Q The dice can be 3 5


R T formed 6 3 6 7
7 8
(c) Q The dice can be 4 8
U R formed
5 4
(d) R The dice can be (c) (d)
S U formed
Sol. (c)
Dice can be formed in A, B, C and D. Here can see that the faces with the digits 7, 6 and 4 are
The correct answer is option (d). adjacent to each other.
44. Choose the box that is not similar to the box formed from the
given sheet of paper when it is folded. 7 5
ml c‚Dl dk p;u djsa tks fn, x, dkxt+ dh 'khV ls cus c‚Dl ds 4 8
6 3
leku ugha gS tc bls eksM+k tkrk gSA

Options:

111
Download Free PDFs & e-Books from Neon Classes App

3 I VI
(c) 7, 6 and 4 are
6 7 8 adjacent to each V III IV II
other so correct
4 option (c) Can see that the faces with the Roman number I, III and V are
5 adjacent to each other.
(a)
5 II
(d) 6 and 4 are opposite
3 so eliminated option I and V are option
6 7 VI V III
(d) so get eliminated
8 IV

4 I

(a) 3 I
(b)
7 and 4 are opposite I and V are option
7 6 4 III II IV so get eliminated
so get eliminated
option (a)
5 V

8 V

(b) 5 (c) V
7 and 6 are opposite
3 6 4 so get eliminated I, III and V are
option (b) III I IV adjacent so each
8 other so correct
7 VI option (c)

The correct answer is option (a). II


46. Two orientations of a dice are shown. This dice can be
obtained by folding which of the option figures along the lines?
,d ikls ds nks >qdko fn[kk, x, gSaA ;g iklk fdl fodYi vk—fr (d)
VI
dks js[kkvksa ds vuqfn'k eksM+dj çkIr fd;k tk ldrk gS\
I and V are option
III V II
so get eliminated
I VI
IV
V III III II
I

The correct answer is option (c).


47. Choose the die that is similar to the die, formed from the open
II I
die.
VI V III III II IV [kqys ikls ls cuus okys ikls ds leku ikls dk p;u djsAa

IV V

I VI

(a) (b)

V VI
(x)
III I IV III V II

VI IV

II I
(1) (2) (3) (4)
(c) (d)
(a) Only 1 (b) Only 2
Sol. (c) (c) 1 and 3 (d) 1, 2 and 3
Sol. (c)

112
Best App for Govt. Jobs : Neonclasses (Download Now)

The figure number 1 is possible as the top face is shaded and Sol. Two alternate blank surfaces will be opposite to each other
adjacent faces are blank. Figure 2 is not possible as the surface while the remaining two alternate blank surfaces will be
with a dot should lie between shaded one, but here it is blank. opposite to each other.
Figure 3 is possible but the 4th one is impossible as the two nks ,dkarj fjDr lrg ,d nwljs ds foijhr gksxh tcfd 'ks"k nks
shaded faces cannot be adjacent to each other. ,dkarj fjDr lrg ,d&nwljs ds foijhr gksxhA
Hence, the correct answer is option c. ∴X↔Y
48. Which of the four cubes given below can be created by folding X & Y can not be seen together and can not be hidden together.
the given design? X vkSj Y dks ,d lkFk ugha ns[kk tk ldrk gS vkSj bls ,d lkFk ugha
fn, x, fMtkbu dks eksM+dj fuEufyf[kr pkj ?kuksa esa ls dkSu lk ?ku fNik;k tk ldrk gSA
cuk;k tk ldrk gS\ ∴ Option (A) is correct.
Question Figure @iz’u vkd`fr % 51. Which of the following options will represent the givendice
when it is folder?
fuEufyf[kr esa ls dkSu lk fodYi fn, x, ikls dk izfrfuf/kRo djsxk
tc bl ikls dks can fd;k tkrk gS\

1
D

Answer Figures @mŸkj&vkd`fr;k¡ % 4 A B C 2


E
3
(a) (b) (c) (d)
Sol. (d)
B D C C
4 4 2 2
1 3 1 3 1 3 3 1
D 2 A 2 D 4 E 4

49. The question figure is folded to form a box. Choose from the
answer figures the box that is similar to the box formed. (A) (B) (C)
(D)
iz'u vkd`fr dks eksM+dj ,d fMCck cuk;k x;k gSA mŸkj vkd`fr;ksa esa Sol. Here open dice will be folder to back side and then 1,2,3 and 4
og fMCck pqfu, tks ml fMCcs tSlk gS\ which are in clockwise direction will be seen in anticlockwise
Question Figure @iz’u vkd`fr % direction i.e. 1, 4, 3 and 2.
Or we can say that

1 1
4 2 Will become 2 4
Answer Figures @mŸkj&vkd`fr;k¡ % 3 3

Among (1, 2, 3 & 4) we can give any place to one number, then
the remaining numbers should be set accordingly.
(a) (b) (c) (d) We took 1 & we can place it at 4 places.
geus 1 fy;k vkSj ge bls 4 LFkkuksa ij j[k ldrs gSaA
Sol. (d)
1 2 3 4
2 4 3 1 4 2 1 3
3 4 1 2

Now: A ↔ C.
50. Which of the following figure is correct? D ↔ E.
fuEufyf[kr esa ls dkSulh vkd`fr lgh gS\ B ↔ ⊠.
Also remember that D & 1 will be togwether as they are
X
connected. Similarly A & 4, C & 2 and E & 3 will be together.
Y ;g Hkh ;kn j[ks fd D vkSj 1 ,d lkFk gksx a s D;ksfa d os tqM+s gq, gSaA
blh rjg A & 4, C & 2, E & 3 ,d lkFk gksx a sA
Y Option (A) is incorrect [B & ⊠ can’t come together.]
X X Y X
fodYi (A) xyr gS [B vkSj ⊠ ,d lkFk ugha gks ldrs]
Option (B) is incorrect [D should be connected with 1.]
fodYi (B) xyr gS [D dks 1 ls tksM+k tkuk pkfg, Fkk]
I II III IV Option (c) is incorrect [∵ 1, 2, 3 and 4 are in clockwise
(a) Only (III) (b) Only (III) & (IV) direction.]
(c) Only (iv) (d) All of these fodYi (C) xyr gS [ D;ksfa d 1]2]3]4 ?kM+h dh fn’kk esa gSa tcfd budks
?kM+h ds foijhr fn’kk esa gksuk pkfg, Fkk]

113
Download Free PDFs & e-Books from Neon Classes App

∴ Option (D) is correct. ;fn 2 ds foijhr 6 gS] rks 5 ds foijhr D;k gksxk \
Concept: VII : Language based question:
Hkk"kk ij vk/kkfjr iz’u 2 5
52. Four standard dice are thrown on the ground. The total of 3 6
1 4
numbers on the top faces of these four dice is 15 as the top
faces showd 3, 2, 6 and 4 respectively. What is the total of the
faces touching the ground? I II
tehu ij pkj lkekU; ikls Qsd a s tkrs gSaA bu pkj iklkvksa ds Åijh (a) 1 (b) 3 (c) 5 (d) 6
lrgksa ij dqy la[;k 15 gS D;ksfa d Åijh lrgksa us dze’k% 3, 2, 6 vkSj Sol. There is no common number in both figures. But it is given
4 fn[kk,A tehu dks Nwus okys dqy lrgksa ij la[;kvksa dk ;ksx D;k that 2 is opposite to 6. In such a situation rotate in clockwise
gksxk\ direction along the one number and rotate in anticlockwise
(a) 10 (b) 13 direction along the second number.
(c) 15 (d) can not bee determined nksuksa vkd`fr;ksa esa dksbZ la[;k leku ugha gSa ysfdu ;g fn;k gS fd 2
Sol. (b) ds foijhr 6 gSA ,slh fLFkfr esa ,d la[;k ls ?kM+h dh foijhr fn’kk esa
As we know that the sum of opposite surfaces in a standard ?kweAsa
dice is equal to 7. Therefore
;gk¡ lkekU; iklk dk vFkZ gS ekud iklk vkSj tSlk fd ge tkurs gS 2 5
fd ekud iklk foijhr lrgksa dk ;ksx 7 ds cjkcj gSA blfy,
Dice-I Dice-II Dice-III Dice-IV 1 3 4 6
Top 3 2 6 4
Bottom (7-3)=4 (7-2)=5 (7-6)=1 (7-4)=3
I II
Hence the total of bottom surfaces
In figure (I) rotate in clockwise direction along the number 2
= 4+5+1+3=13
shile in figure (II) rotate in anticlockwise direction along the
53. Two different positions of the same dice are shown, the six,
number 6 as follows.
faces of which are numbered from 1 to 6. Select the number
that will be on the face opposite to the face having the number
vkd`fr (I) esa la[;k 2 ls ?kM+h dh fn’kk esa ?kwesa tcfd vkd`fr (II) esa
‘3’ la[;k 6 ls ?kM+h dh foijhr fn’kk esa uhps fn[kk,a vuqlkj ?kweAsa
,d gh ikls ds nks vyx&vyx LFkku fn[kk, x, gSa] ftuesa ls Ng ds 2–3–1
Qyd 1 ls 6 rd fxus x, gSaA ml la[;k dk p;u djsa tks 3 la[;k 6–5–3
Hence, 2 is opposite to 6
okys Qyd ds foijhr Qyd ij gksxhA
3 is opposite to 5
1 is opposite to 4
5 5
∴ 3 is opposite to 5.
Note: In one common trick we rotateame direction along the
4 3 2 4
common number. But in case of opposite number s we rotate
in opposite directions along the opposite numbers.
uksV% ou dkWeu fVªd esa ge leku la[;k ls leku fn’kk eesa ?kwers gSA
(a) 2 (b) 1 (c) 6 (d) 4
Sol. (a)
ysfdu foijhr la[;kvksa dh fLFkfr esa ge foijhr la[;kvksa ds lkFk
When two faces are common foijhr fn’kk esa ?kwers gSA
Third faces are opposite to each other 56. Each of the six faces of a cube is numbered by one of the six
3
digits from 1 to 6. This cube is shown in its from different
Here (5, 4) are common than positions in the figures (I), (II), (III) and (IV).
2
?ku dh N% lrgksa esa ls izR;sd 1 ls 6 rd ds N^ vadksa esa ls ,d
The correct answer is option (a).
54. If 1 is adjacent to 2 and 5 is opposite to 3, then which of the
fn[kkrh gSA ;g ?ku blds pkj vyx&vyx fp=ksa (I),(II), (III) vkSj
(IV) esa fn[kk;k x;k gSA
following statements is necessarily true?
;fn 1]2 ds foijhr gS vkSj 5] 3 ds foijhr gS] rks fuEufyf[kr esa ls 1 6
6 5
dkSu lk dFku lR; gS \
3 2 4 2 6 4 2 4
(a) 4 is adjacent to 3 and 6
(b) 2 is adjacent to 4 and 6
I II III IV
(c) 4 is adjacent to 5 and 6
(d) 6 is adjacent to 3 and 4
fuEufyf[kr dFkuksa ij fopkj djs%a
Sol. It is given that Consider the following statements:
1↔2 I. Figures (II) an (III) are sufficient to know as to which face is
5↔3 opposite to the face numbered 6.
Hence, 4 ↔ 6
fp= (II) vkSj (III) ;g tkuus ds fy, i;kZIr gSa fd dkSu lh lrg 6
Since 4 is opposite 6, therefore option (A), (C) and (D) are ds foijhr gSA
incorrect. II. Figures (II) and (III) are sufficient to know as to which face
∴ Option (B) is correct. is opposite to the face numbered 4.
Concept- VIII: Conditional questions. fp= (II) vkSj (III) ;g tkuus ds fy, i;kZIr gSa fd dkSu lh lrg 4
55. If 2 is opposite to 6, then what is opposite to 5? ds foijhr gSA

114
Best App for Govt. Jobs : Neonclasses (Download Now)

III. Figures (I) and (IV) are sufficient to know as to which face (i) Arrange these words according to dictionary: Cat, Boy, Dog,
is opposite to the face numbered 3. Fish
fp= (I) vkSj (IV) ;g tkuus ds fy, i;kZIr gSa fd dkSu lh lrg 3 Cat, Boy, Dog, Fish dks 'kCndks’k ds vuqlkj O;ofLFkr dhft,A
ds foijhr gSA Sol. The first letters of these words are C, B, D, and F. If these
(a) I and III only (b) I and II only letters are arranged according to Englis alphabet then we will
(c) II and III only (d) I, II and III get B, C, D and F respectively. Hence the following order of
Sol. From II and III words will be obtained.
4→1→3
bu 'kCnks ds igys v{kj Øe’k% C, B, D, F gS ;fn bUgs 'kCndks’k ds
3 vuqlkj vFkkZr~ o.kZekyk Øe ds vuqlkj O;ofLFkr fd;k tk, rc B, C,
4→6→5 D, F izkIr gksx
a A
blfy, 'kCnksa dk fuEu Øe izkIr gksxk
1 is opposite to 6 and 3 is opposite to 4.
(a) Boy
Thus figure II and III are sufficient.
(b) Cat
Now, from figure I and IV
(c) Dog
6/2 ↔ 1/5
(d) Fish
3↔4
(ii) Arrange these words according to dictionary: Cut, Cutter, Cow,
4 is opposite to 3. Hence figure I & IV are sufficient.
Crow
Therefore all the statements are correct.
Cut, Cutter, Cow, Crow dks 'kCndks’k ds vuqlkj O;ofLFkr dhft,A
57. If 2 is opposite to 3 then what is opposite to 5?
;fn 2 ds foijhr 3 gS rc 5 ds foijhr D;k gksxk\ Sol. Here the first letters of all the words are the same, so the
second letters will be compared. The second letters of the
4 3 words are U,U,O and R respectively. If they are arranged
according to English alphabetical order, then we will get O, R,
2 6 1 5 U and U.
Therefore ‘Cow’ will come first and then ‘Crow’. Now we will
compare the words ‘Cut’ and ‘Cutter’.
I II The third letters of these two words are the same and ‘Cut’ is a
(a) 1 (b) 3 (c) 5 (d) 6 3 letter word.
Sol. 2 → 4 → 6 Therefore it will come before the word ‘Cutter’. Hence the
3→1→5 following order of the words will be obtained:
Hence option (d) is correct. ;gk¡ lHkh 'kCnks ds igys v{kj ,d leku gS blfy, nwljs v{kjksa dh
58. If A is opposite to E then what is opposite to B? rqyuk gksxhA ;gk¡ 'kCnksa ds nwljs v{kj Øe’k% U, U, O, R gS ;fn bUgs
;fn A ds foijhr E gS rc B ds foijhr D;k gksxk\ o.kZekyk Øe ds vuqlkj O;ofLFkr fd;k tk, rc O, R, U, U izkIr
gksxkA blfy, lcls igys ‘Cow’ vkSj ‘Crow’ 'kCn vk,xkA vc ge
B E ‘Cut’ vkSj ‘Cutter’ 'kCn dh rqyuk djsx a sA
A C D F bu nksuksa 'kCnksa ds rhljs v{kj Hkh ,d leku gS vkSj ‘Cut’ rhu v{kj
dk gh 'kCn gS blfy, igys ;g 'kCn vk,xkA
blfy, 'kCnks dk fuEu Øe izkIr gksxk&
I II (a) Cow
(a) A (b) D (c) E (d) F (b) Crow
Sol. A → B → C (c) Cut
E→ D → F (d) Cutter
Hence op tion (b) is correct. 1. Select the correct option that indicates the arrangement of the
DICTIONARY given words in the order in which they appear in an English
dictionary
The words in the dictionary are arranged according to English
ml fodYi dk p;u dhft,] tks uhps fn, x, 'kCnksa ds ml Øe dks
alphabetical order. To arrange some given words according to
n’kkZrk gS] ftl Øe esa vaxsth 'kCndks’k esa vkrs gSA
the dictionary, they will be compared on the basis of the first
1. Hitch 2. Home 3. Hire 4. Hippo
letters of these words. If the first letters of the words are thw
(a) 4, 3, 2, 1 (b) 4, 3, 1, 2
same, the wordswill be compared on the basis of the second
(c) 1, 2, 3, 4 (d) 1, 3, 2, 4
letters. Similarly if the second letters of the words are the
Sol. (b)
same then their third letters will be compared and so on For
e.g. (1) H IT CH
'kCndks’k esa 'kCn vaxzsth o.kZekyk Øe ds vuqlkj gksrs gSA dqN fn,
x, 'kCnksa dks 'kCndks’k ds vuqlkj O;ofLFkr djus ds fy, mu 'kCnksa (2) H O ME
ds izFke v{kj ds vk/kkj ij rqyuk dh tk,xhA
;fn 'kCnksa ds igys v{kj ,d leku gS rc 'kCnksa ds nwljs v{kj ds (3) H IR E
vk/kkj ij rqyuk dh tk,xhA blh izdkj ;fn 'kCnksa ds nwljs v{kj
,d leku gS] rc muds rhljs v{kjks dh rqyuk dh tk,xh vkSj blh (4) H IP PO
rjg ;g izfØ;k pyrh jgsxhA tSls &

115
Download Free PDFs & e-Books from Neon Classes App

(c) 1, 3, 4, 5, 2 (d) 1, 4, 3, 5, 2
1, 3, 4 2 Option a,c,d gets
Sol. (a)
eliminated
4, 3 , 1, 2 (1) M a n u s c r i p t

2. Select the correct option that indicates the arrangement of the (2) M i n o r
given words in the order in which they appear in an English
dictionary (3) M e l o d y
lgh fodYi dk p;u djsa tks fn, x, 'kCnksa dh O;oLFkk dks mlh Øe
esa bafxr djrk gS ftl Øe esa os vaxzsth 'kCndks'k esa fn[kkbZ nsrs gSa (4) M a l i c i o u s
(1) Petitionary (2) Petitioning
(3) Petition (4) Petitioners (5) M e m o r y
(5) Petitioned
(a) 1, 2, 3, 4, 5 (b) 4, 1, 2, 3, 5 Eliminate option (c) & (d) beacause start from (1) so now
(c) 3, 1, 5, 4, 2 (d) 3, 1, 4, 5, 2 alphabetical order 4, 1 _ _ _ _. word (2) is last word beacause
Sol. (c) (3) & (5) word second letter same so (2) word is last position
(1) P E T I T I O N A R Y 4, 1, 3, 5, 2.
(2) P E T I T I O N I N G 5. Select the correct option that indicates the arrangement of the
(3) P E T I T I O N given words in the order in which they appear in an English
(4) P E T I T I O N E R S dictionary.
og lgh fodYi pqusa tks fn, x, 'kCnksa ds ml Øe dks n’kkZrk gS ftls
(5) P E T I T I O N E D
Øe esa os vaxzsth 'kCndks’k esa ekStwn gksrs gSaA
1. Pediculate
3 1 4 5 2 Option a, b gets eliminated 2. Pandemonium
3. Pancytopenia
3 1 5 4 2 Option d gets eliminated 4. Panelist
3. Select the correct option that indicates the arrangement of the 5. Panegyric
given words in the order in which they appear in an English (a) 1, 3, 2, 5, 4 (b) 3, 2, 5, 4, 1
dictionary. (c) 3, 2, 4, 5, 1 (d) 2, 3, 5, 4, 1
lgh fodYi dk p;u djsa tks fn, x, 'kCnksa dh O;oLFkk dks mlh Sol. (b)
Øe esa bafxr djrk gS ftl Øe esa os vaxzsth 'kCndks'k esa fn[kkbZ nsrs (1) P ed iculate
gSa
1. Junketeered 2. Junction (2) P an demonium
3. Junketers 4. Junketeering
5. Junctures (3) P an cytopenia
(a) 2, 5, 4, 3, 1 (b) 1, 4, 2, 3, 5
(c) 2, 5, 1, 4, 3 (d) 2, 5, 3, 1, 4 (4) P an e list
Sol. (c)
(1) J U N keteered (5) P an e gyric

(2) J U N ct i on 2, 3, 4, 5 1 Option (a) gets eliminated

(3) J U N kete rs 3, 2, 4, 5 1 Option (d) gets eliminated

(4) J U N kete er ing 3, 2, 5, 4 1 Option (c) gets eliminated

(5) J U N ct ure 6. Select the correct option that indicates the arrangement of the
given words in the order in which they appear in an English
First three latters are same obtion fourth & fifth letters are dictionary.
same and sixth letters of alphabetical start so option 2, 5 _ _ 3. og lgh fodYi pqusa tks fn, x, 'kCnksa ds ml Øe dks n’kkZrk gS ftls
beacause third word’s eight letter is different. So eliminate Øe esa os vaxzsth 'kCndks’k esa ekStwn gksrs gSaA
method order is . 2, 5, 1, 4, 3. 1. Speaking 2. Standardize
4. Select the option that represents the correct order of the given 3. Southern 4. Stampede
words as they would appear in an English dictionary. 5. Spacious 6. Sovereignty
lgh fodYi dk p;u djsa tks fn, x, 'kCnksa dh O;oLFkk dks mlh (a) 3, 4, 6, 5, 1, 2 (b) 3, 6, 5, 1, 4, 2
Øe esa bafxr djrk gS ftl Øe esa os vaxzsth 'kCndks'k esa fn[kkbZ nsrs (c) 3, 6, 2, 4, 1, 5 (d) 3, 2, 6, 1, 4, 5
gSa Sol. (b)
1. Manuscript 2. Minor
3. Melody 4. Malicious
5. Memory
(a) 4, 1, 3, 5, 2 (b) 4, 1, 5, 2, 3

116
Best App for Govt. Jobs : Neonclasses (Download Now)

(1) S peaking (1) T e r m i n a l

(2) S ta ndardize (2) T e r mi n o l o g y

(3) S o uthern (3) T e mp e r a t u r e

(4) S pacious (4) T a mp e r e

(5) S o vereignty (5) T a ma r i n d

(6) T e r t i a r y
3, 6 Option a and d gets eliminated

3, 6, 5, 1 Option (c) gets eliminated 4, 5 _ _ _ _ _ _

3, 6, 5, 1, 4, 2 4, 5, 3_ _ _

7. Select the correct option that indicates the arrangement of the 4, 5, 3, 1, 2, 6


given words in the order in which they appear in an English
dictionary. 9. Select the correct option that indicates the arrangement of the
og lgh fodYi pqusa tks fn, x, 'kCnksa ds ml Øe dks n’kkZrk gS ftls given words in the order in which they appear in an English
dictionary.
Øe esa os vaxzsth 'kCndks’k esa ekStwn gksrs gSaA
ml fodYi dk p;u dhft,] tks uhps fn, x, 'kCnksa ds ml Øe dks
1. Disaster 2. Disappointment
3. Discharge 4. Disappear
n’kkZrk gS] ftl Øe esa vaxsth 'kCndks’k esa vkrs gSA
5. Disadvantage 6. Disarticulate 1. Cardinal 2. Cartography
(a) 5, 4, 2, 1, 6, 3 (b) 5, 2, 4, 6, 1, 3 3. Cardiac 4. Carrot
(c) 5, 2, 4, 1, 6, 3 (d) 5, 4, 2, 6, 1, 3 5. Callous 6. Careless
Sol. (d) (a) 5, 3, 1, 6, 4, 2 (b) 5, 3, 1, 6, 2, 4
(c) 5, 1, 3, 6, 4, 2 (d) 5, 3, 1, 4, 6, 2
(1) D I S a s t e r Sol. (a)
(1) c a r d i n a l
(2) D I S a p p o i n t m e n t
(2) c a r t o g r a p h y
(3) c a r d i a c
(3) D I S c h a r g e
(4) c a r r o t
(5) c a l l o u s
(4) D I S a p p e r e
(6) c a r e l e s s
(5) D I S a d v a n t a g e
5, _ _ _ _ _ _ Because third
letter is
(5) D I S a r t i c u l a t e
5, 3, 1 _ _ _ alphabetical

5, _ _ _ _ _ 3
5, 3, 1, 6, 4, 2
5, 4, 2 _ _ 3 10. If the following words are arranged in the order as they would
appear in an English dictionary, which word would come at
5, 4, 2, 6, 1, 3 the third place?
8. Select the correct option that indicates the arrangement of the
;fn fuEufyf[kr 'kCnksa dks mlh Øe esa O;ofLFkr fd;k tk, tSls os
given words in the order in which they appear in an English
vaxzsth 'kCndks'k esa fn[kkbZ nsrs gSa] rks dkSu lk 'kCn rhljs LFkku ij
dictionary. vk,xk\
ml fodYi dk p;u dhft,] tks uhps fn, x, 'kCnksa ds ml Øe dks 1. Place 2. Player 3. Plough
n’kkZrk gS] ftl Øe esa vaxsth 'kCndks’k esa vkrs gSA 4. Plaster 5. Plaque
1. Terminal 2. Terminology (a) Plaque(b) Player
(c) Plough(d) Plaster
3. Temperature 4. Tamarind
5. Tame 6. Tertiary Sol. (d)
(a) 4, 5, 3, 1, 2, 6 (b) 4, 5, 3, 2, 1 , 6 (1) p l a c e
(c) 4, 5, 1 , 3, 2, 6 (d) 4, 5, 3, 1 , 6, 2 (2) p l a y e r
Sol. (a) (3) p l o u g h
(4) p l a s t e r
(5) p l a q u e

117
Download Free PDFs & e-Books from Neon Classes App

1st condition word (3) eliminate now 3, 17, 19 are the


(1) I n h a b i t
sequence order so third place word is plaster.
11. If the given words are arranged as per an English dictionary, (2) I n g e n I o u s
the which word will appear second? (3) I n h e r i t
uhps fn, x, lehdj.k esa ;fn fn, x, 'kCnksa dks vaxzsth 'kCndks'k ds
vuqlkj O;ofLFkr fd;k tk,] rks dkSu lk 'kCn nwljs LFkku ij vk,xk\ (4) I n f l u e n c e
(a) Cumbersome (b) Cucumber (5) I n f a t u a t I o n
(c) Curious (d) Curtains
[5, 4, …………]  Because IInd letter is Alphabetical so get
Sol. (a)
option (a) and (c) eliminate.
(1) c u m b e r s o m e [5, 4, 2……..] Option D is eliminate.
(2) c u c u m b e r [5, 4, 2, 1, 3]  Correct optuion (b).
(3) c u r i o u s 15. Arrange the following words as per order in the dictionary?
(4) c u r t a i n s vaxszth 'kCndks’k ds Øekuqlkj fuEufyf[kr 'kCnks dks O;ofLFkr djs \
(1) VERTERBRAL (2) VERTICAL
Option (b) is first place and option (c) is second place in order.
(3) VERTICIL (4) VERTEX
12. If the given words are arranged as per an English dictionary,
(5) VERTEBENA
then which word will appear last?
(a) 51423 (b) 51432
;fn fn, x, 'kCnksa dks vaxzsth 'kCndks'k ds vuqlkj O;ofLFkr fd;k (c) 54132 (d) 54123
tk,] rks dkSu lk 'kCn vafre fn[kkbZ nsxk\ Sol. (a)
1. Disturb 2. Distance
3. Distilled 4. Distribution (1) V E R T E B R A L
5. Distort (2) V E R T I C A L
(a) Disturb (b) Distribution
(c) Distort (d) Distilled (3) V E R T I C I L
Sol. (a) (4) V E R T E X
(1) D i s t u r b (5) V E R T E B E N A
(2) D i s t a n c e [5, 4, 1 …………]  option (c) and (d) are eliminate.
(3) D i s t l l e d [5, 1, 4, 3, 2] Option (b) is eliminate.
(4) D i s t r i b u t i o n [5, 1, 4, 2, 3]  Correct optuion (a).
(5) D i s t o r t
1, 9, 15, 18, 21
Dot Situation
So word Disturb is last place here. 1. Select the figure which satisfies the same conditions of
13. Among the following words, which one will come fifth if they placement of the dots as in Figure-X.
are arranged as per their order in an English dictionary? ml vk—fr dk p;u djsa tks fcanq ds LFkku dh mUgha 'krksZa dks larq"V
fuEufyf[kr 'kCnksa esa ls dkSu&lk ,d vaxzsth 'kCndks'k esa muds Øe djrh gS tSlk fd fp=&X esa gSA
ds vuqlkj O;ofLFkr gksus ij ikaposa LFkku ij vk,xk\
1. Wink 2. Whip 3. Weary
4. Wing 5. Weigh 6. Weather
(a) Whip (b) Wing
(c) Wink (d) Weather
Sol. (b)
(1) w i n k
(2) w h i p Sol. (c) In fig. (X), the dot is contained in the region
(3) w e a r y common to the triangle and the circle only. Out of the four
(4) w i n g alternatives, only fig. (c) contains a region common to the
triangle and the circle only.
(5) w e i g h
(6) w e a t h e r
2. Select the figure which satisfies the same
Eliminate option (a) & (d) word wing option (b) is fifth place conditions of placement of the dots as in Figure-X.
in order. ml vk—fr dk p;u djsa tks fcanq ds LFkku dh mUgha 'krksZa dks larq"V
14. Arrange the following words as per order in the dictionary? djrh gS tSlk fd fp=&X esa gSA
vaxszth 'kCndks’k ds Øekuqlkj fuEufyf[kr 'kCnks dks O;ofLFkr djs \
(1) Inhabit (2) Ingenious
(3) Inherit (4) Influence
(5) Infatuation
(a) 45312 (b) 54213
(c) 45213 (d) 54312
Sol. (b)

118
Best App for Govt. Jobs : Neonclasses (Download Now)

Sol. (a) In fig. (X), one of the dots is placed in the region 6. Select the figure which satisfies the same conditions of
common to the circle and the triangle only and another dot is placement of the dots as in Figure-X.
placed in the region common to the square and the triangle ml vk—fr dk p;u djsa tks fcanq ds LFkku dh mUgha 'krksZa dks larq"V
only. In each of the three alternatives (b), (c) and (d), there is djrh gS tSlk fd fp=&X esa gSA
no region common to the square and the triangle only. Only fig.
(a) consists of both the types of regions.
3. Select the figure which satisfies the same
conditions of placement of the dots as in Figure-X.
ml vk—fr dk p;u djsa tks fcanq ds LFkku dh mUgha 'krksZa dks larq"V
djrh gS tSlk fd fp=&X esa gSA

Sol. (d) In fig. (X), one of the dots lies in the region common
to the circle and the square only, another dot lies in the region
common to the square, the triangle and the rectangle only and
the third dot lies in the region common to the triangle and the
Sol. (b) In fig. (X), one of the dots lies in the region common
rectangle only. In each of the figures (a), (b) and (c) there is no
to the circle, the square and the rectangle only, another dot lies
region common to the square, the triangle and the rectangle
in the region common to all the four elements - the circle, the
only. Only fig. (d) consists of all the three types of regions.
square, the triangle and the rectangle and the third dot lies in
7. Select the figure which satisfies the same conditions of
the region common to the circle, the triangle and the rectangle
placement of the dots as in Figure-X.
only. So only fig. (b) consists of all the three types of regions.
ml vk—fr dk p;u djsa tks fcanq ds LFkku dh mUgha 'krksZa dks larq"V
4. Question: Select the figure which satisfies the
djrh gS tSlk fd fp=&X esa gSA
same conditions of placement of the dots as in Figure-X.
ml vk—fr dk p;u djsa tks fcanq ds LFkku dh mUgha 'krksZa dks larq"V
djrh gS tSlk fd fp=&X esa gSA

Sol. (c) In fig. (X), the dot is contained in the region common
Sol. (c) In fig. (X), one of the dots lies in the triangle alone,
to the triangle and the circle only. Out of the four alternatives,
another dot lies in the region common to the square and the
only fig. (c) contains a region common to the triangle and the
triangle only and the third dot lies in the region common to the
circle only.
circle and the square only. In each of the figures (a), (b) and
8. Select the figure which satisfies the same conditions of
(d), there is no region common to the square and the triangle
placement of the dots as in Figure-X.
only. Only fig. (c) consists of all the three types of regions.
ml vk—fr dk p;u djsa tks fcanq ds LFkku dh mUgha 'krksZa dks larq"V
5. Select the figure which satisfies the same
djrh gS tSlk fd fp=&X esa gSA
conditions of placement of the dots as in Figure X.
ml vk—fr dk p;u djsa tks fcanq ds LFkku dh mUgha 'krksZa dks larq"V
djrh gS tSlk fd fp=&X esa gSA

Sol. (d) In fig. (X), one of the dots lies in the region common
to the circle and the square only and the other dot lies in the
Sol. (a) In fig. (X), one of the dots lies in the region common
region common to all the three figures -the circle, the square
to the circle and the square only, another dot lies in the region
and the triangle. So only fig. (d) consists of both the types of
common to the square, the triangle and the rectangle only and
regions.

119
Download Free PDFs & e-Books from Neon Classes App

the third dot lies in the region common to the square and the 12. Select the figure which satisfies the same conditions of
rectangle only. Hence, only fig. (a) consists of all the three placement of the dots as in Figure-X.
types of regions. ml vk—fr dk p;u djsa tks fcanq ds LFkku dh mUgha 'krksZa dks larq"V
9. Select the figure which satisfies the same conditions of djrh gS tSlk fd fp=&X esa gSA
placement of the dots as in Fiugure-X.
ml vk—fr dk p;u djsa tks fcanq ds LFkku dh mUgha 'krksZa dks larq"V
djrh gS tSlk fd fp=&X esa gSA

Sol. (d) In fig. (X), one of the dots lies in the region common
to the circle and the triangle only, another dot lies in the
Sol. (b) In fig. (X), one of the dots lies in the region common
region common to all the three figures - the circle, the square
to the circle and the triangle only, another dot lies in the
and the triangle and the third dot lies in the region common to
region common to the circle, the square and the triangle only
the circle and the square only. Hence, only fig. (d) consists of
and the third dot lies in the region common to the circle, the
all the three types of regions.
square and the rectangle only. Hence, only fig. (b) consists of
13. Select the figure which satisfies the same conditions of
all the three types of regions.
placement of the dots as in Figure-X.
ml vk—fr dk p;u djsa tks fcanq ds LFkku dh mUgha 'krksZa dks larq"V
10. Select the figure which satisfies the same conditions of
djrh gS tSlk fd fp=&X esa gSA
placement of the dots as in Figure-X.
ml vk—fr dk p;u djsa tks fcanq ds LFkku dh mUgha 'krksZa dks larq"V
djrh gS tSlk fd fp=&X esa gSA

Sol. (b) In fig. (X), one of the dots lies in the region common
to the circle and the triangle only, another dot lies in the
Sol. (a) In fig. (X), the dot is contained in the region common region common to the circle, the square and the triangle only
to the triangle and the square only. Out of the four alternatives, and the third dot lies in the rectangle alone. Hence, only fig. (b)
only fig. (a) contains a region common to the triangle and the contains all the three types of regions.
square only. 14. Select the figure which satisfies the same conditions of
11. Select the figure which satisfies the same conditions of placement of the dots as in Figure-X.
placement of the dots as in Figure-X. ml vk—fr dk p;u djsa tks fcanq ds LFkku dh mUgha 'krksZa dks larq"V
ml vk—fr dk p;u djsa tks fcanq ds LFkku dh mUgha 'krksZa dks larq"V djrh gS tSlk fd fp=&X esa gSA
djrh gS tSlk fd fp=&X esa gSA

Sol. (b) In fig. (X), the dot lies in the region common to all
Sol. (c) In fig. (X), one of the dots lies in the region common the three figures - square, triangle and circle. Out of the four
to the circle and the triangle only and the other dot lies in the alternatives, only fig. (b) contains a region common to all the
circle alone. Hence, only fig. (c) consists of both the types of three figures. Hence, fig. (b) is the answer.
regions.

120
Best App for Govt. Jobs : Neonclasses (Download Now)

15. Select the figure which satisfies the same conditions of


placement of the dots as in Figure-X.
ml vk—fr dk p;u djsa tks fcanq ds LFkku dh mUgha 'krksZa dks larq"V
djrh gS tSlk fd fp=&X esa gSA
(a) (b) (c) (d )
Sol. (c) Clearly, the question figure is embedded in answer figure
(c) only.
4. Question figure/ iz’u vkd`fr

Answer figure/ mÙkj vkd`fr


Sol. (c) In fig. (X), one of the dots lies in the triangle alone,
another dot lies in the region common to the square and the
triangle only and the third dot lies in the region common to the
circle and the square only. In each of the figures (a), (b) and
(d), there is no region common to the square and the triangle
only. Only fig. (c) consists of all the three types of regions. (a) (b) (c) (d )
Embedded figures Sol. (c) Clearly, the question figure is embedded in answer figure
(c) only.
Direction (1-40)
5. Question figure/ iz’u vkd`fr
Select the option figure which is embedded in the given figure
as its part
ml fodYi vk—fr dk p;u djsa tks nh xbZ vk—fr esa blds Hkkx ds :i
esa lfUufgr gS
1. Question figure/ iz’u vkd`fr
Answer figure/ mÙkj vkd`fr

Answer figure/ mÙkj vkd`fr

(a) (b) (c) (d )


Sol. (b) Clearly, the question figure is embedded in answer figure
(b).
(a) (b) (c) (d ) 6. Question figure/ iz’u vkd`fr
Sol. (b) Clearly, the question figure is embedded in answer figure
(b).
2. Question figure/ iz’u vkd`fr

Answer figure/ mÙkj vkd`fr


Answer figure/ mÙkj vkd`fr

(a) (b) (c) (d )


(a) (b) (c) (d )
Sol. (c) On close observation we find that the question figure is
Sol. (b) On close observation we find that the question figure is
embedded in answer figure (c).
embedded in answer figure (b).
7. Question figure/ iz’u vkd`fr
3. Question figure/ iz’u vkd`fr

Answer figure/ mÙkj vkd`fr

121
Download Free PDFs & e-Books from Neon Classes App

Sol. (b) Clearly, the question figure is embedded in answer figure


(b).
11. From the given answer figures, select the one in which the
question figure is hidden/embedded.
nh xbZ vkd`fr;ksa esa ls mls pqus ftlesa iz’u vkd`fr varfuZfgr gSA
(a) (b) (c) (d ) Question figure/ iz’u vkd`fr
Sol. (b) Clearly, the question figure is embedded in answer figure
(b).
8. In which answer figure is the given figure embedded nh xbZ iz’u
vkd`fr fdl mrj vkd`fr esa varfuZfgr gSA
Question figure/ iz’u vkd`fr
Answer figure/ mÙkj vkd`fr

Answer figure/ mÙkj vkd`fr Sol. (a) On close observation we find that the question figure is
embedded in answer figure (a).
12. From the given answer figures, select the one in which the
question figure is hidden/embedded.
nh xbZ vkd`fr;ksa esa ls mls pqus ftlesa iz’u vkd`fr varfuZfgr gSA
Question figure/ iz’u vkd`fr

Sol. (d) Clearly, the question figure is embedded in answer figure


(d) only.
9. From the given answer figures, select the one in which the
question figure is hidden/embedded.
nh xbZ vkd`fr;ksa esa ls mls pqus ftlesa iz’u vkd`fr varfuZfgr gSA
Question figure/ iz’u vkd`fr
Answer figure/ mÙkj vkd`fr

Answer figure/ mÙkj vkd`fr

Sol. (d) Clearly, the question figure is embedded in answer figure


(d) only.
13. Question figure/ iz’u vkd`fr

Sol. (a) Clearly, the question figure is embedded in answer figure


(a). Answer figure/ mÙkj vkd`fr
10. In which answer figure is the given figure embedded nh xbZ iz’u
vkd`fr fdl mrj vkd`fr esa varfuZfgr gSA
Question figure/ iz’u vkd`fr

(a) (b) (c) (d )


Sol. (a) On close observation we find that the question figure is
embedded in answer figure (a).
14. Question figure/ iz’u vkd`fr
Answer figure/ mÙkj vkd`fr

Answer figure/ mÙkj vkd`fr

122
Best App for Govt. Jobs : Neonclasses (Download Now)

(a) (b) (c) (d )


Sol. (c) Clearly, the question figure is embedded in answer figure
Answer figure/ mÙkj vkd`fr
(c) only.
15. Question figure/ iz’u vkd`fr

Answer figure/ mÙkj vkd`fr (a) (b) (c) (d )


Sol. (c) Clearly, the question figure is embedded in answer figure
(c) only.
↓ ↓ ↓ 19. Question figure/ iz’u vkd`fr

(a) (b) (c) (d )


Sol. (c) Clearly, the question figure is embedded in answer figure
(c) only.
16. Question figure/ iz’u vkd`fr
Answer figure/ mÙkj vkd`fr

Answer figure/ mÙkj vkd`fr


(a) (b) (c) (d )
Sol. (b) Clearly, the question figure is embedded in answer figure
(b).
20. Question figure/ iz’u vkd`fr

(a) (b) (c) (d )


Sol. (c) Clearly, the question figure is embedded in answer figure
(c).
17. Question figure/ iz’u vkd`fr

Answer figure/ mÙkj vkd`fr

Answer figure/ mÙkj vkd`fr

(a) (b) (c) (d )


Sol. (a) On close observation we find that the question figure is
embedded in answer figure (a).
21. Question figure/ iz’u vkd`fr
(a) (b) (c) (d )
Sol. (c) Clearly, the question figure is embedded in answer figure
(c) only.
18. Question figure/ iz’u vkd`fr

Answer figure/ mÙkj vkd`fr

123
Download Free PDFs & e-Books from Neon Classes App

Answer figure/ mÙkj vkd`fr

(a) (b) (c) (d )


Sol. (c) Clearly, the question figure is embedded in answer figure Sol. (c) Clearly, the question figure is embedded in answer figure
(c) only. (c).
22. Question figure/ iz’u vkd`fr 26. From the given answer figures, select the one in which the
question figure is hidden/embedded.
nh xbZ vkd`fr;ksa esa ls mls pqus ftlesa iz’u vkd`fr varfuZfgr gSA
Question figure/ iz’u vkd`fr

Answer figure/ mÙkj vkd`fr

Answer figure/ mÙkj vkd`fr

(a) (b) (c) (d )


Sol. (b) Clearly, the question figure is embedded in answer figure
(b).
23. Question figure/ iz’u vkd`fr
Sol. (a) On close observation we find that the question figure is
embedded in answer figure (a).
27. Question figure/ iz’u vkd`fr
Answer figure/ mÙkj vkd`fr

Answer figure/ mÙkj vkd`fr

(a) (b) (c) (d )


Sol. (c) Clearly, the question figure is embedded in answer figure
(c).
24
Question figure/ iz’u vkd`fr
(a) (b) (c) (d )
Sol. (c) Clearly, the question figure is embedded in answer figure
(c).
28. Question figure/ iz’u vkd`fr

Answer figure/ mÙkj vkd`fr

Answer figure/ mÙkj vkd`fr

Sol. (b) Clearly, the question figure is embedded in answer figure


(b).
25.
Question figure/ iz’u vkd`fr (a) (b) (c) (d )
Sol. (a) Clearly, the question figure is embedded in answer figure
(a).
29. Question figure/ iz’u vkd`fr

124
Best App for Govt. Jobs : Neonclasses (Download Now)

Answer figure/ mÙkj vkd`fr

(a) (b) (c) (d ) (a) (b) (c) (d )


Sol. (a) Clearly, the question figure is embedded in answer figure Sol. (b) Clearly, the question figure is embedded in answer figure
(a). (b).
30. Question figure/ iz’u vkd`fr 34. Question figure/ iz’u vkd`fr

Answer figure/ mÙkj vkd`fr Answer figure/ mÙkj vkd`fr

(a) (b) (c) (d )


(a) (b) (c) (d )
Sol. (a) Clearly, the question figure is embedded in answer figure
Sol. (c) Clearly, the question figure is embedded in answer figure
(a).
(c) only.
31. Question figure/ iz’u vkd`fr
35. Question figure/ iz’u vkd`fr

Answer figure/ mÙkj vkd`fr

Answer figure/ mÙkj vkd`fr

(a) (b) (c) (d )


Sol. (b) Clearly, the question figure is embedded in answer figure
(a) (b) (c) (d ) (b).
Sol. (b) Clearly, the question figure is embedded in answer figure 36. Question figure/ iz’u vkd`fr
(b).
32. Question figure/ iz’u vkd`fr

Answer figure/ mÙkj vkd`fr

Answer figure/ mÙkj vkd`fr

(a) (b) (c) (d )


(a) (b) (c) (d ) Sol. (c) Clearly, the question figure is embedded in answer figure
Sol. (b) Clearly, the question figure is embedded in answer figure (c) only.
(b). 37. Question figure/ iz’u vkd`fr
33. Question figure/ iz’u vkd`fr

Answer figure/ mÙkj vkd`fr

Answer figure/ mÙkj vkd`fr

125
Download Free PDFs & e-Books from Neon Classes App

Answer figure/ mÙkj vkd`fr

Sol. (c)
(a) (b) (c) (d ) From 1st figure to 2nd figure the inner circle rotates through 90º
Sol. (b) On close observation we find that the question figure is
anti clockwise while the black dot in outer circle is
embedded in answer figure (b).
38. Question figure/ iz’u vkd`fr
moving clockwise. Similar changes would occur from
3rd figure to 4th figure.
2. Question figure/iz’u vkd`fr

Answer figure/ mÙkj vkd`fr : :: : ?


Answer figure/ mÙkj vkd`fr

(a) (b) (c) (d )


Sol. (d) Clearly, the question figure is embedded in answer figure
(d) only. (a) (b) (c) (d)
39. Question figure/ iz’u vkd`fr
Sol. (b)
From 1st figure to 2nd figure the number of sides got one less.
Similarly we will get 4th figure (option B) from the 3rd figure.
3. Question figure/iz’u vkd`fr
Answer figure/ mÙkj vkd`fr

Answer figure/ mÙkj vkd`fr


:: :
?
(a) (b) (c) (d )
Sol. (d) Clearly, the question figure is embedded in answer figure
(d) only.
40. Question figure/ iz’u vkd`fr
(a) (b) (c) (d)
Sol. (d)
From 1st figure to 2nd figure the horizontal arrow rotates through
450 clockwise.
Answer figure/ mÙkj vkd`fr 4. Question figure/iz’u vkd`fr

Answer figure/ mÙkj vkd`fr

(a) (b) (c) (d )


Sol. (b) On close observation we find that the question figure is
embedded in answer figure (b).

Figure Analogy Sol. (b)


Which one of the Answer figures shall complete the given We get the 2nd figure after rotating the first figure through 1800.
analogy? Similarly we will get 4th figure after rotating the 3rd figure
1. Question figure/iz’u vkd`fr through 1800.
5. Question figure/iz’u vkd`fr

126
Best App for Govt. Jobs : Neonclasses (Download Now)

: :: :?
Answer figure/ mÙkj vkd`fr

Sol. (c)
From 1st figure to 2nd figure the main design rotates through 45°
clockwise and the line segments got doubled. Similarly we will
get 4th figure (option C) from the 3rd figure.
9. Question figure/iz’u vkd`fr
(a) (b) (c) (d)
Sol. (b)
We get the 2nd figure after rotating the first figure through 1800.
Similarly we will get 4th figure after rotating the 3rd figure
: :: :?
through 1800.
6. Question figure/iz’u vkd`fr
Answer figure/ mÙkj vkd`fr

Answer figure/ mÙkj vkd`fr

(a) (b) (c) (d)


Sol. (b) From first figure to second figure to the left of sign
(::), the triangle is converted into square, i.e., one side is added
Sol. (b) to each design and a circle is introduced inside the concentric
From 1st figure to 2nd figure the number of sides got one less. squares touching the sides. So, Answer figure b is the missing
Similarly we will get 4th figure (option B) from the 3rd figure. figure in the given Analogy. Hence, option B is correct.
7. Question figure/iz’u vkd`fr 10. Question figure/iz’u vkd`fr

: :: :?
Answer figure/ mÙkj vkd`fr Answer figure/ mÙkj vkd`fr

(a) (b) (c) (d) Sol. (c)


Sol. (a) From 1st figure to 2nd figure the two designs are converted
From 1st figure to 2nd figure the given circle is not changed and individually into mirror image but the middle line segment of
figure rotates 1800 . Similarly we will get 4th figure (option a) the left design remains unchanged. Similar changes would
from the 3rd figure. occur from 3rd figure to 4 th figure.
8. Question figure/iz’u vkd`fr 11. Question figure/iz’u vkd`fr

: :: :?
Answer figure/ mÙkj vkd`fr
Answer figure/ mÙkj vkd`fr

127
Download Free PDFs & e-Books from Neon Classes App

Sol. (a)
From 1st figure to 2nd figure the number of sides got one less.
Also the inner circles moved out of the figure and the outer
circle moved inside the figure. Similar changes would occur
form 3rd figure to 4th figure.
15. Question figure/iz’u vkd`fr
(a) (b) (c) (d)
Sol. (b)
From 1 st figure to 2nd figure the left and right designs move to : :: :?
the top and middle positions respectively while the middle
design moves to the bottom.
12. Question figure/iz’u vkd`fr Answer figure/ mÙkj vkd`fr

Answer figure/ mÙkj vkd`fr

(a) (b) (c) (d)


Sol. (c)
From 1st figure to 2nd figure the main design ro-tates through 45°
clockwise and the line segments got doubled. Similarly we will
get 4th figure (option C) from the 3rd figure.
16. Question figure/iz’u vkd`fr
Sol. (d)
From 1st figure to 2nd figure the horizontal arrow rotates
through 45° clockwise.
13. Question figure/iz’u vkd`fr

: :: :? Answer figure/ mÙkj vkd`fr

Answer figure/ mÙkj vkd`fr

Sol. (d)
(a) (b) (c) (d) From 1st figure to 2nd figure the left and right designs
Sol. (c) interchanged positions. Also the top and bottom designs
From 1st figure to 2nd figure the number of sides of outer figure interchanged positions .
and inner figure increased by one . Similarly we will get 4th Similar changes would occur from 3rd figure to 4th figure.
figure (option c) from the 3rd figure. 17. Question figure/iz’u vkd`fr
14. Question figure/iz’u vkd`fr

: :: :?
Answer figure/ mÙkj vkd`fr
Answer figure/ mÙkj vkd`fr

(a) (b) (c) (d)


Sol. (d)
From 1st figure to 2nd figure the two designs are converted
individually into mirror image but the middle line segment of

128
Best App for Govt. Jobs : Neonclasses (Download Now)

the left design remains unchanged. Similar changes would


occur form 3rd figure to 4th figure.
18. Question figure/iz’u vkd`fr

?
: :: :?
Answer figure/ mÙkj vkd`fr
(a) (b) (c) (d)
Sol. (c)
From 2nd figure to 1st figure the given figure gets completed by
outer lines. Similarly we will get 4th figure (option C) from the
3rd figure.
22. Question figure/iz’u vkd`fr
(a) (b) (c) (d)
Sol. (c)
From 1st figure to 2nd figure the main design converted into mirror
image and the line segments got doubled. Similarly we will get
4th figure (option C) from the 3rd figure.
19. Question figure/iz’u vkd`fr
Answer figure/ mÙkj vkd`fr

Answer figure/ mÙkj vkd`fr

Sol. (d)
The 2nd figure is the mirror image of the 1st figure. Similarly the
4th figure will be the mirror image of the 3rd figure.
Sol. (b) 23. Question figure/iz’u vkd`fr
From 1st figure to 2nd figure there lefts only third design and
the other designs are removed.
20. Question figure/iz’u vkd`fr
Answer figure/ mÙkj vkd`fr

: :: :?
Answer figure/ mÙkj vkd`fr Sol. (c)
Here the 3rd figure is the mirror image of 2nd figure. Similarly 4th
figure will be mirror image of the 1st figure.
24. Question figure/iz’u vkd`fr

(a) (b) (c) (d)


Sol. (a)
From 1st figure to 2nd figure the outer figure sides drecreased by
two (hexagon to square) and outer figure rotates through 180°
Answer figure/ mÙkj vkd`fr
clockwise . Similarly we will get 4th figure (option a) from the
3rd figure.
21.

129
Download Free PDFs & e-Books from Neon Classes App

From 1st figure to 2nd figure the main design coverted into water
image and the segment got doubled. Similarly we will get 4th
figure (option C) from the 3rd figure.
28.

?
Sol. (d)
From 1st figure to 2nd figure the outer and inner designs
interchanged positions.
25. Question figure/iz’u vkd`fr

(a) (b) (c) (d)


Sol. (b)
Answer figure/ mÙkj vkd`fr The two parts of the first figure are rearranged and joined along
the longer sides. The common side is then lost to form the
second figure.
29.

Sol. (a)
We get the 2nd figure after rotating the first figure through 180°. ?
Similarly we will get 4th figure after rotating the 3rd figure
through 180°.
26. Question figure/iz’u vkd`fr

: :: :? Sol. (c)
(a) (b) (c) (d)

The second figure is obtained from the first figure by moving the
line segment to the opposite side of the square boundary and
Answer figure/ mÙkj vkd`fr
replacing it with two similar line segments. Also, the element
in the lower-left corner gets replaced by two similar elements -
one placed in the upper-left and the other placed in the lower-
right corner.
30.

(a) (b) (c) (d)


Sol. (b)
From 1st figure to 2nd figure all the inner lines in main design
increased by one . Similarly we will get 4th figure (option b)
from the 3rd figure.
27. Question figure/iz’u vkd`fr
?

: :: : ? (a) (b) (c) (d)


Sol. (b)
Answer figure/ mÙkj vkd`fr In each row, the first figure is rotated through 900 CW to
obtain the second figure and the second figure is rotated
through 900 CW to obtain the third figure.
31.

(a) (b) (c) (d)


Sol. (c)

130
Best App for Govt. Jobs : Neonclasses (Download Now)

? Sol. (b)
(a) (b) (c) (d)

In each row, the second figure is obtained from the first figure by
adding two mutually perpendicular line segments at the centre
and the third figure is obtained from the first figure by adding
four circles outside the main figure.
35.
(a) (b) (c) (d)
Sol. (a)
In each row, the second figure is obtained from the first figure by
reversing the direction of the RHS arrow and the third figure is
obtained from the second figure by reversing the direction of

32.
both the arrows.
?

? Sol. (a)
(a) (b) (c) (d)

Each row (as well as each column) contains a figure consisting


of a circle and two line segments, a figure consisting of a circle
and three line segments and a figure consisting of a circle and
four line segments.
36.
(a) (b) (c) (d)
Sol. (d) In each row, the third figure is a combination of the first
and the second figure.
33.

?
?
(a) (b) (c) (d)
Sol. (d)
In each row, the second figure is obtained by removing the
outermost element of the first figure and the third figure is
(a) (b) (c) (d) obtained by removing the outermost element of the second
Sol. (a) figure.
In each row, the second figure is obtained from the first figure by 37.
increasing the number of smaller elements by one and the
1 1 1
third figure is obtained from the second figure by increasing 2 2 2
the number of smaller elements by one.
34.
2 2 2

1
2
1
2 ?
1 1 1 1
? 2 2 2 2

(a) (b) (c) (d)


Sol. (c)

131
Download Free PDFs & e-Books from Neon Classes App

In each column, the second figure (middle figure) is obtained by


removing the upper part of the first figure (uppermost figure)
and the third figure (lowermost figure) is obtained by
vertically inverting the upper part of the first figure.
38.
?

? (a) (b) (c) (d)


Sol. (d)
In each row, the second figure forms the innermost and the
outermost elements of the third figure and the first figure
forms the middle element of the third figure.
(a) (b) (c) (d) 42.
Sol. (d)
In each row (as well as each column), the third figure is a
combination of all the, elements of the first and the second
figures.
39.

?
(a) (b) (c) (d)
Sol. (a)
The number of components in each row either increases or
decreases from left to right. In the third row, it increases.
43.
(a) (b) (c) (d)
Sol. (b)
In each row, the second figure is obtained by shading one of the
four parts of the first figure and the third figure is obtained by
shading two out of the four parts of the first figure.
40.
?

? (a) (b) (c) (d)


Sol. (d)
In each row, there are three types of outer elements (circle, triangle
and square), three types of inner elements (circle, triangle and
square) and three types of shadings in the inner elements
(black, white and lines).
(a) (b) (c) (d) 44.
Sol. (a)
In each row, the third figure comprises of a black circle and
only those line segments which are not common to the first
and the second figures.
41.

132
Best App for Govt. Jobs : Neonclasses (Download Now)

The third figure in each row comprises of parts which are not
common to the first two figures.

Figure Classification
1. Choose the figure which is different from the rest.
(a) (b) (c) (d)
Sol. (c)
The third figure in each row comprises of the parts common to
the first two figures.
45.

(a) 1 (b) 2 (c) 3 (d) 4


(e) 5
Sol. (c)
In all other figures, the two line segments are parallel to each

? other but figure 3 is different.


2. Choose the figure which is different from the rest.

(a) (b) (c) (d)


Sol. (b) (a) 1 (b) 2 (c) 3 (d) 4
In each row, the second figure is obtained by rotating the first (e) 5
figure through 900 CW or 900 ACW and adding a circle to it. Sol. (a)
Also, the third figure is obtained by adding two circles to the
first figure (without rotating the figure).
46.
The pins, equal in number to the number of sides in the main
figure are attached to the midpoint of a side of the main figure
in case of figures (2), (3), (4) and (5). In fig. (1), these pins are
attached to a vertex of the main figure.
3. Choose the figure which is different from the rest.
?

(a) 1 (b) 2 (c) 3 (d) 4


(a) (b) (c) (d) (e) 5
Sol. (a)
Sol. (d)
In each row, the central part of the first figure rotates either
900 CW or 900 ACW to form the central part of the second
figure and the central part of the first figure rotates
through 1800 to form the central part of the third figure.
Also, in each row, there are 3 types of side elements - In all other figures, the arrow and the V sign lie towards the
rectangles, circles and triangles. black end of the main figure.
47. 4. Choose the figure which is different from the rest.
• •
• •
• •
• •
• • • •••
• •
••• •
? (a) 1
(e) 5
(b) 2 (c) 3 (d) 4

Sol. (a)
In all other figures, the lower-right quarter portion is shaded
except figure 1.
5. Choose the figure which is different from the rest.
(a) (b) (c) (d)
Sol. (b)

133
Download Free PDFs & e-Books from Neon Classes App

In each one of the figures except fig. (5), two arrows point
towards left and one arrow points towards right.
10. Choose the figure which is different from the rest.

(a) 1 (b) 2 (c) 3 (d) 4


(e) 5
Sol. (e)

(a) 1 (b) 2 (c) 3 (d) 4


(e) 5
Only in fig. (5), the arrowhead along the circumference of the Sol. (c)
circle indicates motion in an ACW direction. All other figures can be rotated into each other.
6. Choose the figure which is different from the rest. 11. Choose the figure which is different from the rest.

(a) 1 (b) 2 (c) 3 (d) 4 (a) 1 (b) 2 (c) 3 (d) 4


(e) 5 (e) 5
Sol. (d) Sol. (d)

Figure (4) is formed by four line segments while each one of


All other figures are divided into equal parts.
the other figures is formed by three line segments.
7. Choose the figure which is different from the rest.
12. Choose the figure which is different from the rest.

(a) 1 (b) 2 (c) 3 (d) 4


(a) 1 (b) 2 (c) 3 (d) 4
(e) 5
(e) 5
Sol. (b)
Sol. (c)
Figure (1) and fig. (3) can be rotated into each other and fig.
(4) and fig. (5) can be rotated into each other.
8. Choose the figure which is different from the rest.

In all other figures, the dot appears in the angle formed


between the arrow and the pin.
13. Choose the figure which is different from the rest.

(a) 1 (b) 2 (c) 3 (d) 4


(e) 5
Sol. (b)
Each one of the figures except fig. (2), consists of five
(a) 1 (b) 2 (c) 3 (d) 4
arrowheads.
(e) 5
9. Choose the figure which is different from the rest.
Sol. (a)
All other figures can be rotated into each other. (In each figure
except fig, (1), the middle element is obtained by rotating the
outer element through 900 CW and the inner element is
obtained by rotating the middle element through 900 CW).
14. Choose the figure which is different from the rest.
(a) 1 (b) 2 (c) 3 (d) 4 (e) 5
Sol. (e)

134
Best App for Govt. Jobs : Neonclasses (Download Now)

(a) 1 (b) 2 (c) 3 (d) 4


(e) 5
Sol. (b) Sol. (d)
Except (B), in all other figures two same designs are attached
together.
15. Choose the figure which is different from the rest.
Except fig. (4), in all other figures, one of the two arrows
points towards the centre of the circle.
20. Choose the figure which is different from the rest.

(a) 1 (b) 2 (c) 3 (d) 4


(e) 5
Sol. (b)
Each one of the remaining figures is composed of straight lines
only.
Sol. (c)
16. Choose the figure which is different from the rest.
Except (C), in all other options the figure has been divided into
three parts.
21. Choose the figure which is different from the rest.

(a) 1 (b) 2 (c) 3 (d) 4


Sol. (c)
(e) 5
The figure given in option (C) is symmetric while all other
Sol. (c)
figures are not symmetric. The line segment inside the square is attached to the corner
17. Choose the figure which is different from the rest.
which lies opposite to the corner to which the outer L-shaped
element is attached.
22. Choose the figure which is different from the rest.

(a) 1 (b) 2 (c) 3 (d) 4


(e) 5
Sol. (c)
Only in fig. (3), the marked angles are unequal. (a) 1 (b) 2 (c) 3 (d) 4
18. Choose the figure which is different from the rest. (e) 5
Sol. (a)
Only in fig. (1), two of the four elements are oriented in the
same direction.
23. Choose the figure which is different from the rest.

Sol. (d)
Except (D), all other figures have two types of designs (1 circle and
1 triangle). The figure in option (D) has three types of designs
(1 circle and 2 types of triangles). (a) 1 (b) 2 (c) 3 (d) 4
19. Choose the figure which is different from the rest. (e) 5
Sol. (a)
All other figures can be rotated into each other.
24. Choose the figure which is different from the rest.

135
Download Free PDFs & e-Books from Neon Classes App

Except (A), all other figures there are one vertical line and two
slant lines while in option (A) there are one horizontal line and
two slant lines.
29. Choose the figure which is different from the rest.

(a) 1 (b) 2 (c) 3 (d) 4


(e) 5
Sol. (c)
All other figures can be rotated into each other.
25. Choose the figure which is different from the rest.
Sol. (a)
Except (A), all other figures are similar as they can be rotated
into each other.
30. Choose the figure which is different from the rest.

(a) 1 (b) 2 (c) 3 (d) 4


(e) 5
Sol. (d)

(a) 1 (b) 2 (c) 3 (d) 4


(e) 5
Sol. (c)
Except fig. (4), in all other figures, the elements are arranged
in the increasing order of the number of sides sequentially,
either in a CW or in an ACW direction.
26. Choose the figure which is different from the rest.
All other figures can be rotated into each other.
31. Choose the figure which is different from the rest.

(a) 1 (b) 2 (c) 3 (d) 4


(e) 5
Sol. (e) (a) 1 (b) 2 (c) 3 (d) 4
Sol. (d)

In all other figures, the lower element is an alphabet and the


upper element is a number which indicates the position of the Only fig. (4) contains two circles while all other figures have
lower element in the set of English alphabet. only one circle.
27. Choose the figure which is different from the rest. 32. Choose the figure which is different from the rest.

(a) 1 (b) 2 (c) 3 (d) 4 (a) 1 (b) 2 (c) 3 (d) 4


(e) 5 (e) 5
Sol. (d) Sol. (b)
Only in fig. (4), all the four cup-shaped elements are opening in All other figures can be rotated into each other.
different directions. 33. Choose the figure which is different from the rest.
28. Choose the figure which is different from the rest.

Sol. (b)
Sol. (a)

136
Best App for Govt. Jobs : Neonclasses (Download Now)

In all other figures two different designs partially intersect Sol. (a)
each other. So , correct answer will be option B All other figures can be rotated into each other.
34. Choose the figure which is different from the rest. 39. Choose the figure which is different from the rest.

(a) 1 (b) 2 (c) 3 (d) 4 (a) 1 (b) 2 (c) 3 (d) 4


(e) 5 (e) 5
Sol. (a) Sol. (c)
All other figures can be rotated into each other.
35. Choose the figure which is different from the rest.

Only in fig. (4), all the leaves have their outer halves shaded.
40. Choose the figure which is different from the rest.

(a) 1 (b) 2 (c) 3 (d) 4


(e) 5
Sol. (d)
(a) 1 (b) 2 (c) 3 (d) 4
(e) 5
Sol. (c)
Only in fig. (4), the black triangle and the black circle lie at the Except (C), all figures have opposite figure.
two ends of the same diameter. 41. Choose the figure which is different from the rest.
36. Choose the figure which is different from the rest.

(a) 1 (b) 2 (c) 3 (d) 4


(e) 5
Sol. (c)
Sol. (a) In each one of the figures except fig. (3), the two crosses (x)
Except (A), all other figures are the same. appear in the diagonally opposite corners.
37. Choose the figure which is different from the rest. 42. Choose the figure which is different from the rest.

(a) 1 (b) 2 (c) 3 (d) 4


(e) 5 Sol. (b)
Sol. (b) Except (B) all other figures are formed using six lines.
43. Choose the figure which is different from the rest.

In each one of the figures, except fig. (2), the number of sides
in the inner element is one more than the number of sides in
the outer element.
38. Choose the figure which is different from the rest.

Sol. (d)
Except (D), in all other options there are similar figures at
equal distance.
44. Choose the figure which is different from the rest.

(a) 1 (b) 2 (c) 3 (d) 4


(e) 5

137
Download Free PDFs & e-Books from Neon Classes App

(a) 1 (b) 2 (c) 3 (d) 4 Sol. (d)


(e) 5 Except (D), in all other options, the figures have been divided
Sol. (d) into six parts.
Figure formation
Directions (1- 21): Which answer figure can be formed from
the pieces given in question figure.
Figure (4) has a rectangle in place of a '+' sign. fn;s x;s iz'uksa esa VqdM+ks ls pkj mÙkj vkd`fr;ksa esa ls dkSu lh mÙkj vkd`fr
45. Choose the figure which is different from the rest. cu ldrh gS\
1.

(a) 1 (b) 2 (c) 3 (d) 4


(e) 5
Sol. (c)
Only in fig. (3), the line segment is not a diameter of the circle. A B C D
46. Choose the figure which is different from the rest. Sol. (a) Figure given in option (a) can be formed by joining the
pieces given in the question figure .
2.

(a) 1 (b) 2 (c) 3 (d) 4


(e) 5
Sol. (a)
All other figures can be rotated into each other. A B C D
47. Choose the figure which is different from the rest.
Sol. (a) Figure given in option (a) can be formed by joining the
pieces given in the question figure .
3.

(a) 1 (b) 2 (c) 3 (d) 4


(e) 5
Sol. (a)
In all other figures, as we move from the innermost to the
outermost element, the numbers of sides of the elements A B C D
either increase or decrease in a sequence. Sol. (b) Figure given in option (b) can be formed by joining the
48. Choose the figure which is different from the rest. pieces given in the question figure .
4.

(a) 1 (b) 2 (c) 3 (d) 4


(e) 5
Sol. (d)
A B C D
Only in fig. (4), both the parallel lines are bent in the same
Sol. (d) Figure given in option (d) can be formed by joining the
direction (i.e. towards the left).
pieces given in the question figure .
49. Choose the figure which is different from the rest.
5.

138
Best App for Govt. Jobs : Neonclasses (Download Now)

A B C D
Sol. (c) Figure given in option (c) can be formed by joining the
pieces given in the question figure .
11.
A B C D
Sol. (b) Figure given in option (b) can be formed by joining the
pieces given in the question figure .
6.

A B C D
Sol. (b) Figure given in option (b) can be formed by joining the
pieces given in the question figure .
12.
A B C D
Sol. (c) Figure given in option (c) can be formed by joining the
pieces given in the question figure .
7.

A B C D
Sol. (b) Figure given in option (b) can be formed by joining the
pieces given in the question figure .
8.

Sol. (b) Figure given in option (b) can be formed by joining the
pieces given in the question figure .
13.

A B C D
Sol. (c) Figure given in option (c) can be formed by joining the
pieces given in the question figure .
9.

A B C D
Sol. (c) Figure given in option (c) can be formed by joining the
pieces given in the question figure .
10.

139
Download Free PDFs & e-Books from Neon Classes App

Sol. (a) Figure given in option (a) can be formed by joining the
pieces given in the question figure .
14.

Sol. (a) Figure given in option (a) can be formed by joining the
pieces given in the question figure .
17.

Sol. (a) Figure given in option (a) can be formed by joining the
pieces given in the question figure .
15.

Sol. (b) Figure given in option (b) can be formed by joining the
pieces given in the question figure .
18.

Sol. (b) Figure given in option (b) can be formed by joining the
pieces given in the question figure .
16.

140
Best App for Govt. Jobs : Neonclasses (Download Now)

Sol. (c) Figure given in option (c) can be formed by joining the
pieces given in the question figure .
19.

Sol. (c) Figure given in option (c) can be formed by joining the
pieces given in the question figure .
Directions (22- 31):
By which answer figure, question figure is formed.
bl iz’u eas fn, x, pkj fodYiksa esa ls dkSu&lk fodYi iz’u esa nh
xbZ vkd`fr dks lgh&lgh cuk ldrk gS\
22.

Sol. (a) Figure given in option (a) can be formed by joining the
pieces given in the question figure .
20.

A B C D
Sol. (c) Here, the question figure can be formed by joining the
pieces given in option (c) .
23.

• •
• •

A B C D
Sol. (c) Here, the question figure can be formed by joining the
pieces given in option (c) .
24.

Sol. (a) Figure given in option (a) can be formed by joining the
pieces given in the question figure .
21.

141
Download Free PDFs & e-Books from Neon Classes App



• •

A B C D A B C D
Sol. (a) Here, the question figure can be formed by joining the Sol. (a) Here, the question figure can be formed by joining the
pieces given in option (a) . pieces given in option (a) .
25. 30.

A B C D A B C D
Sol. (c) Here, the question figure can be formed by joining the Sol. (b) Here, the question figure can be formed by joining the
pieces given in option (c) . pieces given in option (b) .
26. 31.

A B C D A B C D
Sol. (c) Here, the question figure can be formed by joining the Sol. (d) Here, the question figure can be formed by joining the
pieces given in option (c) . pieces given in option (d) .
27.
Figure Series
1. Select the figure from among the given options that can
replace the question mark (?) in the following series.
fn, x, fodYiksa esa ls ml vk—fr dk p;u djsa tks fuEufyf[kr
J`[a kyk esa ç'u fpà ¼\½ dks çfrLFkkfir dj ldrk gSA

A B C D
Sol. (d) Here, the question figure can be formed by joining the
pieces given in option (d) .
28.

Sol. (c)
Figure rotates 450 clockwise in every step. so correct option
(c)

2. Select the figure from among the given options that can
A B C D replace the question mark (?) in the following series.
Sol. (b) Here, the question figure can be formed by joining the fn, x, fodYiksa esa ls ml vk—fr dk p;u djsa tks fuEufyf[kr
pieces given in option (b) . J`[a kyk esa ç'u fpà ¼\½ dks çfrLFkkfir dj ldrk gSA
29.

142
Best App for Govt. Jobs : Neonclasses (Download Now)

Sol. (d) Clockwise 45° rotation joint/both figure letter and symbol so
Figure rotates 900 clockwise in every step. so correct option correct option (a)
(c)

6. Which of the option figures rotated 180° clockwise and


3. Which of the option figures, when rotated 135° clockwise then 45° anticlockwise will result in the given Question
followed by 45° anticlockwise, will result in the given figure?
Question figure? dkSu lh fodYi vk—fr 180° nf{k.kkorZ ?kqekbZ xbZ vkSj fQj 45°
dkSulh fodYi vkd`fr] tc 135° nf{k.kkorZ vkSj mlds ckn 45° okekorZ ?kqekbZ xbZ rks nh xbZ ç'u vk—fr esa ifj.kke gksxk\
okekorZ ?kqekbZ tkrh gS] rks nh xbZ iz’u vkd`fr izkIr gksxh\

Sol. (a)
Sol. (b) 135° clockwise rotation so the given Question figures. So the
Clockwise rotation in 90° angle so correct option (b) correct option (a).
Clock wise Anti clockwise Clock wise Anti clockwise
135° 45° 180° 45°
135–45=90° (Clockwise 180–45=135° (Clockwise)

7. Which of the option figures when rotated 270°


anticlockwise and then 45° clockwise will result in the
given Question figure?
4. Select the figure from among the given options that can 270° okekorZ ?kqekus ij vkSj fQj 45° nf{k.kkorZ ?kqekus ij dkSu lh
replace the question mark (?) in the following series.
fodYi vk—fr nh xbZ ç'u vk—fr esa ifj.kr gksxh\
fn, x, fodYiksa esa ls ml vk—fr dk p;u djsa tks fuEufyf[kr
J`[a kyk esa ç'u fpà ¼\½ dks çfrLFkkfir dj ldrk gSA

Sol. (b)
All position anticlockwise rotation 45°, 90°, 135° and 180° so Sol. (a)
correct option (b) 225° Anticlockwise rotation so the given Question figures. So
the correct option (a).
Clock wise Anti clockwise
45° 270°
270–45=225° (Clockwise)
5. Select the figure from among the given options that can
replace the question mark (?) in the following series.
fn, x, fodYiksa esa ls ml vk—fr dk p;u djsa tks fuEufyf[kr
J`[a kyk esa ç'u fpà ¼\½ dks çfrLFkkfir dj ldrk gSA 8. Which of the option figures when rotated 135° clockwise
and then 45° anticlockwise will result in the given
Question figure?
135° nf{k.kkorZ vkSj fQj 45° okekorZ ?kqek, tkus ij dkSulh fodYi
vkd`fr nh xbZ iz’u vkd`fr ds tSlh gks tk,xh\

Sol. (a)

143
Download Free PDFs & e-Books from Neon Classes App

Sol. (c)
Dark quarter change the position in next quarter and
arrowhead change the position anticlockwise and 90° rotation
so correct option (c).

Sol. (c)
90° clockwise rotation so the given Question figures. So the 12. Select the figure from among the given options that can
correct option (c). replace the question mark (?) in the following series.
Clock wise Anti clockwise fn, x, fodYiksa esa ls ml vk—fr dk p;u djsa tks fuEufyf[kr
135° 45° J`[a kyk esa ç'u fpà ¼\½ dks çfrLFkkfir dj ldrk gSA
135–45=90° (Clockwise)

9. Select the figure from among the given options that can
replace the question mark (?) in the following series.
fn, x, fodYiksa esa ls og vkd`fr pqusa tks fuEufyf[kr J`a[kyk esa Sol. (c)
iz’uokpd (?) ds LFkku ij vk ldrh gSA Arrowhead mid position changed anticlockwise 90°angle.
Square and heart shap anticlockwise rotation 90° angle. So
correct opition (c).

13. Select the figure that will replace the question mark (?) in
Sol. (d) the following figure series.
Hexagon figure are same but line is changed incorrect so ml vk—fr dk p;u djsa tks fuEufyf[kr vk—fr J`[a kyk esa ç'u
correct option (d). fpà (?) dks çfrLFkkfir djsxhA

10. Select the figure from among the given options that can
replace the question mark (?) in the following series.
fn, x, fodYiksa esa ls og vkd`fr pqusa tks fuEufyf[kr J`a[kyk esa
iz’uokpd (?) ds LFkku ij vk ldrh gSA
Sol. (c)
Star changed in position row to row and other two diagram
changed in row to row. So correct option (c).

14. Select the figure from among the given options that can
Sol. (b) replace the question mark (?) in the following series.
Clock wise 45° rotation so the given Question figures. So the fn, x, fodYiksa esa ls ml vk—fr dk p;u djsa tks fuEufyf[kr
correct option (b). J`[a kyk esa ç'u fpà ¼\½ dks çfrLFkkfir dj ldrk gSA

11. Select the figure that will replace the question mark (?) in
the following figure series.
ml vkd`fr dk p;u djs]a tks fuEu vkd`fr Ja`[kyk esa iz’uokpd
fpgu (?) ds LFkku ij vk,xhA Sol. (a)
Dark square present or left top corner is moved to middle
from figure to figure.
Dark hexgon on top right corner moves three steps anti-
clockwise from figure to figure.

144
Best App for Govt. Jobs : Neonclasses (Download Now)

Dark star present on below middle moves three steps anti- 18. Select the figure from among the given options that can
clockwise from figure to figure. replace the question mark (?) in the following series.
Dark triangle in middle moves to top right corner from figure fn, x, fodYiksa esa ls og vkd`fr pqusa tks fuEufyf[kr J`a[kyk esa
to figure. So correct option (a). iz’uokpd (?) ds LFkku ij vk ldrh Gsa

15. Select the figure from among the given options that can
replace the question mark (?) in the following series.
fn, fodYiksa esa ls ml la[;k dk p;u djsa tks fuEufyf[kr J`a[kyk esa
Sol. (c)
iz’uokpd fpUg (?) ds LFkku ij vk ldrh gSA
Number of line/shape increment next figure alternative
previous figure is eliminated so correct option (c).

19. Select the figure from among the given options that can
replace the question mark (?) in the following series.
fn, x, fodYiksa esa ls og vkd`fr pqusa tks fuEufyf[kr J`a[kyk esa
Sol. (b) iz’uokpd (?) ds LFkku ij vk ldrh gSA
All figure changed next position but oval figure changed next
same position, rectangle figure changed next position and 90°
rotation, arrow 90° rotation next position and dark arrow 90°
rotation next position. So correct option (b).

16. Select the figure from among the given options that can Sol. (c)
replace the question mark (?) in the following series. Arrowhead increment and rotation anticlockwise and
fn, x, fodYiksa esa ls og vkd`fr pqusa tks fuEufyf[kr J`a[kyk esa clockwise direction so correct option (c).
iz’uokpd (?) ds LFkku ij vk ldrh gSA

20. Select the figure from among the given options that can
replace the question mark (?) in the following series.
fn, x, fodYiksa esa ls og vkd`fr pqusa tks fuEufyf[kr J`a[kyk esa
iz’uokpd (?) ds LFkku ij vk ldrh gSA

Sol. (c)
All symbol clockwise alternative interchange position so
correct option (c).

Sol. (b)
Symbol increment and number of symbol increment correct
17. Select the figure that will replace the question mark (?) in option (b).
the following figure series.
ml vk—fr dk p;u djsa tks fuEufyf[kr vk—fr J`[a kyk esa ç'u
fpà (?) dks çfrLFkkfir djsxhA
21. Select the figure from among the given options that can
replace the question mark (?) in the following series.
fn, fodYiksa esa ls ml la[;k dk p;u djsa tks fuEufyf[kr J`a[kyk esa
iz’uokpd fpUg (?) ds LFkku ij vk ldrh gSA

Sol. (c)
Top symbol left to right interchanged and bottom symbol right
to left interchanged so correct option (c).
Sol. (b)
Letter increment and line increment next figure so correct
option (b).

145
Download Free PDFs & e-Books from Neon Classes App

Answer figure/ mÙkj vkd`fr

22. Select the figure from among the given options that can
replace the question mark (?) in the following series. (a) (b) (c) (d )
fn, x, fodYiksa esa ls og vkd`fr pqusa tks fuEufyf[kr J`a[kyk esa Sol. (d)
iz’uokpd (?) ds LFkku ij vk ldrh gSA According to above given Question figures , we can say that
In each subsequent figure one line segment is added to the main
design, i.e., the triangle becomes square (quadrilateral) in the
next figure. Again, the oval rotates through 900 in each
subsequent figure.
Hence , figure ( d ) will come on the place of ? from Answer figures
.
26. Question figure/ iz’u vkd`fr
Sol. (b)
Arrowhead decrement and other symbol increment so correct
option (b) ?
Answer figure/ mÙkj vkd`fr

23. Question figure/ iz’u vkd`fr

?
(a) (b) (c) (d )
Answer figure/ mÙkj vkd`fr
Sol. (d)
According to above given Question figures , we can say that
In each subsequent figure one line segment is added to the right
side.
Therefore , figure ( d ) will come on the place of ? in Question
figure .As shown in Answer figures .
(a) (b) (c) (d )
27. Question figure/ iz’u vkd`fr
Sol. (d)
According to above given Question figures , we can say that
The three types of designs move upward step wise and the
uppermost design becomes the lowermost design in each
subsequent figure.
?
Thus , figure ( d ) will come on the place of ? in Question figure . As
shown in Answer figures . Answer figure/ mÙkj vkd`fr
24. Question figure/ iz’u vkd`fr

?
Answer figure/ mÙkj vkd`fr
(a) (b) (c) (d )
Sol. (d)
On the basis of given figures in above question , we can see
(a) (b) (c) (d ) that
Sol. (c) In each subsequent figure all the designs move one step in
On the basis of given figures in above question , we can see anticlockwise direction.
that Hence , figure ( d ) will come on the place of ? from Answer figures
In each subsequent figure the design rotates 450 in clockwise .
direction. 28. Question figure/ iz’u vkd`fr
Hence , figure ( c ) will come on the place of ? from Answer figures
.

?
25. Question figure/ iz’u vkd`fr

146
Best App for Govt. Jobs : Neonclasses (Download Now)

Answer figure/ mÙkj vkd`fr

(a) (b) (c) (d )


Sol. (c)
(a) (b) (c) (d ) As per the given figure in above question, we can say that
Sol. (d) In each subsequent figure the main design rotates through 45°
On the basis of given figures in above question , we get anticlockwise. Similarly, the three smaller designs move in
In each subsequent figure the three small squares move in anticlockwise direction. Therefore , figure ( c ) will come on
anticlockwise direction. the place of ? in Question figure .As shown in Answer figures
Thus , figure ( d ) will come on the place of ? in Question figure .As .
shown in Answer figures . 32. Question figure/ iz’u vkd`fr
29. Question figure/ iz’u vkd`fr

?
? Answer figure/ mÙkj vkd`fr
Answer figure/ mÙkj vkd`fr

(a) (b) (c) (d )


Sol. (a)
(a) (b) (c) (d ) As per the given figure in above question, it is clear that In
Sol. (a) each subsequent figure one dot is increasing and one curve is
The given line diagram is increased from corner to corner in being deleted. Thus , figure ( a ) will come on the place of ? in
every step. Question figure .As shown in Answer figures .
30. Question figure/ iz’u vkd`fr 33. Question figure/ iz’u vkd`fr

? ?
Answer figure/ mÙkj vkd`fr Answer figure/ mÙkj vkd`fr

(a) (b) (c) (d ) (a) (b) (c) (d )


Sol. (b) Sol. (b)
As per the given figure in above question, we can say that
In each subsequent figure the main design rotates through 45° In this problem, the line segment in subsequent figure is rotating in
anticlockwise. Similarly, dark dots present in the figure is 90° angle clockwise. Also two arrow heads are added one at
decreased by one in every step. Therefore , figure ( b ) will each end. So, Answer figure 2 is the missing figure in the
come on the place of ? in Question figure .As shown in given series. Hence option b is correct.
Answer figures . 34. Question figure/ iz’u vkd`fr
31. Question figure/ iz’u vkd`fr

?
? Answer figure/ mÙkj vkd`fr

Answer figure/ mÙkj vkd`fr

147
Download Free PDFs & e-Books from Neon Classes App

(a) (b) (c) (d )


Sol. (b) (a) (b) (c) (d )
In this problem, one hand of the clock is moving clockwise and the Sol. (c)
other in anticlockwise direction. So, Answer figure 2 is the In this problem, the square is rotating at 45° angle in clockwise
missing figure in the given series. Hence option b is correct. direction. White and black dots are moving in anti-clockwise
35. Question figure/ iz’u vkd`fr direction. So, Answer figure c is the missing figure in the
0 × × given series. Hence option c is correct.
×
× 0 0 ? 38. Question figure/ iz’u vkd`fr

Answer figure/ mÙkj vkd`fr


0
?
0 ×
×
0 Answer figure/ mÙkj vkd`fr
× × 0
(a) (b) (c) (d )
Sol. (a)
Here, in each subsequent figure the small circle moves along the
side of pentagon in anticlockwise direction while the cross
sign moves in clockwise direction. So, Answer figure a is the (a) (b) (c) (d )
missing figure in the given series. Hence option a is correct. Sol. (b)
36. Question figure/ iz’u vkd`fr In this question, the bigger design rotates through 90° clockwise
and moves along the diagonal. The small designs moves one
step in clockwise direction and the shaded part moves in

? anticlockwise direction. So, Answer figure b is the missing


figure in the given series.Hence option b is correct.
39. Question figure/ iz’u vkd`fr

Answer figure/ mÙkj vkd`fr


?
Answer figure/ mÙkj vkd`fr

(a) (b) (c) (d )


Sol. (c)
Here, in each subsequent figure the main design rotates through
(a) (b) (c) (d )
45° clockwise and one leaflet is added. So, Answer figure c is Sol. (d)
the missing figure in the given series. Hence option C is Here, in each subsequent figure one line segment is deflected in a
correct. set pattern. So, Answer figure d is the missing figure in the
37. Question figure/ iz’u vkd`fr given series. Hence option d is correct.
40. Question figure/ iz’u vkd`fr

?
?
Answer figure/ mÙkj vkd`fr
Answer figure/ mÙkj vkd`fr

148
Best App for Govt. Jobs : Neonclasses (Download Now)

(a) (b) (c) (d )


(a) (b) (c) (d ) Sol. (b)
Sol. (a) Both the vertical lines given in first figure is increased by one in
Here, in subsequent figure the rectangle moves from right to left in every next step.
one step and then moves to the right stepwise. So, Answer
figure a is the missing figure in the given series. 44. Question figure/ iz’u vkd`fr
Hence option A is correct.
41. Question figure/ iz’u vkd`fr

?
? Answer figure/ mÙkj vkd`fr

Answer figure/ mÙkj vkd`fr

(a) (b) (c) (d )


Sol. (c)
(a) (b) (c) (d ) Diagonal line is in increasing order in every step.
Sol. (b) 45. Question figure/ iz’u vkd`fr
Here, in each subsequent figure the designs move from left to right
and the right most design moves to the left most place while
the shading pattern changes in the reverse order. So, Answer
figure b is the missing figure in the given series. Hence option
b is correct. Answer figure/ mÙkj vkd`fr
42. Question figure/ iz’u vkd`fr

?
(a) (b) (c) (d )
Answer figure/ mÙkj vkd`fr
Sol. (d)
Mirror image of the circle is draw in next step and circle is
increased by one in every step. This pattern is follow in same
manner in every next step.

Grouping of images
(a) (b) (c) (d ) 1. Group the given figures into three classes using each figure
Sol. (d) only once.
Here, the triangle moves two steps in anticlockwise direction while çR;sd vk—fr dk dsoy ,d ckj ç;ksx djrs gq, nh xbZ vk—fr;ksa dks rhu
the circle moves one step in anticlockwise direction. So, oxksZa esa lewfgr dhft,A
Answer figure d is the missing figure in the given series.
Hence option d is correct.
43. Question figure/ iz’u vkd`fr

?
(a) 1,4 ; 2,3 ; 5,6
Answer figure/ mÙkj vkd`fr (b) 1,5 ; 2,6 ; 4,3
(c) 1,6 ; 2,3 ; 4,5
(d)1,2 ; 3,6 ; 4,5
Sol. (a) (1,4),(2,3) and (5,6)are three different pairs of
identical figures.

149
Download Free PDFs & e-Books from Neon Classes App

2.Group the given figures into three classes using each figure
only once.
çR;sd vk—fr dk dsoy ,d ckj ç;ksx djrs gq, nh xbZ vk—fr;ksa dks rhu
oxksZa esa lewfgr dhft,A

(a) 1,2,3 ; 4,5,6 ; 7,8,9


(b) 1,3,5 ; 2,4,6 ; 7,8,9
(c) 1,5,9 ; 3,6,2 ; 4,7,8
(d) 1,9,7 ; 2,8,5 ; 3,4,6
Sol. (a)
1, 2, 3 are figures composed of two straight lines.
(a) 1,3,9 ; 2,5,6 ; 4,7,8 4, 5, 6 are figures composed of three straight lines.
(b) 1,3,9 ; 2,7,8 ; 4,5,6 7, 8, 9 are figures composed of four straight lines.
(c) 1,2,4 ; 3,5,7 ; 6,8,9 5.Group the given figures into three classes using each figure
(d) 1,3,6 ; 2,4,8 ; 5,7,9 only once.
Sol. (a) çR;sd vk—fr dk dsoy ,d ckj ç;ksx djrs gq, nh xbZ vk—fr;ksa dks rhu
1, 3, 9 have one element placed inside a different element. oxksZa esa lewfgr dhft,A
2, 5, 6 contain two mutually perpendicular lines dividing the
figure into four parts.
4, 7, 8 have two similar elements (unequal in size) attached to
each other.
3.Group the given figures into three classes using each figure
only once.
çR;sd vk—fr dk dsoy ,d ckj ç;ksx djrs gq, nh xbZ vk—fr;ksa dks rhu
oxksZa esa lewfgr dhft,A

(a) 1,4,7 ; 2,5,8 ; 3,6,9


(b) 1,4,7 ; 2,5,9 ; 3,6,7
(c) 1,3,4 ; 2,5,8 ; 6,7,9
(d) 1,2,3 ; 4,5,6 ; 7,8,9
Sol. (a)
1, 4, 7 are all (two-dimensional) quadrilaterals.
2, 5, 8 are all three-dimensional figures.
3, 6, 9 are all (two-dimensional) triangles.
6.Group the given figures into three classes using each figure
(a) 7,8,9 ; 2,4,3 ; 1,5,6 only once.
(b) 1,3,2 ; 4,5,7 ; 6,8,9 çR;sd vk—fr dk dsoy ,d ckj ç;ksx djrs gq, nh xbZ vk—fr;ksa dks rhu
(c) 1,6,8 ; 3,4,7 ; 2,5,9 oxksZa esa lewfgr dhft,A
(d) 1,6,9 ; 3,4,7 ; 2,5,8
Sol. (d)
1, 6, 9, are all triangles.
3, 4, 7 are all four-sided figures.
2, 5, 8 are all five-sided figures.
4.Group the given figures into three classes using each figure
only once.
çR;sd vk—fr dk dsoy ,d ckj ç;ksx djrs gq, nh xbZ vk—fr;ksa dks rhu
oxksZa esa lewfgr dhft,A

(a) 1,5,7 ; 2,4,6 ; 3,9,8


(b) 1,5,7 ; 2,4,8 ; 3,6,9
(c) 1,4,7 ; 2,5,8 ; 3,6,9
(d) 1,7,9 ; 3,5,8 ; 2,4,6
Sol.(b)
1, 5, 7 have two similar elements, one inside the other.

150
Best App for Govt. Jobs : Neonclasses (Download Now)

2, 4, 8 have one element placed inside a different element.


3, 6, 9 have two similar elements, one inside the other and the
area between the two elements is shaded.
7.Group the given figures into three classes using each figure
only once.
çR;sd vk—fr dk dsoy ,d ckj ç;ksx djrs gq, nh xbZ vk—fr;ksa dks rhu
oxksZa esa lewfgr dhft,A

(a) 2,4,7 ; 1,8,9 ; 3,5,6


(b) 2,6,9 ; 1,5,7 ; 3,4,8
(c) 2,6,7 ; 1,5,8 ; 3,4,9
(d) 2,8,7 ; 1,5,9 ; 3,4,6
(a) 1,2,6 ; 3,4,7 ; 5 Sol. (b)
(b) 1,3 ; 2,6 ; 4,5,7 2, 6, 9 contain a triangle with its three medians as the outer
(c) 1,2,6,7 ; 3 ; 4,5 element and another element (similar or different) placed
(d) 1,3 ; 2,4,5 ; 6,7 inside it.
Sol. (d) 1, 5, 7 contain a rectangle with its two diagonals as the outer
1, 3 contain a V-shaped element inside a geometrical figure. element and another element (similar or different) placed
2, 4, 5 contain two similar elements, one placed inside the inside it.
other and touching it. 3, 4, 8 contain a circle with its two mutually perpendicular
6, 7 contain geometrical figures which are divided into four diameters as the outer element and another element
equal parts by two mutually perpendicular straight lines. (similar or different) placed inside it.
8.Group the given figures into three classes using each figure 10.Group the given figures into three classes using each figure
only once. only once.
çR;sd vk—fr dk dsoy ,d ckj ç;ksx djrs gq, nh xbZ vk—fr;ksa dks rhu çR;sd vk—fr dk dsoy ,d ckj ç;ksx djrs gq, nh xbZ vk—fr;ksa dks rhu
oxksZa esa lewfgr dhft,A oxksZa esa lewfgr dhft,A

(a) 1,2,5 ; 3,7,8 ; 4,6,9


(b) 1,7,2 ; 3,9,6 ; 4,5,8
(c) 2,3,8 ; 4,6,9 ; 1,5,7 (a) 1,3,5 ; 2,6,9 ; 4,7,8
(d) 5,6,9 ; 3,4,1 ; 2,7,8 (b) 2,3,4 ; 5,6,8 ; 9,1,7
Sol. (a) (c) 1,3,5 ; 2,6,8 ; 4,7,9
1, 2, 5 are figures that have patterns formed from four lines (d) 3,2,4 ; 6,5,8 ; 7,9,1
curved in a concave direction. Sol. (c)
3, 7, 8 are figures that have patterns formed from four lines 1, 3, 5 are figures having partially or completely curved
curved in a convex direction. boundaries.
4, 6, 9 are figures that have patterns formed from these 2, 6, 8 are all triangles.
straight lines. 4, 7, 9 are all quadrilaterals.
9.Group the given figures into three classes using each figure 11.Group the given figures into three classes using each figure
only once. only once.
çR;sd vk—fr dk dsoy ,d ckj ç;ksx djrs gq, nh xbZ vk—fr;ksa dks rhu çR;sd vk—fr dk dsoy ,d ckj ç;ksx djrs gq, nh xbZ vk—fr;ksa dks rhu
oxksZa esa lewfgr dhft,A oxksZa esa lewfgr dhft,A

151
Download Free PDFs & e-Books from Neon Classes App

3, 7, 9 are closed figures shaded by oblique line segments.


2, 4, 5 are figures composed of straight lines only.
14.Group the given figures into three classes using each figure
only once.
çR;sd vk—fr dk dsoy ,d ckj ç;ksx djrs gq, nh xbZ vk—fr;ksa dks rhu
oxksZa esa lewfgr dhft,A

(a) 3,4,9 ; 5,7,8 ; 1,2,6


(b) 1,5,6 ; 2,4,8 ; 3,7,9
(c) 4,6,8 ; 3,5,7 ; 1,2,9
(d) 1,2,7 ; 3,5,9 ; 4,6,8
Sol. (d) (a) 1,2,4 ; 3,5,6 ; 7,8,9
1, 2, 7 are simple geometrical figures. (b) 1,7,8 ; 3,5,6 ; 2,4,9
3, 5, 9 have one element placed inside a different element. (c) 1,3,4 ; 2,8,9 ; 5,6,7
4, 6, 8 have two different elements attached to each other. (d) 1,7,8 ; 2,3,6 ; 4,5,9
12.Group the given figures into three classes using each figure Sol. (b)
only once. 1, 7, 8 are all undivided geometrical figures.
çR;sd vk—fr dk dsoy ,d ckj ç;ksx djrs gq, nh xbZ vk—fr;ksa dks rhu 3, 5, 6 are geometrical figures divided into two parts.
oxksZa esa lewfgr dhft,A 2, 4, 9 are geometrical figures divided into four parts.
15.Group the given figures into three classes using each figure
only once.
çR;sd vk—fr dk dsoy ,d ckj ç;ksx djrs gq, nh xbZ vk—fr;ksa dks rhu
oxksZa esa lewfgr dhft,A

(a) 1,5,8 ; 3,4,7 ; 2,6,9


(b) 1,3,6 ; 4,5,9 ; 2,7,8
(c) 1,3,6 ; 2,5,7 ; 4,8,9
(d) 6,7,8 ; 1,3,7 ; 2,4,9
Sol. (c) (a) 1,3,7 ; 2,4,6 ; 5,8,9
1, 3, 6 contain one complete circle each. (b) 1,4,6 ; 2,5,7 ; 3,8,9
2, 5, 7 contain a semi-circle each. (c) 1,4,8 ; 2,5,6 ; 3,7,9
4, 8, 9 contain a triangle each. (d) 1,4,8 ; 2,7,9 ; 3,5,6
13.Group the given figures into three classes using each figure Sol. (c)
only once. 1, 4, 8 contain similar elements (not equal in size) each
çR;sd vk—fr dk dsoy ,d ckj ç;ksx djrs gq, nh xbZ vk—fr;ksa dks rhu divided into four parts and attached to each other.
oxksZa esa lewfgr dhft,A 2, 5, 6 contain three elements (two of which are similar)
placed one inside the other.
3, 7, 9 contain one element inside the other, which may or may
not be similar.
16.Group the given figures into three classes using each figure
only once.
çR;sd vk—fr dk dsoy ,d ckj ç;ksx djrs gq, nh xbZ vk—fr;ksa dks rhu
oxksZa esa lewfgr dhft,A

(a) 1,3,6 ; 4,5,8 ; 2,7,9


(b) 2,3,9 ; 4,5,8 ; 1,6,7
(c) 1,6,8 ; 3,7,9 ; 2,4,5
(d) 3,8,9 ; 1,2,7 ; 4,5,6
Sol. (c)
1, 6, 8 are figures composed of straight as well as curved lines.

152
Best App for Govt. Jobs : Neonclasses (Download Now)

(c) 1,5,8 ; 2,4,7 ; 3,6,9


(d) 1,5,8 ; 2,6,9 ; 3,4,7
Sol. (c)
1, 5, 8 consist of a circle and a triangle intersecting it. The
triangle is also divided into two equal parts by a straight
line.
2, 4, 7 are all funnel shaped figures.
3, 6, 9 are geometrical figures containing line segments (the
number of these line segments is half the number of sides
in the figure) parallel to the sides of the figure.
(a) 1,7,9 ; 2,3,6 ; 4,5,8 19.Group the given figures into three classes using each figure
(b) 1,2,9 ; 3,4,6 ; 5,7,8 only once.
(c) 1,6,8 ; 2,4,7 ; 3,5,9 çR;sd vk—fr dk dsoy ,d ckj ç;ksx djrs gq, nh xbZ vk—fr;ksa dks rhu
(d) 1,7,8 ; 2,9,3 ; 6,4,5 oxksZa esa lewfgr dhft,A
Sol. (a)
1, 7, 9 contain two similar elements one inside the other but
not touching each other.
2, 3, 6 contain two similar elements one inside the other and
both touching each other.
4, 5, 8 are divided into equal parts by straight lines emerging
from the centre.
17.Group the given figures into three classes using each figure
only once.
çR;sd vk—fr dk dsoy ,d ckj ç;ksx djrs gq, nh xbZ vk—fr;ksa dks rhu
oxksZa esa lewfgr dhft,A
(a) 2,4,7 ; 1,6,9 ; 3,5,8
(b) 1,3,5 ; 2,6,7 ; 4,8,9
(c) 1,5,7 ; 2,3,6 ; 4,8,9
(d) 1,3,5 ; 2,4,7 ; 6,8,9
Sol. (a)
2, 4, 7 are all divided into equal parts (either three or four
parts) by straight lines and also have a black circle at the
centre.
1, 6, 9 are figures which are half shaded by slanting lines.
3, 5, 8 have similar designs and have their four corners shaded
black.
20.Group the given figures into three classes using each figure
(a) 1,4,7 ; 2,5,9 ; 3,8,6
only once.
(b) 2,6,9 ; 1,4,7 ; 5,8,3
(c) 1,4,7 ; 2,3,6 ; 5,8,9
çR;sd vk—fr dk dsoy ,d ckj ç;ksx djrs gq, nh xbZ vk—fr;ksa dks rhu
(d) 3,5,1 ; 4,7,8 ; 6,2,9 oxksZa esa lewfgr dhft,A
Sol. (c)
1, 4, 7 tare objects which have neither a base nor an upper lid
attached to them.
2, 3, 6 are objects having base but not upper lid.
5, 8, 9 are objects having both base as well as upper lid.
18. Group the given figures into three classes using each figure
only once.
çR;sd vk—fr dk dsoy ,d ckj ç;ksx djrs gq, nh xbZ vk—fr;ksa dks rhu
oxksZa esa lewfgr dhft,A

(a) 1,2,3 ; 4,5,8 ; 6,7,9


(b) 1,5,6 ; 2,3,4 ; 7,8,9
(c) 1,3,5 ; 2,4,8 ; 6,7,9
(d) 1,4,7 ; 2,5,8 ; 3,6,9
Sol. (b)
1, 5, 6 have two similar elements, one inside the other.
2, 3, 4 contain straight lines each dividing, the figure into two
equal parts.
(a) 1,5,8 ; 2,6,7 ; 3,4,9
7, 8, 9 have one element placed inside a different element.
(b) 1,5,7 ; 2,6,8 ; 3,4,5

153
Download Free PDFs & e-Books from Neon Classes App

21.Group the given figures into three classes using each figure
only once.
çR;sd vk—fr dk dsoy ,d ckj ç;ksx djrs gq, nh xbZ vk—fr;ksa dks rhu
oxksZa esa lewfgr dhft,A

(a) 1,5,9 ; 2,7,8 ; 3,4,6


(b) 1,5,6 ; 4,7,8 ; 2,3,9
(c) 2,4,9 ; 6,7,8 ; 1,3,5
(d) 3,7,8 ; 4,5,9 ; 1,2,6
(a) 1,4,7 ; 3,6,9 ; 2,5,8 Sol. (c)
(b) 1,6,9 ; 2,4,7 ; 3,5,8 2, 4, 9 have two similar elements intersecting each other.
(c) 1,4,7 ; 2,6,9 ; 3,5,8 6, 7, 8 have two different elements one placed inside the other.
(d) 1,5,7 ; 2,6,9 ; 3,4,8 1, 3, 5 have two different elements intersecting each other.
Sol. (c) 24.Group the given figures into three classes using each figure
1, 4, 7 have similar designs (there are four small circles at(the only once.
corners of the main figure and there is a wheel shaped çR;sd vk—fr dk dsoy ,d ckj ç;ksx djrs gq, nh xbZ vk—fr;ksa dks rhu
element at the centre of the figure). oxksZa esa lewfgr dhft,A
2, 6, 9 have similar designs (three of the corners of the main
figure are shaded black and there is a pattern formed
around a '+' sign at the centre of the figure).
3, 5, 8 have similar designs (four leaves placed close to a small
circle and forming a symmetrical design at the centre of
the figure).
22.Group the given figures into three classes using each figure
only once.
çR;sd vk—fr dk dsoy ,d ckj ç;ksx djrs gq, nh xbZ vk—fr;ksa dks rhu
oxksZa esa lewfgr dhft,A

(a) 1,2,3 ; 6,5,4 ; 7,9,8


(b) 1,6,7 ; 2,5,9 ; 3,4,8
(c) 1,6,7 ; 2,5,8 ; 3,4,9
(d) 1,3,7 ; 2,5,9 ; 4,6,8
Sol. (b)
1, 6, 7 have only one element.
2, 5, 9 have two elements.
3, 4, 8 consist of three elements.
(a) 1,3,9 ; 2,5,8 ; 4,6,7 25.Group the given figures into three classes using each figure
(b) 1,5,8 ; 4,6,7 ; 2,3,9 only once.
(c) 2,5,9 ; 1,3,8 ; 2,6,7 çR;sd vk—fr dk dsoy ,d ckj ç;ksx djrs gq, nh xbZ vk—fr;ksa dks rhu
(d) 1,8,9 ; 4,6,7 ; 2,3,5 oxksZa esa lewfgr dhft,A
Sol. (b)
1, 5, 8 are all open figures bisected by a line segment.
4, 6, 7 are all closed figures touching a line segment.
2, 3, 9 are all closed figures intersected by a line.
23.Group the given figures into three classes using each figure
only once.
çR;sd vk—fr dk dsoy ,d ckj ç;ksx djrs gq, nh xbZ vk—fr;ksa dks rhu
oxksZa esa lewfgr dhft,A
(a) 5,7,1 ;2,4,9 ; 3,6,8
(b) 1,5,7 ; 2,8,9 ; 3,4,6
(c) 1,7,8 ; 2,4,9 ; 3,5,6
(d) 1,4,6 ; 5,8,9 ; 2,3,7
Sol. (a)
Figures 1, 5 and 7 contain an English letter, two circles, one
star and oblique lines.

154
Best App for Govt. Jobs : Neonclasses (Download Now)

Figures 2, 4 and 9 contain an English letter and one dot. III. M < K IV. L > K
Figures 3, 6 and 8 contain an English letter and one or two (a) Only conclusion I is true/ dsoy fu"d"kZ I lR; gS
cross sign (s). (b) Only conclusions I and III are true
INEQUALITY dsoy fu"d"kZ I vkSj III lR; gSa
(c) Only conclusions I and IV are true
In the question related to this chapter we are given some dsoy fu"d"kZ I vkSj IV lR; gSa
statements and conclusions and we have to tell which of the
(d) Only Conclusions II is true
given conclusions will be follow or not. Both statement and
dsoy fu"d"kZ II lR; gS
conclusions are created using signs of inequality (>,, <, , =)
Sol. (b)
We can divided these signs into two groups.
Statement:- K ≥ L > M < N ≤ X
Group (i) : - In this group ‘>’, ‘’ and = signs are included this group
Conclusions:- (I.) X > M
is called greater family. (I) X>M ,
While defining the relationship between any two elements, if all when we go from X to M, path possible. So relation is possible
three signs ‘>’, ‘’ and = come in between choose the sign ‘>’. Highest preference is of ‘≤’ so correct relation is X>M
If the ‘>’ and = come in between choose the sign ‘>’. (II) X>L,
If the ‘>’ and ‘’ come in between choose the sign ‘>’. when we go from X to L, Path Not possible, so relation is not
If the ‘’ and = come in between choose the sign ‘’. possible
Group (ii) : - In this group ‘<’, ‘’ and = signs are included this group (III) M<K,
is called less than family. when we go from K to M path possible, So relation is possible
While defining the relationship between any two elements, if all Highest preference is of ‘≥’ so correct relation is M < K
three signs ‘<’, ‘’ and = come in between choose the sign ‘<’. (IV) L>K,
If the ‘<’ and = come in between choose the sign ‘<’. when we go from L to K. Path not possible so, Relation is not
possible.
If the ‘<’ and ‘’ come in between choose the sign ‘<’.
correct answer is option (b)
If the ‘’ and = come in between choose the sign ‘’.
3. Statement/dFku:
CONCEPT (1)-SINGLE STATEMENT
W=H≥G=C≥T≤I≤M
1. Read the given statements and conclusions carefully. Decide
which of the given conclusion/s/is/are true based on the given
Conclusions/fu"d"kZ:
statement. I. T ≤ W
fn, x, dFkuksa vkSj fu"d"kksaZ dks è;kuiwoZd i<+Asa fn, x, dFku ds II. C ≥ M
vkèkkj ij r; djsa fd fn, x, fu"d"kksaZ esa ls dkSu lk@ls lR; (a) Only conclusion II is true
gS@gSaA (b) Both conclusions I and II are true
Statements: (c) Only conclusion I is true
A>B=CD (d) Neither conclusion I nor II is true
Conclusions: Sol. (c)
I. A  C II. B = D Conclusions:-
(a) Only conclusion I is true/ dsoy fu"d"kZ I lR; gS (I) T≤W
(b) Only conclusions I and II are true When we go from W to T, Path possible So relation is possible.
dsoy fu"d"kZ I vkSj II lR; gSa Highest Preference is of ≥ So correct relation is T ≤ W
(c) Neither I nor II are true (II) C≥M, When we go from C to M, Path not possible, So
u rks fu"d"kZ I u gh II lR; gSa relation is Not Possible. (Condition block).
(d) Only Conclusions II is true Correct answer is option (c)
dsoy fu"d"kZ II lR; gS 4. Statement/dFku %
Sol. (c) Z>H≥Y>R<D≥F>X
I. Here we have to make the relation form A to C in when we go Conclusion/fu"d"kZ%
from A to C in statement then the sign and = come in (i) Z ≥ X (ii) F > H
between hence we will choose the sign ‘>’ . Thus relation from (a) Both conclusions I and II follow
A to C : A > C fu"d"kZ I vkSj II nksuksa vuqlj.k djrs gSa
There for the given conclusion first does’t follow. (b) Only conclusion I follows
II. Here we have make the relation some B to D and when we dsoy fu"d"kZ I vuqlj.k djrk gS
go from B to D in statement then the sign = and  come in (c) Neither conclusion I nor conclusion II follows
u rks fu"d"kZ I vkSj u gh fu"d"kZ II vuqlj.k djrk gS
between so we will choose ‘’. Thus the relation from B to D : B
(d) Only conclusion II follows
 D. dsoy fu"d"kZ II vuqlj.k djrk gS
Hence the conclusion II does not follow. Sol. (c)
Neither I nor II follow Conclusions – (I) Z ≥ X
2. Statements: When we go from Z to X, Path not possible, So relation is not
K≥L>M<N≤X possible
Conclusions: (II) F>H
I. X > M II. X > L

155
Download Free PDFs & e-Books from Neon Classes App

When we go from F to H, Path Not possible, So relation is not (c) Neither conclusions I nor II follows
possible. u rks fu"d"kZ I vkSj u gh II vuqlj.k djrk gS
Correct answer is option (c) (d) Both conclusions I and II follow
5. Select the correct conclusion that could be drawn from fu"d"kZ I vkSj II nksuksa vuqlj.k djrs gSa
the given expression. Sol. (d)
ml lgh fu"d"kZ dk p;u djsa tks fn, x, O;atd ls fudkyk tk I. A  C [>, =] hence the conclusion I follow.
ldrk gSA
II. B  D [=,] hence the conclusion II follow.
F≤E<C=V>A≥Y>N Both I and II follow.
(a) E > Y (b) N < V (c) F = V (d) V < Y 8. Statement/dFku %
Sol. (b) P > Z, N < T, M > N, Z > M
Option (a) E>Y Conclusion/fu"d"kZ%
When we go from E to Y, Path not possible , So relation is Not
(i) P < T (ii) N < Z
possible.
(a) Both conclusions I and II follow
Option (b) N<V
fu"d"kZ I vkSj II nksuksa vuqlj.k djrs gSa
When we go from N to V, path possible , So relation is possible.
(b) Only conclusion I follows
highest preference is of ‘≥’ So correct relation is N<V
dsoy fu"d"kZ I vuqlj.k djrk gS
Option (c) F = V
(c) Neither conclusion I nor conclusion II follows
When be go from from F to V , Path not possible so relation is
u rks fu"d"kZ I vkSj u gh fu"d"kZ II vuqlj.k djrk gS
not possible.
(d) Only conclusion II follows
Option (d) V<Y
When we go from V to Y, path not possible.
dsoy fu"d"kZ II vuqlj.k djrk gS
Correct answer is option (b) Sol. (d)
06. Statements ¼dFku½: Conclusion (I) P<T
When we go from P to T, Path not possible So relation is not
A<D>C<E>B
possible
Conclusions ¼fu"d"kZ½ :
(II) N<Z
I. C > B
When we go from Z To N path possible so relation is possible.
II. A < E
highest preference is of ‘>’ So correct relation is N<Z
III. D > B
Correct answer is option (d)
(a) Only I conclusion follows/dsoy I fu"d"kZ vuqlj.k djrk gS
9 . Statements/dFku:
(b) Only II conclusion follows/dsoy II fu"d"kZ vuqlj.k djrk gS
(c) Only III conclusions follows/dsoy III fu"d"kZ vuqlj.k djrk gS £ < ≤ ≥= 
(d) Neither I, II nor III is true/uk gh I, II ,oa uk gh III fu"d"kZ lR; gS Conclusions/fu"d"kZ:
(e) Either I and II is true/;k rks I vkSj II lR; gSA I. £ ≤  < 
(f) Both conclusion I or II, III is true/nksuks fu"d"kZ I ;k II, III lR; gS (a) Both conclusions I and II follow
Sol. (d) I. C > D → C < D fu"d"kZ I vkSj II nksuksa vuqlj.k djrs gSa
Does not follow (b) Only conclusion I follows
II. A < E → E > C < D > A dsoy fu"d"kZ I vuqlj.k djrk gS
Does not follow (c) Neither conclusion I nor conclusion II follows
III. D > B → D > C < E > B u rks fu"d"kZ I vkSj u gh fu"d"kZ II vuqlj.k djrk gS
Does not true (d) Only conclusion II follows
CONCEPT (2)- TWO OR MORE THAN TWO STATEMENTS dsoy fu"d"kZ II vuqlj.k djrk gS
ARE GIVEN Sol. (c)
More then one statement are given in such a situations any Conclusions:-
two given statements can be combined only when there is Option(I) £ ≤ 
common term between.
 not possible
Eg. M > N ; U  P; N > U
Option (II) < 
The statement M > N and N > U will be used to go from M to U.
The common term in both statement is N, hence these  Not relation possible
statement can be combined. Correct option is option (c)
7. Statements/dFku: 10 . Statements/dFku:
A > B; C  D; B = C M > N  T; R < N  P
Conclusions/fu"d"kZ: Conclusions/fu"d"kZ:
I. A  C I. M > R  P III. T  P
(a) Both conclusions I and III follow
II. B  D
fu"d"kZ I vkSj III nksuksa vuqlj.k djrs gSa
(a) Only conclusions II follows
(b) Only conclusion I and II follows
dsoy fu"d"kZ II vuqlj.k djrk gS
dsoy fu"d"kZ I vkSj II vuqlj.k djrs gSa
(b) Only conclusions I follows (c) None follows
dsoy fu"d"kZ I vuqlj.k djrk gS dksbZ Hkh vuqlj.k ugha djrk gSA

156
Best App for Govt. Jobs : Neonclasses (Download Now)

(d) Only conclusion III follows I. F > N


dsoy fu"d"kZ III vuqlj.k djrk gS II. H < N
Sol. (a) (a) Only I conclusion follows/dsoy I fu"d"kZ vuqlj.k djrk gS
I. M > R [>,>] hence the conclusion I follow. (b) Only II conclusion follows/dsoy II fu"d"kZ vuqlj.k djrk gS
  P [>,] hence the conclusion II does not follow. (c) Either I or II follows/;k rks I ;k II fu"d"kZ vuqlj.k djrk gS
III. T  P [,] hence the conclusion III follow. (d) Neither I nor II follows/uk gh I uk gh II fu"d"kZ vuqlj.k djrk gS
(e) Both I and II follows/nksuks fu"d"kZ vuqlj.k djrs gS
 hus Only I and III follow.
Sol. (a) I. F > N → F > K > N
Correct option is option (a)
Ist Conclusion follow
11 . Statements/dFku:
II. H < N → N > H
R > P  U = S; P < W  Y Does not follow
Conclusions/fu"d"kZ: 15. Statements ¼dFku½:
I. R > W U W M > N > R > W, E = J > L > W
(a) Both conclusions I and II follow Conclusions ¼fu"d"kZ½ :
fu"d"kZ I vkSj II nksuksa vuqlj.k djrs gSa I. E > W
(b) Only conclusion II follows II. M > L
dsoy fu"d"kZ II vuqlj.k djrk gS (a) Only I conclusion follows/dsoy I fu"d"kZ vuqlj.k djrk gS
(c) Only conclusion I follows (b) Only II conclusion follows/dsoy II fu"d"kZ vuqlj.k djrk gS
dsoy fu"d"kZ I vuqlj.k djrk gS (c) Either I or II follows/;k rks I ;k II fu"d"kZ vuqlj.k djrk gS
(d) None follows (d) Neither I nor II follows/uk gh I uk gh II fu"d"kZ vuqlj.k djrk gS
dksbZ Hkh vuqlj.k ugha djrk gSA (e) Both I and II follows/nksuks fu"d"kZ vuqlj.k djrs gS
Sol. (b) Sol. (a) I. E > W → E = J > L > W
I. R > W [>,<] hence the conclusion I does not follow. Ist Conclusion follow
 U W [, <] hence the conclusion II follow. II. M > L → M > N > R > W < L
 hus Only II follow. IInd Conclusion does not follow
CONCEPT (3) - POSSIBILITY
Correct option is option (b)
There will be possibility such conclusions which are neither
12 . A __ B __ C __ P __ Q __ R
100% true nor 100% false.
Which of the following options makes ‘A >P’ and ‘ C  R ’ 100% false conclusion
definitely true? The conclusions which are neither 100% true nor there
(a) > > > > > (b)  is any possibility for them, will be 100% false.
(c) > > =  = (d) > > = <  16 . Statements/dFku:
Sol. (c)
A  B  C; B < D  F; D E
‘A > P’ will be true only when there comes the signs of greater
Conclusions/fu"d"kZ:
family from A to P and there should be at least one ‘>’ sign.
Hence option (b) and (c) are incorrect. I. A = C  F III. D  E
Similarly : - IV. B < E V. F > E
(i) How many conclusions are 100% true?
C  R will be true only when there comes the signs of less than
 (a) Only I follow
family from U to R and there should be at least one ‘’ sign. (b) Only III follow
Hence option (d) is incorrect. (c) Only IV follow
Thus option (c) is follow. (d) Only II and IV follows
13. Statements ¼dFku½: (ii) How many conclusions are 100% false?
A < B, B < C, C > D > E, E < F, F < L (a) Only I does not follow
Conclusions ¼fu"d"kZ½ : (b) Only III does not follow
I. B > E (c) Only V does not follow
II. L > D (d) Only III and IV does not follow
(a) Only I conclusion follows/dsoy I fu"d"kZ vuqlj.k djrk gS Sol. (i & ii)
(b) Only II conclusion follows/dsoy II fu"d"kZ vuqlj.k djrk gS I. The relation from ‘A to C; A  C’
(c) Either I or II follows/;k rks I ;k II fu"d"kZ vuqlj.k djrk gS Hence there is possibility for the conclusion ‘A = C’.
(d) Neither I nor II follows/uk gh I uk gh II fu"d"kZ vuqlj.k djrk gS II. The relation from ‘B to F’ the signs of both families are
(e) Both I and II follows/nksuks fu"d"kZ vuqlj.k djrs gS coming between B and F.
Sol. (d) I. B > E ⟶ B > A Hence there is no relation between them.
B<C>D>E In such a situation there is possibility for the conclusion ‘B = F’
Does not follow III. The relation from ‘D to E; D  E’.
II. L > D → L > F > E < D Hence there is possibility for the conclusion ‘D = E’.
Does not follow IV. The relation from ‘B to E; B < E’.
14. Statements ¼dFku½: Hence the conclusion IV follows.
H < N, N < K, K < F V. The relation from ‘F to E; F < E’.
Conclusions ¼fu"d"kZ½ : Hence the conclusion ‘F > E’ is 100% false.

157
Download Free PDFs & e-Books from Neon Classes App

(i) Thus only IV follows answer (c) option. When we go from D to J Path Not possible so relation is not
(ii) Thus only conclusion V does not follow answer (c) possible
Option (c) B#L B>L
option.
When we go from path not possible
CONCEPT (4) – EITHER - OR CONDITION
Option (d) B$J B≥J
There will be either-or case between any two given
When we go from path not possible so relation not possible
conclusions when the following three condition. Correct answer is option (a)
(1) Both conclusion must be wrong.
(2) The subjects and objects of both conclusions must 19. If G = E < D < R and E = Y > K > Q, then which of the following
be same. options is NOT correct?
(3) (i) If there is the relation of ‘’ between any two ;fn G = E < D < R vkSj E = Y > K > Q, rks fuEufyf[kr esa ls dkSulk
fodYi lgh ugha gS\
given elements, then there will be either-or case
(a) G < Q (b) R > Q (c) G < R (d) Y < R
between the signs ‘> and =’. Because the sign  is made Sol. (a)
of these two signs. Option (a) G<Q
(ii) If there is the relation of  between any two given When we go from Q to G, Path not possible so relation is not
elements then there will be either-or case between the possible
Option (b) R>Q
signs ‘< and =’. Because the sign  is made of these
two signs.
(iii) If there is no relation between two elements i.e. E<D<R, E= Y>K>Q
condition is block, then either-or case will be between
the three signs ‘>, < and =’. when we go from R to Q, Path possible so relation is possible
17. Statements/dFku: Highest preference is of ‘>’ So correct relation is R>Q
Option (c) G<R G  E  D  R
P  R  T  S.
Conclusions/fu"d"kZ: When we go from R to G, path possible so relation is possible
highest preference is of ‘<’ So correct relation is G<R
I. P > T P T III. S  R
(a) Only III follow Option (d) Y<R R  D  E, E  Y
(b) Either I or II follow When we go from R to Y, path possible so relation is possible
(c) Only II follows highest preference is of ‘>’ so correct relation is Y<R
(d) Only either I or II and III follow Concept(5): Coded Inequalites:
Sol. (d) In this, the relation between two or more elements is not
I. P > T [, =] hence the conclusion I does not follows. shown derectly using the signs of inequalities but they are
shown using some symbols.
 P T [ , =] hence the conclusion II does not follow.
While solving such questions we make a small box. In this box
III. S  R [ , =] hence the conclusion III follow. we will show all the inequality signs and the symbols related
Here the definite result from ‘P to T, P  T’ to those. Let’s understand it with the help of the following
Here the both conclusion I and II are wrongs, the subjects and examples:
object of both conclusion are the same and both the signs Directions (20 to 22): In the following questions the
and = required for  are present. symbols $,©,@,# and % are used with the following
Hence there will be either-or case between I and II meanings:
conclusions. A$B means A is not less than B.
Thus only either I or II and III follows. A©B means A is greater than B.
18. ‘#’ means ‘>’, ‘$’ means ‘≥’, ‘%’ means ‘<’, and ‘@’ A@B means A is equal to B.
means ‘≤’. A#B means A is not greater than B
A%B means A is less than B.
Assuming the following statements to be true, which of
Now in each or the following questions, assuming the three
the conclusions given in the options is definitely true. statements to be true, find which of the two conclusions I and
fuEufyf[kr dFkuksa dks lR; ekurs gq, fodYiksa esa fn, x, fu"d"kksaZ esa II is/are true. Give answer
ls dkSu lk fuf'pr :i ls lR; gSA (A) Only I follows
B$D$A (B) Only II follows
R%D%L@J (C) Either I or II follows
(a) A % L (b) J % D (c) B # L (d) B $ J (D) Neither I nor II follows
Sol. (a) (E) Both I and II follow
Symbol dks fpUg esa change djus ds ckn 20. Statements: A$B ; B©D ; D@I
Conclusions: I. A©D II. B@I
⇒ B≥D≥A, R<D<L≤J
21. Statements: M%O©S ; R#T
Option (a) A%L A<L, A≤D<L
Conclusions: I. M©S II. M%S
When we go from R to L , path possible so relation is possible
22. Statements: M©N ; R%N
Highest preference is of ‘<’ so correct relation is A<L
Conclusions: I. M©R II. M%R
Option (b) J%D J<D

158
Best App for Govt. Jobs : Neonclasses (Download Now)

Sol. To Solve such questions easily we will make a small box and ‘A $ B dk eryc ^A, B ls u rks cM+k vkSj u gh NksVk gSA
show the inequality signs and the symbols related to those in ‘A @ B’ means ‘A is neither greater than nor equal to B’
it. Here: ‘A @ B dk eryc ^A, B ls u rks cM+k vkSj u gh cjkcj gSA
$ => ≥ ; ©=> > ; @=> = ; #=> ≤ ; %=> < ‘A % B’ means ‘A is neither smaller than nor equal to B’.
These all will be shown as follows in the box: ‘A % B dk eryc ^A, B ls u rks NksVk vkSj u gh cjkcj gSA
‘A & B’ means A is not smaller than B’
© % ‘A & B dk eryc ^A, B ls NksVk ugha gS
‘A # B’ means A is not greater than B’.
> < ‘A # B dk eryc ^A, B ls cM+k ugha gSA
(a) Only I is true
dsoy I lR; gSA
(b) Only II is true
dsoy II lR; gSA
@= (c) Either I or II is true
;k rks I ;k II lR; gSA
(d) Neither I nor II is true
uk rks I uk gh II lR; gSA
(e) Both I and II is true
≥ ≤ nksuks I vkSj II lR; gSA
23. Statements/¼dFku½:
S $ Q, Q @ B, B & K, K # W
$ #
Now if the symbols Conclusions ¼fu"d"kZ½ :
©, $ and @ come in between the two elements, then choose I. W % K II. S @ B
the symbol ‘©’. Similarly choose the symbol ‘©’ between © 24. Statements/¼dFku½:
and @ and choose the symbol ‘$’ between $ and @. Z % N, N # K, K $ M, M @ R
Simillarly choose the symbol ‘%’ among %,# and @, choose the Conclusions ¼fu"d"kZ½ :
symbol ‘%’ between % and @ and choose the symbol ‘#’ I. M $ N II. M % N
between # and @. Apart from that, condition will be block if 25. Statements/¼dFku½:
the signs of both families come in between. W % P, P % G, G @ I, I # N
,sls iz’kuksa dks vklkuh ls gy djus ds fy, ge ,d ckWDlcuk,axs vkSj Conclusions ¼fu"d"kZ½ :
mlesa vlekurk okys fpàksa o muls lacaf/kr izrhdksa dks fn[kk,axsA ;gk¡% I. N % W II. N # W
$ => ≥ ; ©=> > ; @=> = ; #=> ≤ ; %=> < 23. sol.(b)
bu lHkh dks ckWDl esa fuEu izdkj n’kkZ,axs% S = Q, Q < B, B > K, K < W
vc ;fn nks vo;oksa ds chp esa ‘©’, ‘$’ vkSj ‘@’ vk, rks ‘©’ dks I. W % K → W > K → W > K
pqusaxsA blh izdkj ‘©’ vkSj ‘@’ esa ‘©’ dks pqusaxs ‘$’ vkSj ‘@’ vkSj ‘$’ Ist Conclusion does not follow
sa sA blh rjg ‘%’,’#’ vkSj ‘@’ esa ‘%’ dks pqusaxs] ‘%’ vkSj
esa dks pqux II. S @ B → S < B → B > Q = S
‘@’ esa ‘%’ dks pqux sa s vkSj ’#’ vkSj ‘@’ esa ’#’ dks pqusaxsA IInd Conclusion is follow
blds vykok nksuksa QSfeyht ds fpà chp esa vkus ij dafM’ku CykWd 24. sol.
gksxhA (c)
20. (A) Z > N, N < K, K = M, M < R
I. A©D [$,©] I. M $ N → M = N → M = K > N
Hence the conclusion I follows. I. Conclusion does not follow
II. B@I [©,@] II. M % N →M > N → M = K > N
Hence the conclusion II dose not follow. Does not follow Conclusion I and II follow 3 symbol (>, <, =)
Thus only I follows. case
21. (B) So either or case will be apply
I. M©S[%,©] 25. sol.
There is condition block between R and T. Hence the (c)
conclusion I does not follow. W > P, P > G, G < I, I < N
II. Hence the conclusion II follows. I. N % W → N > W → N > I > G < P < W
Thus only II follows.
Does not follow
22. (A)
II. N # W → N < W → W > P > G < I < N
I. M©R [©,©]
Does not follow Conclusion I and II follow 3 symbol (>, < =)
Hence conclusion I follows and II doesnot follows.
case
Thus only conclusion I follows.
So either or case will be apply.
Notes: The symbols will be reversed while moving from M to R.
INEQUALITY
Hence ‘%’ is converted into ‘©’.
In the question related to this chapter we are given some
Directions (23 to 25):: In these questions Symbols $, @, %, # and
statements and conclusions and we have to tell which of the
& are used with the different meaning as follows.
given conclusions will be follow or not. Both statement and
‘A $ B’ means ‘A is neither greater than nor smaller than B.

159
Download Free PDFs & e-Books from Neon Classes App

conclusions are created using signs of inequality (>,, <, , =) Conclusions:- (I.) X > M
We can divided these signs into two groups. (I) X>M ,
Group (i) : - In this group ‘>’, ‘’ and = signs are included this group when we go from X to M, path possible. So relation is possible
is called greater family. Highest preference is of ‘≤’ so correct relation is X>M
While defining the relationship between any two elements, if all (II) X>L,
three signs ‘>’, ‘’ and = come in between choose the sign ‘>’. when we go from X to L, Path Not possible, so relation is not
If the ‘>’ and = come in between choose the sign ‘>’. possible
If the ‘>’ and ‘’ come in between choose the sign ‘>’. (III) M<K,
If the ‘’ and = come in between choose the sign ‘’. when we go from K to M path possible, So relation is possible
Group (ii) : - In this group ‘<’, ‘’ and = signs are included this group Highest preference is of ‘≥’ so correct relation is M < K
is called less than family. (IV) L>K,
While defining the relationship between any two elements, if all when we go from L to K. Path not possible so, Relation is not
three signs ‘<’, ‘’ and = come in between choose the sign ‘<’. possible.
If the ‘<’ and = come in between choose the sign ‘<’. correct answer is option (b)
If the ‘<’ and ‘’ come in between choose the sign ‘<’. 3. Statement/dFku:
If the ‘’ and = come in between choose the sign ‘’. W=H≥G=C≥T≤I≤M
Conclusions/fu"d"kZ:
CONCEPT (1)-SINGLE STATEMENT
1. Read the given statements and conclusions carefully. Decide I. T ≤ W
which of the given conclusion/s/is/are true based on the given II. C ≥ M
(a) Only conclusion II is true
statement.
(b) Both conclusions I and II are true
fn, x, dFkuksa vkSj fu"d"kksaZ dks è;kuiwoZd i<+Asa fn, x, dFku ds
(c) Only conclusion I is true
vkèkkj ij r; djsa fd fn, x, fu"d"kksaZ esa ls dkSu lk@ls lR;
(d) Neither conclusion I nor II is true
gS@gSaA
Sol. (c)
Statements:
Conclusions:-
A>B=CD (I) T≤W
Conclusions: When we go from W to T, Path possible So relation is possible.
I. A  C II. B = D Highest Preference is of ≥ So correct relation is T ≤ W
(a) Only conclusion I is true/ dsoy fu"d"kZ I lR; gS (II) C≥M, When we go from C to M, Path not possible, So
(b) Only conclusions I and II are true relation is Not Possible. (Condition block).
dsoy fu"d"kZ I vkSj II lR; gSa Correct answer is option (c)
(c) Neither I nor II are true 4. Statement/dFku %
u rks fu"d"kZ I u gh II lR; gSa Z>H≥Y>R<D≥F>X
(d) Only Conclusions II is true Conclusion/fu"d"kZ%
dsoy fu"d"kZ II lR; gS (i) Z ≥ X (ii) F > H
Sol. (c) (a) Both conclusions I and II follow
I. Here we have to make the relation form A to C in when we go fu"d"kZ I vkSj II nksuksa vuqlj.k djrs gSa
from A to C in statement then the sign and = come in (b) Only conclusion I follows
between hence we will choose the sign ‘>’ . Thus relation from dsoy fu"d"kZ I vuqlj.k djrk gS
A to C : A > C (c) Neither conclusion I nor conclusion II follows
There for the given conclusion first does’t follow. u rks fu"d"kZ I vkSj u gh fu"d"kZ II vuqlj.k djrk gS
II. Here we have make the relation some B to D and when we (d) Only conclusion II follows
go from B to D in statement then the sign = and  come in dsoy fu"d"kZ II vuqlj.k djrk gS
between so we will choose ‘’. Thus the relation from B to D : B Sol. (c)
 D. Conclusions – (I) Z ≥ X
Hence the conclusion II does not follow. When we go from Z to X, Path not possible, So relation is not
Neither I nor II follow possible
2. Statements: (II) F>H
K≥L>M<N≤X When we go from F to H, Path Not possible, So relation is not
Conclusions: possible.
I. X > M II. X > L Correct answer is option (c)
III. M < K IV. L > K 5. Select the correct conclusion that could be drawn from the given
(a) Only conclusion I is true/ dsoy fu"d"kZ I lR; gS expression.
(b) Only conclusions I and III are true ml lgh fu"d"kZ dk p;u djsa tks fn, x, O;atd ls fudkyk tk
dsoy fu"d"kZ I vkSj III lR; gSa ldrk gSA
(c) Only conclusions I and IV are true F≤E<C=V>A≥Y>N
dsoy fu"d"kZ I vkSj IV lR; gSa (a) E > Y (b) N < V (c) F = V (d) V < Y
(d) Only Conclusions II is true Sol. (b)
dsoy fu"d"kZ II lR; gS Option (a) E>Y
Sol. (b) When we go from E to Y, Path not possible , So relation is Not
Statement:- K ≥ L > M < N ≤ X possible.

160
Best App for Govt. Jobs : Neonclasses (Download Now)

Option (b) N<V dsoy fu"d"kZ I vuqlj.k djrk gS


When we go from N to V, path possible , So relation is possible. (c) Neither conclusion I nor conclusion II follows
highest preference is of ‘≥’ So correct relation is N<V u rks fu"d"kZ I vkSj u gh fu"d"kZ II vuqlj.k djrk gS
Option (c) F = V (d) Only conclusion II follows
When be go from from F to V , Path not possible so relation is dsoy fu"d"kZ II vuqlj.k djrk gS
not possible. Sol. (d)
Option (d) V<Y Conclusion (I) P<T
When we go from V to Y, path not possible. When we go from P to T, Path not possible So relation is not
Correct answer is option (b) possible
06. Statements ¼dFku½: (II) N<Z
A<D>C<E>B When we go from Z To N path possible so relation is possible.
Conclusions ¼fu"d"kZ½ : highest preference is of ‘>’ So correct relation is N<Z
I. C > B Correct answer is option (d)
II. A < E 9. Statements/dFku:
III. D > B £ < ≤ ≥= 
(a) Only I conclusion follows/dsoy I fu"d"kZ vuqlj.k djrk gS Conclusions/fu"d"kZ:
(b) Only II conclusion follows/dsoy II fu"d"kZ vuqlj.k djrk gS I. £ ≤  < 
(c) Only III conclusions follows/dsoy III fu"d"kZ vuqlj.k djrk gS (a) Both conclusions I and II follow
(d) Neither I, II nor III is true/uk gh I, II ,oa uk gh III fu"d"kZ lR; gS fu"d"kZ I vkSj II nksuksa vuqlj.k djrs gSa
(e) Either I and II is true/;k rks I vkSj II lR; gSA (b) Only conclusion I follows
(f) Both conclusion I or II, III is true/nksuks fu"d"kZ I ;k II, III lR; gS dsoy fu"d"kZ I vuqlj.k djrk gS
Sol. (d) I. C > D → C < D (c) Neither conclusion I nor conclusion II follows
Does not follow u rks fu"d"kZ I vkSj u gh fu"d"kZ II vuqlj.k djrk gS
II. A < E → E > C < D > A (d) Only conclusion II follows
Does not follow dsoy fu"d"kZ II vuqlj.k djrk gS
III. D > B → D > C < E > B Sol. (c)
Does not true Conclusions:-
CONCEPT (2)- TWO OR MORE THAN TWO STATEMENTS ARE Option(I) £ ≤ 
GIVEN  not possible
More then one statement are given in such a situations any Option (II) < 
two given statements can be combined only when there is  Not relation possible
common term between. Correct option is option (c)
Eg. M > N ; U  P; N > U 10 . Statements/dFku:
The statement M > N and N > U will be used to go from M to U. M > N  T; R < N  P
The common term in both statement is N, hence these Conclusions/fu"d"kZ:
statement can be combined.
I. M > R  P III. T  P
7. Statements/dFku:
(a) Both conclusions I and III follow
A > B; C  D; B = C
fu"d"kZ I vkSj III nksuksa vuqlj.k djrs gSa
Conclusions/fu"d"kZ:
(b) Only conclusion I and II follows
I. A  C dsoy fu"d"kZ I vkSj II vuqlj.k djrs gSa
II. B  D (c) None follows
(a) Only conclusions II follows dksbZ Hkh vuqlj.k ugha djrk gSA
dsoy fu"d"kZ II vuqlj.k djrk gS (d) Only conclusion III follows
(b) Only conclusions I follows
dsoy fu"d"kZ III vuqlj.k djrk gS
dsoy fu"d"kZ I vuqlj.k djrk gS
Sol. (a)
(c) Neither conclusions I nor II follows
I. M > R [>,>] hence the conclusion I follow.
u rks fu"d"kZ I vkSj u gh II vuqlj.k djrk gS
  P [>,] hence the conclusion II does not follow.
(d) Both conclusions I and II follow
III. T  P [,] hence the conclusion III follow.
fu"d"kZ I vkSj II nksuksa vuqlj.k djrs gSa
 hus Only I and III follow.
Sol. (d)
Correct option is option (a)
I. A  C [>, =] hence the conclusion I follow.
11 . Statements/dFku:
II. B  D [=,] hence the conclusion II follow.
R > P  U = S; P < W  Y
Both I and II follow.
Conclusions/fu"d"kZ:
8. Statement/dFku %
I. R > W U W
P > Z, N < T, M > N, Z > M
(a) Both conclusions I and II follow
Conclusion/fu"d"kZ%
fu"d"kZ I vkSj II nksuksa vuqlj.k djrs gSa
(i) P < T (ii) N < Z
(b) Only conclusion II follows
(a) Both conclusions I and II follow
dsoy fu"d"kZ II vuqlj.k djrk gS
fu"d"kZ I vkSj II nksuksa vuqlj.k djrs gSa
(c) Only conclusion I follows
(b) Only conclusion I follows

161
Download Free PDFs & e-Books from Neon Classes App

dsoy fu"d"kZ I vuqlj.k djrk gS (e) Both I and II follows/nksuks fu"d"kZ vuqlj.k djrs gS
(d) None follows Sol. (a) I. E > W → E = J > L > W
dksbZ Hkh vuqlj.k ugha djrk gSA Ist Conclusion follow
Sol. (b) II. M > L → M > N > R > W < L
I. R > W [>,<] hence the conclusion I does not follow. IInd Conclusion does not follow
 U W [, <] hence the conclusion II follow. CONCEPT (3) - POSSIBILITY
 hus Only II follow. There will be possibility such conclusions which are neither
Correct option is option (b) 100% true nor 100% false.
12 . A __ B __ C __ P __ Q __ R 100% false conclusion
Which of the following options makes ‘A >P’ and ‘ C  R ’ The conclusions which are neither 100% true nor there is any
definitely true? possibility for them, will be 100% false.
(a) > > > > > (b)  16 . Statements/dFku:
(c) > > =  = (d) > > = <  A  B  C; B < D  F; D E
Sol. (c) Conclusions/fu"d"kZ:
‘A > P’ will be true only when there comes the signs of greater I. A = C  F III. D  E
family from A to P and there should be at least one ‘>’ sign. IV. B < E V. F > E
Hence option (b) and (c) are incorrect. (i) How many conclusions are 100% true?
Similarly : -  (a) Only I follow
C  R will be true only when there comes the signs of less than (b) Only III follow
family from U to R and there should be at least one ‘’ sign. (c) Only IV follow
Hence option (d) is incorrect. (d) Only II and IV follows
Thus option (c) is follow. (ii) How many conclusions are 100% false?
13. Statements ¼dFku½: (a) Only I does not follow
A < B, B < C, C > D > E, E < F, F < L (b) Only III does not follow
Conclusions ¼fu"d"kZ½ : (c) Only V does not follow
I. B > E (d) Only III and IV does not follow
II. L > D Sol. (i & ii)
(a) Only I conclusion follows/dsoy I fu"d"kZ vuqlj.k djrk gS I. The relation from ‘A to C; A  C’
(b) Only II conclusion follows/dsoy II fu"d"kZ vuqlj.k djrk gS Hence there is possibility for the conclusion ‘A = C’.
(c) Either I or II follows/;k rks I ;k II fu"d"kZ vuqlj.k djrk gS II. The relation from ‘B to F’ the signs of both families are
coming between B and F.
(d) Neither I nor II follows/uk gh I uk gh II fu"d"kZ vuqlj.k djrk gS
Hence there is no relation between them.
(e) Both I and II follows/nksuks fu"d"kZ vuqlj.k djrs gS
In such a situation there is possibility for the conclusion ‘B = F’
Sol. (d) I. B > E ⟶ B > A
III. The relation from ‘D to E; D  E’.
B<C>D>E
Hence there is possibility for the conclusion ‘D = E’.
Does not follow
IV. The relation from ‘B to E; B < E’.
II. L > D → L > F > E < D
Hence the conclusion IV follows.
Does not follow
V. The relation from ‘F to E; F < E’.
14. Statements ¼dFku½:
Hence the conclusion ‘F > E’ is 100% false.
H < N, N < K, K < F
(i) Thus only IV follows answer (c) option.
Conclusions ¼fu"d"kZ½ :
(ii) Thus only conclusion V does not follow answer (c) option.
I. F > N CONCEPT (4) – EITHER - OR CONDITION
II. H < N There will be either-or case between any two given conclusions
(a) Only I conclusion follows/dsoy I fu"d"kZ vuqlj.k djrk gS when the following three condition.
(b) Only II conclusion follows/dsoy II fu"d"kZ vuqlj.k djrk gS (1) Both conclusion must be wrong.
(c) Either I or II follows/;k rks I ;k II fu"d"kZ vuqlj.k djrk gS (2) The subjects and objects of both conclusions must be same.
(d) Neither I nor II follows/uk gh I uk gh II fu"d"kZ vuqlj.k djrk gS (3) (i) If there is the relation of ‘’ between any two given
(e) Both I and II follows/nksuks fu"d"kZ vuqlj.k djrs gS elements, then there will be either-or case between the signs ‘>
Sol. (a) I. F > N → F > K > N and =’. Because the sign  is made of these two signs.
Ist Conclusion follow (ii) If there is the relation of  between any two given elements
II. H < N → N > H then there will be either-or case between the signs ‘< and =’.
Does not follow Because the sign  is made of these two signs.
15. Statements ¼dFku½: (iii) If there is no relation between two elements i.e. condition is
M > N > R > W, E = J > L > W block, then either-or case will be between the three signs ‘>, < and
Conclusions ¼fu"d"kZ½ : =’.
I. E > W 17. Statements/dFku:
II. M > L P  R  T  S.
(a) Only I conclusion follows/dsoy I fu"d"kZ vuqlj.k djrk gS Conclusions/fu"d"kZ:
(b) Only II conclusion follows/dsoy II fu"d"kZ vuqlj.k djrk gS I. P > T P T III. S  R
(c) Either I or II follows/;k rks I ;k II fu"d"kZ vuqlj.k djrk gS (a) Only III follow
(d) Neither I nor II follows/uk gh I uk gh II fu"d"kZ vuqlj.k djrk gS (b) Either I or II follow

162
Best App for Govt. Jobs : Neonclasses (Download Now)

(c) Only II follows In this, the relation between two or more elements is not
(d) Only either I or II and III follow shown derectly using the signs of inequalities but they are
Sol. (d) shown using some symbols.
I. P > T [, =] hence the conclusion I does not follows. While solving such questions we make a small box. In this box
 P T [ , =] hence the conclusion II does not follow. we will show all the inequality signs and the symbols related
III. S  R [ , =] hence the conclusion III follow. to those. Let’s understand it with the help of the following
Here the definite result from ‘P to T, P  T’ examples:
Here the both conclusion I and II are wrongs, the subjects and Directions (20 to 22): In the following questions the
object of both conclusion are the same and both the signs symbols $,©,@,# and % are used with the following
and = required for  are present. meanings:
Hence there will be either-or case between I and II A$B means A is not less than B.
conclusions. A©B means A is greater than B.
Thus only either I or II and III follows. A@B means A is equal to B.
18. ‘#’ means ‘>’, ‘$’ means ‘≥’, ‘%’ means ‘<’, and ‘@’ means ‘≤’. A#B means A is not greater than B
Assuming the following statements to be true, which of the A%B means A is less than B.
conclusions given in the options is definitely true. Now in each or the following questions, assuming the three
fuEufyf[kr dFkuksa dks lR; ekurs gq, fodYiksa esa fn, x, fu"d"kksZa esa statements to be true, find which of the two conclusions I and
ls dkSu lk fuf'pr :i ls lR; gSA II is/are true. Give answer
B$D$A (A) Only I follows
R%D%L@J (B) Only II follows
(a) A % L (b) J % D (c) B # L (d) B $ J (C) Either I or II follows
Sol. (a) (D) Neither I nor II follows
Symbol dks fpUg esa change djus ds ckn (E) Both I and II follow
⇒ B≥D≥A, R<D<L≤J 20. Statements: A$B ; B©D ; D@I
Option (a) A%L A<L, A≤D<L Conclusions: I. A©D II. B@I
When we go from R to L , path possible so relation is possible 21. Statements: M%O©S ; R#T
Highest preference is of ‘<’ so correct relation is A<L Conclusions: I. M©S II. M%S
Option (b) J%D J<D 22. Statements: M©N ; R%N
When we go from D to J Path Not possible so relation is not Conclusions: I. M©R II. M%R
possible Sol. To Solve such questions easily we will make a small box and
Option (c) B#L B>L show the inequality signs and the symbols related to those in
When we go from path not possible it. Here:
Option (d) B$J B≥J $ => ≥ ; ©=> > ; @=> = ; #=> ≤ ; %=> <
When we go from path not possible so relation not possible These all will be shown as follows in the box:
Correct answer is option (a)
© %
19. If G = E < D < R and E = Y > K > Q, then which of the following
options is NOT correct? > <
;fn G = E < D < R vkSj E = Y > K > Q, rks fuEufyf[kr esa ls dkSulk
fodYi lgh ugha gS\
(a) G < Q (b) R > Q (c) G < R (d) Y < R
Sol. (a)
Option (a) G<Q @=
When we go from Q to G, Path not possible so relation is not
possible
Option (b) R>Q

E<D<R, E= Y>K>Q
≥ ≤
when we go from R to Q, Path possible so relation is possible $ #Now if the symbols
Highest preference is of ‘>’ So correct relation is R>Q
©, $ and @ come in between the two elements, then choose
Option (c) G<R G  E  D  R the symbol ‘©’. Similarly choose the symbol ‘©’ between ©
When we go from R to G, path possible so relation is possible and @ and choose the symbol ‘$’ between $ and @.
highest preference is of ‘<’ So correct relation is G<R Simillarly choose the symbol ‘%’ among %,# and @, choose the
Option (d) Y<R R  D  E, E  Y symbol ‘%’ between % and @ and choose the symbol ‘#’
between # and @. Apart from that, condition will be block if
When we go from R to Y, path possible so relation is possible
highest preference is of ‘>’ so correct relation is Y<R the signs of both families come in between.
Concept(5): Coded Inequalites:
,sls iz’kuksa dks vklkuh ls gy djus ds fy, ge ,d ckWDlcuk,axs vkSj
mlesa vlekurk okys fpàksa o muls lacaf/kr izrhdksa dks fn[kk,axsA ;gk¡%

163
Download Free PDFs & e-Books from Neon Classes App

$ => ≥ ; ©=> > ; @=> = ; #=> ≤ ; %=> < 23. sol.(b)


bu lHkh dks ckWDl esa fuEu izdkj n’kkZ,axs% S = Q, Q < B, B > K, K < W
vc ;fn nks vo;oksa ds chp esa ‘©’, ‘$’ vkSj ‘@’ vk, rks ‘©’ dks I. W % K → W > K → W > K
pqusaxsA blh izdkj ‘©’ vkSj ‘@’ esa ‘©’ dks pqusaxs ‘$’ vkSj ‘@’ vkSj ‘$’ Ist Conclusion does not follow
sa sA blh rjg ‘%’,’#’ vkSj ‘@’ esa ‘%’ dks pqusaxs] ‘%’ vkSj
esa dks pqux II. S @ B → S < B → B > Q = S
‘@’ esa ‘%’ dks pqux sa s vkSj ’#’ vkSj ‘@’ esa ’#’ dks pqusaxsA IInd Conclusion is follow
blds vykok nksuksa QSfeyht ds fpà chp esa vkus ij dafM’ku CykWd 24. sol.
gksxhA (c)
20. (A) Z > N, N < K, K = M, M < R
I. A©D [$,©] I. M $ N → M = N → M = K > N
Hence the conclusion I follows. I. Conclusion does not follow
II. B@I [©,@] II. M % N →M > N → M = K > N
Hence the conclusion II dose not follow. Does not follow Conclusion I and II follow 3 symbol (>, <, =)
Thus only I follows. case
21. (B) So either or case will be apply
I. M©S[%,©] 25. sol.
There is condition block between R and T. Hence the (c)
conclusion I does not follow. W > P, P > G, G < I, I < N
II. Hence the conclusion II follows. I. N % W → N > W → N > I > G < P < W
Thus only II follows.
Does not follow
22. (A)
II. N # W → N < W → W > P > G < I < N
I. M©R [©,©]
Does not follow Conclusion I and II follow 3 symbol (>, < =)
Hence conclusion I follows and II doesnot follows.
case
Thus only conclusion I follows.
So either or case will be apply.
Notes: The symbols will be reversed while moving from M to R.
Hence ‘%’ is converted into ‘©’. 4. Letter Analogy
Directions (23 to 25):: In these questions Symbols $, @, %, # and FIRST TO FIRST
& are used with the different meaning as follows. Fix addition
‘A $ B’ means ‘A is neither greater than nor smaller than B. Fix subtraction
‘A $ B dk eryc ^A, B ls u rks cM+k vkSj u gh NksVk gSA Alternate addition
‘A @ B’ means ‘A is neither greater than nor equal to B’ Mix variation alternate
‘A @ B dk eryc ^A, B ls u rks cM+k vkSj u gh cjkcj gSA Series (addition)
‘A % B’ means ‘A is neither smaller than nor equal to B’. Series (subtraction)
‘A % B dk eryc ^A, B ls u rks NksVk vkSj u gh cjkcj gSA Alternate addition/subtraction series
‘A & B’ means A is not smaller than B’ Series (Odd ,Even)
‘A & B dk eryc ^A, B ls NksVk ugha gS Opposite relation
‘A # B’ means A is not greater than B’. Vowel/Consonant based
‘A # B dk eryc ^A, B ls cM+k ugha gSA Increasing/Decreasing order
(a) Only I is true 1. Select the option that is related to the fifth letter-cluster in
dsoy I lR; gSA the same way as the second letter-cluster is related to the
(b) Only II is true first letter-cluster and the fourth letter-cluster is related
dsoy II lR; gSA to the third letter-cluster.
(c) Either I or II is true ml fodYi dk p;u djsa tks ikaposa v{kj&lewg ls mlh çdkj lacfa èkr
;k rks I ;k II lR; gSA gS tSls nwljk v{kj&lewg igys v{kj&lewg ls lacafèkr gS vkSj pkSFkk
(d) Neither I nor II is true v{kj&lewg rhljs v{kj&lewg ls lacafèkr gSA
uk rks I uk gh II lR; gSA VOCAL : CVJHS :: TROOF : AYVVM :: RTAGE : ?
(e) Both I and II is true (a) YAHNN (b) YAHML
nksuks I vkSj II lR; gSA (c) YAHNM (d) YAHNL
23. Statements/¼dFku½: Sol. (d) YAHNL
S $ Q, Q @ B, B & K, K # W Pattern : First to first
Conclusions ¼fu"d"kZ½ : Logic : addition of 7 in every letter ot letter .
I. W % K II. S @ B 22 15 3 1 12
24. Statements/¼dFku½: V O C A L
+7 +7 +7 +7 +7
Z % N, N # K, K $ M, M @ R
Conclusions ¼fu"d"kZ½ : C V J H S
I. M $ N II. M % N 3 22 10 8 19
25. Statements/¼dFku½:
W % P, P % G, G @ I, I # N
Conclusions ¼fu"d"kZ½ :
I. N % W II. N # W

164
Best App for Govt. Jobs : Neonclasses (Download Now)

20 18 15 15 6 19 8 15 23 5 18
T R O O F S H O W E R
+7 +7 +7 +7 +7 +2 +0 +2 +0 +2 +0
A Y V V M U H Q W G R
1 25 22 22 13 21 8 17 23 7 18
18 20 1 7 5 14 5 5 4 12 5
R T A G E N E E D L E
+7 +7 +7 +7 +7 +2 +0 +2 +0 +2 +0
Y A H N L P E G D N E
25 1 8 14 12 16 5 7 4 14 5
Elimination method : Elimination:-
(a) from last letter E + 7 = 12(L) So, N is wrong gets (a). Because of 1st letter N, option gets eliminated.
eliminated. (c) Because of 1st letter N, option gets eliminated.
(b) 4th letter M wrong in option so gets eliminated. (d) Because of 3rd letter D, option gets eliminated.
(c) 5th letter L wrong in option so gets eliminated. 4. Select the option that is related to the fifth letter-cluster in
2. Select the option that is related to the fifth letter-cluster in the same way as the second letter-cluster is related to the
the same way as the second letter-cluster is related to the first letter-cluster and the fourth letter-cluster is related
first letter-cluster and the fourth letter-cluster is related to the third letter-cluster.
to the third letter-cluster. ml fodYi dk p;u djsa tks ikaposa v{kj&lewg ls mlh çdkj lacfa èkr
ml fodYi dk p;u djsa tks ikaposa v{kj&lewg ls mlh çdkj lacfa èkr gS tSls nwljk v{kj&lewg igys v{kj&lewg ls lacafèkr gS vkSj pkSFkk
gS tSls nwljk v{kj&lewg igys v{kj&lewg ls lacafèkr gS vkSj pkSFkk v{kj&lewg rhljs v{kj&lewg ls lacafèkr gSA
v{kj&lewg rhljs v{kj&lewg ls lacafèkr gSA AZBZ : CYDY :: EXFX : GWHW :: IVJV : ?
GOOD: CKKZ :: BEST : XAOP :: COOL : ? (a) KVLV (b) KULU (c) IUJU (d) IJKV
(a) WKKH (b) YKKI Sol. (b) KULU
(c) YKKH (d) YJJH Pattern: first to first, alternate
Sol. (c) YKKH Logic: odd position letter + 2
Pattern : First to first, fix subtraction Even position letter – 1
Logic : (letter – 4) 1 26 2 26
A Z B Z
7 15 15 4 2 5 19 20 +2 -1 +2 -1
G O O D B E S T C Y D Y
-4 -4 -4 -4 -4 -4 -4 -4 3 25 4 25
C K K Z X A O P 5 24 6 24
3 11 11 26 24 1 15 16 E X F X
3 15 15 12 +2 -1 +2 -1
C O O L G W H W
-4 -4 -4 -4 7 23 8 23
Y K K H 9 22 10 22
25 11 11 8 I V J V
Elimination method : +2 -1 +2 -1
(a) 1st letter W is wrong , So option gets eliminated. K U L U
(b) 4th letter I given wrong, so option gets eliminated. 11 21 12 21
(d) 2nd , 3rd letter J given wrong, so option gets eliminated. Elimination:-
3. Select the option that is related to the fifth term in the (a) 2nd letter V wrong so option eliminated.
same way as the second term is related to the first term (c) 1st letter I wrong so option eliminated.
and the fourth term is related to the third term. (d) 1st letter I wrong so option eliminated.
ml fodYi dk p;u djsa tks ikaposa in ls mlh çdkj lacfa èkr gS tSls 5. Select the option that is related to the third term in the
nwljk in igys in ls vkSj pkSFkk in rhljs in ls lacafèkr gSA same way as the second term is related to the first term.
COMPUTER : EOOPWTGR :: SHOWER : UHQWGR :: NEEDLE ml fodYi dk p;u djsa tks rhljs in ls mlh izdkj lacaf/kr gS tSls
:? nwljk in igys in ls lacaf/kr gSA
(a) NGEFLG (b) PEGDNE CANOPY : DCQSUE : : TONSIL : ?
(c) NGEFLE (d) PEDGNE (a) URQWNS (b) UQQXOR
Sol. (b) PEGDNE (c) UQRVNR (d) UQQWNR
Pattern: Add mix first to first Sol. (d) UQQWNR
Logic: even position letter as it is and odd position letter + 2 Pattern : Adddition series
3 15 13 16 21 20 5 18 Logic : increasing in consecucative number series order . in
C O M P U T E R every next letter.
+2 +0 +2 +0 +2 +0 +2 +0
E O O P W T G R
5 15 15 16 23 20 7 18

165
Download Free PDFs & e-Books from Neon Classes App

3 1 14 15 16 25 18 10 2 17 16 7
C A N O P Y R J B Q P G
+1 +2 +3 +4 +5 +6 +2 -3 +4 +2 -3 +4
D C Q S U E T G F S M K
4 3 17 19 21 5 20 7 6 , 19 13 11
20 15 14 19 9 12 Elimination:-
T O N S I L (b) last letter J wrong so eliminated.
+1 +2 +3 +4 +5 +6 (c) first letter T wrong so eliminated.
U Q Q W N R (d) first letter R wrong so gets eliminated.
21 17 17 23 14 18 8. Select the option that is related to the fifth letter-cluster in
Elimination:- the same way as the second letter-cluster is related to the
(a) from last letter S, option gets eliminated. first letter-cluster and the fourth letter-cluster is related
(b) from 2nd last letter O, option gets eliminated. to the third letter-cluster.
(c) from 3rd letter R, option gets eliminated. ml fodYi dk p;u djsa tks ikaposa v{kj&lewg ls mlh çdkj lacfa èkr
6. Select the option that is related to the fifth letter-cluster in gS tSls nwljk v{kj&lewg igys v{kj&lewg ls lacafèkr gS vkSj pkSFkk
the same way as the second letter-cluster is related to the v{kj&lewg rhljs v{kj&lewg ls lacafèkr gSA
first letter-cluster and the fourth letter-cluster is related METHOD : OIZPYP :: SAFETY : UELMDK :: YELLOW : ?
to the third letter-cluster. (a) AIRTXI (b) AIRTYJ
ml fodYi dk p;u djsa tks ikaposa v{kj&lewg ls mlh çdkj lacfa èkr (c) AIRTXJ (d) AIRTYI
gS tSls nwljk v{kj&lewg igys v{kj&lewg ls lacafèkr gS vkSj pkSFkk Sol. (d) AIRTYI
v{kj&lewg rhljs v{kj&lewg ls lacafèkr gSA Pattern :- Addition series
BEWMTT : ACTION :: KWQMTX : JUNIOR :: QTRJNZ : ? Logic:- increasing in even number order.
(a) PROFIT (b) PROEIT 13 5 20 8 15 4
(c) PROFTI (d) PROFIU M E T H O D
Sol. (a) PROFIT +2 +4 +6 +8 +10 +12
Pattern: Sub series O I Z P Y P
Logic:- Subtraction in consecutive numbers series. 15 9 26 16 25 16
2 5 23 13 20 20 19 1 6 5 20 25
B E W M T T S A F E T Y
-1 -2 -3 -4 -5 -6 +2 +4 +6 +8 +10 +12
A C T I O N U E L M D K
1 3 20 9 15 14 21 5 12 13 4 11
11 23 17 13 20 24 25 5 12 12 15 23
K W Q M T X Y E L L O W
-1 -2 -3 -4 -5 -6 +2 +4 +6 +8 +10 +12
J U N I O R A I R T Y I
10 21 14 9 15 18 1 9 18 20 25 9
17 20 18 10 14 26 Eliminition:
Q T R J N Z (a) from 2nd last letter X, option gets eliminated.
-1 -2 -3 -4 -5 -6 (b) from last letter J, option gets eliminated.
P R O F I T (c) from last letter J, option gets eliminated.
16 18 15 6 9 20 9. Select the option that is related to the third letter-cluster
Elimination: in the same way as the second letter-cluster is related to
(b) 4th letter E is wrong so get eliminate. the first letter-cluster.
(c) last letter I is wrong so gets eliminated. ml fodYi dk p;u djsa tks rhljs v{kj&lewg ls mlh çdkj lacafèkr
(d) last letter U is wrong so gets eliminated. gS tSls nwljk v{kj&lewg igys v{kj&lewg ls lacafèkr gSA
7. Select the option that is related to the third letter-cluster PV : MQ :: FB : ?
in the same way as the second letter-cluster is related to (a) DX (b) CW (c) CX (d) DW
the first letter cluster. Sol. (b) CW
ml fodYi dk p;u djsa tks rhljs v{kj&lewg ls mlh çdkj lacfa èkr Pattern: Sub Series
gS tSls nwljk v{kj&lewg igys v{kj lewg ls lacafèkr gSA Logic: (1st letter – 3), (2nd letter -5) odd number series
RJB : TGF :: QPG : ? 16 22 6 2
(a) SMK (b) SOJ (c) TNJ (d) RMK P V F B
Sol. (a) SMK -3 -5 -3 -5
Pattern: Alternate series, first to first M Q C W
Logic: addition, subtraction altenate in series. 13 17 , 3 23
Elimination:-
(a) Both letter wrong so gets eliminated.
(c) 2nd letter X wrong so gets eliminated.

166
Best App for Govt. Jobs : Neonclasses (Download Now)

(d) 1st letter D wrong do gets eliminated. 7 15 15 4


10. Select the option that is related to the third letter-cluster G O O D
in the same way as the second letter-cluster is related to +1 +4 +9 +16
the first letter-cluster. H S X T
ml fodYi dk p;u djsa tks rhljs v{kj&lewg ls mlh çdkj lacaf/kr 8 19 24 20
gS tSls nwljk v{kj&lewg igys v{kj&lewg ls lacaf/kr gSA 2 5 19 20
BECD : YVXW : : DGEF : ? B E S T
(a) VRTS (b) WTVU +1 +4 +9 +16
(c) WUTV (d) XUWV C I B J
Sol. (b) WTVU 3 9 2 10
Pattern : first to first
Logic:- Opposite letter to letter. 3 15 15 12
2 5 3 4 C O O L
B E C D +1 +4 +9 +16
D S X B
Y V X W 4 19 24 2
25 22 24 23
Elimination:-
4 7 5 6
(a) last letter Z is wrong so option eliminated.
D G E F
(c) 2nd last letter S is wrong so option eliminated
(d) 2nd last letter Z is wrong so option eliminated.
W T V U 13. Select the option that is related to the third term in the
23 20 22 21 same way as the second term is related to the first term.
Elimintation:- ml fodYi dk p;u djsa tks rhljs in ls mlh çdkj lacafèkr gS tSls
(a) first letter V is wrong so gets eliminated. nwljk in igys in ls lacafèkr gSA
(c) from last letter V option gets eliminated. FRONTIER : GSNOUHDS :: CLOSING : ?
(d) from first letter X option gets eliminated. (a) DMNTHOH (b) BMPTHOH
11. Select the option that is related to the third letter-cluster (c) DMNRHLH (d) DKNTJOH
in the same way as the second letter-cluster is related to Sol. (a) DMNTHOH
the first letter-cluster. Pattern: first to first
ml fodYi dk p;u djsa tks rhljs v{kj&lewg ls mlh izdkj lacaf/kr Logic:- (Consonant +1), (Vowel -1)
gS ftl izdkj nwljk v{kj&lewg igys v{kj&lewg ls lacaf/kr gSA 6 18 15 14 20 9 5 18
DFC : WHX : : KFQ : ? F R O N T I E R
(a) PHJ (b) QHJ (c) RGK (d) QHK +1 +1 -1 +1 +1 -1 -1 +1
Sol. (a) PHJ G S N O U H D S
Pattern: First to first 7 19 14 15 21 8 4 19
Logic:- (2nd letter +2), 1st & 3rd letters opposite 3 12 15 19 9 14 7
4 6 3 11 6 17 C L O S I N G
D F C K F Q +1 +1 -1 +1 -1 +1 +1
+2 +2
D M N T H O H
W H X P H J 4 13 14 20 8 15 8
23 8 24 , 16 8 10
Elinination:-
Elimination:- (b) 1st letter B wrong so option gets eliminated.
(b) from 1st letter Q wrong, so eliminated. (c) 2nd last letter L wrong so option gets eliminated.
(c) All letters wrong so option eliminate. (d) 2nd letter K wrong so option gets eliminated.
(d) from 1st letter Q option gets eliminated. 14. Select the option that is related to the third letter-cluster
12. Select the option that is related to the fifth letter-cluster in in the same way as the second letter-cluster is related to
the same way as the second letter-cluster is related to the the first letter-cluster.
first letter-cluster and the fourth letter-cluster is related ml fodYi dk p;u djsa tks rhljs v{kj&lewg ls mlh çdkj
to the third letter-cluster. lacaf/kr gS tSls nwljk v{kj&lewg igys v{kj&lewg ls lacaf/kr gSA
ml fodYi dk p;u djsa tks ikaposa v{kj&lewg ls mlh çdkj lacfa èkr NEUTSE : EFPVXZ : : ZWURST : ?
gS tSls nwljk v{kj&lewg igys v{kj&lewg ls lacafèkr gS vkSj pkSFkk (a) RTSVEA (b) QXPXAE
v{kj&lewg rhljs v{kj&lewg ls lacafèkr gSA (c) RTVXAE (d) QXPTWA
GOOD : HSXT :: BEST : CIBJ :: COOL : ? Sol. (c) RTVXAE
(a) DSXZ (b) DSXB (c) DSSB (d) DSZB Pattern: first to first (increasing)
Sol. (b) DSXB Logic : letters are arranged and then each letter increased in
Pattern :- first to first series order (+0, +1, +2,……)
Logic : (Square series in increasing order) NEUTSE
In dictionary order

167
Download Free PDFs & e-Books from Neon Classes App

5 5 14 19 20 21 13 1 18 19
E E N S T U M A R S
+0 +1 +2 +3 +4 +5 +1 -1 +1 -1 +1
-1 +1 -1
E F P V X Z
5 6 16 22 24 26 N L B Z S Q T R
ZWURST 14 12 2 26 19 17 20 18
In dictionary order
18 19 20 22 23 26 16 12 21 20 15
R S T U W Z P L U T O
+0 +1 +2 +3 +4 +5 +1 -1 +1
-1 +1 -1 +1 -1 +1 -1
R T V X A E
18 20 21 24 1 5 Q O M K V T U S P N
Elimination: 17 15 13 11 22 20 21 19 16 14
(a) Last letter A wrong So option gets eliminated. Similarly
(b) from first letter Q, option gets eliminated. 5 1 18 20 8
(d) first letter Q wrong, So option gets eliminated. E A R T H
15. Select the option that is related to the third term in the +1 -1 +1
-1 +1 -1 +1 -1 +1 -1
same way as the second term is related to the first term.
ml fodYi dk p;u djsa tks rhljs in ls mlh çdkj lacafèkr gS tSls
F D B Z S Q U S I G
nwljk in igys in ls lacafèkr gSA 6 4 2 26 19 17 21 19 9 7
IVORY : ZWSPJ :: CREAM : ?
(a) BQDZL (b) NFDQB
(c) DSFCN(d) SNFDB
17. Select the option that is related to the third term in the
Sol. (d) SNFDB
same way as the second term is related to the first term.
Pattern: first to first (decreasing)
ml fodYi dk p;u djsa tks rhljs in ls mlh çdkj lacafèkr gS tSls
Logic: letters are arranged in decreasing order then letter
+1
nwljk in igys in ls lacafèkr gSA
IVORY ZPEEXFLCRQ : SHIFT ∷ ATEXXDIBVO : ?
Decreasing order (a) ADORE (b) WAGER
25 22 18 15 9 (c) VADER (d) EAGLE
Y V R O I Sol. (b) WAGER
+1 +1 +1 +1 +1 Pattern: One to two letters.
Z W S P J Logic: (Only even position letter +3)
26 23 19 16 10 16 5 6 3 17
ZP E E X F L C R Q
CREAM
+3 +3 +3 +3 +3
Decreasing order
18 13 5 3 1 S H I F T
R M E C A 19 8 9 6 20
+1 +1 +1 +1 +1 20 24 4 2 15
AT E X X D I B V O
S N F D B
+3 +3 +3 +3 +3
19 14 6 4 2
Elimination: W A G E R
(a) from first letter B, option gets eliminated. 23 1 7 5 18
(b) from first letter N, option gets eliminated. (a) from first letter A, option gets eliminated.
(c) first letter D wrong, so option gets eliminated. (c) from first letter V, option gets eliminated.
16. Select the option that is related to the fifth letter-cluster in (d) from first letter E, option gets eliminated.
the same way as the second letter-cluster is related to the FIRST TO LAST (LETTER TO LETTER)
first letter-cluster and the fourth letter-cluster is related Fix addition
to the third letter-cluster. Fix subtraction
ml fodYi dk p;u djsa tks ikaposa v{kj&lewg ls mlh çdkj lacfa èkr Mix variation alternate
gS tSls nwljk v{kj&lewg igys v{kj&lewg ls lacafèkr gS vkSj pkSFkk Opposite relation
v{kj&lewg rhljs v{kj&lewg ls lacafèkr gSA Series (addition)
MARS : NLBZSQTR :: PLUTO : QOMKVTUSPN :: EARTH : ? Opposite relation
(a) QOSFDDIUAS (b) QUIESEDILA 18. Select the option that is related to the third term in the
(c) FDBZSQUSIG (d) FDZBQSSUIG same way as the second term is related to the first term.
Sol. (c) FDBZSQUSIG ml fodYi dk p;u djsa tks rhljs in ls mlh izdkj lacaf/kr gS tSls
nwljk in igys in ls lacaf/kr gSA
QUICKER : TGMEKWS : : REPORTS : ?
(a) UVTQRGT (b) VUTQRGS
(c) USVQRGT (d) UVTRRHT

168
Best App for Govt. Jobs : Neonclasses (Download Now)

Sol. (a) UVTQRGT SEVERAL : MZSDWDT :: TRACKED : EDLBBQU :: HEADING :


Pattern: Addition cross ?
Logic: (letter +2) in crossing order (a) IDBCJMH (b) HMJCBDI
(c) JDBCIMH (d) HMICBDJ
17 21 9 3 11 5 18 Sol. (b) HMJCBDI
Q U I C K E R Pattern: Alternate cross
+2
Logic: (letter+1), (letter -1) in alternate order cross
+2 +2 +2 +2 +2 +2 19 5 22 5 18 1 12
+2 +2 +2 S E V E R A L
-1 +1 +1 -1 +1
T G M E K W S +1 -1
20 7 13 5 11 23 19
18 5 16 15 18 20 19 M Z S D W D T
R E P O R T S 13 26 19 4 23 4 20
+2
20 18 1 3 11 5 4
+2 +2 +2 +2 +2 +2
T R A C K E D
+2 +2 +2 -1 +1 +1 -1 +1
+1 -1
U V T Q R G T
21 22 20 17 18 7 20
E D L B B Q U
Elimination: 5 4 12 2 2 17 21
(b) first letter V, so get eliminated.
(c) 2nd letter is S, so gets eliminated. 8 5 1 4 9 14 7
(d) 2nd last letter H, so gets eliminated. H E A D I N G
19. Select the option in which the pair of letter-clusters share -1 +1 +1 -1 +1
+1 -1
the same relationship as that shared by the given pair of
letter-clusters.
H M J C B D I
ml fodYi dk p;u djsa ftlesa v{kj&lewgksa dk ;qXe ogh lacaèk
8 13 10 3 2 4 9
j[krk gS tks fn, x, v{kj&lewgksa ds ;qXe esa laca/k gSA
Elimination:-
TRACK : IAYPR :: ?
(a) 1st letter I wrong so eliminated.
(a) PROUD : NPMSB
(c) 1st letter J wrong so eliminated.
(b) GAMES : UGOCE
(d)last letter J wrong so eliminated.
(c) STRAW : UYPRQ
21. Select the option that is related to the third letter-cluster
(d) POLDY : WBJQN
in the same way as the second letter-cluster is related to
Sol. (c) STRAW : UYPRQ
the first letter-cluster.
Pattern: fix sub cross
ml fodYi dk p;u djsa tks rhljs v{kj&lewg ls mlh çdkj lacafèkr
Logic: (letter -2) in crossing order.
20 18 1 3 11
gS tSls nwljk v{kj&lewg igys v{kj&lewg ls lacafèkr gSA
T R A -2 C -2 K LON : RRN :: MAD : ?
-2 -2 -2 (a) HEO (b) HDO (c) IDO (d) IEO
Sol. (b) HDO
Pattern: Add series cross
I A Y P R
Logic: addition in series order +2, +3, +4
9 1 25 16 18
12 15 14 13 1 4
19 20 18 1 23 L O N M A D
S T R -2 A -2 W +2 +3 +4 +2 +3 +4
-2 -2
-2
R R N H D O
18 18 14 , 8 4 15
U Y P R Q
21 25 16 18 17
Elimination:
(a) 2nd letter E wrong, so eliminated.
Elimination:
(c) 1st letter I wrong , so eliminated.
(a) P (16)-2 = 14 (N) last letter not N so eliminated.
(d) 1st letter I wrong, so eliminated.
(b) S & U pair wrong so eliminated.
22. Select the option that is related to the third term in the
(d) O & Q Pair wrong so eliminated.
same way as the second term is related to the first term.
20. Select the option that is related to the fifth letter-cluster in
ml fodYi dk p;u djsa tks rhljs in ls mlh izdkj lacaf/kr gS tSls
the same way as the second letter-cluster is related to the
nwljk in igys in ls lacaf/kr gSA
first letter-cluster and the fourth letter-cluster is related
CRAW : DZIX :: MOCK : ?
to the third letter-cluster.
(a) PYLN (b) PXMN (c) PXLN (d) PLXN
ml fodYi dk p;u djsa tks ikaposa v{kj&lewg ls mlh çdkj lacfa èkr
Sol. (c) PXLN
gS tSls nwljk v{kj&lewg igys v{kj&lewg ls lacafèkr gS vkSj pkSFkk
Pattern: Opposite Crossing
v{kj&lewg rhljs v{kj&lewg ls lacafèkr gSA
Logic: Letter of opposite letter cross

169
Download Free PDFs & e-Books from Neon Classes App

3 18 1 23 16 1 20 5 12 19
C R A W P A T E L S
+1 +1 +1 +1 +1 +1

D Z I X B Q F U T M
4 26 9 24 2 17 6 21 20 13

13 15 3 11 14 5 3 20 1 18
M O C K N E C T A R
+1 +1 +1 +1 +1 +1

P X L N F O U D S B
16 24 12 14 6 15 21 4 19 2
Elimination: Elimination:
(a) 2nd letter Y wrong so eliminated. (a) last letter Z is wrong so gets eliminated.
(b) 3rd letter M wrong so eliminated. (c) last letter S is wrong so gets eliminated.
(d) 2nd & 3rd letter wrong so eliminated. (d) 1st letter O is wrong so gets eliminated.
23. Select the option that is related to the third term in the 25. Select the option that is related to the third term in the
same way as the second term is related to the fnst term. same way as the second term is related to the first term.
ml fodYi dk p;u djsa tks rhljs in ls mlh çdkj lacafèkr gS tSls ml fodYi dk p;u dhft,] ftldk rhljs in ls ogh lac/a k gS] tks
nwljk in çFke in ls lacafèkr gSA nwljs in dk igys in ls gSA
GOWE : OWOG :: MXML : ........ SANITY : OBTZUJ :: FOSTER : ?
(a) LMLM (b) MXMX (c) XLXL (d) XMXM (a) TGPHFU (b) TPGSFU
Sol. (d) XMXM (c) TPESFV (d) TQGSGU
Sol. (b) TPGSFU
7 15 23 5 19 1 14 9 20 25
G O W E S A N I T Y
+1
+1 +1 +1 +1 +1

O W O G O B T Z U J
15 23 15 7 15 2 20 26 21 10
6 15 19 20 5 18
13 24 13 12 F O S T E R
+1
M X M L +1 +1 +1 +1 +1

T P G S F U
X M X M 20 16 7 19 6 21
24 13 24 13 26. Select the option that is related to the third term in the
Elimination: same way as the second term is related to the first term.
(a) 1st letter L wrong so eliminated. ml fodYi dk p;u djsa tks rhljs in ls mlh çdkj lacaf/kr gS tSls
(b) 1st letter M wrong so eliminated. nwljk in igys in ls lacaf/kr gSA
(c) last letter L wrong so eliminated. JOCKY : PKCXJ : : BHAJI : ?
CROSS/GROUPING (LETTER TO LETTER) (a) ICIAH (b) ICAIH (c) ICAHI (d) CAIHI
Fix addition (2*2, 3*3, 4*4, 5*5) Sol. (c) ICAHI
Fix addition,Subtraction Pattern: fix addition, sub and cross
Mix Variation Logic: 1st cross (letter +1), Middle letter as it is 2nd cross (letter
Opposite/Mix opposite -1)
24. Select the option that is related to the third term in the 10 15 3 11 25
same way as the second term is related to the first term. J O C K Y
ml fodYi dk p;u djsa tks rhljs in ls mlh çdkj lacafèkr gS tSls +1 +1 +0 -1 -1
nwljk in igys in ls lacafèkr gSA
PATELS : BQFUTM :: NECTAR : ? P K C X J
(a) FODUSZ (b) FOUDSB 16 11 3 24 10
(c) FOVDBS (d) OEUDQB
Sol. (b) FOUDSB
Pattern: Fix addition and cross
Logic: (letter +1) cross order

170
Best App for Govt. Jobs : Neonclasses (Download Now)

2 8 1 10 9 Logic : 4th & 5th letter opposite letter and remaining 1st three &
B H A J I last three increased by 1 in cross order.
+1 +1 +0 -1 -1 3 1 18 14 1 20 9 3
C A +1 R N A T I C
+1 +1 opp opp +1 +1 +1
I C A H I
9 3 1 8 9
Elimination: S B D M Z D J U
(a) last letter H wrong so eliminated. 19 2 4 13 25 4 10 21
(b) last letter H wrong so eliminated.
(d) 1st letter C wrong so eliminated. 6 18 5 5 4 15 13 19
27. Select the option that is related to the fifth letter- cluster F R +1 E E D O M+1S
+1 +1 opp opp +1 +1
in the same way as the fourth letter – cluster is related to
the third letter – cluster and the second letter – cluster is
related to the first letter – cluster. F S G V W T N P
ml fodYi dk p;u djsa tks ikaposa v{kj leqg ls mlh rjg lacafèkr 6 19 7 22 23 20 14 16
gS tSls pkSFkk v{kj leqg rhljs v{kj leqg ls lacafèkr gS vkSj nqljk Elimination:
v{kj leqg igys v{kj leqg ls lacafèkr gSA (b) 3rd last letter P wrong so eliminated.
AYTRES : SFTUZB :: MINTED : UFEOJN :: LTREAD : ? (c) last letter V wrong so eliminated.
(a) FBETRS (b) DFRHRE (d) 1st letter G wrong so eliminated.
(c) FBESUM (d) DFRERT 29. Select the option that is related to the third term in the
Sol. (c) FBESUM same way as the second term is related to the first term.
Pattern: Random cross ml fodYi dk p;u djsa tks rhljs in ls mlh çdkj lacaf/kr gS tSls
 1st half in last cross  nwljk in igys in ls lacaf/kr gSA
Logic: (letter + 1)  
 2ndhalf in first cross  TURNIP : QTSQJO : : RADISH : ?
(a) DZRITJ (b) CZQGRH
1 25 20 18 5 19
(c) BZPHSJ (d) CZQITJ
A Y T+1 R +1E +1 S
+1 +1 +1 Sol. (d) CZQITJ
Pattern: Fix subtraction 3 ×3 cross mix
Logic: (letter -1) in 3 × 3 cross
S F T U Z B
(letter +1) in 3 × 3 cross
19 6 20 21 26 2 20 21 18 14 9 16
T U -1 R N I +1P
13 9 14 20 5 4 -1 -1 +1 +1
M I
+1 +1 N+1 +1
T +1E +1 D
Q T S Q J O
17 20 19 17 10 15
U F E O J N
21 6 5 15 10 14 18 1 4 9 19 8
R A -1 D I S +1H
-1 -1 +1 +1
12 20 18 5 1 4
L T R+1 E +1A +1 D
+1 +1 +1 C Z Q I T J
3 26 17 9 20 10
Elimination:
F B E S U M (a) 1st letter D wrong in option so gets eliminated.
6 2 5 19 21 13
(b) last letter H wrong in option so gets eliminated.
Elimination: (c) 1st letter B wrong in option so gets eliminated.
(a) last letter S wrong, so eliminated. Rearrangment (Cross/Forward)
(b) 1st letter D wrong, so eliminated. 30. Select the option that is related to the third term in the
(d) 1st letter D wrong, so eliminated. same way as the second term is related to the first term.
28. Select the option that is related to the third term in the ml fodYi dk p;u djsa tks rhljs in ls mlh çdkj lacaf/kr gS tSls
same way as the second term is related to the first term. nwljk in igys in ls lacaf/kr gSA
ml fodYi dk p;u djsa tks rhljs in ls mlh izdkj lacaf/kr gS tSls FOSTER : OFTSRE : : LAWYER : ?
nwljk in igys in ls lacaf/kr gSA (a) AYLWRE (b) ALYWRE
CARNATIC : SBDMZDJU : : FREEDOMS : ? (c) ALYREW (d) ALYRWE
(a) FSGVWTNP (b) FSGVXPNP Sol. (b) ALYWRE
(c) FSGUNPEV (d) GSFVWPEP Pattern: cross rearrangement
Sol. (a) FSGVWTNP Logic: 2 ×2 cross
Pattern : Fix addition opposite cross

171
Download Free PDFs & e-Books from Neon Classes App

6 15 19 20 5 18 7 18 5 1 20 12 25
F O S T E R G R E A T L Y

O F T S R E E R G A Y L T
15 6 20 19 18 5 5 18 7 1 25 12 20

12 1 23 25 5 18 13 9 24 20 21 18 5
L A W Y E R M I X T U R E

A L Y W R E X I M T E R U
1 12 25 23 18 5 24 9 13 20 5 18 21
Elimination:
(a) 2nd letter Y wrong so eliminated. 20 18 15 21 2 12 5
(c) last letter W wrong so eliminated. T R O U B L E
(d) 2nd last letter W wrong so eliminated.
31. Select the option that is related to the third letter cluster
in the same way as the second way as the second letter O R T U E L B
cluster is related to the first letter cluster. 15 18 20 21 5 12 2
Ml fodYi dk p;u djs]a tks rhljs v{kj&lewg ls Bhd mlh izdkj
lacaf/kr gS] ftl izdkj nwljk v{kj&lewg] igys v{kj&lewg ls lacfa /kr Elimination:
gSA (a) 1st letter B wrong so eliminated.
ZODIAC : DOZCAI :: MISTER : ? (b) last letter L wrong so eliminated.
(a) SFUTJN (b) SIMRET (c) First letter T wrong so eliminated.
(c) NITUFS (d) SMIRTE 34. Select the option that is related to the third term in the
Sol. (b) SIMRET same way as the second term is related to the first term.
Patterrn : cross rearrangement ml fodYi dk p;u djsa tks rhljs in ls mlh çdkj lacaf/kr gS tSls
25 15 4 9 1 3 nwljk in igys in ls lacaf/kr gSA
Z O D I A C CELEBRITY : ELECYYTIR :: MONUMENTS : ?
(a) UONMNSTNE (b) UNOMGSTEN
(c) UNOMNSTNE (d) UNOMNSNTE
D O Z C A I Sol. (c) UNOMNSTNE
Logic: 3 ×3 cross 4 15 25 3 1 9 Pattern: cross rearrangement
Logic: 4 ×4 cross, middle letter opposite letter
13 9 19 20 5 18 3 5 12 5 2 18 9 20 25
M I S T E R C E L E B R I T Y

E L E C Y Y T I R
S I M R E T
5 12 5 3 25 25 20 9 18
19 9 13 18 5 20
13 15 14 21 13 5 14 20 19
Elimination:
M O N U M E N T S
(a) Last letter N wrong so gets eliminated.
(c) 1st letter S wrong so gets eliminated.
(d) last letter E, wrong so gets eliminated. U N O M N S T N E
33. Select the option that is related to the fifth term in the 21 14 15 13 14 19 20 14 5
same way as the second term is related to the first term
Elimination:
and the fourth term is related to the third term.
(a) 2nd letter O, wrong so option eliminated.
ml fodYi dk p;u djsa tks ikaposa in ls mlh çdkj lacfa èkr gS tSls
(b) last letter N, wrong so option eliminated.
nwljk in igys in ls vkSj pkSFkk in rhljs in ls lacafèkr gSA (d) 2nd last letter T, wrong so option eliminated.
GREATLY : ERGAYLT :: MIXTURE : XIMTERU :: TROUBLE : ? 35.. Select the option that is related to the fifth term in the
(a) BORTUEL (b) ORTUEBL same way as the second term is related to the first term
(c) TUELBOR (d) ORTUELB and the fourth term is related to the third term.
Sol. (d) ORTUELB ml fodYi dk p;u djsa tks ikaposa in ls mlh çdkj lacfa èkr gS tSls
Pattern: cross rearrangement
nwljk in igys in ls vkSj pkSFkk in rhljs in ls lacafèkr gSA
Logic: 3 × 3 cross, middle as it is
SOCCER : OSFFRE :: ROLLER : OROORE :: TOPPER : ?
(a) TOERPK (b) OTSSRE
(c) OTPPRE (d) TOSSRE
Sol. (b) OTSSRE

172
Best App for Govt. Jobs : Neonclasses (Download Now)

Pattern: Rearrangement Mix cross (1) Series


Logic: 2×2 cross, addition (Middle letter +3) (2) Diff
19 15 3 3 5 18
(3) Sum
S O C C E R
+3 +3 (4) Pattern ()
(5) Vowel / Consonant
O S F F R E (d) does not follows pattern, So (d) is odd among all.
15 19 6 6 18 5 CONCEPT: DIFFERENCE BETWEEN ADJECENT LETTERS
18 15 12 12 5 18 02. Four letter-clusters have been given, out of which three
R O L L E R are alike in some manner and one is different. Select the
+3 +3
letter-cluster that is different.
pkj v{kj & lewg fn, x, gSa] ftuesa ls rhu ,d fuf’pr izdkj ls
O R O O R E leku gS] tcfd ,d muls vlaxr gSA ml vlaxr v{kj & lewg
15 18 15 15 18 5 p;u dhft,A
20 15 16 16 5 18 (a) SPMJ (b) CZWT (c) PMJG (d) GDAW
T O P P E R Sol. (d)
+3 +3
(a) 19 16 13 10
S P M J
O T S S R E
15 20 19 19 18 5 –3 –3 –3
Elimination:
3 26 23 20
(a) first letter T, wrong so eliminated. (b)
(c) Middle letter P, wrong so eliminated. C Z W T
(d) first letter T wrong so eliminated. –3 –3 –3
LETTER CLASSIFICATION (c) 16 13 10 7
CONCEPT: DIFFERENCE BETWEEN ADJECENT LETTERS IN P M J G
SERIES
01. Four letter-clusters have been given, out of which three –3 –3 –3
are alike in some manner and one is different. Select the
letter-cluster that is different. 7 4 1 23
(d)
pkj v{kj & lewg fn, x, gSa] ftuesa ls rhu ,d fuf’pr izdkj ls G D A W
leku gS] tcfd ,d muls vlaxr gSA ml vlaxr v{kj & lewg –3 –3 –4
p;u dhft,A
(a) PRNT (b) FHDJ (1) Series
(c) LNJP (d) GIEY (2) Diff ()
Sol. (d) GIEY (3) Sum
(a) 16 18 14 20 (4) Pattern
P R N T (5) Vowel / Consonant
a, b and c follows same pattern and d does not. So (d) is odd
+2 -4 +6 among all option.
6 8 4 10 CONCEPT: SUM OF LETTERS
(b) 03. Four letter-clusters have been given, out of which three
F H D J
are alike in some manner and one is different. Select the
+2 -4 +6 letter-cluster that is different.
pkj v{kj & lewg fn, x, gSa] ftuesa ls rhu ,d fuf’pr izdkj ls
(c) 12 14 10 16 leku gS] tcfd ,d muls vlaxr gSA ml vlaxr v{kj & lewg
L N J P p;u dhft,A
+2 -4 (a) ATH (b) RFB (c) DMI (d) CGP
+6
Sol. (a)
7 9 5 25 1 20 8
(d) (a)
G I E Y A T H
1 + 20 + 8 =29  26
+2 -4 +20
18 6 2
(b)
R F B
18 + 6 + 2 = 26

173
Download Free PDFs & e-Books from Neon Classes App

4 13 9 2019 18
(c) (a)
D M I T S R
4 + 13 + 9 = 26 Decrease order
3 7 16 25 24 23
(d) (b)
C G P Y X W
3 + 7 + 16 = 26 Decrease order
(1) Series
14 13 12
(2) Diff (c)
N M L
(3) Sum ()
Decrease order
(4) Pattern
6 7 8
(5) Vowel / Consonant (d)
F G H
b, c and d does not follows same pattern. So (a) is odd among Increase order
all option.
04. Three of the following four letter-clusters are alike in a (1) Series
certain way and one is different. Pick the odd one out. (2) Diff
fuEufyf[kr pkj v{kj lewgksa esa ls rhu ,d fuf'pr rjhds ls leku (3) Sum
gSa vkSj ,d vyx gSA fo"ke dks pqusAa (4) Pattern () : Decrease order
(a) BDGKP (b) SUXBG (5) Vowel / Consonant
(c) OQTXC (d) JLKRW
Sol. (d) a, b and c follows same pattern and d does not. So (d) is odd
among all option.
(a) 2 4 7 11 16
B D G K P 06. Four letter-clusters have been given, out of which three
are alike in some manner and one is different. Select the
+2 +3 +4 +5 letter-cluster that is different.
pkj v{kj & lewg fn, x, gSa] ftuesa ls rhu ,d fuf’pr izdkj ls
19 21 24 2 7 leku gS] tcfd ,d muls vlaxr gSA ml vlaxr v{kj & lewg
(b)
S U X B G p;u dhft,A
(a) TVQX(b) NPKR(c) BDYF (d) XZTB
+2 +3 +4 +5
Sol. (d)
(c) 15 17 30 24 3
(a) 20 22 17 24
O Q T X C T V Q X
+2 +3 +4 +5 –3 +2
5
10 12 11 18 23 14 16 11 18
(d) (b)
J L K R W N P K R
+2 –1 +7 +5 –3 +2
(1) Series ()
(c) 2 4 25 6
(2) Diff B D Y F
(3) Sum
(4) Pattern –3 +2
(5) Vowel / Consonant 24 26 20 2
a, b and d follows same pattern and d does not. So (c) is odd (d)
X Z T B
among all option.
CONCEPT: DECRESING ORDER –4 +2
05. Four letter-cluster's have been given, out of which three
(1) Series
are alike in some manner, while one is different. Select the
odd letter-cluster. (2) Diff ()
pkj v{kj&lewg fn, x, gSa] ftuesa ls rhu fdlh u fdlh :i esa ,d (3) Sum
tSls gSa] tcfd ,d fHkUu gSA fo"ke v{kj&lewg dk p;u djsAa (4) Pattern
(a) TSR (b) YXW (c) NML (d) FGH (5) Vowel / Consonant
Sol. (d) a, b and c follows same pattern and d does not. So (d) is odd
among all option.
07. Three of the following four letter-clusters are alike hi a
certain way and one is different. Pick the odd one out.

174
Best App for Govt. Jobs : Neonclasses (Download Now)

fuEufyf[kr pkj v{kj&lewgksa esa ls rhu ,d fuf'pr rjhds ls leku b, c and d follows same pattern and a does not. So (a) is odd
gSa vkSj ,d vyx gSA fo"ke dks pqusAa among all option.
(a) PQTXC (b) GHKOT OPPOSITE LETTER
(c) JKXRV (d) UVYCH 09. Three of the following four letter-clusters are alike in a
Sol. (c) certain way and one is different. Pick the odd one out.
fuEufyf[kr pkj v{kj lewgksa esa ls rhu ,d fuf'pr rjhds ls leku
(a) 16 17 20 24 3
P Q T X C gSa vkSj ,d vyx gSA fo"ke dks pqusAa
(a) GTXC (b) HSYB
+1 +3 +4 +5 (c) QJUF (d) DWKO
Sol. (d)
7 8 11 15 20 7 20 24 3
(b) (a)
G H K O T G T X C
+1 +3 +4 +5 Opp Opp

(c) 10 11 24 18 22
8 19 25 2
J K X R V (b)
H S Y B
+1 +13 -6 +4
Opp Opp
21 22 25 3 8 17 10 21 6
(d) (c)
U V Y C H Q J U F
+1 +3 +4 +5 Opp Opp
(1) Series
4 23 11 15
(2) Diff () (d)
D W K O
(3) Sum
(4) Pattern Opp Not Opp
(5) Vowel / Consonant (1) Series
a, b and d follows same pattern and d does not. So (c) is odd (2) Diff
among all option. (3) Sum
MULTIPLICATION PATTERN (4) Pattern () : Opposite
08. Four letter -cluster have been given, out of which three (5) Vowel / Consonant
are alike in some manner and one is different. Select the
odd letter – cluster a, b and c follows same pattern and d does not. So (d) is odd
pkj v{kj&lewg fn, x, gSa] ftuesa ls rhu fdlh u fdlh :i esa ,d among all option.
tSls gSa vkSj ,d fHkUu gSA fo"ke v{kj lewg dk p;u djsa DIVISIBILITY PATTERN
(a) GCV (b) BJT (c) DDP (d) FBL 10. Identify the odd one from the given groups of letters.
Sol. (a) fn, x, v{kjksa ds lewgksa esa ls fo"ke dks igpkfu,A
7 3 22 (a) FZJ (b) BVH (c) FPE (d) BDA
(a) Sol. (b)
G C V
Pattern : 
7 × 3 = 21  22
2nd letter code  1st letter code
2 10 20
(b) 2
B J T
 3rd letter code
2 × 10 = 20
4 4 16 (a) 6 26 10
(c) F Z J
D D P
4 × 4 = 16
6 2 12
(d) 2 22 8
F B L (b)
B V H
6 × 2 = 12
(1) Series
(2) Diff
(3) Sum
(4) Pattern () : Multiplication
(5) Vowel / Consonant

175
Download Free PDFs & e-Books from Neon Classes App

Sol. (d)
(c) 6 16 5
F P E 15 4 19
(a)
O D S
15 + 4 = 19
2 4 1 1 8 9
(d) (b)
B D A A H I
1+8=9
7 14 21
(c)
G N U
(1) Series
7 + 14 = 21
(2) Diff
4 15 21
(3) Sum (d)
D O U
(4) Pattern ()
4 + 15 = 19  21
(5) Vowel / Consonant
(1) Series
(b) does not follows pattern, so (b) is odd among all. (2) Diff
BASED ON REARRANGEMENT (3) Sum
11. Three of the following four letter-cluster pairs are alike in
(4) Pattern () : Sum Based
a certain way and one is different. Pick the odd one out.
fuEufyf[kr pkj v{kj&lewg tksM+s esa ls rhu ,d fuf'pr rjhds ls (5) Vowel / Consonant
leku gSa vkSj ,d vyx gSA fo"ke dks pqusaA a, b and c follows same patern and d does not. So (d) is odd
(a) PSRB : SRPB (b) TUKS : SKUT among all option.
(c) GMDI : IDMG (d) LKJR : RJKL BASED ON VOWEL/CONSONANT
Sol. (a) 13. Four letter-clusters have been given, out of which three
Pattern : - Rearrangement are alike in some manner and one is different. Select the
(a) 16 19 18 2 letter-cluster that is different.
P S R B pkj v{kj & lewg fn, x, gSa] ftuesa ls rhu ,d fuf’pr izdkj ls
leku gS] tcfd ,d muls vlaxr gSA ml vla xr v{kj & lewg
S R P B p;u dhft,A
(a) VAQ (b) XED (c) ZOV (d) WRU
(b) 20 21 11 19 Sol. (d)
T U K S Pattern : - [Middle letter is vowel]

S K U T (a) V A Q
(Vowel)

(c) 7 13 4 9 (b) X E D
G M D I (Vowel)
(c) Z O V
I D M G (Vowel)
(d) W R U
(d) 12 11 10 18
(Consonant)
L K J R

R J K L (1) Series
(2) Diff
(1) Series (3) Sum
(2) Diff (4) Pattern
(3) Sum (5) Vowel / Consonant ()
(4) Pattern ()
(5) Vowel / Consonant (d) does not follows pattern, so (d) is odd among all.

(a) does not follows pattern, so (a) is odd among all


Logical sequence of words
10. BASED ON LETTERS SUM 01. Select the correct option that indicates the arrangement of
12. Four letter-clusters have been given, out of which three the given words in a logical and meaningful order.
are alike in some manner and one is different. Select the ml lgh fodYi dk p;u djsa tks fn, x, 'kCnksa dh O;oLFkk dks
letter-cluster that is different. rkfdZd vkSj lkFkZd Øe esa bafxr djrk gS
pkj v{kj & lewg fn, x, gSa] ftuesa ls rhu ,d fuf’pr izdkj ls 1. Starfish/LVkjfQ’k
leku gS] tcfd ,d muls vlaxr gSA ml vlaxr v{kj & lewg 2. Blue whale/Cyw Ogsy
p;u dhft,A 3. Shark/’kkdZ
(a) ODS (b) AHI (c) GNU (d) DOU 4. Giant tortoise/fo’kkydk; dNqvk

176
Best App for Govt. Jobs : Neonclasses (Download Now)

5. Penguin/isx a qbu 4. Script – First of all the story of a movie or video is written in
(a) 1, 5, 4, 2, 3 (b) 1, 5, 3, 2, 4 script form.
(c) 1, 5, 4, 3, 2 (d) 4, 5, 1, 3, 2 2. Rehearsal – Rehearsal are done based on the script written.
Sol. (c) 1. Recording – Recording is done after the rehearsal.
The organisms have been arranged according to their size. 5. Video Editing – Then video editing of the recording.
Hence (c) has correct logical order. 3. Viewers – Finaly it is ready for viewers.
02. Arrange the following words in a logical and meaningful Hence, the correct is in option b.
order.
fuEufyf[kr 'kCnksa dks rkfdZd vkSj vFkZiw.kZ Øe esa O;ofLFkr dhft,A 05. Select the correct option that indicates the arrangement of
1. Hexagon/"kVHkqt the given words in a logical and meaningful order.
2. Nonagon/uoHkqt ml lgh fodYi dk p;u djsa tks fn, x, 'kCnksa dh O;oLFkk dks
3. Pentagon/iapHkqt rkfdZd vkSj lkFkZd Øe esa bafxr djrk gS
4. Heptagon/lIrHkqt 1. Treatment/mipkj
5. Octagon/v"VHkqt 2. Recovery/fjdojh
(a) 1-3-4-5-2 (b) 4-3-1-2-5 3. Virus/ok;jl
(c) 1-4-5-2-3 (d) 3-1-4-5-2 4. Symptoms/y{k.k
Sol. (d) 5. Infection/laØe.k
Hexagon – six sides (a) 3, 5, 4, 2, 1 (b) 3, 5, 4, 1, 2
Nonagon – nine sides (c) 2, 5, 1, 4, 5 (d) 4, 5, 3, 1, 2
Pentagon – five sides Sol. (b)
Heptagon – seven sides The process of virus infected to a person and finally getting
Octagon – eight sides recovery is as shown.
Arranging the following polygons in ascending order. Hence 3. Virus
(d) has correct logical order. 5. infection
03. Select the correct option that indicates the arrangement of 4. Symptoms
the given course of actions in a logical and meaningful 1. Treatment
order. 2. Recovery
ml lgh fodYi dk p;u djsa tks fn, x, dk;ksaZ dh O;oLFkk dks Hence, Option b is correct.
rkfdZd vkSj lkFkZd Øe esa bafxr djrk gSA 06. Select the correct option that indicates the arrangement of
1. Open email account/bZesy [kkrk [kksys the given words in a logical and meaningful order.
2. Compose email/bZesy fy[ks ml lgh fodYi dk p;u djsa tks fn, x, 'kCnksa dh O;oLFkk dks
3. Start computer/dEI;qVj izkjEHk djsa rkfdZd vkSj lkFkZd Øe esa bafxr djrk gS
4. Enter email address/bZesy irk ntZ djsa 1. Chapterlist/v/;k; lwph
5. Write the content/lkexzh fy[ksa 2. Bibliography/xzaFk lwph
6. Send the mail/esy Hksts 3. Preface/izLrkouk
(a) 3, 1, 2, 4, 5, 6 (b) 2, 5, 1, 3, 4, 6 4. Cover page/vkoj.k i`"B
(c) 3, 2, 6, 1, 5, 4 (d) 3, 1, 2, 6, 5, 4 5. Chapters/v/;k;
Sol. (a) (a) 2, 5, 1, 4, 3 (b) 4, 3, 1, 5, 2
The mailing process from the given sentences are arranged in (c) 4, 5, 3, 1, 2 (d) 3, 5, 4, 2, 1
order is as shown below : The words have been arranged
3. Start computer according to the book format.
1. Open email account 4. Cover page – it is one of the first pages you encounter after you
2. Compose email open the book where title is written.
4. Enter email address 3. Preface – After cover page a preface page is written from the
5. Write the content point of view of the book author.
6. Send the mail 1. Chapter list – After preface page the contents list of a book show
Hence (a) has correct logical order. list of different chapters and sections.
04. Select the correct option that indicates the arrangement of 5. Chapters – After the content list pof chapters comes details of
the given words in a logical and meaningful order. chapter.
ml lgh fodYi dk p;u djsa tks fn, x, 'kCnksa dh O;oLFkk dks 2. Bibliography – Last page of any book where written sources of
rkfdZd vkSj lkFkZd Øe esa bafxr djrk gS information on a book called bibliography.
1. Recording/fjdkWfMZax Hence, option b have correct order.
2. Rehearsal/iwokZH;kl
3. Viewers/n’kZd 07. Select the correct alternative to indicate the arrangement
4. Script/iVdFkk of the following words in a logical and meaning fill order.
5. Video Editing/ohfM;ks laiknu
rkfdZd vkSj vFkZ Hkj.k Øe esa fuEufyf[kr 'kCnksa dh O;oLFkk dks bafxr
(a) 4, 1, 2, 3, 5 (b) 4, 2, 1, 5, 3
djus ds fy, lgh fodYi dk p;u djsAa
(c) 2, 4, 1, 5, 3 (d) 4, 5, 2, 1, 3 1. Chapter/ vè;k;
Sol. (b) 2. Word/'kCn
The correct meaningful order – 3. Letter/ v{kj

177
Download Free PDFs & e-Books from Neon Classes App

4. Phrase/ okD;ka'k 4. Uttar Pradesh/mÙkj izns’k


5. Paragraph/ ifjPNsn 5. Himachal Pradesh/fgekpy izns’k
6. Sentence/ okD; (a) 4-2-1-3-5 (b) 4-1-2-5-3
(a) 3, 2, 6, 4, 5, 1 (b) 3, 2, 4, 6, 1, 5 (c) 4-2-1-5-3 (d) 4-1-2-3-5
(c) 2, 3, 4, 1, 6, 5 (d) 3, 2, 4, 6, 5, 1 Sol. (c)
Sol. (d) Arranging the states according to the opulation,
The pattern followed here is: Uttar Prades has the hightest population.
A letter is a part of a word; a word is a part of a phrase; a Utter Prades is followed byu Madhya Pradesh.
phrase is a part of sentence; a sentence is a part of a Madhya Pradesh is followed by Andhra Pradesh.
paragraph; a paragraph is a part of a chapter. Andhra Pradesh is followed by Himaxhal Pradesh.
Letter – Word – Phrase – Sentence – Paragraph – Chapter Himachal Pradesh is followed buy Arunachal Pradesh.
Hence, the option d has correct answer. The correct logical order is,
08. Select the correct option that indicates the arrangement of Uttar Pradesh – Madhya Pradesh – Andhra Pradesh – Himachal
the given words in a logical and meaningful order. Pradesh – Arunachal Pradesh
ml lgh fodYi dk p;u djsa tks fn, x, 'kCnksa dh O;oLFkk dks
rkfdZd vkSj lkFkZd Øe esa bafxr djrk gS 11. Arrange the following words in a logical and meaningful
1. Core/dksj order.
2. Atmosphere/ok;qe.My fuEufyf[kr 'kCnksa dks rkfdZd vkSj vFkZiw.kZ Øe esa O;ofLFkr dhft,A
3. Universe/czãk.M 1. Egypt/felz
4. Surface/Hkwry 2. Africa/vÝhdk
5. Galaxy/ vkdk'kxaxk 3. Great Pyramid./egku fijkfeM
(a) 1, 4, 2, 5, 3 (b) 4, 3, 5, 2, 1 4. World/foÜo
(c) 1, 4, 2, 3, 5 (d) 1, 3, 4, 5, 2 5. Giza/xht+k
Sol. (a) (a) 5, 3, 1,2, 4 (b) 5, 3, 1,4, 2
The increasing order of universe from core of a planet is (c) 3, 5, 2, 1, 4 (d) 3, 5, 1,2, 4
shown. Sol. (d)
1. Core Great Pyramid is located in Giza.
4. Surface Giza is located in Egypt.
2. Atmosphere Egypt is located in Africa.
5. Galaxy Africa is located in World.
3. Universe So, logical order is in option d.
Hence, Option a has correct logical order. 12. Arrange the following words in a logical and meaningful
09. Select the correct option that indicates the arrangement order.
of the following words in a logical and meaningful order. fuEufyf[kr 'kCnksa dks rkfdZd vkSj vFkZiw.kZ Øe esa O;ofLFkr dhft,A
lgh fodYi dk p;u djsa tks fuEufyf[kr 'kCnksa dh O;oLFkk dks 1. Rajasthan/ jktLFkku
rkfdZd vkSj lkFkZd Øe esa bafxr djrk gSA 2. India/ Hkkjr
1. Satellite/ mixzg 3. Jaipur/ t;iqj
2. Galaxy/ vkdk'kxaxk 4. North India/ mÙkj Hkkjr
3. Universe/ czãkaM 5. Asia/,f'k;k
4. Planet/ xzg (a) 3, 1, 4, 2, 5 (b) 1, 3, 2, 4, 5
5. Star/ rkjs (c) 3, 1, 2, 4, 5 (d) 1, 3, 4, 2, 5
(a) 1, 3, 5, 4, 2 Sol. (a)
(b) 3, 2, 5, 4, 1 The logical order would be,
(c) 1, 2, 3, 4, 5 State Capital > State > State Location > Country > Continent
(d) 3, 4, 5, 1, 2 Jaipur > Rajasthan > North India > India > Asia
Sol. (b) 13. Arrange the following words in a logical and meaningful
First comes Universe, which is everything. order.
Universe consists of galaxies. fuEufyf[kr 'kCnksa dks rkfdZd vkSj lkFkZd Øe esa O;ofLFkr djsAa
Galaxies consists of stars. 1. Colonel/duZy
Planets are followed by stars. 2. Field Marshal/QhYM ek’kZy
Satellites are followed by planets. 3. Brigadier/fczxsfM;j
The correct order is, 4. Lieutenant/ysf¶VusV a
Universe – Galaxy – Star – Planet – Satellite 5. General/tujy
10. Arrange the following words in a logical and meaningful (a) 2, 5, 3, 1, 4 (b) 3, 5, 2, 4, 1
order (according to population). (c) 1, 2, 3, 5, 4 (d) 2, 5, 1, 4, 3
fuEufyf[kr 'kCnksa dks rkfdZd vkSj lkFkZd Øe esa O;ofLFkr djsa Sol. (a)
¼tula[;k ds vuqlkj½A There are the Ranks in an Army arragned in descending order.
1. Andhra Pradesh/vka/kz izns’k 2. Field Marshal
2. Madhya Pradesh/e/; izns’k 5. General
3. Arunachal Pradesh/v#.kkpy izns’k 3. Brigadier

178
Best App for Govt. Jobs : Neonclasses (Download Now)

1. Colonel 17. Select the correct option that indicates the arrangement of
4. Lieutenant the following words in a logical and meaningful order.
Hence, option a have correct order. lgh fodYi dk p;u djsa tks fuEufyf[kr 'kCnksa ds rkfdZd vkSj
14. Arrange the following words in a logical and meaningful lkFkZd Øe esa O;oLFkk dks bafxr djrk gSA
order. 1. Plant/ikS/kk
fuEufyf[kr 'kCnksa dks rkfdZd vkSj vFkZiw.kZ Øe esa O;ofLFkr dhft,A 2. Germination/vadqj.k
1. Nose/ukd 3. Seed/cht
2. Forehead/ekFkk 4. Fruit/Qy
3. Chin/BksM+h 5. Flower/Qwy
4. Waist/dej (a) 3, 1, 2, 5, 4 (b) 3, 4, 2, 1, 5
5. Chest/Nkrh (c) 3, 2, 1, 5, 4 (d) 4, 2, 3, 5, 1
(a) 2, 1, 3, 5, 4 (b) 2, 3, 1, 4, 5 Sol. (c)
(c) 3, 2, 4, 2, 1 (d) 2, 4, 1, 5, 3 The words given presents the germination process of fruit
Sol. (a) from seed step by step.
Part of a body from head to toe is arranged serially. Seed – Germination – Plant – Flower - Fruit
So the correct order is;
2. Forehead Machine Input Output
1. Nose in the questions realted to this topic, we are given an
3. Chin input and after performing some operations on the given
5. Chest input, we will otain the output. This output is obtained
4. Waist through number of steps following some prefixed
15. Select the correct option that indicates the arrangements patterns or rules.
of the following words in a logical meaningful order. While solving such questions we have to understand the
ml fodYi dk p;u djs]a tks fn, x, 'kCnksa ds rkfdZd ,oa vFkZiw.kZ pattern of process through which we are getting different
Øe foU;kl dks n’kkZrk gSA outputs in different steps.
1. Tissue/mÙkd bl fo"k; ls lcaf/kr iz’uksa es]a gesa ,d buiqV fn;k tkrk gS vkSj fn,
2. Cell/dksf’kdk x, buiqV ij dqN vkWijs’ku djus ds ckn] ge vkmViqV izkIr djsaxsA
3. Organ/vax ;g vkmViqV dqN iqoZ fu/kkZfjr iSVuZ ;k fu;eksa dk ikyu djrs gq,
4. Human Body/ ekuo 'kjhj dbZ pj.kksa esa izkIr fd;k tkrk gSA
5. Organ System/vax ra= ,sls iz’uksa dks gy djrs le; gesa ml iSVuZ ;k izfØ;k dks le>uk
(a) 2, 3, 1, 5, 4 (b) 2, 1, 3, 5, 4 gksxk ftlds ek/;e ls gesa fofHkUu pj.kksa esa vyx&vyx vkmViqV
(c) 2, 1, 5, 4, 3 (d) 1, 2, 3, 5, 4 fey jgs gSA
Sol. (b) When only letters given: Shifting From one side
The logical order is , ¼tc dsoy v{kj fn, tk;s½
2. Cell – Cell is the smallest unit of life and the basic building Direction (1-5): Study the following information carefully
block of the human body. and answer the question given below:
1. Tissue – A tissue is a group of cells. A word arrangement machine when given an input line of
3. Organ – Organs are made of tissues words rearranges it in every step following a certain rule.
5. Organ System, - organ system are made up of organs. Following is an illustration of an input line of words and
4. Human Body – Human body is made up of organ systems. various steps of rearrangement:
Hence, The logical order is 2,1,3,5,4 fuEufyf[kr tkudkjh dk /;kuiwoZd v/;;u dj fn, iz’uksa ds mÙkj
16. Select the correct alternative that shows the logical and nhft,A
meaningful sequence of the given words. 'kCnksa dks O;ofLFkr djus okyh ,d e’khu 'kCnksa dh buiqV ykbu fn,
ml lgh fodYi dk p;u dj]sa tks fn, x, 'kCnksa ds rkfdZd vkSj tkus ij] izR;sd pj.k esa ,d fo’ks"k fu;e dk ikyu djrs gq,] mUgsa
lkFkdZ Øe dks n'kkrkZ gS iqu% O;ofLFkr djrh gSA buiqV vkSj iquO;ZoLFkk dk mnkgj.k fn;k
I. Flowers/Qwy x;k gSA
II. sprout/vadqj input % gone are take enough brought station
III. fruit/Qy Step I: take gone are enough brought station
iv. Plant/ikS/kk Step II: take are gone enough brought station
v. Seed/cht Step III: take are station gone enough brought
(a) ii, v, iv, i, iii (b) v, ii, iv, i, iii Step IV: take are station brought gone enough
(c) ii, i, iii, iv, v (d) v, iv, i, iii, ii vkSj pj.k IV bl buiqV ds fy, vafre pj.k gSA
Sol. (b) mijksDr pj.kkssa esa fn, x, fu;eksa ds vuqlkj] fuEufyf[kr izR;sd iz’u
The process of germination of fruit from seed is as shown : esa fn, x, buiqV ds fy, mi;qDr pj.k dk irk yxk,aA
v. Seed is sown in soil. buiqV % car on star quick demand fat.
ii. After few days seed develop as sprout. 1. How many steps will be required to complete the given
iv. Sprout transforms to plant. arrangement?
i. Plant blossom with flowers. nh xbZ O;oLFkk dks iwjk djus ds fy, fdrus pj.kksa dh vko’;drk
iii. Flower then coverts into fruit. gksxh\
Hence, the correct answer is v, ii, iv, i, iii. (a) 4 (b) 3

179
Download Free PDFs & e-Books from Neon Classes App

(c) 7 (d) 6 Step IV : data 95 for 38 sale 18 23 then


(e) buesa ls dksbZ ugha Step V : data 95 for 38 sale 23 18 then
Sol. (b) 3 Step VI : data 95 for 38 sale 23 then 18
2. Which step number would be the following output? Input for the questions / iz’uksa ds fy, buiqV
fuEufyf[kr esa vkmViqV fdl pj.k la[;k esa gksxk\ 6. input : year 39 stake 47 house full 94 55. How many steps
star car quick on demand fat will be required to complete rearrangement?
(a) Step II (b) Step III bl iquZO;oLFkk dks lekIr djus ds fy, fdrus pj.k cusxs\
(c) Step IV (d) Step V (a) Five (b) Six
(e) None of these (c) Four (d) Seven
Sol. (a) Step II (e) None of these
3. What is the position of “quick” in step III? 7. Step II of on input
pj.k III esa “quick” dh fLFkfr D;k gS\ buiqV dk nwljk pj.k
(a) Second form the left ¼ck,¡ vksj ls nwljk½ Car 83 lost ever 32 46 74 now.
(b) Fourth form the right ¼ck,¡ vksj ls pkSFkk½ How many more steps will be required to complete this
(c) Fourth form the right ¼nk,¡ vksj ls pkSFkk½ arrangement?
(d) Sixth from the right ¼nk,¡ vksj ls NBk½ bl O;oLFkk dks lekIr@iwjk djus ds fy, vkSj fdrus pj.k yxsxs\
(e) None of these ¼buesa ls dksbZ ugha½ (a) Three (b) Four
4. How many words are there between “star” and “demand” (c) Five (d) Six
in step I ? (e) None of these
pj.k 1 esa “star” vkSj “demand” ds chp fdrus 'kCn gSa\ 8. Step III of an input is and 79 code 27 18 new goal 37 which
(a) 1 (b) 2 is following will definitely be the input?
(c) 3 (d) 4 dkSulk fuEufyf[kr esa ls fuf’pr :Ik ls buiqV gS\
(E) None of these ¼buesa ls dksbZ ugha½ (a) Code and 79 27 18 new goal 34
5. Which of the following would be third to the left of second (b) Code 27 18 new goal 34 and 79
to the right of “quick” in step III ? (c) Code 27 and 18 79 new goal 34
fuEufyf[kr esa ls dkSu pj.k III esa “quick” ds nk,¡ nwljs ds ck,¡ ls (d) Cannot be determined
rhljs LFkku ij gksxk\ (e) None of these
9. Step II of an input is high 69 store pay 36 43 15 there.
(a) Car (b) Quick
Which of the following will be step VI?
(c) Star (d) Fat
(e) None of these ¼buesa ls dksbZ ugha½
buesa ls dkSulk pj.k VI gksxk\
(a) high 69 pay 43 store 36 there 15
Sol : (1-5)From the last step it is clear that when we arrange the
(b) high 69 pay 43 store 36 15 there
words of the input as in English dictionary order, then
(c) high 69 pay 36 43 store 15 there
arrangement starts with the last word, then the first word,
(d) there will be no such step
then second last word and so on.
(e) none of the above
vafre pj.k ls ;g Li"V gS fd tc ge buiqV ds 'kCnksa dks vxzt as h
10. input : train more 29 53 fast gear 37 84
'kCndks’k Øe esa O;ofLFr djrs gS] rks O;oLFkk vafre 'kCn ls 'kq:
which of the following steps will be the last but one?
gksrh gS] fQj igyk 'kCn] fQj nwljk 'kCn vkSj blh rjgA
buesa ls dkSulk pj.k vafre ls ,d igyk gS\
buiqV % car on star quick demand fat
(a) IX (b) VII
pj.k I: star car on quick demand fat (c) VIII (d) VI
pj.k II: star car quick on demand fat (e) None of these
pj.k III: star car quick demand on fat Note:- words are arranged in alphabetically ascending order
1.(B) 3 steps and digits are arranged in desending order.
2. (A) Step II 'kCn dks o.kZekyk ds igys v{kj ds vuqlkj c<+rs Øe esa fn[kk;k x;k
3. (C) 4th from the right / nk,¡ vksj ls pkSFkk gS vkSj vadks dks ?kVrs Øe esa O;ofLFkr fd;k x;k gSA
4. (C) 3 Sol: 6. (b)
5. (A) Car Input : Year 39 stake 47 house full 94 55
When numbers and letters are together: Shifting from one Step I: full year 39 stake 47 house 94 55
side Step II: full 94 year 39 stake 47 house 55
(6-10) : Step III: full 94 house year 39 stake 47 55
A string of numbers is given as input the further steps are Step IV: full 94 house 55 year 39 stake 47
given by applying certain logic. Each step is a resultant of Step V: full 94 house 55 stake year 39 47
previous step only. study the following information Step VI: full 94 house 55 stake 47 year 39
carefully and answer the given question.
Step Year 39 Stake 47 house full 94 55
la[;k ls lacaf/kr ,d buiqV fn;k gS vkSj dqN ykWftd ds lkFk pj.k VI A VIB V VI III I II IV
cuk, gSA izR;sd pj.k fiNys pj.k ds ifj.kke ls cuk gSA lwpuk dks
7. (b) Step II is given i.e.
/;ku ls i<+dj uhps fn;s x;s iz'uksa ds mÙkj nhft,A
Input : sale date 18 23 for 95 then 38 Car83
/(IVLostA) ever 32 46 74 now
III VIA V IV VI b2  4ac
Step I : data sale 18 23 for 95 then 38
Step II : data 95 sale 18 23 for then 38 Total = 6
Step III : data 95 for sale 18 23 then 38 Already done = 2

180
Best App for Govt. Jobs : Neonclasses (Download Now)

four more step will be required to complete this arrangement fuEu vkmViqV dk pj.k dkSulk gS & ’37 bake 78 book 33 blast
8. (d) cannot be determined 23 34 chicken chariot spider 24?
Step III is given :- (a) Step II (b) Step III (c) Step IV
and 79 code / 27 18 new good 37 (d) Step V (e) None
if any previous step will be asked always can’t determine 15. In which of the following step ‘23 chicken chariot’ appear
9. (e) Step – II in same order?
High69 fuEufyf[kr esa ls fdl pj.k esa ’23 chicken chariot’ mlh Øe esa
/ (IVStore
 A)
Pay 36 43 15 there
III IV B IV (V  A) V fn[kkà nsrs gSa\
Step VI :- there will be no such type of step because total five (a) Step II (b) Step III (c) Step IV
steps are required to complete this re-arrangement (d) Step V (e) None
10. (e) Sol. (11-15)
train more 29 53 fast gear 37 84 Words are arranged based on number of letters in ascending
VI A V VIB I V I III VI II order if same number of letters are there then they are
Total six steps will be required to complete this re- arranged in dictionary order Numbers are arranged
arrangement but in given question i.e. asked last but one i.e. – alternately first odd then even in descending order
(V) five 'kCn¨a d¨ vkj¨gh Øe esa v{kj¨a dh la[;k ds vk/kkj ij O;ofLFkr
(11-15) : fd;k tkrk gS ;fn leku la[;k esa i= g¨rs gSa r¨ mUgsa 'kCnd¨'k Øe
A string of numbers is given as input the further steps are esa O;ofLFkr fd;k tkrk gS uacj¨a d¨ igys fo"ke rjhds ls O;ofLFkr
given by applying certain logic. Each step is a resultant of fd;k tkrk gS] fQj le voj¨gh Øe esa
previous step only study the following information carefully Input: 23 book 78 bake blast 34 37 chicken chariot 24 33
and answer the given question. spider
la[;k ls lacaf/kr ,d buiqV fn;k gS vkSj dqN ykWftd ds lkFk pj.k Step I: 37 bake 23 book 78 blast 34 chicken chariot 24 33
cuk, gSA izR;sd pj.k fiNys pj.k ds ifj.kke ls cuk gSA lwpuk dks spider
/;ku ls i<+dj uhps fn;s x;s iz'uksa ds mÙkj nhft,A Step II: 37 bake 78 book 23 blast 34 chicken chariot 24 33
Input: 24 plate 79 paste 27 36 boundary candle 44 33 spider
pigeon sparrow Step III: 37 bake 78 book 33 blast 23 34 chicken chariot 24
Step I: 79 paste 24 plate 27 36 boundary candle 44 33 pigeon spider
sparrow Step IV: 37 bake 78 book 33 blast 34 spider 23 chicken chariot
Step II: 79 paste 44 plate 24 27 36 boundary candle 33 pigeon 24
sparrow Step V: 37 bake 78 book 33 blast 34 spider 23 chariot chicken
Step III: 79 paste 44 plate 33 candle 24 27 36 boundary pigeon 24
sparrow Step VI: 37 bake 78 book 33 blast 34 spider 23 chariot 24
Step IV: 79 paste 44 plate 33 candle 36 pigeon 24 27 boundary chicken
sparrow 11. (c) VI steps
Step V: 79 paste 44 plate 33 candle 36 pigeon 27 sparrow 24 12. (b) blast
boundary 13. (a)
Step V is the last step In the same manner Blast is related to 34
Input: 23 book 78 bake blast 34 37 chicken chariot 24 33 14. (e)
spider none
11. What is the last step for above Input? 15. (c) Step IV
buiqV dk vafre pj.k D;k gksxk\ When numbers and letters are together: Shifting from both
(a) Step IV (b) Step V side
(c) Step VI (d) Step VII (16-20) :
(e) None A string of numbers is given as input the further steps are
12. Which of the following element would at 6th position from given by applying certain logic. Each step is a resultant of
left end in step II? previous step only study the following information
fuEufyf[kr esa ls d©u lk rRo pj.k II esa ck,a N¨j ls 6 osa LFkku carefully and answer the given question.
ij g¨xk\ la[;k ls lacaf/kr ,d buiqV fn;k gS vkSj dqN ykWftd ds lkFk pj.k
(a) 23 (b) blast cuk, gSA izR;sd pj.k fiNys pj.k ds ifj.kke ls cuk gSA lwpuk dks
(c) 34 (d) Chicken /;ku ls i<+dj uhps fn;s x;s iz'uksa ds mÙkj nhft,A
(e) Chariot Input : 11 day 34 night 93 place 27 easy 44 joy
13. In Step IV if ‘bake’ is related to ‘78’, ‘book’ is related to ‘33’ Step I : 93 11 day 34 night place 27 easy 44 joy
then ‘blast’ is related to? Step II : 93 11 34 night place 27 easy 44 joy day
pj.k IV esa ;fn ‘bake’ *78* ls lacaf/kr gS] ‘book’ ‘33’ ls lacaf/kr gS Step III : 93 44 11 34 night place 27 easy joy day
r¨ ‘blast’ fdlls lacaf/kr gS\ Step IV : 93 44 11 34 night place 27 joy day easy
(a) 34 (b) spider Step V : 93 44 34 11 night place 27 joy day easy
(c) 23 (d) 24 (e) None Step VI : 93 44 34 11 night place 27 day easy joy
14. Which step number would be the following output – ‘37 Step VII : 93 44 34 27 11 night place day easy joy
bake 78 book 33 blast 23 34 chicken chariot spider 24’? Step VIII : 93 44 34 27 11 place day easy joy night
Step IX : 93 44 34 37 11 day easy joy night place
input : class 25 war 15 race 73 heap 58 just 88 take 38

181
Download Free PDFs & e-Books from Neon Classes App

16. What is the position of “war” in step VII? la[;k ls lacaf/kr ,d buiqV fn;k gS vkSj dqN ykWftd ds lkFk pj.k cuk,
pj.k&7 esa “war” dh fLFkfr dkSulh gS\ gSA izR;sd pj.k fiNys pj.k ds ifj.kke ls cuk gSA lwpuk dks /;ku
(a) Seventh from the right end nk¡, ls 7ok¡ ls i<+dj uhps fn;s x;s iz'uksa ds mÙkj nhft,A
(b) Eight from the right end nk,¡ ls 8ok¡ input 44 38 24 55 16 14 85
(c) Fifth from the left end ck,¡ ls 5ok¡ Step I 8 2 6 1 7 5 4
(d) Fifth from the right end nk,¡ ls 5ok¡ Step II 60 0 32 -3 45 21 12
(e) Eight from the left end ck,¡ ls 8ok¡ Step III 6 0 5 -3 9 3 3
17. Which of the following is the ninth from the right in Step Step IV 7 4 14 13 34 39 52
VI? Step V 15 12 22 21 42 47 60
fuEu esa ls dkSulk pj.k VI esa nk,¡ ls 9ok¡ LFkku gS\ Step VI 6 3 4 3 6 2 6
(a) race (b) 38 (c) war (d) 58
(e) 25 Now, answer the following questions based on the following
18. What is the position of “15” in the step IX? input:
^^15** dh pj.k&9 es D;k fLFkfr gS\ uhps ,d buiqV ns j[kk gS mlds vk/kkj ij fuEu iz’uksa ds mŸkj nhft,
(a) Seventh from the left end ck,¡ ls 7ok¡ input 52 36 12 22 14 16 32
(b) Eighth from the left end ck,¡ ls 8ok¡
(c) Fifth from the right end nk,¡ ls 5ok¡ 21. How many steps will be required to complete the
(d) Seventh from the right end nk,¡ ls 7ok¡ arrangement?
(e) Eighth from the right end nk,¡ ls 8ok¡ dqy fdrus pj.k yxsxs bl O;oLFkk dks lekIr djus esa\
19. How many steps are required to complete this (a) 5 (b) 3
arrangement? (c) 6 (d) 4
bl O;oLFkk dks iwjk djus esa dqy fdrus pj.k yxsxs\ (e) None of these
(a) Eleven (b) Twelve (c) Ten 22. Which number comes in between 1st and 3rd number from
(d) Nine (e) None of these left of 6th step of input?
20. Which of the following represents the step – X? pj.k&6 esa dkSulh la[;k ck,¡ ls igys o rhljs ds chp esa vk,xh\
fuEufyf[kr esa ls 10ok¡ LFkku dkSulk gS\ (a) 18 (b) 14
(a) 88 73 58 38 25 war 15 race take class help just (c) 8 (d) 45
(b) 88 73 58 38 25 15 class heap just race take war (e) 6
(c) 88 73 58 38 25 15 war class heap just race take 23. Which number comes after “19” from left in step 4th?
(d) 88 73 58 38 25 15 war take class heap just race dkSulh la[;k pj.k&4 esa ck,¡ ls ^^19** ds ckn vk,xh\
(e) there will be no such step. (a) 28 (b) 14 (c) 9
Sol : (16-20) (d) 5 (e) 6
input : class 25 war 15 race 73 heap 58 just 88 take 38 24. Which of the following would definitely be the input step
I. 88 class 25 war 15 race 73 heap 58 just take 38 II of an input is?
II. 88 25 war 15 race 73 heap 58 just take 38 class pj.k&2 esa fuEu esa ls dkSulk fuf’pr :i ls buiqV gksxk\
III. 88 73 25 war 15 race heap 58 just take 38 class (a) 45 77 12 9 3 21
IV. 88 73 25 war 15 race 58 just take 38 class heap (b) 10 5 21 12 45 52
V. 88 73 58 25 war 15 race just take 38 class heap (c) 17 18 27 36 53 6
VI. 88 73 58 25 war 15 race take 38 class heap just (d) None of these
VII. 88 73 58 38 25 war 15 race take class heap just (e) cannot be determined
VIII. 88 73 58 38 25 war 15 take class heap just race 25. What is the sum of numbers which are 3rd from the left
IX. 88 73 58 38 25 15 war take class heap just race end in step 2nd and 3rd from the right end in step 5th?
X. 88 73 58 38 25 15 war class heap just race take pj.k&2 esa ck,¡ fdukjs ls rhljh o pj.k&5 esa nk,¡ fdukjs ls rhljh
XI. 88 73 58 38 25 15 class heap just race take war la[;k dk ;ksx fdruk gksxk\
16. (a) Sixth from left end and VII from right end word ‘war’ (a) 41 (b) 36 (c) 63
VIIth from right end (d) 40 (e) 45
17. (e) Step VI – 9th from right is 25 Sol. (21-25)
18. (d) Word 15 is coming VIth from left end and 7th from right input 52 36 12 22 14 16 32
end
Step I 7 9 3 4 5 7 5
19. (a) “Eleven” steps will required to complete this re-
arrangement Step II 45 77 5 12 21 45 21
20. (c) Step – X Step III 9 5 5 3 3 9 3
88 73 58 38 25 15 war class heap just race take Step IV 10 9 14 19 28 45 52
Number with logic Step V 18 17 22 27 36 53 60
(21-25) : A string of numbers is given as input the further
Step VI 9 8 4 9 9 8 6
steps are given by applying certain logic. Each step is a
resultant of previous step only study the following 21. (c) Explanation –
information carefully and answer the given question. Step I – Sum of digit i.e. 5+2=7
Step II – Square of step I and minus 4 in each no. i.e. (72) = 49-4
= 45
Step III Sum of digit which we get in step II

182
Best App for Govt. Jobs : Neonclasses (Download Now)

Step IV – (1)2 + no. in III step, (2)2 + no., 32+no. 42+no. 29. Which of the following number is third to the left of the
Step V – Add -8 in each no. of previous step number which is ninth from the left end in last step?
Step VI - Sum of digit in each number of previous step. fuEufyf[kr esa ls d©u lh la[;k vafre pj.k esa ck,a N¨j ls u©osa
22. (c) Between Ist and IIIrd number in step 6th from left end uacj ds cka;s rhljs LFkku ij gS\
i.e. 8 (a) 17 (b) 90
23. (a) “28” is that number which comes after 19 in step IV (c) 94 (d) 87
from left end? (e) None of these
24. (e) If any step is given and ask for previous step / input 30. If the odd number is increased by 13 and even number is
then always ans will be cannot be determined decreased by 15 in third step then what is the product of
25. (a) Step II – left – IIIrd = 5 lowest and fourth lowest number of the newly formed
Step V - Right – IIIrd = 36 series?
5+36=41 ;fn fo"ke la[;k esa 13 dh o`f) gqà gS vkSj le la[;k esa 15 dh deh
vkà gS] r¨ pj.k&3 esa uoxfBr J`[a kyk ds lcls de vkSj p©Fks uacj
Number with logic from both Sides dh la[;k dk xq.kuQy D;k gS\
(26-30) : (a) 872 (b) 972
A string of numbers is given as input the further steps are (c) 652 (d) 986
given by applying certain logic. Each step is a resultant of (e) 882
previous step only study the following information Sol: (26-30):
carefully and answer the given question. For step -1 lowest number is increased by 2 and it is arranged
la[;k ls lacaf/kr ,d buiqV fn;k gS vkSj dqN ykWftd ds lkFk pj.k at left end and also highest number is decreased by 2 and it is
cuk, gSA izR;sd pj.k fiNys pj.k ds ifj.kke ls cuk gSA lwpuk dks arranged at right end For step -2, Second lowest number is
/;ku ls i<+dj uhps fn;s x;s iz'uksa ds mÙkj nhft,A increased by 2 and it is arranged at left end and also Second
Input: 57 18 25 88 96 34 19 48 77 66 29 54 highest number is decreased by 2 and it is arranged at right
Step I: 20 57 25 88 34 19 48 77 66 29 54 94 end and so on for remaining steps.
Step II: 21 20 57 25 34 48 77 66 29 54 94 86 pj.k &1 ds fy, lcls de la[;k esa 2 dh o`f) gqà gS vkSj bls ck,a
Step III: 27 21 20 57 34 48 66 29 54 94 86 75 N¨j ij O;ofLFkr fd;k x;k gS vkSj mPpre la[;k d¨ 2 ls ?kVk;k
Step IV: 31 27 21 20 57 34 48 54 94 86 75 64 x;k gS vkSj bls nkfgus N¨j ij O;ofLFkr fd;k x;k gSA pj.k & 2 ds
Step V: 36 31 27 21 20 48 54 94 86 75 64 55 fy,] nwljh lcls de la[;k esa 2 dh o`f) gqà gS vkSj ;g O;ofLFkr
Step VI: 50 36 31 27 21 20 94 86 75 64 55 52 gS ck,a N¨j ij vkSj nwljh lcls cM+h la[;k esa 2 dh deh gqà gS vkSj
And Step VI is the last step of the rearrangement of the above
bls nk,a N¨j ij vkSj 'ks"k pj.k¨a ds fy, O;ofLFkr fd;k x;k gSA
input. As per the rules followed in the above steps, find out in
Input: 34 89 27 58 92 15 46 69 96 80
each of the following questions the appropriate steps for the
Step I: 17 34 89 27 58 92 46 69 80 94
given input.
Step II: 29 17 34 89 58 46 69 80 94 90
vkSj pj.k VI mij¨Dr buiqV ds iquZO;oLFkk dk vafre pj.k gSA
Step III: 36 29 17 58 46 69 80 94 90 87
mij¨Dr pj.k¨a esa fn, x, fu;e¨a ds vuqlkj] fuEufyf[kr çR;sd ç'u Step IV: 48 36 29 17 58 69 94 90 87 78
esa fn, x, buiqV ds fy, mi;qDr pj.k¨a dk irk yxk,a Step V: 60 48 36 29 17 94 90 87 78 67
Input: 34 89 27 58 92 15 46 69 96 80 26. c) Step 4 – 48 36 29 17 58 69 94 90 87 78 The sum of the
26. What is the sum of the number which is third from the number which is third from the right end and the number
right end and the number which is fourth from the left end which is fourth from the left end in step 4
in step 4? 90+17 =107
la[;k dk ;¨x D;k gS t¨ nkfgus N¨j ls rhljk gS vkSj pj.k 4 esa 27. (d) In step -4 58 69 94 appeared in the same sequence
ck,a N¨j ls p©Fks uacj ij gS\ 28. (b) (Second number from the left end in step -2) – (Fifth
(a) 130 (b) 119 number from the right end in step -3)
(c) 107 (d) 97 69 - 17 = 52
(e) None of these 7 29. (c) Step -5 (last step) – 60 48 36 29 17 94 90 87 78 67 94
27. In which of the following step “58 69 94” were found in the Third to the left of the number which ninth from the left end in
same order? last step is 94.
fuEufyf[kr esa ls fdl pj.k esa ^^58 69 94** mlh Øe esa ik, x, 30. (e) step 3 – 36 29 17 58 46 69 80 94 90 87
Fks\ Now, Odd number is increased by 13 and even number is
(a) Step -1 (b) Step -2 decreased by 15 in third step
(c) Step -3 (d) Step -4 i.e. 21 42 30 43 31 82 65 79 75 100
(e) Step -5 then product of lowest and fourth lowest number is:
28. What will be the resultant if the second number from the 21 × 42 = 882
left end in step -2 is subtracted from fifth number from the
right end in step -3? Mathematical Operation
pj.k &2 esa ck,a N¨j ls nwljk uacj pj.k &3 esa nkfgus N¨j ls ikaposa VBODMAS RULE →
uacj ls ?kVk;k x;k r¨ ifj.kke D;k g¨xk\ V → Var [A+B]
(a) 54 (b) 52 Small(-)
(c) 67 (d) 88 B → Bracket Curly{-}
(e) None of these Square (-)
O → of (dk →×)

183
Download Free PDFs & e-Books from Neon Classes App

D → Division ( ÷ ) → 29 + 8 × 7 – 22
M → Multiplication (×) → 29 + 56 – 22
A → Addition ( + ) → 85 – 22 ⇒ 63
S → Substraction ( - ) → Correct Answer of (c)
1. Select the correct combination of mathematical signs that 3. Which two numbers (Not Digits) need to be interchanged
can sequentially replace the * signs and balance the given to make the following equation correct?
equation. fuEufyf[kr lehdj.k dks lgh cukus ds fy, fdu nks la[;kvksa ¼vadksa
xf.krh; fpàksa ds ml lgh la;kstu dk p;u djsa tks vuqØfed :i dks ugha½ dks vkil esa cnyus dh vko';drk gS
ls * fpàksa dks çfrLFkkfir dj ldrk gS vkSj fn, x, lehdj.k dks 15 + 90 ÷ 9 × 5 – 11 = 28
larqfyr dj ldrk gSA (a) 11 and 9 (b) 15 and 9 (c) 15 and 5 (d) 9
60 * 48 * 36 * 6 * 15 * 53 and 5
(a) +, ÷, −, ×, = (b) +, ÷, ×, −, = Sol. (b)
(c) ÷, +, ×, −, = (d) ×, ÷, +, −, = 15 + 90 ÷ 9 × 5 – 11 = 28
Sol. (b) Option (a) 11 and 9 PUT IN EQUATION
Option (a) → +, ÷, −, ×, = → 15+90÷11×5-9 = 28
xf.krh; fpUgksa dks * ls izfrLFkkfir djds] gesa izkIr gksrk gS % ⤷Factor Form, So gets eliminated
→ 60+48÷36-6×15 = 53
Option (b) 15 and 9 PUT IN EQUATION
⤷Factor Form, So gets eliminated
→ 9+90÷15×5-11 = 28
Option b → +, ÷, ×, −, = → 9+6×5-11 = 28
xf.krh; fpUgksa dks * ls izfrLFkkfir djds] gesa izkIr gksrk gS % → 9+30-11 = 28
→ 60+48÷36×6-15 = 53 → 28 = 28
4 Option (c) 15 and 5 PUT IN EQUATION
→ 60+ × 6 – 15 = 53
3 → 5+90÷9×15-11 = 28
→ 60 + 8 – 15 = 53 → 5+10×15-11 = 28
→ 68 – 15 = 53 ⤷Higher value, So gets Eliminated
→ 53 = 53 Option (d) 9 and 5 PUT IN EQUATION
Option (c) ÷, +, ×, −, = → 15+90÷5×9-11 = 28
xf.krh; fpUgksa dks * ls izfrLFkkfir djds] gesa izkIr gksrk gS % → 15+18×9-11 = 28
→ 60÷48+36×6-15 = 53 ⤷ Higher value, So gets Eliminated
⤷ Factor Form, So gets eliminated ÷ → a Eliminated
Option (d) ×, ÷, +, −, = × → c, d Eliminated
xf.krh; fpUgksa dks * ls izfrLFkkfir djds] gesa izkIr gksrk gS % +
→ 60×48÷36+6-15 = 53 -
4 Correct Answer Option (b)
→ 60  + 6 - 15 = 53
3 4. Which two signs need to be interchanged to make the
→ 80 + 6 – 15 = 53 following equation correct?
→ 86 – 15 = 53 fuEufyf[kr lehdj.k dks larqfyr djus ds fy, fdu nks fpUgksa dks
→ 71 ≠ 53 ijLij cnyus dh vko’;drk gS\
÷ → A, C Eliminated 45 ÷ 15 × 5 + 15 – 11 = 39
× (a) – and × (b) – and +
+ (c) + and × (d) ÷ and +
- Sol. (d)
→ Correct Answer Option (b) 45 ÷ 15 × 5 + 15 – 11 = 39
2. If '@' means 'addition', '%' means 'multiplication', '$' Option (a) – and × PUT IN EQUATION
means 'division' and '#' means 'subtraction', then find the → 45÷15-5+15×11 = 39
value of the following expression. → 3-5+165 = 39
;fn '@' dk vFkZ 'addition'] '%' dk vFkZ 'multiplication'] '$' dk → 163 ≠ 39
vFkZ 'division' vkSj '#' dk vFkZ 'subtraction’ gS] rks fuEufyf[kr Option (b) – and + PUT IN EQUATION
O;atd dk eku Kkr dhft,A → 45÷15×5-15+11 = 39
29 @ 128 $ 16 % 7 # 22 → 3×5-15+11 = 39
(a) 58 (b) 47 (c) 63 (d) 23 → 15-15+11 = 39
Sol. (c) → 11 ≠ 39
29 @ 128 $ 16 % 7 # 22 Option (c) + and × PUT IN EQUATION
@ dk vFkZ → + → 45÷15+5×15-11 = 39
$ dk vFkZ → ÷ → 3+75-11 = 39
% dk vFkZ → × → 78-11 = 39
→ 67 ≠ 39
# dk vFkZ → -
Option (d) ÷ and + PUT IN EQUATION
xf.krh; fpUgksa dks SYMBOL ls izfrLFkkfir djds] gesa izkIr gksrk gS %
→45+15×5÷15-11 = 39
→ 29+128÷16×7-22

184
Best App for Govt. Jobs : Neonclasses (Download Now)

1 ÷ and +, 12 and 14 Interchanged


→ 45+15× -11 = 39
3  14 + 45 ÷ 15 × 12 – 10
→ 45+5-11 = 39  14 + 3 × 12 – 10
→ 50-11 = 39  14 + 36 – 10
→ 39 = 39
Correct Answer Option (d)
 40
5. Which two numbers and which two signs should be Correct option (a)
interchanged to balance the following equation? 8. By interchanging the given two signs + and – and two
fuEufyf[kr lehdj.k dks larqfyr djus ds fy, fdu nks la[;kvksa numbers 17 and 216 (NOT digits), which of the following
equations will be correct?
vkSj fdu nks fpàksa dks vkil esa cnyk tkuk pkfg,\
fn, x, nks fpàksa $ vkSj & vkSj nks la[;kvksa 17 vkSj 216 ¼vadksa dks
24 ÷ 16 – 96 + 48 × 12 = 195
(a) 12 and 16, × and ÷
ugha½ dks vkil esa cnyus ij] fuEufyf[kr esa ls dkSu lk lehdj.k
(b) 24 and 16, – and ÷
lgh gksxk\
(c) 24 and 48, × and ÷ (d) 12 and 48, × and + I. 10 × 216 + 17 ÷ 6 – 13 = 146
Sol. (a) 12 and 16, × and ÷ II. 7 × 216 + 41 – 17 ÷ 36 = 81
24 ÷ 16 – 96 + 48 × 12 = 195 (a) Only II (b) Only I
Option (a) 12 and 16, × and ÷ PUT IN EQUATION (c) Neither I nor II (d) Both I and II
Sol. (c)
 24 × 12 – 96 + 48 ÷ 16 = 195
+ and –, 17 and 216 Interchanging
288 – 96 + 3 = 195 Equation (i) 10 × 216 + 17 ÷ 6 – 13 = 146
 195 = 195  10 × 17 – 216 ÷ 6 + 13 = 146
Option (b) 24 and 16, – and ÷ PUT IN EQUATION
 170 – 36 + 13 = 146
 16 – 24 ÷ 96 + 48 × 12 = 195  147  146
⤷ Factor form, so gets eliminate Equation (ii) 7×216+41–17 ÷ 36 = 81
Option (c) 24 and 48, × and ÷ PUT IN EQUATION
 7 × 17 – 41 + 216 ÷ 36 = 81
 48 × 16 – 96 + 24 ÷ 12 = 195
 119 – 41 + 6 = 81
⤷ Higher value, so gets eliminate
 84  81
Option (d) 12 and 48, × and + PUT IN EQUATION
Correct option (c)
 24 ÷ 16 – 96 × 12 + 48 = 195 9. If the numbers 15 and 21 are interchanged, then which of
⤷ Factor form, so gets eliminate the following equations will be correctly balanced?
÷  D, B eliminated ;fn la[;k 15 vkSj 21 dks vkil esa cny fn;k tk,] rks fuEufyf[kr
×  C eliminated esa ls dkSu lk lehdj.k lgh <ax ls larqfyr gksxk\
(a) 62 ÷ 31 – 21 + 15 × 3 = 80
+
(b) 26 + 21 × 15 ÷ 3 – 14 = 108

(c) 24 – 21 ÷ 3 + 15 – 7 = 23
Correct option (a)
(d) 63 ÷ 15 + 21 × 5 – 4 = 74
6. What will come in the place of ‘?’ in the following equation,
Sol. (d) 63 ÷ 15 + 21 × 5 – 4 = 74
if ‘+’ and ‘–‘ are interchanged and also ‘×’ and ‘÷’ are
Numbers 15 and 21 are interchanged
interchanged?
Option (a) 62 ÷ 31 – 21 + 15 × 3 = 80
fuEufyf[kr lehdj.k esa '?' ds LFkku ij D;k vk,xk] ;fn '+' vkSj
'&' dks vkil esa cny fn;k tk, vkSj lkFk gh '×' vkSj '÷' dks Hkh
 62 ÷ 31 – 15 + 21 × 3 = 80
vkil esa cny fn;k tk,\  2 – 15 + 63 = 80
4 ÷ 6 − 9 + 48 × 8 = ?  50  80
(a) 27 (b) 38 (c) 24 (d) 42 Option (b) 26 + 21 × 15 ÷ 3 – 14 = 108
Sol. (a) 27  26 + 15 × 21 ÷ 3 – 14 = 108
4 ÷ 6 – 9 + 48 × 8 = ?  26 + 15 × 7 – 14 = 108
 [+, – and ×, ÷] Interchanged  26 + 105 – 14 = 108
 4 × 6 + 9 – 48 ÷ 8  117  108
 24 + 9 – 6  27 Option (c) 24 – 21 ÷ 3 + 15 – 7 = 23
Correct option (a)  24 – 15 ÷ 3 + 21 – 7 = 23
7. After interchanging the given two signs and two numbers
(not digits), what will be the value of the given equation?  24 – 5 + 21 – 7 = 23
fn, x, nks fpàksa vkSj nks la[;kvksa ¼vadksa dks ugha½ dks vkil esa cnyus  33  23
ds ckn fn, x, lehdj.k dk eku D;k gksxk\ Option (d) 63 ÷ 15 + 21 × 5 – 4 = 74
÷ and +, 12 and 14  63 ÷ 21 + 15 × 5 – 4 = 74
12 ÷ 45 + 15 × 14 – 10 = ?  3 + 75 – 4 = 74
(a) 40 (b) 60 (c) 80 (d) 20
 74 = 74
Sol. (a)
Correct option (d)
12 ÷ 45 + 15 × 14 – 10 = ?

185
Download Free PDFs & e-Books from Neon Classes App

10. If the signs ‘+’ and ‘÷’ are interchanged, then which of the  16 = 16
following equations can be correctly balanced? 12. Which of the following equation will become correct by
;fn fpà '+' vkSj '÷' dks vkil esa cny fn;k tk,] rks fuEufyf[kr esa replacing the * signs from left to right as per the given
ls dkSu lk lehdj.k lgh <ax ls larqfyr fd;k tk ldrk gS\ sequence of mathematical signs?
(a) 12 + 4 – 8 ÷ 3 = 11 xf.krh; fpUgksa ds fn, x, Øe ds vuqlkj * fpàksa dks ck,¡ ls nk,¡
(b) 24 – 12 ÷ 6 + 3 = 20 cnyus ij fuEufyf[kr esa ls dkSu&lk lehdj.k lgh gks tk,xk\
(c) 16 + 4 × 8 ÷ 3 = 45 The given sequence/fn;k x;k Øe:
(d) 22 + 11 × 8 ÷ 3 = 19 +, ÷, –, ×, =
Sol. (d) (a) 45 * 15 * 5 * 7 * 5 * 13 (b) 45 * 24 * 8 * 7 * 5 * 23
+ and ÷ interchanged (c) 60 * 15 * 5 * 7 * 5 * 25 (d) 45 * 15 * 5 * 5 * 5 * 18
Option (a) 12 + 4 – 8 ÷ 3 = 11 Sol. (a)
 12 ÷ 4 – 8 + 3 = 11 Left to Right  +, ÷, –, ×, =
 3 – 8 + 3 = 11 Option (a) 45 * 15 * 5 * 7 * 5 * 13
 –2  11 xf.krh; fpUgksa dks * ls izfrLFkkfir djds] gesa izkIr gksrk gS %
Option (b) 24 – 12 ÷ 6 + 3 = 20  45 + 15 ÷ 5 – 7 × 5 = 13
 24 – 12 + 6 ÷ 3 = 20  45 + 3 – 35 = 13
 12 + 2 = 20  48 – 35 = 13
 14  20  13 = 13
Option (c) 16 + 4 × 8 ÷ 3 = 45 Option (b) 45 * 24 * 8 * 7 * 5 * 23
 16 ÷ 4 × 8 + 3 = 45 xf.krh; fpUgksa dks * ls izfrLFkkfir djds] gesa izkIr gksrk gS %
 4 × 8 + 3 = 45  45 + 24 ÷ 8 – 7 × 5 = 23
 32 + 3 = 45  45 +3 – 35 = 23
 35  45  13  23
Option (d) 22 + 11 × 8 ÷ 3 = 19 Option (c) 60 * 15 * 5 * 7 * 5 * 25
 22 ÷ 11 × 8 + 3 = 19 xf.krh; fpUgksa dks * ls izfrLFkkfir djds] gesa izkIr gksrk gS %
 2 × 8 + 3 = 19  60 + 15 ÷ 5 – 7 × 5 = 25
 16 + 3 = 19  60 + 3 – 35 = 25
 19 = 19  28  25
11. If × stands for subtraction, ÷ stands for addition, + stands Option (d) 45 * 15 * 5 * 5 *5 * 18
for division, and – stands for multiplication, which of the xf.krh; fpUgksa dks * ls izfrLFkkfir djds] gesa izkIr gksrk gS %
following equations is correct?   45 + 15 ÷ 5 – 5 × 5 = 18
;fn × dk vFkZ ?kVko gS, ÷ dk vFkZ ;ksx gS] $ dk vFkZ Hkkx gS] vkSj –  45 + 3 – 25 = 18
dk vFkZ xq.kk gS] rks fuEufyf[kr esa ls dkSu lk lehdj.k lgh gS\  23  18
(a) 25 + 5 × 20 ÷ 5 – 16 = 0
Correct option (a)
(b) 45 – 15 ÷ 15 × 60 + 16 = 42
(c) 9 ÷ 28 – 10 × 15 + 9 = 10 MATRIX
(d) 16 ÷ 20 – 6 + 10 × 12 = 16 Directions/funs’Z k: Read the following information carefully and
Sol. (d) 16 ÷ 20 – 6 + 10 × 12 = 16 answer the questions given below.
Option (a) fuEufyf[kr tkudkjh dks è;kuiwoZd i<+sa vkSj uhps fn, x, ç'uksa ds
25 + 5 × 20 ÷ 5 – 16 = 0 mÙkj nsAa
 25 ÷ 5 – 20 + 5 × 16 = 0 Directions (1-5) : A Word is represented by only one set of
 5 – 20 + 80 = 0 numbers as given in anyone of the alternatives. The set of
 85 – 20 = 0 numbers given in the alternatives are represented by two
classes of alphabets as in two matrics given below. The
 65 ≠ 0
columns and rows of Matrix I are numbered from 0 to 4 and
Option (b) 45 – 15 ÷ 15 × 60 + 16 = 42
that of Matrix Il are numbered from 5 to 9.
 45 × 15 + 15 – 60 ÷ 16 = 42 fdlh Hkh fodYi esa fn, x, 'kCnksa ds dsoy ,d lsV }kjk ,d 'kCn
⤷Factor form, so gets eliminated dk çfrfufèkRo fd;k tkrk gSA fodYiksa esa fn, x, la[;kvksa ds lewg
(c) 9 ÷ 28 – 10 × 15 + 9 = 10 dks v{kjksa ds nks oxksaZ }kjk n'kkZ;k x;k gS tSlk fd uhps fn, x, nks
 9 + 28 × 10 – 15 ÷ 9 = 10 vkO;wgksa esa gSA eSfVªDl ds d‚ye vkSj iafä;ksa dh la[;k 0 ls 4 rd
⤷Factor form, so gets eliminate vkSj eSfVªDl-I ds d‚ye vkSj iafä;ksa dh la[;k 5 ls 9 rd gSA
(d) 16 ÷ 20 – 6 + 10 × 12 = 16 A letter from these matrics can be represented first by its row
 16 + 20 × 6 ÷ 10 – 12 = 16 and next by its column. e.g., 'A' can be represented by 00, 12,
23 etc. and 'P' can be represented by 58, 69, 75 etc. Similarly,
6
 16 +20× -12 = 16 you have to identify the set for the word given in each
10
question.
 16 + 12 - 12 = 16

186
Best App for Govt. Jobs : Neonclasses (Download Now)

bu vkO;wgksa esa ls ,d v{kj dks igys mldh iafä vkSj ckn esa mlds (b) 86, 12, 31, 76
LraHk }kjk n'kkZ;k tk ldrk gSA tSls] 'A' dks 00] 12] 23 vkfn }kjk (c) 58, 41, 12, 67
n'kkZ;k tk ldrk gS vkSj 'P' dks 58] 69] 75 vkfn }kjk n'kkZ;k tk (d) 88, 77, 41, 67
ldrk gSA blh rjg] vkidks çR;sd ç'u esa fn, x, 'kCn ds fy, Sol. (b)
lsV dh igpku djuh gksxhA According to the matrics,
Matrix I P - 58, 69, 75, 86, 97
A - 00, 12, 23, 34, 41
0 1 2 3 4
S - 02, 14, 20, 31, 43
0 A R S N C
T - 59, 65, 76, 87, 98
1 N C A R S
Therefore, PAST ⇒ 86, 12, 31, 76
2 S N C A R
5. Identify the representation of NEST?
3 R S N C A
(a) 32, 56, 20, 89
4 C A R S N
(b) 10, 65, 41, 76
(c) 32, 76, 34, 98
Matrix II (d) 21, 67, 14, 59
5 6 7 8 9 Sol. (d)
5 O E L P T According to the matrics,
6 T O E L P N - 03, 10, 21, 32, 44
7 P T O E L E - 56, 67, 78, 89, 95
8 L P T O E S - 02, 14, 20, 31, 43
9 E L P T O T - 59, 65, 76, 87, 98
Therefore, NEST ⇒ 21, 67, 14, 59.
1. Identify the representation of RATE? Directions (6-9) : In each of the following questions, a word is
(a) 13, 12, 98, 67 represented by only one set of numbers as given in any of the
(b) 42, 23, 56, 76 alternatives. The sets of numbers given in the alternatives are
(c) 30, 14, 95, 89 represented by two classes of alphabets as in the two given
(d) 24, 43,89, 95 matrics. The columns and rows of Matrix I are numbered from
Sol. (a) O to 4 and those of Matrix II from 5 to 9. A letter from these
According to the matrics, matrics can be represented first by its row and then the
R - 01, 13, 24, 30, 42 column number e.g., in the matrics for question 1 to 4, M can
A - 00, 12, 23, 34, 41 be represented by 14, 21 etc,; O can be represented by 20, 32,
T - 59, 65, 76, 87, 98 etc. Similarly you have to identify the correct set for the word
E - 56, 67, 78, 89, 95 given in each question.
Therefore, RATE ⇒ 13, 12, 98, 67. izR;sd iz’u esa] ,d ’kCn dsoy lap;k&lewg }kjk n’kkZ;k x;k gs] tSls
2. Identify the representation of POET? fd fodYiks esa ls fdlh ,d esa fn;k x;k gSA fodYiks esa fn, x,
(a) 69, 88, 67, 65 la[;k lewg v{kjks ds nks oxksZ }kjk n’kkZ;s x, gS] tSls fd uhps fn,
(b) 75, 56, 65, 67 x, nks vkO;wgks esa gSA vkO;wg I ds LraHk vkSj iafDr dh la[;k 0 ls 4
(c) 77, 88, 98, 78 vkSj vkO;wg II dh 5 ls 9 nh xbZ gSA bu vkO;wgks ls ,d v{kj dks
(d) 75, 66, 76, 78 igys mldh iafDr vkSj ckn esa LrEHk la[;k }kjk n’kkZ;k tk ldrk
Sol. (a) gSA
According to the matrics, mnkgj.k ds fy, vkO;wg 1 ls 4 rd dks vkfn }kjk n’kkZ;k tk
P - 58, 69, 75, 86, 97 ldrk gSA rFkk ‘M’ dks 14] 21 vkfn }kjk n’kkZ;k tk ldrk gS vkSj
O - 55, 66, 77, 88, 99 O dks 20 vkSj 32 vkfn }kjk n’kkZ;k tk ldrk gSA blh izdkj izR;sd
E - 56, 67, 78, 89, 95
iz’u esa fn, x, lgh lewg dks igokfu,A
T - 59, 65, 76, 87, 98
Matrix I
Therefore, POET ⇒ 69, 88, 67, 65. 0 1 2 3 4
3. Identify the representation of PEST?
0 F O M S R
(a) 97, 89, 34, 59
(b) 58, 67, 43, 98 1 S R F O M
(c) 57, 59, 31, 98 2 O M S R F
(d) 68, 95, 31, 76 3 R F O M S
Sol. (b) 4 M S R F O
According to the matrics,
Matrix II
P - 58, 69, 75, 86, 97
5 6 7 8 9
E - 56, 67, 78, 89, 95
5 A T D I P
S - 02, 14, 20, 31, 43
T - 59, 65, 76, 87, 98 6 I P A T D
7 T D I P A
Therefore, PEST ⇒ 58, 67, 43, 98.
8 P A T D I
4. Identify the representation of PAST?
(a) 75, 21, 14, 65 9 D I P A T
6. MOST

187
Download Free PDFs & e-Books from Neon Classes App

(a) 40, 44, 22, 89 (b) 33, 20, 11, 79 Matrix I


(c) 21, 00, 03, 88 (d) 02, 13, 34, 56 0 1 2 3 4
Sol. (d) 0 A E S T H
According to the matric s,
1 T H A E S
M - 02, 14, 21, 33, 40
O - 01, 13, 20, 32, 44 2 E S T H A
S - 03, 10, 22, 34, 41 3 H A E S T
T - 56, 68, 75, 87, 99 4 S T H A E
Therefore, MOST ⇒02, 13, 34, 56.
7. ROAD Matrix – II
(a) 42, 32, 79, 58 (b) 23, 32, 98, 99 5 6 7 8 9
(c) 11, 13, 67, 69 (d) 04, 20, 55, 78 5 P O R K L
Sol. (c)
6 K L P O R
According to the matrics,
R - 04, 11, 23, 30, 42 7 O R K L P
O - 01, 13, 20, 32, 44 8 L P O R K
A - 55, 67, 79, 86, 98 9 R K L P O
D – 57,69,76,88,95
Therefore, ROAD ⇒11, 13, 67, 69. 10. EAST
8. STOP (a) 44, 32, 21, 03 (b) 32, 31, 02, 04
(a) 10, 56, 44, 97 (b) 41, 68, 01, 77 (c) 20, 43, 33, 11 (d) 13, 12, 14, 10
(c) 22, 75, 32, 86 (d) 33, 99, 42, 59 Sol. (d)
Sol. (a) According to the matrics,
According to the matrics, E – 01, 13, 20, 32, 44
S - 03, 10, 22, 34, 41 A – 00, 12, 24, 31, 43
T - 56, 68, 75, 87, 99 S- 02, 14, 21, 33, 40
O -01, 13, 20, 32, 44 P – 59,66, 78, 85,97 T – 03, 10, 22, 34, 41
Therefore, STOP ⇒ 10, 56, 44, 97 Therefore, EAST ⇒13, 12, 14, 10.
9. FOAM 11. ROSE
(a) 24, 01, 55, 22 (b) 00, 01, 67, 33 (a) 95, 75, 02, 32 (b) 88, 76, 31, 32
(c) 12, 13, 67, 23 (d) 43, 52, 56, 33 (c) 86, 67, 33, 44 (d) 57, 87, 32, 33
Sol. (b) Sol. (a)
According to the matrics, According to the matrics,
F – 00, 12, 24, 31,43 R – 57, 69, 76, 88, 95
O - 01, 13, 20, 32, 44 O – 56, 68, 75, 87, 99
A- 55, 67, 79, 86, 98 S- 02, 14, 21, 33, 40
M - 02, 14, 21, 33, 40 E - 01, 13, 20, 32, 44
Therefore, FOAM ⇒ 00, 01, 67, 33. Therefore, ROSE ⇒95, 75, 02, 32.
Directions (10-14) : In each of the following questions, a word 12. SOLE
is represented by only one set of numbers as given in any of (a) 41, 57, 87, 31 (b) 33, 99, 66, 44
the alternatives. The sets of numbers given in the alternatives (c) 21, 75, 44, 02 (d) 02, 78, 87, 13
are represented by two clases of alphabets as in the two given Sol. (b)
matrics. The columns and rows of Matrix I are numbered from According to the matrics,
O to 4 and those of Matrix II from 5 to 9. A letter from these S – 02, 14, 21, 33, 40
matrics can be represented first by its row and then the O – 56, 68, 75, 87, 99
column number e.g., in the matrics for question 1 to 4, M can L- 59, 66, 78, 85, 97
be represented by 14, 21 etc,; O can be represented by 20, 32, E - 01, 13, 20, 32, 44
etc. Similarly you have to identify the correct set for the word Therefore, SOLE ⇒33, 99, 66, 44.
given in each question. 13. LAKE
izR;sd iz’u esa] ,d ’kCn dsoy lap;k&lewg }kjk n’kkZ;k x;k gs] tSls (a) 97, 00, 77, 12 (b) 66, 12, 58, 40
fd fodYiks esa ls fdlh ,d esa fn;k x;k gSA fodYiks esa fn, x, (c) 85, 31, 77, 44 (d) 77, 43, 76, 31
la[;k lewg v{kjks ds nks oxksZ }kjk n’kkZ;s x, gS] tSls fd uhps fn, Sol. (c)
x, nks vkO;wgks esa gSA vkO;wg I ds LraHk vkSj iafDr dh la[;k 0 ls 4 According to the matrics,
vkSj vkO;wg II dh 5 ls 9 nh xbZ gSA bu vkO;wgks ls ,d v{kj dks L – 59, 66, 78, 85, 97
igys mldh iafDr vkSj ckn esa LrEHk la[;k }kjk n’kkZ;k tk ldrk A - 00, 12, 24, 31, 43
gSA K- 58, 65, 77, 85, 97
mnkgj.k ds fy, vkO;wg 1 ls 4 rd dks vkfn }kjk n’kkZ;k tk E - 01, 13, 20, 32, 44
ldrk gSA rFkk ‘M’ dks 14] 21 vkfn }kjk n’kkZ;k tk ldrk gS vkSj Therefore, LAKE ⇒ 85, 31, 77, 44
O dks 20 vkSj 32 vkfn }kjk n’kkZ;k tk ldrk gSA blh izdkj izR;sd 14. LEST
iz’u esa fn, x, lgh lewg dks igokfu,A (a) 97, 32, 21, 34 (b) 87, 32, 21, 31

188
Best App for Govt. Jobs : Neonclasses (Download Now)

(c) 85, 02, 04, 22 (d) 66, 00, 20, 34 From matrix II, A can be coded as 55, 67, 79, 86 or 98.
Sol. (a) From matrix II, D can be coded as 57, 69, 76, 88 or 95.
According to the matrics, Clearly only (c) contains the correct codes.
L – 59, 66, 78, 85, 97 17. STOP
E - 01, 13, 20, 32, 44 (a) 10, 56, 44, 97 (b) 41, 68, 01, 77
S- 02, 14, 21, 33, 40 (c) 22, 75, 32, 86 (d) 33, 99, 42, 59
T - 03, 10, 22, 34, 41 Sol. (a)
Therefore, LEST ⇒97, 32, 21, 34. From matrix I, S can coded as 03, 10, 22, 34 or 41.
Directions (15-18) : In each of the following questions, a From matrix II, T can be coded as 56, 68, 75, 87 or 99.
word is represented by only one set of numbers as given in From matrix I, O can be coded as 01, 13, 20, 32 or 44.
any of the alternatives. The sets of numbers given in the From matrix II, P can be coded as 59, 66, 78, 85 or 97.
alternatives are represented by two clases of alphabets as in 18. FOAM
the two given matrics. The columns and rows of Matrix I are (a) 24, 01, 55, 22 (b) 00, 01, 67, 33
numbered from O to 4 and those of Matrix II from 5 to 9. A (c) 12, 13, 67, 23 (d) 43, 52, 56, 33
letter from these matrics can be represented first by its row Sol. (b)
and then the column number e.g., in the matrics for question 1 From matricx I, F can be coded as 00, 12, 24, 31 or 43
to 4, M can be represented by 14, 21 etc,; O can be represented From matrix I, O can be coded as 01, 13, 20, 32 or 44.
by 20, 32, etc. Similarly you have to identify the correct set for From matrix II, A can be coded as 55, 67, 79 86 or 98.
the word given in each question. From matrix I, M can be coded as 02, 14, 21, 33, or 40.
izR;sd iz’u esa] ,d ’kCn dsoy lap;&lewg }kjk n’kkZ;k x;k gs] tSls Direction (19) : The hundred cells in the square below have
fd fodYiks esa ls fdlh ,d esa fn;k x;k gSA fodYiks esa fn, x, been filled with letter. The columns and the rows are identified
la[;k lewg v{kjks dks nks oxksZ }kjk n’kkZ;k x;k gS] tSls fd uhps fn, by the numbers 0 to 9. A letter in a cell is represented first by
x, nks vkO;wgks esa gSA vkO;wg I ds LraHk vkSj iafDr dh la[;k 0 ls 4 its column number and then by its column number and then
vkSj vkO;wg II dh 5 ls 9 nh xbZ gSA bu vkO;wgks ls ,d v{kj dks by its column number and then by its row number e.g., G in
column 3 and row 1 represented by 31. In each of the
igys mldh iafDr vkSj ckn esa LrEHk la[;k }kjk n’kkZ;k tk ldrk
following questions, a word has been given which is
gSA
represented by one of the four alternatives given under it. Find
mnkgj.k ds fy, vkO;wg 1 ls 4 rd dks vkfn }kjk n’kkZ;k tk the correct alternative.
ldrk gSA rFkk ‘M’ dks 14] 21 vkfn }kjk n’kkZ;k tk ldrk gS vkSj uhps fn, x, ,d oxZ ds lkS [kkuks dks v{kjks ls Hkjk x;k gSA ftuesa
O dks 20 vkSj 32 vkfn }kjk n’kkZ;k tk ldrk gSA blh izdkj izR;sd LraHk rFkk iafDr dks la[;k 0 rFkk 9 }kjk n’kkZ;k x;k gSA mnkgj.k
iz’u esa fn, x, lgh lewg dks igpkfu,A ds fy, G LraHk esa 3 vkSj iafDr esa 1 gS mls 31 }kjk iznf’kZr fd;k
Matrix I x;k gSA izR;sd fn, x, iz’u es]a ftlesa ,d ’kCn dks vanj fn, x,
0 1 2 3 4 pkj oSfdYi ’kCnks }kjk n’kkZ;k x;k gSA rks lgh fodYi Kkr djs \
0 F O M S R
0 1 2 3 4 5 6 7 8 9
1 S R F O M
0 I L B P K N H S A E
2 O M S R F
3 R F O M S 1 M A Q G T V I O N U
4 M S R F O 2 H R W J A X B E C I
3 T Y A I U U O N J F
Matrix II 4 F O B M E G U K W R
5 6 7 8 9 5 A C L J X R A A X T
5 A T D I P 6 P S U E Z K V W D L
6 I P A T D 7 Z D Y V F O H Y I O
7 T D I P A 8 M I Z Q E A U F I S
8 P A T D I 9 P E O D E U Q O C G
9 D I P A T 19. BLOT
15. MOST (a) 20, 10, 71, 22 (b) 24, 10, 26, 48
(a) 40, 44, 22, 89 (b) 33, 20, 11, 79 (c) 34, 35, 63, 03 (d) 62, 25, 57, 95
(c) 21, 00, 03, 88 (d) 02, 13, 34, 56 Sol. (d)
Sol. (d) B can be coded as 20, 62 or 24;
From matrix I, M can be coded as 02, 14, 21, 33 or 40. L can be coded as 10, 25 or 96;
From matrix I, O can be coded as 01, 13, 20, 32 or 44. O can be coded as 71, 63, 14, 57, 97, 29 or 79;
From matrix I, S can be coded as 03, 10, 22, 34 or 41. T can be coded as 41, 03 or 95.
From matrix II, T can be coded as 56, 68, 75, 87 or 99. 20. A word is represented by only one set of numbers as given in
Clearly, (d) is the only set of correct codeds. any one of the alternatives. The sets of numbers given in the
16. ROAD alternatives are represented by two classed of alphabets as in
(a) 42, 32, 79, 58 (b) 23, 32, 98, 99 two matrics given below. The columns and rows of matrix I are
(c) 11, 13, 67, 69 (d) 04, 20, 55, 78 numbered from 0 to 4 and that of Matrix II are numbered from
Sol. (c) 5 to 9. A letter from these matrics can be represented first by
From matgrix I, R can be coded as 04, 11, 23, 30 or 42. its row and next by its column, e.g., ‘A’ can be represented by
From matrix I, O can be coded as 01, 13, 20, 32 or 44.

189
Download Free PDFs & e-Books from Neon Classes App

01, 14, etc., and ‘O’ can be represented by 59,57 67etc. 0 1 2 3 4


Similarly, you have to identify the set for the word ‘PEARL’
0 P A G R Z
,d 'kCn dsoy ,d la[;k & lewg }kjk n’kkZ;k x;k gS] tSls fd
fodYiksa esa ls fdlh ,d esa fn;k x;k gSA fodYiksa esa fn, x, la[;k 1 G R Z P A
lweg v{kjksa ds nks oxksZa }kjk n’kkZ;s x, gSa] tSls fd uhps fn, x, nks 2 Z P A G R
vkO;wgksa esa gSaA vkO;wg I ds LraHk vkSj iafDr dh la[;k 0 ls 4 vkSj 3 A G R Z P
vkO;wg II dh 5 ls 9 nh xbZ gSA bu vkO;wgksa ls ,d v{kj dks igys
4 R Z P A G
mldh iafDr vkSj ckn esa LraHk la[;k }kjk n’kkZ;k tk ldrk gSA
mnkgj.k ds fy, ‘A’ dks 01] 14 rFkk ‘O’ dks 59, 67 bR;kfn }kjk Matrix II
n’kkZ;k tk ldrk gSA bl izdkj 'kCn ‘PEARL’ dks dSls n’kkZ;k 5 6 7 8 9
tk;sxk \ 5 E M L N O
Matrix I 6 L E O M N
7 O N E L M
0 1 2 3 4
8 N O M E L
0 P A G R Z 9 M L N O E
1 G R Z P A (a) 99, 01, 44, 96, 77
(b)66, 43, 44, 79, 88
2 Z P A G R
(c) 55, 14, 11, 78, 66
3 A G R Z P (d) 88, 22, 31, 89, 76
4 R Z P A G Sol. (a)
According to the matrics,
E – 55, 66, 77, 88, 99,
Matrix II
A – 01, 14, 22, 30, 43
5 6 7 8 9 G – 02, 10, 23, 31, 44
5 E M L N O L – 55, 65, 78, 89, 96
6 L E O M N E - 55, 66, 77, 88, 99
7 O N E L M Therefore, EAGLE ⇒ 99, 01, 44, 96, 77
8 N O M E L 22. A word is reperesenetd by only one set of numbers as given in
9 M L N O E any one of the alternatives. The sets of numbers given in the
(a) 00, 55, 22, 11, 96 alternatives are represented by two classes of alphabets as in
(b) 00, 66, 14, 32, 56 two matrics given below. the columns and rows of Matrix I are
(c) 13, 77, 30, 14, 88 numbered from 0 to 4 and that of Matrix II are numbered from
(d) 12, 88, 43, 32, 89 5 to 9. A letter from these matrics can be represented first by
Sol. (a) its row and next by its column, e.g. ‘A’ can be represented by
According to the matrics, 01, 14 etc. and ‘T’ can be represented by 55’ 68 etc. Similarly,
P – 00, 13, 21, 34, 42 you have to identify the set for the word ‘PERSON’
E – 55, 66, 77, 88, 99 ,d 'kCn dsoy ,d la[;k & lewg }kjk n’kkZ;k x;k gS] tSls fd
A – 01, 14, 22, 30, 43 fodYiksa esa ls fdlh ,d esa fn;k x;k gSA fodYiksa esa fn, x, la[;k
R – 03, 11, 24, 32, 40 lewg v{kjksa ds nks oxksaZ dks n’kkZ;s x, gSa] tSls fd uhps fn, x, nks
L - 57, 65, 78, 89, 96
vkO;wgksa esa gSaA vkO;wg I ds LraHk vkSj iafDr dh la[;k 0 ls 4 vkSj
Therefore, PEARL ⇒ 00, 55, 22, 11, 96 vkO;wg II dh 5 ls 9 nh xbZ gSA bu vkO;wgksa ls ,d v{kj dks igys
21. A word is represented by only one set of numbers as given in mldh iafDr vkSj ckn esa LrEHk la[;k }kjk n’kkZ;k tk ldrk gSA
any one of the alternatives. The sets of numbers given in
mnkgj.k ds fy, A’ dks 01] 14 rFkk ‘T’ rFkk 55*68 dks bR;kfn
alternatives are represented by two classes of alphabets as in
}kjk n’kkZ;k tk ldrk gSA bl izdkj 'kCn ‘PERSON’ dks dSls
two matrics given below. the columns and rows of Matrix I are
numbered from 0 to 4 and that of Matrix II are numbered from
n’kkZ;k tk;sxk \
5 to 9. A letter from these matrics can be represented first by Matrix I
its row and next by its columns, e.g. ‘A’ can be represented by 0 1 2 3 4
01, 14 etc. and ‘M’ can be represented by 56,68 etc Similarly, 0 R A S E N
you have to identify the set for the word “EAGLE”
,d 'kCn dsoy ,d la[;k & lewg }kjk n’kkZ;k x;k gS] tSls fd 1 N E S R A
fodYiksa esa ls fdlh ,d esa fn;k x;k gSA fodYiksa esa fn, x, la[;k 2 E A R N S
lewg v{kjksa ds nks oxksZa }kjk n’kkZ;s x, gSa] tSls fd uhps fn, x, nks 3 A S N R E
vkO;wgksa esa gSaA vkO;wg I ds LraHk vkSj iafDr dh la[;k 0 ls 4 vkSj
4 E A R N S
vkO;wg II dh 5 ls 9 nh xbZ gSA bu vkO;wgksa ls ,d v{kj dks igys
mldh iafDr vkSj ckn esa LrEHk la[;k }kjk n’kkZ;k tk ldrk gSA
mnkgj.k ds fy, ‘A’ dks 01] 14 rFkk ‘M’ dks 56] 68 bR;kfn }kjk Matrix II
n’kkZ;k tk ldrk gSA bl 'kCn “EAGLE” dks dSls n’kkZ;k tk ldrk 5 6 7 8 9
gS \ 5 T O P I C
Matrix I

190
Best App for Govt. Jobs : Neonclasses (Download Now)

6 C P O T I R – 59, 68, 77, 86, 95


U – 56, 65, 78, 87, 99
7 P O T C I
D – 01, 10, 23, 34, 42
8 T O P I C E– 02, 11, 20, 33, 44
9 I P O T C Therefore, RUDE ⇒ 59, 99, 34, 11
(a) 66, 03, 10, 33, 56, 03, 24. A word is represented by only one set of numbers as given in
(b) 96, 12, 32, 40, 77, 34, any one of the alternatives. The sets of numbers given in
(c) 75, 20, 43, 04, 98, 42, alternatives are represented by two classes of alphabets as in
(d) 87, 11, 22, 24, 67, 04, two matrics given below. the columns and rows of Matrix I are
Sol. (d) numbered from 0 to 4 and that of Matrix II are numbered from
According to the matrics, 5 to 9. A letter from these matrics can be represented first by
P – 57, 66, 75, 87, 96 its row and next by its columns, e.g. ‘A’ can be represented by
E – 03, 11, 20, 34, 40 00, 21 etc. and ‘S’ can be represented by 55, 98 etc Similarly,
R – 00, 13, 22, 33, 42 you have to identify the set for the word “SLOW”
S – 02,12, 24, 31, 44, ,d 'kCn dsoy ,d la[;k & lewg }kjk n’kkZ;k x;k gS] tSls fd
O - 56, 67, 76, 86, 97 fodYiksa esa ls fdlh ,d esa fn;k x;k gSA fodYiksa esa fn, x, la[;k
N – 04, 10, 23, 32, 43 lewg v{kjksa ds nks oxksZa }kjk n’kkZ;s x, gSa] tSls fd uhps fn, x, nks
Therefore, PERSON vkO;wgksa esa gSaA vkO;wg I ds LraHk vkSj iafDr dh la[;k 0 ls 4 vkSj
⇒ 87, 11, 22, 24, 67, 04, vkO;wg II dh 5 ls 9 nh xbZ gSA bu vkO;wgksa ls ,d v{kj dks igys
23. A word is reperesenetd by only one set of numbers as given in mldh iafDr vkSj ckn esa LrEHk la[;k }kjk n’kkZ;k tk ldrk gSA
any one of the alternatives. The sets of numbers given in the mnkgj.k ds fy, ‘A’ dks 00] 21 rFkk ‘S’ dks 58] 98 bR;kfn }kjk
alternatives are represented by two classes of alphabets as in n’kkZ;k tk ldrk gSA bl 'kCn “SLOW” dks dSls n’kkZ;k tk ldrk gS
two matrics given below. The columns and rows of Matrix I are \
numbered from 0 to 4 and that of Matrix II are numbered from Matrix I
5 to 9. A letter from these matrics can be represented by 03, 12
0 1 2 3 4
etc. and ‘M’ can be represented by 55, 67 etc. Similarly, you
have to identify the set for the word ‘RUDE’ 0 A E K G L
,d 'kCn dsoy ,d la[;k & lewg }kjk n’kkZ;k x;k gS] tSls fd 1 H B I J K
fodYiksa esa ls fdlh ,d esa fn;k x;k gSA fodYiksa esa fn, x, la[;k
2 M A C B C
lewg v{kjksa ds nks oxksaZ dks n’kkZ;s x, gSa] tSls fd uhps fn, x, nks
vkO;wgksa esa gSaA vkO;wg I ds LraHk vkSj iafDr dh la[;k 0 ls 4 vkSj 3 D E F D L
vkO;wg II dh 5 ls 9 nh xbZ gSA bu vkO;wgksa ls ,d v{kj dks igys 4 H I J K E
mldh iafDr vkSj ckn esa LrEHk la[;k }kjk n’kkZ;k tk ldrk gSA
mnkgj.k ds fy, A’ dks 03] 12 rFkk ‘M’ rFkk 55]67 dks bR;kfn Matrix II
}kjk n’kkZ;k tk ldrk gSA bl izdkj 'kCn ‘RUDE’dks dSls n’kkZ;k 5 6 7 8 9
tk;sxk \ 5 N S R S T
Matrix I 6 Q O T U X
0 1 2 3 4 7 W X P U V
8 Y Z Y Q X
0 B D E T O
9 Z W R S R
1 D E T O B
2 E B O D T (a) 58, 34, 66, 95 (b) 98, 04, 66, 96
3 T O B E D (c) 58, 34, 66, 76 (d) 98, 04, 66, 95
Sol. (b)
4 O T D B E According to the matrics,
S – 56, 58, 98,
Matrix II L – 04, 34,
O –66,
5 6 7 8 9
W– 75, 96
5 M U I L R Therefore, SLOW ⇒ 98, 04, 66, 96
6 U L M R I 25. A word is represented by only one set of numbers as given in
7 I M R U L any one of the alternatives. The sets of numbers given in
alternatives are represented by two classes of alphabets as in
8 L R U I M two matrics given below. The columns and rows of Matrix I are
9 R I L M U numbered from 0 to 4 and that of Matrix II are numbered from
5 to 9. A letter from these matrics can be represented first by
(a) 59, 99, 34, 11 (b) 77, 56, 02, 01
its row and next by its columns, e.g. ‘A’ can be represented by
(c) 95, 87, 42, 12 (d) 98, 04, 66, 95
00, 11, 20 etc. and ‘P’ can be represented by 59, 68, 75 etc
Sol. (a)
Similarly, you have to identify the set for the word “LOAD”
According to the matrics,

191
Download Free PDFs & e-Books from Neon Classes App

,d 'kCn dsoy ,d la[;k & lewg }kjk n’kkZ;k x;k gS] tSls fd
fodYiksa esa ls fdlh ,d esa fn;k x;k gSA fodYiksa esa fn, x, la[;k
lewg v{kjksa ds nks oxksZa }kjk n’kkZ;s x, gSa] tSls fd uhps fn, x, nks
vkO;wgksa esa gSaA vkO;wg I ds LraHk vkSj iafDr dh la[;k 0 ls 4 vkSj
vkO;wg II dh 5 ls 9 nh xbZ gSA bu vkO;wgksa ls ,d v{kj dks igys
mldh iafDr vkSj ckn esa LrEHk la[;k }kjk n’kkZ;k tk ldrk gSA (a) 1 (b) 2 (c) 3 (d) 4
mnkgj.k ds fy, ‘A’ dks 00, 11, 20 rFkk ‘P’ dks 59, 68, 75 bR;kfn Sol. (b)
}kjk n’kkZ;k tk ldrk gSA bl 'kCn “EAGLE” dks dSls n’kkZ;k tk 3. Choose the alternative which is closely resembles the
ldrk gS \ mirror image of the given combination.
Matrix I og fodYi pqusa tks fn, x, la;kstu dh niZ.k Nfo ls fudV ls
0 1 2 3 4
feyrk tqyrk gksA

0 A B C D E
1 B A E D C
2 A C D B E
3 E A D C B (a) 1 (b) 2 (c) 3 (d) 4
4 C E A D B Sol. (c)
4. Choose the alternative which is closely resembles the
mirror image of the given combination.
Matrix II og fodYi pqusa tks fn, x, la;kstu dh niZ.k Nfo ls fudV ls
5 6 7 8 9 feyrk tqyrk gksA
5 L M N O P
6 M L N O P
7 P L M N O
8 P O M N L (a) 1 (b) 2 (c) 3 (d) 4
9 O M P L N Sol. (c)
5. Choose the alternative which is closely resembles the
mirror image of the given combination.
(a) 55,42, 86, 03 (b) 66, 40, 31, 13 og fodYi pqusa tks fn, x, la;kstu dh niZ.k Nfo ls fudV ls
(c) 89, 86, 11, 99 (d) 76, 95, 20, 32
feyrk tqyrk gksA
Sol. (d)
According to the matrics,
L – 55, 66, 76, 89, 98
O – 58, 69, 79, 86, 95
A – 00, 11, 20, 31, 42
D – 03, 13, 22, 32, 43
Therefore, LOAD ⇒ 76, 95, 20, 32 (a) 1 (b) 2 (c) 3 (d) 4
Sol. (a)
Mirror Image 6. Choose the alternative which is closely resembles the
1. Choose the alternative which is closely resembles the mirror image of the given combination.
mirror image of the given combination. og fodYi pqusa tks fn, x, la;kstu dh niZ.k Nfo ls fudV ls
og fodYi pqusa tks fn, x, la;kstu dh niZ.k Nfo ls fudV ls feyrk tqyrk gksA
feyrk tqyrk gksA

(a) 1 (b) 2 (c) 3 (d) 4 (a) 1 (b) 2 (c) 3 (d) 4


Sol. (c) Sol. (d)
2. Choose the alternative which is closely resembles the 7. Choose the alternative which is closely resembles the
mirror image of the given combination. mirror image of the given combination.
og fodYi pqusa tks fn, x, la;kstu dh niZ.k Nfo ls fudV ls og fodYi pqusa tks fn, x, la;kstu dh niZ.k Nfo ls fudV ls
feyrk tqyrk gksA feyrk tqyrk gksA

192
Best App for Govt. Jobs : Neonclasses (Download Now)

(a) 1 (b) 2 (c) 3 (d) 4


Sol. (c) Sol. (c)
8. Choose the alternative which is closely resembles the 13. Select the correct mirror image of the given combination
mirror image of the given combination. when the mirror is placed at ‘PQ’ as shown.
og fodYi pqusa tks fn, x, la;kstu dh niZ.k Nfo ls fudV ls fn, x, la;kstu dh lgh niZ.k Nfo dk p;u djsa tc niZ.k dks 'PQ'
feyrk tqyrk gksA ij fn[kk;k x;k gSA

(a)
(b)
(c)
(a) 1 (b) 2 (c) 3 (d) 4
(d)
Sol. (a)
9. Choose the alternative which is closely resembles the Sol. (d)
mirror image of the given combination. 14. Select the correct mirror image of the given combination
og fodYi pqusa tks fn, x, la;kstu dh niZ.k Nfo ls fudV ls when the mirror is placed at ‘PQ’ as shown.
feyrk tqyrk gksA tc niZ.k dks fuEukafdr fp= ds vuqlkj ‘PQ’ ij j[kk x;k gks] rc
fn, x, la;kstu dk niZ.k esa fufeZr lgh izfrfcac p;fur djsAa

(a) 1 (b) 2 (c) 3 (d) 4


Sol. (b)
10. Choose the alternative which is closely resembles the
mirror image of the given combination.
og fodYi pqusa tks fn, x, la;kstu dh niZ.k Nfo ls fudV ls Sol. (b)
feyrk tqyrk gksA 15. Which of the Option figure is the exact mirror image of the
given problem figure when the mirror is held to the right
side of the problem figure ?
tc niZ.k dks leL;k vk—fr ds nk;ha vksj j[kk tkrk gS] rks
fuEufyf[kr fodYi vkd`fr;ksa esa ls dkSu lh vkd`fr] nh xbZ leL;k
(a) 1 (b) 2 (c) 3 (d) 4 vkd`fr dh lVhd niZ.k Nfo gS\
Sol. (d)
11. Select the correct mirror image of the given combination
when the mirror is placed at PQ as shown.
niZ.k dks PQ ij j[kus ij fn, x, la;kstu dh lgh niZ.k Nfo dk
p;u djsa tSlk fd fn[kk;k x;k gSA
Sol. (d)
16. Select the correct mirror image of the given figure when
the mirror is placed at MN as shown below.
nh xbZ vk—fr dh lgh niZ.k Nfo dk p;u djsa tc niZ.k dks MN
ij j[kk x;k gS tSlk fd uhps fn[kk;k x;k gSA

Sol. (c)
12. Which of the Option figures is the exact mirror image of
the given figure when the mirror is held at the right side?
dkSu&lh fodYi vk—fr nh xbZ vk—fr dk lVhd niZ.k çfrfcEc gS
tc niZ.k dks nkfguh vksj j[kk tkrk gS\
#PaV$ArK@

Sol. (a)

193
Download Free PDFs & e-Books from Neon Classes App

18. Which of the Option figure is the exact mirror image of the M1
given problem figure when the mirror is held to the right
side of the problem figure ?
tc niZ.k dks leL;k vk—fr ds nk;ha vksj j[kk tkrk gS] rks
fuEufyf[kr fodYi vkd`fr;ksa esa ls dkSu lh vkd`fr] nh xbZ leL;k
vkd`fr dh lVhd niZ.k Nfo gS\ (X)
M2

(X)
(a) (b) (c) (d)
Sol. (a)
22. Which of the Option figure is the exact mirror image of the
given problem figure when the mirror is held to the left
side of the problem figure ?
(a) (b) (c) (d) tc niZ.k dks leL;k vk—fr ds cka;ha vksj j[kk tkrk gS] rks
Sol. (c) fuEufyf[kr fodYi vkd`fr;ksa esa ls dkSu lh vkd`fr] nh xbZ leL;k
19. Select the correct mirror image of the given figure when vkd`fr dh lVhd niZ.k Nfo gS\
the mirror is placed at MN as shown below. M1
nh xbZ vk—fr dh lgh niZ.k Nfo dk p;u djsa tc niZ.k dks MN
ij j[kk x;k gS tSlk fd uhps fn[kk;k x;k gSA

M2 (X)

M N
(X)

(a) (b) (c) (d)


Sol. (c)
(a) (b) (c) (d) 23. Select the correct mirror image of the given figure when
Sol. (d) the mirror is placed at MN as shown below.
20. Select the correct mirror image of the given figure when nh xbZ vk—fr dh lgh niZ.k Nfo dk p;u djsa tc niZ.k dks MN
the mirror is placed at MN as shown below. ij j[kk x;k gS tSlk fd uhps fn[kk;k x;k gSA
nh xbZ vk—fr dh lgh niZ.k Nfo dk p;u djsa tc niZ.k dks MN
ij j[kk x;k gS tSlk fd uhps fn[kk;k x;k gSA

M N
(X)

M N
(X)
M N M N M N M N

(a) (b) (c) (d)


Sol. (d)
(a) (b) (c) (d) 24. Which of the Option figure is the exact mirror image of the
Sol. (c) given problem figure when the mirror is held to the right
21. Which of the Option figure is the exact mirror image of the side of the problem figure ?
given problem figure when the mirror is held to the left tc niZ.k dks leL;k vk—fr ds nk;ha vksj j[kk tkrk gS] rks
side of the problem figure ? fuEufyf[kr fodYi vkd`fr;ksa esa ls dkSu lh vkd`fr] nh xbZ leL;k
tc niZ.k dks leL;k vk—fr ds cka;ha vksj j[kk tkrk gS] rks vkd`fr dh lVhd niZ.k Nfo gS\
fuEufyf[kr fodYi vkd`fr;ksa esa ls dkSu lh vkd`fr] nh xbZ leL;k
vkd`fr dh lVhd niZ.k Nfo gS\

(X)

194
Best App for Govt. Jobs : Neonclasses (Download Now)

28. Which of the Option figure is the exact mirror image of the
given problem figure when the mirror is held to the right
side of the problem figure ?
tc niZ.k dks leL;k vk—fr ds nk;ha vksj j[kk tkrk gS] rks
(a) (b) (c) (d) fuEufyf[kr fodYi vkd`fr;ksa esa ls dkSu lh vkd`fr] nh xbZ leL;k
Sol. (a) vkd`fr dh lVhd niZ.k Nfo gS\
25. Which of the Option figure is the exact mirror image of the
given problem figure when the mirror is held to the right
side of the problem figure ?
tc niZ.k dks leL;k vk—fr ds nk;ha vksj j[kk tkrk gS] rks
fuEufyf[kr fodYi vkd`fr;ksa esa ls dkSu lh vkd`fr] nh xbZ leL;k (X)
vkd`fr dh lVhd niZ.k Nfo gS\

(a) (b) (c) (d)


(X) Sol. (d)
29. Which of the Option figure is the exact mirror image of the
given problem figure when the mirror is held to the right
side of the problem figure ?
tc niZ.k dks leL;k vk—fr ds nk;ha vksj j[kk tkrk gS] rks
(a) (b) (c) (d) fuEufyf[kr fodYi vkd`fr;ksa esa ls dkSu lh vkd`fr] nh xbZ leL;k
Sol. (d) vkd`fr dh lVhd niZ.k Nfo gS\
26. Which of the Option figure is the exact mirror image of the
given problem figure when the mirror is held to the right
side of the problem figure ?
tc niZ.k dks leL;k vk—fr ds nk;ha vksj j[kk tkrk gS] rks
fuEufyf[kr fodYi vkd`fr;ksa esa ls dkSu lh vkd`fr] nh xbZ leL;k (X)
vkd`fr dh lVhd niZ.k Nfo gS\

(a) (b) (c) (d)


(X) Sol. (a)
30. Which of the Option figure is the exact mirror image of the
given problem figure when the mirror is held to the right
side of the problem figure ?
tc niZ.k dks leL;k vk—fr ds nk;ha vksj j[kk tkrk gS] rks
(a) (b) (c) (d) fuEufyf[kr fodYi vkd`fr;ksa esa ls dkSu lh vkd`fr] nh xbZ leL;k
Sol. (d) vkd`fr dh lVhd niZ.k Nfo gS\
27. Which of the Option figure is the exact mirror image of the
given problem figure when the mirror is held to the right
side of the problem figure ?
tc niZ.k dks leL;k vk—fr ds nk;ha vksj j[kk tkrk gS] rks
fuEufyf[kr fodYi vkd`fr;ksa esa ls dkSu lh vkd`fr] nh xbZ leL;k (X)
vkd`fr dh lVhd niZ.k Nfo gS\

(a) (b) (c) (d)


(X) Sol. (d)
31. Which of the Option figure is the exact mirror image of the
given problem figure when the mirror is held to the right
side of the problem figure ?
tc niZ.k dks leL;k vk—fr ds nk;ha vksj j[kk tkrk gS] rks
(a) (b) (c) (d) fuEufyf[kr fodYi vkd`fr;ksa esa ls dkSu lh vkd`fr] nh xbZ leL;k
Sol. (c) vkd`fr dh lVhd niZ.k Nfo gS\

195
Download Free PDFs & e-Books from Neon Classes App

35. Which of the answer figure is the mirror image of given


figure? which is based on AB.
dkSu lh mÙkj vkd`fr iz’u vkd`fr dk niZ.k izfrfcEc gS] tcfd ni.kZ
AB dh fLFkfr esa j[kk tkrk gSA
(X) Question figure/iz’u vkd`fr

(a) (b) (c) (d)


Sol. (b) Answer figure/ mÙkj vkd`fr
32. Which of the Option figure is the exact mirror image of the
given problem figure when the mirror is held to the right
side of the problem figure ?
tc niZ.k dks leL;k vk—fr ds nk;ha vksj j[kk tkrk gS] rks
fuEufyf[kr fodYi vkd`fr;ksa esa ls dkSu lh vkd`fr] nh xbZ leL;k
vkd`fr dh lVhd niZ.k Nfo gS\
Sol. (d)
36. If mirror is based on line LM, then which of the answer
figure is the mirror image of given figure?
;fn fdlh niZ.k dks js[kk LM ij j[kk tk,] rks nh xbZ mÙkj]
(X)
vkd`fr;ksa esa ls dkSu lh vkd`fr iz'u vkd`fr dk lgh izfrfcEc gksxh \
Question figure/iz’u vkd`fr

L
(a) (b) (c) (d)
Sol. (d)
33. Choose the correct mirror image of the given figure (X)
from amongst the four alternatives. M
pkj fodYiksa esa ls nh xbZ vk—fr (X) dh lgh niZ.k Nfo pquAsa Answer figure/ mÙkj vkd`fr

(X) (1) (2) (3) (4) (a) (b) (c) (d)


(a) 1 (b) 2 (c) 3 (d) 4 Sol. (b)
Sol. (d) 37. Which of the answer figure is the mirror image of given
34. Which of the Option figure is the exact mirror image of the figure? which is based on AB.
given problem figure when the mirror is held to the right dkSu lh mÙkj vkd`fr iz’u vkd`fr dk niZ.k izfrfcEc gS] tcfd niZ.k
side of the problem figure ? AB dh fLFkfr esa j[kk tkrk gSA
tc niZ.k dks leL;k vk—fr ds nk;ha vksj j[kk tkrk gS] rks Question figure/iz’u vkd`fr
fuEufyf[kr fodYi vkd`fr;ksa esa ls dkSu lh vkd`fr] nh xbZ leL;k A
vkd`fr dh lVhd niZ.k Nfo gS\
Question figure/iz’u vkd`fr
B
Answer figure/ mÙkj vkd`fr

(a) (b) (c) (d )


Answer figure/ mÙkj vkd`fr
Sol. (d)
38. If mirror is based on line MN, then which of the answer
figure is the mirror image of given figure?
;fn fdlh niZ.k dks MN js[kk ij j[kk tk,] rks nh xbZ mÙkj]
vkd`fr;ksa esa ls dkSu lh vkd`fr iz'u vkd`fr dk lgh izfrfcEc gksxh \
Question figure/iz’u vkd`fr
Sol. (a)

196
Best App for Govt. Jobs : Neonclasses (Download Now)

Sol. (b)
42. Select the correct mirror image of the given figure when
Answer figure/ mÙkj vkd`fr
the mirror is placed to the right of the figure.
;fn niZ.k dks vk—fr ds nk;ha vksj j[kk x;k gS] rks fuEufyf[kr vk—
fr ds niZ.k çfrfcEc dks igpkfu,A

Sol. (b)
39. If mirror is based on line MN, then which of the answer
figure is the mirror image of given figure?
;fn fdlh niZ.k dks MN js[kk ij j[kk tk,] rks nh xbZ mÙkj]
vkd`fr;ksa esa ls dkSu lh vkd`fr iz'u vkd`fr dk lgh izfrfcEc gksxh \
Sol. (c)
Question figure/iz’u vkd`fr
43. Select the correct mirror image of the given figure when
the mirror is placed at ‘AB’ as shown.
nh xbZ vk—fr ds lgh niZ.k çfrfcEc dk p;u djsa tc niZ.k dks
'AB' ij j[kk x;k gS tSlk fd fn[kk;k x;k gSA
A

Answer figure/ mÙkj vkd`fr R

Sol. (b) Sol. (b)


40. Which of the answer figure is the mirror image of 44. Select the correct mirror image of the given combination
given figure? which is based on MN. when the mirror is placed at MN as shown below.
dkSu lh mÙkj vkd`fr iz’u vkd`fr dk ni.kZ izfrfcEc gS] tcfd niZ.k dks MN ij j[kus ij fn, x, la;kstu dh lgh niZ.k Nfo dk
ni.kZ MN dh fLFkfr esa j[kk tkrk gSA p;u djsa tSlk fd uhps fn[kk;k x;k gSA
Question figure/iz’u vkd`fr

Answer figure/ mÙkj vkd`fr


Sol. (b)
45. Select the correct mirror image of the given figure when
the mirror is placed at ‘AB’ as shown.
tc niZ.k dks fuEukafdr fp= ds vuqlkj ‘AB’ ij j[kk x;k gS] rks nh
xbZ vkd`fr ds niZ.k esa fufeZr lgh izfrfcac dk p;u djsAa
A
Sol. (d)
41. Select the correct mirror image of the given figure when
the mirror is placed on the right of the figure.
nh xbZ vk—fr ds lgh niZ.k çfrfcEc dk p;u djsa tc niZ.k dks B
vk—fr ds nkbZa vksj j[kk tk,A

(b)
(a)

197
Download Free PDFs & e-Books from Neon Classes App

A
(c) (d)
c d
e
Sol. (b) b a
46. Select the correct mirror image of the given figure when a B
mirror is placed on the right side of the figure.
b ɔ d c d ɔ d c
nh xbZ vk—fr dh lgh niZ.k Nfo dk p;u djsa tc niZ.k dks vk—fr
ɘ e ɘ
ds nkbZa vksj j[kk tkrk gSA d a b a d a b
(a) (b) (c) (d)
Sol. (a)
51. Select the correct mirror image of the given figure when
the mirror is placed to the right side of the figure.
nh xbZ vk—fr dh lgh niZ.k Nfo dk p;u djsa tc niZ.k dks vk—
fr ds nkbZa vksj j[kk tk,A
Sol. (a) 3 8
47. Which of Select the correct mirror image of the given 9 6
figure when the mirror is placed to the right of the figure.
8 3
;fn niZ.k dks vk—fr ds nk;ha vksj j[kk x;k gS] rks fuEufyf[kr vk—
fr ds niZ.k çfrfcEc dks igpkfu,A

(a) (b) (c) (d)


Sol. (d)
52. Select the correct mirror image of the given figure when
the mirror is placed at AB as shown.
Sol. (b) nh xbZ vk—fr ds lgh niZ.k çfrfcEc dk p;u djsa tc niZ.k dks AB
48. Select the correct mirror image of the given figure when a ij j[kk x;k gS tSlk fd fn[kk;k x;k gSA
mirror is placed on the right of the figure
nh xbZ vk—fr ds lgh niZ.k çfrfcEc dk p;u djsa tc niZ.k dks
vk—fr ds nkbZa vksj j[kk tk,A

Sol. (a)
10. Missing Number
01. Study the given pattern carefully and select the number
Sol. (b)
that can replace the question mark (?) in it.
49. Select the correct mirror image of the given combination
fn, x, iSVuZ dk è;kuiwoZd vè;;u djsa vkSj ml la[;k dk p;u
when the mirror is placed at MN as shown below.
niZ.k dks MN ij j[kus ij fn, x, la;kstu dh lgh niZ.k Nfo dk djsa tks mlesa ç'uokpd fpà ¼\½ dks çfrLFkkfir dj lds
p;u djsa tSlk fd uhps fn[kk;k x;k gSA 3 18 6

27 ?

(a) 38 (b) 18 (c) 29 (d) 54


Sol. (d)
Pattern: - [Mid Number = XY]
Sol. (d)
50. If the mirror is placed as shown in the following figure, 3 3×6 6
choose the correct image formed in the mirror of the =18
given figure.
;fn niZ.k dks fuEu vkd`fr ds vuqlkj ‘AB’ ij j[kk tkrk gS] rks nh 3×9 6×9
xbZ vkd`fr dk niZ.k esa fufeZr lgh izfrfcac p;u djsAa
= 27 =54

198
Best App for Govt. Jobs : Neonclasses (Download Now)

Correct option (c) 52 + 22 = 74, 74 + 26 = 100


02. Study the given pattern carefully and select the number 100 + 30 = 130
that can replace the question mark (?) in it. Correct option (a)
fn, x, iSVuZ dk /;kuiwoZd vè;;u djs]a vkSj ml la[;k dk p;u 04. Study the given pattern carefully and select the number
djsa tks iz’uokpd fpUg (?) ds LFkku ij vk ldrh gSA that can replace the question mark (?) in it.
7 9 fn, x, iSVuZ dk /;kuiwoZd vè;;u djs]a vkSj ml la[;k dk p;u
djsa tks iz’uokpd fpUg (?) ds LFkku ij vk ldrh gSA
24 57 12 23
5 11 8 15

12 13 25 51 17 37 58 21

33 ? 31 19
3 3 7 21
5 22
(a) 58 (b) 70 (c) 62 (d) 68
Sol. (b)
Pattern : - 47 41 35 39 ? 27
A
(A×B)–C=D, 24 15
D
B C (a) 51 (b) 45 (c) 49 (d) 53
Sol. (c)
b
 (7×5) – 11  35 – 11 = 24
 (9×8) – 15  72 – 15 = 57
Pattern : - a e c [(a+b+d)–c =e]
 (12×3) – 3  36 – 3 = 33
Similarly:-
 (13×7) – 21  91 – 21 = 70 d
Correct option (b)  (25+12+31) – 17  68–17 = 51
03. Study the given pattern carefully and select the number  (37+23+19) – 21  79 – 21 = 58
that can replace the question mark (?) in it.  (47+5+24) – 35  76 – 35 = 41
fn, x, iSVuZ dk è;kuiwod
Z vè;;u djsa vkSj ml la[;k dk p;u djsa Similarly:-
tks mlesa ç'uokpd fpà ¼\½ dks çfrLFkkfir dj ldsA
 (39+22+15) – 27  76 – 27 = 49
05. Study the given pattern carefully and select the number
that can replace the question mark (?) in it
fn, x, iSVuZ dk è;kuiwod
Z vè;;u djsa vkSj ml la[;k dk p;u djsa
tks mlesa ç'uokpd fpà ¼\½ dks çfrLFkkfir dj lds

11
(a) 74 (b) 64 (c) 96 (d) 82 5125 28
Sol. (a) 3
Pattern:-
A + 6 = B, B + 10 = C, C + 14 = D ?
+4 +4
The pattern used here is column wise :- (a) 4126 (b) 444 (c) 464 (d) 446
Sol. (c)
Pattern : - Digit, (1st Digit)3
130 4 +4
100 10 N
+26

20 113 = 1, 2  23 = 8
+4 +4
34
52
11 28
+4 +4

or
4 + 6 = 10 , 10 + 10 = 20
20 + 14 = 34, 34 + 18 = 52

199
Download Free PDFs & e-Books from Neon Classes App

3 3
333 = 27, 5  53 = 125 7, 3, 370  7 + 3  343 + 27 = 370
Similarly:-
3 3
327 5125 6, 4, ?  6 + 4  216 + 64 = 280
Correct option (a)
Similarity:- Concept: Row Matrix: Cube: A3–B3 = C (Difference of cubes of
4  43 = 64 any 2 numbers out of 3 is related to third number)
08. In each of the given number-clusters, the number on the right
side of `=’ (the equal to sign) is calculated by performing
464
certain mathematical operations on the two numbers on the
1,13 left of = (the equal to sign). All three number-clusters follow
the same pattern. Select the number from among the given
11 2,23 options that can replace the question mark (?) in the third
5125 number-cluster.
28
3 fn, x, çR;sd la[;k&lewgksa es]a ¾ ¼fpà ds cjkcj½ ds nkbZa vksj dh
la[;k dh x.kuk ¾ ¼fpà ds cjkcj½ ds ckbZa vksj nks la[;kvksa ij dqN
? xf.krh; lapkyu djds dh tkrh gSA lHkh rhu la[;k&lewg leku
iSVuZ dk vuqlj.k djrs gSaA fn, x, fodYiksa esa ls og la[;k pqfu,
tks rhljs la[;k&lewg esa ç'uokpd fpUg ¼\½ dks çfrLFkkfir dj
ldsA
Correct option (c)
5, 3 = 98
Concept: Row Matrix: [ Triplet] [L2+ B2= K2]
7, 4 = 279
06. Study the given pattern carefully and select the number
9, 5 = ?
that can replace the question mark (?) in it.
(a) 904 (b) 604 (c) 854 (d) 754
fn, x, iSVuZ dk /;kuiwoZd vè;;u djs]a vkSj ml la[;k dk p;u
Sol. (b)
djsa tks iz’uokpd fpUg (?) ds LFkku ij vk ldrh gSA Pattern :- [A3–B3 = C]
9 7 12 3 3
5, 3 = 98  5 – 3  125 – 27 = 98
12 24 ? 3 3
7, 4 = 279  7 – 4  343 – 64 = 279
15 25 37 Similarly:-
3 3
(a) 42 (b) 35 (c) 36 (d) 40 9, 5  9 – 5  729 – 125 = 604
Sol. (b) Correct option (b)
Pattern : - [ Traplete] [L2+ B2= K2] Concept: Row Matrix: Cube: Cube roots of perfect cube
The pattern used here is column wise :- [A2+ B2 = C2] Numbers are operated
First column 09. Study the given pattern carefully and select the number
9, 12, 15  92 + 122 = 152 81 + 144 = 225 that can replace the question mark (?) in it.
225 = 225 fn, x, iSVuZ dk /;kuiwoZd vè;;u djs]a vkSj ml la[;k dk p;u
Second column djsa tks iz’uokpd fpUg (?) ds LFkku ij vk ldrh gSA
7, 24, 25  72+ 242 = 252 49 + 576 = 625 64 12 27
625 = 625 216 ? 343
Similarly:- 512 40 125
Third column (a) 42 (b) 72 (c) 64 (d) 95
 122+ X2 = 372 Sol. (a)
144 + X2 = 1369 The Pattern followed here is :- [ 3 a  3 c  b ]
X2 = 1369 – 144 3 64  3 27  4  3  12
X2 = 1225
512,40,125 3 512  3 125  8  5  40
X = 1225  35
Correct option (b) Similarly:- 216,?,343  3 216 3 343  6 7  42 Correct
Concept:Row Matrix: Cube: [A3+B3 = C] Sum of cubes of option (a)
two numbers is related to third number Concept:Row Matrix: Square: [A,B,C][A2–B2 = C] (Difference of
07. Find the missing number from the below options. squares of any two numbers is related to third number)
uhps fn, x, fodYiksa esa ls yqIr la[;k Kkr dhft,A 10. Find the missing number from the below options.
uhps fn, x, fodYiksa esa ls yqIr la[;k Kkr dhft,A
8 2 520 12 7 95
7 3 370 14 8 132
6 4 ? 16 9 ?
(a) 280 (b) 270 (c) 240 (d) 300 (a) 154 (b) 164 (c) 185 (d) 175
Sol. (a) Sol. (d)
Pattern :- [A3+B3 = C] Pattern :- [A,B,C][A2–B2 = C]
st 2 2
3 3
8, 2, 520  8 + 2  512 + 8 = 520 I row(12, 7, 95)  12 – 7  144 – 49 = 95

200
Best App for Govt. Jobs : Neonclasses (Download Now)

nd 2
II row(14, 8, 132)  14 – 8  196 – 64 = 132
2 (a) 50 (b) 55 (c) 45 (d) 40
Similarly:- Sol. (c)
rd 2 2 Pattern :- [A,B,C][A2+ C = B]
III row(16, 9, ?)  16 – 9  256 – 81 = 175
The Pattern followed here is :- [A,B,C] [A2+ C = B]
Correct option (d) st 2
I row(3, 13, 4)  3 + 4  9 + 4 = 13
Concept:Row Matrix: Square: [A,B,C][ A  B ]2 = C] ( here
nd 2
perfect squares are given in rows and columns and their II row(6, 67, 31)  6 + 31  36 + 31 = 67
roots is related ) Similarly:-
rd 2
11. Study the given pattern carefully and select the number III row(5, ?, 20)  5 + 20  25 + 20 = 45
that can replace the question mark (?) in it. Correct option (c)
fn, x, iSVuZ dk /;kuiwoZd vè;;u djs]a vkSj ml la[;k dk p;u Concept:Row Matrix: Square: (A+B+C), [(A-B) = C]
2

djsa tks iz’uokpd fpUg (?) ds LFkku ij vk ldrh gSA (difference of any two numbers is squared to get the third
25 49 144 the number
36 64 196 14. Study the given pattern carefully and select the number
121 225 ? that can replace the question mark (?) in it.
(a) 652 (b) 576 (c) 676 (d) 346 fn, x, iSVuZ dk è;kuiwod
Z vè;;u djsa vkSj ml la[;k dk p;u djsa
Sol. (c) tks mlesa ç’uokpd fpà ¼\½ dks çfrLFkkfir dj ldsA
Pattern :- [A,B,C][ A  B ]2 = C] First row: 8, 13, 25
Second row: 12, 7, 25
 
st 2 2 2
I row(25, 49, 144)  25  49  (5+7)  12 = 144
Third row: 9, 15, ?

  (NOTE: Operations should be performed on the whole


nd 2 2 2
II row(36, 64, 196)  36  64  (6+8)  14 =
numbers, without breaking down the number into its
196 constituent digits. For example, 13 – Operations on 13 such as
Similarly:- adding/deleting/multiplying etc. to 13 can be performed.

  Breaking down 13 into 1 and 3 and then performing


rd 2 2 2
III row(12, 225, ?)  121  225  (11+15)  26
mathematical operations on 1 and 3 is not allowed.)
= 676 ¼uksV: la[;kvksa dks mlds ?kVd vadksa esa foHkkftr fd, fcuk] iw.kZ
Correct option (c) la[;kvksa ij xf.krh; lafØ;k,a dh tkuh pkfg,A mnkgj.k ds fy,
Concept:Row Matrix: Square: [A,B,C][(A+1) 2+(B+1) 2 = C]( 13ds ekeys esa & 13 tksM+us @ gVkus @ xq.kk vkfn tSlh xf.krh;
Addition of Squares of numbers succeeding or preceding lafØ;k,W 13 ij dh tkr ldrh gSA 13 dks 1 vkSj esa rksM+dj vkSj
the numbers is related to third number) fQj 1 vkSj 3 ij xf.krh; lafØ;k,a djus dh vuqefr ugha gSA
12. Study the given pattern carefully and select the number (a) 36 (b) 30 (c) 16 (d) 49
that can replace the question mark (?) in it. Sol. (a)
fn, x, iSVuZ dk /;kuiwoZd v/;;u djsa vkSj ml la[;k dk p;u 2
Pattern:- (A+B+C), [(A-B) = C]
djsa tks mlesa ç'uokpd fpà (?) dks çfrLFkkfir dj ldsA st 2 2
5 7 100 I row  (8, 13, 25)  (8 –13)  5 = 25
nd 2 2
8 9 181 II row (12, 7, 25)(12 –7) 5 = 25
11 10 ? Similarly:-
(a) 260 (b) 244 (c) 255 (d) 265 rd
III row (9, 15, ?)(9 –15)  6 = 36
2 2

Sol. (d) Correct option (a)


Pattern :- [A,B,C][(A+1) 2+(B+1) 2 = C] 2 2
st 2 2 2 2 Concept:Row Matrix: Square: (A,B,C), [(A+B) – (A–B) = C](
I row(5, 7, 100)  (5+1) +(7+1)  6 8  36 + 64 = in a row The difference of Square of sum and difference is
100 equal to the third number)
nd 2 2 2 2
II row(8, 9, 181)  (8+1) +(9+1)  9 10  81 + 100 15. Study the given pattern carefully and select the number
= 181 that can replace the question mark (?) in it.
Similarly:- fn, x, iSVuZ dk /;kuiwoZd v/;;u djsa vkSj ml la[;k dk p;u
rd 2
III row(11,10, ?)  (11+1) +(10+1)  12 11  144
2 2 2 djsa tks mlesa ç'uokpd fpà (?) dks çfrLFkkfir dj ldsA
+ 121 = 265 8 7 224
Correct option (d) 11 13 572
Concept: Row Matrix: Square: [A,B,C][A2+ C = B] (Square of 12 15 ?
a number is related to a second number to achieve the (a) 360 (b) 720 (c) 456 (d) 320
third number.) Sol. (b)
2 2
13. Study the given pattern carefully and select the number Pattern:- (A,B,C), [(A+B) – (A–B) = C]
that can replace the question mark (?) in it. st 2
I row  (8, 7, 224)  (8 + 7) – (8 – 7)  15 – 1  225 –
2 2 2

fn, x, iSVuZ dk è;kuiwoZd vè;;u djsa vkSj ml la[;k dk p;u 1 = 224


djsa tks mlesa ç'uokpd fpà ¼\½ dks çfrLFkkfir dj lds nd 2 2 2 2
II row  (11, 13, 572)  (11 + 13) – (11 – 13)  24 – 2
3 13 4
6 67 31  576 – 4 = 572
5 ? 20 Similarly:-

201
Download Free PDFs & e-Books from Neon Classes App

rd 2
III row  (12, 15, ?)  (12 + 15) – (12 – 15)  27 – 3 
2 2 2 Pattern :- [Series][(a,b,c), [a, a+2, a+6]]
+6
729 – 9 = 720 +2
Correct option (b) Ist row(11, 13, 17) 11, 13, 17
2 +6
Concept: Row Matrix: Square: (A,B,C), [A = BC] square of +2
any of the number in a matrix is equal to multiplication of IInd row(33, 35, 39) 33, 35, 39
other two numbers
Similarity:- +6
16. Study the given pattern carefully and select the number +2
rd
that can replace the question mark (?) in it. III row(189, 191, ?) 189, 191, 195
fn, x, iSVuZ dks /;ku ls i<+as vkSj ml la[;k dk p;u djs]a tks blesa Correct option (d)
iz’uokpd fpgu (?) ds LFkku ij vk ldrh gSaA B2  C2
12 9 16 Concept: Row Matrix: Square: [A,B,C][  A ]( Sum of
2
9 3 27
squares of any two number is divided by any number)
14 ? 28 19. Study the given pattern carefully and select the number
(a) 11 (b) 8 (c) 7 (d) 9 that can replace the question mark (?) in it.
Sol. (c) fn, x, iSVuZ dk è;kuiwoZd vè;;u djsa vkSj ml la[;k dk p;u
2
Pattern:- (A,B,C), [A = BC] djsa tks mlesa ç'uokpd fpà ¼\½ dks çfrLFkkfir dj lds
st 2
I row  (12, 9, 16)  12 = 9 × 16  144 = 144 45 9 3
nd 2
II row (9, 3, 27)9 = 3 × 27 81 = 81 122 10 12
Similarly:- 116 6 ?
rd 2 (a) 16 (b) 12 (c) 14 (d) 10
III row (14, ?, 28)14 = ? × 28  196 = ? × 28  ? =
Sol. (c)
196
7 B2  C2
28 Pattern :- [A,B,C][ A ]
2
Correct option (c)
Concept: Row Matrix: Square: [A,B,C][(C–A)2– 1 = B st 92  32 81  9 90
I row(45,9,3)    45
(Difference of two number is squared and operated to 2 2 2
nd
achieve the third number) II row (45, 9, 3) 
17. Study the given pattern carefully and select the number
102  122 100  144 244
that can replace the question mark (?) in it.    122
2 2 2
fn, x, iSVuZ dk è;kuiwoZd vè;;u djsa vkSj ml la[;k dk p;u
djsa tks mlesa ç'uokpd fpà ¼\½ dks çfrLFkkfir dj lds Similarly:-
rd
7 35 13 III row(116,6, ?) 
9 24 14 62  ? 2 2 2 2
 116  36 + ? = 116 × 2  36 + ? = 232  ? =232
13 120 ? 2
(a) 24 (b) 20 (c) 28 (d) 26 2
– 36  ? = 196  ? = 196 = 14
Sol. (a)
2
Correct option (c)
Pattern :- [A,B,C][(C–A) – 1 = B]
st 2 2 Concept: Row Matrix: Square: in a row (A,B,C), [A, B, a  b
I row(7, 35, 13)  (13–7) – 1  6 –1  36 – 1 = 35
] (Root of a number is multiplied to another number is
nd 2 2
II row(9, 24, 14)  (14–9) –1  5 –1 25 – 1 = 24 equal to the third number)
Similarly:- 20. Study the given pattern carefully and select the number
rd 2
III row(13,120,?) (?–13) – 1 = 120 (? – 13) = 120 +
2 that can replace the question mark (?) in it.
2 fn, x, iSVuZ dk è;kuiwod
Z vè;;u djsa vkSj ml la[;k dk p;u djsa
1  (? –13) 121 ? – 13 = 121  ? – 13 = 11  ? = 11 +
tks mlesa ç’uokpd fpà ¼\½ dks çfrLFkkfir dj ldsA
13 = 24
2 2
First row:16, 12, 48
or (24–13) –1  (11) –1 121 – 1 = 120 Second row: 9, 8, 24
Correct option (a) Third row:4,24 ?
Concept: Row Matrix: Square: [Series][(a,b,c), [a, a+k, (NOTE: Operations should be performed on the whole
a+2k]] is a AP pattern , there can also be series relation in numbers, without breaking down the number into its
terms of GP and other series progressions constituent digits. For example, 13 – Operations on 13
18. Study the given pattern carefully and select the number such as adding/deleting/multiplying etc. to 13 can be
that can replace the question mark (?) in it. performed. Breaking down 13 into 1 and 3 and then
fn, x, iSVuZ dk è;kuiwoZd vè;;u djsa vkSj ml la[;k dk p;u performing mathematical operations on 1 and 3 is not
djsa tks mlesa ç'uokpd fpà ¼\½ dks çfrLFkkfir dj lds allowed.)
11 13 17 ¼uksV: la[;kvksa dks mlds ?kVd vadksa esa foHkkftr fd, fcuk] iw.kZ
33 35 39 la[;kvksa ij xf.krh; lafØ;k,a dh tkuh pkfg,A mnkgj.k ds fy, 13
189 191 ? ds ekeys esa & 13 tksM+us @ gVkus @ xq.kk vkfn tSlh xf.krh;
(a) 217 (b) 203 (c) 199 (d) 195 lafØ;k,W 13 ij dh tkr ldrh gSA 13 dks 1 vkSj esa rksM+dj vkSj
Sol. (d) fQj 1 vkSj 3 ij xf.krh; lafØ;k,a djus dh vuqefr ugha gSA

202
Best App for Govt. Jobs : Neonclasses (Download Now)

(a) 48 (b) 46 (c) 50 (d) 44 (a) 148 (b) 131 (c) 141 (d) 137
Sol. (a) Sol. (c)
Pattern:- (A,B,C), [A, B, a b ] Pattern:- (A,B,C), [A + B = C]
st
st
I row  (16, 12, 48)  16, 12, 16  12  4 × 12 = 48 I row  (86, 72, 158)  86 + 72 = 158
nd
nd II row (98, 111, 209)98 + 111 = 209
II row (9, 8, 24)9, 8, 9  8  3 × 8 = 24
Similarly:-
Similarly:- rd
rd III row (76, 65, ?)76 + 65 = 141
III row (4, 24, ?) 4, 24, 4  24  2 × 24 = 48
Correct option (c)
Correct option (a)
Concept: Row Matrix: Cube & Square: (A,B,C), [A + C = B]
3 2 
Concept: Row Matrix: Addition: (A,B,C), [ A,B, 3 A  B ] 
(Cube of a number is added to square of another number 23. Study the given pattern carefully and select the number
21. Study the given pattern carefully and select the number that can replace the question mark (?) in it.
that can replace the question mark (?) in it. fn, x, iSVuZ dk /;kuiwoZd v/;;u djsa vkSj ml la[;k dk p;u
fn, x, iSVuZ dk è;kuiwod
Z vè;;u djsa vkSj ml la[;k dk p;u djsa djsa tks mlesa ç'uokpd fpà (?) dks çfrLFkkfir dj ldsA
tks mlesa ç’uokpd fpà ¼\½ dks çfrLFkkfir dj ldsA 125 25 10
First row- 3, 43, 4 216 49 13
Second row- 6, 241, 5 27 121 ?
Third row- 2, ?, 11 (a) 16 (b) 8 (c) 12 (d) 14
(NOTE: Operations should be performed on the whole Sol. (d)
numbers, without breaking down the numbers into its
constituent digits. E.g. 13 – Operations on 13 such as

Pattern:- (A,B,C), [ A,B, 3 A  B ] 
st
adding /deleting /multiplying etc. to 13 can be performed. I row  (125, 25, 10)  3 125  25  5  5  10
Breaking down 13 into 1 and 3 and then performing nd
II row (216, 49, 13) 3 216  49  6  7  13
mathematical operations on 1 and 3 is NOT allowed)
¼uksV: la[;kvksa dks mlds ?kVd vadksa esa foHkkftr fd, fcuk] iw.kZ Similarly:-
rd
la[;kvksa ij xf.krh; lafØ;k,a dh tkuh pkfg,A mnkgj.k ds fy, 13 III row (27, 121, ?) 3 27  121  3  11  14
ds ekeys esa & 13 tksM+us @ gVkus @ xq.kk vkfn tSlh xf.krh; Correct option (d)
lafØ;k,W 13 ij dh tkr ldrh gSA 13 dks 1 vkSj esa rksM+dj vkSj Concept: Row Matrix: Addition: (A,B,C), [ A  3 B  C ]
fQj 1 vkSj 3 ij xf.krh; lafØ;k,a djus dh vuqefr ugha gSA (Sum of square roots and cube roots is equal to third
(a) 173 (b) 192 (c) 137 (d) 129 number)
Sol. (d) 24. Study the given pattern carefully and select the number
3 2
Pattern:- (A,B,C), [A + C = B] that can replace the question mark (?) in it.
st 3 2
I row  (3, 43, 4)  3  4  27 + 16 = 43
fn, x, iSVuZ dk /;kuiwoZd v/;;u djsa vkSj ml la[;k dk p;u
nd 3 2 djsa tks mlesa ç'uokpd fpà (?) dks çfrLFkkfir dj ldsA
II row (6, 241, 5)6 5  216 + 25 = 241
16 27 7
Similarly:-
rd 3 2
25 8 7
III row (2, ?, 11)2 + 11  8 + 121 = 129 121 ? 12
Correct option (d) (a) 4 (b) 1 (c) 7 (d) 16
Concept: Row Matrix: Addition: (A,B,C), [A + B = C]( Sum of Sol. (b)
any 2 numbers is equal to the third number)
22. Study the given pattern carefully and select the number Pattern:- (A,B,C), [ A  3 B  C ]
st
that can replace the question mark (?) in it. I row  (16, 27, 7)  16  3 27  4  3  7
fn, x, iSVuZ dk è;kuiwod
Z vè;;u djsa vkSj ml la[;k dk p;u djsa nd
II row (25, 8, 7)
tks mlesa ç’uokpd fpà ¼\½ dks çfrLFkkfir dj ldsA
25  3 8  5  2  7
First row: 86, 72, 158
Second row: 98, 111, 209 Similarly:-
rd
Third row: 76, 65, ? III row (121, ?, 12)
(NOTE: Operations should be performed on the whole 121  3 ?  12
numbers, without breaking down the number into its
 11  1  12
constituent digits. For example, 13 – Operations on 13
Correct option (b)
such as adding/deleting/multiplying etc. to 13 can be
Concept: Row Matrix: Addition: [A,B,C],[(A+B+1)×4]
performed. Breaking down 13 into 1 and 3 and then
25. Study the given pattern carefully and select the number
performing mathematical operations on 1 and 3 is not
that can replace the question mark (?) in it.
allowed.)
fn, x, iSVuZ dk è;kuiwod
Z vè;;u djsa vkSj ml la[;k dk p;u djsa
¼uksV: la[;kvksa dks mlds ?kVd vadksa esa foHkkftr fd, fcuk] iw.kZ
tks mlesa ç’uokpd fpà ¼\½ dks çfrLFkkfir dj ldsA
la[;kvksa ij xf.krh; lafØ;k,a dh tkuh pkfg,A mnkgj.k ds fy, 13
First row- 9, 21, 124
ds ekeys esa & 13 tksM+us @ gVkus @ xq.kk vkfn tSlh xf.krh;
Second row- 11, 25, 148
lafØ;k,W 13 ij dh tkr ldrh gSA 13 dks 1 vkSj esa rksM+dj vkSj Third row- 17, ?, 220
fQj 1 vkSj 3 ij xf.krh; lafØ;k,a djus dh vuqefr ugha gSA

203
Download Free PDFs & e-Books from Neon Classes App

(NOTE: Operations should be performed on the whole fn, x, iSVuZ dks /;ku ls i<+sa vkSj ml la[;k dk p;u djs]a tks
numbers, without breaking down the numbers into its blesa iz’uokpd fpgu (?) ds LFkku ij vk ldrh gSaA
constituent digits. E.g. 13 – Operations on 13 such as 4507 7077 37
adding /deleting /multiplying etc. to 13 can be performed. 2503 3067 26
Breaking down 13 into 1 and 3 and then performing 3307 4061 ?
mathematical operations on 1 and 3 is NOT allowed)
(a) 72 (b) 67 (c) 24 (d) 45
¼uksV: la[;kvksa dks mlds ?kVd vadksa esa foHkkftr fd, fcuk] iw.kZ
Sol. (c)
la[;kvksa ij xf.krh; lafØ;k,a dh tkuh pkfg,A mnkgj.k ds fy, 13 Pattern:- (A,B,C), [Sum Digit A + Sum Digit B = C]
ds ekeys esa & 13 tksM+us @ gVkus @ xq.kk vkfn tSlh xf.krh; st
I row  (4507, 7077, 37)  (4 + 5 + 0 + 7) + (7 + 0 + 7 + 7 )
lafØ;k,W 13 ij dh tkr ldrh gSA 13 dks 1 vkSj esa rksM+dj vkSj
fQj 1 vkSj 3 ij xf.krh; lafØ;k,a djus dh vuqefr ugha gSA 16 + 21 = 37
nd
(a) 34 (b) 30 (c) 35 (d) 37 II row (2503, 3067, 26) (2 + 5 + 0 + 3) + (3 + 0 + 6 + 7
Sol. (d) ) 10 + 16 = 26
Pattern:- [A,B,C],[(A+B+1)×4] Similarly:-
st
I row  (9, 21, 124)  (9+21+1)×4  31 × 4 = 124 rd
III row (3307, 4061, ?)  (3 + 3 + 0 + 7) + (4 + 0 + 6 + 1 )
nd
II row (11, 25, 148)  (11+25+1)×4  37 × 4 = 148 13 + 11 = 24
Similarly:- Correct option (c)
rd
III row (17, ?, 220)  (17+?+1)×4 =220  (17+?+1) = A
Concept: Row Matrix: Addition: (A,B,C), [ BC ]
220 B
 (17+?+1) = 55  18 + ? = 55  ? = 55–18 = 37
4 29. Select the set in which the numbers are related in the
Correct option (d) same way as are the numbers of the following set.
Concept: Row Matrix: Addition: (A,B,C), [A + B = C × 9] ml leqPp; dk p;u djsa ftlesa la[;k,¡ mlh çdkj lacfa èkr gSa tSls
26. Find the missing number from the below options. fuEufyf[kr leqPp; dh la[;k,¡ gSaA
uhps fn, x, fodYiksa esa ls yqIr la[;k Kkr dhft,A (56, 8, 15)
50 31 9 (32, 16, 18)
43 11 6 (NOTE : Operations should be performed on the whole
42 21 ? numbers, without breaking down the numbers into its
constituent digits. E.g. 13 – Operations on 13 such as
(a) 9 (b) 5 (c) 7 (d) 6
adding /deleting /multiplying etc. to 13 can be performed.
Sol. (c)
Breaking down 13 into 1 and 3 and then performing
Pattern:- (A,B,C), [A + B = C × 9]
st
mathematical operations on 1 and 3 is NOT allowed)
I row  (50, 31, 9)  50 + 31 = 9 × 9  81 = 81 ¼uksV: la[;kvksa dks mlds ?kVd vadksa esa foHkkftr fd, fcuk] iw.kZ
nd
II row (43, 11, 6) 43 + 11 = 6 × 9  54 = 54 la[;kvksa ij lapkyu fd;k tkuk pkfg,A mnkgj.k ds fy, 13 & 13
Similarly:- ij lapkyu tSls fd tksM+us @ gVkus @ xq.kk djus vkfn dks 13 esa
rd
III row (42, 21, ?)  42 + 21 = ? × 9  63 = ? × 9 fd;k tk ldrk gSA 13 dks 1 vkSj 3 esa rksM+dj vkSj fQj 1 vkSj 3
63 ij xf.krh; lafØ;kvksa dks djus dh vuqefr ugha gS½
?= =7 (a) (75, 5, 15) (b) (70, 5, 13)
9
(c) (66, 6, 17) (d) (68, 4, 17)
Correct option (c)
Sol. (c)
Concept: Row Matrix: Addition: Sum of all three Numbers
is equal in each set. A
Pattern:- (A,B,C), [ BC ]
27. Study the given pattern carefully and find the missing B
number [ADD. NO =20] st 56
I row  (56, 8, 15)   8  7  8  15
fn, x, iSVuZ dk /;kuiwoZd v/;;u djsa vkSj yqIr la[;k Kkr djsAa 8
(9, 9, 2) (8, ___, 1) (4, 8, 8) nd 32
(a) 11 (b) 8 (c) 6 (d) 1 II row (32, 16, 18)   16  2  16  18
16
Sol. (a)
Similarly:-
Pattern:- [Sum row =20]
rd 66
(9, 9, 2)  9 + 9 + 2 = 20 III row (66, 6, 17)   6  11  6  17
6
(4, 8, 8)4 + 8 + 8 = 20 Correct option (c)
Similarly:- Concept: Row Matrix: Addition: [A+C = B+ D] Sum of first
(8, ?, 1)8 + 1 + ? = 20  9 + ? = 20  ? = 20 – 9 = 11 and third number is equal to second and fourth number
Correct option (a) 30. Select the option that can replace the question mark (?) in
Concept: Row Matrix: Addition: (A,B,C), [Sum Digit A + Sum the third row.
Digit B = C]( When Sum of digits of two numbers is ml fodYi dk p;u djsa tks rhljh iafä esa ç'u fpà ¼\½ dks
operated is equal to the third number) çfrLFkkfir dj ldrk gS
28. Study the given pattern carefully and select the number Row 1 : 26, 17, 14, 23
that can replace the question mark (?) in it. Row 2 : 17, 12, ?, 9
Row 3 : 29, 32, 21, 18

204
Best App for Govt. Jobs : Neonclasses (Download Now)

(a) 19 (b) 2 (c) 4 (d) 7 13 30 34


Sol. (c) 27 ? 14
Pattern:- [A+C = B+ D] 19 30 22
st
I row  (26, 17, 14, 23) 26 + 14 = 17 + 23  40 = 40 (a) 30 (b) 34 (c) 32 (d) 36
nd
II row (29, 32, 21, 18) 29 + 21 = 32 + 18  50 = 50 Sol. (b)
Similarly:- Pattern:- (A,B,C), [2(A–B) =C]
st
rd
III row (17, 12, ?, 9) 17 + ? = 12 + 9  17 + ? = 21  ? I row  (13, 30, 34) 2(13 – 30)  2 × 17 = 34
nd
21 –- 17 = 4 II row ( 19, 30, 22)  2(19 – 30)  2 × 11 = 22
Correct option (c) Similarly:-
3 2 rd
Concept: Row Matrix: Difference: (A,B,C), [A – B = C] III row (27, ?, 14)  2(27 – 34)  2 × 7 = 14
(Difference of cube of a number to square of another Correct option (b)
number is equal to third number) Concept:Row Matrix: Multiplication: (A,B,C), [(A × B) – 3 =
31. Study the given matrix carefully and select the number C] (A number is subtracted from multiplication of two
from among the given options that can replace the numbers)
question mark (?) in it. 34. Select the number that can replace the question mark (?)
fn, x, eSfVªDl dk è;kuiwoZd vè;;u djsa vkSj fn, x, fodYiksa esa ls in the given table.
ml la[;k dk p;u djsa tks mlesa ç'u fpà ¼\½ dks çfrLFkkfir dj ml la[;k dk p;u djsa tks nh xbZ rkfydk esa ç'u fpà ¼\½ dks
ldsA çfrLFkkfir dj ldrh gSA
7 13 174 5 16 77
9 25 104 8 19 149
11 30 ? 13 12 ?
(a) 431 (b) 335 (c) 100 (d) 129 (a) 153 (b) 157 (c) 155 (d) 151
Sol. (a) Sol. (a)
3 2
Pattern:- (A,B,C), [A – B = C] Pattern:- (A,B,C), [(A × B) – 3 = C]
st
st 3 2
I row  (7, 13, 174)  7 – 13  343 – 169 = 174 I row  (5, 16, 77)  (5 × 16) – 3  80 – 3 = 77
nd
nd 3 2
II row (9, 25, 104)  9 – 25  729 – 625 = 104 II row (8, 19, 149)(8 × 19) – 3  152 – 3 = 149
Similarly:- Similarly:-
rd
rd 3 2
III row (11, 30, ?)  11 – 30  1331 – 900 = 431 III row (13, 12, ?)(13 × 12) – 3  156 – 3 = 153
Correct option (a) Correct option (a)
Concept: Row Matrix: Multiplication: (A,B,C), [(A–1)×C = B]
Concept: Row Matrix: Difference: (A,B,C), [2(A–B)2 =C]
successor of a number is multiplied with another number
Difference of any two numbers is doubled and then
is equal to the third number
squared
35. Study the given pattern carefully and select the number
32. Study the given pattern carefully and select the number
that can replace the question mark (?) in it.
that can replace the question mark (?) in it.
fn, x, iSVuZ dk /;kuiwoZd v/;;u djsa vkSj ml la[;k dk p;u fn, x, iSVuZ dks /;ku ls i<+as vkSj ml la[;k dk p;u djs]a tks blesa
djsa tks mlesa ç'uokpd fpà (?) dks çfrLFkkfir dj ldsA iz’uokpd fpgu (?) ds LFkku ij vk ldrh gSaA
15 196 14
12 16 32
18 255 ?
24 36 288
7 96 16
37 49 ?
(a) 8 (b) 12 (c) 15 (d) 13
(a) 188 (b) 178 (c) 278 (d) 288
Sol. (c)
Sol. (d)
Pattern:- (A,B,C), [(A–1)×C = B]
Pattern:- (A,B,C), [2(A–B)2 =C] st
st I row  (15, 196, 14) (15 – 1) × 14  14 × 14 = 196
I row  (12, 16, 21) 2(12 – 16)2  2 × 42  2 × 16 = 32 nd
nd II row ( 7, 96, 16)  (7 – 1) × 16  6 × 16 = 96
II row ( 24, 36, 288)  2(24 – 36)2  2 × 122  2 ×
Similarly:-
144 = 288 rd
Similarly:- III row (18, 255, ?)  (18 – 1) × ? = 255  17 × ? = 255
rd 255
III row (37, 49, ?)  2(37 – 49)2  2 × 122  2 × 144 = ?= = 15
288 17
Correct option (d) Correct option (c)
Concept: Row Matrix: Difference: (A,B,C), [n(A–B)=C] Concept:Row Matrix: Multiplication: (A,B,C), [ (A + c)×2 =
Difference of any two number’s multiple is equal to the B] (sum of number is multiplied)
third number 36. Find the missing number from the below options.
33. Study the given pattern carefully and select the number uhps fn, x, fodYiksa esa ls yqIr la[;k Kkr dhft,A
that can replace the question mark (?) in it. 19 78 20
fn, x, iSVuZ dk /;kuiwoZd vè;;u djs]a vkSj ml la[;k dk p;u 25 144 47
djsa tks iz’uokpd fpUg (?) ds LFkku ij vk ldrh gSA 16 ? 13

205
Download Free PDFs & e-Books from Neon Classes App

(a) 58 (b) 76 (c) 29 (d) 96 17 120 ?


Sol. (a) (a) 11 (b) 13 (c) 9 (d) 16
Pattern:- (A,B,C), [ (A + c)×2 = B] Sol. (b)
st
I row  (19, 78, 20)  (14 + 7)×2  39 × 2 = 78 Pattern:- (A,B,C), [(A + C) × (A – C) = B]
st
nd
II row (25, 144, 47)(25 + 47)×2  72 × 2 = 144 I row  (15, 81, 12)  (15 + 12) × (15 – 12)  27 × 3 = 81
nd
Similarly:- II row (18, 99, 15)(18 + 15) × (18 – 15)  33 × 3 = 99
rd
III row (16, ?, 13) (16 + 13)×2  29 × 2 = 58 Similarly:-
rd
Correct option (a) III row (17, 120, ?)(17 + ?) × (17 – ?) = 120
Concept: Row Matrix: Multiplication: (A,B,C), [n(A × B) = C],  (17 + 13) × (17 – 13)
n is any number . ( Multiplication of the numbers is
Multiplied is equal to third number .)
30 × 4 = 120
37. Study the given pattern carefully and select the number Correct option (b)
that can replace the question mark (?) in it. Concept: Row Matrix: Multiplication: (A,B,C,D), [A×C = B×D]
fn, x, iSVuZ dk /;kuiwoZd v/;;u djsa vkSj ml la[;k dk p;u multiplication of any two numbers is equal to remaining
two numbers
djsa tks mlesa ç'uokpd fpà (?) dks çfrLFkkfir dj ldsA
40. Study the given pattern carefully and select the number
14 7 196
that can replace the question mark (?) in it.
15 6 ? fn, x, iSVuZ dk è;kuiwod
Z vè;;u djsa vkSj ml la[;k dk p;u djsa
21 7 294 tks mlesa ç'uokpd fpà ¼\½ dks çfrLFkkfir dj ldsA
(a) 180 (b) 199 (c) 253 (d) 144 10 4 14 35
Sol. (a) 14 7 6 12
Pattern:- (A,B,C), [2(A × B) = C] 18 ? 8 16
st
I row  (14, 7, 196)  2(14 × 7)  2 × 98 = 196 (a) 6 (b) 7 (c) 9 (d) 8
nd
II row (21, 7, 294)2(21 × 7)  2 × 147 = 294 Sol. (c)
Similarly:- Pattern:- (A,B,C,D), [A × C = B × D]
st
rd
III row (15, 6, ?)2 (15 × 6)  2 × 90 = 180 I row  (10, 4, 14, 35) 10 × 14 = 4 × 35  140 = 140
nd
Correct option (a) II row (14, 7, 6, 12) 14 × 6 = 7 × 12  84 = 84
Concept: Row Matrix: Multiplication: (A,B,C), [(A + 2) × (B + Similarly:-
2) = C]( Succeeders of two numbers are multiplied) rd 144
III row (18, ?, 8, 16) 18 × 8 = 16 × ?  144 = 9
38. Study the given pattern carefully and select the number 16
that can replace the question mark (?) in it. Correct option (c)
fn, x, iSVuZ dk /;kuiwoZd v/;;u djsa vkSj ml la[;k dk p;u Concept: Row Matrix: Multiplication: (A,B,C), [nA + mB =
djsa tks mlesa ç'uokpd fpà (?) dks çfrLFkkfir dj ldsA C]( difference of multiple of any two numbers is equal to
7 14 144 third number)
11 16 234 41. Study the given pattern carefully and select the number
12 ? 266 that can replace the question mark (?) in it.
(a) 17 (b) 18 (c) 19 (d) 20 fn, x, iSVuZ dk è;kuiwod
Z vè;;u djsa vkSj ml la[;k dk p;u djsa
Sol. (a) tks mlesa ç’uokpd fpà ¼\½ dks çfrLFkkfir dj ldsA
Pattern:- (A,B,C), [(A + 2) × (B + 2) = C] First row:7, 9, 39
st
I row  (7, 14, 144)  (7 + 2) × (14 + 2)  9 × 16 = 144 Second row:4, 8, 28
nd Third row:6, 7, ?
II row (11, 16, 234)(11 + 2) × (16 + 2)  13 × 18 = (NOTE: Operations should be performed on the whole
234 numbers, without breaking down the number into its
Similarly:- constituent digits. For example, 13 – Operations on 13
rd
III row (12, ?, 266)(12 + 2) × (? + 2)  14 × (? + 2) = such as adding/deleting/multiplying etc. to 13 can be
266 performed. Breaking down 13 into 1 and 3 and then
266 ? + 2 =  19  ? = 19 – 2 = 17 performing mathematical operations on 1 and 3 is not
14
allowed.)
Correct option (a)
Concept: Row Matrix: Multiplication: (A,B,C), [(A + C) × (A –
¼uksV: la[;kvksa dks mlds ?kVd vadksa esa foHkkftr fd, fcuk] iw.kZ
C) = B]( Multiplication of sum and differences of two la[;kvksa ij xf.krh; lafØ;k,a dh tkuh pkfg,A mnkgj.k ds fy, 13
numbers is equal to the third number) ds ekeys esa & 13 tksM+us @ gVkus @ xq.kk vkfn tSlh xf.krh;
39. Study the given pattern carefully and select the number lafØ;k,W 13 ij dh tkr ldrh gSA 13 dks 1 vkSj esa rksM+dj vkSj
from among the given options that can replace the fQj 1 vkSj 3 ij xf.krh; lafØ;k,a djus dh vuqefr ugha gSA
question mark (?) in it.15 (a) 32 (b) 38 (c) 40 (d) 37
fn, x, iSVuZ dk è;kuiwoZd vè;;u djsa vkSj fn, x, fodYiksa esa ls Sol. (a)
ml la[;k dk p;u djsa tks mlesa ç'u fpà ¼\½ dks çfrLFkkfir dj Pattern:- (A,B,C), [3A + 2B = C]
st
ldsA I row  (7, 9, 39)  3 × 7 + 2 × 9  21 + 18 = 39
15 81 12 nd
II row (4, 8, 28) 3 × 4 + 2 × 8  12 + 16 = 28
18 99 15

206
Best App for Govt. Jobs : Neonclasses (Download Now)

Similarly:- fn, x, iSVuZ dk è;kuiwod


Z vè;;u djsa vkSj ml la[;k dk p;u djsa
rd
III row (6, 7, ?)  3 × 6 + 2 × 7  18 + 14 = 32 tks mlesa ç’uokpd fpà ¼\½ dks çfrLFkkfir dj ldsA
Correct option (a) First row - 12, 25, 19
Concept: Row Matrix: Multiplication: (A,B,C), [A +( B×4+1) Second row - 35, 50, 60
= C] : (A Number is added to multiple of another number is Third row - 24, 70, ?
equal to third number (NOTE : Operations should be performed on the whole
42. Study the given pattern carefully and select the number numbers, without breaking down the numbers into its
that can replace the question mark (?) in it. constituent digits. E.g. 13 – Operations on 13 such as
fn, x, iSVuZ dk è;kuiwod
Z vè;;u djsa vkSj ml la[;k dk p;u djsa adding /deleting /multiplying etc. to 13 can be performed.
tks mlesa ç’uokpd fpà ¼\½ dks çfrLFkkfir dj ldsA Breaking down 13 into 1 and 3 and then performing
First row- 14, 4, 31 mathematical operations on 1 and 3 is not allowed)
Second row- 38, 10, 79 ¼uksV: la[;kvksa dks mlds ?kVd vadksa esa foHkkftr fd, fcuk] iw.kZ
Third row- 30, 8, ? la[;kvksa ij lapkyu fd;k tkuk pkfg,A mnkgj.k ds fy, 13 & 13
(NOTE: Operations should be performed on the whole ij lapkyu tSls fd tksM+us @ gVkus @ xq.kk djus vkfn dks 13 esa
numbers, without breaking down the numbers into its fd;k tk ldrk gSA 13 dks 1 vkSj 3 esa rksM+dj vkSj fQj 1 vkSj 3
constituent digits. E.g. 13 – Operations on 13 such as ij xf.krh; lafØ;kvksa dks djus dh vuqefr ugha gS½
adding /deleting /multiplying etc. to 13 can be performed. (a) 58 (b) 46 (c) 30 (d) 34
Breaking down 13 into 1 and 3 and then performing Sol. (d)
mathematical operations on 1 and 3 is NOT allowed) B
¼uksV: la[;kvksa dks mlds ?kVd vadksa esa foHkkftr fd, fcuk] iw.kZ Pattern:- (A,B,C), [2A –C = ]
5
la[;kvksa ij xf.krh; lafØ;k,a dh tkuh pkfg,A mnkgj.k ds fy, st
I row  (12, 251, 19) 
13ds ekeys esa & 13 tksM+us @ gVkus @ xq.kk vkfn tSlh xf.krh;
25
lafØ;k,W 13 ij dh tkr ldrh gSA 13 dks 1 vkSj esa rksM+dj vkSj 2  12  19   24  19  5  5  5
5
fQj 1 vkSj 3 ij xf.krh; lafØ;k,a djus dh vuqefr ugha gSA
nd
(a) 78 (b) 73 (c) 63 (d) 68 II row (35, 50, 60) 
Sol. (c) 50
2  35  60   70  60  10  10  10
Pattern:- (A,B,C), [A +( B×4+1) = C] 5
st
I row  (14, 4, 31)  14 + (4 × 4 + 1) 14 + 17 = 31 Similarly:-
rd
nd
II row (38, 10, 79) 38 + (10 × 4 + 1) 38 + 41 = 79 III row (24, 70, ?) 
Similarly:- 70
rd
2  24  ?   48  ?  14  ?  48  14  34 Correct option
III row (30, 8, ?)  30 + (8 × 4 + 1) 30 + 33 = 63 5
Correct option (c) (d)
Concept: Row Matrix: Multiplication: Pattern:- (A,B,C), Concept: Row Matrix: Multiplication: (A,B,C), [
[3A+1 = B, 3B +1 =C] Successor of multiple of two numbers A  2  B
 C ](sum of Factors of two numbers is equal to
is equal to next number , forms a multiplication series 2
43. Find the missing number from the below options. the third number)
uhps fn, x, fodYiksa esa ls yqIr la[;k Kkr dhft,A 45. Study the given pattern carefully and select the number
9 28 85 that can replace the question mark (?) in it.
12 37 ? fn, x, iSVuZ dk è;kuiwod
Z vè;;u djsa vkSj ml la[;k dk p;u djsa
16 49 148 tks mlesa ç’uokpd fpà ¼\½ dks çfrLFkkfir dj ldsA
(a) 134 (b) 112 (c) 96 (d) 140 First row: 124, 38, 100
Sol. (b) Second row: 78, 25, 64
Pattern:- (A,B,C), [3A+1 = B, 3B +1 =C] Third row: 94, 31, ?
×3+1 ×3+1 (NOTE: Operations should be performed on the whole
Ist row(9, 28, 85) 9, 28, 85 numbers, without breaking down the number into its
×3+1 ×3+1 constituent digits. For example, 13 – Operations on 13
IInd row(16, 49, 148) 16, 49, 148 such as adding/deleting/multiplying etc. to 13 can be
performed. Breaking down 13 into 1 and 3 and then
Similarity:- performing mathematical operations on 1 and 3 is not
×3+1 ×3+1
IIIrd row(12, 37, ?) 12, 37, 112 allowed.)
¼uksV: la[;kvksa dks mlds ?kVd vadksa esa foHkkftr fd, fcuk] iw.kZ
Correct option (b)
la[;kvksa ij xf.krh; lafØ;k,a dh tkuh pkfg,A mnkgj.k ds fy, 13
B
Concept: Row Matrix: Multiplication: (A,B,C), [2A –C = ] ds ekeys esa & 13 tksM+us @ gVkus @ xq.kk vkfn tSlh xf.krh;
5
lafØ;k,W 13 ij dh tkr ldrh gSA 13 dks 1 vkSj esa rksM+dj vkSj
difference of any two Number’s Multiple or factor is
fQj 1 vkSj 3 ij xf.krh; lafØ;k,a djus dh vuqefr ugha gSA
subtracted is equal to multiple or factor of third number
(a) 84 (b) 80 (c) 74 (d) 78
44. Study the given pattern carefully and select the number
Sol. (d)
that can replace the question mark (?) in it.

207
Download Free PDFs & e-Books from Neon Classes App

A  2  B Breaking down 13 into 1 and 3 and then performing


Pattern:- (A,B,C), [ C ] mathematical operations on 1 and 3 is NOT allowed.)
2
st ¼uksV: la[;kvksa dks mlds ?kVd vadksa esa foHkkftr fd, fcuk] iw.kZ
I row  (124, 38, 100)  la[;kvksa ij xf.krh; lafØ;k,a dh tkuh pkfg,A mnkgj.k ds fy, 13
124   2  38  124  76 200 ds ekeys esa & 13 tksM+us @ gVkus @ xq.kk vkfn tSlh xf.krh;
   100
2 2 2 lafØ;k,W 13 ij dh tk ldrh gSA 13 dks 1 vkSj esa rksM+dj vkSj fQj
nd 78   2  25 78  50 128 1 vkSj 3 ij xf.krh; lafØ;k,a djus dh vuqefr ugha gSA
II row (78, 25, 64)     64
(a) 184 (b) 148 (c) 146 (d) 164
2 2 2
Similarly:- Sol. (a)
Pattern:- (A,B,C), [A×4–2 = B, A×8 =C]
rd 94   2  31 94  62 156
III row (94, 31, ?)     78 The pattern followed here is : -(A,B,C), [A×4–2 = B, A×8 =C]
2 2 2
Correct option (d)
Concept: Row Matrix: Multiplication: consecutive numbers ×4–2 ×8
Ist row(9, 34, 72) 9, 34, 72
in a row or column are related with some multiplication
logic ×4–2 ×8
46. Study the given pattern carefully and select the number IInd row(7, 26, 56) 7, 26, 56
that can replace the question mark (?) in it.
fn, x, iSVuZ dk è;kuiwod
Z vè;;u djsa vkSj ml la[;k dk p;u djsa Similarly:-
×8
×4–2
tks mlesa ç’uokpd fpà ¼\½ dks çfrLFkkfir dj ldsA IIIrd row(23, 90, ?)23, 90, 184
First row- 6,39, 124
Second row- 8 , 53, ?
Third row- 11, 74, 229 Correct option (a)
(NOTE: Operations should be performed on the whole AB
numbers, without breaking down the numbers into its Concept: Row Matrix: Division : (A,B,C), [ = C] : (sum
4
constituent digits. E.g. 13 – Operations on 13 such as is divided by a number)
adding /deleting /multiplying etc. to 13 can be performed. 48. In each of the given number groups, the number to the
Breaking down 13 into 1 and 3 and then performing right of the (equal sign) is obtained by performing some
mathematical operations on 1 and 3 is NOT allowed). mathematical operations on the two numbers to the left of
¼uksV: la[;kvksa dks mlds ?kVd vadksa esa foHkkftr fd, fcuk] iw.kZ the (equal sign). The same pattern is followed in all the
la[;kvksa ij xf.krh; lafØ;k,a dh tkuh pkfg,A mnkgj.k ds fy, 13 three number-groups. Select the number from the given
ds ekeys esa & 13 tksM+us @ gVkus @ xq.kk vkfn tSlh xf.krh; alternatives which can come in place of the question mark
lafØ;k,W 13 ij dh tkr ldrh gSA 13 dks 1 vkSj esa rksM+dj vkSj (?) in the third set of numbers.
fQj 1 vkSj 3 ij xf.krh; lafØ;k,a djus dh vuqefr ugha gSA fn, x, çR;sd la[;k lewgksa esa] '=' ¼fpà cjkcj½ ds nkbZa vksj ekStwn
(a) 111 (b) 161 (c) 116 (d) 166 la[;k] ‘=’ (cjkcj fpUg) ds ckbZ vksj ekStwn nks la[;kvksa ij dqN
Sol. (d) xf.krh; lafØ;k,a djds izkIr dh tkrh gSA rhuksa la[;k&lewgksa esa
Pattern:- (A,B,C), [A×7–3 = B, B×3+7 =C] leku iSVuZ dk ikyu fd;k tkrk gSA fn, x, fodYiksa esa ls ml
The pattern followed here is : -(A,B,C), [A×7–3 = B, B×3+7 =C] la[;k dk p;u dhft,] tks rhljs la[;k & lewg esa iz’u fpUg (?) ds
×7–3 ×3+7 LFkku ij vk ldrh gSaA
Ist row(6, 39, 124) 6, 39, 124
69, 87, = 39
×7–3 ×3+7 93, 95 = 47
IInd row(11, 74, 229) 11, 74, 229
52, 64 = ?
(a) 29 (b) 37 (c) 45 (d) 53
Similarly:-
×7–3 ×3+7 Sol. (a)
IIIrd row(8, 53, ?) 8, 53, 166
AB
Correct option (d) The pattern followed here is : -(A,B,C), [ = C]
4
Concept: Row Matrix: Multiplication: Multiplication of
st 69  87 156
successor or predecessors of any two numbers is related I row  (69, 87 = 39)    39
to the third number 4 4
47. Study the given pattern carefully and select the number nd 93  95 188
II row(93, 95 = 47)    47
that can replace the question mark (?) in it. 4 4
fn, x, iSVuZ dk è;kuiwod
Z vè;;u djsa vkSj ml la[;k dk p;u djsa Similarly:-
tks mlesa ç’uokpd fpà ¼\½ dks çfrLFkkfir dj ldsA rd 52  64 116
First row- 9, 34, 72 III row (52, 64, ?)    29
4 4
Second row- 7, 26, 56 Correct option (a)
Third row- 23, 90, ?
C
(NOTE: Operations should be performed on the whole Concept: Row Matrix: Division : (A,B,C), [  n  B ], here n
A
numbers, without breaking down the numbers into its
constituent digits. E.g. 13 – Operations on 13 such as can be any numbers. ( division , Multiple of the ratio of any
adding /deleting /multiplying etc. to 13 can be performed. two numbers is equal to third numbers)

208
Best App for Govt. Jobs : Neonclasses (Download Now)

49. Study the given pattern carefully and select the number III
rd
row (68, 47, ?) 
that can replace the question mark (?) in it.
3 3
fn, x, iSVuZ dk /;kuiwoZd v/;;u djsa vkSj ml la[;k dk p;u (68  47)   115   23  3  69
5 5
djsa tks mlesa ç'uokpd fpà (?) dks çfrLFkkfir dj ldsA
Correct option (b)
7 18 21
A BC
20 15 50 Concept: Row Matrix: Division : (A,B,C,D), [  D ]: in
5
? 42 98
a 3*4 matrix sum of 3 numbers is divided by a random
(a) 18 (b) 12 (c) 14 (d) 16
number is equal to the third number
Sol. (c)
51. In each of the given number-clusters, the number on the
C right side of = (the equal to sign) is calculated by
Pattern:- (A,B,C), [ 6  B ]
A performing certain mathematical operations on the three
st
I row  (7, 18, 21)  numbers on the left of= (the equal to sign). All three
21 number-clusters follow the same pattern. Select the
 6  3  6  18 number from among the given options that can replace
7
nd
the question mark (?) in the third number-cluster.
II row (20, 15, 50)  fn, x, çR;sd la[;k&lewgksa es]a ¾ ¼fpà ds cjkcj½ ds nkbZa vksj dh
50 30 la[;k dh x.kuk ¾ ¼fpà ds cjkcj½ ds ckbZa vksj rhu la[;kvksa ij
6   15
20 2 dqN xf.krh; lapkyu djds dh tkrh gSA lHkh rhu la[;k&lewg
Similarly:- leku iSVuZ dk vuqlj.k djrs gSaA fn, x, fodYiksa esa ls og la[;k
rd
III row (?, 42, 98)  pqfu, tks rhljs la[;k&lewg esa ç'uokpd fpUg ¼\½ dks çfrLFkkfir
98 588 dj ldsA
 6  42  588  42  ?  ?   14 Correct option (c) 36, 58, 21 = 23
? 42
41, 39, 20 = 20
p
Concept: Row Matrix: Division : (A,B,C), [ (A  B)  C ], 39, 52, 34 = ?
q (a) 17 (b) 30 (c) 25 (d) 15
where p and q can be any number(Multiple of sum of two Sol. (c)
numbers is divided by a third number ) A BC
50. Study the given pattern carefully and select the number Pattern:- (A,B,C,D), [ D ]
5
that can replace the question mark (?) in it.
A BC
fn, x, iSVuZ dk è;kuiwod
Z vè;;u djsa vkSj ml la[;k dk p;u djsa The pattern followed here is : -(A,B,C), [ D ]
tks mlesa ç’uokpd fpà ¼\½ dks çfrLFkkfir dj ldsA 5
First row: 73, 52, 75 st 36  58  21 115
I row  (36, 58, 21)    23
Second row:64, 41, 63 5 5
Third row: 68, 47, ? nd 41  39  20 100
(NOTE: Operations should be performed on the whole II row ( 41, 39, 20)    20
5 5
numbers, without breaking down the number into its Similarly:-
constituent digits. For example, 13 – Operations on 13
rd 39  52  34 125
such as adding/deleting/multiplying etc. to 13 can be III row (39, 52, 34)    25
5 5
performed. Breaking down 13 into 1 and 3 and then
Correct option (c)
performing mathematical operations on 1 and 3 is not
Concept: Row Matrix: Division : (A,B,C,D), [A×B–C = D]
allowed.)
Multiplication of any two numbers is subtracted by third
¼uksV: la[;kvksa dks mlds ?kVd vadksa esa foHkkftr fd, fcuk] iw.kZ
number is equal to the fourth number
la[;kvksa ij xf.krh; lafØ;k,a dh tkuh pkfg,A mnkgj.k ds fy, 13
52. In each of the given number-clusters, the number on the
ds ekeys esa & 13 tksM+us @ gVkus @ xq.kk vkfn tSlh xf.krh;
right side of `=' (the equal to sign) is calculated by
lafØ;k,W 13 ij dh tkr ldrh gSA 13 dks 1 vkSj esa rksM+dj vkSj performing certain mathematical operations on the four
fQj 1 vkSj 3 ij xf.krh; lafØ;k,a djus dh vuqefr ugha gSA numbers on the left of = (the equal to sign). All three
(a) 71 (b) 69 (c) 63 (d) 65 number-clusters follow the same pattern. Select the
Sol. (b) number from among the given options that can replace
2 the question mark (?) in the third number-cluster
Pattern:- (A,B,C), [ (A  B)   C ]
5 fn, x, çR;sd la[;k&lewgksa esa] `=' ¼fpà ds cjkcj½ ds nkbZa vksj dh
st 3 3 la[;k dh x.kuk ¾ ¼fpà ds cjkcj½ ds ckbZa vksj dh pkj la[;kvksa ij
I row  (73, 52, 75)  (73  52)   125   25  3  75
5 5 dqN xf.krh; lafØ;k,¡ djds dh tkrh gSA lHkh rhu la[;k&lewg
II
nd
row (64, 41, 63)  leku iSVuZ dk vuqlj.k djrs gSaA fn, x, fodYiksa esa ls ml la[;k
dk p;u djsa tks rhljs uacj lewg esa ç'u fpà ¼\½ dks çfrLFkkfir
3 3
(64  41)   105   21  3  63 dj ldrh gS
5 5
21, 9, 40 = 149
Similarly:- 18, 7, 25 = 101
15, 11, 30 = ?
(a) 110 (b) 120 (c) 125 (d) 135

209
Download Free PDFs & e-Books from Neon Classes App

Sol. (d) Concept: Row Matrix: 3*5 matrix : (A,B,C,D,E), [A2 + B2 + C2


Pattern:- (A,B,C,D), [A×B–C = D] + D2 = E] Sum of squares of four Numbers is equal to fifth
st
I row  (21, 9, 40 = 149) 21 × 9 – 40  189 – 40 = 149 number
nd
II row ( 18, 7, 25 = 101)  18 × 7 – 25  126 – 25 = 101 55. Select the option that can replace the question mark (?) in
Similarly:- the third row.
rd ml fodYi dk p;u djsa tks rhljh iafä esa ç'u fpà ¼\½ dks
III row (15, 11, 30 = ?)  15 × 11 – 30  165 – 30 = 135
çfrLFkkfir dj ldrk gS
Correct option (d)
C
Row 1 : 5, 6, 2, 4, 81
Concept: Row Matrix: power based: (A,B,C) [A =B] (When Row 2 : 1, 3, 2, 4, ?
one number is powered by another Row 3 : 2, 3, 1, 5, 39
53. Study the given pattern carefully and select the number (a) 73 (b) 40 (c) 30 (d) 52
that can replace the question mark (?) in it. Sol. (c)
fn, x, iSVuZ dk /;kuiwoZd vè;;u djs]a vkSj ml la[;k dk p;u Pattern:- (A,B,C,D,E), [A2 + B2 + C2 + D2 = E]
djsa tks iz’uokpd fpUg (?) ds LFkku ij vk ldrh gSA
The pattern followed here is : -(A,B,C,D,E), [A2 + B2 + C2 + D2 =
7 343 3
E]
5 ? 3 st
I row  (5, 6, 2, 4, 81) 52 + 62 + 22 + 22  25 + 36 + 4 + 16
2 256 8
= 81
(a) 225 (b) 250 (c) 525 (d) 625 nd
Sol. (d) II row (2, 3, 1, 5, 39) 22 + 32 + 12 + 52  4 + 9 + 1 + 25
c = 39
Pattern:- (A,B,C)[A = B]
3
Similarly:-
(7,343, 3)7  7×7×7 = 343 rd
3
III row (1, 3, 2, 4, ?) 12 + 32 + 22 + 42  1 + 9 + 2 + 16 =
(5,?, 4)5  5×5×5 = 625 30
8
(2,256, 8)2  2×2×2×2×2×2×2×2 = 256 Correct option (c)
Correct option (d) Concept: Row Matrix: 3*5 matrix : (A,B,C,D,E), [A×B–
Concept: Row Matrix: 3*4 matrix square : (A,B,C), [(A2 + B2 C×D=E] Multiplication of any two numbers is subtracted
from multiplication of another two numbers is equal to
+ C2 )–8 = D] (Sum of square of all three numbers is
56. In each of the given number – cluster, the number on the
subtracted)
right side of = (the equal to sign) is calculated by
54. In each of the given number clusters. the number on the
performing certain mathematical operations on the four
right side of ‘=’ (the equal to sign) is calculated by
numbers on the left of = (the equal to sign). All three
performing certain mathematical operations as the three
number – clusters follow the same pattern. Select the
numbers on the left of '=' (the equal to sign). All three
number from among the given options that can replace
number-clusters follow the same pattern. Select the
the question mark (?) in the third number – cluster.
number from among the given options that can replace
fn, x, çR;sd la[;k & lewg esa] ¾ ¼cjkcj fpའds nkbZa vksj ekStwn
the question mark (?) in the third number-cluster.
la[;k ‘=’ ¼cjkcj fpའds ckbZa vksj ekStwn pkj la[;kvksa ij dqN
fn, x, çR;sd la[;k lewgksa esaA '=' ¼fpà ds cjkcj½ ds nkbZa vksj dh
xf.krh; lafØ;k,a djds izkIr dh tkrh gSA rhuksa la[;k & lewgksa esa
la[;k dh x.kuk dqN xf.krh; lafØ;kvksa dks '=' ¼fpà ds cjkcj½ ds
leku iSVuZ dk ikyu fd;k tkrk gSA fn, x, fodYiksa esa ls ml
ckbZa vksj rhu la[;kvksa ds :i esa djds dh tkrh gSA lHkh rhu
la[;k dk p;u djsa tks rhljs la[;k & lewg esa ç'u fpà ¼\½ dks
la[;k&lewg leku iSVuZ dk vuqlj.k djrs gSaA fn, x, fodYiksa esa ls
çfrLFkkfir dj ldrh gSA
og la[;k pqfu, tks rhljs la[;k&lewg esa ç'uokpd fpUg ¼\½ dks
11, 8, 5, 9 = 43
çfrLFkkfir dj ldsA
12, 9, 7, 6 = 66
4, 3, 2 = 21
14, 13, 9, 10 = ?
5, 6, 3 = 52
(a) 92 (b) 80 (c) 98 (d) 89
5, 8, 4 = ?
Sol. (a)
(a) 41 (b) 106 (c) 97 (d) 85
Pattern:- (A,B,C,D,E), [A×B–C×D=E]
Sol. (c) st
I row  (11, 8, 5, 9 = 43) 11 × 8 – 5 × 9  88 – 45 = 43
Pattern:- (A,B,C), [(A2 + B2 + C2 )–8 = D]
nd
The pattern followed here is : -(A,B,C), [(A2 + B2 + C2 )–8 = D] II row (12, 9, 7, 6 = 66) 12 × 9 – 7 × 6  108 – 42 = 66
st Similarly:-
I row  (4, 3, 2 = 21) (42 + 32 + 22 )–8  16 + 9 + 4 – 8  rd
29 – 8 = 21 III row (14, 13, 9, 10 = ?) 14 × 13 – 9 × 10  182 – 90
nd = 92
II row ( 5, 6, 3 = 52)  (52 + 62 + 32 ) – 8  25 +36 + 9 –
Correct option (a)
8  70 – 8 = 62 AB
Similarly:- Concept: Row Matrix: 3*5 matrix :  E in a 3*5 matrix
rd
CD
III row (5, 8, 4 = ? )  (52 + 82 + 42 )–8  25 + 64 + 16 – , multiplication of two number is divided by difference of
8  105 – 8 = 97 other remaining number is equal to a fifth number
Correct option (c) 57. In each of the given number-clusters, the number on the
right side of `=' (the equal to sign) is calculated by
performing certain mathematical operations on the four

210
Best App for Govt. Jobs : Neonclasses (Download Now)

numbers on the left of = (the equal to sign). All three


3 G N Q 8
number-clusters follow the same pattern. Select the
number from among the given options that can replace 7 14 17  7 + 14 + 17 = 38
the question mark (?) in the third number-cluster 3 R B S 9
fn, x, çR;sd la[;k&lewgksa esa] `=' ¼fpà ds cjkcj½ ds nkbZa vksj dh 18 2 19  18 + 2 + 19 = 39
la[;k dh x.kuk ¾ ¼fpà ds cjkcj½ ds ckbZa vksj dh pkj la[;kvksa ij
dqN xf.krh; lafØ;k,¡ djds dh tkrh gSA lHkh rhu la[;k&lewg 4 T U A 2
leku iSVuZ dk vuqlj.k djrs gSaA fn, x, fodYiksa esa ls ml la[;k 20 21 1  20 + 21 + 1 = 42
dk p;u djsa tks rhljs uacj&DyLVj esa ç'u fpà ¼\½ dks çfrLFkkfir 2 F P C 5
dj ldrh gS 6 16 3  6 + 16 + 3 = 25
12, 4, 37, 25 = 4
Similarly:-
2, 36, 87, 51 = 2
18, 3, 58, 49 = ? 3 O L H ?
(a) 7 (b) 5 (c) 6 (d) 8 15 12 8  15 + 12 + 8 = 35
Sol. (c)
Correct option (d)
AB
Pattern:- E Concept: Column Matrix: Cube & Operations:
CD
A
AB B   A3  B3  C 
The pattern followed here is : -
CD
E     Cube of two number’s sum is equal to
C 
st 12  4 48
I row  (12, 4, 37, 25 = 4)   4 the third number
37  25 12
60. Study the given pattern carefully and select the number
nd 2  36 72 that can replace the question mark (?) in it.
II row (2, 36, 87, 51 = 2)   2
87  51 36 fn, x, iSVuZ dk /;kuiwoZd vè;;u djs]a vkSj ml la[;k dk p;u
Similarly:- djsa tks iz’uokpd fpUg (?) ds LFkku ij vk ldrh gSA
rd 18  3 54 789
III row (18, 3, 58, 49 = ?)   6
58  49 9 42?
Correct option (c) 407 520 756
Concept: Row Matrix: 4*2 matrix : [A+B = k ] where k is (a) 1 (b) 2 (c) 3 (d) 4
constant(sum of row remains constant) Sol. (c)
58. Find the missing number from the below options. A
uhps fn, x, fodYiksa esa ls yqIr la[;k Kkr dhft,A  
Pattern:- B   A3  B3  C 
3 10 C 
5 8
7 ? A
 
5 8 The pattern followed here is : - B   A3  B3  C 
(a) 7 (b) 5 (c) 6 (d) 8 C 
Sol. (c)
Pattern:- [A+B = 13] 7 
I column  4 
st
The pattern followed here is : - [A+B = 13]
st  407 
I row  3, 10 3 + 10 = 13
nd
II row 5, 8  5 + 8 = 13 73  43  7  7  7  4  4  4
th
IV row 5, 8  5 + 8 = 13  343  64  407
Similarly:- 8  3 3
8  2  8  8  8  2 2 2
II column 2 
rd nd
III row 7, ?  7 + ? = 13
 ? = 13 – 7 = 6 520  512  8  520
Correct option (c) Similarly:-
Concept: Row Matrix: Letter & Number Relation:[Sum 9  9  ?  756  ?  756  9
3 3 3 3

Alphabet = (A) + B+ (C) = BC] rd ?   9  9  9  756  729  27


III column  
59. Find the missing number from the below options.
756  ?3  27  ?  3 27  3
uhps fn, x, fodYiksa esa ls yqIr la[;k Kkr dhft,A
3 GNQ 8 Concept:Column Matrix: Cube & Operations:
3 RBS 9 A
4 TUA 2 B  B3  A  C 
   
2 FPC 5 C 
3 OLH ?
(a) 6 (b) 7 (c) 8 (d) 5 61. Study the given pattern carefully and select the number
Sol. (d) that can replace the question mark (?) in it.
Pattern:- [Sum Alphabet = (A) + B+ (C) = BC]

211
Download Free PDFs & e-Books from Neon Classes App

fn, x, iSVuZ dk /;kuiwoZd v/;;u djsa vkSj ml la[;k dk p;u Concept: Square & Operations based column matrix:
djsa tks mlesa ç'uokpd fpà (?) dks çfrLFkkfir dj ldsA A
4 7 11 B   A 2  B  C 
   
5 3 4 C 
? 34 75
(a) 29 (b) 129 (c) 41 (d) 138 64. Study the given pattern carefully and select the number
Sol. (b) that can replace the question mark (?) in it.
fn, x, iSVuZ dk /;kuiwoZd vè;;u djs]a vkSj ml la[;k dk p;u
A
djsa tks iz’uokpd fpUg (?) ds LFkku ij vk ldrh gSA
Pattern:- B  B3  A  C 
9 12 13
C 
42 98 ?
nd
II column 3  7  3  3  3  7  27  7  34
3 39 46 80
rd (a) 93 (b) 99 (c) 140 (d) 89
III column 4  11  4  4  4  11  64  11  75
3
Sol. (d)
Similarly:-
st
A
I column 5  4  5  5  5  4  125  4  129 Pattern:- B   A 2  B  C 
3

Correct option (b) C 


Concept: Square & Operations based column matrix:
A
B   A 2  B2  C  9 
    I column 42 92  42  81  42  39
st
C 
39 
62. Select the missing number from the given options
fn, x, fodYiksa esa ls yqIr la[;k dk p;u djsAa 12 
nd  
7 11 14 II column 98  122  98  144  98  46
8 12 9 46 
113 265 ?
Similarly:-
(a) 277 (b) 625 (c) 361 (d) 281 rd
Sol. (a) III column
A 13 
  ?  132  ?  80  169  ?  80
Pattern:- B   A 2  B2  C   
80  ?  80  169  89
C 
st Correct option (d)
I column 7  8  49  64  113
2 2

Concept: Square & Operations based column matrix:


nd
II column 11  12  121  144  265
2 2
A
Similarly:- B   A  C 3  B 
rd     
III column 14  9  196  81  277
2 2
C 
Correct option (a)
65. Select the missing number from the given options.
Concept: Square & Operations based column matrix: fn, x, fodYiksa esa ls yqIr la[;k dk p;u djsAa
A 2 3 5
B   A 2  B2  C 
    125 343 216
C  3 4 ?
63. Select the missing number from the below options. (a) 2 (b) 4 (c) 3 (d) 1
uhps fn, x, fodYiksa esa ls yqIr la[;k dk p;u djsAa Sol. (d)
9 11 8 A
 
Pattern:- B   A  C   B
3
683  
45 57 ? C 
(a) 55 (b) 54 (c) 52 (d) 53
I column 2  3  53  5  5  5  125
st 3
Sol. (a)
A II column 3  4  73  7  7  7  343
nd 3

 
Pattern:- B   A 2  B2  C 
Similarly:-
C 
5  ? 
3
rd  216  5  ?  3 216
st III column
I column 9  6  81  36  45
2 2
5 ?  6  ?  6 5  1
nd
II column 11  8  121  64  57
2 2
Correct option (d)
Similarly:-
rd
III column  8  3  64  9  55
2 2

Correct option (a)

212
Best App for Govt. Jobs : Neonclasses (Download Now)

Concept:Square & Operations based column matrix: fn, x, iSVuZ dks /;ku ls i<+as vkSj ml la[;k dk p;u djs]a tks blesa
A iz’uokpd fpgu (?) ds LFkku ij vk ldrh gSaA
B   A  B  C  8 27 32 16
   
C  12 18 16 20
18 12 ? 25
66. Select the missing number from the given options.
uhps fn, x, fodYiksa esa ls yqIr la[;k Kkr dhft,A (a) 14 (b) 12
(c) 8 (d) 10
16 25 81
Sol. (c)
36 49 9
10 12 ? A
(a) 15 (b) 12 (c) 21 (d) 36 Pattern:- B   A  C  B
Sol. (b) C 
A
8 
Pattern:- B   A  B  C  st 12   8  18  144  12
I column  
C 
18 
st
I column 16  36  4  6  10
27
II column 18   27  12  324  18
nd
II column 25  49  5  7  12 nd

Similarly:- 12 
rd
III column  81  9  9  3  12
16
Correct option (b) th  
IV column 20  16  25  400  20
Concept: Square & Operations based column matrix:
25
A
B  B  A  C  Similarly:-
   
C  32 
rd 16   32  ?  16
III column  
67. Study the given pattern carefully and select the number
? 
that can replace the question mark (?) in it.
fn, x, iSVuZ dk è;kuiwoZd vè;;u djsa vkSj ml la[;k dk p;u
djsa tks mlesa ç'uokpd fpà ¼\½ dks çfrLFkkfir dj lds 32  ?  16  32  ?  162  256
25 64 81
256
23 41 38 ? 8
32
18 33 ?
(a) 29 (b) 17 (c) 27 (d) 19 Correct option (c)
Sol. (a)
A A
 
Pattern:- B  B  A  C  Concept: Combination: B   A  B  C
C  C 
69. Study the given pattern carefully and select the number
25
st   that can replace the question mark (?) in it.
I column 23 23  25  23  5  18 fn, x, iSVuZ dk è;kuiwoZd vè;;u djsa vkSj ml la[;k dk p;u
18 djsa tks mlesa ç'uokpd fpà ¼\½ dks çfrLFkkfir dj lds
64  357
nd   23 27 31
II column 41 41  64  41  8  33
69 135 ?
33 
(a) 217 (b) 93 (c) 266 (d) 155
Similarly:- Sol. (a)
81 A
rd    
III column 38  38  81  38  9  29 Pattern:- B   A  B  C
?  C 
Correct option (a)
Concept:Square & Operations based column matrix: 3 
A st  
I column 23  3  23  69
B   A  C  B
   69
C 
5 
68. Study the given pattern carefully and select the number nd  
II column 27   5  27  135
that can replace the question mark (?) in it.
135

213
Download Free PDFs & e-Books from Neon Classes App

Similarly:- 12 
17   12  1  17  1  13  18  234
    
nd
7  II column 
III column  31  7  31  217
rd
234
? 
Similarly:-
Correct option (a) 15
16  15  1  16  1  16  17  272
     
rd
A III column
Concept:Combination: B   A  B  2  C  ? 
C 
Correct option (a)
70. Study the given pattern carefully and select the number A
that can replace the question mark (?) in it.  
Concept: Combination: B   A   B  K   C 
fn, x, iSVuZ dk /;kuiwoZd vè;;u djs]a vkSj ml la[;k dk p;u C 
djsa tks iz’uokpd fpUg (?) ds LFkku ij vk ldrh gSA
15 30 33 72. Study the given matrix carefully and select the number
16 26 19 from among the given options that can replace the
62 112 ? question mark (?) in it
(a) 124 (b) 128 (c) 117 (d) 104 fn, x, eSfVªDl dk è;kuiwoZd vè;;u djsa vkSj fn, x, fodYiksa esa ls
Sol. (d) ml la[;k dk p;u djsa tks mlesa ç'u fpà ¼\½ dks çfrLFkkfir dj
A
ldsA
7 9 5
Pattern:- B   A  B  2  C 
3 8 ?
C 
28 81 50
15 (a) 9 (b) 5 (c) 7 (d) 8
  Sol. (a)
I column 16  15  16   2  31  2  62
st

A
62  
Pattern:- B   A   B  1  C 
30 
  C 
II column 26   30  26   2  56  2  112
nd

112 7 
I column 3   7  3  1   7  4  28
st
Similarly:-
28 
33
 
III column 19  33  19  2  52  2  104
rd
9 
 
II column 8   9   8  1  9  9  81
nd
? 
Correct option (d) 81

A Similarly:-
 
Concept: Combination: B   A  K    B  K   C  5  50
rd   5  (? 1)  50  ? 1   10
C  III column ?  5
71. Study the given pattern carefully and select the number 50   ?  10  1  9
that can replace the question mark (?) in it. Correct option (a)
fn, x, iSVuZ dk /;kuiwoZd vè;;u djs]a vkSj ml la[;k dk p;u A
djsa tks iz’uokpd fpUg (?) ds LFkku ij vk ldrh gSA    A  B  C
Concept: Combination: B   
2
14 12 15 C   
11 17 16
180 234 ? 73. Select the missing number from the given options.
(a) 272 (b) 369 (c) 298 (d) 425 fn, x, fodYiksa esa ls yqIr la[;k dk p;u djsAa
Sol. (a) 36 52 86
A 28 40 12
  32 46 ?
Pattern:- B   A  1    B  1   C 
(a) 53 (b) 49 (c) 43 (d) 51
C 
Sol. (b)
A
14     A  B  C
Pattern:- B   
   2 
I column 11   14  1  11  1  15  12  180
st
C 
180
36 
st   36  28  64  32
I column 28 
2 2
32 

214
Best App for Govt. Jobs : Neonclasses (Download Now)

52  6 
52  40 92
II column 40   I column 8    6  8   (6  8)  48  14  62
nd st
  46
2 2
46  62
Similarly:- 11 
3   11  3  (11  3)  33  14  47
   
nd
86 II column
rd   86  12  98  49
III column 12   47
2 2
? 
Similarly:-
Correct option (b) 14 
(14  ?)  (14  ?)  89  (14  5)  (14  5)
III column ? 
rd
A
   70  19  89
Concept: Combination: B   A  B   (A  B)  C  89 
C 
Correct option (a)
74. Study the given pattern carefully and select the number A
that can replace the question mark (?) in it. Concept: Combination: B   A  B   C 
2

fn, x, iSVuZ dk è;kuiwoZd vè;;u djsa vkSj ml la[;k dk p;u  


C 
djsa tks mlesa ç'uokpd fpà ¼\½ dks çfrLFkkfir dj lds
15 18 ? 76. Study the given pattern carefully and select the number
8 9 12 that can replace the question mark (?) in it.
161 243 432
fn, x, iSVuZ dk /;kuiwoZd v/;;u djsa vkSj ml la[;k dk p;u
(a) 20 (b) 24 (c) 22 (d) 26
djsa tks mlesa ç'uokpd fpà (?) dks çfrLFkkfir dj ldsA
Sol. (b) 55 63 ?
48 58 40
A
  49 25 49
Pattern:- B   A  B   (A  B)  C  (a) 58 (b) 51 (c) 47 (d) 49
C  Sol. (c)
15  A
  Pattern:- B   A  B   C 
2
I column 8   15  8   (15  8)  23  7  161
st
 
161 C 

18  55 
9   18  9  (18  9)  27  9  243 I column 48  (55  48)2  72  49
st
  
nd
II column 
243 49
Similarly:- 63
nd  
?  II column 58   (63  58)2  52  25
rd  
III column  12  25
 432 Similarly:-
(? 12)  (? 12)  432  (24  12)  (24  12) ? 
Correct option (b) rd  
 36  12  432 III column 40  (47  40)2  72  49
A 49
 
Concept: Combination: B   A  B   (A  B)  C  or
C  (? 40)2  49  ? 40  49
75. Study the given pattern carefully and select the number  ? 40  7  ?7  7  40  47
that can replace the question mark (?) in it. Correct option (c)
fn, x, iSVuZ dk è;kuiwoZd vè;;u djsa vkSj ml la[;k dk p;u A
djsa tks mlesa ç'uokpd fpà ¼\½ dks çfrLFkkfir dj lds   B  C  A
Concept: Combination: B   
2
6 11 14 C   
83?
62 47 89 77. Study the given pattern carefully and select the number
(a) 5 (b) 6 (c) 15 (d) 4 that can replace the question mark (?) in it.
Sol. (a) fn, x, iSVuZ dk /;kuiwoZd v/;;u djs]a vkSj ml la[;k dk p;u
djsa tks iz’uokpd fpUg (?) ds LFkku ij vk ldrh gSA
A
  ? 23 17
Pattern:- B   A  B   (A  B)  C 
74 95 62
C  44 49 28
(a) 16 (b) 32 (c) 15 (d) 36
Sol. (c)

215
Download Free PDFs & e-Books from Neon Classes App

A A
B  C 
Pattern:- B    A Pattern:- B  5A  B  C 
 2 
C  C 

23 
st   95  49  46  23
I column 95  9 
I column 18   5  9  18  45  18  27
2 2 st
49
27
17 
62  28 34
II column 62  
nd
  17 18
2 2 nd  
28  II column 32  5  18  32  90  32  58
58
Similarly:-
?  Similarly:-
74  44 30
III column 74  
rd
  15 ?  5? 79  46  5?  46  79
2 2 rd  
44  III column 79  125
 5?  125  ?   25
Correct option (c)  46  5

A Correct option (d)


AB 
Concept: Combination: B    C A
Concept: Combination: B   XA  YB  C
 5 
C 
C 
78. Study the given pattern carefully and select the number
that can replace the question mark (?) in it.
fn, x, iSVuZ dk /;kuiwoZd v/;;u djsa vkSj ml la[;k dk p;u where X and Y can be any Number
djsa tks mlesa ç'uokpd fpà (?) dks çfrLFkkfir dj ldsA 80. Study the given pattern carefully and select the number
15 25 45 that can replace the question mark (?) in it.
5 6 8 fn, x, iSVuZ dk /;kuiwoZd v/;;u djs]a vkSj ml la[;k dk p;u
15 30 ? djsa tks iz’uokpd fpUg (?) ds LFkku ij vk ldrh gSA
(a) 60 (b) 61 (c) 76 (d) 72 10 13 17
Sol. (d) 12 11 ?
A 160 185 235
AB  (a) 15 (b) 13 (c) 10 (d) 17
Pattern:- B    C
 5  Sol. (b)
C 
A
 
15 Pattern:- B  10A  5B  C
st   15  5  75  15
I column 5  
5 5 C 
15 st
I column
25
nd   25  6  150  30 10 
II column  6   12   10  10  5  12  100  60  160 nd
II column
5 5  
30 160
Similarly:-
13 
45 11   10  13  5  11  130  55  185
45  8 360 Similarly:-
III column 8    
rd
  72
5 5 185
? 
17  10  17  5  ?  235  170  5?  235
Correct option (d)
III column ?   5?  235  170  5?  65
rd
A
 
Concept: Combination: B  5A  B  C  235  ?  65  13
5
C 
Correct option (b)
79. Study the given pattern carefully and select the number
A
that can replace the question mark (?) in it.  
fn, x, iSVuZ dk /;kuiwoZd v/;;u djs]a vkSj ml la[;k dk p;u Concept:Combination: B   A  K   B  C 
djsa tks iz’uokpd fpUg (?) ds LFkku ij vk ldrh gSA C 
9 18 ? 81. Study the given pattern carefully and select the number
18 32 79 that can replace the question mark (?) in it.
27 58 46 fn, x, iSVuZ dk è;kuiwoZd vè;;u djsa vkSj ml la[;k dk p;u
(a) 22 (b) 21 (c) 26 (d) 25 djsa tks mlesa ç'uokpd fpà ¼\½ dks çfrLFkkfir dj lds
Sol. (d) 23 32 16
345

216
Best App for Govt. Jobs : Neonclasses (Download Now)

72 132 ? (a) 29 (b) 25 (c) 32 (d) 27


(a) 85 (b) 80 (c) 88 (d) 162 Sol. (b)
Sol. (a) Pattern:- [Sum All Numbers = 40]
A (15, 13, 10, 2)
Pattern:- B   A  1  B  C  15 + 13 + 10 + 2 = 40
C   (20, 5, 4, 11)
20 + 5 + 4 + 11 = 40
23
   (13, 8, 13, 6)
I column 3    23  1  3  24  3  72
st
13 + 8 + 13 + 6 = 40
72
Similarly:-
32  (7, 6, 2, ?)
II column 4   32  1  4  33  4  132 7 + 6 + 2 + ? = 40  15 + ? = 40 
nd

132  ? = 40 – 15 = 25
Correct option (b)
Similarly:-
Concept: Matrix ratio Form: When the difference of group
16 of two numbers are in some ratio
III column 5   16  1  5  17  5  85
rd
84. Study the given pattern carefully and select the number
?  that can replace the question mark (?) in it.
Correct option (a)
fn, x, iSVuZ dks /;ku ls i<+as vkSj ml la[;k dk p;u djs]a tks blesa
iz’uokpd fpgu (?) ds LFkku ij vk ldrh gSaA.
A 
B 12 33 28
Concept: Combination: B  [ A   C ], where k can be
k 9 12 ?
C 
13 40 17
any number. when square roots of a perfect square is (a) 61 (b) 15 (c) 16 (d) 51
added to factor of another number Sol. (a)
82. Study the given pattern carefully and select the number
A 
that can replace the question mark (?) in it.    A  B  3
Pattern:- B    [Ratio = 3 : 4]
fn, x, iSVuZ dk /;kuiwoZd v/;;u djsa vkSj ml la[;k dk p;u C  B  4 
C 
djsa tks mlesa ç'uokpd fpà (?) dks çfrLFkkfir dj ldsA
16 36 81
50 42 60 12 9 13
29 27 ? Ist column
~3 ~4
(a) 39 (b) 37 (c) 21 (d) 25 or 12 – 9 = 3,
Sol. (a) 13 – 9 = 4
A 
  B 33 12 40
Pattern :- B  [ A C ]
2 IInd column
C  21 28
~3 ~4
16  or 33 – 12 = 21,
st    16  50  4  25  29
I column 50  40 – 12 = 28
2
29 
Similarly:-
36  28 61 17
rd    36  42  6  21  27
II column 42  IIIrd column
2 33 44
27 
~3 ~4
Similarly:- or 61 – 28 = 33,
81  61 – 17 = 44
nd    81  60  9  30  39
III column 60  Correct option (a)
2
?  Concept:4*4 Matrix Combination: The pattern followed
Correct option (a) here is : - [(A+C)+(B×10)=D]
Concept: 4 Matrix Sum All Numbers Constant: [Sum All 85. Study the given pattern carefully and select the number
Numbers = k] k is Constant Number(sum of all numbers is that can replace the question mark (?) in it.
constant) fn, x, iSVuZ dk /;kuiwoZd v/;;u djsa vkSj ml la[;k dk p;u
83. Study the given pattern carefully and select the number djsa tks mlesa ç'uokpd fpà (?) dks çfrLFkkfir dj ldsA
that can replace the question mark (?) in it. (15, 1, 7, 32) (20, 2, 7, 47) (13, 3, 7, 50) (7, 4, ?, 48)
fn, x, iSVuZ dk /;kuiwoZd v/;;u djsa vkSj ml la[;k dk p;u (a) 1 (b) 8 (c) 5 (d) 3
djsa tks mlesa ç'uokpd fpà (?) dks çfrLFkkfir dj ldsA Sol. (a)
(15, 13, 10, 2) (20, 5, 4, 11) (13, 8, 13, 6) (7, 6, 2, ?) Pattern:- [(A+C)+(B×10)=D]

217
Download Free PDFs & e-Books from Neon Classes App

(15, 1, 7, 32) fn, x, iSVuZ dks /;ku ls i<+as vkSj ml la[;k dk p;u djs]a tks blesa
(15+7)+(1×10)22+10=32 iz’uokpd fpgu (?) ds LFkku ij vk ldrh gSaA
 (20, 2, 7, 47) 18 21 24
(20+7)+(2×10)27+20=47 24 11 27
 (13, 3, 7, 50) 12 ? 16
(13+7)+(3×10)20+30=50 16 22 18
Similarly:- (a) 42 (b) 32 (c) 36 (d) 38
(7, 4, 7, 48) Sol. (a)
(7+?)+(4×10) = 48 7+?+ 40 = 48  7 + ? = 48 – 40  7 A
 
B
+?=8?=8–7=1 Pattern:-    A  D  B  C 
C 
or (7+1)+(4×10)  8 + 40 = 48  
Correct option (a) D 
A 18 
   
B
Concept:4*4 Matrix Combination:    AB  C  D
st
I column 24   18  16  24  12  288  288
C  12 
   
D  16 
86. Study the given pattern carefully and select the number 24 
that can replace the question mark (?) in it.  
27
III column    24  18  27  16  432  432
nd
fn, x, iSVuZ dks /;ku ls i<+as vkSj ml la[;k dk p;u djs]a tks blesa 16 
iz’uokpd fpgu (?) ds LFkku ij vk ldrh gSaA  
18 
9 8 7
6 7 8 Similarly:-
4 5 3 21 
 
50 ? 53 11
II column   
rd
(a) 52 (b) 61 (c) 51 (d) 47 ? 
Sol. (c)  
22 
A 21  22  11  ?  462  11  ?
 
B
Pattern:-    AB  C  D ?
462
 42
C  11
 
D  Correct option (a)
A
st
I column  
B
Concept:4*4 Matrix Combination:    A  C   D  B
9  C 
   
6   9  6  4  54  4  50 D 
4  88. Study the given pattern carefully and select the number
 
50 that can replace the question mark (?) in it.
fn, x, iSVuZ dks /;ku ls i<+as vkSj ml la[;k dk p;u djs]a tks blesa
7 
  iz’uokpd fpgu (?) ds LFkku ij vk ldrh gSaA
8
III column    7  8  3  56  3  53
nd
798
3  97 73 64
 
53 67?
Similarly:- 55 10 48
rd (a) 2 (b) 5 (c) 4 (d) 3
II column 
Sol. (a)
8 
  A
7   8  7  5  56  5  51  
B
5  Pattern:-    A  C   D  B
  C 
?   
D 
Correct option (c)
A st
  I column
B
Concept:4*4 Matrix Combination:    A  D  B  C  7 
C   
  97  (7  6)  55  42  55  97
D  6 
87. Study the given pattern carefully and select the number  
that can replace the question mark (?) in it. 55 

218
Best App for Govt. Jobs : Neonclasses (Download Now)

9  A
   
73 B
II column    (9  7)  10  63  10  73 Concept:4*4 Matrix Combination:    B  C   A  D
nd
7  C 
   
10 D 
Similarly:- 90. Study the given pattern carefully and select the number
rd
III column  that can replace the question mark (?) in it.
fn, x, iSVuZ dk è;kuiwodZ vè;;u djsa vkSj ml la[;k dk p;u djsa
8 
  tks mlesa ç'uokpd fpàksa dks çfrLFkkfir dj ldsA
64   (8  2)  48  16  48  64 10 4 7 5
?  15 7 8 25
 
 48  14 3 6 12
or 10 ? 14 65
(8  ?)  48  64  (8  ?)  64  48 (a) 16 (b) 14
16 (c) 10 (d) 18
 8  ?  16  ?  2 Sol. (a)
8
Correct option (a) A
 
B
A Pattern:-    B  C   A  D
  C 
B  
Concept:4*4 Matrix Combination:   B  C  D  A  D 
C 
 
D 
st
89. Study the given pattern carefully and select the number I column
that can replace the question mark (?) in it. 10 
fn, x, iSVuZ dk è;kuiwoZd vè;;u djsa vkSj ml la[;k dk p;u  
15   (15  14)  10  1  10  10
djsa tks mlesa ç'uokpd fpà ¼\½ dks çfrLFkkfir dj lds 14 
29 71 64  
10 
8 11 14
4 7 ? 7 
 
3 6 3 8
III column    (8  6)  7  2  7  14
nd

(a) 4 (b) 3 6 
 
(c) 5 (d) 6 14 
Sol. (c) th
IV column
A
  5 
B  
Pattern:-   B  C  D  A 
C  25  (25  12)  5  13  5  65
  12
D   
65
29
  Similarly:-
st
I Column 8   8  4  3  32  3  29 rd
II column 
4 
  4
3   
7   (7  3)  4  4  4  16
71 3 
   
11
II Column    11  7  6  77  6  71 ? 
nd
7 
  Correct option (a)
6 
Similarly:-
Number Analogy
Concept: Cube: N : N3
64 
  01. Select the option that is related to the third number in the
14
III Column   
rd
same way as the second number is related to the first
?  number.
 
6  ml fodYi dk p;u djsa tks rhljh la[;k ls mlh çdkj lacfa èkr gS
14  ? 6  64  14  ?  64  6 tSls nwljh la[;k igyh la[;k ls lacafèkr gSA
70 6 : 216 :: 8 : ?
 14  ?  70  ?  5 (a) 64 (b) 154 (c) 1024 (d) 512
14
Pattern : - [CUBE, N : N3]
14 × 5 – 6 70 – 6 = 64 
 6  ××× 6 = 216
Correct option (c)
Similarly : -

219
Download Free PDFs & e-Books from Neon Classes App

 ml la[;k&;qXe dk p;u djsa ftlesa nks la[;k,¡ mlh çdkj lacafèkr


8  ××× 8 = 512
Correct Answer Option (d) gSa tSls nh xbZ tksM+h esa nks la[;k,¡ gSaA
Concept: CUBE: [N : N3+k] , K will be any random Number 65 : 126
02. (a) 26 : 65(b) 2 : 10
4:69:: 11: ? (c) 37 : 50 (d) 9 : 28
(a) 1029 (b) 198 (c) 176 (d) 1336 Sol. (d)
Sol. (d) Pattern : - [N3+1 : (N + 1)3+1 ]
Pattern : - [N : N3+5] 65:12643+1 : (4+1)3+1→ 64 + 1 : 53 + 1
4 : 69 :: 11 : ?  
4 : 43 + 5  64 + 5   ⇒9:28
Similarly : -
 
11 : 113 + 5  1331 + 5 
 
Correct Answer Option (d)
Correct Answer Option (d)
N
Concept : CUBE : [N : N3+ ] Concept : Cube: [(N + k)3 + (N + k) : N3 + N], where k can be
2 any number
03. 07.
8 : 516 :: 6 : 219 :: 4 : ? 222: 130
(a) 69 (b) 72 (c) 66 (d) 68 (a) 125 :27 (b) 350 : 49
N (c) 68 : 30 (d) 25 : 36
Pattern : - [N : N3+ ]
2 Sol. (c)
8 Pattern : - [(N + 1)3 + (N + 1) : N3 + N]
8 : 83 +  512 + 4 
2 222 : 130 53 + 5 (N=5)
6 (5+1)3 +(5+1) 63 + 6  216 + 6 = 222
  216 + 3 = 219
2 Similarly :
Similarly : - 68 : 30 33 + 3 (N=3)
4 (3+ 1)3 + (3 + 1)  43 + 4  64 + 4 = 68
4 : 43 +  64 + 2 = 
2 Correct Answer Option (c)
Correct Answer Option (c) Concept : CUBE: [N : N3-k], K will be any random Number
Concept: CUBE :[N : (N + k)3]; k ,will be any random
number 08.
04. 5 : 117 :: 6 : 208 :: 8 : ?
7 : 512 :: ? : 2744 :: 15 : 4096 (a) 405 (b) 450 (c) 504 (d) 540
(a) 21 (b) 8 (c) 17 (d) 13 Sol. (c)
Sol. (d) Pattern : - [N : N3 – 8]
Pattern : - [N : (N + 1)3] 5 : 53 – 8  125 – 8 = 117
7 : (7 + 1)3  83 = 512 6 : 63 – 8  216 – 8 = 208
 15 : (15 + 1)3 163 = 4096 Similarly :
Similarly : - 8 : 83 – 8  512 – 8 = 504
13 : (13 + 1)3  143 = 2744 Correct Answer Option (c)
Or Concept : Cube: [N : N3 – N]
3 2744 - 14 – (14 – 1) = 13 09. Select the option that is related to the fifth number in the
same way as the second number is related to the first
Correct Answer Option (d)
number and the fourth number is related to the third
Concept : Cube: - [(N + k)3 + N : N] , k can be any number
number.
05.
ml fodYi dk p;u djsa tks ikapoha la[;k ls mlh çdkj lacafèkr gS
1008 : 8 :: ? : 6 :: 220 : 4
tSls nwljh la[;k igyh la[;k ls lacafèkr gS vkSj pkSFkh la[;k rhljh
(a) 518 (b) 350 (c) 443 (d) 524
Sol. (a)
la[;k ls lacafèkr gSA
Pattern : - [(N + 2)3 + N : N] (NOTE: Operations should be performed on the whole
numbers, without breaking down the numbers into its
8:[(8+2)3 +8]103+81008+ 8 = 1008
constituent digits. E.g. 13 – Operations on 13 such as adding
4:[(4+2)3 + 4]  63 + 4  216 + 4 = 220 /deleting /multiplying etc. to 13 can be performed. Breaking
Similarly : down 13 into 1 and 3 and then performing mathematical
6:[(6+2)3 + 6]  83 + 6  512 + 6 = 518 operations on 1 and 3 is not allowed)
Correct Answer Option (a) ¼uksV % la[;kvksa dks mlds ?kVd vadksa esa foHkkftr fd, fcuk] iw.kZ
CUBE : [N3+1 : (N + 1)3+1 ] la[;kvksa ij lapkyu fd;k tkuk pkfg,A mnkgj.k ds fy, 13&13
06. Select the number-pair in which the two numbers are ij lapkyu tSls fd tksMus@gVkus@xq.kk djus vkfn dks 13 esa fd;k
related in the same way as are the two numbers in the tk ldrk gSA 13 dks 1 vkSj 3 esa foHkkftr fd;k tk ldrk gSSA vkSj
given pair. fQj 1 vkSj 3 ij xf.krh; lafØ;kvksa dks djus dh vuqefr ugha gSa½

220
Best App for Govt. Jobs : Neonclasses (Download Now)

2 : 6 :: 4 : 60 :: 7 : ? 14.
(a) 600 (b) 201 (c) 105 (d) 336 11 : 1309 :: 15 : 3345 :: 7: ?
Sol. (d) (a) 329 (b) 333 (c) 330 (d) 335
Pattern : - [N : N3 – N] Sol. (a)
2 : 23 – 2  8 – 2 = 6 Pattern : - [N : N3 – 2N]
4 : 43 – 4  64 – 4 = 60 11 : 113 – 2 × 11  1331 – 22  1309
Similarly : 15 : 153 – 2 × 15  3375 – 30  3345
7 : 73 – 7  343 – 7 = 336 Similarly :
Correct Answer Option (d) 7 : 73 – 2 × 7  343 – 14  329
CUBE [ N : (N – k)3], where k can be any number Correct Answer Option (a)
10. Concept : Cube: -[N : N3 × k], where k can be any number
13 : 1331 : : 17 : ? 15.
(a) 2642 (b) 1453 (c) 3375 (d) 1829 3 : 51 :: 7 : ?
Sol. (c) (a) 319 (b) 609 (c) 335 (d) 679
Pattern : - [N : (N – 2)3] Sol. (d)
13 : 1331 :: 17 : ? Pattern : - [N : N3 × 2 – N]
13 : (13 – 2)3  113 = 1331 3 : 33 × 2 – 3  27 × 2 – 3  54 – 3 = 51
Similarly : Similarly :
17 : (17 – 2)3  153 = 3375 7 : 73 × 2 – 7  343 × 2 – 7  686 – 7 = 679
Correct Answer Option (c) Correct Answer Option (d)
Concept : Cube: - [N : (N – k) 3 + N], where k can be any Concept : CUBE : X3 : Y3 (both numbers are perfect cube)
number 16. Select the number-pair in which the two numbers are
11. related in the same way as are the two numbers of the
4 : 31::7 : 223::6 : ? given number pair.
(a) 137 (b) 133 (c) 123 (d) 131 ml la[;k&;qXe dk p;u djsa ftlesa nks la[;k,¡ mlh çdkj lacafèkr
Sol. (d) gSa tSls nh xbZ la[;k ;qXe dh nks la[;k,¡ gSaA
Pattern : - [N : (N – 1) 3 + N] 1:8
4 : [(4 – 1)3 + 4]  33 + 4  27 + 4 = 31 (a) 8 : 27 (b) 6 : 30
7:[(7 – 1)3 + 7] 63 + 7  216 + 7 = 223 (c) 16 : 49 (d) 9 : 36
Similarly : Sol. (a)
Pattern : - [CUBE, X3 : Y3]
6:[(6 – 1)3 + 6] 53 + 6  125 + 6 = 131  
1 : 8   ××××
Correct Answer Option (d)
Concept : Cube: - [N3 – k : N + k], where k can be any Similarly : -
 
number 8 :27  ×××3×3=8 : 27
12. Correct Answer Option (a)
121 : 9 :: 212 : 10 :: 508 : ? Concept : CUBE: [X3+n : Y3+n]
(a) 10 (b) 13 (c) 12 (d) 11 17.
Sol. (c) 223 : 350 :: 519 : ?
Pattern : - [N3 – 4 : N + 4] (a) 736 (b) 645 (c) 687 (d) 654
53 – 4 : 5 + 4  125 – 4 : 9  121 : 9 Sol. (a)
Pattern : - [X3+7 : Y3+7]
63 – 4 : 6 + 4  216 – 4 : 10  212 : 10
Similarly : 63 + 7 : 73 + 7 :: 83 + 7 : 93 + 7
508 : ?, 508 + 4 = 512 ,
(9 × 9 × 9 + 7)  729 + 7 = 736
3 512 8, 8 + 4 = 12
Correct Answer Option (c) Correct Answer Option (a)
Concept : Cube: [N : (N – k) 3 + Y], where k & Y, can be any Concept : CUBE: [N : N3 – (N – 1)3], difference between
number cubes of consecutive numbers
13. 18.
7:9::8:?::6:2 3 : 19::2 : 7::5 : ?
(a) 25 (b) 28 (c) 22 (d) 26 (a) 69 (b) 51 (c) 59 (d) 61
Sol. (b) Sol. (d)
Pattern : - [N : (N – 5) 3 + 1] Pattern : - [N : N3 – (N – 1)3]
3
7 : (7 – 5)3 + 1  23 + 1  8 + 1 = 9 333 – (3–1)3  27 –2  27 – 8 = 19
3
6 : (6 – 5)3 + 1  13 + 1  1 + 1 = 2 2  23 – (2 – 1)3  8 –1  8 – 1 = 7
Similarly : Similarly :
8 : (8 – 5)3 + 1  33 + 1  27 + 1 = 28 3
5  53 – (5 – 1)3  125 –4  125 – 64 = 61
Correct Answer Option (b) Correct Answer Option (d)
Concept : Cube: - [N : N3 – kN], where k can be any number

221
Download Free PDFs & e-Books from Neon Classes App

Concept : CUBE: [(N – k1)3 – k2 : N], where k1 and k2 can 22.


be any numbers 108 : 6 :: 256 : ?
19. Select the same way as the first term is related to the (a) 16 (b) 12 (c) 8 (d) 9
second term. Sol. (c)
ml fodYi dk p;u djsa tks pkSFks in ls mlh çdkj lacafèkr gS tSls N3
igyk in nwljs in ls lacafèkr gSA Pattern : - [ : N]
2
213 : 7 : : _______ : 8 108 : 6 :: 256 : ?
(a) 343 (b) 512 (c) 340 (d) 509
63 216
Sol. (c) 6   =108
Pattern : - [(N – 1)3 – 3 : N] 2 2
3 Similarly : -
7  (7–1)3– 3  6 – 3  216 – 3 = 213
Similarly : 83 512
8   = 256
3 2 2
8  (8 – 1)3 – 3  7 – 3  343 – 3 = 340
Correct Answer Option (c)
Correct Answer Option (c) 
Concept : CUBE (A,B,C) C ]
Concept : CUBE- [
X  Y ]Eg.
3
 Z3 
23. Select the set in which the numbers are related in the
n same way as are the numbers of the following sets.
20. Select the option in which the numbers are related in the ml lsV dk p;u djsa ftlesa la[;k,a mlh rjg lacaf/kr gSa tSls
same way as are the numbers of the following set. fuEufyf[kr lsV dh la[;k,a gSaA
mu fodYiksa dk p;u djsa ftuesa la[;k,¡ mlh çdkj lacafèkr gSa tSls (NOTE : Operations should be performed on the whole
fuEufyf[kr lsV dh la[;k,¡ gSaA numbers, without breaking down the numbers into its
(4, 532, 10) constituent digits. E.g. 13- Operations on 13 such as adding
(a) (2, 121, 6) (b) (9, 396, 4) /subtracting /multiplying etc. to 13 can be performed.
(c) (4, 140, 6) (d) (3, 269, 8) Breaking down 13 into 1 and 3 and then performing
Sol. (c) mathematical operations on 1 and 3 is not allowed.)

Pattern : - [
X 3
 Z3 
Y ]
¼uksV: la[;kvksa dks muds ?kVd vadksa esa foHkkftr fd, fcuk] iw.kZ
2 la[;kvksa ij lapkyu fd;k tkuk pkfg,A mnkgj.k ds fy, 13& 13
(4, 532, 10) 
ij lapkyu tSls 13 dks tksM+uk / ?kVkuk / xq.kk djuk vkfn fd;k
tk ldrk gSA 13 dks 1 vkSj 3 esa rksM+uk vkSj fQj 1 vkSj 3 ij

4 3
 103 

64  1000
,
1064
= 532 xf.krh; lafØ;k,¡ djus dh vuqefr ugha gSA½
2 2 2 (6, 12, 228), (8, 3, 515)
Similarly : - (a) (3, 28, 56) (b) (13, 11, 172)
(4, 140, 6)  (c) (4, 18, 82) (d) (9, 23, 119)


4 3
 63 

64  216  280
, = 140
Sol. (c)

2 Pattern : (A,B,C) C ]
2 2
(6, 12, 228)
Correct Answer Option (c)
(6)3+12216 + 12 = 228
N3
Concept : CUBE: [N :
]  (8, 3, 515)
2
21. Select the number-pair in which the two numbers are
(8)3+3 512 + 3 = 515
related in the same way as are the two numbers of the Similarly : -
following number-pair (4, 18, 82)
ml la[;k&;qXe dk p;u djsa ftlesa nks la[;k,¡ mlh çdkj lacaf/kr (4)3+18 64 + 18 = 82
gSa tSls fuEufyf[kr la[;k&;qXe dh nks la[;k,¡ gSaA  Correct Answer Option (c)
 
4 : 32 Concept (A,B,C)  C ]
(a) 8 : 248 (b) 5 : 62 24. Select the set in which the numbers are related in the
(c) 10 : 160 (d) 6 : 108 same way as are the numbers of the given sets.
Sol. (d) ml leqPp; dk p;u djsa ftlesa la[;k,¡ mlh çdkj lacaf/kr gSa tSls
N3 fn, x, leqPp;ksa dh la[;k,¡ gSaA
Pattern : - [N : ] (NOTE : Operations should be performed on the whole
2
3 numbers, without breaking down the numbers into its
4 64
4 : 32  4 :  =32 constituent digits. E.g. 13 – Operations on 13 such as adding
2 2 /subtracting /multiplying etc. to 13 can be performed.
Similarly : - Breaking down 13 into 1 and 3 and then performing
63 216 mathematical operations on 1 and 3 is NOT allowed)
6:108  =108
2 2 ¼uksV& la[;kvksa dks muds ?kVd vadksa esa foHkkftr fd, fcuk] iw.kZ
Correct Answer Option (d) la[;kvksa ij lapkyu fd;k tkuk pkfg,A mnkgj.k ds fy, 13 & 13
N3 ij lapkyu tSls 13 dks tksM+uk @ ?kVkuk @ xq.kk djuk vkfn fd;k
Concept : CUBE: [ : N] tk ldrk gSA 13 dks 1 vkSj 3 esa rksM+uk vkSj fQj 1 vkSj 3 ij
2
xf.krh; lafØ;k,¡ djus dh vuqefr ugha gSA½

222
Best App for Govt. Jobs : Neonclasses (Download Now)

(2, 3, 35) (11, 6, 1547) N2


(a) (4, 15, 3439) (b) (10, 9, 1730) Pattern : - [N : ]
9
(c) (7, 8, 2565) (d) (3, 12, 1296)
272 27  27
Sol. (a) 27    27  3  81
  9 9
Pattern : (A,B,C)  C ]
(2, 3, 35) 182 18  18
18    18  2  36
9 9
23 + 33 8 + 27 = 35
Similarly :
(11, 6, 1547)
332 33 33 1089
113 + 63 1331 + 216 = 1547 33     121
9 9 9
Similarly :
(4, 15, 3439) Correct Answer Option (a)
Concept : SQUARE: [N : N2 + k], where k can be any number
43 + 153 64 + 3375 = 3439 29.
Correct Answer Option (a)
16 : 263 :: 18 : ? :: 22 : 491
Concept : CUBE: [Digit : (Add Digit)3]
(a) 317 (b) 361 (c) 331 (d) 340
25. Select the option in which the two numbers are related in
Sol. (c)
the same way as are the two numbers of the following
Pattern : - [N : N2 + 7]
number-pair.
16 : 162 + 7  256 + 7 = 263
ml fodYi dk p;u djsa ftlesa nks la[;k,a ,d&nwljs ls mlh izdkj
lacaf/kr gSa tSls fuEufyf[kr la[;k&;qXe dh nks la[;k,a ,d&nwljs ls 22 : 222 + 7  484 + 7 = 491
lacaf/kr gSaA Similarly :
25 : 343 18 : 182 + 7  324 + 7 = 331
(a) 24 : 216 (b) 29 : 121 Correct Answer Option (c)
(c) 34 : 510 (d) 30 : 729 Concept : SQUARE: [N + k : N2], where k can be any number
Sol. (a) 30.
Pattern : - [Digit : (Add Digit)3] 11 : 81 : : ? : 121 : : 8 : 36
25 : 343  (2 + 5)3 73  (a) 12 (b) 10 (c) 18 (d) 13
Similarly : - Sol. (d)
Pattern : - [N + 2 : N2]
24 : 216  (2 + 4)3 63  2
Correct Answer Option (a) 11 : 9 + 2  9 = 81
2
Concept : SQUARE: [X : X2] 8 : 6 + 2  6 = 36
26. Similarly :
110 : 12100 : : 170 : ?
? : 121  121  11  11 + 2 = 13
(a) 2890 (b) 28900(c) 18900(d) 36100
Correct Answer Option (d)
Sol. (b)
Concept : SQUARE: [N : (N + k)2], where k can be any
Pattern : - [X : X2]
number
110  (110)2 = 12100
31.
Similarly : 19 : 400 : : 24 : ____
170  (170)2 = 28900 (a) 566 (b) 625 (c) 676 (d) 652
Correct Answer Option (b) Sol. (b)
Concept : SQUARE: 2 X : X (square root of number is Pattern : - [N : (N + 1)2]
related to number) 2
19  (19 + 1)  20 = 400
2

27. Similarly :
2809 : 53 :: 1024 : ? 2 2
24  (24 + 1)  25 = 625
(a) 35 (b) 33 (c) 31 (d) 32
Correct Answer Option (b)
Sol. (d)
Concept : SQUARE: [X2 : (X + k)2], where k can be any
Pattern : - 2 X : X
number
2
2809 = 53 32. Select the number-pair in which the two numbers are
Similarly : related in the same way as are the two numbers of the
2
given number pair.
1024 = 32 ml la[;k&;qXe dk p;u djsa ftlesa nks la[;k,¡ mlh çdkj lacafèkr
Correct Answer Option (d) gSa tSls nh xbZ la[;k ;qXe dh nks la[;k,¡ gSaA
N2 36:49
Concept : SQUARE: [N : ], where k can be any number
k (a) 25 : 64(b) 289 : 324
28. (c) 256 : 361 (d) 16 : 36
27:81 :: 18:36 :: 33:? Sol. (b)
(a) 121 (b) 135 (c) 125 (d) 131 Pattern : - [X2 : (X + 1)2]
Sol. (a) 36 : 49  62 : (6 + 1)2  62 : 72 =36 : 49
Similarly :

223
Download Free PDFs & e-Books from Neon Classes App

289 : 324  172 : 182 196 : 196  10  14 + 10 = 24


Correct Answer Option (b) Similarly :
Concept : SQUARE: [N : N2 + (N + 1)2]
529 : 529  10  23 + 10 = 33
33.
11 : 265 :: 20 : ? Correct Answer Option (d)

 
2
(a) 385 (b) 841 (c) 388 (d) 480 Concept : SQUARE: [X : X  k ],
Sol. (b)
Pattern : - [N : N2 + (N + 1)2] 37.
2 2 2 289 : 324 :: 441 : ?
11 : 11 + (11 + 1)  121 + 12  121 + 144 = 265
(a) 483 (b) 484 (c) 401 (d) 529
Similarly :
Sol. (b)
2 2 2
20 : 20 + (20 + 1)  400 + 21  400 + 441 = 841
 X  1 ]
2

Correct Answer Option (b) Pattern : - [X :


Concept : SQUARE: - [N : (N + k)2+(N + 2k)2], where k can
289:  289  1 (17+1)
2

be any number
2  182 = 324
34. Select the option in which the numbers are related in the Similarly :
same way as are the numbers in the given set .
 
2

ml fodYi dk p;u djsa ftlesa la[;k,a mlh rjg lacfa /kr gSa tSls 441: 441  1 (21+1)2  222 = 484
fn, x, lsV esa la[;k,a gSaA Correct Answer Option (b)
(18, 400, 484) Concept : Concept:SQUARE: [N2 : (N + k)2+k],
(a) (20, 400, 576) (b) (14, 256, 400) where k can be any
(c) (24, 676, 784) (d) (22, 400, 324) 38.
Sol. (c) 196: 226:: 256: 290:: 324: ?
Pattern : - [N : (N + 2)2+(N + 4)2] (a) 358 (b) 362 (c) 352 (d) 372
(18, 400, 484) Sol. (b)
2
18  (18 + 2)  (18+4)
2 Pattern : - [N2 : (N + 1)2+1]
18 : 202 : 222 196 = 14, (14 +1)2 + 1  152 + 1 225+1 = 226
18 : 400 : 484
256 = 16, (16 +1)2 + 1  172 + 1 289+1 = 290
Similarly :
(24, 576, 784) Similarly :
2
24  (24 + 2)  (24+4)
2 324 = 18, (18 +1)2 + 1  192 + 1 361+1 = 362
24 : 262 : 282 Correct Answer Option (b)
24 : 576 : 784
Correct Answer Option (c) Concept :SQUARE: [N2 + N : (N + k)2 + (N + k)]
Concept : SQUARE: [X : X2+k1 : X+k2], where k1 and k2 are 40. Select the option that is related to the fifth number in the
any two numbers same way as the second number is related to the first
35. Select the option in which the numbers are NOT related in number and fourth number is related to third number.
the same way as are the numbers of the following set. ml fodYi dk p;u djsa tks iakpoh la[;k ls mlh izdkj lacaf/kr gS
ml fodYi dk p;u djsa ftlesa la[;k,a mlh rjg lacafèkr ugha gSa tSls nwljh la[;k igyh la[;k ls lacaf/kr gS vkSj pkSFkh la[;k rhljh
tSls fuEufyf[kr lsV dh la[;k,a gSaA la[;k ls lacaf/kr gSA
(7, 51, 22) (NOTE: Operations should be performed on the whole
(a) (11, 123, 26) (b) (6, 38, 22) numbers, without breaking down the numbers into its
(c) (3, 11, 18) (d) (4, 18, 19) constituent digits. E.g. 13 – Operations on 13 such as adding
Sol. (a) /deleting /multiplying etc. to 13 can be performed. Breaking
Pattern : - [X : X2+2 : X+15] down 13 into 1 and 3 and then performing mathematical
(7, 51, 22) 7 72 + 2 : 7 + 15 72 : 22 = operations on 1 and 3 is not allowed)
¼uksV % la[;kvksa dks mlds ?kVd vadksa esa foHkkftr fd, fcuk] iw.kZ
722
la[;kvksa ij lapkyu fd;k tkuk pkfg,A mnkgj.k ds fy, 13&13
Similarly :
ij lapkyu tSls fd tksMus@gVkus@xq.kk djus vkfn dks 13 esa fd;k
(11, 123, 26) 11 112 + 2 : 11 + 15 112 : 26 =
tk ldrk gSA 13 dks 1 vkSj 3 esa foHkkftr fd;k tk ldrk gSSA vkSj
1126 fQj 1 vkSj 3 ij xf.krh; lafØ;kvksa dks djus dh vuqefr ugha gSa½
Correct Answer Option (a) 20 : 30 :: 42 : 56 :: 72 : ?
Concept : SQUARE: [X : X + n], (a) 82 (b) 90 (c) 84 (d) 96
36. Sol. (b)
64 : 18 :: 196 : 24 :: 529 : ? Pattern : - [N2 + N : (N + 1)2 + (N + 1)]
(a) 30 (b) 35 (c) 39 (d) 33 20 : 30  42 + 4 : 52 + 5
Sol. (d) 16 + 4 : 25 + 5  20 : 30
Pattern : - [X : X + 10] 42 : 56  62 + 6 : 72 + 7
64 : 64  10  8 + 10 = 18 36 + 6 : 49 + 7  42 : 56

224
Best App for Govt. Jobs : Neonclasses (Download Now)

Similarly : 14
737, 541
72 : ?  82 + 8 : 92 + 9
64 + 8 : 81 + 9  72 : 90 737 - 541 = 196
Correct Answer Option (b)
Concept:SQUARE: [N : N2 – N] (541, 14, 737)  (541 – 737)  196 = 14
41. 16 : 240 :: 6 : ? Similarly :
(a) 30 (b) 25 (c) 40 (d) 35 (697, 13, 866)  (697 – 866)  169 = 13
Sol. (a) Correct Answer Option (c)
Pattern : - [N : N2 – N]
16  162 – 16 256 – 16 = 240 Concept : SQUARE: X : Y , X - Y = 3
Similarly : 46. Select the option that is related to the sixth number in the
same way as the first number is related to the second
6  62 – 6 36 – 6 = 30
number and the third number is related to the fourth
Correct Answer Option (a)
number.
Concept : SQUARE: [N : N2 – k], where k can be any number ml fodYi dk p;u djsa tks NBh la[;k ls mlh çdkj lacfa èkr gS
42. 5 : 7 :: ? : 271 :: 22 : 466
tSls igyh la[;k nwljh la[;k ls lacafèkr gS vkSj rhljh la[;k pkSFkh
(a) 22 (b) 24 (c) 17 (d) 18
la[;k ls lacafèkr gSA
Sol. (c)
144 : 225 :: 169 : 256 :: ? : 361
Pattern : - [N : N2 – 18]
(a) 324 (b) 256 (c) 196 (d) 144
5  52 – 18 25 – 18 = 7 Sol. (b)
22  222 – 18 484 – 18 = 466
Pattern: - [ X : Y , X - Y = 3 (Difference)]
Similarly :
17  172 – 18 289 – 18 = 271 144 : 225  12 ; 15 = 3

or 271 + 18  289, 289 = 17 169 : 256 13 : 16 = 3


Correct Answer Option (c) Similarly:
Concept : SQUARE: Pattern: - [N : (N – k)2], ? : 361  361  19
43.
19 – 3 = 16  162 = 256
23 : 441 :: 28 : ?
Correct Answer Option (b)
(a) 692 (b) 494 (c) 528 (d) 676
Sol. (d) Concept : (A,B,C)  [ (B)2 - (A)2 = C ]
Pattern: - [N : (N – 2)2] 47. Select the option in which the numbers are related in the
same way as are the numbers of the following set.
23  (23 – 2)2212 = 441
ml fodYi dk p;u djsa ftlesa la[;k,W mlh izdkj lacaf/kr gSa tSls
Similarly :
fuEufyf[kr lsV dh la[;k,¡ gSa
28 (28 – 2)2  262 = 676
(6, 9, 45)
Correct Answer Option (d) (a) (2, 8, 46) (b) (6, 8, 60)
Concept : SQUARE: [N : N2–(N–k)2], where k can be any (c) (7, 4, 48) (d) (8, 10, 36)
number
Sol. (d)
44. 8 : 48 :: 19 : ? :: 16 : 112 Pattern : [ (B)2 - (A)2 = C ]
(a) 136 (b) 142 (c) 145 (d) 138
(6, 9, 45)(9)2 – (6)281-36 = 45
Sol. (a)
Pattern : - [N : N2–(N–4)2] Similarly :

8  82–(8–4)2  64–(4)2  64 – 16 = 48 (8, 10, 36)(10)2 – (8)2100-64 = 36


Correct Answer Option (d)
16  162–(16–4)2  256–(12)2  256 – 144 = 112
Concept : SQUARE: [N : (N – 1)2 : (N+1)2],
Similarly : 48.. Select the option in which the numbers are related in the
19  192–(19–4)2  361–(15)2  361 – 225 = 136 same way as are the numbers of the following set.
Correct Answer Option (a) ml fodYi dk p;u djsa ftlesa la[;k,W mlh izdkj lacaf/kr gSa tSls
Concept : SQUARE:[X - Z = Y ] fuEufyf[kr lsV dh la[;k,¡ gSa
45. Select the options in which the numbers are related in the (12, 121, 169)
same way as are the numbers of the following set. (a) (11, 144, 196) (b) (17, 256, 324)
mu fodYiksa dk p;u djsa ftuesa la[;k,¡ mlh çdkj lacafèkr gSa tSls (c) (14, 196, 225) (d) (16, 161, 256)
fuEufyf[kr lsV dh la[;k,¡ gSaA Sol. (b)
(541, 14, 737) Pattern : - [N : (N – 1)2 : (N+1)2]
(a) (832, 8, 895) (b) (635, 18, 924) (12, 121, 169)
(c) (697, 13, 866) (d) (432, 25, 1108) 12 (12 - 1)2 : (12 + 1) 2
Sol. (c) 12 : 112 : 132
Y] 12 : 121 : 169
Pattern : - [X - Z =
Similarly :
(17, 256, 324) 17 (17-1)2 :(17 + 1) 2

225
Download Free PDFs & e-Books from Neon Classes App

17 : 162 : 182 ml fodYi dk p;u djsa tks pkSFkh la[;k ls mlh çdkj lacafèkr gS
17 : 256 : 324 tSls igyh la[;k nwljh la[;k ls lacafèkr gS vkSj ikapoha la[;k NBh
Correct Answer Option (b) la[;k ls lacafèkr gSA
Concept : SQUARE: [N : (N+k1)2–k2], where k1 and k2 can 64 : 4 :: ? : 6 :: 100 : 5
be any numbers (a) 108 (b) 188 (c) 228 (d) 144
49. Select the option that is related to the third term in the Sol. (d)
same way as the second term is related to the first term Pattern : - [(2N)2 : N]
and the sixth term is related to the fifth term. 
4 : (4 × 2)2  = 64
ml fodYi dk p;u djsa tks rhljs in ls mlh çdkj lacafèkr gS ftl 
5 : (2 × 5)2  = 100
izdkj nwljk in igys in ls lacaf/kr gS vkSj NBk in ikaposa in ls
Similarly :
lacafèkr gSA 
5 : 35 :: 9 : ? :: 14 : 224 6 : (2 × 6)2  = 144
(a) 99 (b) 113 (c) 107 (d) 87 Correct Answer Option (d)
Sol. (a) Concept : SQUARE: Pattern: - [N : (N+1)2×k] , where k can
Pattern : - [N : (N+1)2–1] be any number
53. Select the option in which the two numbers are related in
5  (5+1)2–1  62–1  36 – 1 = 35
the same way as are the two numbers of the following
14  (14+1)2–1  152–1  225 – 1 = 224 number-pair.
Similarly : ml fodYi dk p;u djsa ftlesa nks la[;k,a mlh rjg lacafèkr gSa tSls
9  (9+1)2–1  102–1  100 – 1 = 99 fuEufyf[kr la[;k&;qXe dh nks la[;k,a gSaA
Correct Answer Option (a) 14 : 450
Concept : SQUARE: [N : N2+2N –2], [N:(N+1)2–k], where k (a) 31 : 961 (b) 30 : 902
can be any number (c) 22 : 490 (d) 18 : 722
50. Select the option that is related to the fifth number in the Sol. (d)
same way as the second number is related to the first Pattern: - [N : (N+1)2×2]
number and the fourth number is related to the third 2
14 : 450  (14 + 1)2 15 × 2 = 450
number.
Similarly:
ml fodYi dk p;u djsa tks ikapoha la[;k ls mlh çdkj lacafèkr gS 2
tSls nwljh la[;k igyh la[;k ls lacafèkr gS vkSj pkSFkh la[;k rhljh 18 : 722  (18 + 1)219 × 2 = 722
la[;k ls lacafèkr gSA Correct Answer Option (d)
SQUARE: [N : N2 × k1 + k2], where k1 and k2 can be any
5 :33 :: 11 : 141 :: 7 : ?
two numbers
(a) 58 (b) 50 (c) 61 (d) 56
54. Select the option that is related to the third number in the
Sol. (c)
same wayas the second numberis related to the first
Pattern : - [N : N2+2N –2]
number.
5  52+2×5–2  25+10–235 – 2 = 33
ml fodYi dk p;u djsa tks rhljh la[;k ls mlh çdkj lacfa èkr gS
11  112+2×11–2  121+22–2  143 – 2 = 141 ftl çdkj nwljh la[;k igyh la[;k ls lacfa èkr gSA
Similarly: 3 : 20 :: 7 : ?
7  72+2×7–2  49+14–2  63–2 = 61 (a) 105 (b) 96 (c) 100 (d) 98
Correct Answer Option (c) Sol. (c)
SQUARE: [N : N2 × k] , K can be any number Pattern: - [N : N2 × 2 + 2]
51. Select the number that is related to the third number in 3 : 32×2+2  9×2+2  18 + 2 = 20
the same way as the second number is related to the first Similarly:
number.
7 : 72×2+2  49×2+2  98 + 2 = 100
ml la[;k dk p;u djsa tks rhljh la[;k ls mlh çdkj lacfa /kr gS
Correct Answer Option (c)
tSls nwljh la[;k igyh la[;k ls lacaf/kr gSA Concept : SQUARE: [N : k1N2+k2], where k1 and k2 can be
14 : 392 : : 21 : ? any two numbers
(a) 691 (b) 778 (c) 882 (d) 880 55. Select the option that is related to the fifth number in the
Sol. (c) same way as the second number is related to the first
Pattern : - [N : N2 × 2] number and the fourth number is related to the third

14 : 142 × 2 × 2 = 392 number.
Similarly : ml fodYi dk p;u djsa tks ikapoha la[;k ls mlh çdkj lacafèkr gS

21 : 212 × 2 × 2 = 882 tSls nwljh la[;k igyh la[;k ls lacafèkr gS vkSj pkSFkh la[;k rhljh
Correct Answer Option (c) la[;k ls lacafèkr gSA
Concept : SQUARE: [(kN)2 : N], where k can be any number 6 : 218::3 : 56::5 : ?
52. Select the option that is related to the fourth number in (a) 156 (b) 152 (c) 168 (d) 160
the same way as the first number is related to the second Sol. (b)
number and the fifth number is related to the sixth Pattern: - [N : 6N2+2]
number. 6  6×62+2  6×36+2  216 + 2 = 218
3  6×32+2  6×9+2  54 + 2 = 56

226
Best App for Govt. Jobs : Neonclasses (Download Now)

Similarly: 59. Select the option that is related to the fifth number in the
5  6×52+2  6×25+2  150 + 2 = 152 same way as the second number is related to the first
Correct Answer Option (b) number and the fourth number is related to the third
Concept : SQUARE: [N : 2N2+2N +1] number.
56. Select the option that is related to the third number in the ml fodYi dk p;u djsa tks ikapoha la[;k ls mlh çdkj lacafèkr gS
same way as the second number is related to the first tSls nwljh la[;k igyh la[;k ls lacafèkr gS vkSj pkSFkh la[;k rhljh
number and the sixth number is related to the fifth la[;k ls lacafèkr gSA
number. 9 : 102 :: 11 : 146 :: 13 : ?
ml fodYi dk p;u dhft, ftldk rhljh la[;k ls ogh laca/k gS (a) 198 (b) 200 (c) 175 (d) 171
tks nwljh la[;k dk igyh la[;k ls vkSj NBh la[;k dk ikapoha la[;k Sol. (a)
ls gSA Pattern : - [N : N2 + 2N+3]
7 : 113 :: 11 : ? :: 15 : 481 9  92 + 2 × 9 + 3  81 + 18 + 3 = 102
(a) 265 (b) 155 (c) 388 (d) 340 11  112 + 2 × 11 + 3  121 + 22 + 3 = 146
Sol. (a) Similarly :
Pattern: - [N : 2N2+2N +1]
13  132 + 2 × 13 + 3  169 + 26 + 3 = 198
7  2×72+2×7+1  2×49+14+1  98+14+1 = 113 Correct Answer Option (a)
15  2×152+2×15+1  2×225+30+1  450+30+1 = 481 Concept :
Similarly:
SQUARE: [  
X  Z  k  Y ]where k can be any number
11  2×112+2×11+1  2×121+22+1  242+22+1 = 265
Correct Answer Option (a) 60. Select the set in which the numbers are related in the
SQUARE: [N2–2(N–1) : N] same way as are the numbers of the following set.
57. Select the option that is related to the fourth number in ml leqPp; dk p;u djsa ftlesa la[;k,¡ mlh çdkj lacaf/kr gSa tSls
the same way as the first number is related to the second fuEufyf[kr leqPp; dh la[;k,¡ gSaA
number and the fifth number is related to the sixth (9, 35, 16)
number. (a) (16, 50, 64) (b) (36, 55, 25)
ml fodYi dk p;u djsa tks pkSFkh la[;k ls mlh çdkj lacafèkr gS (c) (81, 65, 36) (d) (25, 30, 4)
tSls igyh la[;k nwljh la[;k ls lacafèkr gS vkSj ikapoha la[;k NBh Sol. (b)
la[;k ls lacafèkr gSA Pattern : Square
26 : 6 :: ? : 11 :: 37 : 7 The pattern followed here is :- [  
X  Z 5  Y ]
(a) 101 (b) 122 (c) 82 (d) 145
Sol. (a) 9, 16 35
Pattern : - [N2–2(N–1) : N]
6  62–2 (6–1)  36–2(5)  36–10 = 26 =Y

7  72–2 (7–1)  49–2(6)  49–12 = 37 3 + 4 = 7×5


Similarly: (9, 35, 16)  ( 9 + 16 )×5 ××5 = 35
11  112–2 (11–1)  121–2(10)  121–20 = 101 Similarly :
Correct Answer Option (a)
(36, 55, 25)  ( 36 + 25 )×5 ××5 = 55
SQUARE: [N:(N+k1)2 _ k2] where k1 and k2 can be any
numbers Correct Answer Option (b)
58. Select the option that is related to the fifth number in the  N2  N 
Concept : SQUARE: [N :   ]
same way as the second number is related to the first  2 
number and the fourth number is related to the third
61. Select the option that is related to the third number in the
number.
same way as the second number is related to the first
ml fodYi dk p;u djsa tks ikapoha la[;k ls mlh çdkj lacafèkr gS
number and the sixth number is related to the fifth
tSls nwljh la[;k igyh la[;k ls lacafèkr gS vkSj pkSFkh la[;k rhljh
number.
la[;k ls lacafèkr gSA ml fodYi dk p;u djsa tks rhljh la[;k ls mlh çdkj lacfa èkr gS
10 : 142 :: 17 : 359 :: 21 : ?
tSls nwljh la[;k igyh la[;k ls lacafèkr gS vkSj NBh la[;k ikapoha
(a) 525 (b) 529 (c) 527 (d) 576
la[;k ls lacafèkr gSA
Sol. (c)
16 : 120 :: 18 : ? :: 22 : 231
Pattern : - [N : (N+2)2–2]
2 2
(a) 134 (b) 153 (c) 531 (d) 513
10 10 + 2) – 2 2 – 2  144 – 2 = 142 Sol. (b)
2 2
17 17+2) –2 –2361 – 2 = 359  N2  N 
Similarly : Pattern : - [N :   ]
2 2
 2 
21 21 + 2) – 2  – 2  529 – 2 = 527
256  16 240
Correct Answer Option (c) 16  169   = 120
2 2
Concept : SQUARE: [N : N2 + k1N+k2], where k1 and k2 can
be any two random numbers  222 -22  484-22 462
22    = 231
 2  2 2

227
Download Free PDFs & e-Books from Neon Classes App

Similarly :  N2  1 
Pattern : - [N :  ]
 182  18  324  18 306  2 
18     = 153
 2  2 2
 52  1  25  1 24
Correct Answer Option (b) 5    = 12
 2  2 2
2
Nk  Similarly :
Concept : SQUARE: [N :   ], where k can be any
 2   112  1  121  1 120
number 11     = 60
 2  2 2
62. Select the option that is related to the fifth term in the
same way as the second term is related to the first term Correct Answer Option (a)
and fourth term related to third term. N
:
 N  2
Concept: SQUARE: [ ](The number is
ml fodYi dk p;u djsa tks ikaposa in ls mlh çdkj lacfa èkr gS tSls N2  2  N  22  2
nwljk in igys in ls vkSj pkSFkk in rhljs in ls lacafèkr gSA denominator is square of numerator subtract a number)
27 : 196 :: 19 : 100 :: 17 : ? 65. Select the option that is related to the third term in the
(a) 57 (b) 81 (c) 96 (d) 89 same way as the second term is related to the first term.
Sol. (b) ml fodYi dk p;u djsa tks rhljs in ls mlh çdkj lacafèkr gS tSls
2
 N 1  nwljk in igys in ls lacafèkr gSA
Pattern: - [N :   ]
 2  6/34 : 8/62 ∷ 11/119 : ?
 27  1 
2
 28 
2 (a) 13/165 (b) 17/167
27        14 = 196
2
(c) 15/171 (d) 13/167
 2   2 
Sol. (d)
2 2
 19  1   20   N  2
19        10 = 100
2 N
 2   2  Pattern : - [ : ]
N2  2  N  22  2
Similarly:
2 2 6 6 6 62
 17  1   18    :
17        9 = 81
2
62  2 36  2 34  6  22  2
 2   2 
Correct Answer Option (b) 8 8 8
  
N
2 82  2 64  2 62
Concept : SQUARE: [N : 2   ], where k can be any Similarly :
k
11 11 11 11  2
number(double of square of factor of the number)   :
63. Select the number that is related to the third number in 112  2 121  2 119 11  22  2
the same way as the second number is related to the first 13 13 13
number.   
132  2 169  2 167
ml la[;k dk p;u djsa tks rhljh la[;k ls mlh çdkj lacaf/kr gS
Correct Answer Option (d)
tSls nwljh la[;k igyh la[;k ls lacaf/kr gSA
N Nk
28 : 98 : : 36 : ? Concept: SQUARE: [ 2 : ],
(a) 190 (b) 160 (c) 192 (d) 162 N  1 (N  k)2  1
Sol. (d) 66. Select the option that is related to the third term in the
2 same way as the second term is related to the first term.
N
Pattern : - [N : 2   ] ml fodYi dk p;u djsa tks rhljs in ls mlh çdkj lacafèkr gS tSls
4
nwljk in igys in ls lacafèkr gSA
2
 28  7/50 : 9/82 :: 13/170 : ?
28  2    2(7)2  2 × 49 = 98
 4  (a) 225 (b) 15/226
Similarly : (c) 15/225 (d) 15/224
2 Sol. (b)
 36 
36  2    2(9)2  2 × 81 = 162 N N2
 4  Pattern : - [ 2 : ]
N  1 (N  2)2  1
Correct Answer Option (d)
 N2  k  7 7 7 72 9 9
Concept :SQUARE: [N :   ] where k can be any
  :  2  
 7  2  1
2
7 1
2
49  1 50 9 1 81  1
 2 
number 9
64. Select the option that is related to the third term in the 82
same way as the second term is related to the first term. Similarly :
ml fodYi dk p;u djsa tks rhljs in ls mlh çdkj lacaf/kr gS tSls 13 13 13 13 13  2 15
nwljk in igys in ls lacaf/kr gSA  2   :  2
170 13  2  1
2
170 13  1 169  1 15  1
5 : 12 : : 11 _____
(a) 60 (b) 40 (c) 20 (d) 80 15 15
Sol. (a)  
225  1 226

228
Best App for Govt. Jobs : Neonclasses (Download Now)

Correct Answer Option (b) (4, 9, 16) [ 22, 32, 42 ]


Concept : SQUARE: in a set (x,y,z) Pattern : - [(X + Y)2 = Z]
(81, 100, 121) [ 92, 102, 112 ]
67. Select the set in which the numbers are related in the
Similarly :
same way as are the numbers of the following sets.
ml leqPp; dk p;u djsa ftlesa la[;k,¡ mlh çdkj lacfa èkr gSa tSls (36, 49, 64) [ 62, 72, 82 ]
fuEufyf[kr leqPp;ksa dh la[;k,¡ gSaA Correct Answer Option (c)
Concept : SQUARE: [Triplet X2 + Y2 = Z2]
(NOTE : Operations should be performed on the whole
70. Select the set in which the numbers are related in the
numbers, without breaking down the numbers into its
same way as are the numbers of the following set.
constituent digits. E.g 13 – Operations on 13 such as adding
/deleting /multiplying etc. to 13 can be performed. Breaking
ml leqPp; dk p;u djsa ftlesa la[;k,¡ mlh çdkj lacaf/kr gSa tSls
down 13 into 1 and 3 and then performing mathematical fuEufyf[kr leqPp; dh la[;k,¡ gSaA
operations on 1 and 3 is NOT allowed) (5, 13, 12)
¼uksV % la[;kvksa dks mlds ?kVd vadksa esa foHkkftr fd, fcuk] iw.kZ (a) (6, 10, 8) (b) (11, 15, 9)
la[;kvksa ij lapkyu fd;k tkuk pkfg,A mnkgj.k ds fy, 13&13 (c) (13, 17, 11) (d) (15, 19, 13)
Sol. (a)
ij lapkyu tSls fd tksMus@gVkus@xkk djus vkfn dks 13 esa fd;k
Pattern : - [Triplet X2 + Y2 = Z2]
tk ldrk gSA 13 dks 1 vkSj 3 esa foHkkftr fd;k tk ldrk gSSA vkSj
fQj 1 vkSj 3 ij xf.krh; lafØ;kvksa dks djus dh vuqefr ugha gSa½ 5, 12 13
(2, 3, 25)
(3, 4, 49) 25 +144 =
(a) (4, 5, 81) (b) (4, 7, 111)
(5,13,12)52+122
(c) (4, 6, 110) (d) (4, 5, 91)
Sol. (a) = 25 + 144 = 169 ⇒ 169 = 13
Pattern : - [(X + Y)2 = Z] Similarly :
(6, 10, 8)  62+82  ⇒ 100 = 10
2 , 3 25 Correct Answer Option (a)
Concept : SQUARE: [N : N4 ]
2 + 3= 5 71. Select the option that is related to the fifth number in the
2 2 same way as the second number is related to the first
(2, 3, 25) (2 + 3)  5 = 25
2 2 number and the fourth number is related to the third
(3, 4, 49) (3 + 4)  7 = 49 number.
Similarly : ml fodYi dk p;u djsa tks ikapoha la[;k ls mlh çdkj lacafèkr gS
2 2
(4, 5, 81) (4 + 5)  9 = 81 tSls nwljh la[;k igyh la[;k ls lacafèkr gS vkSj pkSFkh la[;k rhljh
Correct Answer Option (a) la[;k ls lacafèkr gSA
68. Select the option in which the numbers are NOT related in (NOTE: Operations should be performed on the whole
the same way as are the numbers of the following set. numbers, without breaking down the numbers into its
ml fodYi dk p;u djsa ftlesa la[;k,a mlh rjg lacafèkr ugha gSa constituent digits. E.g. 13 – Operations on 13 such as adding
tSls fuEufyf[kr lsV dh la[;k,a gSaA /deleting /multiplying etc. to 13 can be performed. Breaking
(7, 21, 49) down 13 into 1 and 3 and then performing mathematical
(a) (9, 27, 72) (b) (8, 24, 64) operations on 1 and 3 is not allowed)
(c) (5, 15, 25) (d) (6, 18, 36) ¼uksV % la[;kvksa dks mlds ?kVd vadksa esa foHkkftr fd, fcuk] iw.kZ
Sol. (a) la[;kvksa ij lapkyu fd;k tkuk pkfg,A mnkgj.k ds fy, 13&13
Pattern : (A,B,C) [ A, 3×A=B, (A)2=C ] ij lapkyu tSls fd tksMus@gVkus@xq.kk djus vkfn dks 13 esa fd;k
(7,21,49)  7×3=21 , (7)2 = 49 tk ldrk gSA 13 dks 1 vkSj 3 esa foHkkftr fd;k tk ldrk gSSA vkSj
Similarly : fQj 1 vkSj 3 ij xf.krh; lafØ;kvksa dks djus dh vuqefr ugha gSa½
2 : 16 :: 3 : 81 :: 6 : ?
(8, 24, 64) 8×3 = 24, (8)2 = 64
(a) 1296 (b) 1326 (c) 1524 (d) 1666
(5, 15, 25) 5×3 =15, (5)2 = 25 Sol. (a)
(6, 18, 36) 6×3 = 18, (6)2 = 36 Pattern : - [N : N4 ]
Not Similarly :
(9,27,72) 9×3=27, (9)2= 81 ≠ 72  2  24  2×2×2×2 = 16
Correct Answer Option (a) 3  34 3×3×3×3 = 81
Concept : In a set square roots of numbers are in series Similarly :
69. Select the option in which the numbers are related in the
6  64 6×6×6×6 = 1296
same way as are the numbers in the given sets.
Correct Answer Option (a)
ml fodYi dk p;u djsa ftlesa la[;k,a mlh rjg lacfa /kr gSa tSls
Concept : cube and Square: [N3 : N2]
fn, x, lsV esa la[;k,a gSaA
72. Select the option that is related to the fifth number in the
(4, 9, 16) (81, 100, 121)
same way as the second number is related to the first
(a) (25, 49, 81) (b) (16, 36, 25)
number and the fourth number is related to the third
(c) (36, 49, 64) (d) (9, 16, 36)
number.
Sol. (c)

229
Download Free PDFs & e-Books from Neon Classes App

ml fodYi dk p;u djsa tks ikapoha la[;k ls mlh çdkj lacafèkr gS ml fodYi dk p;u djsa ftlesa la[;k,W mlh izdkj lacaf/kr gSa tSls
tSls nwljh la[;k igyh la[;k ls lacafèkr gS vkSj pkSFkh la[;k rhljh fuEufyf[kr lsV dh la[;k,¡ gSa
la[;k ls lacafèkr gSA (7, 52, 346)
NOTE: Operations should be performed on the whole (a) (5, 25, 128) (b) (4, 19, 70)
numbers, without breaking down the numbers into its (c) (8, 67, 515) (d) (6, 39, 217)
constituent digits. E.g. 13 – Operations on 13 such as adding Sol. (c)
/deleting /multiplying etc. to 13 can be performed. Breaking Pattern : - [N : N2+3 : N +3]
3

down 13 into 1 and 3 and then performing mathematical (7, 52, 346)
operations on 1 and 3 is not allowed) 2 3
7 : 7 + 3 : 7 + 3
¼uksV % la[;kvksa dks mlds ?kVd vadksa esa foHkkftr fd, fcuk] iw.kZ
 7 : 49 + 3 : 343 + 3 = 7 : 52 : 346
la[;kvksa ij lapkyu fd;k tkuk pkfg,A mnkgj.k ds fy, 13&13
Similarly :
ij lapkyu tSls fd tksMus@gVkus@xq.kk djus vkfn dks 13 esa fd;k (8, 67, 515)
tk ldrk gSA 13 dks 1 vkSj 3 esa foHkkftr fd;k tk ldrk gSSA vkSj 2 3
8 : 8 + 3 : 8 + 3
fQj 1 vkSj 3 ij xf.krh; lafØ;kvksa dks djus dh vuqefr ugha gSa½
125 : 25 :: 64 : 16 :: 216 : ?
 8 : 64 + 3 : 512 + 3 = 8 : 67 : 515
Correct Answer Option (c)
(a) 36 (b) 39 (c) 22 (d) 24
2 3
Sol. (a) Concept : cube and Square: In a set (X,Y,Z) [X + Y = Z]
Pattern : - [N3 : N2] 76. Select the option in which the numbers are related in the
125 : 25  53 : 52, same way as are the numbers of the following set.
ml fodYi dk p;u djsa ftlesa la[;k,W mlh izdkj lacaf/kr gSa tSls
64 : 16  43 : 42
fuEufyf[kr lsV dh la[;k,¡ gSa
Similarly:
(4, 7, 359)
216 : ?  63 : 62  36 = 216 : 36 (a) (3, 2, 71) (b) (5, 4, 98)
Correct Answer Option (a) (c) (10, 3, 172) (d) (6, 8, 548)
Concept : cube and Square: [X : X3 + X2] Sol. (d)
73. Select the option that is related to the third number in the 2 3
Pattern : - [X + Y = Z]
same way as the second number is related to the first
number. 4, 7 359
ml fodYi dk p;u djsa tks rhljh la[;k ls mlh çdkj lacfa èkr gS
tSls nwljh la[;k igyh la[;k ls lacafèkr gSA 42=16 +73 =343
6 : 252 :: 5 : ? 2 3
(a) 150 (b) 175 (c) 225 (d) 125 (4, 7, 359)  4 +7 
Sol. (a) Similarly :
Pattern : - [X : X3 + X2] 6, 8 548
6  63 + 62  216 + 36 = 252
Similarly : 62=36 +83 =512
5  53 + 52  125 + 25 = 150 2 3
(6, 8, 548)  6 +8 
Correct Answer Option (a)
Correct Answer Option (d)
cube and Square: [N : N3–N2] 2 3
74. Select the option that is related to the third number in the Concept : cube and Square: [k(X + Z ) = y]
same way as the second number is related to the first 77. Select the option in which the numbers are related in the
number and the sixth number is related to the fifth same way as are the numbers of the following set.
number. ml fodYi dk p;u djsa ftlesa la[;k,W mlh izdkj lacaf/kr gSa tSls
ml fodYi dk p;u djsa tks rhljh la[;k ls mlh çdkj lacfa èkr gS fuEufyf[kr lsV dh la[;k,¡ gSa
tSls nwljh la[;k igyh la[;k ls lacafèkr gS vkSj NBh la[;k ikapoha (4, 282, 5)
la[;k ls lacafèkr gSA (a) (5, 25, 1) (b) (3, 405, 6)
3 : 18 :: 9 : ? :: 5 : 100 (c) (2, 84, 4) (d) (7, 152, 3)
(a) 500 (b) 236 (c) 648 (d) 435 Sol. (d)
2 3
Sol. (c) Pattern : - [2(X + Z ) = y]
Pattern : - [N : N3–N2] 282
4, 5
3  33 – 3227 – 9 = 18
5  53 – 52125 – 25 = 100
Similarly: 42=16 +53 =125 (141)
9  93 – 92729 – 81 = 648 (4, 282, 5)
Correct Answer Option (d) 2 3
 2(4 +5 )2)
3
Concept : cube and Square: [N : N2+k
: N +k]  2×141 = 282
75. Select the option in which the numbers are related in the Similarly :
same way as are the numbers of the following set.

230
Best App for Govt. Jobs : Neonclasses (Download Now)

3
7, 3 152 N
Concept : cube and Square: : [N : N2 +   ]
k
80. Select the option that is related to the fifth term in the
72=49 +33 =27 (76) same way as the second term is related to the first term
2 3
(7, 152, 3)  2(7 +3 )227) and fourth term related to third term.
ml fodYi dk p;u djsa tks ikaposa in ls mlh çdkj lacfa èkr gS tSls
 2×76 = 152
nwljk in igys in ls vkSj pkSFkk in rhljs in ls lacafèkr gSA
Correct Answer Option (d)
3
12:208 :: 9:108 ::15:?
Concept : cube and Square: [N : N2 : N ] (a) 350 (b) 360 (c) 325 (d) 365
78. Select the option in which the numbers are related in the Sol. (a)
same way as are the numbers of the following sets. 3
N
ml fodYi dk p;u djsa ftlesa la[;k,¡ mlh çdkj lacfa èkr gSa tSls Pattern : - [N : N2 +   ]
fuEufyf[kr lsVksa dh la[;k,¡ gSaA 3
3
NOTE: Operations should be performed on the whole  12 
12  122 +    144 + 43  144 + 64 = 208
numbers, without breaking down the numbers into its  3
constituent digits. E.g. 13 – Operations on 13 such as 3
 9
adding/deleting/multiplying etc. to 13 can be performed. 9  92 +    81 + 33  81 + 27 = 108
Breaking down 13 into 1 and 3 and then performing 3
mathematical operations on 1 and 3 is not allowed) Similarly :
uksV: la[;kvksa dks mlds ?kVd vadksa esa foHkkftr fd, fcuk] iw.kZ  15 
3

la[;kvksa ij lapkyu fd;k tkuk pkfg,A mnk- 13 & 13 ij lapkyu 15  152 +    225 + 53  225 + 125 = 350
 3
tSls 13 dks tksM+uk @ gVkuk @ xq.kk djuk vkfn fd;k tk ldrk Correct Answer Option (a)
gSA 13 dks 1 vkSj 3 esa rksM+uk vkSj fQj 1 vkSj 3 ij xf.krh; 2
lafØ;k,¡ djus dh vuqefr ugha gS½ N
Concept :cube and Square: [N : N3+   ]
(4, 16, 64) 2
(7, 49, 343) 81. Select the option that is related to the fifth term in the
(a) (5, 25, 115) (b) (8, 64, 412) same way as the second term is related to the first term
(c) (9, 81, 729) (d) (11, 121, 1231) and fourth term related to third term.
Sol. (c) ml fodYi dk p;u djsa tks ikaposa in ls mlh çdkj lacfa èkr gS tSls
Pattern : - [N : N2 :N ]
3 nwljk in igys in ls vkSj pkSFkk in rhljs in ls lacafèkr gSA
12:1764 :: 8:528 :: 16:?
(4, 16, 64)  4 × 16 = 64
(a) 4156 (b) 4266 (c) 4260 (d) 4160
or 4 : 42 : 43 4 : 16 : 64 Sol. (d)
(7, 49, 343)  7 × 49 = 343 N
2

Pattern : - [N : N3+   ]
or 7 : 72 : 73 7 : 49 : 343 2
Similarly : 2
 12 
(9, 81, 729)  9 × 81 = 729 12  123+    1728 + 62  1728 + 36 = 1764
 2
or 9 : 92 : 93 9 : 81 : 729 2
Correct Answer Option (c) 8
8  83+    512 + 42  512 + 16 = 528
Concept :cube and Square: [N : (N+1)2–1 : (N+2)3–2] 2
79. Select the option that is most like the given set. Similarly :
ml fodYi dk p;u djsa tks fn, x, lsV ds leku gSA 2
 16 
(3, 15,123) 16  163+    4096 + 82  4096 + 64 = 4160
 2 
(a) (8, 63, 998) (b) (5, 36, 341)
(c) (2, 26, 125) (d) (4, 24, 214) Correct Answer Option (d)
3 2
Sol. (d) N N
Concept : Cube and Square: [N :   +   ]
Pattern : - [N : (N+1)2–1 : (N+2)3–2] 2 2
(3, 15, 123) 3 : (3 + 1)2–1 : (3 + 2)3–2 82. Select the option that is related to the fifth term in the
 3 : 42–1 : 53–2 same way as the second term is related to the first term
3 : 16 – 1 : 125 – 2 and fourth term related to third term. [
= 3 : 15 : 123 ml fodYi dk p;u djsa tks ikaposa in ls mlh çdkj lacfa èkr gS tSls
Similarly : nwljk in igys in ls vkSj pkSFkk in rhljs in ls lacafèkr gSA
16:576 :: 8:80 :: 22:?
(4, 24, 214) 4 : (4 + 1)2–1 : (4 + 2)3–2
(a) 1455 (b) 1452 (c) 1458 (d) 1456
 4 : 52–1 : 63–2 Sol. (b)
4 : 25 – 1 : 216 – 2 3 2
N N
= 4 : 24 : 215 Pattern : - [N :   +   ]
2 2
Correct Answer Option (d)

231
Download Free PDFs & e-Books from Neon Classes App

 16 
3
 16 
2
 12 
16 : 576    +    83 + 82  512 + 64 = 576 12 : 60  12 ×   1   12 × (6–1)  12 × 5 = 60
 2   2   2 
8 8
3 2 Similarly :
8 : 80    +    43 + 42  64 + 16 = 80
2 2  16 
16 :  16 ×   1   16 × (8–1)  16 × 7 = 112
Similarly :  2 
3 2 Correct Answer Option (b)
 22   22 
22    +    113 + 112  1331 + 121 = 1452 Concept : Multiplication : [N : kN]
 2  2
86. Select the option that is related to the third number in the
Correct Answer Option (b) same way as the second number is related to the first
N number.
Concept : Multiplication : [N : N × ]
2 ml fodYi dk p;u djsa tks rhljh la[;k ls mlh çdkj lacfa èkr gS
83. Select the option that is related to the fifth number in the tSls nwljh la[;k igyh la[;k ls lacafèkr gSA
same way as the second number is related to the first 17 : 102 :: 23 :
number and the fourth number is related to the third (a) 136 (b) 256 (c) 138 (d) 152
number. Sol. (c)
ml fodYi dk p;u djsa tks ikapoha la[;k ls mlh çdkj lacafèkr gS Pattern : - [N : 6N]
tSls nwljh la[;k igyh la[;k ls lacafèkr gS vkSj pkSFkh la[;k rhljh 17 : 102 17 × 6  17 : 102
la[;k ls lacafèkr gSA Similarly :
8 : 32 :: 12 : 72 :: 18 : ? 23 23 × 6  138 = 23 : 138
(a) 234 (b) 208 (c) 162 (d) 180
Correct Answer Option (c)
Sol. (c)
Concept : Multiplication : [N : N(N+k)], where k can be any
N number
Pattern : - [N : N × ]
2 87. Select the option that is related to the third number in the
8 same way as the second number is related to the first
8 8 ×  8 × 4 = 32
2 number.
12 ml fodYi dk p;u djsa tks rhljh la[;k ls mlh çdkj lacfa èkr gS
12 12 ×  12 × 6 = 72 tSls nwljh la[;k igyh la[;k ls lacafèkr gSA
2
20 : 480 :: 25 : ?
Similarly :
(a) 700 (b) 725 (c) 655 (d) 625
18
18 18 ×  18 × 9 = 162 Sol. (b)
2 Pattern : - [N : N(N+4)]
Correct Answer Option (c)
20 20 (20+4)  20 × 24 = 480
Concept :Multiplication : Multiplication[N × k1 : N × k2] Similarly :
84. Select the option that is related to the third number in the
25 25 (25+4)  25 × 29 = 725
same way as the second number is related to the first number.
Correct Answer Option (b)
ml fodYi dk p;u djsa tks rhljh la[;k ls mlh çdkj lacfa èkr gS
Concept :Multiplication : [N : N(N–1)] ,
tSls nwljh la[;k igyh la[;k ls lacafèkr gSA
88. Select the option that is related to the fourth number in
52 : 91 : : 72 : ?
the same way as the first number is related to the second
(a) 98 (b) 126 (c) 109 (d) 138
number and the fifth number is related to the sixth
Sol. (b)
number.
Pattern : - Multiplication[N × 4 : N × 7]
ml fodYi dk p;u djsa tks pkSFkh la[;k ls mlh izdkj lacaf/kr gS
52 : 91 13 × 4 : 13 × 7 tSls igyh la[;k nwljh la[;k ls lacaf/kr gS vkSj ikapoh la[;k NBh
Similarly : la[;k ls lacaf/kr gSA
72 : ? 18 × 4 : 18 × 7 = 126 7: 42 :: ? : 110 :: 9 : 72
Correct Answer Option (b) (a) 11 (b) 8 (c) 6 (d) 12
N  Sol. (a)
Concept :Multiplication : [N: N ×   1 
2  Pattern : - [N : N(N–1)]
85. Select the option that is related to the third number in the 7 : 42 7(7–1) 7 × 6 = 42
same way as the second number is related to the first 9 : 72 9(9–1) 9 × 8 = 72
number. Similarly :
ml fodYi dk p;u djsa tks rhljh la[;k ls mlh çdkj lacfa èkr gS ? : 11 11(11–1) 11 × 10 = 110
tSls nwljh la[;k igyh la[;k ls lacafèkr gSA Correct Answer Option (a)
12 : 60 :: 16 : ? Concept :Multiplication : [N(kN) : N], where k can be any
(a) 210 (b) 112 (c) 121 (d) 201 number
Sol. (b) 89. Select the option that is related to the fourth number in
N  the same way as the first number is related to the second
Pattern : - [N: N ×   1  ]
2  number and the fifth number is related to the sixth
number.

232
Best App for Govt. Jobs : Neonclasses (Download Now)

ml fodYi dk p;u djsa tks pkSFkh la[;k ls mlh çdkj lacafèkr gS (18, 38, 116)  18 : 2×18+ 2 : 6 × 18 + 8
tSls igyh la[;k nwljh la[;k ls lacafèkr gS vkSj ikapoha la[;k NBh 18 : 36+2 : 108 + 8
la[;k ls lacafèkr gSA 18 : 38 : 116
72 : 6 :: ? : 7 :: 128 : 8 ×6+8
×2+2
(a) 95 (b) 101 (c) 98 (d) 90 18, 38, 116
Sol. (c)
Similarly :
Pattern : - [N(2N) : N]
(10, 22, 68)  10 : 2 × 10 + 2 : 6 × 10 + 8
6 6×(2×6) 6 × 12 = 72
10 : 20 +2 : 60 + 8
8 8×(2×8) 8 × 16 = 128 10 : 22 : 68
Similarly :
×6 +8
7 7×(2×7) 7 × 14 = 98 ×2 +2
Correct Answer Option (c) 10, 22, 68
Multiplication [N: kN + p], where k and p can be any Correct Answer Option (a)
numbers N
Concept : Multiplication [N: kN + ], where k can be any
90. Select the option that is related to the third number in the 2
same way as the second number is related to the first number
number and the sixth number is related to the fifth 92. Select the number-pair in which the two numbers are
number. related in the same way as are the two numbers of the
ml fodYi dk p;u djsa tks rhljh la[;k ls mlh çdkj lacafèkr gS following number-pair.
tSls nwljh la[;k igyh la[;k ls lacafèkr gS vkSj NBh la[;k ikapoha ml la[;k ;qXe dk p;u djsa ftlesa nks la[;k,¡ mlh çdkj lacfa èkr gSa
la[;k ls lacafèkr gS tSls uhps fn, x, ;qXe dh nks la[;k,¡A
3 : 17 :: 6: ? :: 9 : 47 12 : 42
(a) 34 (b) 36 (c) 38 (d) 32 (a) 15 : 52(b) 24 : 82
Sol. (d) (c) 8 : 28 (d) 16 : 53
Pattern : - [N: 5N + 2] Sol. (c)
3  5 × 3 + 2  15 + 2 = 17 N
Pattern : - [N: 3N + ]
9  5 × 9 + 2  45 + 2 = 47 2
Similarly : 12
12 : 42  12 : 3 × 12 + 36+ 6 = 42
6  5 × 6 + 2  30 + 2 = 32 2
Correct Answer Option (d) Similarly :
Concept :Multiplication [N : k1N + p1 : k2N + p2], 8
91. Select the set in which the numbers are related in the 88:3×8+  24 + 4 = 28
2
same way as are the numbers of the following sets. [ N: Correct Answer Option (c)
2N+2 : 6N+8] Concept : Multiplication [k(X+Y) = Z], where k can be any
ml leqPp; dk p;u djsa ftlesa la[;k,¡ mlh çdkj lacfa èkr gSa tSls number
fuEufyf[kr leqPp;ksa dh la[;k,¡ gSaA 93. Study the given pattern carefully and select the number
(NOTE : Operations should be performed on the whole from among the given options that can replace the
numbers, without breaking down the numbers into its question mark (?) in it.
constituent digits. E.g. 13 – Operations on 13 such as adding fn, x, iSVuZ dk /;kuiwoZd v/;;u djsa vkSj fn, x, fodYiksa esa ls
/deleting /multiplying etc. to 13 can be performed. Breaking ml la[;k dk p;u djsa tks mlesa iz’uokpd fpUg ¼\½ dks izfrLFkkfir
down 13 into 1 and 3 and then performing mathematical dj ldsA
operations on 1 and 3 is not allowed.) (8, 4, 48), (3, 6, 36), (8, 6, ?)
¼uksV: la[;kvksa dks mlds ?kVd vadksa esa foHkkftr fd, fcuk] iw.kZ (a) 28 (b) 14 (c) 56 (d) 52
la[;kvksa ij lapkyu fd;k tkuk pkfg,A mnkgj.k ds fy, 13 & 13 Sol. (c)
ij lapkyu tSls fd tksM+us @ gVkus @ xq.kk djus vkfn dks 13 esa Pattern : - [4(X+Y) = Z]
fd;k tk ldrk gSA 13 dks 1 vkSj 3 esa rksM+dj vkSj fQj 1 vkSj 3
8, 4 48
ij xf.krh; lafØ;k,¡ djus dh vuqefr ugha gSA½
(13, 28, 86) (18, 38, 116)
(a) (10, 22, 68) (b) (9, 18, 56)
8 + 4 = 12
(c) (11, 24, 76) (d) (8, 18, 54)
Sol. (a) (8, 4, 48)  8 + 4 × 4 48
Pattern : - [N : 2N + 2 : 6N + 8] 3, 6 36
(13, 28, 86) 
13 : 2 × 13 + 2 : 6 × 13 + 8
13 : 26 + 2 : 78 + 8 3 + 6= 9
13 : 28 : 86
(3, 6, 36)  3 + 6 × 4 36
×6 +8
×2 +2 Similarly :
13, 28, 86

233
Download Free PDFs & e-Books from Neon Classes App

56 Sol. (c)
8, 6
X
Pattern : - [X 0 ]
Y
8 + 6 = 14 X
The pattern followed here is :- [X 0 ]
(8, 6, ?)  8 + 6 × 4 = 56 Y
Correct Answer Option (c) 350 : 5 :: ? : 13 :: 990 : 9
Concept :Multiplication [N: N(N+k) + 1], 350
350 – 5, = 70
94. Select the option that is related to the fifth number in the 5
same way as the second number is related to the first 990
number and the fourth number is related to the third 990–9, = 110
9
number. Similarly :
ml fodYi dk p;u djsa tks ikapoha la[;k ls mlh çdkj lacafèkr gS
1950
tSls nwljh la[;k igyh la[;k ls lacafèkr gS vkSj pkSFkh la[;k rhljh = 150
13
la[;k ls lacafèkr gSA
Correct Answer Option (c)
7 : 71::13 : 209::16 : ?
Concept : Multiplication : [A, 2A, 2(2A -6)]
(a) 291 (b) 317 (c) 305 (d) 309
97. Select the set in which the numbers are related in the
Sol. (c)
same way as are the numbers of the given sets.
Pattern : - [N: N(N+3) + 1]
ml leqPp; dk p;u djsa ftlesa la[;k,¡ mlh çdkj lacaf/kr gSa tSls
7  7 (7+3)+1  7 × 10 + 1  70 +1 = 71 fn, x, leqPp;ksa dh la[;k,¡ gSaA
13  13(13+3)+1  13 × 16 + 1  208 + 1 = 209 (NOTE : Operations should be performed on the whole
Similarly : numbers, without breaking down the numbers into its
16  16(16+3)+1  16 × 19 + 1  304 + 1 = 305 constituent digits. E.g. 13 – Operations on 13 such as adding
Correct Answer Option (c) /subtracting /multiplying etc. to 13 can be performed.
Concept : Multiplication : [k(X × Z) = Y] Breaking down 13 into 1 and 3 and then performing
95. Select the option in which the numbers are related in the mathematical operations on 1 and 3 is not allowed.)
same way as are the numbers of the following set. ¼uksV: la[;kvksa dks muds ?kVd vadksa esa foHkkftr fd, fcuk] iw.kZ
ml fodYi dk p;u djsa ftlesa la[;k,W mlh izdkj lacaf/kr gSa tSls la[;kvksa ij lapkyu fd;k tkuk pkfg,A mnkgj.k ds fy, 13 & 13
fuEufyf[kr lsV dh la[;k,¡ gSa ij lapkyu tSls 13 dks tksM+uk / ?kVkuk / xq.kk djuk vkfn fd;k
(15, 315, 7), tk ldrk gSA 13 dks 1 vkSj 3 esa rksM+uk vkSj fQj 1 vkSj 3 ij
(a) (14, 197, 4) (b) (11, 188, 6) xf.krh; lafØ;k,¡ djus dh vuqefr ugha gSA½
(c) (18, 486, 9) (d) (17, 401, 12) (18, 36, 60)
Sol. (c) (14, 28, 44)
Pattern : - [3(X × Z) = Y] (a) (21, 42, 164) (b) (17, 32, 60)
15, 7 315 (c) (16, 28, 64) (d) (22, 44, 76)
Sol. (d)
Pattern: - [A, 2A, 2(2A -6)]
15 × 7 =105 (18, 36, 60)  18, 2 × 18, 2 (2×18–6)

(15, 315, 7)  3(15×7) 18, 36, 2(36–6)


= 3 (105)3×105 = 315  2 (30) = 60
Similarly : (14, 28, 44)  14, 2 × 14, 2 (2×14–6)

18, 9 486 14, 28, 2(28–6)


 2 (22) = 44
Similarly :
18 × 9 =162 (22, 44, 76)  22, 2 × 22, 2 (2×22–6)
(18, 486, 9)  3(18 × 9) = 3(162) = 486 22, 44, 2(44–6)
Correct Answer Option (c)  2 (38) = 76
X Correct Answer Option (d)
Concept : Multiplication : [X  0 ]
Y N  k   2
Concept :Multiplication : [N: ] , where k can be
96. Select the option that is related to the fourth number in 5
the same way as the first number is related to the second any number
number and the fifth number is related to the sixth 98. Select the option that is related to the third number in the
number. same way as the second number is related to the first
ml fodYi dk p;u djsa tks pkSFkh la[;k ls mlh çdkj lacafèkr gS number and the sixth number is related to the fifth
tSls igyh la[;k nwljh la[;k ls lacafèkr gS vkSj ikapoha la[;k NBh number.
la[;k ls lacafèkr gSA ml fodYi dk p;u djsa tks rhljh la[;k ls mlh çdkj lacfa èkr gS
350 : 5 :: ? : 13 :: 990 : 9 tSls nwljh la[;k igyh la[;k ls lacafèkr gS vkSj NBh la[;k ikapoha
(a) 1052 (b) 1620 (c) 1950 (d) 1258 la[;k ls lacafèkr gSA

234
Best App for Govt. Jobs : Neonclasses (Download Now)

6 : 4 :: 11 : ? :: 16 : 8 la[;kvksa ds fuEufyf[kr lsV ds leku la[;kvksa ds lewg dk p;u


(a) 6 (b) 5 (c) 8 (d) 7 djsAa
Sol. (a) (20, 42, 72)
N  4  2 (a) (10, 22, 52) (b) (16, 40, 70)
Pattern : - [N: ] (c) (2, 12, 30) (d) (18, 38, 68)
5
Sol. (c)
Pattern : - [Factor of multiplication]
6  4   2 10  2 20
6   =4 (20, 42, 72) 
5 5 5
 20  4 × 5
16  4  2 20  2 40
16    =8 45 6 × 7
5 5 5
Similarly : 72  8 × 9
Similarly:
11  4   2 15  2 30
11    =6 (2, 12, 30) 
5 5 5
Correct Answer Option (a)  21×2
Concept : Multiplication: same multiplication pattern in all 12 3 × 4
three numbers 30  5 × 6
99. Select the option in which the numbers are related in the Correct Answer Option (c)
same way as are the numbers in the given set. Concept :Addition : [N : N + k], where k can be any number
ml fodYi dk p;u djsa ftlesa la[;k,a mlh rjg lacfa /kr gSa tSls .
fn, x, lsV esa la[;k,a gSaA 102. Select the option that is related to the third
(11, 23, 47) number in the same way as the second number is related
(a) (23, 46, 93) (b) (17, 35, 69) to the first number.
(c) (19, 39, 79) (d) (13, 27, 54) ml fodYi dk p;u djsa tks rhljh la[;k ls mlh çdkj lacfa èkr gS
Sol. (c) tSls nwljh la[;k igyh la[;k ls lacafèkr gSA
Pattern : - [X : 2X+1 : 4X+3] 68 : 321 :: 525 : ?
×4+3 (a) 852 (b) 681 (c) 778 (d) 792
×2+1 Sol. (c)
(11, 23, 47)  11, 23, 47
Pattern : - [N : N + 253]
Similarly : 68 : 68 + 253 321
×4+3 Similarly :
×2+1 525 : 525 + 253 778
(19, 39, 79)  19, 39, 79
Correct Answer Option (c)
Correct Answer Option (c) Concept : Addition : sum of Number is constant
Concept : [ X: (X+X/2) : 2Y] 103. Select the set in which the numbers are related
100. Select the set in which the numbers are related in the same way as are the numbers of the following set.
in the same way as are the numbers of the following set. ml leqPp; dk p;u djsa ftlesa la[;k,¡ mlh çdkj lacfa èkr gSa tSls
ml leqPp; dk p;u djsa ftlesa la[;k,¡ mlh çdkj lacaf/kr gSa tSls fuEufyf[kr leqPp; dh la[;k,¡ gSaA
fuEufyf[kr leqPp; dh la[;k,¡ gSaA (8, 15, 9)
(8, 12, 24) (a) (5, 17, 11) (b) (4, 16, 12)
(a) (12, 20, 40) (b) (9, 18, 27) (c) (7, 13, 14) (d) (9, 12, 15)
(c) (6, 9, 18) (d) (6, 10, 18) Sol. (b)
Sol. (c) Pattern : - [X+Y+Z = 32]

 X (8, 15, 9)  8 + 15 + 9 32


Pattern : - [ X :  X   : 3X ] Similarly :
 2
(4, 16, 12)  4 + 16 + 12 = 32
8
(8, 12, 24)  8 : 8 + :3×8 Correct Answer Option (b)
2 Concept :Addition : [N4 +k : N]
 8 : 8 + 4 : 24 = 8 : 12 : 24 104. Select the option that is related to the fourth
Similarly : number in the same way as the first number is related to
6 the second number and the fifth number is related to the
(6, 9, 18)  6 : 6 + :3×6 sixth number.
2
ml fodYi dk p;u djsa tks pkSFkh la[;k ls mlh çdkj lacafèkr gS
 6 : 6 + 3 : 18
tSls igyh la[;k nwljh la[;k ls lacafèkr gS vkSj ikapoha la[;k NBh
 la[;k ls lacafèkr gSA
Correct Answer Option (c) 83 : 3 :: ? : 5 :: 258 : 4
Concept : [N(N+1), (N+2) (N+3), (N+4) (N+5)] (a) 627 (b) 527 (c) 123 (d) 222
101. Select the set of numbers that is similar to the Sol. (a)
following set of numbers. Pattern : - [N4 +2 : N]

235
Download Free PDFs & e-Books from Neon Classes App

3  34 + 2  (3×3×3×3)+2  81 + 2 = 83 down 13 into 1 and 3 and then performing mathematical


operations on 1 and 3 is not allowed)
4  + 2  (4×4×4×4)+2256 + 2 = 258
44
¼uksV % la[;kvksa dks mlds ?kVd vadksa esa foHkkftr fd, fcuk] iw.kZ
Similarly :
la[;kvksa ij lapkyu fd;k tkuk pkfg,A mnkgj.k ds fy, 13&13
5  54 + 2  (5×5×5×5)+2  625 + 2 = 627
ij lapkyu tSls fd tksMus@gVkus@xq.kk djus vkfn dks 13 esa fd;k
Correct Answer Option (a)
tk ldrk gSA 13 dks 1 vkSj 3 esa foHkkftr fd;k tk ldrk gSSA vkSj
Concept : Addition : [X+Y = Z]
fQj 1 vkSj 3 ij xf.krh; lafØ;kvksa dks djus dh vuqefr ugha gSa½
105. Choose the set of numbers that is similar to the
7854 : 5632 :: 8424 : 6202 :: 4444 : ?
following set.
la[;kvksa ds ml leqPp; dks pqfu, tks fuEufyf[kr leqPp; ds leku (a) 1242 (b) 1424 (c) 2222 (d) 2424
Sol. (c)
gksA
Pattern : - Subtraction [N: N – 2222]
(4, 12, 16)
(a) (81, 36, 9) (b) (64, 36, 100 ) 7854  7854 – 2222 = 5632
(c) (16, 20, 25) (d) (36, 25, 49) 8424  8424 – 2222 = 6202
Sol. (b) Similarly :
Pattern : - [X+Y = Z] 4444  4444 – 2222 = 2222
4, 12 16 Correct Answer Option (c)
Concept : Subtraction: The difference between adjacent
letters is constant
4 + 12 = 16 108. Select the option in which the numbers are NOT
related in the same way as are the numbers in the given
(4, 12, 16)  4 + 12 16 set.
Similarly : ml fodYi dk p;u djsa ftlesa la[;k,a mlh rjg lacfa èkr ugha gSa
64, 36 100 tSls fn, x, lsV esa la[;k,a gSa
(143, 110, 88)
(a) (69, 36, 14) (b) (61, 28, 6)
64 + 36 = 100 (c) (114, 82, 60) (d) (123, 90, 68)
Sol. (c)
(64, 36, 100)  64 + 36 = 100
Pattern : - [X : X – 33, Y – 22]
Correct Answer Option (b)
Concept : Addition : [X : X+k1 : X+k2] (143, 110, 88)  143 : 143 – 33 = 110 , 110 – 22 = 88
106. Select the option in which the numbers shares (a) (69, 36, 14)  69 : 69 – 33 = 36, 36 – 22 = 14
the same relationship as that shared by the given pair of (b) (61, 28, 6)  61 : 61 – 33 = 28, 28 – 22 = 6
numbers.
(c) (114, 82, 60)  114 : 114 – 33 = 81, 81 – 22 = 59  60
ml fodYi dk p;u djsa ftlesa la[;kvksa ds chp ogh lacaèk gSa tks nh
(d) (123, 90, 68)  123 : 123 – 33 = 90, 90 – 22 = 68
xbZ la[;kvksa ds ;qXe ds chp gSA
Correct Answer Option (c)
(73, 78, 93)
Concept : Subtraction: In a set [X-(Y+Z)=k]
(a) (67,72 ,82) (b) (29, 34, 39)
109. Select the set in which the numbers are related
(c) (23, 33, 38) (d) (145, 150, 165)
in the same way as are the numbers of the following sets.
Sol. (d)
Pattern : - [X : X+5 : X+20]
ml leqPp; dk p;u djsa ftlesa la[;k,¡ mlh çdkj lacfa èkr gSa tSls
+20
fuEufyf[kr leqPp;ksa dh la[;k,¡ gSaA
+5 (8, 4, 1)
(73, 78, 93)  73, 78, 93 (12, 4, 5)
Similarly : (a) (17, 8, 6) (b) (33, 10, 21)
(c) (44, 20, 20) (d) (20, 2, 12)
+5 +20 Sol. (a)
(145, 150, 165)  140, 150, 165 Pattern : - [Y + Z + 3 = X]
Correct Answer Option (d) 4, 1 8
Concept :Subtraction : [N: N – k],
107. Select the option that is related to the fifth
number in the same way as the second number is related 4 + 1= 5
to the first number and the fourth number is related to the
third number. (8, 4, 1)  4 + 1 + 3 5
ml fodYi dk p;u djsa tks ikapoha la[;k ls mlh çdkj lacafèkr gS 4, 5 12
tSls nwljh la[;k igyh la[;k ls lacafèkr gS vkSj pkSFkh la[;k rhljh
la[;k ls lacafèkr gSA
(NOTE: Operations should be performed on the whole 4 + 5= 9
(12, 4, 5)  4 + 5 + 3 12
numbers, without breaking down the numbers into its
Similarly :
constituent digits. E.g. 13 – Operations on 13 such as adding
/deleting /multiplying etc. to 13 can be performed. Breaking

236
Best App for Govt. Jobs : Neonclasses (Download Now)

17 the second number and the fifth number is related to the


8, 6
sixth number.
ml fodYi dk p;u djsa tks pkSFkh la[;k ls mlh çdkj lacafèkr gS
8 + 6 = 14 tSls igyh la[;k nwljh la[;k ls lacafèkr gS vkSj ikapoha la[;k NBh
la[;k ls lacafèkr gSA
(17, 8, 6)  8 + 6 + 3 = 17 396 : 24 : : ? : 28 : : 672 : 30
Correct Answer Option (a) (a) 588 (b) 504 (c) 536 (d) 572
Concept : Multiplication & Dividation : [(X*Y)/3=Z] Sol. (d)
110. Select the option in which the numbers are Pattern : - [N (N–8) + 12 : N]
related in the same way as are the numbers of the
24  24(24–8) + 12 
following set.
ml fodYi dk p;u djsa ftlesa la[;k,W mlh izdkj lacaf/kr gSa tSls  24 (16) + 12  384+12 = 396
fuEufyf[kr lsV dh la[;k,¡ gSa 30  30(30–8) + 12 
(18, 24, 144)  30 (22) + 12  660+12 = 672
(a) (26, 15, 130) (b) (16, 12, 109) Similarly :
(c) (18, 20, 137) (d) (22, 18, 246) 28  28(28–8) + 12 
Sol. (a)
 28 (20) + 12  560+12 = 572
XY
Pattern : - [ z ] Correct Answer Option (d)
3
N
(18, 24, 144) Concept : Multiplication & Dividation : [N : kN + (
)]
3
18, 24 144 113. Select the option that is related to the third
number in the same way as the second number is related
to the first number and the sixth number is related to the
18 × 24 = 432 fifth number.
Similarly : ml fodYi dk p;u djsa tks rhljh la[;k ls mlh çdkj lacafèkr gS
(26, 15, 130) tSls nwljh la[;k igyh la[;k ls lacafèkr gS vkSj NBh la[;k ikapoha
26, 15 130 la[;k ls lacafèkr gS
21 : 112 :: 36 : ? :: 51 : 272
(a) 192 (b) 252 (c) 72 (d) 198
26 × 15 = 390 Sol. (a)
N
Correct Answer Option (a) Pattern : - [N : 5N + ( )]
3
3X
Concept : Multiplication & Dividation : X :  21 
2 21  5 × 21 +    105 + 7 = 112
111. Select the option that is related to the third  3 
number in the same way as the second number is related  51 
to the first number and the sixth number is related to the 51  5 × 51 +    255 + 17 = 272
 3
fifth number.
Similarly :
ml fodYi dk p;u djsa tks rhljh la[;k ls mlh çdkj lacfa èkr gS
tSls nwljh la[;k igyh la[;k ls lacafèkr gS vkSj NBh la[;k ikapoha  36 
36  5 × 36 +    180 + 12 = 192
la[;k ls lacafèkr gSA  3 
72 : 108 :: 84 : ? :: 102 : 153 Correct Answer Option (a)
(a) 135 (b) 126 (c) 144 (d) 117 Concept : Multiplication & Dividation : [X, Y, (X-1) (Y–1)],
Sol. (b) 126 114. Select the option in which the numbers are
3X related in the same way as are the numbers of the
Pattern : - [ X : ] following set.
2
ml fodYi dk p;u djsa ftlesa la[;k,W mlh izdkj lacaf/kr gSa tSls
3  72 216
72   = 108 fuEufyf[kr lsV dh la[;k,¡ gSa
2 2
(12, 14, 143)
3  102 306
102   = 153 (a) (16, 10, 235) (b) (17, 12, 134)
2 2 (c) (15, 19, 252) (d) (12, 18, 246)
Similarly : Sol. (c)
3  84 252 Pattern : - [X, Y, (X-1) (Y–1)]
84   = 126
2 2 (12 : 14 : 143)
Correct Answer Option (b) 12, 14 143
Concept :Multiplication & Dividation : N (N–k) + p : N],
–1 –1
where k can be any number .
112. Select the option that is related to the fourth 14 × 13 = 143
number in the same way as the first number is related to
Similarly :

237
Download Free PDFs & e-Books from Neon Classes App

(15, 19, 252) (a) 13 : 98(b) 12 : 85


252 (c) 3 : 11 (d) 16 : 145
15, 19
Sol. (a)
–1 –1
 X 1  X 1
Pattern : - [ X:X   ]
14 × 18 = 252  2   2 
Correct Answer Option (c)  7 1  7 1
Concept : Multiplication & Dividation : [N3 : N(2N) : 3N], 7:327   
 2   2 
115. Select the option in which the number are
related in the same way are as the the number in the given 8 8
 7   
set.  2  2
ml fodYi dk p;u djsa ftlesa la[;k,a mlh çdkj lacafèkr gSa tSls  7 × 4 + 4  28 + 4 = 32
fn, x, lsV esa la[;k,a gSaA Similarly :
[343, 98, 21]  13  1   13  1 
(a) [1728, 288, 24] (b) [512, 126, 27] 13 : 98  13   
 2   2 
(c) [217, 72, 18] (d) [1331, 242, 33]
Sol. (d)  14   14 
 13     
Pattern : - [N3 : N(2N) : 3N]  2  2
The pattern followed here is :- [N3 : N(2N) : 3N]
 13× 7  91 + 4 = 98
(343, 98, 21)
Correct Answer Option (a)
21
= 7, 7 (2×7)  7× 14 = 98, 73 = 343 Concept : Multiplication & square : In a set Pattern : - [X : Y
3 : X2Y],
Similarly : 118. Select the option in which the numbers are
(1331, 242, 33) related in the same way as are the numbers of the
33 following set.
= 11, 11 (2×11)  11× 22 = 242, 113 = 1331
3 ml fodYi dk p;u djsa ftlesa la[;k,W mlh izdkj lacaf/kr gSa tSls
Correct Answer Option (d) fuEufyf[kr lsV dh la[;k,¡ gSa
Y
2 (5, 11, 275)
Concept :Multiplication & Dividation : [X, Y X2 +   ], (a) (4, 16, 256) (b) (9, 15, 270)
2
(c) (8, 14, 490) (d) (6, 18, 180)
116. Select the option in which the number sare
Sol. (a)
related in the same way as are the numbers in the given
Pattern : - [X : Y : X2Y]
set.
(5 : 11 : 275)
ml fodYi dk p;u djs] ftlesa nh xbZ la[;kvksa ds chp ogh laca/k
gS] tks uhps fn, x, leqPp; dh la[;kvksa ds chp gSA 5, 11 275
(14, 18, 277)
(a) (20,22, 521) (b) (12, 32, 320)
(c) (18, 16, 485) (d) (24, 20, 576) 52 = 25 ×11 = 275
Sol. (a) Similarly :
Y
2 (4, 16, 256)
Pattern : - [X, Y X2 +   ]
2 4, 16 256
2
 18 
(14, 18, 277)  14, 18, 142 +  
 2  42 = 16 ×16 = 256
 196 + 92  196 + 81 = 277 Correct Answer Option (a)
Similarly : Concept : Multiplication & Subtraction: [N : 2N –
2
 22  4]
(20, 22, 521)  20, 22, 202 +  
 2 119. Select the option that is related to the fifth
number in the same way as the second number is related
 400 + 112  400 + 121 = 521 to the first number and the fourth number is related to the
Correct Answer Option (a) third number.
Concept :Multiplication & Dividation : [ ml fodYi dk p;u djsa tks ikapoha la[;k ls mlh çdkj lacafèkr gS
 X 1  X 1 tSls nwljh la[;k igyh la[;k ls lacafèkr gS vkSj pkSFkh la[;k rhljh
X:X   ]
 2   2  la[;k ls lacafèkr gSA
117. Select the number-pair in which the two (NOTE: Operations should be performed on the whole
numbers are related in the same way as are the two numbers, without breaking down the numbers into its
numbers of the following number-pair. constituent digits. E.g. 13 – Operations on 13 such as adding
ml la[;k&;qXe dk p;u djsa ftlesa nks la[;k,¡ mlh çdkj lacaf/kr /deleting /multiplying etc. to 13 can be performed. Breaking
gSa tSls fuEufyf[kr la[;k&;qXe dh nks la[;k,¡ gSaA down 13 into 1 and 3 and then performing mathematical
7 : 32 operations on 1 and 3 is not allowed)

238
Best App for Govt. Jobs : Neonclasses (Download Now)

¼uksV % la[;kvksa dks mlds ?kVd vadksa esa foHkkftr fd, fcuk] iw.kZ X
Concept : Dividation : Dividation, : N]
la[;kvksa ij lapkyu fd;k tkuk pkfg,A mnkgj.k ds fy, 13&13 7
ij lapkyu tSls fd tksMus@gVkus@xq.kk djus vkfn dks 13 esa fd;k 122. Select the option that is related to the third
tk ldrk gSA 13 dks 1 vkSj 3 esa foHkkftr fd;k tk ldrk gSSA vkSj number in the same way as the second number is related
fQj 1 vkSj 3 ij xf.krh; lafØ;kvksa dks djus dh vuqefr ugha gSa½ to the first number.
12 : 20 :: 8 : 12 :: 34 : ? ml fodYi dk p;u djsa tks rhljh la[;k ls mlh çdkj lacfa èkr gS
(a) 56 (b) 64 (c) 52 (d) 48 tSls nwljh la[;k igyh la[;k ls lacafèkr gSA
Sol. (b) 77 : 11 :: 259 : ?
Pattern : - [N : 2N – 4] (a) 31 (b) 43 (c) 29 (d) 37
12  2 × 12 – 4  24 – 4 = 20 Sol. (d)
8  2 × 8 – 4  16 – 4 = 12 Pattern : - [Dividation,
X
: N]
Similarly:- 7
34  2 × 34 – 4  68 – 4 = 64 77
 11
Correct Answer Option (b) 7
Concept : Multiplication & Subtraction: [2C–A = B] Similarly:-
120. Select the option in which the numbers are 259
related in the same way as are the numbers in the given 259   37
7
set. [2C-A=B] Correct Answer Option (d)
ml fodYi dk p;u djs] ftlesa nh xbZ la[;kvksa ds chp ogh laca/k
N
gS] tks uhps fn, x, leqPp; dh la[;kvksa ds chp gSA Concept :Dividation : [ N :]
2
44, 70, 57
123. Select the number that is related to the third
(a) 34, 68, 51 (b) 46, 80, 70
number in the same way as the second number is related
(c) 28, 56, 112 (d) 32, 50, 46
to the first number.
Sol. (a)
ml la[;k dk p;u djsa tks rhljh la[;k ls mlh çdkj lacaf/kr gS
Pattern : - [2C–A = B]
tSls nwljh la[;k igyh la[;k ls lacaf/kr gSA
(44,70,57)  2 × 57 – 44 = 114 – 44 = 70
Select the number that is related to the third number in
Similarly:- the same way as the second number is related to the first
(34, 68, 51) 2 × 51 – 34 = 102 – 34 = 68 number.
Correct Answer Option (a) ml la[;k dk p;u djsa tks rhljh la[;k ls mlh çdkj lacaf/kr gS
Concept : Multiplication & Subtraction: [2 (X-Y) =Z] tSls nwljh la[;k igyh la[;k ls lacaf/kr gSA
121. Select the set in which the numbers are related 1762 : 881 :: 2578 : ?
in the same way as are the numbers of the following sets. (a) 1280 (b) 2150 (c) 1758 (d) 1289
ml leqPp; dk p;u djsa ftlesa la[;k,¡ mlh çdkj lacfa èkr gSa tSls Sol. (d)
fuEufyf[kr leqPp;ksa dh la[;k,¡ gSaA N
(13, 12, 2), (25, 22, 6) Pattern : - [ N : ]
2
(a) (26, 22, 10) (b) (24, 12, 19)
1762
(c) (6, 3, 6) (d) (15, 5, 15) 1762   881
Sol. (c) 2
Pattern : - [2 (X-Y) =Z] Similarly:-
(13, 12, 2) 2578
2578   1289
13, 12 2 2
Correct Answer Option (d)
Concept : Subtraction & Addition: [X : (2X–2) : (4X–2)]
13 – 12 = 1 124. Select the set in which the numbers are related
in the same way as are the numbers of the following set
(25, 22, 6)
ml leqPp; dk p;u djsa ftlesa la[;k,¡ mlh çdkj lacaf/kr gSa tSls
25, 22 6 fuEufyf[kr leqPp; dh la[;k,¡ gSaA
(10, 18, 38)
(a) (4, 12, 22) (b) (14, 12, 8)
25 – 22 = 3 (c) (12, 22, 46) (d) (18, 6, 14
Similarly : Sol. (c)
(6, 3, 6) Pattern : - [X : (2X–2) : (4X–2)]
6, 3 6 (10, 18, 38)  10 : (2×10–2) : (4×10–2)
10 : 20 – 2 : 40 – 2
10 : 18 : 38
6 – 3 = 3 Similarly :-
Correct Answer Option (c) (12, 22, 46) 12 : (2×12–2) : (4×12–2)
12 : 24 – 2 : 48 – 2
12 : 22 : 46

239
Download Free PDFs & e-Books from Neon Classes App

Correct Answer Option (c) (c) (276, 260, 184) (d) (174, 212, 166)
Concept : Subtraction & Addition: [A+B–10 = C] Sol. (c)
125. Select the set in which the numbers are related Pattern : - [(A+C) –200 = B]
in the same way as are the numbers of the given sets. (241, 230, 189)
ml lsV dk p;u djsa ftlesa la[;k,a mlh rjg lacaf/kr gSa tSls fn, 241, 189 230
x, lsV dh la[;k,a gSaA
(NOTE : Operations should be performed on the whole
numbers, without breaking down the numbers into its 241 +189 = 430
constituent digits. E.g. 13 – Operations on 13 such as adding
(218, 250, 232)
/subtracting /multiplying etc. to 13 can be performed.
Breaking down 13 into 1 and 3 and then performing 218, 232 250
mathematical operations on 1 and 3 is not allowed.)
¼uksV: la[;kvksa dks muds ?kVd vadksa esa foHkkftr fd, fcuk] iw.kZ
la[;kvksa ij lapkyu fd;k tkuk pkfg,A mnkgj.k ds fy, 13 & 13 218 +232 = 450
ij lapkyu tSls 13 dks tksM+uk / ?kVkuk / xq.kk djuk vkfn fd;k Similarly :
tk ldrk gSA 13 dks 1 vkSj 3 esa rksM+uk vkSj fQj 1 vkSj 3 ij (276, 260, 184)
xf.krh; lafØ;k,¡ djus dh vuqefr ugha gSA½ 276, 184 260
(16, 22, 28)
(14, 19, 23)
(a) (12, 18, 25) (b) (13, 21, 27) 276 +184 = 460
(c) (15, 26, 31) (d) (17, 24, 41)
Correct Answer Option (c)
Sol. (c)
Concept : Digit Operation : [Digit + k]where k, can any
Pattern : - [A+B–10 = C]
random number
(16, 22, 28)
127. Select the option that is related to the third
16, 22 28 number in the same way as the second number is related
to the first number.
ml fodYi dk p;u dhft, ftld rhljh la[;k ls Bhd ogh laca/k
16 + 22 = 38 gS tks nwljh la[;k dk igyh la[;k ls gSA
(14, 19, 23) 3824 : 4935 : : 5716 : ?
14, 19 23 (a) 6782 (b) 6827 (c) 6087 (d) 6287
Sol. (b)
Pattern : - [Digit + 1]
14 + 19 = 33 3824 3 8 2 4
+1 +1 +1 +1
Similarly :
(15, 26, 31) 4 9 3 5
Similarty : -
15, 26 31
5716 5 7 1 6
+1 +1 +1 +1
15 + 26 = 41 6 8 2 7

Correct Answer Option (c) Correct Answer Option (b)


Concept : Subtraction & Addition : [(A+C) –200 = B] Concept : Digit Operation : [Digit (ab) : a × b]
126. Select the set in which the numbers are related 128. Select the option that is related to the third
in the same way as are the numbers of the given sets. number in the same way as the second number is related
ml leqPp; dk p;u djsa ftlesa la[;k,¡ mlh çdkj lacaf/kr gSa tSls to the first number.
fn, x, leqPp;ksa dh la[;k,¡ gSaA ml fodYi dk p;u djsa tks rhljh la[;k ls mlh çdkj lacfa èkr gS
(NOTE : Operations should be performed on the whole tSls nwljh la[;k igyh la[;k ls lacafèkr gSA
numbers, without breaking down the numbers into its 39 : 27 :: 43 : ?
constituent digits. E.g. 13 – Operations on 13 such as adding (a) 38 (b) 20 (c) 6 (d) 12
/subtracting /multiplying etc. to 13 can be performed. Sol. (d)
Breaking down 13 into 1 and 3 and then performing Pattern : - [Digit (ab) : a × b]
mathematical operations on 1 and 3 is not allowed.)
¼uksV& la[;kvksa dks muds ?kVd vadksa esa foHkkftr fd, fcuk] iw.kZ 39 : 27 :: 43 : ?
la[;kvksa ij lapkyu fd;k tkuk pkfg,A mnkgj.k ds fy, 13 & 13 39  3×9 = 27
ij lapkyu tSls 13 dks tksM+uk @ ?kVkuk @ xq.kk djuk vkfn fd;k Similarly :
tk ldrk gSA 13 dks 1 vkSj 3 esa rksM+uk vkSj fQj 1 vkSj 3 ij 43  4 × 3 = 12
xf.krh; lafØ;k,¡ djus dh vuqefr ugha gSA½ Correct Answer Option (d)
(241, 230, 189), (218, 250, 232) Concept : Digit Operation : Sum Digit × k = N (where k, can
(a) (158, 194, 182) (b) (197, 233, 190) any random number

240
Best App for Govt. Jobs : Neonclasses (Download Now)

129. Select the option that is related to the fifth 123  12 + 22 + 33  1 + 4 + 27 = 32


number in the same way as the second number is related Similarly:-
to the first number and the fourth number is related to the
541  52 + 42 + 13  25 + 16 + 1 = 42
third number.
Correct Answer Option (d)
ml fodYi dk p;u djsa tks ikapoha la[;k ls mlh çdkj lacafèkr gS
Concept : Digit Operation : [Digit AB  A3, B2]
tSls nwljh la[;k igyh la[;k ls lacafèkr gS vkSj pkSFkh la[;k rhljh
133. Select the option that is related to the third term
la[;k ls lacafèkr gSA
in the same way as the second term is related to the first
232 : 21 :: 343 : 30 :: 157 : ?
term.
(a) 36 (b) 39 (c) 42 (d) 33
ml fodYi dk p;u djsa tks rhljs in ls mlh çdkj lacaf/kr gS tSls
Sol. (b)
Pattern : - [Sum Digit × 3 = N]
nwljk in igys in ls lacaf/kr gSA
32 : 274 : : 44 : ______
232 : 21  2 + 3 + 2 × 3  7 × 3 = 21
(a) 6146 (b) 4661 (c) 4616 (d) 6416
343 : 30  3 + 4 + 3 × 3  10 × 3 = 30 Sol. (d)
Similarly:- Pattern : - [Digit AB  A3, B2]
157  1 + 5 + 7 × 3  13 × 3 = 39
32 : 274  32  33 , 22  27, 4  274
Correct Answer Option (b)
Similarly:-
Concept : Digit Operation : Digit AB  A2 + B2]
44  43 , 42  64, 16 = 6416
130. Select the number that is related to the third
Correct Answer Option (d)
number in the same way as the second number is related
B
to the first number. Concept : Digit Operation : [AB : (A–1),, AB–Digit]
ml la[;k dk p;u djsa tks rhljh la[;k ls mlh çdkj lacaf/kr gS 2
tSls nwljh la[;k igyh la[;k ls lacaf/kr gSA 134. Select the option in which the numbers share the
same relationship as that shared by the given pair of
45 : 41 : : 27 : ?
numbers.
(a) 53 (b) 57 (c) 49 (d) 78
Sol. (a)
ml fodYi dk p;u djsa] ftldh la[;kvksa ds chp ogh lac/a k gS] tks
fn, x, la[;k ;qXe dh la[;kvksa ds chp gS\
Pattern : - [Digit AB  A2 + B2]
84 : 72
45  42 – 52  16 + 25 = 41 (a) 68 : 54(b) 64 : 56
Similarly:- (c) 77 : 64 (d) 76 : 62
27  22 – 72  4 + 49 = 53 Sol. (a)
Correct Answer Option (a) B
Concept : Digit Operation : Two digit – Last Two digit : N] Pattern : - [AB : (A–1), , AB–Digit]
2
131. Select the option that is related to the fourth
number in the same way as the first number is related to 84 : 72  (8–1)
the second number and the fifth number is related to the
sixth number.
ml fodYi dk p;u djsa tks pkSFkh la[;k ls mlh çdkj lacafèkr gS 7 2 = 72
tSls igyh la[;k nwljh la[;k ls lacafèkr gS vkSj ikapoha la[;k NBh Similarly:-
la[;k ls lacafèkr gSA 68 : 54  (6–1)
4523 : 22 :: ? : 14 :: 9178 : 13
(a) 8228 (b) 7448 (c) 9152 (d) 5440
Sol. (d) 5 4 = 54
Pattern : - [1st Two digit – Last Two digit : N]
Correct Answer Option (a)
4523  45 – 23 = 22 Concept : Digit Operation : [AB : AB + (A+B), AB – Digit]
9178  91 – 78 = 13 135. Select the option in which the numbers share the
Similarly:- same relationship as that shared by the given pair of
5440  54 – 40 = 14, (5440 : 14) numbers.
Correct Answer Option (d) ml fodYi dk p;u djsa] ftldh la[;kvksa ds chp ogh lac/a k gS] tks
Concept : Digit Operation : ABC : A2+ B2 +C3, ABC- fn, x, ;qXe dh la[;kvksa ds e/; gSA
Digit] 48 : 60
132. Select the option that is related to the third term (a) 86 : 100 (b) 78 : 94
in the same way as the second term is related to the first (c) 64 : 72 (d) 98 : 114
term. Sol. (a)
ml fodYi dk p;u djsa tks rhljs in ls mlh çdkj lacaf/kr gS tSls Pattern : - [AB : AB + (A+B), AB Digit]
nwljk in igys in ls lacaf/kr gSA 48 : 60  48 + (4 + 8)  48 + 12 = 60
123 : 32 : : 541 : ______ Similarly:-
(a) 12 (b) 22 (c) 32 (d) 42 86 : 100  86 + (8 + 6)  86 + 14 = 100
Sol. (d) Correct Answer Option (a)
Pattern : - [ABC : A2+B2 c3, ABC- Digit]

241
Download Free PDFs & e-Books from Neon Classes App

1 1 17 :102:153 
Concept : Digit Operation : : , AB
AB  A  3 ,  B  4  17 102 153
: : 1:6:9
Digit 17 17 17
136. Select the option that is related to the third term Similarity : 
in the same way as the second term is related to the first (16,96,144)
term and the sixth term is related to the fifth term. 16 96 144
, , 1:6:9
ml fodYi dk p;u djsa tks rhljs in ls mlh rjg lacaf/kr gS tSl s 16 16 16
nwljk in igys in ls lacaf/kr gS vkSj NBk in ikaposa in ls lacaf/kr Correct Answer Option (c)
gSA Concept : Difference Based
1 1  1  1 1 139. Select the option in which the numbers are NOT
 97  : 63 :: 114  : ? ::  89  : 55 related in the same way as are the numbers of the
following set.
1 1 1 1 ml fodYi dk p;u djsa ftlesa la[;k,a mlh rjg lacafèkr ugha gSa
(a)   (b)   (c)   (d)  
 80   77   86   83  tSls fuEufyf[kr lsV dh la[;k,a gSaA
Sol. (a) (63, 76, 46)
1 1 (a) (45, 58, 28) (b) (72, 85, 55)
Pattern : - [ : , AB  Digit ]
AB  A  3 ,  B  4  (c) (83, 96, 66) (d) (69, 84, 52)
Sol. (d)
1 1 1 1
:   Pattern : - [Difference, Y-X = 13, Y- Z = 30]
97  9  3 , 7  4 6,3 63
(63,76, 46) 76–63=13, 76–46=30
1 1 1 1
:   Or
89  8  3 , 9  4 5,5 55
63 76 46
Similarly:-
1
:
1

1

1 Difference:-- 13 - 30
114 11  3 ,  4  4 8,0 80 (69,84, 52) 84–69=15, 84–52= 32
Correct Answer Option (a) Or
Concept : Ratio Based 63 76 46
137. Select the option that is related to the third
number in the same way as the second number is related - 15 - 32
to the first number.
Correct Answer Option (d)
ml fodYi dk p;u djsa tks rhljh la[;k ls mlh çdkj lacfa èkr gS
140. Select the option in which the numbers are NOT
tSls nwljh la[;k igyh la[;k ls lacafèkr gSA related in the same way as are the numbers of the
39 : 91 : : 51 : ? following set.
(a) 125 (b) 178 (c) 119 (d) 93 ml fodYi dk p;u djsa ftlesa la[;k,a mlh rjg lac afèkr ugha gSa
Sol. (c)
tSls fuEufyf[kr lsV dh la[;k,a gSaA
Pattern : - [Ratio – 3 : 7]
(64, 91, 118)
39 91 (a) (55, 82, 109) (b) (46, 73, 101)
39 : 91  :  3:7
13 13 (c) (72, 99, 126) (d) (48, 75, 102)
Similarly:- Sol. (b)
51 Pattern : - [Difference = [+27]]
51 :   3, 17  7  119
17 (64,91, 118) 64 91 118
Correct Answer Option (c)
138. Select the option in which the numbersare -27 -27
Difference:-
related in the same way as are the numbers in the given
(46,73, 101) 46 73 101
set.
ml fodYi dk p;u djs] ftlesa nh xbZ la[;kvksa ds chp ogh laca/k
-27 -28
gS] tks uhps fn, x, leqPp; dh la[;kvksa ds chp gSA
17: 102 : 153 Correct Answer Option (b)
(a) 23 : 162 : 207 (b) 18: 104: 171 141. Select the set in which the numbers are related
(c) 13:78: 108 (d) 16:96: 144 in the same way as are the numbers of the following sets.
Sol. (d) ml leqPp; dk p;u djsa ftlesa la[;k,¡ mlh çdkj lacaf/kr gSa tSls
Pattern : - [Ratio – 1 : 6 : 9] fuEufyf[kr leqPp;ksa dh la[;k,¡ gSaA
(NOTE : Operations should be performed on the whole
numbers, without breaking down the numbers into its
constituent digits. E.g. 13- Operations on 13 such as
adding /subtracting /multiplying etc. to 13 can be
performed. Breaking down 13 into 1 and 3 and then
performing mathematical operations on 1 and 3 is not
allowed.)

242
Best App for Govt. Jobs : Neonclasses (Download Now)

¼uksV : la[;kvksa dks muds ?kVd vadksa esa foHkkftr fd, fcuk] iw.kZ 382 : 283, 3 8 22 8 3
la[;kvksa ij lapkyu fd;k tkuk pkfg,A mnkgj.k ds fy, 13& 13
ij lapkyu tSls 13 dks tksM+uk / ?kVkuk / xq.kk djuk vkfn fd;k Similarity:-
tk ldrk gSA 13 dks 1 vkSj 3 esa rksM+uk vkSj fQj 1 vkSj 3 ij 564 : 465, 5 6 44 6 5
xf.krh; lafØ;k,¡ djus dh vuqefr ugha gSA½
(111, 208, 313), (36,12, 42)
Correct Answer Option (b)
(a) (41, 69, 107) (b) (66, 92, 151)
144. Select the option that is related to the third
(c) (120, 224, 342) (d) (28, 51, 73)
number in the same way as the second number is related
Sol. (d)
to the first number and the sixth number is related to the
Pattern : - [X+Y-6=Z]
fifth number.
(111 : 208 : 313)
ml fodYi dk p;u djsa tks rhljh la[;k ls mlh çdkj lacfa èkr gS
11, 208 313 tSls nwljh la[;k igyh la[;k ls lacafèkr gS vkSj NBh la[;k ikapoha
la[;k ls lacafèkr gSA
78 : 87 :: 43 : ? :: 65 : 56
111 + 208 = 319 (a) 45 (b) 33 (c) 44 (d) 34
(36 : 12 : 42) Sol. (d)
36, 12 42 Pattern : - [Digit Reverse]
78 : 87 :: 43 : ? :: 65 : 56
7 8 : 8 7, 6 5 : 5 6
36 + 12 = 48
Similarly : Similarity:-
(28 : 51 : 73) 43:34

28, 51 73
Correct Answer Option (d)
Concept : Rearrangement: Digit Rearrangement
28 + 51 = 79 145. Select the option that is related to the sixth
number in the same way as the first number is related to
Correct Answer Option (d) the second number and third number is related to fourth
142. Select the set in which the numbers are related number.
in the same way as are the numbers of the following set. ml fodYi dk p;u djsa tks NBh la[;k ls mlh çdkj lacfa èkr gS
ml leqPp; dk p;u dhft, ftlesa la[;k,¡ mlh çdkj lacaf/kr gSa tSls igyh la[;k nwljh la[;k ls lacafèkr gS vkSj rhljh la[;k pkSFkh
tSls fuEufyf[kr leqPp; dh la[;k,¡ gSaA la[;k ls lacafèkr gSA
(17, 12, 7)
8432 : 4382 :: 2496 : 4926 :: ? : 2174
(a) (42, 34, 27) (b) (23, 32, 39)
(a) 7421 (b) 7412 (c) 7214 (d) 7142
(c) (15, 19, 23) (d) (39, 28, 19)
Sol. (c)
Sol. (c)
Pattern : - [Digit Rearrangement]
Pattern : - [Difference between 1st and 2nd, and 2nd and 3rd are
8432 : 4382 :: 2496 : 4926 :: ? : 2174
same]
8432 : 4382 8 4 3 2
–5 –5 
 4 3 8 2
(17, 12, 7)  17 , 12 , 7
2496 : 4926 
Similarly:-

+4 +4
 
(15, 19, 23) , 19, 23
? : 2174 7 2 1 4 
Correct Answer Option (c)

Concept : Reverse Pattern: Digit Reverse
 
143. Select the option in which the two numbers are
related in the same way as are the numbers in the given Correct Answer Option (a)
number-pair. 146. Select the option in which the two numbers are
ml fodYi dk p;u djsa ftlesa nks la[;k,a mlh rjg lacafèkr gSa tSls related in the same way as are the numbers in the given
nh xbZ la[;k&;qXe esa la[;k,a gSaA number-pair.
382 : 283 :: ? ml fodYi dk p;u djsa ftlesa nks la[;k,a mlh rjg lacafèkr gSa tSls
(a) 962 : 629 (b) 564 : 465 nh xbZ la[;k&;qXe esa la[;k,a gSaA
(c) 384 : 843 (d) 784 : 847 248 : 428
Sol. (b) (a) 268 : 628 (b) 284 : 842
Pattern: - [Digit Reverse] (c) 462 : 264 (d) 428 : 824
Sol. (a)
Pattern : - [Digit Rearrangement]

243
Download Free PDFs & e-Books from Neon Classes App

Concept: Cube: When Three Numbers are Perfect


248 : 428 
Analogical Cube and One is not, The Not Analogical cube

pair option is different from the rest
 
4. Four number– pairs have been given, out of which three
Similarly:-
are alike in some manner and one is different. Select the
268 : 628  one that is different.
 pkj vad & tksM+s fn, x, gSa] ftuesa ls rhu fdlh u fdlh :i esa ,d
  tSls gSa vkSj ,d vyx gSA tks vyx gS mls pquAsa
Correct Answer Option (a) (a) 64 : 216 (b) 216 : 512
9. Number Classification (c) 125 : 343
Sol. (d)
(d) 343 : 728

Concept: Prime Number Pattern : - [CUBE]


Prime : Case when three out of 4 numbers are Primes The pattern followed here is : CUBE
1. Four numbers have been given out of which three are [ n3:(n+2)3]
alike in some manner, while one is different. Choose the (a) 43 : 63 (b) 63 = 83
odd one. (c) 53 : 73 (d) 73 : 728
pkj la[;k,¡ nh xbZ gSa ftuesa ls rhu fdlh u fdlh :i esa leku gSa] Except d, all other follow same pattern so d is odd one among
tcfd ,d vlaxr gSA vlaxr dk p;u djsAa all options.
(a) 117 (b) 103 (c) 107 (d) 113 Concept: Cube: N:N3 (In three Options THREE Analogical
Sol. (a) Numbers Pairs Follows this Pattern)
Pattern : - [Prime Number] 5. The second number in the given number pairs is obtained
(a) 117  [ 39 × 3] Not Prime by performing certain mathematical operation(s) on the
(b) 103 first number. The same operation(s) are followed in all
(c) 107 the number pairs, EXCEPT one. Find that odd number pair.
(d) 113 nh xbZ la[;k ;qXeksa esa nwljh] la[;k igyh la[;k ij fuf’pr xf.krh;
Theory : - Prime Numbers are natural numbers that are lafØ;k,¡ djds çkIr dh xbZ gSA fuEukafdr fodYiksa esa fn, x, la[;k
divisible by only I and the number it self. ;qXeksa esa ls ,d dks NksM+dj 'ks"k lHkh esa ogh lafØ;k,sa viukbZ xbZ gSA
Except a, all other follow same pattern so a is odd one among ml vlaxr la[;k ;qXe dk p;u dhft,A
all options. (a) 8 : 64 (b) 8 : 512
Concept: Prime : consecutive prime number numbers are (c) 6 : 216 (d) 4 : 64
related Sol. (a)
2. Four pairs of numbers have been given, out of which three Pattern : - [CUBE]
are alike in some manner, while one is different. Choose The pattern followed here is : CUBE
out the odd one. [ N:N3]
Pkj tksM+h la[;k,¡ nh xbZ gSa] ftuesa ls rhu fdlh u fdlh :© esa ,d (a) 82 = 64 (b) 83 = 512
tSlh gSa] tcfd ,d fHkUu gSA vyx dks pquAsa (c) 63 : 216 (d) 43 : 64
(a) 73 – 79 (b) 31 – 41 Except a, all other follow same pattern so a is odd one among
© 61 – 71 (d) 83 – 97 all options.
Sol. (a) Concept: Cube: CUBE+P , Where P can be any number . (In
Pattern : - [Prime No Next] Three options Some operation is performed with Cube)
(a) 73 – 79 [Not mid] 6. Three of the following four numbers are alike in a certain
(b) 31 – 41 [Mid – 37] way and one is different. Pick the number that is different
© 61 – 71 [Mid – 67] from the rest.
(d) 83 – 97 [Mid – 89] fuEufyf[kr pkj la[;kvksa esa ls rhu ,d fuf'pr rjhds ls leku gSa
Except a, all other follow same pattern so a is odd one among vkSj ,d vyx gSA og la[;k pqusa tks ckdh ls vyx gksA
all options. (a) 65 (b) 217 (c) 28 (d) 338
Concept: Cube: When Three Numbers are Perfect Cube and Sol. (d)
One is not. Pattern : - [CUBE+1]
3. Three of the following four numbers are alike in a certain The pattern followed here is : CUBE
way and one is different. Pick the number that is different [ n3+1]
from the rest.
(a) 43+1  64 +1 = 65
fuEufyf[kr pkj la[;kvksa esa ls rhu ,d fuf'pr rjhds ls leku gSa
(b) 63+1 216+1 = 217
vkSj ,d vyx gSA og la[;k pqusa tks ckdh ls vyx gksA
(a) 343 (b) 2197 (c) 731 (d) 1331 (c) 33 +1  27 +1 = 28
Sol. (c) Sol. (d) 338
Pattern : - [CUBE] Except d, all other follow same pattern so d is odd one among
The pattern followed here is all options.
(a) 73 = 343 (b) 133 = 2197 Concept: Cube: [N : N3-N] Analogical relation for a Number
(c) 731 (d) 113 = 1331 N , where P can be any random number
Except c, all other follow same pattern so c is odd one among
all options.

244
Best App for Govt. Jobs : Neonclasses (Download Now)

7. Four pair of numbers have been given out of which three 93  1 729  1 730
are alike in some manner, while one is different. Choose (d) 9: = = = 365 355
2 2 2
the odd one. Except d, all other follow same pattern so d is odd one among
Pkj tksM+h la[;k,¡ nh xbZ gSa ftuesa ls rhu fdlh u fdlh :© esa leku all options.
gS]a tcfd ,d fHkUu gSA vthc dk p;u djsAa 3
N 
(a) 13 : 2186 (b) 7 : 336 Concept: Cube: [N:   1  ] Analogical relation for a
© 11 : 1320 (d) 5 : 120 2 
Sol. (a) Number N , where P can be any random number
Pattern : - [N :N3-N] 10. The second number in the given number pairs is obtained
(a) 13 : 133 – 13 – 2197 – 13 = 2184  2186 by performing certain mathematical operation(s) on the
(b) 7 : 73 – 7 – 343 – 7 = 336 first number. The same operation(s) are followed in all
© 11 : 113 – 11 – 1331 – 11 = 1320 the number pairs except one. Find that odd number pair.
(d) 5 : 53 – 5 – 125 – 5 = 120 Nh xbZ la[;k ;qXeksa esa nwljh la[;k igyh la[;k ij ,d fuf’pr
Except a, all other follow same pattern so a is odd one among xf.krh; lafØ;k,¡ djds çkIr dh xbZ gSA ,d la[;k ;qXe dks NksM+dj
all options lHkh la[;k ;qXeksa esa leku xf.krh; lafØ;kvksa dk vuqlj.k fd;k x;k
Concept: Cube: [(N-p)3: N] Analogical relation for a gSA ml vlaxr la[;k ;qXe dks Kkr djsAa
Number N , where P can be any random number (a) 14 : 512 (b) 22 : 1728
8. In each of the number-pairs, the second number is © 46 : 12167 (d) 38 : 8000
obtained by performing a certain mathematical operation Sol. (c)
on the first number. Three of the following pairs follow 3
N 
the same pattern and thus form a group. Select the Pattern : - [N:   1  ]
number – pair that does NOT belong to that group. 2 
3
Fn, x, la[;k&;qXeksa esa ls izR;sd esa] nwljh la[;k] igyh la[;k ij  14 
(a) 14 :   1  = 7  1 = 8 = 512
3 3
,d fuf’pr xf.krh; lafØ;k djds izkIr dh tkrh gSA fuEufyf[kr esa  2 
ls rhu la[;k & ;qXe ,d leku iSVuZ dk ikyu djrs gS] vkSj bl 3
 22 
11  1
3
izdkj ,d lewg cukrs gSA ml la[;k & ;qXe dk p;u djs]a tks ml (b) 22 :   1  = 123 = 1728
lewg ls lacaf/kr ugha gSaA  2 
(a) 125 – 6 (b) 729 – 10  46 
3

© 46 :   1  =  23  1 = 243 = 13824  12167


3
© 512 – 9 (d) 343 – 5
 2 
Sol. (d)
3
Pattern : - [(N-1)3: N]  38 
(d) 38 :   1  = 19  1 = 20 = 8000
3 3

(a) (6 – 1)3 = 53 = 125  2 


(b)(10 – 1)3 =93 = 729 Except c, all other follow same pattern so c is odd one among
© (9 – 1)3 = 83 = 512 all options.
(d) ( 5 – 1)3 –=43 = 64  343 3
 N 1
Except d, all other follow same pattern so d is odd one among Concept: Cube: [N:   ] Analogical relation for a
all options.  2 
N3  1 Number N , where P can be any random number
Concept: Cube: [N: ] Analogical relation for a 11. The second number in the given number pairs is obtained
2
by performing certain mathematical operation(s) on the
Number N , where P can be any random number
first number. The same operation(s) are followed in all
9. The second number in the given number-pairs is obtained
the number pairs except one. Find that odd number pair.
by performing certain mathematical operation(s) on the
first number. The same operation(s) are followed in all
nh xbZ la[;k ;qXeksa esa nwljh la[;k igyh la[;k ij ,d fuf’pr
the number-pairs EXCEPT one. Find that odd number-pair. xf.krh; lafØ;k,¡ djds çkIr dh xbZ gSA ,d la[;k ;qXe dks NksM+dj
Nh xbZ la[;k ;qXeksa esa nwljh] la[;k igyh la[;k ij fuf’pr xf.krh; lHkh la[;k ;qXeksa esa leku xf.krh; lafØ;kvksa dk vuqlj.k fd;k x;k
lafØ;k,¡ djds çkIr dh xbZ gSA fuEukafdr fodYiksa esa fn, x, la[;k gSA ml vlaxr la[;k ;qXe dks Kkr djsAa
;qXeksa esa ls ,d dks NksM+dj ‘ks”k lHkh esa ogh lafØ;k,sa viukbZ xbZ (a) 9:125 (b) 11:49
gSA ml vlaxr la[;k ;qXe dk p;u dhft,A (c) 13:343 (d) 7:64
Sol. (b)
(a) 3 : 14 (b) 5 : 63
3
© 7 : 172 (d) 9 : 355  N 1
Pattern : - [N:   ]
Sol. (d)  2 
N3  1 3 3
Pattern : - [N: ]  91  10  3
2 (a) 9 : 
2  =  2  = 5 = 125
   
33  1 27  1 28
(a) 3 : = = = 14 3
 11  1   12 
3
2 2 2 (b) 11 : 
3
 =   = 6 = 216  49
53  1 125  1 126  2   2
(b) 5 : = = = 63 3 3
2 2 2  13  1   14  3
(c) 13 :   =   = 7 = 343
73  1 343  1 344  2   2
©7: = = = 172
2 2 2

245
Download Free PDFs & e-Books from Neon Classes App

3 3
71 8 3 (a) 225 : 15  225 = 15
(d) 7 :   =  2  = 4 = 64
 2    (b) 144 : 12  144 = 15
Except b, all other follow same pattern so b is odd one among
all options. (c) 123 : 11  123 = 11 (Not proper)
Concept: Square : When Three Numbers are Perfect (d) 196 : 14  196 = 14
Square and One is not. Except c, all other follow same pattern so c is odd one among
12. Four number-pairs have been given, out of which three all options.
are alike in some manner and one is different. Select the Concept: Square : Multiple of N2 (Numbers are related with
number-pair that is different. some operation on square)
pkj la[;k&lewg fn, x, gSa] ftuesa ls rhu fdlh u fdlh :i esa 15. Four number-pairs have been given, out of which three
,d leku gSa vkSj ,d vaxlr gSA ml la[;k lewg dk p;u dhft, are alike in some manner and one is different. Select the
tks vlaxr gSA number-pair that is different.
(a) 289 : 64 (b) 324 : 64 Pkj la[;k&lewg fn, x, gSa] ftuesa ls rhu fdlh u fdlh :© esa ,d
(c) 169 : 14 (d) 256 : 49 leku gSa vkSj ,d vaxlr gSA ml la[;k lewg dk p;u dhft, tks
Sol. (b) vlaxr gSA
Pattern : - X : Y, X = digit, (Add digit)2=Y (a) 19 : 622 (b) 14 : 392
© 11 : 242 (d) 12 : 288
(a) 289 – 289 = 17= (1+7) = 8, 8 = 64 2
Sol. (a)
(b) 324 – 324 = 18= (1+8) = 9, 92 = 81  64 Pattern : - [ N : N2  2 ]
(c) 169 – 169 = 13 = (1+3) = 4, 42 = 16 (a) 19 : 192  2  361  2 = 722  622
(d) 256 – 256 = 16 = (1 +6) = 7, 72 = 49 (b) 14 : 142  2  196  2 = 392
Except b, all other follow same pattern so b is odd one among © 11 : 112  2  121  2 = 242
all options. (d) 12 : 122  2  144  2 = 288
2
Concept: Square : [ N : N ] Two Numbers are analogically Except a, all other follow same pattern so a is odd one among
square numbers related to each other all options.
13. Four number-pairs have been given, out of which three Concept: Square : (Multiple of N)2
are alike in some manner and one is different. Select the 16. The second number in the given number pairs is obtained
number-pair that is different from the rest. by performing certain mathematical operation(s) on the
Pkj la[;k&tksM+s fn, x, gSa] ftuesa ls rhu fdlh u fdlh :© esa ,d first number. The same operation(s) are followed in all
tSls gSa vkSj ,d fHkUu gSA ml la[;k&;qXe dk p;u dhft, tks vU; the number pairs except one. Find that odd number pair.
ls fHkUu gSA Nh xbZ la[;k ;qXeksa esa nwljh la[;k igyh la[;k ij ,d fuf’pr
(a) 17 : 289 (b) 13 : 169 xf.krh; lafØ;k,¡ djds çkIr dh xbZ gSA ,d la[;k ;qXe dks NksM+dj
© 11 : 121 (d) 15 : 250 lHkh la[;k ;qXeksa esa leku xf.krh; lafØ;kvksa dk vuqlj.k fd;k x;k
Sol. (d) gSA ml vlaxr la[;k ;qXe dks Kkr djsAa
Pattern : - Square [ N : N2 ] (a) 16:1024 (b) 14:784
(a) 17 : 172 = 289 (c) 18:1620 (d) 12:576
(b) 13 : 132 = 169 Sol. (c)
Pattern : - [ N : 2N  ]
2
© 11 : 112 = 121
(d) 15 : 152 = 225  250
(a) 16 : (2  16)2  (32)2 = 1024
Except d, all other follow same pattern so d is odd one among
all options. (b) 14 : (2  14)2  (28)2 = 784
14. Three of the following four number-pairs are alike in a (c) 18 : (2  18)2  (36)2 = 1296  1620
certain way and one is different. Pick the odd number-pair (d) 12 : (2  12)2  (24)2 = 576
out. Except c, all other follow same pattern so c is odd one among
fuEufyf[kr pkj la[;k&tksM+ksa esa ls rhu ,d fuf'pr rjhds ls leku all options.
gSa vkSj ,d vyx gSA fo"ke la[;k&;qXe dks pquAsa Concept: Square : [N : N2 + N]
(a) 225 : 15 (b) 144 : 12 17. Four pairs of numbers have been given, out of which three
(c) 123 : 11 (d) 196 : 14 are alike in some manner, while one is different. Choose
Sol. (c) out the odd one.
Pattern : - Square [ N2 : N ] Pkj tksM+h la[;k,¡ nh xbZ gSa] ftuesa ls rhu fdlh u fdlh :i esa ,d
The pattern followed here is : Square [ N2 : N ] tSlh gSa] tcfd ,d fHkUu gSA fo"ke dks pquAsa
(a) 225 : 15  152 : 15 (a) 25 : 650 (b) 14 : 210
(c) 7 : 50 (d) 21 : 462
(b) 144 : 12  122 : 12
Sol. (c)
(c) 123 : 11  112 : 11 [121 123] Pattern : - [N : N2 + N]
(d) 196 : 14  142 : 14 (a) 25 : 252 + 25  625 + 25 = 650
method: 2 (b) 14 : 142 + 14  196 + 14 = 210
Pattern : - Proper Underoot

246
Best App for Govt. Jobs : Neonclasses (Download Now)

© 7 : 72 + 7  49 + 7 = 56  50 20. Four number-pairs have been given, out of which three


are alike in some manner and one is different. Select the
(d) 21 : 212 +21  441 + 21 = 462
number-pair that is different.
Except c, all other follow same pattern so c is odd one among
Pkj la[;k&lewg fn, x, gSa] ftuesa ls rhu fdlh u fdlh :© esa ,d
all options.
leku gSa vkSj ,d vaxlr gSA ml la[;k lewg dk p;u dhft, tks
Concept: Square : [ 2N:N 2 ] The relation between the given
vlaxr gSA
numbers (a) 22 : 490 (b) 21 : 447
18. Four number-pairs are given out of which three are alike © 25 : 631 (d) 30 : 908
in a certain way and one is different. Select the number- Sol. (d)
pair that is different from the rest.
Pattern : - [ N : N2  6 ]
pkj la[;k&lewg fn, x, gSa] ftuesa ls rhu fdlh u fdlh :i esa ,d
leku gSa vkSj ,d vaxlr gSA ml la[;k lewg dk p;u dhft, tks (a) 22 : 222+ 6  484 + 6 = 490
vlaxr gSA (b) 21 : 212 + 6  441 + 6 = 447
(a) 16 : 64(b) 34 : 289 (c) 25 : 252 + 6  625 + 6 = 631
(c) 24 : 142 (d) 14 : 49 (d) 30 : 302 + 6 900 + 6 = 906  908
Sol. (c)
Except d, all other follow same pattern
Pattern : [ 2N : N2]
Concept:Square : [ N2 : (N  K) ] Where k can be any
(a) 16 : 64  2  8 : 82
number
(b) 34 : 289  2  17 : 172 21. The second number in the given number-pairs is obtained
(c) 24 : 142  2  12 : 122 = 144  142 by performing certain mathematical operation(s) on the
(d) 14 : 49  2  7 : 72 first number. The same operation(s) are followed in all
Method: 2 the number-pairs EXCEPT one. Find that odd number-pair.
2 Nh xbZ la[;k ;qXeksa esa nwljh] la[;k igyh la[;k ij fuf’pr xf.krh;
X
Pattern : - [ X :   ] lafØ;k,¡ djds çkIr dh xbZ gSA fuEukafdr fodYiksa esa fn, x, la[;k
2
;qXeksa esa ls ,d dks NksM+dj ‘ks”k lHkh esa ogh lafØ;k,sa viukbZ xbZ
2
 16  gSA ml vlaxr la[;k ;qXe dk p;u dhft,A
(a) 16 :    82 = 64
 2  (a) 81 : 12(b) 64 : 11
2 (c) 100 : 13 (d) 121 : 16
 34  Sol. (d)
(b) 34 :    172 = 289
 2  Pattern : - [ N2 : N +3]
2 (a) 81 : 12, 92 : 9 + 3
 24 
(c) 24 :    122 = 144  142 (b) 64 : 11, 82 : 8 + 3
 2 
(c) 100 : 13, 102 : 10 + 3
2
 14  (d) 121 : 16, 112 : 11 + 3 = 14  16
(d) 14 :    72 = 49
 2  method : 2
Except c, all other follow same pattern so c is odd one among Pattern : - [ N: N  3 ]
all options. 81 + 3  9 + 3 = 12
(a) 81 :
Concept: Square : [ N :2N:N2 ] The numbers of given sets
(b) 64 : 64 + 3  8 + 3 = 11
are driven from First Number
19. Four number triads have been given, out of which three © 100 : 100 + 3  10 + 3 = 13
are alike in some manner and one is different. Select the
(d) 121 : 121 + 3  11 + 3 = 14  16
number triad that is different.
Pkj la[;k f=d fn, x, gS] ftuesa ls rhu fdlh fo’ks”k fu;e ds Except d, all other follow same pattern so d is odd one among
all options.
vuqlkj laxr gS vkSj dksbZ ,d vlaxr gSA ml vlaxr la[;k f=d dk
Concept: Square : [ N :  N  P ] , Where P can be any
2
p;u dhft,A
(a) (13, 26, 169) (b) (17, 34, 278) number
(c) (15, 30, 225) (d) (12, 24, 144) 22. The second number in the given number-pairs is obtained
Sol. (b) by performing certain mathematical operation(s) on the
Pattern : - [ N :2N:N2 ] first number. The same operation(s) are followed in all
the number-pairs EXCEPT one. Find that odd number-pair.
(a) (13, 26, 169)  (13 : 2  13 : 132)
Nh xbZ la[;k ;qXeksa esa nwljh] la[;k igyh la[;k ij fuf’pr xf.krh;
(b) (17, 34, 278)  (17 : 2  17 : 172) = 278  289 lafØ;k,¡ djds çkIr dh xbZ gSA fuEukafdr fodYiksa esa fn, x, la[;k
(c) (15, 30, 225)  (15 : 2  15 : 152) ;qXeksa esa ls ,d dks NksM+dj ‘ks”k lHkh esa ogh lafØ;k,sa viukbZ xbZ
(d) (12, 24, 144)  (12 : 2  12 : 122) gSA ml vlaxr la[;k ;qXe dk p;u dhft,A
Except b, all other follow same pattern (a) 25 : 3 (b) 64 : 6
Concept: Square : [ N : N2  P ] , Where P can be any © 100 : 9 (d) 81 : 7
Sol. (c)
number (Square of the Number is divided by a Number)
Pattern : - [ N: N 2 ]

247
Download Free PDFs & e-Books from Neon Classes App

The pattern followed here is : [ N: N 2 ] N


Concept: Square : [ N : N 
2
] Half of the Number is
2
(a) 25 : 3  25  2  5 – 2 = 3
added to the Square of the same number
(b) 64 : 6  64  2  8 – 2 = 6 25. The second number in each of the number – pairs is
(c) 100 : 9  100  2  10 – 2 = 8  9 obtained by performing certain mathematical operations
on the first number. Three of the following four number –
(d) 81 : 7  81  2  9 – 2 = 7 pairs follow the same pattern and thus form a group.
method : 2 Select the number-pair that does NOT belong to that
Pattern : - [  N  2 : N ]
2 group.
çR;sd la[;k&;qXe esa nwljh la[;k igyh la[;k ij dqN xf.krh;
(a) (3 + 2)2  52  25 : 3 lafØ;k,¡ djus ls çkIr gksrh gSA fuEufyf[kr pkj la[;kvksa esa ls rhu
(b) (6 + 2)2  82  64 : 6 & tksM+s ,d gh iSVuZ dk vuqlj.k djrs gSa vkSj bl çdkj ,d lewg
© (9 + 2)2  112 = 121  100 : 9 cukrs gSaA ml la[;k&;qXe dk p;u djsa tks ml lewg ls lacafèkr ugha
(d) (7 + 2)2  92  81 : 7 gSA
Except c, all other follow same pattern so c is odd one among (a) 2 : 5 (b) 4 : 18 (c) 6 : 39 (d) 8 : 66
all options. Sol. (d)
Concept: Square : [(N +P)2 : N], Where P can be any N
Pattern : - [ N : N2  ]
number 2
23. The second number in the given number-pairs is obtained 2
(a) 2 : 22  4+1=5
by performing certain mathematical operation(s) on the 2
first number. The same operation(s) are followed in all 4
the number-pairs EXCEPT one. Find that odd number-pair. (b) 4 : 42   16 + 2 = 18
2
Nh xbZ la[;k ;qXeksa esa nwljh] la[;k igyh la[;k ij fuf’pr xf.krh; 6
lafØ;k,¡ djds çkIr dh xbZ gSA fuEukafdr fodYiksa esa fn, x, la[;k (c) 6 : 62   36 + 3 = 39
2
;qXeksa esa ls ,d dks NksM+dj ‘ks”k lHkh esa ogh lafØ;k,sa viukbZ xbZ
8
gSA ml vlaxr la[;k ;qXe dk p;u dhft,A (d) 8 : 82   64 + 4 = 68  66
2
(a) 64 : 6 (b) 25 : 2
Except d, all other follow same pattern so d is odd one among
(c) 225 : 12 (d) 169 : 10
all options.
Sol. (a)
Pattern : - [(N +3)2 : N] N2
Concept: Square : [ N : ] , Where P can be any number
The pattern followed here is : [(N +3)2 : N] P
(a) 64 : 6  (6 + 3 )2  92 = 81  64 (Square of the Number is divided by a Number)
26. The second number in the given number-pairs is obtained
(b) 25 : 2  (2 + 3 )2  52 = 25
by performing certain mathematical operation(s) on the
(c) 225 : 12  (12 + 3 )2  152 = 225 first number. The same operation(s) is/are followed in all
(d) 169 : 10  (10 + 3 )2  132 = 169 the number-pairs, except one. Find that odd number-pair.
Except a, all other follow same pattern so a is odd one among Nh xbZ la[;k&;qXeksa esa nwljh la[;k igyh la[;k ij dqN xf.krh;
all options. lafØ;k,¡ djds çkIr dh tkrh gSA ,d dks NksM+dj lHkh la[;k&tksM+ksa
Concept: Square : [N : (N-k)2] , Where k can be any number esa leku lafØ;kvksa dk ikyu fd;k tkrk gS@gSaA og fo”ke
24. In each of the number-pairs, the second number is la[;k&;qXe Kkr dhft,A
obtained by performing a certain mathematical operation (a) 12:9 (b) 8:4 (c) 4:15 (d) 20:25
on the first number. Three of the following pairs follow Sol. (c)
the same pattern and thus form a group. Select the
N2
number-pair that does NOT belong to that group. Pattern : - [ N : ]
16
çR;sd la[;k&;qXe esa] igyh la[;k ij ,d fuf’pr xf.krh; lafØ;k
djds nwljh la[;k çkIr dh tkrh gSA fuEufyf[kr esa ls rhu tksM+s ,d 122 144
(a) 12 :  =9
gh iSVuZ dk ikyu djrs gSa vkSj bl çdkj ,d lewg cukrs gSaA ml 16 16
la[;k&;qXe dk p;u djsa tks ml lewg ls lacafèkr ugha gSA 82 64
(b) 8 :  =4
(a) 324 – 19 (b) 625 – 26 16 16
(c) 441 – 22 (d) 144 – 15 42 16
Sol. (d) ©4:  = 1  15
16 16
Pattern : - [(N – 1)2 : N]
The pattern followed here is : [(N – 1)2 : N] 202 400
(d) 20 :  = 25
16 16
(a) 324 – 19 (19 – 1)2  182 = 321
Except c, all other follow same pattern so c is odd one among
(b) 625 – 26 (26 – 1)2  252 = 325
all options.
(c) 441 – 22 (22 – 1)2  212 = 441 2
N
(d) 144 – 15 (15 – 1)2  142 = 196  144 Concept: Square: [ N :   ], Where P can be any number.
P
Except d, all other follow same pattern so d is odd one among
all options. (Square of the Number is divided by a Number)

248
Best App for Govt. Jobs : Neonclasses (Download Now)

27. Four pairs of numbers have been given, out of which three 32  1 91 10
are alike in some manner, while one is different. Choose (b) 3 :   =5
2 2 2
out the odd one.
72  1 49  1 50
Pkj tksM+h la[;k,¡ nh xbZ gSa] ftuesa ls rhu fdlh u fdlh :© esa ,d (c) 7 :   = 25
tSlh gSa] tcfd ,d fHkUu gSA fo”ke dks pquAsa 2 2 2
(a) 34 – 289 (b) 16 – 64 52  1 25  1 26
(d) 5 :   = 13
© 26 – 168 (d) 18 – 81 2 2 2
Sol. (c) Except a, all other follow same pattern so a is odd one among
N
2 all options.
Pattern : - [ N :   ] Concept: Square : [ N : (N  P1)2  P2 ] , Where P1 and P2
2
2 can be any number
 34  30. Four number-pairs have been given, out of which three
(a) 34 :    172 = 289
 2  are alike in some manner and one is different. Select the
2 number-pair that is different.
 16 
(b) 16 :    82 = 64 pkj uacj&isy fn, x, gSa] ftuesa ls rhu fdlh rjg ls ,d tSls gSa
 2 
vkSj ,d vyx gSA ml la[;k&;qXe dk p;u djsa tks fHkUu gSA
2
 26  (a) 10 : 123 (b) 17 : 290
© 26 :    132 = 169  168
 2  (c) 11 : 146 (d) 14 : 227
2
Sol. (b)
 18 
(d) 18 :    92 = 81 Pattern : - [ N :  N  1  2 ]
2

 2 
Except c, all other follow same pattern so c is odd one among (a) 10 : (10 + 1)2 + 2  112 + 2  121 + 2 = 123
all options. (b) 17 : (17 + 1)2 + 2  182 + 2  324 + 2 = 326  290
Concept: Square : [N : N2 – P] , Where P can be any (c) 11 : (11 + 1)2 + 2  122 + 2  144 + 2 = 146
number(A Number is Subtracted from Square of a (d) 14 : (14 + 1)2 + 2  152 + 2  225 + 2 = 227
number)
Except b, all other follow same pattern so b is odd one among
28. Four number-pairs have been given, out of which three
all options.
are alike in some manner and one is different. Select the
Concept: Square : [ N2  P : (N  K)2  P ] , Where P and K can
number-pair that is different from the rest.
La[;kvksa ds pkj ;qXe fn, x, gSa] ftuesa ls rhu fdlh u fdlh rjg ls be any number
leku gSa vkSj ,d vleku gSA ml vleku la[;k&;qXe dk p;u djsAa 31. Three of the following four number-pairs are alike in a
(a) 35 : 1221 (b) 21 : 437 certain way and one is different. Pick the odd one out.
(c) 11 : 118 (d) 33 : 1085 fuEufyf[kr pkj la[;k&;qXeksa esa ls rhu ,d fuf’pr rjhds ls leku
Sol. (c) gSa vkSj ,d fHkUu gSA fo”ke dks pqusAa
Pattern : - [N : N2 – 4] (a) 17-37 (b) 65-101
The pattern followed here is : [N : N2 – 4] (c) 49-82 (d) 26-50
Sol. (c)
(a) 35 : 352 – 4  1225 – 4 = 1221
Pattern : - [ N2  1 : (N  2)2  1 ]
(b) 21 : 212 – 4  441 – 4 = 437
(a) 17 : 37  42+ 1 : 62 + 1
(c) 11 : 112 – 4  121 – 4 = 117  118
(b) 65 : 101  82 + 1 : 102 + 1
(d) 33 : 332 – 4  1089 – 4 = 1085
Except c, all other follow same pattern so c is odd one among (c) 49 : 82 72 + 1( 50  49) : 92 + 1
all options. (d) 26 : 50  52 +1 : 72 + 1
N2  1 Except c, all other follow same pattern so c is odd one among
Concept: Square : [N : ] all options.
2
29. The second number in the given number-pairs is obtained Concept: Square : [1ST TWO DIGHT ROOT+LAST TWO
by performing certain mathematical operation(s) on the DIGHT ROOT =N] (Operations on pair of digits , on group
first number. The same operation(s) are followed in all of 2 digits)
the number-pairs EXCEPT one. Find that odd number-pair. 32. Four pairs of numbers have been given, out of which three
nh xbZ la[;k ;qXeksa esa nwljh] la[;k igyh la[;k ij fuf’pr xf.krh; are alike in some manner, while one is different. Choose
out the odd one.
lafØ;k,¡ djds çkIr dh xbZ gSA fuEukafdr fodYiksa esa fn, x, la[;k
La[;kvksa ds pkj tksM+s fn, x, gSa] ftuesa ls rhu fdlh u fdlh :© esa
;qXeksa esa ls ,d dks NksM+dj 'ks"k lHkh esa ogh lafØ;k,sa viukbZ xbZ gSA
,d tSls gSa] tcfd ,d fHkUu gSA fo”ke dks pqusaA
ml vlaxr la[;k ;qXe dk p;u dhft,A
(a) 4925 : 35 (b) 1625 : 9
(a) 1 : 2 (b) 3 : 5 (c) 7 : 25 (d) 5 : 13
(c) 4964 : 15 (d) 3681 : 15
Sol. (a)
Sol. (a)
N2  1 Pattern : - [1ST TWO DIGHT ROOT+LAST TWO DIGHT ROOT
Pattern : - [N : ]
2 =N]
12  1 11 2 49 + 25  7 + 5 = 12  35
(a) 1 :   =12 (a) 4925 :
2 2 2

249
Download Free PDFs & e-Books from Neon Classes App

(b) 1625 : 16 + 25  4 + 5 = 9 (b) 9 : 93+92  729 + 81 = 810

49 + 64  7 + 8 = 15 © 15 : 153+152  3375 + 225 = 3600  3150


(c) 4964 :
(d) 12 : 123+122  1728 + 144 = 1872
(d) 3681 : 36 + 81  6 + 9 = 15 Except c, all other follow same pattern so c is odd one among
Except a, all other follow same pattern so a is odd one among all options.
all options.
Concept: Square & Cube Mixed Concept: Square : [N : N3 – N2] (When a Number is related
Square : [X2 : Y3] (Perfect Square and Perfect Cubes of a to Cube – Square of the same number
number is given (When a perfect Square number is 36. Four number-pairs have been given, out of which three
related to perfect cube in a analogical relation.)) are alike in some manner and one is different. Select the
33. Three of the following four number-pair are alike in a number-pair that is different.
certain way and one is different. Pick the odd pair out. pkj la[;k&lewg fn, x, gSa] ftuesa ls rhu fdlh u fdlh :i esa
fuEufyf[kr pkj esa ls rhu la[;k&;qXe ,d fuf'pr rjhds ls leku gSa ,d leku gSa vkSj ,d vaxlr gSA ml la[;k lewg dk p;u dhft,
vkSj ,d vyx gSA fo"ke tksM+h dks pqusAa tks vlaxr gSA
(a) 4 : 27 (b) 16 : 125 (a) 6 : 180(b) 7 : 294
(c) 9 : 64 (d) 49 : 218 (c) 5 : 100 (d) 4 : 52
Sol. (d) Sol. (d)
Pattern : - [X2 : Y3] Pattern : - [N : N3 - N2]
The pattern followed here is : [X2 : Y3]
(a) 6 : 180  63 - 62  216 - 36 = 180
(a) 4 : 27  22 : 33
(b) 7 : 294  73 - 72  343 - 49 = 294
(b) 16 : 125  42 : 53
(c) 5 : 100  53 - 52  125 - 25 = 100
(c) 9 : 64  32 : 43
(d) 4 : 52  43 - 42  64 - 16 = 48  52
(d) 49 : 218  72 : 63  216  218 Except d, all other follow same pattern so d is odd one among
Except d, all other follow same pattern so d is odd one among all options.
all options. Concept: Square : [N2 + P : N3], Where p, can any number
Concept: Square : [N : N2 : N3] (: When Square and Cube of a (When Operation on perfect Square number is related to
number are related in a set) perfect cube in a analogical relation)
34. Four sets of numbers are given out of which three are 37. In each of the number-pairs, the second number is
alike in a certain way and one is different. Select the set of obtained by performing a certain mathematical operation
numbers that is different from the rest. on the first number. Three of the following pairs follow
La[;kvksa ds pkj lsV fn, x, gSa ftuesa ls rhu ,d fuf’pr rjhds ls the same pattern and thus form a group. Select the
leku gSa vkSj ,d vyx gSA la[;kvksa ds ml leqPp; dk p;u djsa number-pair that does NOT belong to that group.
tks vU; ls fHkUu gSA çR;sd la[;k&;qXe esa] igyh la[;k ij ,d fuf’pr xf.krh; lafØ;k
(a) 5 – 25 – 225 (b) 6 – 36 – 216 djds nwljh la[;k çkIr dh tkrh gSA fuEufyf[kr esa ls rhu tksM+s ,d
© 9 – 81 – 729 (d) 3 – 9 – 27 gh iSVuZ dk ikyu djrs gSa vkSj bl çdkj ,d lewg cukrs gSaA ml
Sol. (a) la[;k&;qXe dk p;u djsa tks ml lewg ls lacafèkr ugha gSA
Pattern : - [N : N2 : N3] (a) 82 – 729 (b) 26-125
(a) (5 : 25 : 225)  (5 : 52 : 53) = 125  225 (c) 50-343 (d) 68-512
(b) (6 : 36 : 216)  (6 : 62 : 63) Sol. (d)
© (9 : 81 : 729)  (9 : 92 : 93) Pattern : - [N2 + 1 : N3]
The pattern followed here is : [N2 + 1 : N3]
(d) (3 : 9 : 27)  (3 : 32 : 33)
Except a, all other follow same pattern so a is odd one among (a) 82 : 729  92 +1 : 93
all options. (b) 26 : 125  52 +1 : 53
Concept: Square : [N3 + N2 : N] Here Sum of Square of a (c) 50 : 343  72 +1 : 73
number is related to Number . (d) 68 : 512  82 + 4 ( 82+1): 83
35. The second number in the given number pairs is obtained Except d, all other follow same pattern so d is odd one among
by performing certain mathematical operation(s) on the all options.
first number. The same operation(s) are followed in all Concept: Square : SQUARE-P : CUBE-P], where P can be any
the number pairs except one. Find that odd number pair. Number . (When square and Cube of a number is related in
Nh xbZ la[;k ;qXeksa esa nwljh la[;k igyh la[;k ij ,d fuf’pr this manner.
xf.krh; lafØ;k,¡ djds çkIr dh xbZ gSA ,d la[;k ;qXe dks NksM+dj 38. The second number in the given number pairs is obtained
lHkh la[;k ;qXeksa esa leku xf.krh; lafØ;kvksa dk vuqlj.k fd;k x;k by performing certain mathematical operation(s) on the
gSA ml vlaxr la[;k ;qXe dks Kkr djsAa first number. The same operation(s) are followed in all
(a) 7 : 392(b) 9 : 810 the number pairs, except one. Find that odd number pair.
© 15 : 3150 (d) 12 : 1872 Nh xbZ la[;k ;qXeksa esa nwljh] la[;k igyh la[;k ij fuf’pr xf.krh;
Sol. (c) lafØ;k,¡ djds çkIr dh xbZ gSA fuEukafdr fodYiksa esa fn, x, la[;k
Pattern : - [N : N3 + N2] ;qXeksa esa ls ,d dks NksM+dj ‘ks”k lHkh esa ogh lafØ;k,sa viukbZ xbZ
(a) 7 : 73+72  343 + 49 = 392 gSA ml vlaxr la[;k ;qXe dk p;u dhft,A

250
Best App for Govt. Jobs : Neonclasses (Download Now)

(a) 3 : 7 (b) 15 : 63 (a) 14:21014(14+ 1)  14  15 = 210


(c) 8 : 26 (d) 23 : 123
(b) 18:34218(18+ 1)  18  19 = 342
Sol. (d)
Pattern : - [N2 -1 : N3 – 1] (c) 17 : 307  17(17 + 1)  17  18 = 306  307
(a) 3 : 7  22 – 1 : 23 – 1  4 – 1 : 8 – 1 (d) 12: 156  12(12+1)  1213 = 156
Except c, all other follow same pattern so c is odd one among
(b) 15 : 63  42–1 : 16 – 1 : 64 – 1
43–1
all options.
(c) 8 : 26  32– 1 : 33 – 1  9 – 1 : 27 – 1 Concept: Multiplication: N-(kN-P) : when second number
(d) 23 : 123  52 – 1 : 53 – 1  52 – 2 : 53 – 2 25 – 1 = 24  23 is k times first – some number P
: 125 – 1 = 124  123 42. Four number-pairs have been given, out of which three
Except d, all other follow same pattern so d is odd one among are alike in some manner and one is different. Select the
all options. number-pair that is different from therest.
Concept: Multiplication Based La[;kvksa ds pkj ;qXe fn, x, gSa] ftuesa ls rhu fdlh u fdlh rjg ls
Multiplication: [XY=N] the multiplication of both leku gSa vkSj ,d vleku gSA ml vleku la[;k&;qXe dk p;u djsAa
analogically related terms is same in options. (a) 19-114 (b) 17-102
39. Four number – pairs have been given, out of which three (c) 12-72 (d) 28-164
are alike in some manner and one is different. Select the Sol. (d)
number – pairs that is different. Pattern : - [MULTIPLICATION, N – 6]
Pkj la[;k&lewg fn, x, gSa] ftuesa ls rhu fdlh u fdlh :© esa ,d (a) 19 – 114, 6 × 19  114
tSls gSa vkSj ,d fHkUu gSA la[;k lewg dk p;u djsa tks vyx gSaA (b) 17 – 102, 6 × 17 =102
(a) 16, 7 (b) 14, 8 (c) 28, 5 (d) 56, 2 (c) 12 – 72, 6 × 12  72
Sol. (c)
(d) 28 – 164, 6 × 28  168  112
The pattern followed here is : [MULTIPLICATION =N (XY =N]
Except d, all other follow same pattern so d is odd one among
(a) (16, 7), 16 × 7  112
all options.
(b) (14, 8), 14 × 8 =112
Concept: Multiplication: [AB : A × B] digit based
(c) (28, 5), 28 × 5  140  112 Multiplication
(d) (56, 2), 56 × 2  112 43. Four number-pairs have been given, out of which three
Except c, all other follow same pattern so c is odd one among are alike in some manner and one is different. Select the
all options. number-pair that is different from the rest.
Concept: Multiplication: [N: kN], where k can be any la[;kvksa ds pkj ;qXe fn, x, gSa] ftuesa ls rhu fdlh u fdlh rjg
number(: second Number is Fix times the first number) ls leku gSa vkSj ,d vleku gSA ml vleku la[;k&;qXe dk p;u
40. Four number-pairs have been given, out of which three djsAa
are alike in some manner and one is different. Select the (a) 76 : 42(b) 44 : 16
number-pair that is different. (c) 55 : 25 (d) 28 : 56
Pkj la[;k&lewg fn, x, gSa] ftuesa ls rhu fdlh u fdlh :© esa ,d Sol. (d)
leku gSa vkSj ,d vaxlr gSA ml la[;k lewg dk p;u dhft, tks Pattern : - [AB : A × B ]
vlaxr gSA (a) 76 : 42, 76 : 7 × 6  42
(a) (16, 112) (b) (18, 128) (b) 44 : 16, 44 : 4 × 4 =16
(c) (15, 105) (d) (17, 119) © 55 : 25, 55 : 5 × 5  25
Sol. (a)
(d) 28 : 56, 28 : 2 × 8  16  56
Pattern : - [N : 7N]
Except d, all other follow same pattern so d is odd one among
(a) (16, 112), 16 : 7 × 16  112
all options.
(b) (18, 128), 18 : 7 × 18 =126  128 Concept: Multiplication: kN – P , where k and P can be any
(c) (15, 105), 15 : 7 × 15  105 Numbers
(d) (17, 119), 17 : 7 × 17  119 44. Four number-pairs have been given, out of which three
Except b, all other follow same pattern so b is odd one among are alike in some manner and one is different. Select the
all options. number-pair that is different.
Concept: Concept: Multiplication: [N: N(N+P)], When P can Pkj la[;k&lewg fn, x, gSa] ftuesa ls rhu fdlh u fdlh :© esa ,d
be any number leku gSa vkSj ,d vaxlr gSA ml la[;k lewg dk p;u dhft, tks
41. Four number-pairs have been given, out of which three vlaxr gSA
are alike in some manner and one is different. Select the (a) 27 : 181 (b) 21 : 139
number-pair that is different. © 25 : 167 (d) 15 : 197
Pkj la[;k&lewg fn, x, gSa] ftuesa ls rhu fdlh u fdlh :© esa ,d Sol. (d)
leku gSa vkSj ,d vaxlr gSA ml la[;k lewg dk p;u dhft, tks Pattern : - [N : 7N – 8]
vlaxr gSA (a) 27:181 7  27 – 8  189 – 8 = 181
(a) 14 : 210 (b) 18 : 342 (b) 21 : 1397 21 – 8  147 – 8 = 139
(c) 17 : 307 (d) 12 : 156
© 25 : 167 725 – 8  175 – 8 = 167
Sol. (c)
Pattern : - [N : N(N +1)] (d) 15 : 197  7  15 – 8  105 – 8 = 97  197

251
Download Free PDFs & e-Books from Neon Classes App

Except d, all other follow same pattern so d is odd one among 48. Four number-pairs have been given, out of which three
all options. are alike in some manner and one is different. Select the
Concept: Multiplication:[N:N(N-1)] (Multiplication of number-pair that is different.
Number and its preceded Number) Pkj la[;k&lewg fn, x, gSa] ftuesa ls rhu fdlh u fdlh :© esa ,d
45. In each of the number-pairs, the second number is leku gSa vkSj ,d vaxlr gSA ml la[;k lewg dk p;u dhft, tks
obtained by performing a certain mathematical operation vlaxr gSA
on the first number. Three of the following pairs follow (a) 7 : 147(b) 8 : 192
the same pattern and thus form a group. Select the © 12 : 432 (d) 13 : 506
number-pair that does NOT belong to that group. Sol. (d)
çR;sd la[;k&;qXe esa] igyh la[;k ij ,d fuf’pr xf.krh; lafØ;k Y
djds nwljh la[;k çkIr dh tkrh gSA fuEufyf[kr esa ls rhu tksM+s ,d Pattern : - [X : Y, = Remainder(0)]
X
gh iSVuZ dk ikyu djrs gSa vkSj bl çdkj ,d lewg cukrs gSaA ml
147
la[;k&;qXe dk p;u djsa tks ml lewg ls lacafèkr ugha gSA (a) 7 : 147, = 21 (R0)
7
(a) 21 : 420 (b) 25 : 600
© 17 : 272 (d) 15 : 208 192
(b) 8 : 192, = 24 (R0)
Sol. (d) 8
Pattern : - [N : N(N-1)] 432
© 12 : 432, = 36 (R0)
(a) 21:420  21(21-1)  21  20 = 420 12
(b) 25:600  25(25-1)  25  24 = 600 (d) 13 : 506,
506
= 38
12
(R12)  (R0)
© 17:272  17(17-1)  17  16 = 272 13 13
Except d, all other follow same pattern so d is odd one among
(d) 15 : 208  15(15-1)  15  14 = 210  208
all options.
Except d, all other follow same pattern so d is odd one among
Concept: Division : Divisibility of 3
all options.
49. Three of the followmg four numbers are alike in a certain
Concept: Multiplication:[FACTOR MULTIPLICATION]
way and one is different. Pick the number that is different
(Multipliation of consecutive terms.)
from the rest.
46. Three of the following four numbers are alike in a certain
fuEufyf[kr pkj la[;kvksa esa ls rhu ,d fuf’pr rjhds ls leku gSa
way and one is different. Pick the number that is different
from the rest. [FACTOR OF MULTIPLICATION]
vkSj ,d vyx gSA og la[;k pqusa tks ckdh ls vyx gksA
fuEufyf[kr pkj la[;kvksa esa ls rhu ,d fuf’pr rjhds ls leku gSa (a) 39 (b) 573 (c) 43 (d) 63
Sol. (c)
vkSj ,d vyx gSA og la[;k pqusa tks ckdh ls vyx gksA
(a) 12 (b) 14 (c) 56 (d) 30 N
Pattern : - [Dividation, = R(0)]
Sol. (b) 3
(a) 12=3×4 (b) 14=2×7 39
(a) 39, = 13 (R0)
© 56= 7×8 (d) 30= 5×6 3
Concept: Multiplication: (A , k1A , k2A), where k1 and k2 573
are any numbers . ((A,B.C) is a set where B is k1 times and (b) 573, = 191 (R0)
3
C is k2 times the Number . )
43
47. Select the number-triad in which the three numbers share © 43, = 14 (R1)  (R0)
a different relationship from that shared by the three 3
numbers in the rest of the number-triads. 63
(d) 63, = 21 (R0)
Ml la[;k&f=dks.k dk p;u djsa ftlesa rhu la[;k,a ‘ks”k 3
la[;k&f=dksa esa rhu la[;kvksa }kjk lk>k fd, x, lacèa k ls fHkUu lacaèk Except c, all other follow same pattern so c is odd one among
lk>k djrh gSaA all options.
(a) (13, 39, 65) (b) (19, 57, 95) Concept: Division : Divisibility of 4
© (23, 69, 105) (d) (17,51, 85) 50. Our numbers have been given below. Out of which three
Sol. (c) are alike in some manner, while the fourth one is
Pattern : - [N : 3N : 5N] different. Choose out the odd one.
(a) (13, 39, 65), 13 : 3 × 13 : 5 × 13 gekjs uacj uhps fn, x, gSaA ftuesa ls rhu fdlh u fdlh :i esa ,d
(b) (19, 57, 95), 19 : 3 × 19 : 5 × 19 tSls gSa] tcfd pkSFkk vyx gSA fo"ke dks pquAsa
© (23, 69, 105), 23 : 3 × 23 : 5 × 23  115  105 (a) 72 (b) 56 (c) 82 (d) 52
(d) (17, 51, 58), 17 : 3 × 17 : 5 × 17 Sol. (c)
Except c, all other follow same pattern so c is odd one among N
Pattern : - [Dividtion, = R(0)]
all options. 4
Y 72
Concept: Division : [X : Y, = Remainder(0)] (in a (a) 72, = 18 (R0)
X 4
analogical relation one number is proper multiple of 56
other number) (b) 56, = 14 (R0)
4
82
(c) 82, = 20 (R2)  (R0)
4

252
Best App for Govt. Jobs : Neonclasses (Download Now)

52 135
(d) 52, = 13 (R0) © 135, = 15 (R0)
4 9
Except c, all other follow same pattern so c is odd one among 105
all options. (d) 105, = 11 (R6)  (R0)
9
Concept: vDivision : Divisibility of 7 Except d, all other follow same pattern so d is odd one among
51. Three of the following four numbers are alike in a certain all options
way and one is different. Pick the number that is different Concept: Division : Divisibility of 11
from the rest. 54. Three of the following four numbers are alike in a certain
fuEufyf[kr pkj la[;kvksa esa ls rhu ,d fuf’pr rjhds ls leku gSa way and one is different. Pick the number that is different
vkSj ,d vyx gSA og la[;k pqusa tks ckdh ls vyx gksA from the rest.
(a) 126 (b) 217 (c) 189 (d) 254 fuEufyf[kr pkj la[;kvksa esa ls rhu ,d fuf’pr rjhds ls leku gSa
Sol. (d) vkSj ,d vyx gSA og la[;k pqusa tks ckdh ls vyx gksA
N (a) 341 (b) 132 (c) 166 (d) 209
Pattern : - [Dividation, = R(0)]
7 Sol. (c)
126 N
(a) 126, = 18 (R0) Pattern : - [Dividation, = R(0)]
7 11
217 341
(b) 217, = 31 (R0) (a) 341, = 31 (R0)
7 11
189 132
© 189, = 27 (R0) (b) 132, = 12 (R0)
7 11
256 166
(d) 256, = 16 (R4)  (R0) © 166, = 15 (R1)  (R0)
7 11
Except d, all other follow same pattern so d is odd one among 209
all options. (d) 209, = 19 (R0)
11
Concept: Division : Divisibility of 8 Except c, all other follow same pattern so c is odd one among
52. Three of the following four numbers are alike in a certain all options.
way and one is different. Find the odd one out. [N /8] Concept: Division : Divisibility of 19
fuEufyf[kr pkj la[;kvksa esa ls rhu ,d fuf’pr rjhds ls leku gSa 55. Four numbers have been given out of which three are
vkSj ,d fHkUu gSA fo”ke pqusAa alike in some manner, while one is different. Choose the
(a) 104 (b) 28 (c) 32 (d) 48 odd one.
Sol. (b) pkj la[;k,¡ nh xbZ gSa ftuesa ls rhu fdlh u fdlh :i esa leku gSa]
N tcfd ,d vlaxr gSA vlaxr dk p;u djsAa
Pattern : - [Dividation, = R(0)]
8 (a) 209 (b) 195 (c) 171 (d) 133
104 Sol. (b)
(a) 104, = 13 (R0)
8 N
Pattern : - [Dividation, = R(0)]
28 19
(b) 28, = 3 (R4)  (R0)
8 209
(a) 209, = 11 (R0)
32 19
© 32, = 4 (R0)
8 195
(b) 195, = 10 (R5)  (R0)
48 19
(d) 48, = 6 (R0)
8 171
(c) 171, = 9 (R0)
Except b, all other follow same pattern so b is odd one among 19
all options. 133
Concept: Division : Divisibility of 9 (d) 133, = 7 (R0)
19
53. Four number have been given out of which three are alike Except b, all other follow same pattern so b is odd one among
in some manner, while one is different. Choose the odd all options.
one. Concept: Difference : The difference is same in THREE
Pkj la[;k,¡ nh xbZ gSa ftuesa ls rhu fdlh u fdlh :© esa leku gSa] options(The difference between numbers is same in 3
tcfd ,d fHkUu gSA fo”ke la[;k dk p;u dhft,A options and different in one option)
(a) 99 (b) 117 (c) 135 (d) 105 56. Select the number-pair in which the two numbers share a
Sol. (d) different relationship from that shared by the two
N numbers in the rest of the number-pairs.
Pattern : - [Dividation, = R(0)]
9 Ml la[;k&;qXe dk p;u djsa ftlesa nks la[;k,¡ ‘ks”k la[;k&;qXeksa
99 esa nks la[;kvksa }kjk lk>k fd, x, lacaèk ls fHkUu lacaèk lk>k djrh
(a) 99, = 11 (R0) gSaA
9
117 (a) (178 , 308) (b) (215, 338)
(b) 117, = 13 (R0) © (169, 292) (d) (11 , 134)
9

253
Download Free PDFs & e-Books from Neon Classes App

Sol. (a) (b) 5121 : 12  5 + 1 + 2 + 1  = 9  12


Pattern : - [Same Difference]
© 8101 : 10  8 + 1 + 0 + 1  = 10
(a) (178, 308)  308 – 178 = 130
(d) 7456 : 22  7 + 4 + 5 + 6  = 22
(b) (215, 338)  338 – 215 = 123 Except b, all other follow same pattern so b is odd one among
© (169, 292)  292 – 169 = 123 all options.
(d) (11, 134)  134 – 11 = 123 Concept: Digit: [Digit Increase] Digit follows a increasing
Except a, all other follow same pattern so a is odd one among pattern
all options. 60. Three of the following four numbers are alike in a certain
Concept: Difference : In a set when the difference between way and one is different. Pick the number that is different
adjacent numbers follows same pattern from the rest.
57. Four number triads have been given, out of which three fuEufyf[kr pkj la[;kvksa esa ls rhu ,d fuf’pr rjhds ls leku gSa
are alike in some manner and one is different. Select the vkSj ,d vyx gSA og la[;k pqusa tks ckdh ls vyx gksA
number triad that is different. (a) 123 (b) 789 (c) 457 (d) 567
Pkj la[;k f=d fn, x, gSa] ftuesa ls rhu fdlh u fdlh :© esa ,d Sol. (c)
tSls gSa vkSj ,d fHkUu gSA ml la[;k =; dk p;u djsa tks fHkUu gSA Pattern : - [Digit Increase]
(a) (46, 65, 82) (b) (54, 73, 90) The pattern followed here is : [Digit Increase]
© (21, 40, 57) (d) (54, 91, 128) (a) 123  1 < 2 < 3
Sol. (d) (b) 789  7 < 8 < 9
Pattern : - [Same Difference]
© 457  4 < 5 < 7  6
(a) (46, 65, 82)  65 – 46 = 19, 82 – 65 = 17, [19, 17]
(d) 567  5 < 6 < 7
(b) (54, 73, 90)  73 – 54 = 19, 90 – 73 = 17, [19, 17] Type : 2
© (21, 40, 57)  40 – 21 = 19, 57 – 40 = 17, [19, 17] Pattern : - [yxkrkj c<+rs gq,s Digit]
(d) (54, 91, 128)  91 – 54 = 37, 128 – 91 = 37, [37, 37] (a) 1, 1 + 1  2, 2 + 1  3
Except d, all other follow same pattern so d is odd one among (b) 7, 7 + 1  8, 8 + 1  9
all options.
© 4, 4 + 1  5, 5 + 1  6  7
Concept: Difference : The Number are related analogically
and follows logic of multiplication that can also be (d) 5, 5 + 1  6, 6 + 1  7
represented in Ratio form Except c, all other follow same pattern so c is odd one among
58. Three of the following four number-pairs are alike in a all options.
certain way and one is different. Pick the odd one out. Concept: Digit: [(Add Digit)2 = N] (Square of the sum of
fuEufyf[kr pkj la[;k&;qXeksa esa ls rhu ,d fuf’pr rjhds ls leku digits is related to the number)
gSa vkSj ,d fHkUu gSA fo”ke dks pqusAa 61. Four number-pairs have been given, out of which three
(a) 56 : 49(b) 104 : 91 are alike in some manner and one is different. Select the
© 32 : 28 (d) 64 : 54 number-pair that is different.
Sol. (d) Pkj la[;k&lewg fn, x, gSa] ftuesa ls rhu fdlh u fdlh :© esa ,d
Pattern : - [Ratio – 8 : 7] leku gSa vkSj ,d vaxlr gSA ml la[;k lewg dk p;u dhft, tks
56 49
vlaxr gSA
(a) 56 : 49  : 8:7 (a) (812, 121) (b) (546, 225)
7 7
© (547, 258) (d) (723, 144)
104 91
(b)104 : 91  : 8:7 Sol. (c)
13 13 Pattern : - [(Add Digit)2 = N]
32 28 (a) (812 : 121), (8+1+2)2  (11)2 = 121
© 32 : 28  : 8:7
4 4 (b) (546 : 225), (5 + 4 + 6)2  (15)2 = 225
64 54 © (547 : 258), (5 + 4 + 7)2  (16)2 = 256  258
(d) 64 : 54  :  8 : 6.7  7
8 8
(d) (723 : 144), (7 + 2 + 3)2  (12)2 = 144
Except d, all other follow same pattern so d is odd one among
Except c, all other follow same pattern so c is odd one among
all options.
all options.
Concept: Digit: [Add Digit = N] (Case : digit sum of the
Concept: Digit: [Digit : Reverse Digit]( when digits are
digits of a number are related to the number)
reversed in order
59. Four number-pairs have been given, out of which three
62. The second number in the given number pairs is obtained
are alike in some manner and one is different. Select the
by performing certain mathematical operation(s) on the
number-pair that is different from the rest.
first number. The same operation(s) are followed in all
La[;kvksa ds pkj ;qXe fn, x, gSa] ftuesa ls rhu fdlh u fdlh rjg ls the number pairs except one. Find that odd number pair.
leku gSa vkSj ,d vleku gSA ml vleku la[;k&;qXe dk p;u djsAa Nh xbZ la[;k ;qXeksa esa nwljh la[;k igyh la[;k ij ,d fuf’pr
(a) 4209 : 15 (b) 5121 : 12 xf.krh; lafØ;k,¡ djds çkIr dh xbZ gSA ,d la[;k ;qXe dks NksM+dj
© 8101 : 10 (d) 7456 : 22
lHkh la[;k ;qXeksa esa leku xf.krh; lafØ;kvksa dk vuqlj.k fd;k x;k
Sol. (b)
gSA ml vlaxr la[;k ;qXe dks Kkr djsAa
Pattern : - [Add Digit = N]
(a) 4328 : 8324 (b) 2437 : 7342
(a) 4209 : 15  4 + 2 + 0 + 9  = 15

254
Best App for Govt. Jobs : Neonclasses (Download Now)

© 3569 : 9653 (d) 1786 : 6871 Concept: Digit: [Digit : Add Digit  k], where k can be any
Sol. (a) number
Pattern : - [Digit : Reverse Digit] 66. The second number in the given number pairs is obtained
(a) 4328 : 8234  8324 by performing certain mathematical operation(s) on the
(b) 2437 : 7342 first number. The same operation(s) are followed in all
© 3569 : 9653 the number pairs except one. Find that odd number pair.
(d) 1786 : 6871 Nh xbZ la[;k ;qXeksa esa nwljh la[;k igyh la[;k ij ,d fuf’pr
Except a, all other follow same pattern so a is odd one among xf.krh; lafØ;k,¡ djds çkIr dh xbZ gSA ,d la[;k ;qXe dks NksM+dj
all options. lHkh la[;k ;qXeksa esa leku xf.krh; lafØ;kvksa dk vuqlj.k fd;k x;k
63. The second number in the given number pairs is obtained gSA ml vlaxr la[;k ;qXe dks Kkr djsAa
by performing certain mathematical operation(s) on the (a) 678:42 (b) 546:30
first number. The same operation(s) are followed in all © 283:28 (d) 125:16
the number pairs except one. Find that odd number pair. Sol. (c)
nh xbZ la[;k ;qXeksa esa nwljh la[;k igyh la[;k ij ,d fuf’pr Pattern : - [Digit : Add Digit  2]
xf.krh; lafØ;k,¡ djds çkIr dh xbZ gSA ,d la[;k ;qXe dks NksM+dj (a) 678 : (6 + 7 + 8 )  2  21  2 = 42
lHkh la[;k ;qXeksa esa leku xf.krh; lafØ;kvksa dk vuqlj.k fd;k x;k
(b) 546 : (5 + 4 + 6 )  2  15  2 = 30
gSA ml vlaxr la[;k ;qXe dks Kkr djsAa
(a) 678 : 7(b) 282 : 6 © 283 : (2 + 8 + 3 )  2132= 26  28
(c) 366 : 5 (d) 546 : 5 (d) 125 : (1 + 2 + 5 )  2  8  2 = 16
Sol. (b) Except c, all other follow same pattern so c is odd one among
Add Digit all options.
Pattern : - [Digit : ] Concept: Digit: same digit is repeated in rotation form
3
67. Four pairs of numbers have been given, out of which three
6  7  8  21
(a) 678 :  =7 are alike in some manner, while one is different. Choose
3 3 out the odd one.
 2  8  2 12 Pkj tksM+h la[;k,¡ nh xbZ gSa] ftuesa ls rhu fdlh u fdlh :© esa ,d
(b) 282 :  =46
3 3 tSlh gSa] tcfd ,d fHkUu gSA fo”ke dks pquAsa
3  6  6  15 (a) 321 : 213 : 123 (b) 432 : 324 : 231
(c) 366 :  =5 © 798 : 897 : 798 (d) 564 : 645 : 456
3 3
Sol. (b)
5  4  6 15
(d) 546 :  =5 Pattern : - [Same Digit]
3 3
(a) 321 : 213 : 123 [1, 2, 3]
Except b, all other follow same pattern so b is odd one among
all options. (b) 432 : 324 : 231 [1, 2, 3, 4]  [1, 2, 3]
64. Choose the odd number pair. © 798 : 897 : 798 [7, 8, 9]
Fo”ke la[;k ;qXe pquAsa (d) 564 : 645 : 456 [4, 5, 6]
(a) 123 : 49 (b) 453 : 144 Except b, all other follow same pattern so b is odd one among
© 143 : 81 (d) 751 : 196 all options.
Sol. (b) Concept: Digit: [Digit Increase : Add Digit Increase] (when
Pattern : - [Digit : (Add Digit + 1)2] a series in followed in all digit combined . Continuation of
(a) 123 : (1 + 2 + 3 + 1)2  72 = 49 a series)
(b) 453 : (4 + 5 + 3 + 1)2  132 = 169  144 68. Three of the following four number-pairs are alike in a
certain way and one is different. Find the odd one out.
© 143 : (1 + 4 + 3 + 1)2  92 = 81
fuEufyf[kr pkj esa ls rhu la[;k&;qXe ,d fuf’pr rjhds ls leku gSa
(d) 751 : (7 + 5 + 1 + 1)2  142 = 196 vkSj ,d fHkUu gSA fo”ke pqusAa
Except b, all other follow same pattern so b is odd one among (a) 123 : 456 (b) 456: 798
all options. © 234: 567 (d) 345: 678
65. Select the odd pair of numbers. Sol. (b)
la[;kvksa ds fo"ke ;qXe dk p;u dhft,A Pattern : - [Digit Increase : Add Digit Increase]
(a) 32 : 19(b) 21 : 51 (a) 123 : 456, 1- 2 – 3 : 4 – 5 – 6
(c) 58 : 26 (d) 43 : 95 (b) 456 : 798, 4 – 5 – 6 : 7 – 9 – 8  [7 – 8 – 9]
Sol. (c) © 234 : 567, 2 – 3 – 4 : 5 – 6 – 7
Pattern : - [(A + B) : 2 (A + B) = (X + Y)][A, B, X, Y – DIGIT] (d) 345 : 678, 3 – 4 – 5 : 6 – 7 – 8
(a) 32 : 19, 2(3 + 2) : (1 + 9)  2  5 : 10 = 10 : 10 Except b, all other follow same pattern so b is odd one among
(b) 21 : 51, 2(2 + 1) : (5 + 1)  2  3 : 6 = 6 : 6 all options.
(c) 58 : 26, 2(5 + 8) : (2 + 6)  2  13 : 8 = 26 : 8  8 : 8 Concept: Digit: [AB = A2 – B2, A, B = Digit]( A Two digit
Number AB , where the difference between the digit
(d) 43 : 95, 2(4 + 3) : (9 + 5)  2  7 : 14 = 14 : 14
squares is related to the number .)
Except c, all other follow same pattern so c is odd one among
69. Four number-pairs have been given, out of which three
all options.
are alike in some maimer and one is different. Select the
number-pan that is different from the rest.

255
Download Free PDFs & e-Books from Neon Classes App

Pkj la[;k&;qXe fn, x, gSa] ftuesa ls rhu fdlh eSej esa leku gSa vkSj (b) 16 : 6, 42 : 2 × 4 – 2  8 – 2 = 6
,d fHkUu gSA ml uacj&iSu dks pqusa tks ckfd;ksa ls vyx gksA © 25 : 8, 52 : 2 × 5 – 2  10 – 2 = 8
(a) 57-38 (b) 42 -12
(d) 49 : 12, 72 : 2 × 7 – 2  14 – 2 = 12
© 39 -72 (d) 28-60
Except a, all other follow same pattern so a is odd one among
Sol. (a)
all options.
Pattern : - [AB = A2 – B2, A, B = Digit]
Concept: Multiple Operations : power of a number is
(a) 57 – 38, 52 – 72  25 – 49 = 24  38
pattern in options [Xn]
(b) 42 – 12, 42 – 22  16 – 4 = 12 73. Three of the following four numbers are alike in a certain
© 39 – 72, 32 – 92  9 – 81 = 72 way and one is different. Identify the different one.
(d) 28 – 60, 22 – 82  4 – 64 = 60 fuEufyf[kr pkj la[;kvksa esa ls rhu ,d fuf'pr rjhds ls leku gSa
Except a, all other follow same pattern so a is odd one among vkSj ,d fHkUu gSA fHkUu dks igpkusAa
all options. (a) 8 (b) 34 (c) 64 (d) 16
Concept: Digit: [B + C = A, A, B, C – Digit]( A three digit Sol. (b)
Number where Sum of any Two digits is equal to the third Pattern : - [2n]
digit) (a) 8, 2 × 2 × 2  23
70. Three of the following four numbers are alike in a certain (b) 34, 2 × 17  2 × 17
way and one is differen. Pick the number that is different
(c) 64, 2 × 2 × 2 × 2 × 2 × 2  26
from the rest.
fuEufyf[kr pkj la[;kvksa esa ls rhu ,d fuf'pr rjhds ls leku gSa (d) 16, 2 × 2 × 2 × 2  24
vkSj ,d fHkUu gSA og la[;k pqusa tks ckdh ls vyx gksA Except b, all other follow same pattern so b is odd one among
all options.
(a) 953 (b) 523 (c) 312 (d) 734
Concept: Multiple Operations : Both numbers are co-prime
Sol. (a)
of each other
Pattern : - [B + C = A, A, B, C - Digit]
74. Three of the following four number-pairs are alike in a
(a) 953, 5 + 3 = 8  9
certain way and one is different. Identify the difierent one.
(b) 523, 2 + 3 = 5
fuEufyf[kr pkj esa ls rhu la[;k&;qXe ,d fuf'pr rjhds ls leku gSa
(c) 312, 1 + 2 = 3
(d) 734, 3 + 4 = 7
vkSj ,d vyx gSA fHkUu dks igpkusAa
Except a, all other follow same pattern so a is odd one among (a) 14 : 3 (b) 7 : 6 (c) 12 : 4 (d) 21 : 2
all options Sol. (c)
Concept: Digit: [(A + B) : 2 (A + B) = (X + Y)][A, B, X, Y – Pattern : - [CO – Prime Number]
DIGIT] (XY : AB where 2(x+y) = A+B) (a) 14 : 3
71. Select the odd pair of numbers. (b) 7 : 6
la[;kvksa ds fo"ke ;qXe dk p;u dhft,A 12
(c) 12 : 4  = 3 (Not Co–Prime)
(a) 32 : 19(b) 21 : 51 4
(c) 58 : 26 (d) 43 : 95 (d) 21 : 2
Sol. (c) Theory : - Co–Prime Numbers are those numbers that have
Pattern : - [(A + B) : 2 (A + B) = (X + Y)][A, B, X, Y – DIGIT] only one common factor (HCF as1) namely 1, that means a pair
(a) 32 : 19, 2(3 + 2) : (1 + 9)  2  5 : 10 = 10 : 10 of numbers are said to be CO – Prime when they have their
highest common factor as one.
(b) 21:51,2(2+1) : (5+1)  23:6 = 6 : 6
Except c, all other follow same pattern so c is odd one among
(c) 58 : 26, 2(5 + 8) : (2 + 6)  2  13 : 8 = 26 : 8  8 : 8 all options.
(d) 43 : 95, 2(4 + 3) : (9 + 5)  2  7 : 14 = 14 : 14 Concept: Multiple Operations : pair of numbers follows
Except c, all other follow same pattern so c is odd one among continuation of same operation , x(x+1): (x+2)(x+3)
all options. 75. Four pairs of numbers have been given, out of which three
Concept: Multiple Operations : N2 : 2N - 2] Both first and are alike in some manner, while one is different. Choose
second number in analogical relation are obtained by out the odd one.
performing operation on same number pkj tksM+h la[;k,¡ nh xbZ gSa] ftuesa ls rhu fdlh u fdlh :i esa ,d
72. The second number in the given number-pairs is obtained tSlh gSa] tcfd ,d fHkUu gSA fo"ke dks pquAsa
by performing certain mathematical operation(s) on the (a) 42, 72 (b) 56, 63
first number. The same operation(s) is/are followed in all (c) 20, 42 (d) 132, 182
the number-pairs, except one. Find that odd number-pair. Sol. (b)
Nh xbZ la[;k&;qXeksa esa nwljh la[;k igyh la[;k ij dqN xf.krh; The pattern followed here is : [Multiplication of consecutive
lafØ;k,¡ djds çkIr dh tkrh gSA ,d dks NksM+dj lHkh la[;k&tksM+ksa numbers]
esa leku lafØ;kvksa dk ikyu fd;k tkrk gS@gSaA og fo”ke (a) 42, 72 [6*7,8*9 ]
la[;k&;qXe Kkr dhft,A (b) 56, 63 [7*8, 7*9 ]
(a) 4:18 (b) 16:6 (c) 25:8 (d) 49:12 (c) 20, 42 [4*5, 6*7]
Sol. (a) (d) 132, 182 [11*12, 13*14]
Pattern : - [N2 : 2N – 2] Except b, all other follow same pattern so b is odd one among
(a) 4 : 18, 22 : 2 × 2 – 2  4 – 2 = 2  18 all options.

256
Best App for Govt. Jobs : Neonclasses (Download Now)

Concept: Multiple Operations : [N2 : (N+1)3] (when square Pattern : - [X : Y , Y2 + 3Y + 1]


of the number is related to cube of succeeding number) (a) 182 : 12, 122 + 3  12 + 1  144 + 36 + 1 = 181  182
76. Select the number-pair in which the two numbers share a
(b) 305 : 16, 162 + 3  16 + 1  256 + 48 + 1 = 305
different relationship from that shared by the two
numbers in the rest of the number-pairs. (c) 239 : 14, 142 + 3  14 + 1  196 + 42 + 1 = 239
ml la[;k&;qXe dk p;u djsa ftlesa nks la[;k,¡ 'ks"k la[;k&;qXeksa esa (d) 461 : 20, 202 + 3  20 + 1  400 + 60 + 1 = 461
nks la[;kvksa }kjk lk>k fd, x, lacaèk ls fHkUu lacaèk lk>k djrh gSaA Except a, all other follow same pattern so a is odd one among
(a) (36, 353) (b) (49, 512) all options.
2
(c) (25, 216) (d) (64, 729)  N 1
Sol. (a) Concept: Multiple Operations: [N :   ] when
 2 
Pattern : - [N2 : (N+1)3]
operation on number is performed and then its squared
(a) 36 :353, 62 : (6+1)3  73 = 343  353 79. The second number in the given number pairs is obtained
(b) 49 : 512, 72 : (7+1)3  83 = 512 by performing certain mathematical operation(s) on the
(c) 25 : 216, 52 : (5+1)3  63 = 216 first number. The same operation(s) are followed in all
(d) 64 : 729, 82 : (8+1)3  93 = 729 the number pairs except one. Find that odd number pair.
Method : 2
nh xbZ la[;k ;qXeksa esa nwljh la[;k igyh la[;k ij ,d fuf’pr
xf.krh; lafØ;k,¡ djds çkIr dh xbZ gSA ,d la[;k ;qXe dks NksM+dj
 
3
Pattern : - [N : N 1 ] lHkh la[;k ;qXeksa esa leku xf.krh; lafØ;kvksa dk vuqlj.k fd;k x;k
gSA ml vlaxr la[;k ;qXe dks Kkr djsAa
   (6+1)  7 = 343  353
3
(a) 36 : 36  1 3 3
(a) 29:225 (b) 17:81
(c) 45:576 (d) 31:256
(b) 49 :  49  1  (7+1)  8 = 512
3
3 3
Sol. (c)
2

(c) 25 :  25  1  (5+1)  6 = 216


3
 N 1
3 3 Pattern : - [N :   ]
 2 
(d) 64 :  64  1  (8+1)  9 = 729
3
3 3 2 2
 29  1   30 
(a) 29:225,       152 = 225
Except a, all other follow same pattern so a is odd one among  2   2
all options. 2 2
 17  1   18 
Concept: Multiple Operations: [N2 : (N+1)2+1] (when a (b) 17 : 81, 
2     92 = 81
number’s square is related to succeeding number’s
   2
2 2
square.)  45  1   46 
77. Four number – pairs have been given, out of which threare (c) 45 : 576,       232 = 529  576
 2   2 
alike in some manner and one is different. Select the 2 2
number-pair that is different.  31  1   32 
(d) 31 : 256,       162 = 256
pkj la[;k&tksM+s fn, x, gSa] ftuesa ls rhu fdlh u fdlh :i esa  2   2
leku gSa vkSj ,d fHkUu gSA ml la[;k&;qXe dk p;u djsa tks fHkUu Except c, all other follow same pattern so c is odd one among
gSA all options.
(a) 289-325 (b) 169-197 80. The second number in the given number pairs is obtained
(c) 225-241 (d) 121-145 by performing certain mathematical operation(s) on the
Sol. (c) first number. The same operation(s) are followed in all
Pattern : - [N2 : (N+1)2+1] the number pairs EXCEPT one. Find that odd number pair.
(a) 289 : 325, 172 : (17+1)2+1  182 +1  324 + 1 = 325 nh xbZ la[;k ;qXeksa esa nwljh la[;k igyh la[;k ij ,d fuf’pr
(b) 169 : 197, 132 : (13+1)2+1  142 +1  196 + 1 = 197 xf.krh; lafØ;k,¡ djds çkIr dh xbZ gSA ,d la[;k ;qXe dks NksM+dj
lHkh la[;k ;qXeksa esa leku xf.krh; lafØ;kvksa dk vuqlj.k fd;k x;k
(c) 225 : 241, 152 : (15+1)2+1  162 +1  256 + 1 = 257 
gSA ml vlaxr la[;k ;qXe dks Kkr djsAa
241
(a) 48 : 168 (b) 52 : 182
(d) 121 : 145, 112 : (11+1)2+1  122 +1  144 + 1 = 145
(c) 26 : 91 (d) 34 : 118
Except c, all other follow same pattern so c is odd one among Sol. (d)
all options.
N
Concept: Multiple Operations: [X : Y , Y2 + 3Y + 1] (: square Pattern : - [N :  7 ]
2
of succeeding number is added to number itself.)
78. Four number-pairs have been given, out of which three 48
(a) 48 :  7  24  7 = 168
are alike in some manner and one is different. Select the 2
number-pair that is different. 52
(b) 52 :  7 = 26  7 = 182
pkj la[;k&lewg fn, x, gSa] ftuesa ls rhu fdlh u fdlh :i esa ,d 2
leku gSa vkSj ,d vaxlr gSA ml la[;k lewg dk p;u dhft, tks 26
vlaxr gSA (c) 26 :  7  13  7 = 91
2
(a) 182 : 12 (b) 305 : 16
34
(c) 239 : 14 (d) 461 : 20 (d) 34 :  7  17  7 = 119  118
Sol. (a) 2

257
Download Free PDFs & e-Books from Neon Classes App

Except d, all other follow same pattern so d is odd one among Logic: - Difference prime no
all options.
35 54 77 106 137 ?
81. Four pairs of numbers have been given, out of which three
are alike in some manner, while one is different. Choose 19 23 29 31 37
out the odd one. 137+37 = 174
pkj tksM+h la[;k,¡ nh xbZ gSa] ftuesa ls rhu fdlh u fdlh :i esa ,d Correct option (d).
tSlh gSa] tcfd ,d fHkUu gSA fo"ke dks pquAsa  Concept -
(a) 51 - 25 (b) 50 – 7
(c) 16 - 3 (d) 64 - 32 Prime Number :Product series is prime number series (
Sol. (d) Prime Number: Difference prime no )(Alternative
XY multiplication and subtraction)
Pattern : - [X : Y, ] 2. Which of the following numbers will replace the question
19
mark (?) in the given series?
51  25 76
(a) 51 : 25,  =4 fuEufyf[kr esa ls dkSu lh la[;k nh xbZ J`a[kyk esa ç'u fpà ¼\½ dks
19 19 çfrLFkkfir djsxh\
50  7 57 13, 26, 23, 115, 108, ?
(b) 50 : 7,  =3
19 19 (a) 1118 (b) 1818 (c)1881 (d) 1188
16  3 19 Sol. (d)
(c) 16 : 3,  =1 Logic: - Difference prime no (Alternative multiplication and
19 19
subtraction)
64  32 96
(d) 64 : 32,  = 5.15  Not proper divides 13 26 23 115 108 ?
19 19
Except d, all other follow same pattern so d is odd one among ×2 -3 ×5 -7 ×11
all options. 108×11 = 1188
82. Four number-pairs have been given, out of which three Correct option (d).
are alike in some mannerand one is different. Select the
 Concept -
number-pair that is different.
la[;kvksa ds pkj ;qXe fn, x, gSa] ftuesa ls rhu fdlh u fdlh rjg Prime Number : The difference of the series are
ls leku gSa vkSj ,d vleku gSA ml vleku la[;k&;qXe dk p;u alternate prime number series
djsAa 3. Select the number from among the given options that can
4 16 replace the question mark (?) in the following series.
(a) : 3 (b) :6 fn, x, fodYiksa esa ls og la[;k pqfu, tks fuEufyf[kr J`[a kyk esa
5 20
25 30
ç'uokpd fpà ¼\½ dks çfrLFkkfir dj ldsA
(c) :8 (d) :9 27, 30, 37, 50, ?, 98
41 51
(a) 82 (b) 62 (c) 69 (d) 78
Sol. (c)
Sol. (c)
X
Pattern : - [ :  X  Y  Z ] Logic: - Prime no.
Y
27 30 37 50 ? 98
4
(a) :  4  5  9 =3
+3 +7 +13 +19 +29
5
16 50+19 = 69
Correct option (c).
(b) : 16  20  36 = 6
20  Concept -
25
(c) : 25  41  66  8.12  8 Cube
41

(d)
30
51
: 30  51  81 = 9 Cube when the
Number:

Except c, all other follow same pattern so c is odd one among


all options difference of the
Number Series
 Concept - series are perfect
1.
Prime Numbers
Prime Number: Difference of consecutive primes
Select the number from among the given options that can
cubes ( when Cube [Difference](

replace the question mark (?) in the following series.


fn, fodYiksa esa ls og la[;k pqfu, tks fuEufyf[kr Js.kh esa the Numbers are in
iz’uokpd fpUg (?) ds LFkku ij vk ldrh gSA
35, 54, 77, 106, 137, ?
(a) 179 (b) 151 (c) 170 (d) 174
cubes of near to cube
Sol. (d)

258
Best App for Govt. Jobs : Neonclasses (Download Now)

(d) The first bracketed number is correct and the second


of numbers or the bracketed number is incorrect.
igyh czSdsV okyh la[;k lgh gS vkSj nwljh czSdsV okyh la[;k xyr

difference between Sol. (b)


gSA

Logic: - Cube+add [a3+3=x]

the term is cube) 4


3
11
3
1 +3 2 +3 3 +3 4 +3
30
3
(67) 128
3 3
219
3
5 +3 6 +3 7 +3 8 +3
(346) 515
3 3

4. Select the number from among the given options that can =346
replace with the question mark (?) in the following series. Both the bracketed numbers are correct.
fn, x, fodYiksa esa ls og la[;k pqfu tks fuEufyf[kr Js.kh esa
 Concept -
iz’uokpd fpUg (?) ds LFkku ij vk ldrh gSA
55, 63, ?, 343, 855, 1855 Cube Number: Cube – Subtraction [a3–K=x] Where k is
(a) 135 (b) 117 (c) 223 (d) 127 can any number (operation performed on
Sol. (d) perfect cube numbers)
Logic: - Cube [Difference] 6. Select the option that will fill in the blank and complete
the given series.
55 63 ? 343 855 1855
ml fodYi dk p;u djsa tks fjä LFkku dks Hkjsxk vkSj nh xbZ J`[a kyk
+8 +64 +216 +512 +1000 dks iwjk djsxkA
3 3 3 3 3
2 4 6 8 10 0, 7, 26, 63, 124, 215, 342, 511, 728,
63+64 = 127 Correct option (d). (a) 996 (b) 999 (c) 1001 (d) 1000
Sol. (b)
 Concept -
Logic: - Cube–Subtraction [a3–1=x]
Cube Number: series is perfect cube series , but the 0 7 26 63 124 215 342 511 728 ?

numbers that are cubed are double of previous 3


0 –1=0
3
3
3 –1=26
3
3
5 –1=124
3
7 –1=342
3
9 –1=728
3 3 3
2 –1=7 4 –1=63 6 –1=215 8 –1=511 10 –1=999
term/follows some other logic  103–1 1001 –1 =999
Eg. Select the number from among the given options that can
Correct option (b).
replace the question mark (?) in the following series.
fn, x, fodYiksa esa ls og la[;k pqusa tks fuEufyf[kr J`a[kyk esa  Concept -
iz’uokpd (?) ds LFkku ij vk ldrh gSA Cube Number: [a=n3+k] where k can any random number
1, 8, 64, 512, ? (some number is added to perfect cube number series)
(a) 17282 (b) 2744 7. Which number will come next in the following series?
(c) 4096 (d) 2197 fuEufyf[kr J`[a kyk esa dkSu lh la[;k vkxs vk,xh\
Sol. (c) 5, 12, 31, 68, 129, ..........
Logic: - Cube [Difference] (a) 220 (b) 218 (c) 131 (d) 190
1 8 64 512 ? Sol. (a)
1
3
2
3
4
3 3
8 16
3 Logic: - Cube+4 [a=n2+4]

×2 ×2 ×2 ×2 5 12 31 68 129 ?
3
16  16×16×16 = 4096 3 3
1 +4 2 +4 3 +4 4 +4 5 +4 6 +4
3 3 3 3

Correct option (c).  63+4 216+4 =220


 Concept - Correct option (a).

Cube Number: Cube + add [a3+K=x] Where k can be any  Concept -


number (Numbers are perfect cube added to some Square Numbers
number) Square Number: When the given series is perfect square
5. Select the option that is correct for the bracketed numbers number
with respect to their inclusion in the given series. Square Number: The difference is perfect square when
nh xbZ J`a[kyk esa muds 'kkfey fd, tkus ds lacèa k esa czSdsV okyh Numbers are.
la[;kvksa ds fy, lgh fodYi pqusAa 8. Select the number from among the given options that can
4,11, 30, (67), 128, 219, (346), 515 replace the question mark (?) in the following series.
(a) The first bracketed number is incorrect and the second fn, x, fodYiksa esa ls og la[;k pqfu, tks fuEufyf[kr J`[a kyk esa
bracketed number is correct. ç'uokpd fpà ¼\½ dks çfrLFkkfir dj ldsA
igyh czSdsV okyh la[;k xyr gS vkSj nwljh czSdsV okyh la[;k lgh 28, 32, 41, 57, ?
gSA (a) 68 (b) 82 (c) 74 (d) 90
(b) Both the bracketed numbers are correct. Sol. (b)
nksuksa dks"Bd la[;k,a lgh gSaA Logic: - Square [Difference square]
(c) Both the bracketed numbers are incorrect.
dks"Bd esa nh xbZ nksuksa la[;k,¡ xyr gSaA

259
Download Free PDFs & e-Books from Neon Classes App

Square Number: Difference is decreasing perfect square


28 32 41 57 ?
numbers
4 9 16 25 11. Which of the following numbers will replace the question
2 3 2 2
42
52 mark (?) in the given series?
57+25=82 fuEufyf[kr esa ls dkSu lh la[;k nh xbZ J`a[kyk esa ç'u fpà ¼\½ dks
Correct option (b). çfrLFkkfir djsxh\
1778, 1609, 1465, 1344, 1244, ?
 Concept - (a) 1144 (b) 1109 (c) 1178 (d) 1163
Square Number : Perfect square and square of Sol. (d)
consecutive numbers, also sign follow alternate pattern Logic: - Square [Difference]
9. Select the number from among the given options that can
1778 1609 1465 1344 1244 ?
replace the question mark (?) in the following series.
fn, x, fodYiksa esa ls ml la[;k dk p;u djsa tks fuEufyf[kr 169 144 121 100 81
J`[a kyk esa ç'u fpà ¼\½ dks çfrLFkkfir dj ldrh gSA 13 122
112 2
10 2
92
28, 27, 31, 22, 38, 13, 49, ? 1244–81=1163
(a) 42 (b) 0 (c) 2 (d) 22 Correct option (d).
Sol. (b)
Logic: - Square [Alternative symbols]  Concept -
Square Number: Square [Alternative prime no] Difference
28 27 31 22 38 13 49 ?
is square of consecutive prime numbers.
–1 +4 –9 +16 –25 +36 –49 12. Select the number that can replace the question mark (?)
1 2 2 2
32 42 52 62 72 in the following series.
49–49=0 ml la[;k dk p;u djsa tks fuEufyf[kr J`[a kyk esa ç'uokpd fpà
Correct option (b). ¼\½ dks çfrLFkkfir dj ldrh gSA
4, 25. 121, ? , 529, 961
 Concept - (a) 289 (b) 256 (c) 324 (d) 361
Square Number: (Even no2+k ) Where k, can any random Sol. (a)
number(Approx. square , Number are near perfect Logic: - Square [Alternative prime no]
square Numbers are squares of even Numbers .)
4 25 121 ? 529 961
10. In the following number series, two numbers have been
put within brackets. Select the most appropriate option 22 52 112 172 232 312
for these numbers in relation to their inclusion in the 17×17=289
series. The series given above is squares of the alternate prime
fuEufyf[kr la[;k J`a[kyk esa] nks la[;kvksa dks dks"Bd esa j[kk x;k gSA numbers.
bu uacjksa dks J`a[kyk esa 'kkfey djus ds lacaèk esa muds fy, lcls Correct option (a).
mi;qä fodYi dk p;u djsa
 Concept -
1, 5, 17, (39), 65, 101, (145), 197
(a) Both the bracketed numbers are correct. Square Number : Square [number × square]
nksuksa dks"Bd la[;k,a lgh gSaA multiplication with perfect square Numbers in a series.
(b) Both the bracketed numbers are incorrect. 13. Which of the following numbers will replace the question
nksuksa czSdsV okyh la[;k,a xyr gSaA mark (?) in the given series?
(c) The first bracketed number from the left is correct and the fuEufyf[kr esa ls dkSu lh la[;k nh xbZ J`a[kyk esa ç'u fpà ¼\½ dks
second bracketed number from the left is incorrect. çfrLFkkfir djsxh\
ckbZa vksj ls igyh czSdsV okyh la[;k lgh gS vkSj ckbZa vksj ls nwljh 3, 3,12,108,?
czSdsV okyh la[;k xyr gSA (a) 1528 (b) 1512
(d) The first bracketed number from the left is incorrect and (c) 1728 (d) 1684
the second bracketed number from the left is correct. Sol. (c)
ckbZa vksj ls igyh czSdsV okyh la[;k xyr gS vkSj ckbZa vksj ls nwljh Logic: - Square [number×square]
czSdsV okyh la[;k lgh gSA 3 3 12 108 ?
Sol. (d) 2 2 2 2
×1 ×2 ×3 ×4
Logic: - Square+1 [a2+1=x][even no2+1]
108×42108×16 = 1728
1 5 17 (39) 65 101 (145) 197 Correct option (c).
2 2 2 2 2 2 2 2
0 +1 2 +1 4 +1 6 +1 8 +1 10 +1 12 +1 14 +1  Concept -
62+1  37  39
Correct option (d). Division Based
 Concept - Division: Every term is divided by a fix number
14. Select the number from among the given options that can
Square Number: (Odd no2+k ) Where k, can any random
replace the mark (?) in the following series.
number (Approx. square , Number are near perfect
fn, x, fodYiksa esa ls question ml la[;k dk p;u djsa tks
square .Numbers are squares of Odd Numbers .)
fuEufyf[kr J`[a kyk esa ç'u fpà ¼\½ dks çfrLFkkfir dj ldrh gSA

260
Best App for Govt. Jobs : Neonclasses (Download Now)

864, 432, 216, 108,?


240 120 ? 180 360 900
(a) 36 (b) 54 (c) 12 (d) 72
Sol. (b)
Logic: - Dividation
120×1= 120
864 432 216 108 ? Correct option (a).
÷2 ÷2 ÷2 ÷2
108÷2 = 54
 Concept -
Correct option (b).
Multiplication: Multiplication (×k) where k, can any
 Concept -
random number (Every new term is some multiple of
Division :Next term in series is after dividing the previous previous one.)
term with some number 18. Select the number from among the given options that can
15. Which number will replace the question mark (?) in the replace the question mark (?) in the following series.
following series? fn, x, fodYiksa ea ls og la[;k pqusa tks fuEufyf[kr J`a[kyk esa
fuEufyf[kr J`[a kyk esa ç'uokpd fpUg ¼\½ ds LFkku ij dkSu lh iz’uokpd (?) ds LFkku ij vk ldrh gSA
la[;k vk,xh\ 7, 14, 28, 56, ?
16, 24, 36, ?, 81 (a) 150 (b) 72 (c) 90 (d) 112
(a) 54 (b) 52 (c) 61 (d) 58 Sol. (d)
Sol. (a) Logic: - Multiplication (×2)
Logic: - Multiplication
7 14 28 56 ?
16 24 36 ? 81
×2 ×2 ×2 ×2
56×2= 112
Correct option (d).
 Concept -
Correct option (a). Multiplication: Multiplication (Alternative odd number
multiplication) (Alternate series, Two series following
 Concept -
different logics.)
Multiplication Based 19. Select the correct option that will fill in the blank and
Multiplication: Multiplication (Prime no.) product series complete the series.
numbers are consecutive prime numbers lgh fodYi dk p;u djsa tks fjä LFkku dks Hkjsxk vkSj J`[a kyk dks
16. Study the given pattern carefully and select the number iwjk djsxkA
that can replace the question mark (?) in it. 1, 3, 3, 6, 5, 12, 7, 24, 9, 48, 11, _______
fn, x, iSVuZ dk /;kuiwoZd v/;uu djs]a vkSj ml la[;k dk p;u (a) 95 (b) 69 (c) 59 (d) 96
djsa tks iz’uokpd fpUg (?) ds LFkku ij vk ldrh gSA Sol. (d)
3, 6, 18, 90, 630, ? Logic: - Multiplication(Alternative odd number multiplication)
(a) 6930 (b) 6300 (c) 5670 (d) 5040
Sol. (a) 1 3 3 6 5 12 7 24 9 48 11 ?
Logic: - Multiplication (Prime no.) ×2 ×2 ×2 ×2 ×2
48×2= 96
3 6 18 90 630 ?
Correct option (d).
×2 ×3 ×5 ×7 ×11
630×11= 6930  Concept -
Correct option (a). Multiplication: Multiplication (Alternative Even number
 Concept - multiplication)(Alternate series, Two series following
different logics .)
Multiplication:Multiplication Product series Numbers are x
in AP Multiplication: Multiplication[ ] (Multiplication with
y
17. Select the number from among the given options that can
replace the question mark (?) in the following series. some fixed number)
fn, x, fodYiksa esa ls og la[;k pqfu, tks fuEufyf[kr J`[a kyk esa 20. Which of the following numbers will replace the question
ç'uokpd fpà ¼\½ dks çfrLFkkfir dj ldsA mark (?) in the given series?
240, 120, ?, 180, 360, 900
fuEufyf[kr esa ls dkSu lh la[;k nh xbZ J`a[kyk esa ç'u fpà ¼\½ dks
(a) 120 (b) 148 (c) 150 (d) 130 çfrLFkkfir djsxh\
Sol. (a) 600,1500, 3750, 9375, ?
Logic: - Multiplication (a) 23437.50 (b) 21427.50
(c) 23126.50 (d) 22125.25
Sol. (a)
5
Logic: - Multiplication [ ]
2

261
Download Free PDFs & e-Books from Neon Classes App

Correct option (c).


600 1500 3750 9375 ?
 Concept -
Addition: addition of fixed number or following an AP
24. Which of the following numbers will replace the question
mark (?) in the given series?
fuEufyf[kr esa ls dkSu lh la[;k nh xbZ J`a[kyk esa ç'u fpà ¼\½ dks
Correct option (a). çfrLFkkfir djsxh\
 Concept - 134,186,238,290,342,?
(a) 370 (b) 386 (c) 394 (d) 378
Multiplication: multiplication numbers are in AP , an Sol. (c)
decimal series Logic: - Addition (+52)
21. Select the number that can replace the question mark (?)
in the following series. 134 186 238 290 342 ?
ml la[;k dk p;u djsa tks fuEufyf[kr J`[a kyk esa ç'u fpà ¼\½ dks +52 +52 +52 +52 +52
çfrLFkkfir dj ldrh gSA 342+52 = 394
40, 20, 20, 30, 60, ?, 450, 1575, 6300 Correct option (c).
(a) 40 (b) 180 (c) 150 (d) 120  Concept -
Sol. (c)
Logic: - Multiplication Addition: Addition based alternate series
25. Select the number from among the given options that can
40 20 20 30 60 ? 450 1575 6300
replace the question mark (?) in the following series.
×0.5 ×1 ×1.5 × 2 ×2.5 ×3 ×3.5 ×4 fn, x, fodYiksa esa ls ml la[;k dk p;u djsa tks fuEufyf[kr
60×2.5=150 J`[a kyk esa ç'u fpà ¼\½ dks çfrLFkkfir dj ldrh gSA
Correct option (c). 100, 93, 103, 96, 106, 99, ?
 Concept - (a) 123 (b) 109 (c) 127 (d) 132
Sol. (b)
Multiplication: Alternative multiplication (Triple
Logic: - Addition
alternate series , three of them follows multiplication of
+3 +3
fixed numbers)
100 93 103 96 106 99 ?
22. Select the numbers from among the given options that can
replace the question marks (?) in the following series. +3 +3 +3
fn, x, fodYiksa esa ls mu la[;kvksa dk p;u djsa tks fuEufyf[kr 106+3 = 109
J`[a kyk esa ç'u fpà ¼\½ dks çfrLFkkfir dj ldrh gSaA Correct option (b).
8, 9, 5, 16, 27, 25, 32, 81, 125, ?, 243, ?  Concept -
(a) 325, 64 (b) 64, 625
Addition: Addition [+x, +y] Addition is alternate, where x,
(c) 375, 64 (d) 64, 375
y any random fix number
Sol. (b)
26. Select the number from among the given options that can
Logic: - Alternative multiplication
replace the question mark (?) in the following series.
×5 ×5 ×5
fn, x, fodYiksa ea ls og la[;k pqusa tks fuEufyf[kr J`a[kyk esa
8 9 5 16 27 25 32 81 125 ? 243 ?
iz’uokpd (?) ds LFkku ij vk ldrh gSA
×3 ×3 ×3
×2 ×2 ×2
145, 175, 235, 265, 325, ?, 415
32×2=64, 125×5=625 (a) 385 (b) 375 (c) 355 (d) 380
Correct option (b). Sol. (c)
Logic: - Addition [+30, +60]
 Concept -
145 175 235 265 325 ? 415
Addition Based
Addition: Addition Based Series +30 +60 +30 +60 +30 +60
Addition: Addition of alternate prime numbers (Alternate 325+30 = 355
Prime number added in series) Correct option (c).
23. Select the number from among the given options that can  Concept -
replace the question mark (?) in the following series.
Addition: Difference is increasing consecutive numbers
fn, x, fodYiksa esa ls og la[;k pqfu, tks fuEufyf[kr J`[a kyk esa
27. Select the number from among the given options that can
ç'uokpd fpà ¼\½ dks çfrLFkkfir dj ldsA
replace the question mark (?) in the following series.
82, 105, 136, 177, 224, 283, ?
fn, x, fodYiksa esa ls ml la[;k dk p;u djs]a tks fuEufyf[kr Js.kh
(a) 412 (b) 320 (c) 350 (d) 349
esa iz’u fpUg (?) ds LFkku ij vk ldrh gSA
Sol. (c)
24, 31, 39, 48, 58, 69, ?
Logic: - Alternate Prime number added in series
(a) 86 (b) 81 (c) 96 (d) 76
82 105 136 177 224 283 ? Sol. (b)
+23 +31 +41 +47 +59 +67 Logic: - Addition
283+67 = 350

262
Best App for Govt. Jobs : Neonclasses (Download Now)

31. Which of the following numbers will replace the question


24 31 39 48 58 69 ?
mark (?) in the given series?
+7 +8 +9 +10 +11 +12 fuEufyf[kr esa ls dkSu lh la[;k nh xbZ J`a[kyk esa ç'u fpà ¼\½ dks
69+12 = 81
çfrLFkkfir djsxh\
Correct option (b).
762, 755, 741, 720, 692, ?
(a) 657 (b) 655 (c) 664 (d) 641
 Concept - Sol. (a)
Addition : Difference between adjacent numbers follows Logic: - Subtraction Multiplication
AP 762 755 741 720 692 ?
28. Select the number that can replace the question mark (?) –7 –14 –21 –28 –35
in the following series. 692–35 = 657
fn, x, fodYiksa ea ls og la[;k pqusa tks fuEufyf[kr J`a[kyk esa Correct option (a).
iz’uokpd (?) ds LFkku ij vk ldrh gSA Parallel series Tn= a+(n-1)d, (n<0) (A.P. Subtraction)
45, 47, 53, 65, 85, ? 32. Which term of the series 195, 188, 181, ….. is its first
(a) 115 (b) 130 (c) 145 (d) 105 negative term?
Sol. (a) J`[a kyk 195] 188] 181] ……. dk dkSu lk in bldk igyk _.kkRed
Logic: - Addition in gS\
45 47 53 65 85 ? (a) 29 (b) 32 (c) 35 (d) 42
Sol. (a)
+2 +6 +12 +20 +30
Logics A.P. Subtraction
+4 +6 +8 +10
85+30 = 115 Parrallel series Tn= a+(n-1)d, (n<0)
Correct option (a). So a+(n+1)d<0
a = 195
 Concept - d = 188 – 195 = (–7)
Addition: Difference is in GP a + (n–1)d<0
29. Select the number that can replace the question mark in 195+(n–1)(–7)<0
the following series. 195–7n+7<0
ml la[;k dk p;u djsa tks fuEufyf[kr J`a[kyk esa ç'uokpd fpà dks 202–7n<0
çfrLFkkfir dj ldrh gSA n<29
62, 66, 74, 90, ? Correct option (a)
(a) 106 (b) 116 (c) 122 (d) 120  Concept -
Sol. (c)
Logic: - Addition (Multiplication) Alternating Two pattern Based
Multiplication & Addition Based
62 66 74 90 ? Multiplication & Addition: Every term is multiplied and
+4 +8 +16 +32 some number is added
33. Select the number from among the given options that can
×2 ×2 ×2
90+32 = 122 replace the question mark (?) in the following series.
Correct option (c).
fn, x, fodYiksa esa ls ml la[;k dk p;u djs]a tks fuEufyf[kr Js.kh
esa iz’u fpUg (?) ds LFkku ij vk ldrh gSA
 Concept - 6, 22, 70, 214, ?, 1942
Subtraction Based (a) 280 (b) 430 (c) 646 (d) 638
Subtraction: A fix number is subtracted Sol. (c)
30. Select the number from among the given options that can Logic: - Alternative Multiplication and Addition[×3 +4]
replace the question mark (?) in the following series.
6 22 70 214 ? 1942
fn, x, fodYiksa esa ls og la[;k pqfu, tks fuEufyf[kr J`[a kyk esa
ç'uokpd fpà ¼\½ dks çfrLFkkfir dj ldsA ×3+4 ×3+4 ×3+4 ×3+4 ×3+4
214×3+4642+4 = 646
237, 196, 155, 114, ?
(a) 47 (b) 98 (c) 73 (d) 64 Correct option (c).
Sol. (c)  Concept -
Logic: - Subtraction [–41]
Multiplication & Addition: Alternatemultiplication and
237 196 155 114 ? addition series. (some series generally doesn’t change
–41 –41 –41 –41 with some regular change , instead a shifting is observed
114–41 = 73 in every alternate step .)
34. Which of the following numbers will replace the question
Correct option (c).
mark (?) in the given series?
 Concept - fuEufyf[kr esa ls dkSu lh la[;k nh xbZ J`a[kyk esa ç'u fpà ¼\½ dks
Subtraction : Subtracted number are multiple of çfrLFkkfir djsxh\
some number 16, 96, 101, ?, 407, 814

263
Download Free PDFs & e-Books from Neon Classes App

(a) 405 (b) 396


131 1179 1171 ? 8191
(c) 400 (d) 404 ×9 –8 ×7 –6
Sol. (d)
131 × 9 = 1179
Logic: - Alternative Multiplication and Addition
1179 – 8 = 1171
16 96 101 ? 407 814 1171 × 7 = 8197
×6 +5 ×4 +3 ×2 8197 – 6 = 8191
101×4 = 404 Correct option (b).
Correct option (d).
 Concept -
 Concept -
Multiplication & Subtraction: Numbers are increasing and
Multiplication & Addition: Every term is multiplied with signs and operation are different , one is multiplication
fix number and together added numbers that follow a and other is subtraction
series logic 38. Select the number from among the given options that can
35. Select the number from among the given options that can replace the question mark (?) in the following series.
replace the question mark (?) in the following series. fn, x, fodYiksa esa ls og la[;k pqfu, tks fuEufyf[kr J`[a kyk esa
fn, x, fodYiksa esa ls ml la[;k dk p;u djs tks fuEufyf[kr Js.kh ç'uokpd fpà ¼\½ dks çfrLFkkfir dj ldsA
esa iz’uokpd (?) ds LFkku ij vk ldrh gSA 17, 16, 32, 29, 116, ?
13, 29, 62, 129, 264, ? (a) 121 (b) 111 (c) 71 (d) 81
(a) 535 (b) 395 (c) 470 (d) 505 Sol. (b)
Sol. (a) Logic: -Subtraction and Multiplication
Logic: - Alternative Multiplication and Addition[×2 +No.]
17 16 32 29 116 ?
13 29 62 129 264 ? –1 ×2 –3 ×4 –5
×2+3 ×2+4 ×2+5 ×2+6 ×2+7 116–5 = 111
264×2+7528+7 = 535 Correct option (b).
Correct option (a).
 Concept -
 Concept -
Addition & Subtraction Based
Multiplication & Subtraction Based Addition & Subtraction: Addition, Subtraction & Odd
Multiplication & Subtraction: A term is multiplied and number (when the numbers are increasing and
subtracted to get next term decreasing alternately , they follow alternate pattern
36. Select the correct option that will fill in the blank and depends on alternate logic
complete the series. 39. Select the number that can replace the question mark (?)
lgh fodYi dk p;u djsa tks fjä LFkku dks Hkjsxk vkSj J`[a kyk dks in the following series.
iwjk djsxkA ml la[;k dk p;u djsa tks fuEufyf[kr J`[a kyk esa ç'u fpà ¼\½ dks
15, 42, 123, ______, 1095 çfrLFkkfir dj ldrh gSA
(a) 366 (b) 358 (c) 370 (d) 360 17, 20, 15, 22, 13, ?
Sol. (a) (a) 4 (b) 2 (c) 24 (d) 22
Logic: - Multiplication and Subtraction (×, –) Sol. (c) 24
15 42 123 ? 1095 Logic: - Addition, Subtraction & Odd number
×3–3 ×3–3 ×3–3 ×3–3
17 20 15 22 13 ?
15×3–345–3=42 +3 –5 +7 –9 +11
42×3–3126–3=123 13+11 = 24
123×3–3369–3=366 Correct option (c).
366×3–31098–3=1095
 Concept -
Correct option (a).
Addition & Subtraction: (Alternate Subtraction and
 Concept -
Addition Prime number ) alternate addition and
Multiplication & Subtraction: when the logics are subtraction of consecutive prime numbers
alternate , once it is multiplied and next subtracted 40. Which of the following numbers will replace the question
37. Which of the following numbers will replace the question mark (?) in the given series?
mark (?) in the given series? fuEufyf[kr esa ls dkSu lh la[;k nh xbZ J`a[kyk esa ç'u fpà ¼\½ dks
fuEufyf[kr esa ls dkSu lh la[;k nh xbZ J`[a kyk esa ç'u fpà (?) dks çfrLFkkfir djsxh\
çfrLFkkfir djsxh\ 314, 285, 308, ?, 306, 293, 304
131, 1179, 1171, ?, 8191 (a) 277 (b) 298
(a) 8287 (b) 8197 (c) 8199 (d) 8289 (c) 287 (d) 289
Sol. (b) Sol. (d)
Logic: - Multiplication and Subtraction (×, –) Logic: - Alternative Subtraction and Addition Prime number

264
Best App for Govt. Jobs : Neonclasses (Download Now)

Alternate Logic : Numbers are decreasing series and


314 285 308 ? 306 293 304
signs are multiplication and subtraction alternately
–29 +23 –19 +17 –13 +11 44. Select the number from among the given options that can
308–19 = 289
replace the question mark (?) in the following series.
Correct option (d). fn, x, fodYiksa ea ls og la[;k pqusa tks fuEufyf[kr J`a[kyk esa
 Concept - iz’uokpd (?) ds LFkku ij vk ldrh gSA
25, 150, 145, 580, 577, ?
Addition & Subtraction: Difference follows alternate logic
(a) 980 (b) 1154 (c) 1020 (d) 1731
and two different series logics
Sol. (b)
41. Select the number that can replace the question mark (?)
Logic: - Multiplication and Subtraction
in the following series.
ml la[;k dk p;u djsa tks fuEufyf[kr J`[a kyk esa ç'u fpà ¼\½ dks 25 150 145 580 577 ?
çfrLFkkfir dj ldrh gSA ×6 –5 ×4 –3 ×2
123, 123, 110, 110, 121, 121, 108, 108, ?, ? 577 × 2 = 1154
(a) 123, 123 (b) 119, 119
Correct option (b).
(c) 97, 97 (d) 127, 127
Sol. (b)  Concept -
Logic: - Alternative Subtraction and Addition Alternate Logic: Alternate multiplication and addition of a
123 123 110 110 121 121 108 108 ? ? perfect cube numbers series
+0 –13 +0 +11 +0 –13 +0 +11 +0 45. Which of the following numbers will replace the question
108+11 = 119 119
+0 mark (?) in the given series?
Correct option (b). fuEufyf[kr esa ls dkSu lh la[;k nh xbZ J`a[kyk esa ç'u fpà ¼\½ dks
çfrLFkkfir djsxh\(
 Concept -
13, 14, 14, ?, 176
Alternate Logic Based (a) 148 (b) 22 (c) 38 (d) 42
Alternate Logic: When addition and multiplication Sol. (b)
operation are alternately applied in a series Logic: - Cube Addition and Multiplication
42. Select the number from among the given options that can
replace the question mark (?) in the following series. 13 14 14 ? 176
fn, x, fodYiksa esa ls og la[;k pqfu, tks fuEufyf[kr J`[a kyk esa
+13 ×13 +23 ×23
ç'uokpd fpà ¼\½ dks çfrLFkkfir dj ldsA 14 + 23 14 + 8 = 22
10, 14, 31, 35, 73, 77, ?
Correct option (b).
(a) 154 (b) 90 (c) 157 (d) 81
Sol. (c)  Concept -
Logic: - Alternative Addition and [Multiplication, Addition] Alternate Logic: Difference between alternate terms
10 14 31 35 73 77 ? follows a AP series
46. Select the number from among the given options that can
+4 ×2+3 +4 ×2+3 +4 ×2+3
77×2+3154+3 = 157 replace the question mark (?) in the following series.
fn, x, fodYiksa esa ls og la[;k pqfu, tks fuEufyf[kr J`[a kyk esa
Correct option (c).
ç'uokpd fpà ¼\½ dks çfrLFkkfir dj ldsA
 Concept - 10, 22, 35, 40, 72, 40, ?
Alternate Logic: (n×2+1] alternate multiplication series. (a) 131 (b) 134 (c) 133 (d) 143
43. Select the number from among the given options that can Sol. (c)
replace the question mark (?) in the following series. Logic: - Addition Alternative
fn, x, fodYiksa ea ls og la[;k pqusa tks fuEufyf[kr J`a[kyk esa +18
iz’uokpd (?) ds LFkku ij vk ldrh gSA +18 +0
14, 29, 57, 115, 229, ?
10 22 35 40 72 40 ?
(a) 457 (b) 463 (c) 461 (d) 459
Sol. (d) +25 +37 +61
Logic: - [n×2 and Alternative Addition and Subtraction] +12 +24
[n×2+1] Correct option (c).
14 29 57 115 229 459  Concept -
×2+1 ×2–1 ×2+1 ×2–1 ×2+1 Alternate Logic: Alternate Logic where the difference
(14×2)+1  28 + 1 = 29
follows alternate pattern(Dhadkan)
(29×2)+1  58 – 1 = 57
47. Which of the following numbers will replace the question
(57×2)+1  114 + 1 = 115 mark (?) in the given series?
(229×2)+1  458 + 1 = 459 fuEufyf[kr esa ls dkSu lh la[;k nh xbZ J`a[kyk esa ç'u fpà ¼\½ dks
Correct option (d). çfrLFkkfir djsxh\
 Concept - 5, 16, 28, 36, 71, 90, 234, 342, ?

265
Download Free PDFs & e-Books from Neon Classes App

(a) 739 (b) 793 (c) 893 (d) 839 12+1 22+2 62+6 422+42
Sol. (b)
1 1 2 2 6 10 42 ? 1806
Logic: - Cube and Square
13+1 23+2 103+10
5 16 28 36 71 90 234 342 ?
Correct option (c).
+11 +12 +8 +35 +19 +144 +108 +451
+1 –4 +27 –16 +125 –36 +343  Concept -
13 22 33 42 53 62 73 Alternate Logic: Subtraction Number and cube number(-n,
Correct option (a). -n2) (alternate number subtraction and subtraction of its
square)
 Concept -
51. Which number will replace the question mark (?) in the
Alternate Logic: Power series , here the terms follows following series?
power based logic , where the power terms are in series or fuEufyf[kr J`[a kyk esa ç'uokpd fpUg (?) ds LFkku ij dkSu lh
the base of the power term follows some logic la[;k vk,xh\
48. Select the number from among the given options that can 98, 95, 86, 82, 66, ?, 36
replace the question mark (?) in the following series. (a) 58 (b) 63 (c) 61 (d) 60
fn, x, fodYiksa esa ls ml la[;k dk p;u djs]a tks fuEufyf[kr Js.kh Sol. (c)
esa iz’u fpUg (?) ds LFkku ij vk ldrh gSA Logic: - Subtraction Number and cube number(-n, -n3)
1, 8, 81, ?, 15625
(a) 1015 (b) 1225 (c) 1227 (d) 1024 98 95 86 82 66 ? 36
Sol. (d)
–3 –32 –4 –42 –5 –52
Logic: - Power (xn) 66 – 5 = 61
1 8 81 ? 15625 Correct option (c).
 Concept -
12 23 34 45 56
4  4×4×4×4×4 = 1024
5 Alternate Logic: Alternate multiplication and division
Correct option (d). more than 3 series with various logics are asked
alternately(three series following various series logics .)
 Concept - 52. Select the correct alternative to fill up the blank and
Alternate Logic: Previous number×5–Prime No. complete the series.
multiplication is fixed but the subtraction is a consecutive fjä LFkku dks Hkjus vkSj J`a[kyk dks iwjk djus ds fy, lgh fodYi
prime number series dk p;u djsAa
49. Select the number from among the given options that can 22, 4, 16, 24, 6, 36, 26, 8, 64, 28, 10, ?
replace the question mark (?) in the following series. (a) 100 (b) 30 (c) 3 (d) 9
fn, x, fodYiksa esa ls ml la[;k dk p;u djsa tks fuEufyf[kr Sol. (a)
J`[a kyk esa ç'u fpà ¼\½ dks çfrLFkkfir dj ldrh gSA Logic: - Alternative Addition
+2 +2 +2
6, 27, 128, 629, ?
22 4 16 24 6 36 26 8 64 28 10 ?
(a) 3130 (b) 3131 (c) 2121 (d) 2120
+2 +2 +2
Sol. (a) +20 +28 +36
Logic: - Previous number×5–Prime No. 64 +36 = 100
Correct option (a).
6 27 128 629 ?
×5-3 ×5-7 ×5-11 ×5-15
6 × 5 – 3  30 – 3 = 27  Concept -
27 × 5 – 7  135 – 7 = 128
Arithmetical Progression Based
128 × 5 – 11  640 – 11 = 629
Arithmetical Progression: The given series is in AP
629 × 5 – 15  3145 – 15 = 3130
53. In the following number series, two numbers have been
Correct option (a). put within brackets. Select the most appropriate option
 Concept - for these numbers in relation to their inclusion in the
series.
Alternate Logic: [n2+n and n3+n] alternate logics where
one is square logic and other is cube
fuEufyf[kr la[;k J`a[kyk esa] nks la[;kvksa dks dks"Bd esa j[kk x;k gSA
50. Select the number from among the given options that can
bu uacjksa dks J`a[kyk esa 'kkfey djus ds lacaèk esa muds fy, lcls
replace with the question mark (?) in the following series mi;qä fodYi dk p;u djsAa
fn, x, fodYiksa esa ls og la[;k pqfu tks fuEufyf[kr Js.kh esa 2, 4, 6, 8, 10, (12), 14, 16, (18), 20
iz’uokpd fpUg (?) ds LFkku ij vk ldrh gSA (a) The first bracketed number is correct and the second is
incorrect.
1, 1, 2, 2, 6, 10, 42, ?, 1806
(a) 1302 (b) 1203 (c) 1010 (d) 1389
igyh czSdsV okyh la[;k lgh gS vkSj nwljh xyr gSA
Sol. (c) (b) The first bracketed number is incorrect and the second is
Logic: - [n2+n and n3+n] correct.
igyh czSdsV okyh la[;k xyr gS vkSj nwljh lgh gSA
(c) Both the bracketed numbers are incorrect.

266
Best App for Govt. Jobs : Neonclasses (Download Now)

dks"Bd esa nh xbZ nksuksa la[;k,¡ xyr gSaA +1 +1 +1


(d) Both the bracketed numbers are correct. 1 5 2 6 3 7 4 ?
dks"Bd esa nh xbZ nksuksa la[;k,¡ lgh gSaA
Sol. (d) +1 +1 +1
Logic: - Arithmetical Progression, addition [L0] 7+1= 8
[A. P  A. P. is a sequence of numbers in order in which Correct option (b).
the difference between any two consecutive numbers is a
 Concept -
constant value. It is also called arithmetic sequence.
(A.P. Øe esa la[;kvksa dk ,d Øe gS] ftles fdUgh nks Øekxr Arithmetical Progression: Arithmetical Progression,
la[;kvksa ds chp dk varj ,d fLFkj eku gksrk gS blsa vadxf.krh; Øe Subtraction [L1]
Hkh dgrs gSA)] 57. From the given options, choose the correct one that will
replace the question mark (?) in the following series.
2 4 6 8 10 (12) 14 16 (18) 20
fn, x, fodYiksa esa ls og lgh pqusa tks fuEufyf[kr J`a[kyk esa
+2 +2 +2 +2 +2 +2 +2 +2 +2 ç'uokpd fpà (?) dks çfrLFkkfir djsxkA
Correct option (d). 83, 74, 66, 59, 53, ?, 44, 41
 Concept - (a) 42 (b) 50 (c) 44 (d) 48
Sol. (d)
Arithmetical Progression: Arithmetical Progression, Logic: - Arithmetical Progression, Subtraction [L1]
addition [L0]
54. Which of the following numbers will replace the question 83 74 66 59 53 ? 44 41
mark (?) in the given number series? -9 -8 -7 -6 -5 -4 -3
fuEufyf[kr esa ls dkSu lh la[;k nh xbZ la[;k J`a[kyk esa ç'u fpà 53 – 5 = 48
¼\½ dks çfrLFkkfir djsxh\ Correct option (d).
209, 318, 427, 536, ?, 754  Concept -
(a) 655 (b) 645 (c) 664 (d) 635
Arithmetical Progression: Arithmetical Progression,
Sol. (b)
Multiplication [L1] The terms increase faster and next
Logic: - Arithmetical Progression, addition [L0]
terms are multiple of first term or near to multiple of first
209 318 427 536 ? 754 term
58. Select the number from among the given options that can
+109 +109 +109 +109 +109
replace the question mark (?) in the following series.
536 + 109 = 645
fn, x, fodYiksa esa ls ml la[;k dk p;u djsa tks fuEufyf[kr
Correct option (b). J`[a kyk esa ç'u fpà ¼\½ dks çfrLFkkfir dj ldrh gSA
 Concept - 21, 42, 168, 1008, ?
(a) 8304 (b) 7864 (c) 8834 (d) 8064
Arithmetical Progression: Arithmetical Progression,
Sol. (d)
addition [L1]
Logic: - Arithmetical Progression, Multiplication [L1]
55. Select the number from among the given options that can
replace the question mark (?) in the following series. 21 42 168 1008 ?
fn, x, fodYiksa esa ls og la[;k pqfu, tks fuEufyf[kr J`[a kyk esa
×2 ×4 ×6 ×8
ç'uokpd fpà ¼\½ dks çfrLFkkfir dj ldsA 1008 × 8 = 8064
39, 53, 69, 87, ?
Correct option (d).
(a) 99 (b) 107 (c) 92 (d) 115
Sol. (b)  Concept -
Logic: - Arithmetical Progression, addition [L1] Arithmetical Progression: Arithmetical Progression,
39 53 69 87 ? addition, subtraction [L2]
59. Which of the following numbers will replace the question
+14 +16 +18 +20 mark (?) in the given series?
87+20 = 107 fuEufyf[kr esa ls dkSu lh la[;k nh xbZ J`a[kyk esa ç'u fpà ¼\½ dks
Correct option (b). çfrLFkkfir djsxh\
154, 165, 148, ?, 142, 177
 Concept -
(a) 174 (b) 162 (c) 171 (d) 168
Arithmetical Progression: Arithmetical Progression, Sol. (c)
alternative addition [L1] Logic: - Arithmetical Progression, addition, subtraction [L2]
56. Select the correct option that will fill in the blank and
complete the series. 154 165 148 ? 142 177
lgh fodYi dk p;u djsa tks fjä LFkku dks Hkjsxk vkSj J`[a kyk dks +11 -17 +23 -29 +35
iwjk djsxkA +6 +6 +6 +6
1, 5, 2, 6, 3, 7, 4, ________ 148 + (17+6)148+23 = 171
(a) 9 (b) 8 (c) 12 (d) 6
Sol. (b) Correct option (c)
Logic: - Arithmetical Progression, alternative addition [L1]

267
Download Free PDFs & e-Books from Neon Classes App

 Concept - Correct option (a).

Arithmetical Progression: Arithmetical Progression,  Concept -


addition [L3] Arithmetical Progression: Difference between the terms
60. Select the option that will replace the question mark to are Square Numbers
complete the given series. Arithmetical Progression: Arithmetical Progression, [an= a
ml fodYi dk p;u djsa tks nh xbZ J`[a kyk dks iwjk djus ds fy, + (n-1)d]( Finding unrelated term in an AP series)
ç'u fpà dks çfrLFkkfir djsxkA 63. In the series 11, 19, 27, 35, 43, …… Which of the following
7, 17, 41, 85, ?, 257 will NOT be a number of the series?
(a) 155 (b) 165 (c) 105 (d) 150 J`[a kyk esa 11] 19] 27] 35] 43] …… fuEufyf[kr esa ls dkSu lh J`a[kyk
Sol. (a) dh la[;k ugha gksxh\
Logic: - Arithmetical Progression, addition [L3] (a) 434 (b) 107 (c) 307 (d) 195
Sol. (a)
7 17 41 85 ? 257
Logic :- Arithmetical Progression, [an= a + (n-1)d]
+10 +24 +44 +70 +102 Given – 11, 19, 27, 35, 43 ………………
+14 +20 +26 +32 Option (a) = First term [11]
d = common difference [19-11=8]
+6 +6 +6 434 = 11 + (n-1)×8
85 + 70 = 155
423
Correct option (a).  n  1   52.875
8
 Concept - This value will not be possible in this series, as number of
Arithmetical Progression: Arithmetical Progression, terms can’t be in decimal.
multiplication [L3] (difference of difference are in GP.) Option (b) 107 = 11 + (n-1)×8
61. Select the number from among the given options that can 96
replace the question mark (?) in the following series.  n  1   12
8
fn, x, fodYiksa ea ls og la[;k pqusa tks fuEufyf[kr J`a[kyk esa n = 12+1 = 13
iz’uokpd (?) ds LFkku ij vk ldrh gSA (This is a 13th term in this series)
24, 40, 64, 104, ?, 312 Option (c) 307 = 11+(n-1)×8
(a) 228 (b) 176 (c) 154 (d) 168 296
Sol. (b)  n  1   37
8
Logic: - Arithmetical Progression, multiplication [L3]
n = 37+1 = 38
24 40 64 104 ? 312 (This is a 38th term in this series)
Option (d) 195 = 11 + (n-1)×8
+16 +24 +40 +72 +136 184
 n  1   23
+8 +16 +32 +64 8
n = 23 + 1 = 24
×2 ×2 ×2 The number which cannot be possible in this series in 434
Correct option (b). Correct option (a)
 Concept -  Concept -
Arithmetical Progression: Arithmetical Progression, Geometric Progression Based
Addition [L4] Geometric Progression: G.P. Multiplication by 3 is add [L1)
62. Which of the following numbers will replace the question (the difference are Product series)
mark (?) in the given series? 64. Select the number from among the given options that can
fuEufyf[kr esa ls dkSu lh la[;k nh xbZ J`a[kyk esa ç'u fpà ¼\½ dks replace the question mark (?) in the following series.
çfrLFkkfir djsxh\ fn, x, fodYiksa esa ls og la[;k pqusa tks fuEufyf[kr J`a[kyk esa
27,31,42,72,139,? iz’uokpd (?) ds LFkku ij vk ldrh gSA
(a) 267 (b) 264 (c) 271 (d) 355 67, 70, 76, 85, ?
Sol. (a) (a) 96 (b) 89 (c) 97 (d) 98
Logic: - Arithmetical Progression, Addition [L4] Sol. (c)
Logic: - G.P. Multiplication by 3 is add [L1]
27 31 42 72 139 ?
67 70 76 85 ?
+4 +11 +30 +67 +128 +3 +6 +9 +12
+7 +19 +37 +61 85 + 12 = 97
Correct option (c).
+12 +18 +24 65. Select the number from among the given options that can
+6 +6 replace the question mark (?) in the following series.
139 + 128 =267 fn, x, fodYiksa ea ls og la[;k pqusa tks fuEufyf[kr J`a[kyk esa
iz’uokpd (?) ds LFkku ij vk ldrh gSA

268
Best App for Govt. Jobs : Neonclasses (Download Now)

70, 66, 58, 46, 30,? Geometric Progression: when the terms of the series are
(a) 16 (b) 10 (c) 12 (d) 8 in Product series , The multiplication numbers themselves
Sol. (b) follows AP
Logic: - G.P. Multiplication by 4 is add[L1] 69. Which of the following numbers will replace the question
mark (?) and complete the given number series?
70 66 58 46 30 ?
fuEufyf[kr esa ls dkSu lh la[;k ç'u fpà ¼\½ dks çfrLFkkfir djsxh
-4 -8 -12 -16 -20 vkSj nh xbZ la[;k J`a[kyk dks iwjk djsxh\
30 - 20 = 10 2, 12, 144, 2592, ?
Correct option (b). (a) 62108 (b) 62208
 Concept - (c) 62204 (d) 63408
Sol. (b)
Geometric Progression: G.P. Multiplication [L2] (When Logic: - G.P. Multiplication [L1]
consecutive term’s difference is in GP).
66. Select the number from among the given options that can 2 12 144 2592 ?
replace the question mark (?) in the following series. ×6 ×12 ×18 ×24
fn, fodYiksa esa ls og la[;k pqfu, tks fuEufyf[kr J`s.kh esa iz’uokpd 2592 × 24 = 62208
fpUg (?) ds LFkku ij vk ldrh gSA Correct option (b).
18, 34, 66, 130, ?
 Concept -
(a) 273 (b) 178 (c) 232 (d) 258
Sol. (d) Geometric Progression: Fibonacci [a+b=c] Sum of
Logic: - G.P. Multiplication [L2] predecessors is equal to third term
70. Which number will replace the question mark (?) in the
18 34 66 130 ?
following series?
+16 +32 +64 +128 fuEufyf[kr J`[a kyk esa ç'uokpd fpUg ¼\½ ds LFkku ij dkSu lh
×2 ×2 ×2 la[;k vk,xh\
130 + 128 =258 3, 4, 7, 11, 18, ?
Correct option (d). (a) 31 (b) 22 (c) 25 (d) 29
67. Which number will replace the question mark (?) in the Sol. (d)
following series? Logic: - Fabonacci [a+b=c]
fuEu Js.kh esa iz’uokpd fpgu (?) ds LFkku ij dkSu&lh la[;k 3 4 7 11 18 ?
vk,xh\ +1 (3+4)=7 (7+4)=11(11+7)=18 (11+18)=29
17, 37, 77, 157, 317, ? Correct option (d).
(a) 634 (b) 683 (c) 649 (d) 637
 Concept -
Sol. (d)
Logic: - G.P. Multiplication [L2] Geometric Progression: Fibonacci [a×b=c…..]
(Multiplication of two consecutive terms is equal to the
17 37 77 157 317 ?
third term
+20 +40 +80 +160 +320 71. Select the number that can replace the question mark (?)
×2 ×2 ×2 ×2 in the following series.
317 + 320 =637 fuEufyf[kr J`[a kyk esa ç'uokpd fpUg ¼\½ dks çfrLFkkfir djus okyh
Correct option (d). la[;k dk p;u djsa
1, 1, 2, 3, 7, 22, ?
 Concept - (a) 157 (b) 156 (c) 155 (d) 152
Geometric Progression: The difference terms are in AP Sol. (c)
and their signs are alternate Logic: - Fabbonacci [a×b=c…..]
68. Which of the following numbers will replace the question 1×2+1=3 3×7+1=22
mark (?) in the given series?
fuEufyf[kr esa ls dkSu lh la[;k nh xbZ J`a[kyk esa ç'u fpà ¼\½ dks 1 1 2 3 7 22 ?
çfrLFkkfir djsxh\
1×1+1=2 2×3+1=7 7×22+1=155
14, 26, 11, ?, 8, 32
(a) 29 (b) 22 (c) 25 (d) 18 Correct option (c).
Sol. (a)
 Concept -
Logic: - G.P. Alternative symbols [L1]
Geometric Progression: Fibonacci [a, a×a+1=b] [b,
14 26 11 ? 8 32 b×b+1=c] (every new term depends on square of first
+12 -15 +18 -21 +24 added to some number.)
11 + 18 = 29 72. Which number will come next in the following series?
Correct option (a). fuEufyf[kr J`[a kyk esa dkSu lh la[;k vkxs vk,xh\
1, 2, 5, ...., 677
 Concept -
(a) 26 (b) 25 (c) 95 (d) 11
Sol. (a)

269
Download Free PDFs & e-Books from Neon Classes App

Logic: - Fabbonacci [a, a×a+1=b] [b, b×b+1=c] uhps nh xbZ vkd`fr;ksa esa dkxt ds ,d VqdM+s dks eksM+us dk Øe vkSj
2×2= 4+1=5
ml eqM+s gq, dkxt dks dkVus dk rjhdk n’kkZ;k x;k gSaA bl dkxt
dks [kksyus ij ;g dSlk fn[ksxk\
1 2 5 ? 677

1×1=1+1=2 5×5=25+1=26
Correct option (a).
 Concept -
Geometric Progression: Fabbonacci [a2+K=b…..] where k,
can be any random number
73. Select the number from among the given options that can
replace the question mark (?) in the following series.
fn, x, fodYiksa esa ls ml la[;k dk p;u djsa tks fuEufyf[kr
J`[a kyk esa ç'u fpà ¼\½ dks çfrLFkkfir dj ldrh gSA Sol. (a)
0, 1, 2, 5, 26, ? The line from which the paper is folded. When it is unfolded
(a) 475 (b) 677 (c) 69 (d) 27 mirror image is formed.
Sol. (b)
 Triangle shape changed to mirror image are same. So get
Logic: - Fabbonacci [a2+1=b…..]
eliminated. Option (b)
12+1=2 52+1=26  Option (c) square top and bottom triangle so get eliminated
0 1 2 5 26 ?  Option (d) two times mirror image so square will be 4. So
get eliminated
02+1=1 22+1=5 262+1=677
 Option (a) correct answer
Correct option (b).

Paper folding and cutting


1. The sequence of folding a piece of paper and the manner 3. The sequence of folding a piece of paper and the manner
in which the folded paper has been cut is shown in the in which the folded paper has been cut is shown in the
following figures. How would this paper look when following figures. How would this paper look when
unfolded? unfolded?
dkxt ds ,d VqdM+s dks eksM+us dk Øe vkSj eqM+s gq, dkxt dks dkVus fuEukafdr vkd`fr;ksa esa dkxt ds ,d VqdM+s dks eksM+us dk Øe vkSj
dk rjhdk fuEufyf[kr vk—fr;ksa esa fn[kk;k x;k gSA vuQksYM gksus eqM+s gq, dkxt dks dkVus dk rjhdk fn[kk;k x;k gSA ;g dkxt
ij ;g isij dSlk fn[ksxk\ [kksyus ij dSlk fn[ksxk\

(a) (b)

Sol. (a) (c) (d)


The line from which the paper is folded. When it is unfolded
mirror image is formed.
Sol. (c)
 Option (b) and (d) does not follow the same orientation so The line from which the paper is folded. When it is unfolded
gets eliminated. mirror image is formed.
 Option (c) extra line in shape  Option (a) 2 times folding so mirror image circle shape 4.
 Option (a) correct answer So get eliminated.
 Option (b) Triangle shape change to mirror image so
mirror image circle shape 4. So get eliminated.
 Option (c) Correct answer
2. The sequence of folding a piece of paper and the manner
in which the folded paper has been cut is shown in the
following figures. How would this paper look when
unfolded?

270
Best App for Govt. Jobs : Neonclasses (Download Now)

 Option (d) Correct answer

 Option (d) Second column circle and square are


6. The sequence of folding a piece of paper (figure i) and the
interchanged so get eliminated.
manner in which the folded paper has been cut (figure ii)
4. The sequence of folding a piece of paper and the manner
is shown in the following figures. Select the option that
in which the folded paper has been cut is shown in the
would most closely resemble the unfolded form of figure
following figures. How would this paper look when
(ii).
unfolded?
dkxt ds ,d VqdM+s dks eksM+us dk Øe vkSj eqM+s gq, dkxt dks dkVus dkxt ds ,d VqdM+s dks eksM+us dk Øe ¼vk—fr i½ vkSj ftl rjhds ls
dk rjhdk fuEufyf[kr vk—fr;ksa esa fn[kk;k x;k gSA vuQksYM gksus eqM+s gq, dkxt dks dkVk x;k gS ¼vk—fr ii½ fuEufyf[kr vk—fr;ksa esa
ij ;g isij dSlk fn[ksxk\ fn[kk;k x;k gSA ml fodYi dk p;u djsa tks vk—fr ¼ii½ ds [kqys
:i ds lcls fudV ls feyrk tqyrk gksA

(i) (ii)

(a) (b) (c) (d)


Sol. (d)
The line from which the paper is folded. When it is unfolded
(a) (b) (c) (d)
mirror image is formed.
Sol. (b)
 Option (a) Extra circle outer so gets eliminate
The line from which the paper is folded. When it is unfolded
 Option (b) Minimum/ less circle outer so gets eliminate mirror image is formed.
 Option (c) Less circle outer so get eliminate  Option (a) & (c) Triangle shape changed to mirror image
 Option (d) Correct answer (4 time folding so 2 circle × 8 = are same so get eliminate
116 circle)  Option (b) Correct answer


5. A paper is folded and cut as shown below. How will it 
appear when unfolded? Select the figure that most closely  Option (d) Triangle shape changed so get eliminated.
resembles the formed figure after unfolding. 7. The sequence of folding a piece of paper and the manner
,d dkxt+ dks uhps n'kkZ, vuqlkj eksM+k vkSj dkVk tkrk gSA çdV in which the folded paper has been cut is shown in the
gksus ij ;g dSls fn[kkbZ nsxk\ ml vk—fr dk p;u djsa tks lkeus following figures. How would this paper look when
vkus ds ckn cuh gqbZ vk—fr ls lcls vfèkd feyrh tqyrh gSA unfolded?
dkxt ds ,d VqdM+s dks eksM+us dk Øe vkSj eqM+s gq, dkxt dks dkVus
dk rjhdk fuEufyf[kr vk—fr;ksa esa fn[kk;k x;k gSA vuQksYM gksus
ij ;g isij dSlk fn[ksxk\

Sol. (d)
The line from which the paper is folded. When it is unfolded
mirror image is formed.
 Option (a) Two 1/2 circle so get eliminate
 Option (b) Extra triangle and 1/2 circle so get eliminate (a) (b) (c) (d)
 Option (c) Only one circle so get eliminate Sol. (a)

271
Download Free PDFs & e-Books from Neon Classes App

The line from which the paper is folded. When it is unfolded


mirror image is formed.
 Option (a) Correct answer (c) (d)

Sol. (b)
The line from which the paper is folded. When it is unfolded
mirror image is formed.
 Option (a)/ (c) does not follow the same orientation so gets
 Option (b) Extra circle so get eliminate
eliminated.
 Option (c) Extra line so get eliminate
 Option (b) correct answer
 Option (d) does not follow the some orientation so get
eliminate
8. The sequence of folding a piece of paper and the manner
in which the folded paper has been cut is shown in the
following figures. How would this paper look when
unfolded?
dkxt ds ,d VqdM+s dks eksM+us dk Øe vkSj eqM+s gq, dkxt dks dkVus  Option (d) one side face triangle so gets eliminated
dk rjhdk fuEufyf[kr vk—fr;ksa esa fn[kk;k x;k gSA vuQksYM gksus
ij ;g isij dSlk fn[ksxk\ 10. The sequence of folding a piece of paper and the manner
in which the folded paper has been cut is shown in the
following figures. How would this paper look when
unfolded?
fuEukafdr vkd`fr;ksa esa dkxt ds ,d VqdM+s dks eksM+us dk Øe vkSj
eqM+s gq, dkxt dks dkVus dk rjhdk fn[kk;k x;k gSA ;g dkxt
[kksyus ij dSlk fn[ksxk\

(a) (b) (c) (d)


Sol. (a)
The line from which the paper is folded. When it is unfolded
mirror image is formed. (i) (ii) (iii)
 Option (a) correct answer

(a) (b)

 Option (b) right angle triangle so get eliminate


 Option (c) triangle face not both side (face to face) so get
eliminate
(c) (d)
 Option (d) 2 times folding so 4 × one circle = 4 circle so get
eliminate
9. The sequence of folding a piece of paper and the manner
in which the folded paper has been cut is shown in the
Sol. (d)
following figures. How would this paper look when
The line from which the paper is folded. When it is unfolded
unfolded?
mirror image is formed.
fuEukafdr vkd`fr;ksa esa dkxt ds ,d VqdM+s dks eksM+us dk Øe vkSj
eqM+s gq, dkxt dks dkVus dk rjhdk fn[kk;k x;k gSA ;g dkxt  Option (a)/ (c) square and circle changed in the position so
[kksyus ij dSlk fn[ksxk\ gets eliminated.
 Option (b) circle position changed so gets eliminated.
 Option (d) correct answer

(a) (b)
11. The sequence of folding a piece of paper and the manner
in which the folded paper has been cut is shown in the

272
Best App for Govt. Jobs : Neonclasses (Download Now)

following figures. How would this paper look when 13. Select the option that depicts how the given transparent
unfolded? sheet of paper would appear- if it is folded at the dotted
dkxt ds ,d VqdM+s dks eksM+us dk Øe vkSj eqM+s gq, dkxt dks dkVus line.
dk rjhdk fuEufyf[kr vk—fr;ksa esa fn[kk;k x;k gSA vuQksYM gksus ml fodYi dk p;u djsa tks n'kkZrk gS fd dkxt dh nh xbZ ikjn'kÊ
ij ;g isij dSlk fn[ksxk\ 'khV dSlh fn[kkbZ nsxh& ;fn bls fcanhnkj js[kk ij eksM+k tk,A

Sol. (b)
The line from which the paper is folded. When it is unfolded (a) (b)
mirror image is formed.
 Option (a) square and triangle are mid so get eliminated
 Option (b) correct answer

(c) (d)
Sol. (b)
 Option (c) one side face triangle so get eliminated When the transparent sheet of paper is folded at the dotted
 Option (d) circle changed the positions so get eliminated line, it will appear as shown below
12. The sequence of folding a piece of paper and the manner  Option (a) top side line diagonal changed
in which the folded paper has been cut is shown in the  Option (b) correct answer
following figures. How would this paper look when
unfolded?
dkxt ds ,d VqdM+s dks eksM+us dk Øe vkSj eqM+s gq, dkxt dks dkVus
dk rjhdk fuEufyf[kr vk—fr;ksa esa fn[kk;k x;k gSA vuQksYM gksus
ij ;g isij dSlk fn[ksxk\

 Option (c) decrease line so get


 Option (d) top right to bottom left diagonal so get
eliminated
14. Select the option that depicts how the given transparent
sheet of paper would appear if it is folded at the dotted
line.
ml fodYi dk p;u djsa tks n'kkZrk gS fd dkxt dh nh xbZ ikjn'kÊ
'khV dks fcanhnkj js[kk ij eksM+us ij dSlk fn[kkbZ nsxkA
(a) (b) (c) (d)
Sol. (b)
The line from which the paper is folded. When it is unfolded
mirror image is formed.
 Option (a) bottom triangle changed positions so get
eliminated
 Option (b) correct answer

(a) (b)

 Option (c) right triangle positions changed. So get


eliminated
 Option (d) circle and right triangle interchanged so get
eliminated
(c) (d)

273
Download Free PDFs & e-Books from Neon Classes App

Sol. (d)
When the transparent sheet of paper is folded at the dotted
line, it will appear as shown below
 Option (a)/ (b) extra dark triangle so get eliminated
 Option (c) does not follow the same orientation so get
eliminated
 Option (d) correct answer

(a) (b)
15. Select the option that depicts how the given transparent
sheet of paper would appear- if it is folded at the dotted
line.
ml fodYi dk p;u djsa tks n'kkZrk gS fd dkxt dh nh xbZ ikjn'kÊ
'khV dSlh fn[kkbZ nsxh& ;fn bls fcanhnkj js[kk ij eksM+k tk,A

(c) (d)
Sol. (b)
When the transparent sheet of paper is folded at the dotted
line, it will appear as shown below
 Option (a) triangle face bottom so get elimination
 Option (b) correct answer

(a) (b)

 Option (c) triangle shape so get elimination


 Option (d) bottom triangle face west so get elimination
17. A paper is folded in a particular way and a number of
holes are made in it. When the paper is opened it looks
like a question figure. From the given alternatives, select
(c) (d) the figure which shows how the paper was folded and
Sol. (c) made holes.
When the transparent sheet of paper is folded at the dotted ,d dkxt dks fo’ks"k rjhds ls eksM+k tkrk gS vkSj mlesa dbZ lkjs Nsn
line, it will appear as shown below cuk, tkrs gSA tc dkxt dks [kksyk tkrk gS rks og iz’u vkd`fr dh
 Option (a)/ (d) triangle face one side so get eliminated rjg fn[krk gSA fn, x, fodYiksa esa ls] ml vkd`fr dk p;u djsa tks
 Option (b) joint two triangle so get eliminated ;g n’kkZrh gS fd dkxt dks fdl rjg eksM+k x;k vkSj mlesa Nsn
 Option (c) correct answer cuk, x,A

16. Select the option that depicts how the given transparent
sheet of paper would appear if it is folded at the dotted
line.
ml fodYi dk p;u djsa tks n'kkZrk gS fd dkxt dh nh xbZ ikjn'khZ
'khV dks fcanhnkj js[kk ij eksM+us ij dSlk fn[kkbZ nsxkA (a) (b)

274
Best App for Govt. Jobs : Neonclasses (Download Now)

(a) (b) (c) (d)

Sol. (c)
The line from which the paper is folded. When it is unfolded
mirror image is formed.
(c) (d)
Sol. (d)
 Option (a) triangle face are side so get eliminate
 Option (a) big and small size square so get eliminated  Option (b) right angle triangle position changed. So get
eliminate
 Option (b) extra square so get eliminated
 Option (c) correct answer
 Option (c) small size square so get eliminated
 Option (d) correct answer

 Option (d) triangle face one side so get eliminate


18. The sequence of folding a piece of paper and the manner 20. A paper is folded and cut as shown below. How will it
in which the folded paper has been cut is shown in the appear when unfolded?
following figures. How would this paper look when ,d dkxt dks uhps fn[kk, x, rjhds ds vuqlkj eksM+k vkSj dkVk x;k
unfolded? gSA ;g dkxt [kksyus ij dSlk fn[ksxk\
dkxt ds ,d VqdM+s dks eksM+us dk Øe vkSj eqM+s gq, dkxt dks dkVus
dk rjhdk fuEufyf[kr vk—fr;ksa esa fn[kk;k x;k gSA vuQksYM gksus
ij ;g isij dSlk fn[ksxk\

(a) (b)
(a) (b) (c) (d)

Sol. (a)
The line from which the paper is folded. When it is unfolded
mirror image is formed.
 Option (a) correct answer
(c) (d)
Sol. (d)
The line from which the paper is folded. When it is unfolded
 Option (b) bottom triangle changed so get eliminated mirror image is formed.
 Option (c) & (d) Square and triangle changed the position  Option (a) left bottom triangle changed so get eliminated
so get eliminated  Option (b) left top triangle changed so get eliminated
19. The sequence of folding a piece of paper (figure i) and the  Option (c) bottom circle and triangle position changed so
manner in which the folded paper has been cut (figures ii) get eliminated
is shown in the following. Select the option that would
 Option (d) correct answer
most closely resemble the unfolded form of figure (ii)
dkxt ds ,d VqdM+s dks eksM+us dk Øe ¼vk—fr i½ vkSj ftl rjhds ls
eqM+s gq, dkxt dks dkVk x;k gS ¼vkadM+k ii½ fuEufyf[kr esa fn[kk;k
x;k gSA ml fodYi dk p;u djsa tks vk—fr ds [kqys :i ds lcls
fudV ls feyrk tqyrk gks ¼ii½
21. A paper is folded and cut as shown below. How will it
appear when unfolded?
,d dkxt dks uhps fn[kk, x, rjhds ds vuqlkj eksM+k vkSj dkVk x;k
gSA ;g dkxt [kksyus ij dSlk fn[ksxk\
(i) (ii)

275
Download Free PDFs & e-Books from Neon Classes App

X Y Z
A B C D
Sol. (b)
When the transparent sheet of paper is folded at the dotted
line, it will appear as shown below
A B C D
Sol. (b)

In figure X, the square sheet of paper has been folded along the
horizontal line of the symmetry so that the upper half of the
24. Select the option that depicts how the given transparent
sheet overlaps with the lower half.
sheet of paper would appear if it is folded at the dotted
In figure Y, the sheet is folded further into a quarter. Again in
line.
figure Z, a quadrilateral is cut at the centre.
ml fodYi dk p;u djsa tks n'kkZrk gS fd dkxt dh nh xbZ ikjn'khZ
This quadrilateral will be created in each quarter of the paper.
'khV dks fcanhnkj js[kk ij eksM+us ij dSlk fn[kkbZ nsxkA
As the quadrilateral is punched close to the edge, a full design
is seen when the paper is unfolded. Thus, when the paper is
unfolded four quadrilateral shapes will appear doubled
symmetrically.
Hence option b is correct.
22. Select the option that depicts how the given transparent sheet
of paper would appear if it is folded at the dotted line.
ml fodYi dk p;u djsa tks n'kkZrk gS fd dkxt dh nh xbZ ikjn'khZ
'khV dks fcanhnkj js[kk ij eksM+us ij dSlk fn[kkbZ nsxkA

A B C D
Sol. (b)
When the transparent sheet of paper is folded at the dotted
line, it will appear as shown below

A B C D
25. Select the option that depicts how the given transparent
Sol. (b)
sheet of paper would appear if it is folded at the dotted
When the transparent sheet of paper is folded at the dotted
line.
line, it will appear as shown below
ml fodYi dk p;u djsa tks n'kkZrk gS fd dkxt dh nh xbZ ikjn'khZ
23. Select the option that depicts how the given transparent
'khV dks fcanhnkj js[kk ij eksM+us ij dSlk fn[kkbZ nsxkA
sheet of paper would appear if it is folded at the dotted
line.
ml fodYi dk p;u djsa tks n'kkZrk gS fd dkxt dh nh xbZ ikjn'khZ
'khV dks fcanhnkj js[kk ij eksM+us ij dSlk fn[kkbZ nsxkA

A B C D
Sol. (b)
When the transparent sheet of paper is folded at the dotted
line, it will appear as shown below

276
Best App for Govt. Jobs : Neonclasses (Download Now)

A B C D
26. Select the option that depicts how the given transparent Sol. (d)
sheet of paper would appear if it is folded at the dotted The line from which the paper is folded. When it is unfolded
line. mirror image is formed.
ml fodYi dk p;u djsa tks n'kkZrk gS fd dkxt dh nh xbZ ikjn'khZ
'khV dks fcanhnkj js[kk ij eksM+us ij dSlk fn[kkbZ nsxkA

D
29. A paper is folded and cut as shown below. How will it
appear when unfolded?
,d dkxt dks uhps fn[kk, x, rjhds ds vuqlkj eksM+k vkSj dkVk x;k
gSA ;g dkxt [kksyus ij dSlk fn[ksxk\

A B C D
Sol. (d)
When the transparent sheet of paper is folded at the dotted
line, it will appear as shown below P X Y Z

(X)
27. Select the option that depicts how the given transparent Sol. (a)
sheet of paper would appear if it is folded at the dotted The line from which the paper is folded. When it is unfolded
line. mirror image is formed.
ml fodYi dk p;u djsa tks n'kkZrk gS fd dkxt dh nh xbZ ikjn'khZ
'khV dks fcanhnkj js[kk ij eksM+us ij dSlk fn[kkbZ nsxkA

30. A paper is folded and cut as shown below. How will it


appear when unfolded?
,d dkxt dks uhps fn[kk, x, rjhds ds vuqlkj eksM+k vkSj dkVk x;k
gSA ;g dkxt [kksyus ij dSlk fn[ksxk\

A B C D
Sol. (a)
When the transparent sheet of paper is folded at the dotted
line, it will appear as shown below X Y Z

28. A paper is folded and cut as shown below. How will it A B C D


appear when unfolded? Sol. (b)
,d dkxt dks uhps fn[kk, x, rjhds ds vuqlkj eksM+k vkSj dkVk x;k The line from which the paper is folded. When it is unfolded
gSA ;g dkxt [kksyus ij dSlk fn[ksxk\ mirror image is formed.

P X Y Z 31. A paper is folded and cut as shown below. How will it


appear when unfolded?

277
Download Free PDFs & e-Books from Neon Classes App

,d dkxt dks uhps fn[kk, x, rjhds ds vuqlkj eksM+k vkSj dkVk x;k
gSA ;g dkxt [kksyus ij dSlk fn[ksxk\

A B C D
X Y Z Sol. (c)
The line from which the paper is folded. When it is unfolded
mirror image is formed.

A B

34. Select the option that depicts how the given transparent sheet
of paper would appear if it is folded at the dotted line.
C D
Sol. (c)
ml fodYi dk p;u djsa tks n'kkZrk gS fd dkxt dh nh xbZ ikjn'khZ
The line from which the paper is folded. When it is unfolded
'khV dks fcanhnkj js[kk ij eksM+us ij dSlk fn[kkbZ nsxkA
mirror image is formed.

32. A paper is folded and cut as shown below. How will it


appear when unfolded?
,d dkxt dks uhps fn[kk, x, rjhds ds vuqlkj eksM+k vkSj dkVk x;k
gSA ;g dkxt [kksyus ij dSlk fn[ksxk\ A B C D
Sol. (d)
When the transparent sheet of paper is folded at the dotted
line, it will appear as shown below

X Y Z

PUZZLE
The question based on puzzle can be classified into the
A B C D following categories.
Sol. (d) ity ij vk/kkfjr iz’uksa dks fuEu esa oxhZd`r fd;k tk ldrk gS &
The line from which the paper is folded. When it is unfolded 1. Linking Puzzle ¼fyafdax ity½
mirror image is formed. 2. Grouping puzzle/xzqfiax ity½
3. Comparison puzzle¼dEisfjtu ity½
4. Floor puzzle ¼¶yksj ity½
5. Box puzzle ¼ckWDl ity½
6. Mixed puzzle ¼feDLM ity½
We generally use column method to solve questions based on
33. A paper is folded and cut as shown below. How will it puzzle.
appear when unfolded? ity ij vk/kkfjr iz’uksa dks gy djus ds fy, lkekU;r% dkWye fof/k
,d dkxt dks uhps fn[kk, x, rjhds ds vuqlkj eksM+k vkSj dkVk x;k dk iz;ksx djrs gSA
gSA ;g dkxt [kksyus ij dSlk fn[ksxk\ (1) First, read the given question carefully and after it, make a
column. If dates are given in the question then we will make a
column of dates.
lcls igys fn, x, iz’u dks /;kuiwoZd i<s+ vksj mlds ckn dk dkWye
cuk,A ;fn iz’u eas rkjh[ksa nh xbZ gS rks rkjh[kksa dk dkWye cusxkA
P X Y Z If days are given in the question then a column of days will be
created. If both date and day are not given in the question,
columns will be created on the basis of names of persons.

278
Best App for Govt. Jobs : Neonclasses (Download Now)

;fn iz’u esa fnu fn, x, gS rks fnuksa dk dkWye cusxkA iz’u esa P, Q, R, S, T, U and V are seven professors. Each one teaches a
fnuk¡d vksj fnu nksuksa gh ugh fn, tkus ij O;fDRk;ksa ds uke ds different subject from Physics, Chemistry, Biology, English,
vk/kkj ij dkWye cuk, tk,axsA Mathematics, Economics and Geography, not necessarily in the
(2) We will divide the information given in the question into two same order. Each of them teaches one day each on the seven
parts: Direct and indirect. Statements that are favourable or days of the week from Monday to Sunday not necessarily in
positive will be written in the right place of the column and the the same order. R teaches Biology on Friday. Q teaches
statements that are negative will be written in the left place of Mathematics on the previous day of the day on which the
the column. We will write the indirect information as hints and professor teaches Physics. V teaches on Sunday but does not
will use them as needed. teach Chemistry or English. S teaches Economics on the
First, we will solve the questions based on the linking puzzle. previous day on which U teaches. P teaches Geography on
iz’u esa nh xbZ lwpukvksa dks nks Hkkxksa esa ck¡Vsaxs& izR;{k vkSj vizR;{kA Tuesday. T does no teach English.
izR;{k lwpukvksa esa tks dFku vuqdwy gks ;k ldkjkRed] mUgsa dkWye funs’Z k ¼1--5½ P, Q, R, S, T, U vkSj V lkr izksQslj gSA buesa ls izR;sd
ds nk,¡ LFkku ij fy[ksx a s vkSj tks dFku udkjkRed ;k izfrdwy gks] ,d fHkUu fo"k; i<+krk gS& HkkSfrd foKku] jlk;u foHkku] tho foKku]
mUgsa dkWye ds ck,¡ LFkku ij fy[ksaxsA vxzsath] xf.kr] vFkZ’kkL= vkSj Hkwxksy] ;g t:jh ugha fd blh Øe esAa
vizR;{k lwpukvksa dks fgaV ds :i esas fy[k ysaxs vkSj vko’;drkvuqlkj buesa ls izR;sd lkseokj ls jfookj rd lIrkg ds ,d fnu i<+krk gS]
mUgsa iz;ksx djsaxsA t:jh ugh fd blh Øe esaA
lcls igys] ge ÇyÇdx it+y ij vkèkkfjr ç'uksa dks gy djsaxsA 'kqØokj dks R thc foKku i<+krk gSA izksQslj ftl fnu HkkSfrd
Basic steps to solve Logical Puzzles foKku i<+krs gS mlds iwoZ fnol ij Q xf.kr i<+krs gSA V jfookj dks
rkfdZd igsyh dks gy djus ds fy, cqfu;knh dne i<+krs gS fdarq og jlk;u foKku ;k vxzsath ugha i<+krsA U ftl fnu
1. Take a quick look at the question. i<+krs gS mlds iwoZ fnol ij S vFkZ’kkL= i<+krs gSA eaxyokj dks P
ç'u ij ,d Rofjr utj MkysAa Hkwxksy i<+kbZ gSA T vxzath ugha i<+krs gSA
2. Develop a general idea regarding the theme of the problem. bu lwpvuksa ds vk/kkj ij fuEufyf[kr iz’uksa ds mÙkj nhft, &
leL;k ds fo"k; ds ckjs esa ,d lkekU; fopkj fodflr djukA (1) On Which day does U teach?
3. Select the data that is giving you some concrete information U fdl fnu i<+krs gS\
out of total information given. Also, select the data which helps (A) Wednesday ¼cq/kokj½
in ruling out certain possibilities. (B) Thursday ¼xq:okj½
ml MsVk dk p;u djsa tks vkidks nh xbZ dqy tkudkjh esa ls dqN (C) Wednesday or Thursday
Bksl tkudkjh ns jgk gSA blds vykok] MsVk dk p;u djsa tks dqN ¼cq/kokj ;k xq:okj½
laHkkoukvksa dks iwjk djus esa enn djrk gSA (D) None of these ¼buesa ls dksbZ ugha½
4. Useful secondary information: Generally, the first few (2) Which subject does V teach?
sentences of the given data give you the basic information that V dkSulk fo"k; i<+krs gS\
is required to know the general idea of the problem. This is (A) Chemistry ¼jlk;u foKku½
called the ‘useful secondary information’. (B) English ¼vxzt as h½
mi;ksxh ek/;fed tkudkjh: vke rkSj ij] fn, x, MsVk ds igys dqN (C) Biology ¼tho foKku½
okD; vkidks ewy tkudkjh nsrs gSa tks leL;k ds lkekU; fopkj dks (D) Physics ¼HkkSfrd foKku½
tkuuk vko';d gSA bls 'mi;ksxh ek/;fed tkudkjh' dgk tkrk gSA (3) Which subject does T teach?
5. Actual information: Data left after putting aside the useful T dkSulk fo"k; i<+krs gS\
secondary information is called actual information. While (A) Chemistry ¼jlk;u foKku½
trying to solve the problem, you should begin with the actual (B) Physics ¼HkkSfrd foKku½
information while the useful secondary information should be (C) Biology ¼tho foKku½
kept in mind.
(D) None of these ¼buesa ls dksbZ ugha½
okLrfod tkudkjh: mi;ksxh ek/;fed tkudkjh dks vyx j[kus ds
(4) On Which day does V teach?
ckn NksM+s x, MsVk dks okLrfod tkudkjh dgk tkrk gSA leL;k dks V fdl fnu i<+krs gS\
gy djus dh dksf'k'k djrs le;] vkidks okLrfod tkudkjh ls 'kq: (A) Wednesday ¼cq/kokj½
djuk pkfg, tcfd mi;ksxh ek/;fed tkudkjh dks /;ku esa j[kuk (B) Thursday ¼xq:okj½
pkfg,A (C) Monday ¼lkseokj½
6. Negative information: Some parts of the actual information
(D) Sunday ¼jfookj½
may consist of negative information. Negative information
(5) Which subject does U teach?
does not inform us anything exactly but it gives us a chance to
U dkSulk fo"k; i<+krs gS\
eliminate a possibility. Sentences like “P is not the mother of
(A) Physics ¼HkkSfrd foKku½
Q” or “B is not a hill-station” are called negative information.
udkjkRed tkudkjh: okLrfod tkudkjh ds dqN fgLlksa esa udkjkRed (B) English ¼vxzt as h½
tkudkjh 'kkfey gks ldrh gSA udkjkRed tkudkjh gesa okLro esa (C) Geography ¼Hkwxksy½
dqN Hkh lwfpr ugha djrh gS ysfdu ;g gesa ,d laHkkouk dks [kRe (D) Economics ¼vFkZ’kkL=½
djus dk ekSdk nsrh gSA "P] Q dh ek¡ ugha gS" ;k "B ,d fgy&LVs'ku
ugha gS" tSls okD; dks udkjkRed tkudkjh dgk tkrk gSA

Concept –I: Linking Puzzle


Directions(1-5). Study the following information carefully and
answer the question given below:

279
Download Free PDFs & e-Books from Neon Classes App

Sol: ;gk¡ fnuksa ds uke ds vk/kkj ij dkWye cuk, tk,¡xs Hence, on Thursday she attend singing classes.
Negative st. Day
Profe
Subject 7. There are 3 badminton players in each stream – Commerce,
ssor
Science and Arts. A, B and C are from commerce. P, Q and R are
Monday T Chemistry
Tuesday P Geography from Science. J, K and L are from Arts. A team of 4 badminton
Wednesday S Economics players are to be sent for a tournament with following
Thursday U English
conditions:
Friday R Biology
Saturday Q Mathematics çR;sd LVªhe esa 3 cSMfeaVu f[kykM+h gSa& okf.kT;] foKku vkSj dykA A,
Chemistry, English Sunday V Physics B vkSj C okf.kT; ls gSaA P, Q vkSj R foKku ls gSaA J, K vkSj L dyk ls gSaA
fgaV% (1) Q & Mathematics ¼xf.kr½ 4 cSMfeaVu f[kykfM+;ksa dh ,d Vhe dks fuEufyf[kr 'krksZa ds lkFk ,d
Physics¼HkkSfrd foKku½ VwukZeaVs ds fy, Hkstk tkuk gS
(2) S & Economics¼vFkZ’kkL=½ (i) There must be at least one student from each stream.
U çR;sd LVªhe ls de ls de ,d Nk= gksuk pkfg,A
(3) T doesn’t teach English. (ii) P will not go with Q or R.
T vxzt sa h ugha i<+krsA P, Q ;k R ds lkFk ugha tk,xkA
(1) U teaches on Thursday. (iii) A and C will always go together.
U xq:okj dks i<+krs gSA A vkSj C ges'kk lkFk&lkFk pysaxsA
(2) V teaches Physics. (iv) K and L will always go together.
V HkkSfrd foKku i<+krs gSA K vkSj L ges'kk lkFk&lkFk pysx
a sA
(3) T teaches Chemistry. Which of the following combinations of players is a possible
T jlk;u foKku i<+krs gSA combination for the tournament?
(4) V teaches on Sunday. f[kykfM+;ksa dk fuEufyf[kr esa ls dkSu lk la;kstu VwukZeVsa ds fy, ,d
V jfookj dks i<+krs gSA laHkkfor la;kstu gS\
(5) U teaches English. (a) A, C, P and J (b) A, Q, R and J
U vxzt sa h i<+krs gSA (c) B, Q, J and K (d) B, P, R and J
6. Vishakha joined five different hobby classes viz. painting, Sol. (a)
Dancing, Modelling, Singing and Cooking. She attends each  Commerce: A, B, C
class on different day of the same week from Monday to Friday Science: P, Q, R
but not necessarily in the same order. She attends Modelling Arts: J, K, L
class on Wednesday. She attends neither painting nor Dancing  4 badminton player team selected for tournament.
class on Thursday and Friday. She attends cooking class on  At least one student from each stream.
Friday.
On which day does she attend singing class?
 P will not go with Q or R
fo'kk[kk ikap vyx&vyx g‚ch d{kkvksa esa 'kkfey gqbZA isfa Vax] Mkaflax]  A and C will always go together
e‚Mfyax] flafxax vkSj dqfdaxA og lkseokj ls 'kqØokj rd ,d gh  K and L will always go together.
lIrkg ds vyx&vyx fnu çR;sd d{kk esa Hkkx ysrh gS ysfdu t:jh Commerce Science Arts
ugha fd blh Øe esa gksA og cqèkokj dks e‚Mfyax Dykl vVsM a djrh B/(A + C) P or (Q/R) J or (K + L)
gSA og xq#okj vkSj 'kqØokj dks u rks isfa Vax vkSj u gh Mkaflax Dykl  Hence, possible team is P, A, C, J
esa tkrh gSA og 'kqØokj dks dqfdax Dykl esa tkrh gSA og xk;u dh 8. 1. There are four friends Punit, Kaushal, Rakhi and Jagjeet.
d{kk esa fdl fnu mifLFkr gksrh gS\ pkj nksLr iquhr] dkS'ky] jk[kh vkSj txthr gSaA
(a) Tuesday 2. Punit and Kaushal play rugby and circket.
(b) Monday iquhr vkSj dkS'ky jXch vkSj fdØsV [ksyrs gSaA
(c) Thursday 3. Kaushal and Rakhi play rugby and badminton.
(d) Data is inadequate. dkS'ky vkSj jk[kh jXch vkSj cSMfeaVu [ksyrs gSaA
Sol. (c) 4. punit and Jagjeet play cricket and basketball.
 She attends modelling on Wednesday. iquhr vkSj txthr fØdsV vkSj ckLdsVc‚y [ksyrs gSaA
5. Rakhi and Jagjeet play badminton and basketball.
 She attends cooking on Friday.
jk[kh vkSj txthr cSMfeaVu vkSj ckLdsVc‚y [ksyrs gSaA
Day Colour
Who among the following plays circket, basketball, and
Monday
badminton?
Tuesday Modelling
fuEufyf[kr esa ls dkSu fdØsV] ckLdsVc‚y vkSj cSMfeaVu [ksyrk gS\
Wednesday Cooking
(a) Punit (b) Kaushal
Thrusday
(c) Rakhi (d) jagjeet
Friday
Sol. (d)
 She neither painting nor dancing on Thrusday and Friday.
 Punit and Kaushal play Rugby and Cricket.
Day Colour
Monday Painting/dancing  Punit and Jagjeet play Cricket and Basketball
Tuesday Dancing/painting  Kaushal and Rakhi play Rugby and Badminton
Wednesday Modelling  Rakhi and Jagjeet play Badminton and Basketball.
Thrusday Singing Person Games
Friday Cooking Punit Rugby, Cricket, Basketball

280
Best App for Govt. Jobs : Neonclasses (Download Now)

Kaushal Rugby, Cricket, Badminton Sol. (a)


Rakhi Rugby, Badminton,  Tina like Science but not like Hockey.
Jagjeet Basketball  Arshil like Badminton but not like Computer or English.
Badminton, Basketball,
Cricket
 Tina like Football.
 Hence, Jagjeet play Badminton, Cricket, Basketball.  Puneet like Cricket.
9. Among A, B, C, D, E and F, only one person likes the colour  Gautam like Hockey but not like English or Maths.
Black, among them, only two study in class 12th, while the rest Person Subject Sport
study in class 10th. Only C is in the same class as F. D likes the Puneet Cricket
colour Pink. None of those studying in 12th like the colour Arshil Science Badminton
Black. Neither A nor E likes the colour Black. Who among them Gautam Hockey
likes the colour Black? Jatin Football
A, B, C, D, E vkSj F esa ls dsoy ,d O;fä dks dkyk jax ilan gS] muesa Tina
ls dsoy nks d{kk 12 oha esa i<+rs gSa] tcfd ckdh d{kk 10 oha esa i<+rs  One like Hindi also like Basketball.
gSaA dsoy C, F ds leku d{kk esa gSA D dks xqykch jax ilan gSA 12oha esa Person Subject Sport
i<+us okyksa esa ls fdlh dks Hkh dkyk jax ilan ugha gSA u A u gh E dks Puneet English Cricket
dkyk jax ilan gSA muesa ls dkSu dkyk jax ilan djrk gS\ Arshil Maths Badminton
(a) B (b) F (c) C (d) A Gautam Computer Hockey
Sol. (a) Jatin Hindi Basketball
 Only one person like black colour. Tina Science Football
 Neither A nor E like black colour.
 D likes pink colour. 11. A student has exactly eight books A, B, C, D, E, F, G and H, from
which he must choose exactly five to be put in a bag. Any
 So A, E, D not likes black colour. combination is acceptable, provided it meets the following
 Out of 6 only two study in 12th class. while conditions:
 Only C is in the same class as F so C and F definatly in class ,d Nk= ds ikl Bhd vkB iqLrdsa A] B] C] D] E] F] G vkSj H gSa]
12th. ftuesa ls mls ,d cSx esa j[kus ds fy, Bhd ikap iqLrdsa pquuh gksx a hA
 None of those studying in 12th like black so, C and F not like dksbZ Hkh la;kstu Lohdk;Z gS] c'krZs og fuEufyf[kr 'krksZa dks iwjk djrk
black. gks
Nuber Box If A is chosen, F cannot be chosen.
A 10th ;fn A dks pquk tkrk gS] rks F dks ugha pquk tk ldrk gSA
B Black 10th If B is choosen, E must also be chosen.
C Pink 12th ;fn B pquk tkrk gS] rks E dks Hkh pquk tkuk pkfg,A
D 10th If D is chosen, G must also be chosen.
E 10th ;fn D pquk tkrk gS] rks G dks Hkh pquk tkuk pkfg,A
F 12th Choose from the given options below, the combination of
books which satisfies given three conditions.
10. Five frineds, Puneet, Jatin, Gautam, Arshil and Tina, Like five uhps fn, x, fodYiksa esa ls mu iqLrdksa ds la;kstu dks pqfu, tks nh
different subjects namely Maths, English, Science, computer xbZ rhu 'krksZa dks iwjk djrs gSaA
and Hindi, but not necessarily in the same order. Each of the (a) B, D,E, F, G (b) B, C, D, F, G
friends also like one sport, namely Cricket, Hockey, (c) A, B, D, E, F (d) A, B, C, D, G
Badminton, Football and Basketball, such that no two has the Sol. (a)
same favourtie sport. From eight books exactly five books to be put in a bag
ikap fe=] iquhr] tfru] xkSre] v'kÊy vkSj Vhuk] ikap vyx&vyx  If A chosen, F cannot be chosen.
fo"k;ksa tSls xf.kr] vaxzsth] foKku] daI;wVj vkSj fganh dks ilan djrs gSa] A/ F
ysfdu t:jh ugha fd blh Øe esa gksAa çR;sd fe= dks Hkh ,d [ksy  If B is chosen, E must also chosen.
ilan gS] tSls fd fØdsV] g‚dh] cSMfeaVu] QqVc‚y vkSj ckLdsVc‚y] fd B E together
fdlh Hkh nks dk ilanhnk [ksy leku ugha gSA
 If D is chosen, G must also chosen.
Tina like Science but does not like Hockey. The one who likes
D G together
Hindi also likes Basketball Arshil like badminton but does not
like Computer or English. Gautam likes Hockey but does not 2 possible group  A B E D G
like English or maths. Tina likes Football. Puneet likes Cricket. FBEDG
Who likes Hindi? From options 2nd group is correct.
Vhuk dks foKku ilan gS ysfdu g‚dh ilan ugha gSA og O;fä ftls 12. Five Students A, B, C, D and E study in different schools K, L, M,
fganh ilan gS mls ckLdsVc‚y ilan gS v'kÊy dks cSMfeaVu ilan gS N and O, but not necessarily in the same order. Each one likes
ysfdu daI;wVj ;k vaxzsth ilan ugha gSA xkSre dks g‚dh ilan gS only one subject, Viz., Hindi, Mathematics, Science, Social
ysfdu vaxzsth ;k xf.kr ilan ugha gSA Vhuk dks QqV c‚y ilan gSA Science and English. C studies in M. B does not like Social
iquhr dks fØdsV ilan gSA fganh fdls ilan gS\ Science. D like English and studies in N. The students of L likes
(a) Jatin (b) Arshil Math. E likes hindi but is neither from O nor from L. B studies
(c) Puneet (d) Gautam

281
Download Free PDFs & e-Books from Neon Classes App

in O. Which one of the following students studies in school L (A) Monday ¼lkseokj½
and likes Mathematics? (B) Tuesday ¼eaxyokj½
ikap Nk= A, B, C, D vkSj E vyx&vyx Ldwyksa K, L, M, N vkSj O esa i<+rs (C) Wednesday ¼cq/kokj½
gSa] ysfdu t:jh ugha fd blh Øe esa gksAa çR;sd O;fä dks dsoy ,d (D) Thursday ¼xq:okj½
gh fo"k; ilan gS] vFkkZr fganh] xf.kr] foKku] lkekftd foKku vkSj (14) How many subjects are taught between Botany and Zoology?
vaxzsthA C, M esa i<+rk gSA B dks lkekftd foKku ilan ugha gSA D dks ouLifr’kkL= vkSj tUrqfoKku ds chp fdrus fo"k; i<+k, tkrs gS\
vaxzsth ilan gS vkSj og N esa i<+rk gSA L ds Nk= dks xf.kr ilan gSA E (A) 1 (B) 2
dks fganh ilan gS ysfdu og u rks O ls gS vkSj u gh L ls gSA B, O esa (C) 3 (D) 4
i<+rk gSA fuEufyf[kr esa ls dkSu lk Nk= Ldwy L esa i<+rk gS vkSj mls (15) Which of the following subjects is taught on Saturday?
xf.kr ilan gS\ 'kfuokj dks fuEu esa ls dkSulk fo"k; i<k;k tkrk gS\
(a) B (b) A (c) D (d) C (A) Botany ¼ouLifr’kkL=½
Sol. (b) (B) Statistics ¼lkaf[;dh½
 C studies in M. (C) Zoology ¼tUrq foKku½
 D studies in N and like English. (D) Mathematics ¼xf.kr½
(16) On which of the following days is Statistics taught ?
 E likes Hindi but is neither from O nor from L
lkaf[;dh fuEu esa ls fdl fnu i<+kbZ tkrh gS\
 B studies in O (A) Tuesday ¼eaxyokj ½
 B does not like Social Science. (B) Wednesday ¼cq/kokj½
Student School Subject (C) Thursday ¼xq:okj½
A (D) Friday ¼'kqØokj½
B O English (17) If Statistcs is related to Zoology and Physics is related to
C M Hindi Botany in certain way, then to which of the following would
D N Chemistry be related, following the same pattern?
E ;fn lkaf[;dh dk tUrq foKku ls vkSj HkkSfrdh dk ouLifr’kkL= ls
 L like maths. ,d [kkl rjg dk laca/k gS rks iSVuZ ds vuqlkj jlk;u’kkL= dk laca/k
 Remaining school is K. fuEu esa ls fdlds lkFk gksxk\
Student School Subject (A) Maths ¼xf.kr½
A L Maths (B) Statictics ¼lkaf[;dh½
B O Science (C) Physics ¼HkkSfrdh½
C M Social- (D) English ¼vxzt as h½
D N Scien Sol: Here we will create a column of days. We will get two
E K ce possibilities from the given information.
English ;gk¡ fnuksa ds uke ds vk/kkj ij dkWye cuk, tk,axsA
Hindi nh xbZ lwpukvksa ds vk/kkj ij fuEu nks laHkkouk,¡ cusxh &
Hence, A is studies in school L and like Maths.
Directions(13-17). Study the following information Negative Day Subject Day
Monday
Subject
St. Monday Physics
carefully and answer the question given below: (English Tuesday Zoology
Tuesday
Botany
Wednesday
Wednesday English Maths
One of the seven subjects viz. Maths, Zoology, Botany, )
Thursday Chemistry
Thursday
Chemistry
Friday
Chemistry, Physics, English and Statistics is taught in one day Friday
(English Saturday
Maths
Botany
Saturday
Statistics
Zoology
Sunday
each in a week starting form Monday and ending on Sunday. ) Sunday English

Chemistry is taught on Thursday. English is taught on the day Possibility–(1) Possibility–(2)

immediately next to the day when zoology is taught. English is Hint-(1) Zoology
taught neither on Tuesday nor on Saturday. Only one lectrure english
is held between Chemistry and Botany. Two lectures are held Possibility-(1) is false because Statistics is neither read on Monday
between Chemistry and Botany. Two leacrtures are scheduled nor on Sunday.
between maths and Zoology. Statistcis is taught neither on Possibility-(2) is correct and answer all the questions accordingly.
Monday nor on Sunday. (13) Physics is taught on Monday.
funs’Z k ¼13-17½ lkr fo"k;ksa ;Fkk xf.kr] tarq foKku] ouLifr 'kkL=] (14) Botany and 3 classes between Zoology; are read;
jlk;u 'kkL=] HkkSfrfd] vxzt as h vkSj lk¡f[;dh esa ls ,d lIrkg esa ,d (15) Animal science is taught on Saturdays.
fnu i<+k;k tkrk gS vkSj lIrkg lkseokj ls vkjaHk gksdj jfookj dks (16) Statistics is read on Friday.
lekIr gksrk gSA jlk;u 'kkL= xq:okj dks i<+k;k tkrk gSA (17) Their on the next day of Statistics, on the next day of Zoology
tUrq foKku ftl fnu i<+k;k tkrk gS mlds vxys fnu vxzsath i<+kbZ and Physics, Botany is read.
tkrh gSA vxzt sa h u rks eaxyokj dks] u gh 'kfuokj dks i<+kbZ tkrh Similarly, on the second day of Chemisry, the reading of Statistics is
gSA jlk;u 'kkL= vkSj ouLifr 'kkL= ds chp dsoy ,d O;k[;ku gSA done.
xf.kr vkSj tarq foKku] ds chp esa nks O;k[;ku gSA lkaf[;dh u rks fgaV&(1) tUrq foKku
lkseokj dks vkSj u gh jfookj dks i<+kbZ tkrh gSA vxzt sa h
bu lwpukvksa ds vk/kkj ij fuEufyf[kr iz’uksa ds mÙkj nhft,& laHkkouk&(1) xyr gS D;ksfa d lkaf[;dh u rks lkseokj dks u gh
(13) One which of the following days is Physics taught?
jfookj dks i<+kbZ tkrh gSA
HkkSfrdh fuEu esa ls fdl fnu i<+kbZ tkrh gS\ laHkkouk&(2) lgh gS vkSj blh ds vuqlkj lHkh iz’uksa dk mÙkn nsx
a sA
(13) HkkSfrdh lkseokj dks i<+kbZ tkrh gSA

282
Best App for Govt. Jobs : Neonclasses (Download Now)

(14) ouLifr 'kkL= vkSj tarq foKku ds chp esa 3 fo"k; i<+k;s tkrs gSA  None of those studying in 12th like black so, C and F not like
(15) 'kfuokj dks tUrq foKku i<+k;k tkrk gSA black.
(16) lkaf[;dh 'kqØokj dks i<+kbZ tkrh gSA Nuber Box
(17) ;gk¡ lkaf[;dh ds vxys fnu tUrq foKku vkSj HkkSfrdh ds vxys fnu A 10th
ouLifr’kk= i<+k;k tkrk gSA B Black 10th
blh izdkj jlk;u 'kkL= ds vxys fnu lkaf[;dh i<++kbZ tkrh gSA C Pink 12th
18. Five girls, A, B, C, D and E, travel to only one of the following D 10th
five cities: Mumbai, Delhi, Kolkata, Chennai and Jaipur. E does E 10th
not travel to Kolkata, Mumbai or Chennai. D does not travel to F 12th
Mumbai, Chennai or Delhi. A travel to Jaipur. C does not travel Concept – II: Grouping Puzzle :
to Mumbai. To which city does C travel? Directions(20-24). Study the following information
ikap yM+fd;ka] A, B, C, D vkSj E, fuEufyf[kr ikap 'kgjksa esa ls dsoy carefully and answer the questions given below:
,d dh ;k=k djrh gSa& eqacbZ] fnYyh] dksydkrk] psUubZ vkSj t;iqjA A, B, C, D, E, F and G are seven friends travelling by three cars
E dksydkrk] eqacbZ ;k psUubZ dh ;k=k ugha djrk gSA D eqacbZ] psUubZ X, Y and Z. There are at least two friends in each car. Out of the
;k fnYyh dh ;k=k ugha djrk gSA A t;iqj dh ;k=k djrk gSA C seven friends, there are three women. Each is in the different
eqacbZ dh ;k=k ugha djrk gSA C fdl 'kgj dh ;k=k djrk gS\ car. B travels with only his best friend G who is a woman in car
(a) Kolkata/dksydkrk Y. A travels by car X. E and F do not travel by car Z. C is not a
(b) Chennai/psUubZ woman where as F is a woman.
(c) Jaipur/t;iqj funs’Z k ¼20-24½ A, B, C, D, E, F rFkk G lkr fe= X, Y rFkk Z rhu dkj
(d) Delhi/fnYyh ls ;k=k dj jgs gSA izR;sd dkj esa de ls de nks fe= lokj gSA buesa
3 efgyk;s gS rks izR;sd vyx dkj esa gSA B dsoy viuh lcls vPNh
Sol. (b)
efgyk nksLr G ds lkFk dkj Y esa lokjh djrk gSA A dkj X esa ;k=k
 A travel to Jaipur.
djrk gSA E rFkk F dkj Z esa ;k=k ugh djrsA C efgyk ugha gS]
 C does not travel to Mumbai. tcfd F efgyk gSA bl lwpukvksa ds vk/kkj ij fuEufyf[kr iz’uksa ds
 E does not travel to Kolkata, Mumbai or Chenni. mÙkj nhft, &
 D does not travels to Mumbai, Chennai, or Delhi. (20) By which car does C travels?
A Jaipur C fdl dkj esa ;k=k djrk gS\
B (A) Y (B) X
C (C) Z
D (D) None of these ¼buesa ls dksbZ ugh½
E (21) By which car do three friends travel?
 So, B travel to Mumbai, C travel to Chennai, D travel to fdl dkj esa rhu fe= ;k=k djrs gS\
Kolkata, E travel to Delhi. (A) X (B) Y
A Jaipur (C) Z
B Mumbai (D) None of these ¼buesa ls dksbZ ugh½
C Chennai (22)*By Which car does E travel ?
D Kolkata E fdl dkj esa ;k=k djrk gS\
E Delhi (A) Y (B) X
Hence, C travels to Chennai. (C) Z
19. Among A, B, C, D, E and F, only one person likes the colour (D) None of these ¼buesa ls dksbZ ugh½
Black, among them, only two study in class 12th, while the rest (23) Which of the following groups indicates women?
study in class 10th. Only C is in the same class as F. D likes the fuEu esa ls dkSulk lewg rhu efgykvksa dks O;Dr djrk gSa\
colour Pink. None of those studying in 12th like the colour (A) FGD (B) EGD
Black. Neither A nor E likes the colour Black. Who among them (C) EBD
likes the colour Black? (D) None of these ¼buesa ls dksbZ ugh½
A, B, C, D, E vkSj F esa ls dsoy ,d O;fä dks dkyk jax ilan gS] muesa (24) Which of the following car-person pairs is wrong?
ls dsoy nks d{kk 12 oha esa i<+rs gSa] tcfd ckdh d{kk 10 oha esa i<+rs fuEu esa ls dkSulk O;fDr&dkj dk tksM+k xyr gSa\
gSaA dsoy C, F ds leku d{kk esa gSA D dks xqykch jax ilan gSA 12oha esa (A) X-A (B) Y-G
i<+us okyksa esa ls fdlh dks Hkh dkyk jax ilan ugha gSA U rks A U gh E (C) Z-D (D) X-D
dks dkyk jax ilan gSA muesa ls dkSu dkyk jax ilan djrk gS\ Sol : Here, Columns will be formed on the basis of cars and the
(a) B (b) F (c) C (d) A following arrangement will be obtained from the given
Sol. (a) information.
 Only one person like black colour. ;gk¡ dkj ds vk/kkj ij dkWye cuk, tk,axs vkSj nh xbZ lwpukvksa ls
 Neither A nor E like black colour. fEu O;oLFkk izkIr gksxh&
 D likes pink colour.
 So A, E, D not likes black colour.
 Out of 6 only two study in 12th class while class.
 Only C is in the some class as F so C and F definatly in class
12th.

283
Download Free PDFs & e-Books from Neon Classes App

Car Person (C) D, B, F (D) B, E, F


(29) Which of the following members of familes X & Y travel in the
X A E . F
same car?
B G
ifjokj X vkSj Y ds fuEufyf[kr esa ls dkSuls lnL; ,d gh dkj esa
Y
;k=k dj jgs gS\
Z C D (A) C, F (B) D, F
(C) C, D (D) D, E
(20) C travels by car Z. Sol : There will be separate columns for gourp of cars and families.
C, Z dkj esa ;k=k djrk gSA ;gk¡ ifjokj vkSj dkj ds lewg ds vyx&vyx dkWye cusaxs &
(21) There are three friends in car X.
dkj X esa rhu fe= gSA Family Person
(22) E travels by car X.
E, dkj X esa ;k=k djrk gSA A D . E
(23) F, G & D are women.
B A , F , C
F, G o D rhuksa efgyk,¡ gSA
(24) D does not travel by car X. Therefore it is a wrong pair. C B . G , H
dkj X esa D ;k=k ugh djrk gSA blfy, xyr gSA
. Directions(25-29). Study the following information Family X– D + E
carefully and answer the questions given below:
Family X– F + A
Eight members A, B, C, D, E, F, G and H belonging to three
families X, Y, Z go for weekend outing in three different cars I, C
II, III. Four out of eight memebers are females. Members of Family Z–B + G
each family travel in different cars. Each car has at least one
H
male and one female member. Each family has at least two
members. Car Person
A belongs to family Y and he travels in car III. D is wife of E and D , F , H
I
they travel in cars I & II respectively. H is son of B who is wife
of G and they belong to family Z. C is daughter of F who is wife
II E , C , G
of A. C travels in car II. G does not travel with F.
funs’Z k ¼25-29½ rhu ifjokj X, Y, Z ds vkB lnL; A, B, C, D, E, F, G, III A , B
vkSj H, rhu vyx&vyx dkj I, II, III esa lIrkgkar Hkze.k ds fy, tkrs
gSA vkB esa ls pkj efgyk lnL; gSA fdlh Hkh ,d ifjokj ds lnL; (25) D, F and H travel by car I.
vyx&vyx dkjksa esa ;k=k djrs gSA izR;s dkj esa de ls de ,d dkj I esa D, F vkSj H ;k=k dj jgs gSA
iq:"k vkSj ,d efgyk lnL; gSA izR;sd ifjokj esa de ls de nks (26) There are two members in car III.
lnL; gSA dkj III esa dsoy 2 lnL; gSA
(27) F, who belongs to famiy Y and G, who belongs to family Z, are
A ifjokj Y ls gS vkSj dkj III esa ;k=k djrk gSA D, E dh iRuh gS
travelling in the different cars.
vkSj os Øe’k% dkj I vkSj II esa ;k=k djrs gSA H, B dk iq= gS tks G
ifjokj Y dk lnL; F vkSj ifjokj Z dk lnL; G vyx&vyx dkj
dks iRuh gS vkSj ;s ifjokj Z ls gSA C, F dh iq=h gS tks A dh iRuh
esa gSA
gSA C dkj II esa ;k=k djrh gSA G, F ds lkFk ;k=k ugha djrk gSA bu
(28) D,B & F are females.
lwpukvksa ds vk/kkj ij fuEufyf[kr iz’uksa ds mÙkj nhft, & D, B o F rhuksa efgyk,¡ gSA
(25) Which of the following group of persons travel in car I ?
(29) D, who belongs to family X and F, who belongs to family Y, are
dkj I esa fuEufyf[kr esa ls fdu O;fDr;ksa dk lewg ;k=k dj jgk gSa\
travelling in the same car.
(A) D, F, G (B) D, E, G ifjokj X dk lnL; D vkSj ifjokj Y dk lnL; F ,d gh dkj esa
(C) D, G, H(D) D, F, H
gSA
(26) Which car has only two members travelling in it?
Concept-III : Floor Puzzle
fdl dkj esa dsoy nks lnL; ;k=k dj jgs gSa\
30. A, B, C, D, F, and F are six people living in a six-storey building.
(A) I (B) II
With each person living on a different floor. The lower most
(C) III
floor is numbered 1. The floor above it is 2, and so on till the
(D) None of these ¼buesa ls dksbZ ugha½
top most floor is numbered 6.
(27) Which of the following members of families Y & Z travel in
F lives on floor number 3. B and E live on the floors below F's
different cars?
floor. D lives on a floor that is below A's floor but above C's
ifjokj Y vkSj Z ds fuEufyf[kr esa ls dkSuls lnL; vyx&vyx dkj floor. Who among them lives on the top most floor?
esa ;k=k dj jgs gSa\ A] B] C] D] E vkSj F Ng O;fä gSa tks ,d Ng eaftyk bekjr esa jgrs
(A) F, G (B) C, G gSaA IkzR;sd O;fDr ,d vyx eafty ij jgrk gSA lcls fupyh eafty
(C) F, H (D) C, F dh la[;k 1 gSA blds Åij dh eafty 2 gS] vkSj blh rjg lcls
(28) Which of the following group of persons is a group of all
Åijh eafty rd dh la[;k 6 gSA
females?
F eafty la[;k 3 ij jgrk gSA B vkSj E] F ds ry ds uhps okys ry
fuEufyf[kr esa ls O;fDr;ksa dk dkSulk lewg lHkh efgykvksa dk lewg
ij jgrs gSaA D ,d eafty ij jgrk gS tks A dh eafty ds uhps gS
gS\
(A) B, D, G (B) A, B, C

284
Best App for Govt. Jobs : Neonclasses (Download Now)

ysfdu C dh eafty ds Åij gSA muesa ls dkSu lcls Åijh eafty ij 6 P


jgrk gS\ 5 S
(a) D (b) E (c) C (d) A 4 Q
Sol. (d) 3 T
 F lives on floor number 3. 2 R
1 U
 B and E live on the floor below F’s floor.
Hence, U is on floor 1.
Floor Person
32. Seven friends, M, N, O, P, Q. R and S live on seven different
6
floors of the same building, having floors in such a manner that
5 F
the ground floor is numbered 1, the floor above it number 2,
4 B/E
and so on till the top most floor is numbered 7. M lives on floor
3 E/B
number 1. Q lives immediately above M. Only four friends live
2
between the floors of Q and N. Only three friends live between
1
the floors of P and M. R lives immediately below O. Who lives
 D lives on a floor below A’s floor but above C’s floor. on the sixth floor?
Floor Person lkr nksLr] M] N, O] P] Q, R vkSj S ,d gh bekjr ds lkr
6 A vyx&vyx eaftyksa ij jgrs gSa] ftuds Q'kZ bl rjg ls gSa fd Hkwry
5 D dh la[;k 1 gS] blds Åij dh eafty la[;k 2 gS] vkSj blh çdkj
4 C lcls Åijh eafty dh la[;k 7 gksus rd M eafty la[;k 1 ij jgrk
3 F gSA Q] M ds Bhd Åij jgrk gSA Q vkSj N ds ryksa ds chp dsoy
2 B/E pkj fe= jgrs gSaA dsoy rhu fe= P vkSj M ds ryksa ds chp jgrs gSaA
1 E/B
R, O ds uhps jgrk gSA NBh eafty ij dkSu jgrk gS\
31. Six companies, P, Q, R, S, T and U are situated on different
(a) N (b) O (c) S (d) P
floors of a 6 floored building, but not necessarily in the same
Sol. (c)
order. The lowest floor of the building is numbered 1, the floor
above it is numbered as 2, and so on, up to the top floor, which  M lives on the floor number 1.
is numbered as 6. Company Q is not situated at floor number 1.  Q lives immediately above M.
Company P has five companies situated below its floor. There  Only four friends live between Q and N.
are only two companies situated between the floors of  Only three friends between P and M.
companies T and P and also between the companies Q and U. Floor Person
Company S is situated on an odd numbered floor, but not on
7 N
the lowest floor. Which company is situated on floor number
6 P
1?
5 Q
Ng daifu;ka] P] Q] R] S, T vkSj U ,d 6 eaftyk bekjr ds fofHkUu 4 M
ryksa ij fLFkr gSa] ysfdu t:jh ugha fd blh Øe esa gksAa bekjr dh 3
lcls fupyh eafty dh la[;k 1 gS] blds Åij dh eafty dh la[;k 2
2 gS] vkSj blh rjg] lcls Åij dh eafty rd] ftldh la[;k 6 gSA 1
daiuh Q eafty la[;k 1 ij fLFkr ugha gSA daiuh P ls ikap daifu;ka  R lives imedietely below O.
gSa blds ry ds uhps fLFkr gSA daiuh T vkSj P ds ryksa ds chp vkSj
daifu;ksa Q vkSj U ds chp dsoy nks daifu;ka fLFkr gSaA daiuh S ,d  Only one person S is left.
Floor Person
fo"ke la[;k okyh eafty ij fLFkr gS] ysfdu lcls fupyh eafty ij
7 N
ugha gSA dkSu lh daiuh ¶yksj uacj 1 ij fLFkr gS\
6 S
(a) P (b) U (c) R (d) T
5 P
Sol. (b)
4 O
 Company P has five companies below its floor i.e. P is on 3 R
top floor. 2 Q
 There are two companies between P and T. 1 M
Floor Person  Hence, S lives on the sixth floor.
6 P 33. In a Seven – storeyed building, having floors numbered from 1
5 T to 7, A, B, C, D, E and F each live on a different floor. The
4 ground floor is numbered floor number 1, the floor above it is
3 floor number 2 and so on. One of the floors in the building is
2 vacant. B lives on the floor number 6. No floor below the floor
1 no. 3 is vacant : and no odd numbered floor is vacant. Only B
 S is on odd floor but not lowest floor lives between the floors of A and F. Only one person lives
 Two companies between Q and U and Q is not on floor between E and D. only two people live between C and F. C lives
number 1. on one of the floors below F. More than one person lives
between D and B. Which of the following floor numbers is
 R is remaining.
vacant?
Floor Person

285
Download Free PDFs & e-Books from Neon Classes App

,d nhu&HkaMkj okyh bekjr esa] ftldh eaftyksa dh la[;k 1 ls 7 funs’Z k ¼34-38½ ,d bekjr esa ,d ls lkr uacj dh eaftys bl rjg gS
rd gS] A, B, C, D, E vkSj F çR;sd ,d vyx eafty ij jgrs gSaA Hkwry fd Hkwry uacj ,d gS] blds Åij uacj nks gS vksj mlods Åij Øe
dh la[;k eafty la[;k 1 gS] blds Åij dh eafty eafty la[;k 2 c<+rk tkuk gSA lcls Å¡ph eafty uacj lkr gSA izR;sd eafty ij
gS vkSj blh rjgA Hkou dh ,d eafty [kkyh gSA B eafty la[;k 6 lkr O;fDr;ksa A, B, C, D, E, F vkSj G esa ls ,d&,d O;fDr jgrk gSA A
ij jgrk gSA eafty la[;k 3 ds uhps dksbZ eafty [kkyh ugha gS: vkSj pkSaFkh eafty ij jgrk gSA E, F dh eafty ls ,dne uhps dh eafty
dksbZ fo"ke la[;k okyh eafty [kkyh ugha gSA dsoy B, A vkSj F dh ij jgrk gSA F nwljh ;k lkroha eafty ij ugh jgrk gSA
eaftyksa ds chp jgrk gSA E vkSj D ds chp dsoy ,d O;fä jgrk gSA C C fdlh fo"ke uacj okyh eafty ij ugha jgrk gSA C dh eafty ds
vkSj F ds chp dsoy nks yksx jgrs gSaA C, F ds uhps fdlh ,d eafty ,dne Åij ;k uhps dh eafty ij B ugha jgrk gSA D lcls Åij
ij jgrk gSA D vkSj B ds chp ,d ls vfèkd O;fä jgrs gSaA dh eafty ij ugha jgrk gSA G, E dh eafty ls uhps dh fdlh Hkh
fuEufyf[kr eaftyksa dh la[;k fjä gS\ eafty ij ugha jgrk gSA
(a) 4 (b) 7 (c) 6 (d) 5 bu lwpukvksa ds vk/kkj ij fuEufyf[kr iz’uksa ds mÙkj nhft, &
Sol. (a) (34) Who lives on the top most floor?
 B lives on the floor no 6 lcls Åij dh eafty ij dkSu jgrk gS\
 No floor below floor number 3 is vacant (A) B (B) C (C) E (D) G
(35) Who lives immediate above D’s floor?
 No odd floor is vacant. D dh eafty ds ,dne Åij dkSu jgrk gS\
Floor Person
(A) B (B) C (C) E (D) G
7 (36) Three of the following four are alike in a certain way and so
6 B form a group. Which is the one that does not belong to that
5 Vacant group.
4 fuEufyf[kr pkj esa ls rhu fodYi fdlh izdkj ,d leku gS blfy,
3
mudk ,d lewg curk gsA og dkSulk fodYi gS tks bl lewg esa ugha
2
vkrk\
1
(A) F (B) D (C) B (D) C
 B lives between A and F. (37) Who lives on the lowest floor?
 Only two people live between C and F. lcls uhps dh eafty ij dkSu jgrk gS\
 C lives on one of the floors below F. (A) B (B) C (C) E (D) G
Floor Person (38) How many floors are there between the floor on which E
7 A resides and the floor on which G resides?
6 B E ftl eafty ij jgrk gS vkSj G ftl eafty ij jgrk gS] ds chp
5 F fdruh eaftys gS\
4 Vacant (A) 2 (B) 3 (C) 4 (D) 5
3 C Sol : F lives neither on the second floor nor on the seventh floor and
2 E lives immediately below F’s floor. Third floor and we will get
1 two possibilities.
 Only one person live between E and D ;gk¡ F u rks nwljh] u gh lkroha eafty ij jgrk gS vkSj F ds Bhd
uhps E jgrk gSA vr% F eafty 6 ;k eafty 3 ij jgsxk vkSj nks
 More than one person between D and B.
laHkkouk,¡ cusxh &
Floor Person
Floor Person Floor Person
7 A
7 B 7 G
6 B
6 F 6 C
5 F 5 E 5 D
4 Vacant 4 A 4 A
3 E 3 3 F
2 C 2 C 2 E
1 D 1 1 B
 Hence, 4th floor is vacant. Possibility&(1) Possibility&(2)
Directions(34-38). Study the following information
Possibility-(1) is incorrect becasuse G does not live on any
carefully and answer the questions given below:
floor i.e. below E’s floor.
A building has floors numbered one to seven, in such a way
laHkkouk&(1) xyr gS D;ksafd G, E dh eafty ds uhps ugha jgrk gSA
that the ground floor is numbered one, the floor above it,
Possibility-(2) is correct and we will solve all the questions
numbr two and so on such that seven people viz. A,B,C,D,E,F
according to it.
and G lives on each floor. A lives on fourth floor. E lives on the
laHkkouk&(1) lgh gS vkSj blh ds vuqlkj ge lHkh iz’uksa ds mÙkj
floor immediately below F’s floor. E lives on the floor
nsxAsa
immediately below F’s floor. F does not live on the second or
(34) G lives on the topmost floor.
the seventh floor.
G lcls Åij dh eafty ij jgrk gSA
C does not live on and odd numbered floor. B does not live on a
floor immediately above or below C’s floor. D does not live on (35) C lives immediately above D’s Floor.
the top most floor. G does not live on any floor below E’s floor. D dh eafty ds ,dne Åij C gSA
(36) F, B & D live on odd numbered floors and C lives on even
numbered floor. Therefore C is different from all others.

286
Best App for Govt. Jobs : Neonclasses (Download Now)

F, D vkSj B fo"ke la[;k okyh eafty ij jgrs gS rFkk C le la[;k  Shiva is shorter than Nagesh but taller than Triveni
okyh eafty ij jgrk gSA vr% C vyx gSA Nagesh > Shiva > Triveni
(37) B lives on the lowest floor. So from both conditions
B, lcls uhps dh eafty ij jgrk gSA Ravindra > Nagesh > Shiva > Triveni
(38) There are 4 floors between E & G. Hence, Ravindra is tallest among all.
E vkSj G ds chp esa pkj eaftys gSA 41. Among five boxes A B C D and E .B is thrice as heavy as A. C is
Concept –IV: Comparison Puzzle 40 kg heavier than E. D is three and a half times as heavy as E
blesa nks ;k vf/kd O;fDr;ksa dh fdlh vk/kkj ij rqyuk dh tkrh gS and C is five times as heavy as A. The weight of E is 40 kg.
vkSj fQj mUgsa vkjksgh ;k vojksgh Øe esa O;ofLFkr dj ity dks gy Which object is the lightest of all and what is its weight?
fd;k tkrk gSA blds fuEu mnkgj.kksa ls le> ldrs gS& ikap cDlksa esa A, B, C, D vkSj EA B] A ls rhu xquk Hkkjh gSA C] E ls
In this two or more persons are compared on some basis and 40 fdyks Hkkjh gSA D] E ls lk<+s rhu xquk Hkkjh gS vkSj C] A ls ikap
then the puzzle is solved by arranging them in ascending or xquk Hkkjh gSA E dk otu 40 gS fdyksxzkeA dkSu lh oLrq lcls gYdh
descending order. It can be understood from the following gS vkSj mldk Hkkj fdruk gS\
examples- (a) B 50 kg (b) A 16 kg
39. P, Q and R are standing in a queue one behind the other in (c) B 12 kg (d) D 125 kg
increasing order of heights such that the shortest is at the Sol. (b)
front and the tallest at the back. R is neither the first in the  Weight of E = 40 kg
queue nor the tallest person. P is NOT standing behind R. Who
 C is 40 kg haviour than E
is the shortest person among them?
So, C is 80 kg
P] Q vkSj R ,d nwljs ds ihNs ,d drkj esa c<+rs gq, Å¡pkbZ ds Øe
esa bl çdkj [kM+s gSa fd lcls NksVk vkxs vkSj lcls ihNs lcls yack  C is five time as heavy as A
gSA R u rks drkj esa igyk O;fä gS vkSj u gh lcls yack O;fä gSA 80
So, C = 5(A)  A = = 16 kg
P] R ds ihNs ugha [kM+k gSA muesa ls lcls NksVk O;fä dkSu gS\ 5
(a) Either P or R (b) P  D is three and half times as heavy as E
(c) R (d) Q 3 3
Sol. (b) D= (E)  × 40
2 2
Standing in a queue one behind the other in increasing order
D = 60 kg
of heights, shortest is at the front and tallest at the back.
 B is three is heavy as A
 R is neither the first in the queue nor the tallest person.
B = 3(A)  3 × 16
Shorest  < R <  tallest
B = 48
 P is not stading behind R, so P is standing ahead of R and Q C (80) > D (60) > B (48) > E (40) > A (16)
is standing behind of R. Hence, A = 16 kg lightest.
P<R<Q 43. Raj, Vikram, Amit, Amar and Shivani are five friends. Raj,
Hence, P is shortest person aong all. Vikram and Amit are intelligent. Raj, Amar and Shivani are
Åapkà ds c<+rs Øe esa ,d ds ihNs ,d drkj esa [kM+s gksrs gSa] lcls hard working. Amar, Amit and Shivani are teachers. Raj,
NksVk lcls vkxs vkSj lcls yack lcls ihNs gksrk gSA Vikram and Shivani are scholar. Who among the given friends
 R u rks drkj esa lcls igys gS vkSj u gh lcls yack O;fä gSA is neither hard working nor a scholar?
lcls NksVk  < R <  lcls yack  P ihNs ugÈ [kM+k gS] jkt] foØe] vfer] vej vkSj f'kokuh ikap fe= gSaA jkt] foØe vkSj
blfy, R, so P ds vkxs [kM+k gS R and Q ds ihNs [kM+k gSA vfer cqf)eku gSaA jkt] vej vkSj f'kokuh esgurh gSaA vejA vfer
P<R<Q
vkSj f'kokuh f'k{kd gSaA jkt] foØe vkSj f'kokuh fo}ku gSaA fn, x,
vr%] P lHkh esa lcls NksVk O;fä gSA fe=ksa esa ls dkSu u rks esgurh gS vkSj u gh fo}ku\
40. In a class, there are four students – Ravindra, Shiva, Triveni (a) Amit (b) Vikram
and Nagesh, who want to be selected in the basketball team. (c) Shivani (d) Raj
All of them have different heights. Nagesh is shorter than Sol. (a)
Ravindra but taller than Triveni. Shiva is shorter than Nagesh Intelligent  Raj, Amit, Vikram
but taller than Triveni. If the coach wants to select the tallest Hard working  Raj, Amar, Shivani
student among them, then who will be selected? Teacher  Amit, Shivani
,d d{kk esa pkj fo|kFkÊ gSa & johaæ] f'kok] f=os.kh vkSj ukxs'k] tks
Scholar  Raj, Vikram, Shivani
ckLdsVc‚y Vhe esa p;fur gksuk pkgrs gSaA mu lHkh dh vyx&vyx
Hence, Amit neither hardworking nor a scholar.
Åapkb;ka gSaA ukxs'k johaæ ls NksVk gS ysfdu f=os.kh ls yack gSA f'ko
44. Six students, Abhay Bhanu, Charu, David, Ekta and Farheen,
ukxs'k ls NksVs gSa ysfdu f=os.kh ls yEcs gSaA ;fn dksp muesa ls lcls are the top six rank holders in a school. The rank of Bhanu is
yacs Nk= dk p;u djuk pkgrk gS] rks fdldk p;u fd;k tk,xk\ between the ranks of Charu and David. Bhanu is on the fourth
(a) Nagesh or Ravindra rank. There are two students between the ranks of Ekta and
(b) Triveni David. Ekta is at the lowest rank among all the given students.
(c) Shiva The rank of Abhay is immediately below the rank of Farheen.
(d) Ravindra Who is on the fifth rank?
Sol. (d) N% fo|kFkhZ vHk;] Hkkuq] pk:] MsfoM] ,drk vkSj Qjghu ,d Ldwy esa
Nagesh is shorter than Ravindra but taller than Triveni 'kh"kZ N% jSad /kkjd gSaA Hkkuq dk jSad pk: vkSj MsfoM ds jSad ds chp
Ravindra > Nagesh > Triveni

287
Download Free PDFs & e-Books from Neon Classes App

gSA pkSFks jSad ij Hkkuq gSaA ,drk vkSj MsfoM ds jSad ds chp nks fo|kFkhZ vk;ksftr vafre O;k[;ku fuEu esa ls dkSulk gS\
gSaA fn, x, lHkh fo|kfFkZ;ksa esa ,drk lcls fups jSad ij gSA vHk; (A) Statistics ¼lkaf[;dh½
Qjghu ds jSad ls Bhd uhps gSA ikapos jSad ij dkSu gSA (B) Research Methods ¼vuqla/kku i}fr;k¡½
(a) Ekta (b) Charu (c) David (d) Abhay (C) Psychology ¼euksfoKku½
Sol. (b) (D) Can’t be determined ¼fu/kkZfjr ugha fd;k tk ldrk½
 Bhanu is an the fourth rank. (E) None of these ¼buesa ls dksbZ ugha½
  Ekta is at the lowest (47) If Wednesday if the ‘off day the code would be 2-4, if Thursday
was the ‘off day, the code would be 3-3. Taking into account
 There are two students between Ekta an David.
the ‘off day which of the following code is correct ?
 > > David > Bhanu > > Ekta
;fn cq/kokj ^vodk’k* dk fnu gS rks dksM 2&4 gksxk] ;fn xq:okj
 Bhanu rank is between David and Charu ^vodk’k* dk fnu gS rks dksM 3&3 gksxkA ^vodk’k* ds fnu dks /;ku
  Abhay rank is immediately below the rank of Farheen. esa ysrs gq, fuEufyf[kr esa ls dkSulk dksM lgh gS\
Farheen > Abhay > David > Bhanu > Charu > Ekta (A) 2-4 (B) 3-3
Hence, Charu rank is fifth. (C) 4-2 (D) Can’t be determined ¼fu/kkZfjr ugha fd;k tk
45. Six people, F, G, H, I, J and K, got different marks in their ldrk½
examination. G got more marks than F and K. F got more (E) None of these ¼buesa ls dksbZ ugha½
marks than I, but not the highest marks. G scored more marks (48) Which lecture is scheduled on Friday?
than H. K got more marks than J, but fewer marks than H. I got 'kqØokj dks dkSulk O;k[;u vk;ksftr gksrk gS\
the third-lowest marks. F got more marks than K but less than
(A) Economics ¼vFkZ’kkL=½
H. F got 59 marks and K got 46 marks. Who among the
(B) Psychology ¼euksfoKku½
following got the highest marks?
(C) Computer Science ¼dEI;wVj foKku½
Ng yksxks]a F, G, H, I, J vkSj K us viuh ijh{kk es avyx&vyx vad
(D) Can’t be determined ¼fu/kkZfjr ugha fd;k tk ldrk½
izkIr fd,A G us F vkSj K ls vf/kd vad izkIr fd,A F us I ls vf/kd
(E) None of these ¼buesa ls dksbZ ugha½
vad izkIr fd,] ysfdu mPPkre vad ugha izkIr fd,A G us H ls vf/kd
(49) How many lectures were scheduled between economics and
vad izkIr fd,A K us J ls vf/kd vad izkIr fd,] ysfdu H ls de vad
Psychology?
izkIr fd,A I rhljk lcls de vad izkIr gq;sA F dks K ls vf/kd vad
vFkZ’kkL= vkSj euksfoKku ds chp fdrus O;k[;kvksa dk vk;kstu fd;k
feys ysfdu H ls deA F dks 59 vad feys vkSj K us 46 vad izkIr
x;k Fkk\
fd,A fuEufyf[kr esa ls fdlus mPpre vad izkIr fd,\
(A) 1 (B) 2
(a) H (b) G (c) K (d) J
(C) 3 (D) Can’t be determined ¼fu/kkZfjr ugha fd;k tk
Sol. (b)
ldrk½
 I got third lowest marks. (E) None of these ¼buesa ls dksbZ ugha½a
 F got more marks than I but not the highest. (50) Which day is computer Science scheduled ?
 F got more than K but less than H. dEI;wVj foKku dk fdl fnu vk;kstu fd;k x;k gS\
 H>F>KF>I (A) Monday ¼lkseokj½
 G got more marks than H. G > H (B) Wednesday ¼cq/kokj½
 G got more marks than F and K. G > F, K (C) Thursday ¼xq:okj½
(D) Can’t be determined ¼fu/kkZfjr ugha fd;k tk ldrk½
 K got more marks J but fewer than H.
(E) None of these ¼buesa ls dksbZ ugha½
 H>K>J
(51) If some one wants to attend only two lectures out of
So relate all the points.
Psychology, Research Methods and Computer Science but
G>H>F>I>K>J
wants the two days to be successive (one after the other) than
Hence, G got highest marks.
which lecture-combination may be selected ?
Directions(46-51). Study the following information carefully
and answer the questions given below:
;fn dksbZ euksfoKku] vuqla/kku i}fr;k¡ vkSj dEI;wVj foKku esa ls
6 lectures are scheduled in a week starting from Monday and
dksbZ nks O;k[;kuksa esa mifLFkr gksuk pkgrk gS fdarq pkgrk gS fd nksuksa
ending on Sunday of the same week. Computer science is not fnu mÙkjksrj ¼,d ds ckn ,d½ gksus pkfg, rks fdu O;k[;kuksa ds
on Tuesday or Saturday. Psychology is immediately after la;kstu dk pquko fd;k tkuk pkfg,\
Organizational Behaviour. Statistics is not on firday and there (A) Research Methods, Computer Science ¼vuqla/kku i}fr;k¡]
is one day gap between Statistics and Research Methods. One dEI;wVj foKku½
day prior to the schedule of Economics there is no lecture (as (B) Psychology, Computer Science
that day is the ‘off’ day and Monday is not the ‘off’ day). ¼euksfoKku] dEI;wVj foKku½
funs’Z k ¼46-51½,d lIrkg ds lkseokj ls vkjEHk djrs gq, vkSj jfookj (C) Psychology, Research Methods
dks lekIr djrs gq, 6 O;k[;kuksa dks vk;kstu fd;k x;k gSA dEI;wVj ¼euksfoKku] vuqla/kku i}fr;k¡½
foKku eaxyokj ;k 'kfuokj dks ugha gSA euksfoKku] laxBukRed (D) Any Two of the three is possible
O;ogkj ds rqjar ckn gSA lkaf[;dh 'kqØokj dks ugha gS vkSj lkaf[;dh ¼rhu esa ls dksbZ nks laHko gSA½
vkSj vuqla/kku i)fr;ksa ds chp ,d fnu dk varjky gSA vFkZ’kkL= ds (E) With the condition of successive day it is not possible
vk;kstu ls ,d fnu igys dksbZ O;k[;ku ugha gSA ¼D;ksfa d ml fnu mÙkjksÙkj fnuksa dh 'krZ ds lkFk ;g laHko ugha gSA
*vodk’k* gS vksj lkseokj ^vodk’k* dk fnu ugha gSA½ Sol: There is no definite statement given in the question, using
bu lwpukvksa ds vk/kkj ij fuEufyf[kr iz’uksa ds mÙkj nhft,& which we can start solving the question. Apart from this, the
(46) Which of the following is the last lecture scheduled? following hints are given.

288
Best App for Govt. Jobs : Neonclasses (Download Now)

;gk¡ iz’u esa dksbZ fuf’pr dFku ugha fn;k x;k ftldk iz;ksx djds  G is younger than B
ge iz’u gy dj ldsA blds vykok ;gk¡ fuEu fgaV gS &  B>G>A>C>D>F>E
(1) Organizational Behaviour ¼laxBukRed O;ogkj½ (2) Off Hence, B is eldest.
¼vodk’k½ Concept – V : Box Puzzle
Psychology¼euksfoKku½ Economics¼vFkZ’kkL=½ Directions(54-58). Study the following information
(3) Statistics 
1Day
 Research Methods carefully and answer the question given below:
The off day and Economics will come together while 8 Boxes – A, B, C, D, F, G and H are placed one above the
Organizational behaviour and Psychology will come together. another but not necessarily in the same order. Two boxes are
Therefore, there remains one subject computer science which placed between F and E. F is placed above E. One box is placed
will come between statistics and Research Methods. between A and H. A is placed immediately below F. Two boxes
Thus the following arrangement will be obtained are placed between C and H. B is placed somewhere above D.
vodk’k fnu vkSj vFkZ’kkL= lkFk&lkFk jgsaxs vkSj laxBukRed O;ogkj 8 ckWDl – A, B, C, D, E, F, G vkSj H dks ,d ds Åij ,d j[kk x;k gS
o euksfoKku lkFk&lkFk jgsaxsA vr% ,d fo"k; dEI;wVj foKku ckfd ysfdu t:jh ugha fd mlh Øe esa gksA
jgk tks fd lkaf[;dh vkSj vuqla/kku i)fr;ksa ds chp vk,xkA bl F vkSj E ds chp nks ckWDl j[ks x;s gSA F dks E ls Åij j[kk x;k gSA
izdkj fuEu O;oLFkk izkIr gksxh & F vkSj G ds chp ,d ckWDl j[kk x;k gSA rhu ckWDl A vkSj H ds
Negative st. Day Subject chp j[ks x, gSA A dks F ds Bhd uhps j[kk x;k gSA nks ckWDl C
Off Monday Organization vkSj H ds chp j[ks x, gSA B, D ls dgha Åij j[kk x;k gSA
(Computer Science) Behaviour bu lwpukvksa ds vk/kkj ij fuEufyf[kr iz’uksa ds mÙkj nhft,&
Tuesday Psychology
Wednesday Statistics (54) How many boxes are placed above A?
Computer A ls Åij fdrus ckWDl gS\
Thursday
Science (A) 2 (B) 3 (C) 4 (D) 5
(Statisctics) Friday Research
Methods (55) How many boxes are between E and H?
(Computer Science) Saturday Off ckWDl E vkSj H ds chp fdrus ckWDl gS\
Sunday Economics (A) 2 (B) 3 (C) 1 (D) 5
(56) Which of the following boxes is placed at the top?
(46) Economics if the last lecture. dkSulk ckWDl lcls Åij j[kk x;k gS\
vFkZ’kkL= vafre O;k[;ku gSA vr% buesa ls dksbZ ugha (A) G (B) C (C) H (D) B
(47) 5 lectrues are scheduled before the off day and 1 Threrefore, (57) Which of the following boxes is immediately below box A?
the code will be- ‘5-1’ ckWDl A ds Bhd uhps dkSulk ckWDl gS\
^vodk’k* ls igys 5 O;k[;ku vkSj ckn esa 1 O;k[kku gSA vr% dksM (A) C (B) H (C) G (D) F
gksxk & ‘5-1’ (58) Which of the following pairs represents the boxes placed at the
 buesa ls dksbZ ugha top and at the bottom?
(48) Research Methods is scheduled on friday. fuEu esa ls dkSulk ;qXe lcls Åij vkSj lcls uhps okys ckWDl dks
'kqØokj dks vuqla/kku i}fr;k¡ vk;ksftr gksrk gSA iznf’kZr djrk gS\
 buesa ls dksbZ ugha (A) G, B (B) F, D (C) G, H (D) A, H
(49) Three lectures were scheduled on friday. Sol : One box is between F and G but it is not known which of the
vFkZ’kkL= vkSj euksfoKku ds chp 3 O;k[;ku gSA two boxes is above and which box is below.
Therefore, we will get two possibilities.
(50) Computer science is scheduled on Thursday.
Similarly, there are 3 boxes between A and H but it is not
dEI;wVj foKku dk vk;kstu xq:okj dks fd;k x;kA
known which of the two boxes is above and which one is
(51) Of these three subjects, only computer Science and Research
below. Therefore, we will get two more possilities.
Methods are one after another.
Thus there is a total of 4 possibilities which are given below.
bu rhu fo"k;ksa esa ls dsoy dEI;wVj foKku vkSj vuqla/kku i)fr;k¡
F vkSj G ds chp ,d ckWDl gS ysfdu F vkSj G esa ls dkSulk ckWDl
,d ds ckn ,d gSA
Åij gS vkSj dkSulk uhsp]s ;g Kkr ugha gSA vr% nks laHkkouk,¡ cusxhA
53. Among sevens friends A, B, C, D, E, F and G, G is elder than A
blh izdkj A vkSj H ds chp rhu ckWDl gS ysfdu nksuksa esa ls dkSulk
and C. C is elder than D. F is elder than one friend only. A is
third eldest. E is the youngest, G is younger than B. Who is the
ckWDl Åij gS vkSj dkSulk uhps, ;g Kkr ugha gSA vr% nks laHkkouk,¡
eldest? vkSj cusxhA
lkr fe=ksa A, B, C, D, E, F vkSj G esa G, A vkSj C ls cMk+ gSA C, D ls bl izdkj dqy pkj laHkkouk,¡ cusxh tks fuEu gS &
cM+k gSA F dsoy ,d fe= ls cM+k gSA A rhljk lcls cMk+ gSA E lcls G
B H H
NksVk gS] G, B ls NksVk gSA lcls cMk dkSu gS\
F - F G
(a) C (b) B (c) F (d) G A
A - -
Sol. (b) G F
C F
 F is elder than one friend only E A E A
 A is third eldest. D G - -
H E H E
 E is the youngest
Possibility (1) Possibility (2) Possibility (3) Possibility (4)
 >>A>>>F>E
Possibility-(2) Possibility-(3) and (4) are incorrect as there
 G is elder than A and C should be two boxes between H and C.
 C is elder than D

289
Download Free PDFs & e-Books from Neon Classes App

Possibility-(1) is correct and we will solve all the questions 5


according to it.  Only two boxes are kept above box A.
laHkkouk & (2), (3) vkSj (4) xyr gS D;ksfa d H vkSj C ds chp nks Nuber Box
ckWDl ugha vk ldrsA 1 D
(54) 3 boxes are placed above A. 2 E
A ds Åij 3 ckWDl gSA 3 A
(55) 1 box is between E & H. 4 C
E vkSj H ds chp 1 ckWDl gSA 5 B
(56) Box G is placed at the top.
ckWDl G lcls Åij gSA 61. Seven boxes of different colours, Red, Yellow, Blue, Green,
(57) Box C is placed immediately belwo box A. Orange, Violet and Indigo were stocked one on the other, not
ckWDl A Bhd uhps ckWDl C gSA necessarily in the same order. No other box was placed below
(58) Box G is at the top and Box H is at the bottom. the orange box. The violet box was placed second to the top of
ckWDl G lcls Åij vkWj ckWDl H lcls uhps gSA the Blue box. The green box was second to the bottom of the
59. Six identical boxes of different colours, Purple, Black, Yellow, Indigo box. The red box was placed at the topmost position.
Green, Cyan and Magenta, are arranged one over the other in The yellow box was exactly above the Indigo box. Which of the
the following manner; following is the correct pair of box and its position?
fofHkUu jaxks]a cSaxuh] dkys] ihys] gjs] fl;ku vkSj eStsVa k ds Ng leku yky] ihys] uhys] gjs] ukjaxh] cSaxuh vkSj bafMxks ds fofHkUu jaxksa ds lkr
cDls dks ,d ds Åij ,d fuEufyf[kr rjhds ls O;ofLFkr fd;k x;k cDls ,d nwljs ds Åij j[ks x, Fks] t:jh ugha fd blh Øe esaA
gS( ukjaxh c‚Dl ds uhps dksbZ vU; c‚Dl ugha j[kk x;k FkkA ok;ysV
Only three boxes are kept between the Cyan and Purple boxes. c‚Dl dks Cyw c‚Dl ds 'kh"kZ ij nwljs LFkku ij j[kk x;k FkkA gjs jax
The Yellow box is kept at the bottom most position. Only the dk c‚Dl bafMxks c‚Dl ds uhps ls nwljs LFkku ij FkkA yky c‚Dl dks
black colour box is kept between the Magenta and Green lcls Åij j[kk x;k FkkA ihyk c‚Dl bafMxks c‚Dl ds Bhd Åij FkkA
colour boxes. fuEufyf[kr esa ls dkSu&lk cDlksa dk lgh ;qXe vkSj mldh fLFkfr gS\
fl;ku vkSj iiZy c‚Dl ds chp dsoy rhu c‚Dl j[ks x, gSaA ;syks (a) Violet box – 7th position from the top
c‚Dl lcls fupys LFkku ij j[kk x;k gSA eStsVa k vkSj gjs jax ds ok;ysV c‚Dl & Åij ls 7 oka LFkku
cDlksa ds chp dsoy dkys jax dk c‚Dl j[kk x;k gSA (b) Yellow box -5th position from the top
What coloured box is kept at the third position from the top? ihyk c‚Dl &5 oka LFkku Åij ls
dkSu lk jax dk fMCck Åij ls rhljs LFkku ij j[kk x;k gS\ (c) Green box -3rd position from the top
(a) Cyan (b) Magenta gjk c‚Dl & Åij ls rhljk LFkku
(c) Green (d) Black (d) Blue box – 6th position from the top
Sol. (d) uhyk c‚Dl & Åij ls NBk LFkku
 Yellow is kept bottom most position. Sol. (d)
 Three boxes between Cyan and Purple box.  No other box was placed below the orange box.
 Cyan/Purple > > > > Cyan/Purple > Yellow  The red box was placed at topmost position.
 Black is between magenta and Green box. Red
 Cyan/Purple > Magenta/Green > Black > Magenta/Green > Orange
Cyan/Purple > Yellow
 Green box was second to the bottom of the Indigo box.
Hence, Black is at third from top.
60. Five boxes, A, B, C, D and E are kept in a vertical arrangement.  Yellow box was exactly above Indigo box.
Only one box is kept below box C. there are two boxes kept So, 2 possible cases:-
between boxes D and C. Box E is kept at the second place from Case 1 Case 2
the top. Only two boxes are kept above box A. Which box is Red Red
kept at the bottom? Yellow Yellow
ikap cDls] A, B, C, D vkSj E dks ,d ÅèokZèkj O;oLFkk esa j[kk x;k gSA Indigo Indigo
fMCck C ds uhps dsoy ,d fMCck j[kk gSA fMCck D vkSj C ds chp esa Green Green
nks fMCcs j[ks gSaA fMCck E Åij ls nwljs LFkku ij j[kk x;k gSA c‚Dl Orange Orange
A ds Åij dsoy nks c‚Dl j[ks x, gSaA lcls uhps dkSu lk c‚Dl  Violet box was placed second to the top of blue box so case
j[kk x;k gS\ 2 gets eliminates.
(a) Box D (b) Box C (c) Box E (d) Box B Red
Sol. (d) Yellow
 Only one box is kept below box C Indigo
Violet
 Two boxes kept between boxes D and C.
Green
 E is placed 2nd from the top. Blue
Nuber Box Orange
Top 1 D
Bottom 2 E Directions(62-66). Study the following information carefully
3 C and answer the question given below:
4

290
Best App for Govt. Jobs : Neonclasses (Download Now)

Six Boxes A, B, C, D, E and F are placed one above another, also


A →M E →L
they are occupied with different items i.e. K, L, M, X, Y and Z → L
E B→-
(but not necessarily in the same order). B →K D→X
Box which is occupied with M is not placed at top. A is placed D →X C →Z
either at top or at bottom, only 2 boxes are placed between A C →Z F →-
and D, which is occupied with X. only 1 box is placed between F →Y A→Y
D and F. Box E is not placed just above or just below D. Box E is Possibility-(3) Possibility-(4)
not placed just below the box which is occupied with K. Box
which is occupied with Z is placed between D and F. The box Possibility-(2) & (4) are incorrect because box B is not placed
which is placed just above B is occupied with L. The box which above A.
is placed at bottom occupied with Y. Box B is not placed above laHkkouk &(2) vkSj (4) xyr gS D;ksafd ckWDl B, A ds Åij ugha vk
A. ldrkA
6 ckWDl- A, B, C, D, E vkSj F dks ,d ds Åij ,d j[kk tkrk gSA lkFk Possibility-(3) is incorrect because the box occupied with M
gh muds vanj fofHkUUk oLrqvksa vFkkZr~ K, L, M, X, Y vkSj Z j[kh xbZ gSA can’t be placed at the top.
¼ysfdu t:jh ugha fd blh Øe esa½A laHkkouk &(3) xyr gS D;ksfa d ftl ckWDl esa M gS og lcls Åij
ftl ckWDl esa M j[kk x;k gS og lcls Åij ugha gSA A ;k rks lcls ugha vk ldrkA
Åij gS ;k lcls uhpsA A vkSj D ds chp nks ckWDl j[ks x, gSA D Possibility-(1) is correct and we will solve all the questions
according to it.
ckWDl esa X j[kk x;k gSA D vkSj F ds chp esa dsoy ,d ckWDl gSA
laHkkouk &(1) lgh gS vkSj blh ds vuqlkj ge lHkh iz’uksa ds mÙkj
ckWDl E, D ds u rks Bhd Åij gS vkSj u gh Bhd uhps gSA ckWDl E
nsx
a s
dks ml ckWDl ds Bhd uhps ugha j[kk tkrk ftlesa K gSA og ckWDl
(62) There are 3 boxes between them.
ftlesa Z gS dh D vkSj F ds chp j[kk tkrk gSA og ckWDl tks B ds
buds chp 3 ckWDl gSA
Bhd Åij j[kk x;k gS] mlesa L gSA tks ckWDl lcls uhps gS mlesa Y
(63) Box E is placed at the bottom.
gSA ckWDl B,A ls Åij ugha j[kk tkrkA ckWDl E lcls uhsps gSA
bu lwpukvksa ds vk/kkj ij fuEufyf[kr iz’uksa ds mÙkj nhft, &
(64) Box C is occupied with items M.
(62) How many boxes are there between the boxes which are
ckDl C esa oLrq M j[kh xbZ gSA
occupied with K and M?
(65) Box B is occupied with item Z.
os ckWDl ftuesa K vkSj M gS] ds chp fdrus ckWDl g\
oLrq Z ckWDl B esa gSA
(A) 1 (B) 2 (C) 3 (D) 4
(66) Box C is occupied with item M.
(63) Which of the following boxes is placed at the bottom?
ckWDl C esa M j[kk x;k gSA
lcls uhps dkSulk ckWDl j[kk x;k gS\
(A) A (B) E (C) C (D) D Repeated Series
(64) Which of the following items is in box C?
ckWDl C esa fuEu esa ls dkSulh oLrq gS\ In the questions related to this topic, a series of letters is given
(A) M (B) X (C) Z (D) Y that follows a certain pattern. All-though some letters will be
(65) In which box is the item Z? missing in such series, we have to find the missing letters by
oLrq Z dkSuls ckWDl esa gS\ understanding the pattern.
(A) F (B) S (C) B (D) C To solve such questions, the whole series is divided into a few
(66) Which of the following combinations of box-Objects is correct? groups. These groups can be of any number of letters.
;gk¡ fn, x, ckWDl&oLrq ds l;kst
a uksa esa ls dkSulk l;kastu lgh gS\ blesa vkus okys iz’uksa esa lkekU;r% v{kjksa dh ,d J`[a kyk nh xbZ gksrh
(A) B-L (B) D-M (C) C-M (D) E-L gS tks fd fuf’pr iSVuZ dk vuqlj.k djrh gSA gkykafd ,slh J`a[kyk esa
Sol : Box A is either at the top or the bottom, so two possibilities ls dqN v{kj yqIr gksrs gSA gesa iSVuZ dks le>dj yqIr v{kj Kkr
will be formed. Similarly one box is between D and F. djus gksrs gSA ,sls iz’uksa dks gy djus ds fy, iwjh J`[a kyk dks dqN
Therefore, two more possibilities will be formed. Thus we will lewgksa esa ck¡V fy;k tkrk gSA ;s lewg fdrus Hkh v{kjksa ds gks ldrs gSA
get a total of four possibilities which are given below. Some possible groups are given below:
;gk¡ ckWDl A ;k rks lcls Åij gS ;k lcls uhps] vr% nks laHkkouk,¡ dqN laHkkfor lewg fuEu izdkj ls gks ldrs gS&
cusxhA blh izdkj D vkSj F ds chp esa ,d ckWDl gS vr% nks
laHkkouk,¡ vkSj cusxhA bl izdkj dqy pkj laHkkouk,¡ cusxh tks fuEu (1) When all the group formed are the same:/ tc cuus okys
gS& lHkh lewg ,d tSls gks&
A →K F
→L (i) Groups countaining 2 letters/nks&nks v{kjkssa ds lewg
F →L B →Z ab/ab/ab/ab/ ------
B →Z D→X (ii) Groups containing 3 letters/rhu&rhu v{kjksas ds lewg
D →X C →M abc/abc/abc/abc/-------
C →M E →K (iii) Groups containing 4 letters/pkj&pkj v{kjksas ds lewg
E →Y A →Y abcd/abcd/abcd/abcd/--------
Possibility-(1) Possibility-(2) Similarly there can be 5, 6 ….. etc. letters’ group.
blh izdkj 5] 6 ----- v{kjksa ds lewg cusx
a Asa
(2) When the alternate groups formed are the same:/tc cuus
okys oSdfYid lewg ,d tSls gks&
abc/xyz/abc/xyz/-------

291
Download Free PDFs & e-Books from Neon Classes App

(3) When all groups are different:/tc lHkh lewg vyx&vyx gks& v{kjksa ds ml la;kstu dk p;u dhft,] ftls nh xbZ J`a[kyk ds fjDr
Such groups will also be in a logical sequence./,sls lewg Hkh LFkkuksa esa Øec) :Ik ls j[kus ij J`a[kyk iw.kZ gks tk,xhA
fdlh rdZlax dze esa gksaxsA Z_KF_OKGZ_K_ZO_I
abc/bca/cab/-------- (a) O Z O H K (b) O Z K H K
(4) When the number of letters in each group formed is (c) O Z O H Z (d) K Z O H K
different:/tc cuus okys izR;sd lewg esa v{kjksa dh la[;k Sol. (a)
vyx&vyx gks& Pattern : - Length :- 16 letters
ab/abc/abcd/abcde/------ Possible factors : (4)×4, 8×2, 16×1
1. Select the combination of letters that when sequentially placed ()
in the blanks of the given series will complete the series. Logic :- (Last letter +1) in each group.
v{kjksa dk og la;kstu pqusa ftls fuEukafdr J`[a kyk ds fjDr LFkkuksa ij Z O K F/Z O K G/Z O K H/Z O K I
Øec) :i ls j[ks tkus ij J`a[kyk iwjh gks tk,xhA +1 +1 +1
T_XTU__UXT_X 6. Select the combination of letters that when sequentially placed
(a) U X T U (b) T U U X in the blanks of the given series will complete the series.
(c) T X X U (d) X T U X v{kjksa ds ml la;kstu dk p;u djsa ftls nh xbZ Ja[kyk ds fjä
Sol. (a) LFkkuksa esa Øfed :i ls j[kus ij J`a[kyk iwjh gks tk,xhA
Pattern :- Total length 12 so factor in (3)×4, 6×2 K__NL_NK_N_LNK_M_LM_
 ) (a) L, M, M, M, K, L, K, N (b) L, M, M, K, K, L, N, N
T U X/T U X/T U X/ T U X (c) M, M, L, K, K, L, K, N (d) L, M, M, K, K, M, K, N
So here group of 3 letter made. Sol. (a)
2. Select the combination of letters that when sequentially placed Pattern : - Length :- 20 letters
in the blanks of the given series will complete the series. Possible factors : (5)×4, 10×2
v{kjksa ds ml la;kstu dk p;u dhft,] ftls nh xbZ J`a[kyk ds fjDr ()
LFkkuksa esa Øec) :Ik ls j[kus ij J`a[kyk iw.kZ gks tk,xhA Logic :- 1st letter goes on 4th position in every group.
_BIR_IRB_ KLMN/LMNK/MNKL/NKLM/KLMN
(a) CBH (b) RBI (L, M, M, M, K , L , K, N)
(c) HIG (d) TIC 7. Select the set of letters that when sequentially placed in the
Sol. (b) blanks of the given letter series will complete the series.
Pattern : - Length :- 9 letters v{kjksa ds ml leqPp; dk p;u djsa ftls nh xbZ o.kZ Ja[kyk ds fjä
Possible factors : (3)×3, 9×1 LFkkuksa esa Øfed :i ls j[kus ij Ja[kyk iwjh gks tk,xh
() _m_ln_mlmnk_k_nl
R B I/ R B I/ R B I (a) k, l, k, l, m (b) l, n, k, l, n
3. Select the combination of letters that when sequentially placed (c) m, n, k, l, m (d) k, n, k, l, m
in the blanks of the given series, will complete the series. Sol. (d)
v{kjksa ds ml la;kstu dk p;u djsa ftls nh xbZ Ja[kyk ds fjä Pattern : - Length :- 16 letters
LFkkuksa esa Øfed :i ls j[kus ij Ja[kyk iwjh gks tk,xh Possible factors : (4)×4, 8×2, 16×1
SP___PSTS___SPS_ ()
(a) P S S P P S S (b) T S S P T S S Logic :- 4th letter l remains constant, and k m n changes its
(c) S T S P S T T (d) P S T S P T T places alternatively.
Sol. (c) k m n l/ n k m l / m n k l / k m n l
Pattern : - Length :- 16 letters (k n k l m)
Possible factors : (4)×4, 8×2, 16×1 8. Select the combination of letters that when sequentially placed
() in the blanks of the given series will complete the series.
S P S T/ S P S T/ S P S T/ S P S T v{kjksa ds ml la;kstu dk p;u djsa ftls nh xbZ Ja[kyk ds fjä
4. Select the combination of letters that when sequentially placed LFkkuksa esa Øfed :i ls j[kus ij J`a[kyk iwjh gks tk,xhA
in the blanks of the given series will complete the series. _Z_C_LZ_C_L_X_VL___V
v{kjksa ds ml la;kstu dk p;u dhft,] ftls nh xbZ J`a[kyk ds fjDr (a) L, X, V, X, V, Z, C, Z, X, C
LFkkuksa esa Øec) :Ik ls j[kus ij J`a[kyk iw.kZ gks tk,xhA (b) X, Z, L, X, V, Z, C, C, C, L
BCB_BC_IB_BI (c) L, X, C, X, V, C, Z, C, Z, C
(a) BCI (b) BCC (d) L, X, V, V, X, C, C, X, Z, Z
(c) CBI (d) IBC Sol. (a)
Sol. (d) Pattern : - Length :- 20 letters
Pattern : - Length :- 12 letters Possible factors : (5)×4, 10×2
Possible factors : (4)×3, 6×2, 12×1 ()
() L Z X C V / L Z X C V/ L Z X C V/ L Z X C V
B C B I/ B C B I/ B C B I (L, X, V, X, V, Z, C, Z, X, C)
(I B C) 9. Select the combination of letters that when sequentially placed
5. Select the combination of letters that when sequentially placed in the blanks of the given series will complete the series.
in the blanks of the given series will complete the series. v{kjksa ds ml la;kstu dk p;u djsa ftls nh xbZ Ja[kyk ds fjä
LFkkuksa esa Øfed :i ls j[kus ij J`a[kyk iwjh gks tk,xhA

292
Best App for Govt. Jobs : Neonclasses (Download Now)

C _ B N _ _ V_ _ H C _ B _ H Total number of letters (including blank spaces) = 16


(a) VCBHNVN (b) HVCNBVN Hence we will create groups of ‘4’ letters.
(c) VHCBNVN (d) VHBNCHV M_O M/M N _ M/_N O M/M N _ M/
Sol. (c) Here all the groups are same because the places of letters in
Pattern : - Length :- 15 letters each group are matching.
Possible factors : (5)×3, 15×2 The first letters in three groups are ‘M’ hence the first letter in
() the group will be ‘M’.
CVBNH/CVBNH/CVBNH Thus the group formed is = M N O M
(V, H , C , B , N , V, N) M N OM/M N O M /M N O M/M N O M/
10. Select the combination of letters that when sequentially placed Hence option (b) N O M O
in the blanks of the given series will complete the series. 14. Select the combination of letters that when sequentially placed
v{kjksa ds ml la;kstu dk p;u djsa ftls nh xbZ Ja[kyk ds fjä in the blanks of the given series will complete the series.
LFkkuksa esa Øfed :i ls j[kus ij J`a[kyk iwjh gks tk,xhA v{kjksa ds ml la;kstu dk p;u djsa ftls nh xbZ Ja[kyk ds fjä
K_CTQ_OC__KO_T_KO_TT LFkkuksa esa Øfed :i ls j[kus ij J`a[kyk iwjh gks tk,xhA
(a) O, K, T, R, C, T, C L_UA_Z_N_AP_L_U_PZ
(b) P, K, T, R, C, S, C (a) N, P, L, U, Z, N, A
(c) O, K, U, R, C, S, C (b) N, L, P, A , N, U, Z
(d) O, K, T, R, C, S, C (c) P, N, L, Z, U, A , N
Sol. (d) (d) P, L, U, Z, N, A , N
Pattern : - Length :- 20 letters Sol. (a)
Possible factors : (5)×4, 10×2 Pattern : - Length :- 18 letters
() Possible factors : (6)×3, 9×2
Logic :- (Last letter +1) in each group. ()
+1 +1 +1 L N U A P Z / L N U A P Z/ L N U A P Z
K O C T Q/K O C T R/K O C T S/ K O C T T (N, P, L, U, Z, N, A)
15. Select the combination of letters that when sequentially placed
(O, K, T, R, C, S, C) in the blanks of the given series will complete the series.
11. Select the option that represents the letters that, when placed v{kjksa ds ml la;kstu dk p;u dhft,] ftls nh xbZ J`a[kyk ds fjDr
from left to right in the blanks below, will complete the letter LFkkuksa esa Øec) :Ik ls j[kus ij J`a[kyk iw.kZ gks tk,xhA
series. C2_X2CC_XX2_C2_X_C
ml fodYi dk p;u djsa tks v{kjksa dks fu:fir djrk gS] tc uhps (a) C2X2X (b) X2CX2
fjä LFkkuksa esa ck,a ls nk,a j[kk tkrk gS] rks v{kj J`[a kyk dks iwjk (c) AXX2C (d) XCX2C
djsxkA Sol. (b)
BE_KDF_HKDJEH_D_EHKD Pattern : - Length :- 18 letters
(a) J P K B (b) H E K N Possible factors : (6)×3, 9×2
(c) H B P J (d) I L P K ()
Sol. (b) C2XX2C/C2XX2C/C2XX2C
Pattern : - Length :- 20 letters (X 2 C X 2)
Possible factors : (5)×4, 10×2 16. Select the combination of letters that when sequentially placed
() in the blanks of the given series will complete the series.
Logic :- (1st letter +4) in each group. v{kjksa ds ml la;kstu dk p;u dhft,] ftls nh xbZ J`a[kyk ds fjDr
+4 +4 +4 LFkkuksa esa Øec) :Ik ls j[kus ij J`a[kyk iw.kZ gks tk,xhA
B E H K D/F E H K D/J E H K D/N E H K D PR__UC__RODU__PROD__T
(a) ODCPTCUT (b) ODPTUTCC
(H E K N) (c) ODTCTPUC (d) ODTPCTUC
12. QST_QS_RQ_TR_STR Sol. (d)
(a) SQTR (b) RTSQ (c) TRQS (d) TSRQ Pattern : - Length :- 21 letters
Sol. (b) Possible factors : (7)×3
Total number of letters (including blank spaces)=16 ()
Hence here we will create groups of 4-4 letters. PRODUCT/PRODUCT/PRODUCT
QST_/QS_R/Q_TR/_STR/ (O D T P C T U C)
When we look at the given series carefully then we find that all 17. Select the combination of letters that when sequentially placed
the groups are same. in the blanks of the given series will complete the series.
In three groups the last letters are ‘R’, hence the last letter in v{kjksa dk og la;kstu pqusa ftls fuEukafdr J`[a kyk ds fjDr LFkkuksa ij
the first group will also be ‘R’. Øec) :i ls j[ks tkus ij J`a[kyk iwjh gks tk,xhA
Thus the group formed is = QSTR IMM_ _ILEI_ _OBI_ _IMMO_ _LE
Q S T R/Q S T R/Q S T R/ Q S T R/ (a) OBLMLBIE (b) OBMMLEBI
13. M _ O M M N _ M _ N O M M N _ M (c) BOMLMEIB (d) BOLMLIEB
(a) O N M O (b) N O M O Sol. (b)
(c) M O N M (d) N N M O Pattern : - Length :- 24 letters
Sol. (b)

293
Download Free PDFs & e-Books from Neon Classes App

Possible factors : (8)×3, 6×4 (a) bdacb (b) bdbca


() (c) bdcab (d) cbdbc
I M M O B I L E / I M M O B I L E /I M M O B I L E Sol. (a)
(O B M M L E B I) Total letters ⇒ 18
18. Select the set of letters that when sequentially placed in the a c b d d b/ a c b d d b/a c b d d b
blanks of the given letter series will complete the series. 25. dc_abc_cb_bcd_ba_c
v{kjksa ds ml leqPp; dk p;u djsa ftls nh xbZ v{kj Ja[ kyk ds (a) dbacb (b) cbadb
fjä LFkkuksa esa Øekuqlkj j[kus ij og Ja[kyk iwjh gks tk,xhA (c) bdacb (d) bdcab
_swws_wwws_swwww_sss Sol. (c)
(a) W, W, S, S (b) W, S, W,S Total letters ⇒ 18
(c) W, S, S, S (d) W, S, S, W d c b a b c/d c b a b c/d c b a b c
Sol. (c) 26. db_cbd_ba_bd_bac_d
Length : 20 letters (a) adbcd (b) adcdb
Logic : W is increased by 1 in each group (c) bdcbd (d) dacdb
S is increased by 1 in each group Sol. (b)
WS/WWSS/WWWSSS/WWWWSSSS Total letters ⇒ 18
(W, S, S, S) dbacbd/dbacbd/dbacbd
19. Select the combination of letters that when sequentially placed 27. cb_db_cba_bc_bad_c
in the gaps of the given letter series will complete the series. (a) acdcb (b)cabdc
v{kjksa ds la;kstu dk p;u djsa fd tc nh xbZ v{kj J`a[kyk ds (c) dbcbc (d) cbdab
varjky esa Øfed :i ls j[kk tk, rks J`[a kyk iwjh gks tk,xhA Sol. (a)
b_bab_be_abbb_ba_b Total letters ⇒ 18
(a) ebabe (b) ebbeb cbadbc/cbadbc/cbadbc
(c) bebab (d) cbbac 28. k_lmml_mk_mmk_lkkl_m
Sol. (b) (a) l, k, m, k, k (b)k, l, m, k, k
Pattern : - Length :- 18 letters (c) k, l, k, l, m (d) k, m, m, k, l
Possible factors : (6)×3, 9×2 Sol. (c) kklmm /llmkk/ mmkll/ kklmm
() 29. _swww_wwws_swwww_sss
bebabb/bebabb/bebabb (a) w, s, s, w (b) w, s, s, s
(e b b e b) (c) w, s, w, s (d) w, w, s, s
Direction: Select the set of letters that when sequentially Sol. (b)
placed in the blanks of their given letter series will complete The required pattern is:
the series/v{kjksa ds ml lewg dk p;u djsa ftUgsa nh xbZ v{kj ws/wwss/wwws s s/wwwwssss
J`[a kyk ds [kkyh LFkkuksa esa dzfed :i ls Hkjus ij ;g J`a]kyk iwjh gks 30. _uvwuv_wvtuw_uvwuv_wv_uw
tk,xhA (a) t v w t t (b) t t t t t
20. d _ _ td_s_d_st (c) t u t u t (d) t t u t v
(a) ustuu (b) utsut (c) usutu (d) usust Sol. (b)
Sol. (c) The required pattern is:
Total letters ⇒ 12 Same Same
Group ⇒ 4 letters each
Dust/dust/dust
21. a_x_am_namxn_mx_ tuvw/uvtw/vtuw/tuvw/uvtw/vtuw
(a) nmzan(b) mnxan
(c) mnmxa (d) mmnxn 31. mn_pmno_ _nopm_op
Sol. (b) (a) oppn (b) opmn
Total letters ⇒ 16 (c) mpmo (d) oomn
Group ⇒ 4 letters each Sol. (b)
amxn/amxn/amxn/amxn/ The given series is :- mnop/mnop/mnop /mnop/mnop/
22. bac_cab_cd_a_ac_ca 32. ab _ _ _bca_ _acb_ _ _ aac _ _
(a) bdabc (b) dcbace (a) aabcabccab (b) aacbacabbc
(c) cabde (d) dacbd (c) cabaabcabc (d) cbaabcabca
Sol. (d) Sol. (b)
Total letters ⇒ 18 The given pattern is: abaacbc
b a c d c a/b a c d c a/b a c d c a 33. abb_ baa_a_bad_aba
23. ca_bab_ad_ab_ad_ab (a) abab (b) ccac
(a) bcdcb (b) cdbed (c) abba (d) aabb
(c) dcbcb (d) dcabc Sol. (c)
Sol. (c) Discrepancy is found in the questions. 13th character is given d
cadbabcadbabcadbad (officially by SSC) but it should be b to satisfy the pattern.
24. ac_d_b_cbdd_a_bddb Pattern if 13th character is b:-

294
Best App for Govt. Jobs : Neonclasses (Download Now)

abbabaababbadaaba When we look at the given series carefully we find that the
pattern if 13th character is :- abba/baab/abba/baab/ad letters are following a logical sequence here. For e.g.
34. e _ geef _ gg _ ee _ f _ ggg ;fn ge nh xbZ J`a[kyk dks /;ku ls ns[krs gS rc ;gk¡ v{kjksa dks ,d
(a) f, f, e, f, f (b) f, g, f, g, g rdZ laxr dze esa ikrs gS] tSls&
(c) g, f, e, f, f (d) f, f, f, f, f b c a b ⇒ Here ‘c’ after ‘b’, ‘a’ after ‘c’ and ‘b’ after ‘a’.
Sol. (a)
c a b c ⇒ Here ‘a’ after ‘c’, ‘b’ after ‘a’ and ‘c’ after ‘b’.
The given pattern is:-
Similarly’abca’ is also following the same sequence.
efg/eeffgg/eeefffggg.
blh izdkj a b c a Hkh blh dze esa gSA
35. _ k _ _ mmk _ ikiim _ _ ki _ I _ _ mkk
Out of the given options, that options with ‘a’ while one option
(a) miikmkiim (b) iikimimmk
starts with ‘b’. Firstly we will fill ‘b’ in the first blank.
(c) kiikmkkik (d) iiikmkkim
fn;s x;s fodYiksa esa ls rhu fodYi ‘a’ ls 'kq: gksrs gS tcfd ,d
Sol. (d)
The given pattern is:- ikiimmkk
foYi ‘b’ ls 'kq: gksrk gSA igys ge ‘b’ dks igys fjDr LFkku ij
36. j_mb_a__u_b_adj_m_b_d Hkjdj ns[ksaxsA
(a) u b m j m b u b a bbcab_cabc_abca_b
(b) u b d j m b j b a Here ‘b’ came before ‘b’, then in the second blank space ‘c’
(c) u b d j m b u b a should come before ‘c’, but in option (b) it is not given. Hence
(d) u b d j m b u m a the option (b) is incorrect i.e. definitely the letter ‘a’ will be
Sol. (c) filled in the first blank space.
j u m b b a d/j u m b b a d / j u m b b a d ;gk¡ ‘b’ ls igys ‘b’ vk;k] mlh vuqlkj nwljs fjDr LFkku esa ‘c’ ls
37. r_kn_c__h__bcbt_k_bcb igys ‘c’ vkuk pkfg, ysfdu fodYi (B) esa ,slk ugha fn;k x;k gSA
(a) h b b s k n h c vr% fodYi (B) xyr gSA
(b) h c b s k n b n vFkkZr~ igys fjDr LFkku esa fuf’pr :i ls v{kj ‘a’ vk,xkA
(c) h b b s k n h n abcab_cabc_abca_b
(d) h b c q k n h n Here in the first blank space ‘a’ came before ‘b’, then in the
Sol. (c) second blank space ‘b’ should come before ‘c’ Similarly in the
r h k n b c b /s h k n b c b /t h k n b c b third blank space ‘c’ must come before ‘a’ and in the fourth
38. g_o_dk_m_f_o_kk_gfo_d_km blank space ‘a’ must come before ‘b’.
(b) f o k g o d m o k Thus we will obtain the following series:
(c) f o k g o d f o k vc ;gk¡ igys fjDr LFkku esa fuf’pr igys ‘a’ vk;k gS blfy, nwljs
(d) f o k g o d m o m fjDr LFkku esa ‘c’ ls igys ‘b’] rhljs LFkku esa ‘a’ ls igys gS vkSj pkSFks
Sol. (b) fjDr LFkku esa ‘b’ vkuk pkfg,A
g f o o d k k m/ g f o o d k k m /g f o o d k k m a b c a b/ b c a b c/ c a b c a/ a b
39. d_a_b_qdh_a_cq_h_a_cq Hence each group contains 5 letters, while the 4th letter group
(a) h a c a b d a b is incomplete. Hence option (d) abca (Ans)
(b) h a b a b d a b ;gk¡ ij 5&5 v{kjksa dk lewg gS vkSj pkSFkk lewg viw.kZ gSA
(c) h a c a b d q b vr% fodYi (d) abca (Ans)
(d) h a q a n d a b 42. m n o n o p q o p q r s ----
Sol. (a) (a) m n o p q r (b) o p q r s t
d h a a b c q / d h a a b c q/ d h a a b c q (c) p q r s t u (d) o p r s t u
40. abcc _ _ _ ab _ _ de_ _ ccd _ eab_ _ _ de Sol. (c)
(a) ddeccdabdccd When we look at the given series carefully, we find that here
(b) ddecbdccdcda the groups are created in the increasing order of number off
(c) ccdabdccddde letters.
(d) ddeccdabddcc ;gk¡ nh xbZ J`a[kyk dks /;ku ls ns[kus ij Kkr gksrk gS fd ;gk¡ c<+rs
Sol. (a) gq, v{kjksa ds dze esa lewg cuk, x, gSA
The given pattern is:- M n o / n o p q / o p q r s /----/
Abccdde 3letters 4letters 5letter
41. _bcab_cabc_abca_b Now there will be six letters in the next group.
(a) a a b c (b) b b c a The first letters of the given groups are m, n and o respectively,
(c) a b a c (d) a b c a so the first letter of the fourth group will be ‘p’.
Sol. (d) The group formed is = pqrstu
Total number of letters (including blank spaces) = 17 Hence option (c).
dqy v{kjksa dh la[;k ¼fjDr LFkku lfgr½¾17 bl izdkj vxys lewg esa N% v{kj gksaxsA fn;s x;s lewgksa ds igys v{kj
Since the number of letters is 17 therefore we cannot figure dze% m, n, o gS blfy, pkSFks lewg dk igyk v{kj ‘p’ gksxkA pkSFkk
out how many letters the series should be divided into. Thus okyk lewg p q r s t u gksxkA
we have to solve this questions using options.
vr% fodYi (c)A
;gk¡ v{kjksa dh la[;k 17 gS] blfy, J`[a kyk dks fdrus v{kjksa ds lewg
43. _ b a _ b a b _ b a b b _ b
esa ck¡Vk tkuk pkfg, ;g Kkr ugha dj ldrs gSA blfy, ge ;gk¡ (a) b a a a (b) a b b b
fodYiksa dh lgk;rk ysx a sA (c) b a b b (d)a b a b
Sol. (b)

295
Download Free PDFs & e-Books from Neon Classes App

Total number of letters (including blank spaces) = 14 those conditions. Theere are mainly two types of
Here it is not possible to creat groups in the series. Hence we arrangements that are asked in the examination.
will solve the question using options. blesa vkus okys iz’uksa esa dqN 'krsaZ nh tkrh gS vkSj mu 'krksaZ dks iwjk
The last letters in three options are ‘b’ while in one option it is djrs gq, gesa O;fDr;ksa ds lewg dks O;ofLFkr djuk gksrk gSA
‘a’. Therefore we will first check the option (a) in which the O;fDr;ksa ds lewg ds cSBus dh O;oLFkk eq[;r% nks izdkj dh gksrh gS&
last letter is ‘a’. 1. Open form/[kqyh O;oLFkk
2. Close form/can O;oLFkk
;gk¡ J`[a kyk esa lewg cukuk laHko ugha gSA blfy, ge ;gk¡ fodYiksa Concept-I: Open Form: Single line arrangement (4,5,6,7,8,
dh lgk;rk ysaxsA 9 & More than 9 persons are given)
;gk¡ rhu fodYiksa esa vafre v{kj ‘b’ gS tcfd ,d fodYi esa vfUre Such an arrangement is open from both ends. In this
v{kj ‘a’ gSA vr% igys vfUre v{kj ‘a’ okyk fodYi (a) dk iz;ksx arrangement neither we can make first person sit as per our
djds ns[ksx a sA wish and nor we can make all pensons sits together.
bbaababababbab ,slh O;oLFkk nksuksa Nksj ls [kqyh gksrh gSA bl O;oLFkk esa ge igys
Here we don’t find any logical sequence i.e. option (a) is O;fDr dks viuh bPNkuqlkj ugha cSBk ldrs gS vkSj ugha lHkh O;fDr;ksa
incorrect. Now definitely the letter ‘b’ will come in the last dks ,d lkFk cSBk ldrs gSA
blank space. To solve open form questions we will make separate small
;gk¡ dksbZ Hkh rdZ laxr dze fn[kkbZ ugha ns jgk gS] vFkkZr~ fodYi (a) arrangements for each statement and finally the main
gekjk mÙkj ugha gks ldrkA arrangement will be obtained by adding these small
vc ;gk¡ vafre v{kj fuf’pr :i ls ‘b’ gksxkA arrangements.
_ba_bab_babbbb [kqyh O;oLFkk ds iz’uksa dks gy djus ds fy, izR;sd dFku ds fy,
we look at the given series carefully, we find that there comes vyx&vyx NksVh&NksVh O;oLFkk,¡ cuk,axs vkSj var esa bu NksVh&NksVh
‘b’ 4 times after the last ‘a’. It may be that there comes ‘b’ 3 O;oLFkkvksa dks tksM+dj eq[; O;oLFkk dks iwjk fd;k tk,xkA
times after the preceding ‘a’ and then ‘b’ two times after the Keep in mind that the person whose face is towards north, his
preceding ‘b’ before it. Thus letter ‘a’ will come in the first left-right will be according to our left-right i.e. our left will be
blank space and we will obtain the following series: that person’s left and our right will be that person’s right.
vc J`[a kyk dks /;ku ls ns[kus ij ge ikrs gS fd vafre ‘a’ ds ckn 4 ;g /;ku j[ks fd ftl O;fDr dk eq¡g mÙkj fn’kk dh rjg gksxk mlds
ckj ‘b’ vk;k gSA ,slk gks ldrk gS fd blls igys okys ‘a’ ds ckn 3 ck,¡&nk,¡ gekjs vuqlkj gksx
a s vFkkZr~ gekjk ck;k¡ ml O;fDr dk ck;k¡
ckj ‘b’ gks vkSj mlls igys okys ‘a’ ds ckn 2 ckj ‘b’ gksA bl rjg gksxk vkSj gekjk nk;k¡ ml O;fDr dk nk;k¡ gksxkA
lcls igys fjDr LFkku esa ‘a’ vk,xk vkSj cuus okyh J`a[kyk fuEu The person whose face is towards south, his left-right will be
gksxh& opposite to our left-right i.e. our left will be that person’s right
a b / a a b / a b a b / a b b a b/ and our right will be that person’s left.
Here the groups formed are in the increasing order of the ftl O;fDr dk eq¡g nf{k.k fn’kk dh rjQ gksxk mlds ck,¡&nk,¡ gels
number of letters. foijhr gksaxs vFkkZr~ gekjk ck;k¡ ml O;fDr dk nk;k¡ gksxk vkSj gekjk
;gk¡ c<+rs gq, v{kjksa ds dze esa lewg cuk, x, gSA nk;k¡ ml O;fDr dk ck;k¡ gksxkA
Option (b) a b b b 1. Four friends Rati, Areema, Sonali and Geet are sitting side by
44. a _ c a _ c _ d c _ d _ a d _ side on the bed facing towards north. Sonali is sitting towards
(a) d d c c a a left of Rati. Rati is sitting to immediate left of Areema. Geet is
(b) d d a a c c sitting between Rati and Sonali. Who is sitting at second place
(c) c c a d a d from the right end?
(d) a d c a d c pkj fe= jfr] vjhek] lksukyh vkSj xhr mÙkj dh vksj eq[k djds iyax
Sol. Total number of letters (including blank spaces) = 15 ij daèks ls daèkk feykdj cSBs gSaA lksukyh jfr ds ck;ha vksj cSBh gSA
dqy v{kjksa dh la[;k ¼fjDr LFkku lfgr½¾15 jfr] vjsek ds Bhd ck;sa cSBh gSA xhr jfr vkSj lksukyh ds chp cSBk
Hence here we will create groups of (3) or ‘5’ letters. First we gSA nk;sa Nksj ls nwljs LFkku ij dkSu cSBk gS\
will create groups of ‘3’ letters. (a) Geet (b) Rati
vr% ge ;gk¡ ij ^3^ ;k ^5^ v{kjksa ds lewg cuk,axsA (c) Sonali (d) Areema
a _ c/a _ c/ _d c/ _ d _/ a d _ / Sol. (b) Rati is sitting immediate left of Areema Geet is sitting
Here all the groups are same. The second letters in last three between Rati and sonali.
groups are ‘d’, hence the second letters in first and second
groups will also be ‘d’. Thus the group formed is = adc
;gk¡ ij cuus okys lHkh lewg ,d tSls gSA vafre rhu lewgksa esa nwljk Sonali Geet Rati Areema
v{kj ‘d’ gS blfy, igys vkSj nwljs lewg esa Hkh nwljk v{kj ‘d’
Sonali is sitting left of Rati
vk,xkA
bl izdkj cuus okyk lewg = adc
a d c/a d c/ ad c/ a d c/ a d c / Sonali Geet Rati Areema
Hence option (b) d d a a c c Left  Right
Sitting Arrangement So here, Rati is second from the right end.
2. Four friends K, L, M and N are sitting in a row, and they are
In seating arrangement questions we are given some facing south. L and M are sitting at two extreme ends. K sits
conditions and have to arrange a group of people according to between L and N and is second to the right of M. Who sits
immediate left of K?

296
Best App for Govt. Jobs : Neonclasses (Download Now)

fe= K] L] M vkSj N ,d ifaä esa cSBs gSa] vkSj muds eq¡g nf{k.k M Sits to the immediate right of I and immediate left of J. So
fn'kk dh vksj gSaA L vkSj M nks vfare Nksjksa ij cSBs gSaA K] L vkSj N from this Case 1 gets wrong So from case 2 :
ds chp cSBk gS vkSj M ds nk,¡ nwljk gSA K ds Bhd ck,¡ dkSu cSBk gS\
(a) L (b) M
I M J L K
(c) K (d) N
Sol. (d) Hence, J is in the middle of the row.
L and M are sitting at two ends so here two cases will be made. 5. Five colleagues, Chand, Chandini, Nidhi, Vikram and Ajay, are
Case – 1 Case - 2 sitting in a row facing north. Nidhi and Chandini occupie the
extreme ends of the row. Vikram is sitting exactly in the centre.
No two people with the name starting with the same letter are
L M M L immediate neighbours of each other. Ajay is sitting to the
K sits between L and N immediate left of Vikram. Who sits in the right extreme?
ikap lgdeÊ] pkan] pkafnuh] fufèk] foØe vkSj vt; ,d iafä esa mÙkj
dh vksj eq[k djds cSBs gSaA fufèk vkSj pkanuh iafä ds vafre Nksj ij
L K N M M MN K L
gSaA foØe Bhd dsæa esa cSBk gSA leku v{kj ls 'kq: gksus okys uke
K is second to the right of M so case 2 gets wrong.
okys dksbZ Hkh nks O;fä ,d nwljs ds fudVre iM+kslh ugha gSaA vt;]
foØe ds Bhd ck;sa cSBk gSA nk;sa Nksj ij dkSu cSBk gS\
L K N M M (a) Ajay (b) Chandini (c) Nidhi (d) Chand
So, N is immediate left of K. Sol. (c)
3. Five students I, J, K, L and M are sitting on a bench in a straight Nidhi and Chandini both occupy extreme ends So from this
row, facing North. M and J are immediate neighbours. M and K there are two possible cases.
are also immediate neighbours, I is on the extreme left of the
bench. J is sitting second from the extreme right. Who is sitting Case 1:
between I and J? Nidhi Chandini
ikap Nk= I] J] K] L vkSj M ,d csap ij ,d lhèkh iafä esa mÙkj dh
vksj eq[k djds cSBs gSaA M vkSj J fudVre iMkslh gSa- M vkSj K Hkh Case 2:
fudVre iMkslh gSa] I csp a ds ,dne ck;ha vksj gS- J vafre nk;sa ls Chandini Nidhi
nwljs LFkku ij cSBk gSA I vkSj J ds chp esa dkSu cSBk gS\ Now Vikram Sitting Exactly in Centre and Ajay is immediate
(a) L and M (b) K and M left of Vikram So-
(c) K and L (d) L
Sol. (b) Case 1:
Five Students I, J, K, L M Nidhi Ajay Vikram Chandini
I is on extreme left end and J is sitting Second from the right
Case 2:
end. Chandini Ajay Vikram Nidhi
So No two people with the name starting with the Same letter
I J are not immediate neighbours So chand and Chandini are not
neighbours of each others So Case 1 gets eliminated.
M and J are immediate neighbours & M and K are also
immediate neighbours So here –
Chandini Ajay Vikram Chand Nidhi
Hence, Nidhi sits at the right extreme end.
I K M J L
6. Five girls A, B, C, Z and U are sitting in a row, facing towards
Hence, K and M sitting between I and J. the south (but not necessarily in the same order). No one is
4. Five people, I, J, K, L, and M, are sitting in a straight row, facing sitting to the right of A. Three girls are sitting to the left of U. B
north. K sits to the immediate right of L. Only two people sit is second to the left of C. Who is sitting second to the left of U?
between L and I. M sits to the immediate right of I and to the iakp yM+fd;ka A, B, C, Z vkSj U nf{k.k dh vksj eqag djds ,d iafDr esa
immediate left of J. Who sits in the middle of the row? cSBh gSa ¼ysfdu t:jh ugha fd blh Øe esa gks½a A A ds nkbZa vksj dksbZ
ikap O;fDr I, J, K, L, vkSj M, ,d lh/kh iafDr esa mÙkj dh vksj eq[k ugha cSBk gSA U ds ckbZa vksj rhu yM+fd;ka cSBh gSaA B, C ds ck,a ls
djds cSBs gSaA K, L ds Bhd nk;sa cSBk gSA L vkSj I ds chp dsoy nks nwljs LFkku ij gSA U ds ck,a ls nwljs LFkku ij dkSu cSBk gSa\
O;fDr cSBs gSaA M, I ds Bhd nk;sa vkSj J ds Bhd ck;sa cSBk gSA iafDr ds (a) Z (b) B (c) A (d) U
e/; esa dkSu cSBk gSa\ Sol. (a)
(a) M (b) K All are facing south. No one is Sitting right of A. And three girls
(c) J (d) L are sitting to the left to U.
Sol. (c)
Right Left
Two people Sit between L and I, & K Sits to the immediate
right of L so here will be made by 2 cases- A U
Case – 1 Case - 2
B is Second to the left of C so hence final arrangement

L K I I L B
A U C Z B

297
Download Free PDFs & e-Books from Neon Classes App

Hence, Z is Second to the left of U. people sitting between Aakash and Suraj. Diksha has only one
7. Five friends N, M, O, P and Q are sitting in a column facing east person to her right. Aakash is sitting at the west end of the
(but not necessarily in the same order). O is sitting to the bench. There are three people between Rita and Ranjana. Who
leftmost side. N is the immediate neighbour of M and Q. P is is third from the west end of the bench?
not sitting on any of the end seats. Who is the immediate Ng lgdeÊ] jaxuk] nh{kk] pkgr] vkdk'k] jhrk vkSj lwjt] mÙkj dh
neighbour of O? vksj eq[k djds ,d ikdZ esa ,d csap ij cSBs gSaA jhrk csp
a ds iwoÊ Nksj
ikap fe= N, M, O, P vkSj Q ,d dkWye esa iwoZ dh vksj eq[k djds cSBs ij cSBh gSA jatuk vkSj pkgr ,d nwljs ds Bhd cxy esa cSBs FksA
gSa ¼ysfdu t:jh ugha fd blh Øe esa gks½a A O lcls ckbZa vksj cSBk gSA vkdk'k vkSj lwjt ds chp dsoy nks O;fä cSBs gSaA nh{kk ds nk;ha vksj
N, M vkSj Q dk fudVre iMkslh gS vkSj P fdlh Hkh vafre lhV ij dsoy ,d O;fä gSA vkdk'k csp a ds if'peh Nksj ij cSBk gSA jhrk vkSj
ugha cSBk gS- O dk fudVre iMkslh dkSu gSa\ jatuk ds chp rhu O;fä gSaA csp
a ds if'peh Nksj ls rhljk dkSu gS\
(a) Q (b) P (c) M (d) N (a) Chahat (b) Ranjana
Sol. (b) (c) Diksha(d) Suraj
All are facing in east direction. O is at leftmost Side and P is not Sol. (a)
sitting any of the ends. Rita is sitting at the east end and Aakash is sitting at the west
O and.
Left
Three people between Rita and Ranjana.
East
West

Aakash Ranjana Rita


Ranjana and chahat were immediate neighbours.
Right Two persons between Aakash and Suraj.
Dikhsa has only one person to her right.
N is the immediate neighbour of M and Q So N is only at
Second from right end. Here-
O Aakash Ranjana Chahat Suraj Diksha Rita
Hence, Chahat Rita
is third from the west end.
P
10. Six people Akshay, Anurag, Amit, Anubhav, Aryan and Ashish
M/Q are sitting in a straight row facing North. Neither Akshay nor
Amit are sitting at the corners. Amit and Anubhav are
N
immediate neighbour of each other. Aryan is sitting second
Q/M from the right corner. Only Akshay is sitting in between
Hence, P is the neighbour of O. Anubhav and Ashish. Amit is sitting second to the left of
8. Five friends P, Q, R, S and T are travelling in a bus. All are Anurag. Who is sitting in between Anurag and Amit?
sitting in five different rows one after the other, facing the Ng O;fDr v{k;] vuqjkx] vfer] vuqHko] vk;Zu vkSj vk’kh"k ,d lh/kh
same direction. P is sitting just ahead of S. T is after Q. Equal iafDr esa mÙkj dh vksj eq[k djds cSBs gSAa u rks v{k; vkSj u gh vfer
number of persons are sitting before and after R. T is before R. dksuksa ij cSBs gSaA vfer vkSj vuqHko ,d nwljs ds fudVre iMkslh gSa-
Who is sitting at the end? vk;Zu nk;sa dksus ls nwljs LFkku ij cSBk gSA vuqHko vkSj vk’kh"k ds
ikap fe= P, Q, R, S vkSj T ,d cl esa ;k=k dj jgs gSaA lHkh ,d ds chp dsoy v{k; cSBk gSA vfer] vuqjkx ds ck;as ls nwljs LFkku ij
ckn ,d ikap vyx&vyx iafä;ksa esa ,d gh fn'kk dh vksj mUeq[k cSBk gSA vuqjkx ds ck;sa ls nwljs LFkku ij cSBk gSA vuqjkx vkSj vfer
gksdj cSBs gSaA P, S ds Bhd vkxs cSBk gSA T, Q ds ckn cSBk gSA R ds ds chp esa dkSu cSBk gS\
igys vkSj ckn esa leku la[;k esa O;fä cSBs gSaA T, R ds igys cSBk gSA (a) Akshay (b) Aryan
var esa dkSu cSBk gS\ (c) Anubhav (d) Ashish
(a) Q (b) T Sol. (b)
(c) S (d) P Aryan is sitting second from the right corner
Sol. (c)
All are Sitting in five different rows one after another facing in
the same direction. Now- Aryan
→ Equal number of persons are sitting before and after R. So R Only Akshay is sitting between Anubhav and Ashish so there
is in middle of the row. are two possible cases –
→ T is after Q and before R So – Case 1:

Q T R
Anubhav/Ashish Akhsay Ashish/Anubhav Aryan
→ P is standing just ahead of S
Aryan
Case 2:
Q T R P S
Hence, S is Sitting at the end.
9. Six colleagues, Rangana, Diksha, Chahat, Aakash, Rita and Anubhav/ Akhsay Ashish/ Aryan
Suraj, are sitting on a bench in a park facing north. Rita is Ashish Anubhav
sitting at the east end of the bench. Ranjana and Chahat were
sitting immediately next to each other. There are only two

298
Best App for Govt. Jobs : Neonclasses (Download Now)

Amit and Anubhav are immediate neighbours. And Amit not Sol. (c)
sitting at the corner So case 2 gets eliminated. Q and O are neighbours of M.
L and N are neighbours of P
If O is not the nearest neighbour of L
Ashish Akhsay Anubhav Amit Aryan So two groups are QMO or OMQ
Aryan
Amit is second to the left of Anurag LPN or NPL
So here six possible cases –

Ashish Akhsay Anubhav Amit Aryan Anurag Case 1:


Hence, Aryan isAryan
sitting between Amit and Anurag. Q M O N P L
11. Six friends are sitting in a bench facing east. Falak is third to
the right of Charu. There are only two people sitting between
Case 2:
Sargun and Karan. Charu is at the north end of the bench. Arav
is sitting to the immediate left of Falak. Vaibhav and Karan are O M Q N P L
sitting immediately next to each other. Who is sitting second
Case 3:
from the south end?
Ng fe= iwoZ dh vksj eq[k djds ,d csp a ij cSBs gSaA Qyd pk# ds O M Q L P N
nk;sa ls rhljs LFkku ij gSA ljxqu vkSj dj.k ds chp dsoy nks O;fä
Case 4:
cSBs gSaA pk# csp
a ds mÙkjh Nksj ij gSA vkjo Qyd ds Bhd ck;sa cSBk
gSA oSHko vkSj dj.k ,d nwljs ds Bhd cxy esa cSBs gSaA nf{k.k Nksj ls N P L Q M O
nwljs LFkku ij dkSu cSBk gS\
(a) Arav (b) Sargun Case 5:
(c) Falak (d) Karan L P N Q M O
Sol. (d)
Charu is at north end of bench. Falak is third to right of charu. Case 6:
Aarav is immediate left of Falak.
L P N O M Q
North
↑ Charu Possible neighbour of Q are M, N, L
but in each case M is Q’s neighbour.
13. Seven boys A, B, C, D, E, F and G are sitting in a row, facing
towards the north (not necessarily in the same order). E is at
Aarav
one of the ends. A is fifth to the left of E. B is not the immediate
Falak neighbour of A or E. C is third to the left of B. Only D is sitting
between A and B. Only two boys are sitting between D and F.
lkr yM+ds A] B] C] D] E] F vkSj G ,d iafä esa mÙkj dh vksj eq[k
djds cSBs gSa ¼t:jh ugha fd blh Øe esa½A E fdlh ,d fljs ij gSA
Two people between Sargun and Karan. A] E ds ck;sa ls ik¡pok¡ gSA B] A ;k E dk fudVre iMkslh ugha gSA
Vaibhav and karan are immediate neighbours. C] B ds ck;sa ls rhljk gSA A vkSj B ds chp dsoy D cSBk gSA D vkSj
North F ds chp dsoy nks yM+ds cSBs gSaA
↑ Charu Which of the following statements is/are correct?
fuEufyf[kr esa ls dkSu lk@ls dFku lgh gS@gSa\
Sargun I. G is to the immediate left of D.
Aarav G] D ds Bhd ck;sa gSA
II. A and B are the neighbours of D.
Falak A vkSj B] D ds iM+kslh gSaA
Karan (a) Neither I nor II (b) Both I and II
(c) Only I (d) Only II
Vaibhav Sol. (d)

South E is at one of ends.
Hence, Karan is second from south end. A is Fifth to the left of E. So E is definatly
12. L, M, N, O, P and Q families are living in houses in a row of a at right end.
colony. Q and O are neighbours of M. L and N are neighbours of
P. If O is not the nearest neighbour of L, then who is the
nearest neighbour of Q from the given options? A E
L, M, N, O, P vkSj Q ifjokj ,d dkWyksuh ds ,d iafDr esa cus ?kjksa esa Only D is between A and B.
jg jgs gSaA Q vkSj O, M ds iM+kslh gSaA L vkSj N, P ds iM+kslh gSaA ;fn Only two boys between D and F
O, L dk fudVre iM+kslh ugha gS] rks fn, x, fodYiksa esa ls dkSu Q
dk fudVre iM+kslh gS\
(a) P (b) L A D B F E
(c) M (d) O C is third to the left of B. And G is Sit on remaining position

299
Download Free PDFs & e-Books from Neon Classes App

C A D B G F E S P T V
Hence, A and B are neighbours of D. W is third to the fleft of P and W is to the immediate right of U.
14. Seven women A, B, C, D, E, F and G are sitting in a row facing
the South (not necessarily in the same order).
lkr efgyk,a A] B] C] D] E] F vkSj G nf{k.k dh vksj eq[k djds ,d U W S P T V
iafä esa cSBh gSa ¼t:jh ugha fd blh Øe es½a A → R is third to the right of P.
A is fifth to the right of G. A is at one of the ends. E is exactly in → Q is the remaining person.
the middle of row. F is fourth to the right of G. B is not the
immediate neighbour of E. D is to the immediate left of E.
A] G ds nk;sa ls ikaposa LFkku ij gSA A fdlh ,d Nksj ij gSA E] U W Q S P T V R
iafä ds Bhd eè; esa gSA F] G ds nk;sa pkSFkk gSA B] E dk fudVre Hence, R is fourth to the right of S.
iMkslh ugha gSA D] E ds Bhd ck;sa gSA 16. Eight friends, P, Q, R, S, T, U, V and W, are watching a movie
Which of the following statements is/are NOT correct? sitting in a line. T is sitting at one of the corners. Q is sitting
fuEufyf[kr esa ls dkSu lk@ls dFku lgh ugha gS@gSa\ between W and R. V is sitting between S and U. W is sitting to
I. D is third to the left of F./ D] F ds ck;sa ls rhljs LFkku ij gSA the immediate right of T. R is sitting to the immediate left of P.
II. F is sitting between C and D./ F] C vkSj D ds chp cSBk gSA There are two persons between P and U. Who is sitting at the
(a) Both I and II (b) Only II other corner?
(c) Only I (d) Neither I nor II vkB fe=] P, Q, R, S, T, U, V vkSj W ,d iafä esa cSBs gq, fQYe ns[k
Sol. (b) jgs gSaA T fdlh ,d dksus ij cSBk gSA Q] W vkSj R ds chp esa cSBk
A is fifth to the right of G. gSA V] S vkSj U ds chp esa cSBk gSA W] T ds Bhd nk;sa cSBk gSA R] P
A is at one of the ends. So A is only ds Bhd ck;sa cSBk gSA P vkSj U ds chp esa nks O;fä cSBs gSaA
Sits at Right end. (a) U (b) P (c) R (d) S
E is exactly in the middle of row. Sol. (a)
P, Q, R, S, T, U, V and W are watching a movie sitting in a line.
→ Left
Right ← → T is sitting at one of the corners.
A E G
Case 1:
F is Fourth to the right of G.
D is immediate left of E.
T
Case 2:
A F E D G
B is not the immediate neighbour of E.
And the remaining is woman C. T
→ W is Sitting to the immediate right of T
So Case 2 gets eliminated.
A F C E D G B
→ Q is sitting between W and R.
Hence, F is sitting between C and D not correct. → R is sitting to the immediate left of P.
15. Eight north-facing restaurants named P, Q, R, S, T, U, V and W
are located in a straight line. S is second to the left of T. W is
third to the left of P. T is between P and V. S is third to the left T W Q R P
of V. W is to the immediate right of U. R is third to the right of
→ There are two person between P and U.
P. Who is sitting fourth to the right of S?
→ V is sitting between S and U.
vkB mÙkj eq[kh jsLrjka P, Q, R, S, T, U, V vkSj W ,d lhèkh js[kk esa
fLFkr gSaA S] T ds ck;sa ls nwljs LFkku ij gSA W] P ds ck;sa ls rhljk
gSA T] P vkSj V ds chp esa gSA S] V ds ck;sa ls rhljk gSA W] U ds
T W Q R P S V U
Bhd nk;sa gSA R] P ds nk;sa ls rhljs LFkku ij gSA S ds nk;sa ls pkSFks
Hence, U is Sitting at the other corner.
LFkku ij dkSu cSBk gS\
17. Nine people Asha, Bishal, Carol, Dinky, Emily, Farah, Golu,
(a) U (b) W
Harsh, and Itika are watching a movie in a theatre sitting in a
(c) R (d) V
straight row facing North. Bishal is at one end of the row.
Sol. (c)
Harsh is seated adjacent to both Farah and Golu. Carol is to the
All are facing North.
immediate right of Dinky and in the third place to the right of
→ S is Second to the left of T and S is third to the left of V.
Emily. Asha is on the immediate left of Farah. Farah is in the
third place to the left of Bishal. Who are at the ends of the row?
ukS yksx vk’kk] fc’kky] dSjy] fMadh] ,feyh] Qjkg] xksyw] g"kZ vkSj
S T V
bfrdk ,d fFk,Vj esa ,d fQYe ns[k jgs gSa tks ,d lh/kh iafDr esa
T is between P and V mÙkj dh vksj eq[k djds cSBs gSaA fc’kky iafDr ds ,d Nksj ij gSA g"kZ]
Qjkg vkSj xksyw nksuksa ds vklUu cSBk gSA dSjy] fMadh ds Bhd nk;sa

300
Best App for Govt. Jobs : Neonclasses (Download Now)

vkSj rhljs LFkku ij ,feyh ds nk;ha vksj gSA vk’kk] Qjkg ds Bhd sitting to the immediate right of Sheela, who is facing Damini.
ck;ha vksj gSA Qjkg] fc’kky ds ck;sa ls rhljs LFkku ij gSA iafDr dsa Ankita is not sitting in any of the end position. Pinki is facing
var esa dkSu gSa\ the same side as Kamini. Who is sitting to the left of Ankita?
(a) Bishal and Dinky Ng O;fDr] vafdrk] cchrk] 'khyk] nkfeuh] fiadh vkSj dkfeuh] nks
(b) Carol and Emily lekukarj iafDr;ksa esa cSBs gSa] ftuesa ls rhu mÙkj dh vksj vkSj vU;
(c) Harsh and Itika rhu dk eq[k nf{k.k dh vksj gSA vafdrk dk eq[k nf{k.k dh vksj gSA
(d) Emily and Bishal dkfeuh 'khyk ds Bhd nk;sa cSBh gS] ftldk eq[k nkfeuh dh vksj gSA
Sol. (d) vafdrk fdlh Hkh vafre fLFkfr esa ugha cSBh gSA fiadh dk eq[k dkfeuh
→ Bishal is at one end of the row. ds leku Hkqtk dh vksj gSaA vafdrk ds ck;sa dkSu cSBk gS\
→ Farah is in the third place to the left of Bishal. (a) Kamini (b) Sheela
So Bishal is definatly at Right end – (c) Babita (d) Pinki
Sol. (c)
Total Six, three of them facing north and three of them facing
Farah Bishal South, Sitting in parralles row.
→ Asha is immediate left of Farah. → Ankita is facing south and is not sitting is any of end
→ Harsh is adjacent to both Farah and Golu. position.
Ankita

Asha Farah Harsh Golu Bishal


→ Carol is immediate right of Dinky and in the third place to
the right of Emily.
→ Kamini is sitting to the immediate right of Sheela, who is
Emily Itika Dinky Carol Asha Farah Harsh Golu Bishal facing Damini.
→ Pinki is facing the same side as Kamini –
Hence, Emily and Bishal are the ends of the row.
Damini Ankita
18. A certain number of people are sitting in a row, facing North, E
sits fourth to the right of D. B sits third to the left of D. Exactly
seven people sit between A and D, E and C occupy the extreme
end positions of the row. A sits third to the right of C. If no
other person is sitting in the row, what is the total number of
Sheela Kamini Pinki
persons seated?
,d fuf'pr la[;k esa yksx mÙkj dh vksj eq[k djds cSBs gSa] E] D ds → Babita is on remaining Position.
nk;sa ls pkSFks LFkku ij cSBk gSA B] D ds ck;sa ls rhljs LFkku ij cSBk Damini Ankita Babita
gSA A vkSj D ds chp Bhd lkr yksx cSBs gSa] E vkSj C vafre Nksj ij cSBs
gSaA iafä dh fLFkfrA A] C ds nk;sa ls rhljs LFkku ij cSBk gSA ;fn
dksbZ vU; O;fä iafä esa ugha cSBk gS] rks dqy fdrus O;fä cSBs gSa\
(a) 14 (b) 15
Sheela Kamini Pinki
(c) 16 (d) 17
Sol. (c) Hence, Babita is sitting to the left of Ankita.
→ Facing North 20. Three boys T, R and M are sitting in a row facing towards the
→ E sits fourth to the right of D. north (not necessarily in the same order), and three girls F, G
→ B sits third to the left of D and K are sitting in a row facing towards the face of these boys
(not necessarily in the same order). F and R are diagonally
opposite to each other. K is not at any of the ends. Which of the
B D E following pairs of persons facing towards each other is/are
definitely correct?
→ E and C occupy the extreme end positions.
→ Seven Person sits between A and D.
rhu yM+ds T, R vkSj M ,d iafDr esa mÙkj dh vksj eq[k djds cSBs gSa
¼t:jh ugha fd blh Øe es½a ] vkSj rhu yM+fd;ka F, G vkSj K ,d iafDr
esa bu yM+dksa dh vksj eq[k djds cSBh gSa ¼t:jh ugha fd ,d gh Øe½A
F vkSj R ,d nwljs ds frjNs foijhr gSaA K fdlh Hkh Nksj ij ugha gSaA
A B D E
fuEufyf[kr esa ls dkSu&lk@ls O;fDr;ksa dk ,d&nwljs dh vksj eq[k
→ A sits third to right of C. djds tksM+s fuf’pr :i ls lgh gSa\
I. GR
II. KT
C A B D E (a) Only II (b) Only I
Hence total 16E persons seated in the row. (c) Neither I nor II (d) Both I and II
Concept- II: Open Form : Double line arrangement Sol. (b)
(6,8,10,& 12 Persons are given) Three boys T, R, M facing North
19. Six people, Ankita, Babita, Sheela, Damini, Pinki and Kamini, Three girsl F, G, K facing towards boys i. e. South
are sitting in two parallel rows, three of them facing north and → K is not at any of the ends.
the other three facing south. Ankita is facing south. Kamini is → F and R diagonally opposite to each other

301
Download Free PDFs & e-Books from Neon Classes App

So here two possible cases – → Krishna is Second to the left of Manish while Yugal and
Case 1: Krishna are not in the same row.
K F Farhan Utkarsh Yugal

R Krishna Chaman Manish


Case 2: Hence, Krishna is at the diagonally opposite end to Yugal.
F K 22. Six friends, A, B, C, D, E and F, are sitting in two lines, with
three friends in each line. The friends are facing each other. C
is facing F. F is sitting between B and E. A is facing B. Who is
NOT sitting at one of the four corners?
Ng fe=] A, B, C, D, E vkSj F, nks iafä;ksa esa cSBs gSa] çR;sd iafä esa rhu
R fe= gSaA nksLr vkeus&lkeus gSaA C, F dh vksj mUeq[k gSA F, B vkSj E ds
→ Now G Sits at remaining position in South Facing row. chp esa cSBk gSA A dk eq[k B dh vksj gSA dkSu pkjksa dksuksa esa ls fdlh
→ Position of T & M not decided. ,d ij ugha cSBk gS\
Case 1: (a) D (b) B
G K F (c) A (d) C
Sol. (d)
→ C is facing F
→ F is Sitting between B and E
So here two possible cases –
R T/M M/T Case 1: Case 2:
Case 2: B F E E F B
F K G

A C C
T/M M/T R → A is facing B
Hence, G – R only pairs of persons facing towards each other Case 1: Case 2:
definalty in both of the cases – B F E E F B
So only I is corret.
21. Six persons are sitting in two straight rows, facing north. Their
names are Farhan, Utkarsh, Yugal, Krishna, Chaman and
Manish. Farhan and Manish are sitting at diagonally opposite
ends. Utkarsh is in the top row and to the immediate right of A C D D C A
Farhan. Krishna is second to the left of Manish while Yugal and Hence, In both of the cases C is not Sitting at the corners.
Krishna are not in the same row. Who is sitting at the 23. Four males P1, P2, P3 and P4 are sitting in a row facing
diagonally opposite end to Yugal? towards the North (not necessarily in the same order) and
Ng O;fDr nks lh/kh iafDr;ksa esa mRrj dh vksj mUeq[k gksdj cSBs gSaA four females A1, A2, A3 and A4 are sitting in a row facing
buds uke Qjgku] mRd"kZ] ;qxy] —".kk] peu vkSj euh"k gSaA Qjgku towards the face of these males (not necessarily in the same
vkSj euh"k frjNs foijhr Nksj ij cSBs gSaA mRd"kZ 'kh"kZ iafDr esa gS vkSj order). P1 and P4 sit at the ends. A3 and P2 are facing each
Qjgku ds Bhd nk;sa gSaA —".k euh"k ds ck,a ls nwljs LFkku ij gSa other. A4 is second to the left of A3. P4 is facing A1. A1 and A4
tcfd ;qxy vkSj —".k ,d gh iafDr esa ugha gSaA ;qxy ds frjNs are at the ends.
foijhr Nksj ij dkSu cSBk gS\ pkj iq#"k P1] P2] P3 vkSj P4 ,d iafä esa mÙkj dh vksj eq[k djds
(a) Chaman (b) Manish cSBs gSa ¼t:jh ugha fd blh Øe esa½ vkSj pkj efgyk,a A1] A2] A3
(c) Krishna (d) Utkarsh vkSj A4 ,d iafä esa bu iq#"kksa ds eq[k dh vksj eq[k djds cSBh gSa
Sol. (c) ¼t:jh ugha fd blh Øe esa gksa½A P1 vkSj P4 fljs ij cSBs gSaA A3
All are facing north 3 in one row and other 3 are in another vkSj P2 ,d nwljs dh vksj mUeq[k gSaA A4] A3 ds ck,a ls nwljs LFkku
parallel row. ij gSA P4 dk eq[k A1 dh vksj gSA A1 vkSj A4 fljs ij gSaA
→ Utkarsh is in top row and to the immediate right of farhan. Which of the following statements is correct?
→ Farhan and Manish are at diagonally opposite ends. fuEufyf[kr dFkuksa esa ls dkSu lgh gS\
Farhan Utkarsh
I. A4 is facing P1./ A4 dk eq[k P1 dh vksj gSA
II. P4 is second to the left of that male who is facing A2.
P4 ml iq#"k ds ck,a ls nwljs LFkku ij gS tks A2 dh vksj mUeq[k gSA
(a) Both I and II (b) Only I
Manish (c) Neither I nor II (d) Only II

302
Best App for Govt. Jobs : Neonclasses (Download Now)

Sol. (a) Q P R O
Four males P1, P2, P3, P4 facing North While four females A1,
A2, A3, A4 facing towards the face of males i.e. in South.
→ A1 and A4 are at the ends.
→ A4 is second to the left of A3 So A4 is definitely sits at right
end of row facing South. K N M L
A1 A3 A4 Hence, R faces to M.
25. Eight girls, K, L, M, N, O, P, Q and R, are sitting in two lines, with
four girls in each line. Both the lines are facing the same
direction. One line is behind the other line. K is at a corner and
to the immediate left of P. O is to the immediate left of M. Two
→ P4 is facing A1 and, P1 and P4 sits at the end & A3 and P2 persons are sitting between K and R. L is just behind P. O is
facing each other. between L and M. N is not in the front line. Who is sitting
A1 A3 A4 between P and R?
vkB yM+fd;ka] K, L] M, N, O] P, Q vkSj R, nks iafä;ksa esa cSBh gSa] çR;sd
iafä esa pkj yM+fd;ka gSaA nksuksa js[kk,a ,d gh fn'kk dh vksj mUeq[k gSaA
,d ykbu nwljh ykbu ds ihNs gSA K ,d dksus esa gS vkSj P ds Bhd
P4 P2 P1 ck;sa gSA O] M ds Bhd ck;sa gSA K vkSj R ds chp nks O;fä cSBs gSaA L] P
ds Bhd ihNs gSA O] L vkSj M ds chp esa gSA N lkeus ugha gSA js[kkA P
→ Now, A2 and P3 remaining girls Sits on remaining positions
vkSj R ds eè; dkSu cSBk gS\

(a) Q (b) M
A1 A3 A2 A4
(c) L (d) N
Sol. (a)
K, L, M, N, O, P, Q, R Sitting in two lines with four each. Both
lines are facing the same direction.
P4 P2 P3 P1 → K is at corner and to the immediate left of P.
→ two person are between K and R.
Hence, A4 is facing P1
→ L is just behind P So P is Sitting in first Row and K is at left
P4 is Second to the left of that male who is facing A2 both
end of first row
correct
K P R
24. Eight Friends are playing a game, K, L, M, and N are sitting in a
straight line and are facing North. O, P, Q and R are stitting on a
parallel line and are facing south. Thus each of the persons
sitting in a line, faces another person sitting in the parallel line.
M faces a person who has two people sitting to his right. N is
L
sitting between. M and K. Q faces K, O is neither and immediate
neighbour of P nor Q. Who amongst the following faces M? → O is between L and M.
vkB fe= ,d [ksy [ksy jgs gSa] K] L] M vkSj N ,d lhèkh js[kk esa cSBs → O is immediate left of M.
gSa vkSj mÙkj dh vksj mUeq[k gSaA O] P] Q vkSj R ,d lekukarj js[kk → N is not in front line So Q is on remaining position of front
line –
ij cSBs gSa vkSj nf{k.k dh vksj mUeq[k gSaA bl çdkj ,d iafä esa cSBs
K P Q R
çR;sd O;fä dk eq[k lekukarj js[kk esa cSBs fdlh vU; O;fä dh vksj
gSA M dk eq[k ml O;fä dh vksj gS ftlds nkfguh vksj nks O;fä cSBs
gSaA N chp esa cSBk gSA M vkSj K, Q dk eq[k K dh vksj gS] O u rks P
dk vkSj u gh Q dk fudVre iM+kslh gSA fuEufyf[kr esa ls dkSu M
dh vksj mUeq[k gS\ N L O M
(a) Q (b) O
Hence, Q is Sitting between P and R.
(c) P (d) R
26. A group of 8 students, J, K, L, M, N, O, P and Q are sitting in two
Sol. (d)
parallel rows facing each other. Each row has four students. Q
K, L, M, N Facing North And O, P, Q, R
and N are facing each other. Q is an immediate neighbour of
Sitting parallels facing South.
both P and O. K is sitting to the immediate left of O. K and J are
→ M faces a person who has two people sitting to his right.
diagonally opposite. M and N are not immediate neighbour of
→ N is Sitting between M and K.
each other. Who is in front of L?
8 Nk=ksa dk ,d lewg] J] K] L] M] N] O] P vkSj Q nks lekukarj
iafä;ksa esa ,d nwljs ds lkeus cSBs gSaA çR;sd iafä esa Nk=ksa ds fy, gSA
Q vkSj N ,d nwljs dh vksj mUeq[k gSaA Q] P vkSj O nksuksa dk
K N M fudVre iMkslh gS- K] O ds Bhd ck;sa cSBk gS- K vkSj J frjNs foijhr
gS-a M vkSj N ,d nwljs ds fudVre iMkslh ugha gSa- L ds lkeus dkSu
→ Q faces K.
→ O is neither immediate neighbour of P nor Q.
gS\
(a) Q (b) N
(c) K (d) O

303
Download Free PDFs & e-Books from Neon Classes App

Sol. (d) X Z
→ J, K, L, M, N, O, P and Q in each four in two parallel row.
→ Q and N facing each other.
→ Q is an immediate neighbour of both P and O So 2 Possible
cases –
Case 1: Case 2: M O N
N N → P who is to the immediate left of Q, is opposite to Y.
X Z Y

P Q O O Q P
→ K is Sitting to the immediate left of O. M O P Q N
so Case 1 gets eliminated. → W who is immediately to the left of Y, is opposite Q.
→ K and J are diagonally opposite. X Z Y W V
N J

M O P Q N
K O Q P Hence, V is sitting opposite to N.
→ M and N are not immediate neighbours 28. There are 10 people sitting in 2 parellel opposite Row 1 and 2.
M L N J A,
B, C, D and E sitting faces towards North and V, W, X, Y and Z are
sitting in Row 2 facing the south, in such a way that each
member in row 1 faces a member of row 2. Neither A nor B sit
at any of the extreme ends of the row. A sits second to the right
K O Q P of B. the one who faces B is an immediate neighbour of Z. Y sits
Hence, O is in front of L. second to the left of Z. the one who faces Y sits second to the
27. V, W, X, Y and Z are five men sitting in a line facing the South - left of C. W sits second to the right of V. D is not an immediate
while M, N, O, P and Q are five ladies sitting in a second line neigbhour of A. who faces E?
parallel to the first line and are facing the North. W who is nl yksx nks lekukarj iafä;ksa 1 vkSj 2 esa cSBs gSaA A, B, C, D vkSj E
immediate next to the left of Y, is opposite. X and N are iafä 1 esa mÙkj dh vksj eq[k djds cSBs gSa vkSj V] W] X] Y vkSj Z iafä
diagonally opposite to each other. Z is opposite to O who is 2 esa nf{k.k dh vksj eq[k djds cSBs gSa] bl çdkj ls fd iafDr 1 esa
immediate next to the right of M. P who is to the immediate left çR;sd lnL; iafDr 2 ds lnL; dh vksj mUeq[k gSA u rks A vkSj u gh
of Q, is opposite to Y. M is at one end of the line. Who is sitting B iafä ds fdlh Hkh vafre Nksj ij cSBs gSaA A, B ds nk;sa ls nwljs LFkku
opposite to N? ij cSBk gSA og tks B dh vksj mUeq[k gS og Z dk fudVre iMkslh gSA
V, W, X, Y vkSj Z ,d iafDr esa nf{k.k dh vksj eq[k djds cSBs ikap Y] Z ds ck;sa ls nwljs LFkku ij cSBk gSA tks Y dh vksj mUeq[k gS] og C
iq:"k gSa & tcfd M, N, O, P vkSj Q ikap efgyka, gSa tks igyh iafDr ds ck;sa ls nwljs LFkku ij cSBk gSA W, V ds nk;sa ls nwljs LFkku ij cSBk
ds lekukarj nwljh iafDr esa cSBh gSa vkSj mÙkj dh vksj mUeq[k gSaA W, gSA D, A dk fudVre iM+kslh ugha gSA ftldk eq[k E dh vksj gS\
tks Y ds ck;sa ds Bhd cxy esa gS] ds foijhr gS X vkSj N ,d nwljs ds (a) Y (b) W
frjNs foijhr gSaA Z, O ds foijhr gS] tks M ds Bhd cxy esa gSA P, tks (c) V (d) X
Q ds Bhd ckbZa vksj gS] Y ds foijhr gSaA M js[kk ds ,d Nksj ij gSA N Sol. (a)
ds foijhr dkSu cSBk gS\ A, B, C, D, E Row 1 facing North.
(a) Y (b) W V, W, X, Y, Z Row 2 facing South.
(c) V (d) X → Neither A nor B sit at any of the ends.
Sol. (c) → A sits second to the right of B.
V, W, X, Y, Z facing South → The one who faces B is an immediate neighbour of Z.
M, N, O, P, Q facing North in paralles row So here 2 possible cases –
→ M is at one end of line. Case 1: Case 2:
→ Z is opposite to O who is immediate right of M so M is Z Z
definitely at left end.
Z

B A B A
→ Y sits second to the left of Z.
M O
→ D is not an immediate neighbour of A.
→ X and N are diagonally opposite

304
Best App for Govt. Jobs : Neonclasses (Download Now)

Case 1: Case 2: → C is third to the right of B, C is not an immediate neighbour


Z Y Z of either E or A.
→ A is not facing R.
2 possible cases there
Case 1: Case 2:

R S Q O P T R S Q O P T
D B A D B A
→ The one who faces Y sits second to the left of C. So case 2
gets eliminated.
→ W sits second to the right of V.
Z W Y V X E B A F C D E A B F D C
Hence,
According to option RE, TC pairs are sitting at the ends in case
2
30. There are two benches A and B and one chair. Three persons
can sit on each of these benches and one on the chair. Seven
D B E A C persons, Aman, Bhaskar, Chetan, Dablu. Ekta, Fauzia, and
Hence, Y is faces E. Ganesh', are to be seated on these benches and the chair
29. Twelve friends are sitting in two parallel lines. There are 6 following certain constraints as given.
people sitting in each row and facing a person of the other a A vkSj B vkSj ,d dqlÊ gSaA buesa ls çR;sd csp
nks csp a ij rhu O;fä
row. In row 1, A, B, C, D, E and F are seated and all of them are cSB ldrs gSa vkSj ,d dqlÊ ijA lkr O;fä] veu] HkkLdj] psru]
facing north. In row 2, O, P, Q, R, S and T are seated and all of McywA ,drk] QkSft;k] vkSj x.ks'k ' dks bu cspa ksa ij cSBk;k tkuk gS
them are facing south but not necessarily in the same order. C, vkSj dqN ckèkkvksa dk ikyu djrs gq, dqlÊ nh xbZ gSA
is third to the right of B. C is not an immediate neighbour to (i) Fauzia does not sit on the same bench as Deblu
either E or A. P is third to the left of S. Neither P nor S are QkSft;k nscyw ds leku csap ij ugha cSBrh gS
sitting at the extreme ends of any row. F faces O who is second (ii) Ekta does not sit on the same bench as Dablu.
to the right of T and T is not an immediate neighbour to S. Only ,drk nscyw ds leku csp
a ij ugha cSBrh gSA
two people are sitting between Q and T. A, is not facing R.
(iii) Chetan can sit with Aman but not with Ganesh.
Which of the following pairs are sitting at the extreme ends?
psru veu ds lkFk cSB ldrk gS ysfdu x.ks'k ds lkFk ughaA
ckjg fe= nks lekukarj js[kkvksa esa cSBs gSaA çR;sd iafä esa 6 O;fä cSBs
(iv) Aman can sit with Dablu but not with Bhaskar or Ganesh.
gSa vkSj nwljh iafä ds O;fä dk lkeuk dj jgs gSaA iafä 1 esa] A] B] veu nscyw ds lkFk cSB ldrs gSa ysfdu HkkLdj ;k x.ks'k ds lkFk
C] D] E vkSj F cSBs gSa vkSj mu lHkh dk eq[k mÙkj dh vksj gSA iafä
ughaA
2 esa] O] P] Q] R] S vkSj T cSBs gSa vkSj mu lHkh dk eq[k nf{k.k dh (v) Ganesh does not sit on the same bench as Fauzia.
vksj gS ysfdu vko';d ugha blh Øe esa gksA C] B ds nk;sa ls rhljs x.ks'k QkSft;k ds leku csap ij ugha cSBk gSA
LFkku ij gSA C uk rks E uk A dk fudVre iM+kslh ugha gSA P] S ds If Fauzia sits on Bench A and Chetan sits on Bench B. then who
ck;sa ls rhljk gSA u rks P vkSj u gh S fdlh Hkh iafä ds vafre Nksj is sitting in the chair?
ij cSBs gSaA F] O dh vksj mUeq[k gS] tks T ds nk;sa ls nwljs LFkku ij ;fn QkSft;k csp a A ij cSBrk gS vkSj psru csp
a B ij cSBrk gSA rks
gS] vkSj T] S dk fudVre iM+kslh ugha gSA Q vkSj T ds chp dsoy nks dqlÊ ij dkSu cSBk gS\
O;fä cSBs gSaA A] R dh vksj mUeq[k ugha gSA fuEufyf[kr esa ls dkSu (a) Bhaskar (b) Ekta
lk tksM+k uhps cSBs gSa \ (c) Ganesh (d) Dablu
(a) SB, PC (b) RB, TC Sol. (c)
(c) RE, TC (d) SA, PD Two benches and one chair are there 3 persons each of these
Sol. (c) benches and one on chair so total 7 persons –
In Row1 A, B, C, D, E, F facing North Now from point (iii) and (iv)
In Row2 O, P, Q, R, S, T facing South Chetan, Aman and Dablu together on same bench.
→ P is third to the left of S. Now from point (i), (ii), (iv) position of Bhaskar, Ganesh,
→ Neither P nor S are sitting at the extreme ends. Favzia, Ekta not confirmed so –
S P
Bench
Bench B Chair
A
Fauzia/
Cheta Bha
→ T is not immediate neighbour of S. Fauzia/
n skar
→ F faces O, Who is second to the right of T Bhaskar/
Aman /
S O P T Ganesh/Ekta
Dablu Ganesh/
Ekta

From question If fauzia sits on bench A and Chetan sits on


Bench B -
F
→ Only two people are between Q and T.

305
Download Free PDFs & e-Books from Neon Classes App

Bench A Bench B Chair (1) Left-right –The left-right of a person depend on the direction
Fauzia Chetan of his face. For e.g. 8 persons A,B,C,D,E,F,G and H are sitting
Bhaskar Aman Ganesh around a circle.
Ekta Dablu ck;k¡&nk;k¡&fdlh O;fDr dk ck;k¡&nk;k¡ ml O;fDr ds eq[k dh fn’kk
ij fuHkZj djrk gSA tSls & vkB O;fDr A,B,C,D,E,F,G vkSj H ,d o`r
Ganesh is sitting in the chair. ds pkjksa vksj cSBs gSA
31. In a cafeteria, seven customers (B, G, H, K, L, M and N) sit on A,B,E and H are facing the centre while C,D,F and G are facing
three different benches (X, Y and Z). Each bench must have at opposite to the centre.
least two customers sitting on it. Customer G does not sit with buesa ls A,B,E vkSj H dsna z dh vksj dq¡g djds cSBs gS tcfd C,D,F
customers K, L and M. Customer B sits only with customer N. vkSj G dsUnz ls ckgj eq¡g djds cSBs gSA
Customer K sits on Bench X with his best friends. Customer H A Left
sits on Bench Z. On which bench are there three customers? Right
,d dSQsVsfj;k es]a lkr xzkgd ¼B] G] H] K] L] M vkSj N½ rhu Right Left
vyx&vyx csp a ¼X] Y vkSj Z½ ij cSBrs gSaA çR;sd csp
a ij de ls C D
de nks xzkgd cSBs gksAa xzkgd G xzkgd K, L vkSj M ds lkFk ugha cSBrk Left Right
gSA xzkgd B dsoy xzkgd N ds lkFk cSBrk gSA xzkgd K vius lcls Left Right
a X ij cSBrk gSA xzkgd H csp
vPNs nksLrksa ds lkFk csp a Z ij cSBrk gSA B
fdl csp
a ij rhu xzkgd gSa\ To the right of A = C
(a) Only Bench X (2) Immediate Neighbour: - The immediate neighbours of a
(b) Bench X or Bench Y person are those who are sitting close to that person. For e.g.
(c) Only Bench Z in the above figure-
(d) Only Bench Y iM+kslh ¼Immediate Neighbour½%& fdlh O;fDr ds iM+kslh og
Sol. (a) O;fDr gksaxs tks ml O;fDr ds fcYdqy ikl cSBs gSA tSls mi;ZqDr fp=
Seven Customers B, G, H, K, L, M, N esa gS&
Three benches X, Y, Z The immediate neighbours of A is C & D
atleast two customers on each benches. Similarly we will find out the immediate neighbours of other
→ K sits on bench X with his friends. persons.
→ H sits on bench Z blh izdkj vU; O;fDr ds iM+kslh Kkr djsaxs&
(3) Next to: The words “Next to” and Immediate Neighbour’ have
Bench → X Y Z the same meaning. For e.g. the above figure-
Customers → K H vxys LFkku ij (Next to) &Þvxys LFkku ijÞ vkSj ^iM+kslh^ nksuksa dk
vFkZ fcYdqy ,d tSlk gksrk gSA tSls mi;ZqDr fo= es&
a
Next to are : B & H
→ B sits only with customer N So they both definatly sits on
Similarly we can find out it for other persons.
bench Y.
blh izdkj vU; O;fDr;ksa ds vxys LFkku ij cSBs O;fDr Kkr dj
ldrs gSA
X Y Z
(4) Between: If three persons are sitting together then the person
K B, N H sitting in the middle position is said to be the between of the
other two persons For e.g. in the above figure.
→ G does not sit with K, L, M So G definatly sit with H and L & chp esa [Between]:- fdUgh rhu ,d lkFk cSBs O;fDr;ksa esa e/; LFkku
M sit with K. ij cSBk vU; nks O;fDr;ksa ds Bhd chp esa dgykrk gS
tSlk mi;ZqDr fp= es&
a
X Y Z Between B and H : A
Similarly we can find out it for other persons.
K, L, M B, N H, G
blh izdkj vU; ds fy, Hkh chp esa O;fDr Kkr dj ldrs gSA
(5) Opposite: If two persons are sitting in the opposite directions,
Hence, Only Bench X are three customers.
they are called opposite to each other. For e.g. in the above
Concept-III : Close Form : Circuler arrangement ( Single &
figure.
Double layer) ( 4,5,6,7, & 8 Persons are given) Inner and foijhr (Opposite): nks O;fDr tks ,d&nwljs dh foijhr fn’kkvksa esa
Outer Facing
cSBs gS vFkkZr~ vkeus&lkeus cSBs gSA ,sls O;fDr ,d&nwljs ds lEeq[k gks
Such arrangement is closed from all sides. For e.g.- a group of
Hkh ldrs gS vkSj ugha HkhA tSls mi;ZDq r fp= es&a
people can sit around a circle. Similarly square. Rectangle can
B and F are opposite to each other
sit around a circle. Similarly square, rectangle, pentagon etc.
(6) Facing: Facing means two persons are sitting in the oppsotie
,slh O;oLFkk pkjksa vksj ls can gksrh gSA tSls & ge O;fDr;ksa ds lewg
directions and facing each other For e.g. in the above figure-
dks o`r pkjksa vksj cSBk ldrs gSA blh izdkj oxkZdkj] vk;rkdkj]
lEeq[k [Facing]- nks O;fDr tks ,d&nwljs dh foijhr fn’kkvksa esa cSBs
iapHkqtkdkj vkfn Hkh can O;oLFkk esa gh vk,axsA
gS vkSj ,d&nwljs lEeq[k gSA
To solve close form of questions we should know the meaning
A and E are facing each other.
of some basic word.
Note: B & F and D & H are opposite to each other but they
can O;oLFkk ls lacaf/kr iz’uksa dks gy djus ds fy, lcls igys ge
are not facing each other.
dqN vk/kkjHkwr 'kCnksa dk vFkZ tkusaxsA

306
Best App for Govt. Jobs : Neonclasses (Download Now)

uksV%& ;gk¡ B o F,C o G vkSj D o H ,d &nwljs ds lEeq[k ugha (1) First read the given data carefully. After it draw the shape
cSBs gSA os dsoy ,d&nwljs ds foijhr cSBs gSA mentioned in the question. For e.g. – If all people are sitting
(7) Some other words: around the circle, we will draw a circle. Similarly if all people
dqN vU; 'kCn% are sitting around a rectangular table, we will draw rectangle.
(i) Who, which, that –these words are used for object. lcls igys iz’u esa nh xbZ lwwpukvksa dks /;ku ls i<+sA mlds ckn og
^tks^ ¼Who, which, that½& bu 'kCnksa dk iz;ksx deZ ¼Object½ ds fy, vkd`fr cuk, ftldk iz’u esa mYys[k fd;k x;k gSA tSls&;fn lHkh
gksrk gSA O;fDr o`Ùk ds pkjksa vksj cSBs gS rks o`Ùk cuk,axsA ;fn lHkh O;fDr vk;r
(ii) And, while whereas, But-These words are used for subject. ds pkjksa vkSj cSBs gS rks vk;r cuk,axsA
^vkSj^ ¼and½] ^tcfd^ ¼While, whereas½] ^ysfdu^ (But) &bu 'kCnksa (2) Now, Considering all the people in the question mark the
dk iz;ksx drkZ ¼Subject½ ds fy, gksrk gSA slots as given below:
Close Form Arrangement:- vc izz’u esa ftrus O;fDr gS mu lHkh O;fDr;ksa dks ,d&lkFk cSB k nsA
There are mainly three types of close from arrangements
can O;oLFkk ls lacaf/kr O;oLFkk,¡ eq[;r% rhu izdkj dh gksrh gSA
(1) When all the people are sitting facing the centre.
tc lHkh O;fDr dsna z dh vksj eq¡g djds cSBs gks&

Here 6 persons are sitting around the circle.


tSls &N% O;fDr ,d o`Ùk ds pkjksa vksj cSBs gS rks mls fuEu izdkj
n’kkZ,axs&
(3) If it is known whether people facing the centre or opposite
to the centre, then we will also mark it.
(2) When all the people are sitting facing opposite to the ;fn lHkh O;fDr;ksa ds eq¡g dh fn’kk Kkr gks rks vkd`fr esa og Hkh
centre. fpfUgr dj nsAa
tc lHkh O;fDr dsna z ls ckgj dh vksj eq¡g djds cSBs gks& (4) In Close Form question we can make the first person sit
any whereas per our wish. After it using all definite points, We
will fill up as many empty places as possible.
can O;oLFkk ds iz’uksa esa ge igys O;fDr dks viuh bPNkuqlkj dgh
Hkh cSBk ldrs gS vkSj mlds ckn iz’u esa fn, x, fuf’pr fcUnqvksa dk
iz;ksx djrs gq, T;knk ls T;knk [kkyh LFkkuksa ij vU; O;fDr;ksa dks
cSBk,axs
(5) If there are two or more possibilities of seating individuals
while solving, the questions. In the end the possibility which
does not violate any condition, will be correct.
(3) Mixed Arrangement: When some persons are facing the
;fn iz’u dks gy djrs le; O;fDr;ksa dks cSBkus dh nks ;k vf/kd
centre while others are facing opposite to the centre, it is
laHkkouk lgh gksxhA tks iz’u dh 'krksaZ ds fo:) ugha gksA
called mixed arrangement.
32. Drake, Pawan, Gagan, Laxman are sitting around a circular
fefJr O;oLFkk%& dqN O;fdr dsna z dh vksj eq¡g djds cSBs gS vkSj dqN
table facing the centre(but not necessarily in the same order).
O;fDr dsna z ls ckgj dh vksj eq¡g djds cSBs gS rks bls fefJr O;oLFkk
Drake is to the immediate right of Gagan, and Laxman is
dgrs gSA exactly between Drake and Pawan. Which of the following is
true about the arrangement?
Mªsd] iou] xxu] y{e.k] ,d o`Ùkkdkj est ds pkjks vksj dsna z dh vksj
eq[k djds cSBs gSa ¼ysfdu vko’;d ugha fd blh Øe esa gks½a A Mªsd]
xxu ds Bhd nk;sa gSa] vkSj y{e.k] Mªsd vkSj iou ds Bhd chp esa gSaA
O;oLFkk ds ckjs esa fuEufyf[kr esa ls dkSu lk lR; gS\
(a) Pawan is to the immediate left of Laxman/ iou y{e.k ds
rRdky ckbZa vksj gSA
(b) Drake is to the immediate neighbour of pawan and Gagan/
Note: When it is not mention in the question whether all
Msªd] iou vkSj xxu dk fudVre iMkslh gSaA
the persons are facing centre or opposite to the centre, we
(c) Drake is to the immediate left of Pawan/ Msªd] iou ds Bhd
will solve the question by assuming the face of all the
people towards the centre.
ck;sa gSaA
tc iz’u esa ;g ugha crk;k tk, fd O;fDr;ksa dk eq¡g dsna z dh vksj gS (d) Pawan is to the immediate left of Gagan/ iou xxu ds Bhd
;k dsna z ls ckgj dh vksj gS rks ge lHkh O;fDr;ksa dk eq¡g dsUnz dh ck;sa gSA
vksj ekudj iz’u gy djsx a sA Sol. (d)
 Drake is immediate right of Gagan. Now starting from
To solve the close from questions we follow the steps given
below: Gagan.
can O;oLFkk ls lacaf/kr iz’uksa dks fuEu rjg ls gy djsaxs&

307
Download Free PDFs & e-Books from Neon Classes App

W
Drake
left
T
right V
Gagan

U
 Laxman is exactly between Drake and Pawan so final Hence, W is sitting immediate left of S.
Arrangement is – Correct answer is option (D).
34. Five men O1, O2, O3, O4 and O5 are sitting around a circular
Laxman table facing opposite to the centre (not necessarily in the same
order). Only one man is sitting between O4 and O5 in
clockwise direction. Only one man is sitting between O3 and
O4 in clockwise direction. O1 is sitting to the immediate left of
O3. Who is sitting third from the left of O2?
ikap iq:"k O1, O2, O3, O4, vkSj O5 ,d o`Ùkkdkj est ds pkjksa vksj
Pawan Drake dans z ds foijhr eq[k djds cSBs gSa ¼t:jh ugha fd blh Øe esa gks½a A
dsoy ,d O;fDr O4 vkSj O5 ds chp nf{k.kkorZ fn’kk esa cSBk gSA
dsoy ,d O;fDr O3 vkSj O4 ds chp nf{k.kkorZ fn’kk esa cSBk gSA O1,
O3 ds Bhd ck;sa cSBk gSA O2 ds ck,a ls rhljs LFkku ij dkSu cSBk gS\
(a) O1 (b) O5
Gagan (c) O4 (d) O3
 Hence,
Sol. (a)
option (D), Pawan is the immediate left of Gagan is true.
 01 is sitting to immediate left of 03.
Correct answer is option (D).
33. S, T, U, V and W are sitting around a circular table facing the
centre (but not necessarily in the same order). S is the
immediate neighbour of T and W. U is to the immediate right
of T. V is the immediate neighbour of U and W. Who is sitting
to the immediate left of S?
S, T, U, V vkSj W ,d o`Ùkkdkj est ds pkjks vksj dsanz dh vksj eq[k
djds cSBs gSa ¼ysfdu t:jh ugha fd blh Øe esa gks½a A S, T vkSj W dk 03
fudVre iMkslh gS- U, T ds Bhd nk;sa cSBk gS- V, U vkSj W dk
fudVre iMkslh gS- S ds Bhd ck;sa dkSu cSBk gS\ 01
(a) V (b) U  Only one man is sitting between 03 and 04 in clockwise
(c) T (d) W direction.
Sol. (d) 5 persons S,T,U,V,W facing centre.
 U is immediate right of T.
V is immediate neighbour of U and W. So starting from U.

W
04
03
T V 01
 Only one man is sitting between 04 and 05 in clockwise
left right direction.

 S is immediate neighbour of T and W.

308
Best App for Govt. Jobs : Neonclasses (Download Now)

Chirag
05 (Coach)

04
Bablu
03 (Boxer)
Amit
01 (bo) The Doctor and Boxer are immediate
→ Deepak is a doctor.
 The final arrangement is – neighbours of each other.
Chirag Deepak
05 02 (Coach) (Doctor)

04 Eklavya Bablu
03 Amit (Boxer)
(bo) right of Chirag.
Hence, Eklavya is immediate
01
36. Raghav, manpreet, Sunaina, Prakrit, Vedika and Arwa are
Hence, 01 is sitting third to the left of 02.
sitting around a circular table facing the centre. Raghav is
Correct answer is (A).
second to the left of Prakrit. Manpreet is to the immediate left
35. Five friends Amit, Bablu, Chirag, Deepak and Eklavya, each
of Sunaina. Arwa is second to the right of Vedika. There are an
having different occupations Advocate, Boxer, Coach, Doctor
equal number of persons between Raghav and Manpreet from
and Engineer, are sitting around a circular table facing the
both the ends. Who is sitting third to the left of Sunaina?
cente.
jk?ko] euizhr] lquSuk] izkd`r] osfndk vkSj vjok d, xksy est ds
ikap fe= vfer] ccyw] fpjkx] nhid vkSj ,dyO;] çR;sd ds
ifjr% blds dsna z dh vksj eq[k djds cSBs gSaA jk?ko] izkd`r ds cka nwljs
vyx&vyx O;olk; ,MoksdVs ] c‚Dlj] dksp] M‚DVj vkSj bathfu;j
LFkku ij gSA euizhr] lquSuk dh ck;ha vksj Bhd cxy esa cSBk gSA
gSa] dsaæ dh vksj eqag djds ,d xksykdkj est ds pkjksa vksj cSBs gSaA
vjok] osfndk ds nk;sa nwljs LFkku ij gSA jk?ko vkSj euizhr ds chp
1. Bablu is a Boxer and Deepak is a Doctor.
ccyw ,d c‚Dlj gS vkSj nhid ,d M‚DVj gSA nksuksa Nksj ls leku la[;k esa O;fDr gSaA lquSuk ds ck;sa rhljs LFkku ij
dkSu cSBk gS\
2. Amit sits second to the right of Chirag, who is a coach.
vfer] fpjkx ds nk;sa ls nwljs LFkku ij cSBk gS] tksfd ,d dksp gSA (a) Raghav (b) Vedika
(c) Arwa (d) Prakrit
3. The Boxer will never sit immediately next to the Coach.
Sol. (c)
c‚Dlj dHkh Hkh dksp ds Bhd cxy esa ugha cSBsxkA
→ Raghav is second to the left of prakrit.
4. The Doctor and Boxer are immediate neigbhours of each
→ There are an equal number of persons between Raghav and
other.
Manpreet from both the ends it means they are facing opposite
M‚DVj vkSj c‚Dlj ,d nwljs ds rRdky iM+kslh gSaA
to each other –
Who sits to the immediate right of Chirag?
Raghav
fpjkx ds Bhd nk;sa dkSu cSBk gS\
(a) Deepak (b) Bablu
(c) Amit (d) Eklavya
Sol. (d)
5 friends Amit, Bablu, chirag, Deepak, & Eklavya, 5 Occupation
Advocate, Boxer, Coach, Doctor and Engineer.
Manpreet
→ Amit Sits Second to right of Chirag, Who is a coach.
Prakrit
Chirag
(Coach) → Manpreet is to the immediate left of Sunaina.
Raghav Sunaina

Amit Manpreet
→ Bablu is a Boxer. The Boxer will never sit immediately next Prakrit
to the Coach. → Arwa is second to the right of Vedika.

309
Download Free PDFs & e-Books from Neon Classes App

Vedika (c) Manju (d) Priya


Raghav Sunaina Sol. (a)
→ All are facing outside.
→ Vansh is sitting second to the right of Rajat.
Vansh

Arwa Manpreet
Prakrit
Hence, Arwa is third to the left of Sunaina – Right Left
37. Study the given information carefully and answer the question Rajat
that follows. → Manju is Sitting between Rajat and Arun.
nh xbZ tkudkjh dk è;kuiwoZd vè;;u djsa vkSj uhps fn, x, ç'u Vansh
dk mÙkj nsaA
In a school party six girls are sitting in a circle facing the
centre. Pragathi is sitting to the immediate left of Keerthi.
Kavya is an immediate neighbour of both Pragathi and
Vaishali. Meghana is an immediate neighbour of both Keerthi
and Vasantha. Who is sitting to immediate left of Vaishali? Manju
,d Ldwy ikVÊ esa Ng yM+fd;ka ,d ldZy esa dsæa dh vksj eq[k djds Rajat
cSBh gSaA çxfr dhfrZ ds Bhd ck;sa cSBh gSA dkO;k çxfr vkSj oS'kkyh → Arun and Priya are not the neighbours of Rajat.
nksuksa dh fudVre iMkslh gS- es?kuk dhfrZ vkSj olark nksuksa dh → Bhavna is remaining person.
fudVre iMkslh gS- oS'kkyh ds Bhd ck;sa dkSu cSBk gS\ Priya
(a) Meghana (b) Vasantha Vansh
Arun
(c) Kavya (d) Pragathi
Sol. (b)
→ Pragathi is Sitting immediate left of Keerthi
→ Meghna is an immediate neighbour of both keerthi and
Vasantha.
Vasantha Bhavna Manju
Rajat
Hence, Bhavna is Sitting between Vansh and Rajat.
Figure direction is wrong
39. Seven friends, Parv, Qazi, Raees, Sushil, Tania, Uma, and
Vishnu are sitting around a circular table facing the centre.
Pragathi Meghna Parv is immediate neighbour of both Tania and Sushil. Uma is
Keerthi immediate neighbour of both Qazi and Vishnu. Qazi is sitting
→ Kavya is an immediate neighbour of both Pragathi and second to the right of Tania. Who is sitting to the immediate
Vaishali. left of Qazi?
Vaishali lkr fe= ioZ] dkth] jbZl] lq'khy] rkfu;k] mek vkSj fo".kq ,d
Kavya Vasantha o`rkdkj est ds pkjks vksj dsaæ dh vksj eq[k djds cSBs gSaA ioZ rkfu;k
vkSj lq'khy nksuksa dk fudVre iMkslh gS- mek] dkth vkSj fo".kq nksuksa
dh fudVre iMkslh gS- dkth rkfu;k ds nk;sa ls nwljs LFkku ij cSBk
gSA dkth ds Bhd ck;sa dkSu cSBk gS\
(a) Uma (b) Sushil (c) Raees (d) Vishnu
Sol. (c)
Pragathi Meghna
Qazi is sitting second to the right of Tania.
Keerthi
Hence, Vasantha is immediate left of Vaishali.
38. Six friends, Priya, Arun, Vansh, Rajat, Bhavna and Manju, are
sitting around a circular table, with their backs towards the Qazi
centre. Vansh is sitting second to the right of Rajat. Arun and
Priya are not the neighbours of Rajat. Manju is sitting between
Rajat and Arun. Who is sitting between Vansh and Rajat?
N: fe= fç;k] v#.k] oa'k] jtr] Hkkouk vkSj eatw ,d o`Ùkkdkj est ds Tania
pkjksa vksj viuh ihB dsæa dh vksj j[krs gq, cSBs gSaA oa'k jtr ds nk;sa → Uma is immediate neighbour of both Qazi and Vishnu.
ls nwljs LFkku ij cSBk gSA v#.k vkSj fç;k jtr ds iM+kslh ugha gSaA → Parv is immediate neighbour of both Tania and Sushil
eatw jtr vkSj v#.k ds chp cSBh gSA oa'k vkSj jtr ds chp esa dkSu
cSBk gS\
(a) Bhavna (b) Arun

310
Best App for Govt. Jobs : Neonclasses (Download Now)

Vishnu
R
Sushil
Uma
I
Qazi

S
Parv A
Tania → D and B adjcent to each other So case 1 & case 3 eliminated.
→ Raees is the remaining. So – I
Vishnu B R
Sushil
Uma
T
Qazi

D S
Parv Raees A
Tania Hence, S is the immediate neighbour of T and A.
Hence, Raees is immediate left of Qazi. 41. Seven friends, Subhi, Prince, Ketan, Vishal, Mahima, Krish and
40. A, B, R, D, I, S, and T are sitting around a circular table facing Naitik, are sitting around a circular table with their backs
away from the circle. A does not sit adjacent to I and T. B does towards the centre. Subhi is sitting to the immediate right of
not sit adjacent to A. R sits fourth to the left of A. D and B sit Prince. Naitik and Krish are not sitting to the immediate left or
adjacent to each other. S sits third to the left of I. Who is the right of Vishal. Vishal is sitting third to the left of Prince.
immediate neighbour of both T and A? Mahima is sitting to the immediate right of Vishal. Who is
A, B, R, D, I, S vkSj T ,d o`Ùkkdkj est ds pkjksa vksj o`Ùk dh vksj eq[k sitting third to the right of Prince?
djds cSBs gSaA A, I vkSj T ds vklUu ugha cSBk gSA B, A ds vkLkUu ugha lkr fe=] lqHkh] jktdqekj] dsru] fo'kky] efgek] —"k vkSj uSfrd] ,d
cSBk gSA R, A ds ck,a ls pkSFks LFkku ij cSBk gSA D vkSj B ,d nwljs ds o`Ùkkdkj est ds pkjksa vksj viuh ihB dsæa dh vksj j[krs gq, cSBs gSaA
vklUu cSBs gSaA S, I ds ck;sa ls rhljs LFkku ij cSBk gSA T vkSj A nksuksa lqHkh] fçal ds Bhd nk;sa cSBh gSA uSfrd vkSj —"k fo'kky ds Bhd ck;sa
dk fudVre iMkslh dkSu gS\ ;k nk;sa ugha cSBs gSaA fo'kky] fçal ds ck;sa ls rhljs LFkku ij cSBk gSA
(a) R (b) T efgek] fo'kky ds Bhd nk;sa cSBh gSA fçal ds nk;sa ls rhljs LFkku ij
(c) S (d) B dkSu cSBk gS\
Sol. (c) (a) Naitik (b) Krish
→ R Sits fourth to the left of A. (c) Ketan (d) Mahima
→ A does not sit adjacent to I and T. Sol. (c)
→ B does not sit adjacent to A. → Surbhi is sitting to the immediate right of Prince.

Surbhi
A
Prince
→ S sits third to the left of I.
So three possible Cases – → Vishal is Sitting third to the left of Prince.
Diagram error case 2 → Mahima is Sitting to the immediate right of Vishal.
Case 1: Case 2: Vishal
I
I
R R
Mahima
S

Surbhi
S Prince
A A → Naitik and krish are not Sitting to the immediate left or right
Case 3: of Vishal.
→ Only Ketan left –

311
Download Free PDFs & e-Books from Neon Classes App

Naitik/ Ketan 43. Bindu, Krunal, Manoj, Nilima, Omkar, Piyush, Renu and Sundar
krish Vishal are sitting around a circular table facing the centre, but not
necessarily in the same order. Sundar sits second to the right
of Piyush. Only two persons sit between Sundar and Krunal.
Mahima Omkar sits opposite to Bindu, who is not an immediate
neighbour of Krunal and Piyush. Nilima is the immediate
Krish/ neighbour of Krunal and Bindu. Manoj sits third to the right of
Surbhi Naitik Bindu.
Prince
fcanq] Øq.kky] eukst] uhfyek] vksd
a kj] ih;w"k] js.kq vkSj laqnj ,d xksy
est ds ifjr% blds dsna z dh vksj vfHkeq[k gksdj cSBs gSa] ysfdu
Hence, Ketan is third to the right of prince,
vko’;d ugha fd os blh Øe esa gksAa lqna j] ih;w"k ds nk;sa nwljs LFkku
42. Seven friends S, T, U, V, W, X and Y, are sitting around a circle
with their backs towards the centre. There are three persons
ij cSBk gSA lqanj vkSj Øq.kky ds chp dsoy nks O;fDr cSBs gSaA vksd a kj]
between Y and V. W is sitting to the immediate right of V. U is fcanq ds lkeus cSBk gS] tks fd Øq.kky vkSj ih;w"k dh fudVre iM+kslh
sitting between Y and X. T is not the neighbour of W or Y. S is ugha gSA uhfyek Bhd Øq.kky vkSj fcanq ds iM+ksl esa gSA eukst] fcanq ds
the neighbour of W. Who is sitting between V and X? nk;sa rhljs LFkku ij cSBk gSA
lkr fe= S, T, U, V, W, X vkSj Y ,d o`Ùk ds pkjksa vksj viuh ihB dsæa Which of the following statements is/are true?
dh vksj j[krs gq, cSBs gSaA Y vkSj V ds chp rhu O;fä gSaA W, V ds Bhd fuEufyf[kr esa ls dkSu lk@ls dFku lR; gS@gSa\
nk;sa cSBk gSA U, Y vkSj X ds chp cSBk gSA T, W ;k Y dk iM+kslh ugha gSA (a) Manoj is the immediate neighbour of Nilima./ eukst Bhd
S, W dk iM+kslh gSA V vkSj X ds chp dkSu cSBk gSA uhfyek ds iM+ksl esa gSA
(a) T (b) S (b) Sundar sits third to the right of Krunal./ lqanj] Øq.kky ds nk;sa
(c) Y (d) U rhljs LFkku ij cSBk gSA
Sol. (a) (c) Nilima sits opposite the one who sits to the immediate left
→ W is Sitting to the immediate right of V. of Renu./ uhfyek ml O;fDr ds lkeus cSBh gS tks js.kq ds Bhd ck;sa
→ There are three persons between Y and V. iM+ksl esa cSBk gSA
there 2 possible cases – (d) Omkar is the immediate neighbour of Renu and Piyush./
Case 1: Case 2: Y vksd
a kj Bhd js.kq vkSj ih;w"k ds iM+ksl esa gSA
Sol. (c)
Y
→ Omkar Sits Opposite to Bindu.
→ Manoj Sits third to the right of Bindu.
Omkar
Manoj

W W
V V
→ S is the neighbour of W.
→ U is Sitting between Y and X.
Case 1: Case 2: Y Bindu
S Y S U → Bindu is not an immediate neighbour of Krunal and Piyush.
→ Sundar Sits Second to the right of Piyush.
Omkar
X Piyush
U Manoj

W X W
V V
→ T is not the neighbour of W or Y. Sundar
So case 1 Gets Eliminated. Bindu
Y → Nilima is the immediate neighbour of Krunal and Bindu.
S U → Renu is remainning one.
Omkar
Piyush
Manoj
X

Renu Krunal
W T
V
Sundar Nilima
Hence, T is Sitting between V and X.
Bindu

312
Best App for Govt. Jobs : Neonclasses (Download Now)

Hence, Nilima sits opposite the one who sits to the immediate the same table. P is sitting opposite K at the outer table. N is
left of Renu. between P and K at the same table. L is between M and O at the
44 Eight girls X, Y, Z, U, V, W, S and T are sitting around a circular same table. Who is sitting in the north at the outer table?
table, facing towards the centre (not necessarily in the same vkB fe= K, L, M, N, O, P, Q vkSj R nks o`Ùkkdkj estksa ds ifjr% cSBs gSa]
order). X is second to the left of U. U is third to the left of V. Z is ftuesa ls ,d NksVs vkdkj dh vkrafjd est vkSj ,d cM+s vkdkj dh
third to the right of S. S is not the immediate neighbour of V. T ckgjh est gSA nksuksa o`Ùkkdkj estksa ds dsna z leku gSaA izR;sd est ij
is third to the right of Y. Y is second to the right of S.Three out pkj O;fDr ,d&nwljs ls leku nwjh ij dsUnz dh vksj eq[k djds cSBs
of the following four options are similar by a certain logic and gSaA M, L ds nkbZ vksj Bhd cxy esa cSBk gSA Q vkSj N nf{k.k esa cSBs gSa
form a group. Which of the following does NOT belong to that vkSj os R vkSj L dh vksj eq[k djds cSBs gSaA M, leku est ij O ds
group? lkeus cSBk gSA P, ckgjh est ij K ds lkeus cSBk gSA N leku est
vkB yM+fd;a X, Y, Z, U, V, W, S vkSj T ,d o`Ùkkdkj est ds pkjksa vksj ij P vkSj K ds chp esa cSBk gSA L leku ij M vkSj O ds chp esa
dsUa nz dh vksj mUeq[k gksdj cSBh gSa ¼t:jh ugha fd blh Øe es½a A X, U cSBk gSA ckgjh est ij mÙkj fn’kk esa dkSu cSBk gS\
ds ck;sa ls nwljs LFkku ij gSA U, V ds ck;as ls rhljk gSA Z, S ds nk;sa (a) K (b) L
ls rhljk gSA S, V dk fudVre iMkslh ugha gSA T, Y ds nk;sa dks rhljk (c) R (d) P
gSA Y, Y ds ck;sa ls nwljs LFkku ij gSA S ds nkbZa vksjA fuEufyf[kr Sol. (c)
pkj fodYiksa esa ls rhu ,d fuf’pr rdZ ds leku gSa vksj ,d lewg K, L, M, N, O, P, Q, R are sitting at two circular tables, one small
cukrs gSaA fuEufyf[kr esa ls dkSu ml lewg ls lacfa /kr ugha gS\ inner table and one bigger outer table -
(a) YS (b) YZ → N is between P and K at the same table.
(c) WV (d) UX → Q and N are sitting in the South
Sol. (b) → P is Sitting opposite to K at the outer table –
→ X is second to the left of U. and U is third to the left of V. North

X West P/K K/P East

U
N
→ Z is third to the right of S. and S is not the immediate
South
neighbour of V.
→ M is to the immediate right of L and L is between M and O at
Here S is Possible in two places So –
Case 1:
the same table.
Case 2: → Q and N are facing R and L.
Z
R
V V

L
X X Z

P/K M O K/P
S S
U U Q
→ Y is Second to right of S so Case 1 gets eliminated.
→ T is third to the right of Y. N
T
Hence, R is Sitting in the North at the outer table.
W
V 49. Read the given information carefully and answer the question
the follows.
X
nh xbZ tkudkjh dks è;ku ls i<+sa vkSj uhps fn, x, ç'u dk mÙkj nsaA
Z
Ganesh, Raju, Rohit, Isha, Janak, Preet, Chandan and Madhur
are eight employees who are sitting for a discussion around a
Y circular table, facing its centre. Their professions are pilot,
S
U Doctor, publisher, Engineer, General Manger, Union Leader,
Reseracher and Dean, but not necessarily in the same order.
Hence, Except Option (b), in none of the given options, people
Raju is sitting second to the left of Preeti. The dean is to the
are immediate neighbours.
immediate left of Preeti. Only three people are there between
47. Eight friends, K, L, M, N, O, P, Q and R, are sitting at two circular
the Dean and Ganesh. There is only one person sitting between
tables, one smaller inner table and one bigger outer table. Both
the Engineer and Ganesh. The publisher is sitting to the
the circular tables have the same centre. Four persons are
immediate right of the Engineer. Madhur is sitting second to
sitting at each table at equal distances between them, facing
the right of Preeti. Isha is the Dean, Rohit and janak are
the centre. M is to the immediate right of L. Q and N are sitting
immediate neighbours of each other. Neither Rohit nor Janak
in the south and are facing R and L. M is sitting opposite to O at

313
Download Free PDFs & e-Books from Neon Classes App

is the Engineer. The general Manger is sitting to the immediate (Engineer)


left of Raju. The union Leader is sitting second to the right of
the Publisher. The Researcher is an immediate neighbour of
Ganesh
the Engineer. Rohit is sitting second to the right of the Pilot.
What is the position of Chandan with respect to the Dean?
x.ks'k] jktw] jksfgr] bZ'kk] tud] çhfr] panu vkSj eèkqj vkB deZpkjh gSa Raju
tks fdlh ppkZ ds fy, ,d o`Ùkkdkj est ds ifjr% dsUnz dh vksj eq[k
djds cSBs gSaA muds is’k & ik;yV] fpfdRld] izdk’kd] bathfu;j]
egkizca/kd] ;wfu;u yhMj] 'kks/kdrkZ vkSj Mhu gSa] ysfdu mudk blh (Dean)
Øe esa gksuk vfuok;Z ugha gSa jktw] izhfr ds ckbZa vksj nqljs LFkku ij Preeti
cSBk gSA Mhu] izhfr ds ckbZ vksj Bhd cxy esa cSBk gSA Mhu vkSj x.ks’k ⇒ Publisher immediate right of Engineer.
ds chp dsoy rhu O;fDr cSBs gSA bathfu;j vkSj x.ks’k ds chp dsoy ⇒ Isha is Dean.
,d O;fDr cSBk gSA izdk’kd] bathfu;j ds nkbZa vksj Bhd cxy esa cSBk ⇒ Madhur is Sitting second to the right of Preeti.
gSA e/kqj] izhfr ds nkbZa vksj nqljs LFkku LFkku cSBk gSA bZ’kk Mhu gSA Case 1:
jksfgr vkSj tud ,d & nqljs ds Bhd cxy esa cSBs gSA u rks jksfgr
vkSj u gh tud bathfu;j gSA egkizca/kd] jktq ds ckbZ vksj Bhd cxy
Ganesh
esa cSBk gSA ;wfu;u yhMj] izdk’kd ds nkbZa vksj nqljs LFkku ij cSBk
gSA 'kks/kdrkZ] bathfu;j ds Bhd cxy esa cSBk gSA jksfgr] ik;yV ds
nkbZa vksj nqljs LFkku ij cSBk gSA Mhu ds lkis{k panu dk LFkku dkSu Raju Madhur
lk gS\ (Publisher)
(a) Second to the right
(b) Third to the left (Dean) (Engineer)
(c) Third to the right Preeti
(d) Second to the left Case 2:
Sol. (a) (Engineer)
⇒ Raju is Sitting Second to the left of Preeti.
⇒ Dean is immediate left of Preeti.
Ganesh

Raju Madhur

Raju
Isha
(Dean) Preeti
(Dean) ⇒ The General Manager (GM) is immediate left of Raju.
Preeti
⇒ Rohit and Janak are immediate neighbourrs and Neither of
⇒ Only three people between Dean and Ganesh So both are them is Engineer. So case 2 gets eliminated.
opposite to each other. Rohit/Janak
⇒ Only One person between Engineer and Ganesh So 2 Rohit/janak
possible Cases – (GM) Ganesh
Case 1:
Raju Madhur (Publisher)
Ganesh

Isha
Raju (Dean) Preeti (Engineer)
⇒ Union leader is Second to right of the Publisher.
(Dean) (Engineer) ⇒ Researcher is an immediate neighbour of Engineer.
Preeti Rohit/Janak
Rohit/janak
Case 2: (GM) Ganesh

Raju Madhur (Publisher)

Isha
(Dean) Preeti (Engineer)
(Reasearcher)
⇒ Remainning Person is Chandan.
⇒ Rohit is second to the right of the Pilot.

314
Best App for Govt. Jobs : Neonclasses (Download Now)

So Ganesh is Pilot.
P
⇒ Raju is definatly doctor. Doctor Lecturer
Janak S
Q
(Union leader)
Rohit
(GM) Ganesh (Pilot)
J U

Raju Madhur (Publisher)


(Doctor) R T
Lawyer V
Isha Chandan Scientist
(Dean) Preeti (Engineer)
(Reasearcher) (50) S is a doctor. /S MkWDVj gSA
(51) Q is a lecture./Q ysDpj gSA
Hence, Chandan is Second to the right of Dean.
(52) J is the wife of the lwayer/J odhy dh iRuh gSA
Direction(50-54) Study the following information carefully
and answer the questions given below: (53) R sits on the immediate left of the scientist and R is the
J,P,Q,R,S,T,U and V are married couple sitting around a circle, husband of J.
Facing the centre. The profession of the males within the oSKkfud ds rqjar ck,¡ R cSBk gS vkSj R,J dk ifr gSA
group are lecturer, lawyer, doctor scientist . Among the males (54) T is sitting between U and V.
only R (the lawyer ) and V (the scientist) are sitting together. T,U vkSj V ds chp cSBk gSA
Each man is seated besides his wife. U, the wife of the lecturer Note: (1) Here females are denoted using a circle “”
is seated second to the right of the V. T is seated between U efgyk dks o`Ùk “” ls iznf’kZr fd;k x;k gSA
and V. P is the wife of the doctor. Q is not the doctor. S is a Males are denoted using a triangle “ Δ”/iq:"k dks f=Hkqt “Δ”
male. ls iznf'kZr fd;k x;k gSA
J,P,Q,R,S,T,U vkSj V pkj fookfgr tksM+s gS tks dsUnz dh vksj eq¡g djds “+” sign has been used to represent a married couple.
,d o`Ùk esa cSBs gSA iq:"kksa ds chp O;olk; bl izdkj gS& Fookfgr tksM+s dks iznf’kZr djus ds fy, Þ$Þ fpUg dk izz;ksx fd;k
ysDpjj] odhy] MkWDVj vkSj oSKkfudA iq:"kksa esa ls flQZ R¼odhy½ x;k gSA
vkSj V ¼oSKkfud½ ,d lkFk cSBs gSA izR;sd iq:"k viuh iRuh ds vxy (2) Write down statements as hints which are not used
esa cSBk gSA ysDpjj dh iRuh U,V ds nk,¡ dh nwljs LFkku ij cSBh gSA immediately so that we so not need to read the question
T dk LFkku U vkSj V ds chp esa gSA P MkWDVj dh iRuh gSA Q MkWDVj repeatedly.
ugha gSA S ,d iq:"k gSA iz'u esa fn, x, ftu dFkuksa dk rqjar iz;ksx gy esa ugha gksrk] mUgsa
(50) Who among the following is a doctor? fgaV ds :i esa fy[k ys rkfd iz’u dks ckj&ckj i<+uk ugha iM+As
fuEu esa ls MkWDVj dkSu gS\
(a) Q (b) S (c) T (d) J Direction(60-64) Study the following information carefully
(51) Who among the following is a lecturer? and answer the questiongiven below:
fuEu esa ls ysDpjj dkSu gS\ Eight person A,B,C,D,E,F,G and H are sitting around a circular
(a) Q (b) S (c) T (d) J table facing towards the centre. They like different colours
(52) Who among the following the wife of the lawyer? namely Red, Orange, Violet, Black, Brown, Pink, Blue and
fuEu esa ls odhy dh iRuh dkSu gS\ yellow but not necessarily in the same order.
(a) T (b) J (c) Q (d) S 8 O;fDr A,B,C,D,E,F,G vkSj H ,d xksykdkj est ds pkjksa vksj dsUnz
(53) Who is on the immediate left of the scientist? dh vksj eq¡g djds cSBs gSA mu lHkh dks vyx&vyx jax ilan gSA
oSKkfud ds rqjar ck,¡ dkSu cSBk gS\ vFkkZr~ yky] ukjaxh] cSaxuh] dkyk] Hkwjk] xqykch] uhyk vkSj ihykA
(a) Husband of T (b) Husband of J ysfdu t:jh ugha fd blh dze esa gksA
(c) Husband of P (d) Husband of U P@Q means P sits second to the left of Q.
(54) T is sitting between which of the following? P#Q means P and Q sitting opposite to each other.
T fuEu esa ls fdlds chp cSBk gS\ P$Q means P is an immediate neighbour of Q.
(a) J&P (b) S&J (c) Q&R (d) U&V P%Q means P sits third to left of Q
Sol: Here R and V are sitting together but it is not known whether P & Q means P sits thirf to left of Q
V sits to the left or right of R. Therefore we will read next P^Q means P is not an immediate neighbour of Q.
statement. Here U sit first, second to the right of V. So we will Note: 1 Represent the one who likes Red colour, 2 represent the
make V sit first and after it other persons will sit. one who likes organge colour and so on until 8 which
The following arrangement will be obtained from the given represent the one who likes yellow colour in the given order as
information: mentioned above.
iz'u esa fn;k gS fd R vkSj V ,d lkFk cSBs gS ysfdu ;g Kkr ugha gS 1 ml O;fDr dks n'kkZrk gS ftls yky jax ilan gS] 2 ml O;fDr dks
fd V,R ds nk,¡ cSBk gS ;k ck,¡A vr% ge vxys dFku dks ns[ksaxsA n’kkZrk gS ftls ukjaxh jax ilan gS vkSj ;gh dze tc rd pyrk jgsxk
blesa U,V ds nk,¡ dks nwljs LFkku ij cSBh gS vr% lcls igys V dks fd 8 ml O;fDr dks n’kkZrk gs ftls ihyk jax ilan tSlk fd Åij
o`Ùk esa dgh Hkh cSBk ldrs gS vkSj mlds ckn vU; O;fDr;ksa dks fn, x, dze esa mYysf[kr gSA
cSBk,axsA
nh xbZ lwpukvksa ls fuEu O;oLFkk izkIr gksxh& Similarly, 1@2means the one who likes Red colour sits second
to the left of the one who likes Orange colour and 1#2 the one

315
Download Free PDFs & e-Books from Neon Classes App

who likes Red colour and the one who likes orange colour are D,ml O;fDr ds ck,¡ dks rhljs LFkku ij cSBk gS ftls uhyk jax ilan
opposite to each other and so on. gSA
blh izdkj 1@2 dk vFkZ gS fd og O;fDr ftls yky jax ilan gS] (61) A: 8%H = The one who likes yellow sits third to the left of H. (
ml O;fDr ds ck,¡ nwljs LFkku ij cSBrk gSS ftls ukjaxh jax ilan gSA ✓)
1#2 dk vFkZ gS fd og O;fDr ftls yky jax ilan gS vkSj og O;fDrs A: 8%H =ftl O;fDr dks ihyk jax ilan gS og H ds ck,¡ rhljs
ukjaxh jax ilan gS] ,d&nwljs ds foijhr cSBs gS vkSj vkxs Hkh ;gh LFkku ij cSBk gSA ;g lgh gSA
dze tkjh jgsxkA B: 5%2 = The one who likes Brown colour and the one who
P@1 means P sits second to the left of the one who likes Red
likes Orange colour, are immediate neighbours. (✓)
colour, P&8 means P sits third to the right of the one who likes
B: 5%2 = og O;fDr ftls Hkwjk jax ilan gS vkSj og O;fDr ftls
yellow colour and so on.
P@1 dk vFkZ gS P ml O;fDr ds ck,¡ nwljs LFkku ij cSBk gS ftls ukjaxh jax ilan gS nksuksa iM+kslh gSA ;g lgh gSA
yky jax ilan gSA P&8 dk vFkZ gS P ml O;fDrs nk,¡ rhljs LFkku C: 6# = The one who like Pink colour and the one who likes
ij cSBk gS ftls ihyk jax ilan gS vkSj vkxs Hkh ;gh dze tkjh jgsxkA yellow colour, are sitting opposite to each other(✓)
Given Statement: C: 6# = og O;fDr ftls xqykch jax ilan gS vkSj og O;fDr ftls
7#7 G#4, 2#F, 3$H, ihyk jax ilan gS] ,d&nwljs ds foijhr cSBs gSA ;g lgh gSA
H&E$G, 3#2, A%6, C@G, D: 2$4 = The one who likes Orange colour and the one who
1@7, 8^c, B#E likes Black colour, are immediate neighbour. (×)
(60) What is the position of D with respect to the one who likes Og O;fDr ftls ukjaxh jax ilan gS vkSj og O;fDr ftls dkyk jax
Blue colour? ilan gS] nksuksa iM+kslh gSA ;g xyr gSA
D dk LFkku ml O;fDr ds lanHkZ esa dkSulk gS ftls uhyk jax ilan ∴ Option (D)
gS\ (62) When we will (I) in the blanks – D%7 i.e. D sits third to the left
(a) Third to the left of the who likes blue colour. (✓)
(b) Third to the right When we fill (II) in the blankd-D^A i.e. D is not an immediate
(c) Second to the left
neighbour of A. (✓)
(d) Immediate right
(61) Which of the following is not correct? When we fill (III) in the blankd – D$4 i.e. D is the immediate
fuEu esa ls dkSulk lgh ugha gS\ neighbours of the one who likes black colour. (×)
(a) *%H (b) 5$2 (c) 6#8 (d) 2$4 ∴Only (I) and (II) are correct.
(62) Which of the following correctly fits in the blank D ___ ? (63) Option (A) : F2 = 2 (Orange) is opposite to F.
fuEu esa ls dkSulk [kkyh LFkku ds mi;qDr gS Option (B) : F2 = 3 (Violet) is opposite to B.
D_____? Option (C) : B3 = 5 (Brown) is opposite to A.
(I) %7 (II) ^A (III) $4 Option (D) : A5 = 6 (Pink) is opposite to E.
(a) Only (I) Option (E) : E6 = 6 (red) is opposite to H.
(b) only (II) (64) H sits second to the right of A.
(C) Only (I) and (II) Concept-IV : Close Form : Rectangle/ Square arrangement:
(d) Only (II) and (III) Inner Facing ( Four, six, Eight & Ten Persons)
(63) Four of the five among the following are similar in such a way 65. Study the given information carefully and answer the question
to form a group, which one of the following doesn’t belongs to asked.
the group? nh xbZ tkudkjh dk /;kuiwoZd v/;;u djsa vkSj iwNs x, iz’u dk
uhpss fn, x, ik¡p fodYiksa esa ls pkj fodYi bl rjg leku gS fd os mÙkj nsAa
,d lewg dk fuekZ.k djrs gS] fuEu esa ls dkSulk fodYi lewg ls In an office, four colleagues Lewis, Alex, Philip and Siri are
laca/k ugha j[krk gS\ sitting around a square table, and each of them is sitting at
(a) F2 (b) F3 (c) A5 different corners of the table, facing the center of the table.
(d) E6 (e) H1 Lewis is sitting immediate to the right of Alex. Philip is sitting
(64) Who among the following sits second to the right of A? immediate to the left of Siri.
fuEu esa ls dkSu A ds nk,¡ nwljs LFkku ij cSBk gS\ ,d dk;kZy; esa] pkj lgdehZ yqbZl] ,ysDl] fQfyi vkSj lhjh ,d
(a) H (b) B (c) D (d) G oxkZdkj est ds pkjksa vksj cSBs gsa] vkSj muesa ls izR;sd est ds dsUnz
(e) None of these dh vksj eq[k djds est ds fofHkUu dksuksa ij cSBk gSA yqbZl] ,ysDl
Sol. ds nkbZa vksj Bhd cxy esa cSBk gSA fQfyi] lhjh ds ckbZa vksj Bhd
(Yellow) cxy esa cSBk gSA
Blue E (Red) Which of the following pair sits diagonally opposite to each
A G other?
fuEufyf[kr esa ls dkSu lk ;qXe] ,d nqljs ds fod.kZr% lkeus cSBk gS\
Violet D (Orange) (a) ,ysDl & lhjh (b) fQfyi & ,ysDl
F
(c) yqbZl & fQfyi (d) yqbZl & ,ysDl
Sol. (b)
C Brown
H Lewis, Alex, Philip and Siri are Sitting around Square table and
Black B each on different Corners facing center.
Pink
⇒ Lewis is immediate to the right of Alex.
(60) D sits third to the left person who likes blue colour.

316
Best App for Govt. Jobs : Neonclasses (Download Now)

Alex Lewis
⇒ Philip is immediate to the left of Siri. C
Siri Philip ⇒ L is the neighbour of A and B.
⇒ J and K an another positions.
A/B L B/A

Alex Lewis
Hence, Philip Sits diagonally opposite to Alex.
66. Four persons, Deepak, Gurkiran, Shahid and Vikrant, are
sitting around a square shaped table facing the centre. They J/K C K/J
are sitting in such a way that each person is at the middle of Hence, J and K are immediate neighbours of C.
each side of the table. Vikrant is to the immediate right of 68. 6 boys, J, K, L, M, N and P, are sitting around a rectangular table
Deepak. Shahid is facing Vikrant. Who is sitting to the facing towards the centre (not necessarily in the same order).
immediate left of Gurkiran? 4 boys are sitting in the corners, and 1 boy is sitting in the
pkj O;fä] nhid] xqjfdj.k] 'kkfgn vkSj foØkar] ,d oxkZdkj est ds middle of each shorter side of the table. N is second to the right
pkjks vksj dsæa dh vksj eq[k djds cSBs gSaA os bl çdkj cSBs gSa fd of K and second to the left of J. M is exactly opposite to N, and
M is not sitting in the corner.
çR;sd O;fä est dh çR;sd Hkqtk ds eè; esa gSA foØkar] nhid ds Bhd
6 yM+ds] J, K, L, M, N vkSj P, ,d vk;rkdkj est ds pkjksa vksj dsUnz
nk;sa cSBk gSA 'kkfgn dk lkeuk foØkar ls gSA xqjfdj.k ds Bhd ck;sa
dhvksj eq[k djds cSBs gSa ¼t:jh ugha fd blh Øe esa½A 4 yM+ds dksuksa
dkSu cSBk gS\
esa cSBs gSa] vkSj 1 yM+dk est dh izR;sd NksVh Hkqtk ds e/; esa cSBk gSA
(a) Either Shahid or Deepak
N, K ds nk;sa ls nwljs LFkku ij vkSj J ds ck;sa ls nwljs LFkku ij gSA
(b) Vikrant (c) Shahid
(d) Deepak M, N ds Bhd foijhr gS] vkSj M dksus esa ugha cSBk gSA
Sol. (b) Which of the following statements is definitely correct?
Deepak, Gurkiran, Shahid and Vikrant Sitting Each middle Side fuEufyf[kr esa ls dkSu lk dFku fuf’pr :i ls lgh gSa\
of square table. I. L and P are both next to N/ L vkSj P nksuksa N ds cxy esa gSa
⇒ Vikrant is immediate right of Deepak. II. K is diagonally opposite to L/ K, L ds frjNs foijhr gS
(a) Only I is correct/ dsoy I lgh gS
⇒ Shahid is facing Vikrant.
(b) Both I and II are correct/ I vkSj II nksuksa lgh gSa
Gurkiran
(c) Only II is correct/ dsoy II lgh gS
(d) Neither I nor II is correct/ u rks I vkSj u gh II lgh gS
Shahid Vikrant
Sol. (a)
J, K, L, M, N, P are at rectangular table
Deepak four boys are corner and 2 is in each of Shorter Side -
Hence, Vikrant is immediate left of Gurkiran. ⇒ M is not Sitting in the corner
67. 6 girls J, K, L, A, B and C are sitting around a square table facing ⇒ M is Opposite to N.
the centre (not necessarily in the same order). 4 girls are
sitting in the corners and 2 girls are sitting one each in the
middle of two opposite sides of the square. L and C are sitting
M N
opposite each other. L is the immediate neighbour of A and B.
C is not sitting on the corner.
6 yM+fd;k¡ J] K] L] A] B vkSj C ,d oxkZdkj est ds pkjks vksj dsæa
dh vksj eq[k djds cSBh gSa ¼t:jh ugha fd blh Øe esa gks½a A 4 ⇒ N is second to the right of K and Second to the left of J.
yM+fd;ka dksuksa esa cSBh gSa vkSj 2 yM+fd;ka oxZ ds nks foijhr i{kksa ds ⇒ L and P sits at remainning positions.
chp esa ,d&,d cSBh gSaA L vkSj C ,d nwljs ds foijhr cSBs gSaA L] A L/P
J
vkSj B dk fudVre iMkslh gS- C dksus ij ugha cSBk gS
Who are the immediate neighbours of C? M N
C ds fudVre iMkslh dkSu gSa\
(a) K and B (b) J and K K P/L
(c) K and A (d) J and B
Sol. (b) Hence, L and P are both next to N is correct.
6 girls J, K, L, A, B, and C are Sitting square table facing centre. K is diagonally opposite to L not confirmed
4 girls are at corner and 2 are at middle of two opposite sides. 69. Eight boys A, B, C, D, E, F, G and H are sitting around a square
table, facing towards the centre(but not necessarily in the
⇒ C is not sitting on the corner.
same order). Two boys are sitting on each side. G and C are
⇒ L and C are opposite to each other. sitting on the same side. E is sitting opposite to G and E is to
the immediate right of B. F and H are sitting on the adjacent
side of G and C. F and H are sitting on the same side. H is fourth

317
Download Free PDFs & e-Books from Neon Classes App

to the left of B. D is not the immediate neighbour of G. Which of E


the following pairs of boys represent the boys sitting on the
same side of the square table?
vkB yM+ds A, B, C, D, E, F, G vkSj H ,d oxkZdkj est ds pkjksa vksj
dsUnz dh vksj eq[k djds cSBs gSa ¼ysfdu t:jh ugha dh blh Øe esa gksa½ G
gj rjQ nks yM+ds cSBs gSaA G vkSj C ,d gh vksj cSBs gSaA E, G ds ⇒ F is not immediate neighbour of E.
foijhr cSBk gS vkSj E, B ds Bhd nk;sa cSBk gSA F vkSj H, G vkSj C ds ⇒ three people sit between F and H So 2 Possible cases –
cxy esa cSBs gSaA F vkSj H ,d gh rjQ cSBs gSaA H, B ds ck;sa ls pkSFkk Case 1: Case 2:
gSaA D, G dk fudVre iMkslh ugha gSA fuEufyf[kr esa ls yM+dksa dk E H
dkSu&lk tksM+k oxkZdkj est ds ,d gh vksj cSBs yM+dksa dk E
izfrfuf/kRo djrk gS\ H F
(a) BA (b) BD (c) AC (d) HD
Sol. (a) G F G
⇒ G and C are on the same side. ⇒ B Sits immediate left of H So case 2 gets eliminated.
⇒ E sits opposite to G and E is immediate right of B. H
E B
So two possible cases for G and C
Case1: Case2:
E B E

B G F
⇒ A Sits third to right of C.
E H B
G C C G
⇒ H is fourth to the left of B. So case 1 gets eliminated. C D
⇒ F and H are on same side.
⇒ F and H are Sitting adjacent side of G and C.
G F A
E
Hence, E Sits third to right of D.
Concept-V : Close Form : Rectangle/ Square arrangement:
F B
H I Inner & Outer Facing ( Four, six, Eight & Ten Persons)
71. Study the given information carefully and answer the question
G C that follows.
⇒ D is not the immediate neighbour of G. nh xbZ tkudkjh dk è;kuiwoZd vè;;u djsa vkSj uhps fn, x, ç'u
D E dk mÙkj nsaA
Colleagues Karna, Michelli, Prabhu, Vanaja, Mohan, Raghu,
F B Jockey and Neelima are sitting around a square table in such
H A way that four of them sit at corners and four of them at the
middle of the sides. The ones who sit at the corners face
G C outside (opposite to the centre) and the ones who sit at the
Hence, B and A Sitting on Same Side. middle of the sides face towards centre. Vanaja is seated third
to the right of Jockey. Vanaja faces towards the centre. Mohan
70. A, B, C, D, E, F, G and H are sitting around a square table facing is third to the left of Prabhu. Prabhu is not sitting at middle of
the cenre of the table. Four of them are sitting at each of the any of the sides. Only one person sits between Mohan and
corners, while the other four are sitting at the exact centre of Raghu. Raghu is not an immediate neighbour of Prabhu.
each of the side. G sits at one of the conrners of the table. E sits Neelima faces towards the centre. Karna is not an immediate
second to the left of G. F is not an immediate neighbour of E. neighbour of Raghu.
only three people sit between F and H. B sits to the immediate Who is an immediate neigbhour of both Mohan and Raghu?
left of H. A sits third to the right of C. Who sits third to the right lgdeÊ d.kZ] fe'ksyh] çHkq] outk] eksgu] j?kq] t‚dh vkSj uhfyek ,d
of D? oxkZdkj est ds pkjksa vksj bl çdkj cSBs gSa fd muesa ls pkj dksuksa ij
A, B, C, D, E, F, G vkSj H ,d oxkZdkj est ds pkjksa vksj est ds dsæ a vkSj pkj Hkqtkvksa ds chp esa cSBs gSaA dksuksa ij cSBus okyksa dk eq[k ckgj
dh vksj eq[k djds cSBs gSaA muesa ls pkj çR;sd dksus ij cSBs gSa] tcfd ¼dsaæ ds foijhr½ dh vksj gS vkSj tks Hkqtkvksa ds chp esa cSBs gSa mudk
vU; pkj çR;sd Hkqtk ds Bhd dsæa esa cSBs gSaA G est ds fdlh ,d dksus eq[k dsæa dh vksj gSA outk] t‚dh ds nk;sa ls rhljs LFkku ij cSBh gSA
ij cSBk gSA E, G ds ck;sa ls nwljs LFkku ij cSBk gSA F, E dk fudVre outk dk eq[k dsæa dh vksj gSA eksgu] çHkq ds ck;sa ls rhljs LFkku ij
iMkslh ugha gSA F vkSj H ds chp dsoy rhu yksx cSBs gSaA B, H ds Bhd gSA çHkq fdlh Hkh Hkqtk ds eè; esa ugha cSBk gSA eksgu vkSj j?kq ds chp
ck;sa cSBk gSA A, C ds nk;sa ls rhljs LFkku ij cSBk gSA D ds nk;sa ls dsoy ,d O;fä cSBk gSA j?kq] çHkq dk fudVre iMkslh ugha gSA
rhljs LFkku ij dkSu cSBk gSA uhfyek dk eq[k dsæa dh vksj gSA d.kZ] j?kq dk fudVre iMkslh ugha
(a) G (b) H (c) E (d) B gSA
Sol. (c) eksgu vkSj j?kq nksuksa dk fudVre iMkslh dkSu gS\
⇒ G sits at One of the corners. (a) Prabhu (b) Vanaja
⇒ E sits Second to the left of G. (c) Michelli (d) Jockey

318
Best App for Govt. Jobs : Neonclasses (Download Now)

Sol. (d) mUeq[k gSaA vU; pkj pkjksa Hkqtkvksa esa ls çR;sd ds eè; esa cSBs gSa vkSj
Square table corners faces outside and middle of the sides dsæa ds foijhr mUeq[k gSaA E dk eq[k dsæa dh vksj gS vkSj og G dk
faces inside. fudVre iM+kslh ugha gSA A dk eq[k dsæa dh vksj gS vkSj og G ds nk;sa
⇒ Mohan is third to the left of Prabhu. ls rhljs LFkku ij cSBk gSA D, B ds Bhd nk;sa cSBk gSA B dk eq[k dsæa
⇒ Prabhu is not Sitting at middle of any of the sides. dh vksj gS tcfd C dk eq[k ugha gSA A ds Bhd cxy esa gSA G vkSj H ds
chp dsoy ,d O;fä cSBk gSA fuEufyf[kr esa ls dkSu F ds Bhd nk;sa
cSBk gS\
Mohan (a) E (b) A (c) G (d) D
Sol. (a)
⇒ A is facing centre and third from the right of G.

⇒ Only One person sits between Mohan and Raghu.


⇒ Raghu is not an immediate neighbour of Prabhu. G
Raghu
A
Mohan ⇒ E is facing the centre and is not immediate neighbour of G.
E

G
⇒ Vanaja is third to the right of Jockey.
⇒ Vanaja faces centre – A
Case 1:
⇒ B is facing centre and D is sitting to the immediate right of B.
Raghu E D B

Mohan G

A
Vanaja
⇒ C is not an immediate neighbour of A.
Case 2: ⇒ Only one person Sitting between G and H.
Raghu ⇒ And the remainning person is F.
E D
B
Vanaja Mohan
G G

A C
⇒ Neelima faces towards the centre. H
⇒ Karna is not immediate neighbour of Raghu So case 2 gets Hence, E sits to the immediate right of F.
eliminated. 73. Eleven people are sitting across a rectangular table. Four
persons are sitting on each longest side of the table. Boss is
Raghu sitting alone at one side of the table. N and O are sitting beside
each other on one of the shortest side of table. H is sitting
Mohan opposite to E and between G and N. A is sitting opposite to C
Neelima
and between M and D. B is opposite to D and between C and G.
F is sitting opposite to G. Who among them is a Boss?
Vanaja X;kjg O;fä ,d vk;rkdkj est ds ikj cSBs gSaA est dh çR;sd lcls
Hence, Jocky is an immediate neighbour of Mohan and Raghu.
yach Hkqtk ij pkj O;fä cSBs gSaA c‚l Vscy ds ,d rjQ vdsys cSBs
72. Eight friends, A, B, C, D, E, F, G and H are sitting around a squre gSaA N vkSj O ,d nwljs ds cxy esa est dh lcls NksVh Hkqtk ij cSBs
table. Four of them are sitting at four corners of the table and gSaA H] E ds foijhr vkSj G vkSj N ds chp cSBk gSA A] C ds foijhr
are facing the centre. The other four are sitting at the middle of vkSj M vkSj D ds chp cSBk gSA B] D ds foijhr vkSj C vkSj G ds
each of the four sides and are facing opposite to the centre. E is chp cSBk gSA F] G ds foijhr cSBk gSA muesa ls dkSu c‚l gS\
facing the centre and is not an immediate neigbhour of G. A is (a) F (b) M (c) A (d) G
facing centre and is sitting at the third position from the right Sol. (b)
of G. D is sitting to the immediate right of B. B is facing the Boss is Sitting alone at one side of the table –
centre while C is not an immediate neigbhour of A. There is
only one person sitting between G and H. Who among the Boss
following sits to the immediate right of F?
vkB fe=] A, B, C, D, E, F, G vkSj H ,d oxkZdkj est ds pkjksa vksj cSBs
gSaA muesa ls pkj est ds pkjksa dksuksa ij cSBs gSa vkSj dsæa dh vksj

319
Download Free PDFs & e-Books from Neon Classes App

⇒ N and O are beside each other on one of the Shrotest Side of 75. On all the five corners of pentagon, five people Ram, Rohan,
table. Sunita, Seema and Chandan sit in a specific order facing
Two Possible Cases – towards the centre. Ram is to the immediate left of Rohan.
Case 1: Case 2: Sunita is not an immediate neighbour of Ram. Chandan is an
immediate neighbour of both Sunita and Seema.
iapHkqt ds lHkh ikap dksuksa ij] ikap O;fä jke] jksgu] lquhrk] lhek
N Boss N
Boss vkSj panu ,d fof'k"V Øe esa dsæa dh vksj mUeq[k gksdj cSBrs gSaA jke]
O O
jksgu ds Bhd ck;sa gSA lquhrk jke dh fudVre iMkslh ugha gS- panu
lquhrk vkSj lhek nksuksa dk fudVre iMkslh gS-
⇒ H is Sitting opposite to E and between G and N
What will be the position of sunita with respect to Ram?
Case 1: Case 2:
jke ds lUnHkZ esa lquhrk dk LFkku D;k gksxk\
GH E (a) Sunita occupies third place from Ram in the anticlockwise
N Boss N direction.
Boss
O O lquhrk okekorZ fn'kk esa jke ls rhljs LFkku ij gS
E GH (b) Sunita occupies third place from Ram in the clockwise
⇒ B is opposite to D and between C and G. direction.
⇒ F is Sitting opposite to G. lquhrk nf{k.kkorZ fn'kk esa jke ls rhljs LFkku ij gS
Case 1: Case 2: (c) Sunita occupies second place from Ram in the clockwise
C B GH DF E direction.
N Boss N
nf{k.kkorZ fn'kk esa lquhrk jke ls nwljs LFkku ij gS
Boss (d) Sunita occupies first place from Ram in the anti-clockwise
O O
direction.
DF E C BGH
lquhrk okekorZ fn'kk esa jke ls igys LFkku ij gS
⇒ A is Sitting opposite to C and between M and D.
Sol. (b)
Case 1: Case 2:
⇒ Ram is to the immediate left of Rohan.
C B GH AD F E ⇒ Sunita is not an immediate neighbour of Ram.
N Boss N 2 Possible cases –
Boss
O O Case 1:
AD F E C BGH Sunita
Hence, from both of the cases M is the Boss.
Concept-VI: Close Form : Polygon arrangement: Inner &
Outer Facing ( Five, Six & Eight Persons)
74. Akram, Nagesh, Girish, Vikram and Satpal are sitting at the five
corners of a pentagon shaped table; all are facing the centre of
Ram Rohan
the table. Girish is third to the left of Vikram. Satpal is to the
immediate left of Nagesh. Akram is second to the left of Satpal. Case 2:
Who is sitting second to the right of Nagesh?
vdje] ukxs'k] fxjh'k] foØe vkSj lriky ,d iapHkqt ds vkdkj dh Sunita
est ds ikap dksuksa ij cSBs gSa( lHkh dk eq[k est ds dsæa dh vksj gSA
fxjh'k] foØe ds ck;sa ls rhljs LFkku ij gSA lriky ukxs'k ds Bhd
ck;sa gSA vdje lriky ds ck,a ls nwljs LFkku ij gSA ukxs'k ds nk;sa
ls nwljs LFkku ij dkSu cSBk gS\ Ram Rohan
(a) vdje (b) foØe (c) lriky (d) fxjh’k ⇒ Chandan is an immediate neighbour of both Sunita and
Sol. (a) Seema. So Case 1 gets eliminated
⇒ Satpal is to the immediate left of Nagesh. Chandan
⇒ Akram is Second to the left of Satpal.
Seema Sunita
Satpal Akram

Ram Rohan
Nagesh Hence, Sunita occupies third place from Ram in the clockwise
⇒ Girish is third to the left of Vikram. direction.
Girish 76. Six people, K, L, M, N, O and P, are sitting on the ground in a
hexagonal shape but not necessarily in the same order. K is not
Satpal Akram
sitting next to L or M. N is not sitting next to M or O. L is the
neighbour of M. P is in the middle of M and N. What is the
correct order of seating positions of the people in clockwise
direction?
Nagesh Vikram
Hence, Akram is Second to the right of Nagesh.

320
Best App for Govt. Jobs : Neonclasses (Download Now)

Ng O;fä] K, L, M, N, O vkSj P, tehu ij "kV~dks.kh; vkdkj esa cSBs gSa P


ysfdu vko';d ugha blh Øe esa gksAa K, L ;k M ds cxy esa ugha cSBk
gSA N, M ;k O ds cxy esa ugha cSBk gSA L, M dk iM+kslh gSA P, M vkSj N
ds eè; esa gSA ?kM+h dh fn'kk esa yksxksa ds cSBus dh fLFkfr dk lgh Øe
D;k gS\
(a) L-M-P-N-K-O (b) L-N-O-K-M-P S
(c) L-O-K-P-M-N (d) L-M-N-P-K-O
U
Sol. (a)
⇒ P is in middle of M and N. ⇒ T is between P and S.
So two possible cases – ⇒ P is immediate left of R.
Case 1: ⇒ Q is remaining person.
Case 1: Case 2: P
R T

M N N M
Q S
P P
U
⇒ L is neighbour of M.
Hence, Q is opposite to T.
⇒ K is not next to L or M.
78. Eight friends, Arvind, Priya, Kunal, Kajal, Ashish, Mahak,
Case 1: Case 2:
Shubham and Riddhi are sitting around an octagonal table,
L K L K facing the centre. Kunal is between Mahak and Ashish. Kajal is
between Priya and Arvind. Shubham is to the immediate left of
Riddhi. Arvind is third to the right of Ashish. Who is sitting
between Riddhi and Priya?
M N N M
vkB fe=] vjfoan] fç;k] dq.kky] dkty] vk'kh"k] egd] 'kqHke vkSj
P P
fjf) ,d v"Vdks.kh; est ds pkjksa vksj dsaæ dh vksj eq[k djds cSBs
⇒ N is not Sitting next ot M or O. gSaA dq.kky egd vkSj vk'kh"k ds chp esa gSA dkty] fç;k vkSj vjfoan
Case 1: Case 2: ds chp esa gSA 'kqHke] fjf) ds Bhd ck;sa gSA vjfoan] vk'kh"k ds nk;sa ls
O O
L K L K rhljs LFkku ij gSA fjf) vkSj fç;k ds chp esa dkSu cSBk gS\
(a) Ashish (b) Arvind
(c) Kajal (d) Shubham
Sol. (d)
M N N M
Arvind is third to the right of Ashish.
P P
Hence, according to options (A) L-M-P-N-K-O Sequence right in Arvind
case 2.

77. Six persons P, Q, R, S, T and U were sitting around a hexagon


table facing the centre. U was sitting opposite to P, who was to
Ashish
the immediate left of R. S was sitting to the immediate right of
⇒ Kajal is between Priya and Arvind.
U, and T was exactly between P and S. What was the sitting
So 2 Possible cases here –
location of Q?
Case 1:
N% O;fDr P, Q, R, S, T vkSj U ,d "kVdks.kh; est ds pkjksa vksj dsna z dh
vksj eqag djds cSBs gSaA U, P ds lkeus cSBk gS] tks fd R ds ckbZa vksj
iM+ksl esa cSBk gSA S, U ds nkbZa vksj iM+ksl esa cSBk gS rFkk P rFkk S ds Arvind
Bhd chp esa T cSBk gSA Q dgka cSBk gS\
(a) Opposite to R/R ds lkeus Kajal
(b) Opposite to T/T ds lkeus
(c) Immediate left of P/P ds ckbZa vksj iM+ksl esa Ashish Priya
(d) Opposite to S/S ds lkeus Case 2:
Sol. (b) Priya Kajal
⇒ U is Opposite to P.
Arvind
⇒ S was Sitting immediate to the right of U.

Ashish
⇒ Kunal is between Mahak and Ashish.

321
Download Free PDFs & e-Books from Neon Classes App

⇒ Shubham is immediate left of Ridhi. 2. Situation / ifjfLFkfr %


Case 1: Although we have rating agencies like Crisil, ICRA, there is
Ridhi Shubham demand to have a separate rating agency for IT companies
to protect investors.
Mahak Arvind gkykafd gekjs ikl fØfly] vkÃlhvkj, tSlh jsÇVx ,tsfa l;ka gSa]
ysfdu fuos'kdksa dh lqj{kk ds fy, vkÃVh daifu;ksa ds fy, ,d vyx
jsÇVx ,tsalh dh ekax dh tk jgh gSA
Kunal Kajal
Conclusions/ fu"d"kZ %
Ashish Priya I. Assessment of financial worth of IT companies calls for
separate set of skills, insight and competencies.
Case 2:
Priya Kajal
vkÃVh daifu;ksa ds foÙkh; ewY; dk vkdyu dkS'ky] var–Zf"V vkSj
n{krkvksa ds vyx lsV dh ekax djrk gSA
II. Now the investors investing in IT companies will get
Shubham Arvind
protection of their investment.
vc vkÃVh daifu;ksa esa fuos'k djus okys fuos'kdksa dks muds fuos'k dh
Riddhi Mahak lqj{kk feysxh-
(a) if only conclusion I follows
Ashish Kunal ;fn dsoy fu"d"kZ I vuqlj.k djrk gS
So from case 2 Shubham is Sitting between Riddhi and priya. (b) if only conclusion II follows
Situation-Passage and conclusion ;fn dsoy fu"d"kZ II vuqlj.k djrk gS
Case 1 : Conlusion is based on States/facts (c) if either I and II follows
1. Situation: ;fn ;k rks I vkSj II vuqlj.k djrk gS
Since the past 15 years, 75% of the products in the world’s (d) if neither I nor II follows and
pottery market comes from Country K. However, the ;fn u rks I vkSj u gh II vuqlj.k djrk gS vkSj
employment in the pottery industry of Country K has been (e) if both I and II follow
consistently declining by 5-9% every year since the past 3 ;fn I vkSj II nksuksa vuqlj.k djrs gSa
years. Sol. (a)
fiNys 15 o"kksZa ls] nqfu;k ds feêh ds crZuksa ds cktkj esa 75% mRikn The need for separate rating agency for IT companies clearly
ns'k K ls vkrs gSaA gkykafd] ns'k K ds feêh ds crZu m|ksx esa fiNys 3 indicates that such assessment requires a separate set of skills.
o"kksaZ ls gj lky 5&9% dh yxkrkj fxjkoV vk jgh gSA So, I follows. However, the statement indicates only the need
Conclusions: or demand and neither the future course of action nor its after
I. Even after declining employment, Country K has enough effects can be judged. So, II does not follow.
potters to continue contributing equally to the Case 3 : Directly concluded
international market.
jkstxkj ?kVus ds ckn Hkh] ns'k K ds ikl varjjk"Vªh; cktkj esa leku
lhèks fu"d"kZ fudkyk x;k
:i ls ;ksxnku tkjh j[kus ds fy, i;kZIr dqEgkj gSaA 3. Situation/ifjfLFkfr %
II. The local demand of pottery in Country K has Wind is an inexhaustible source of energy and an aero
substantially decreased leading to less interest among generator can convert it into electricity. Though not much
potters. has been done in this field, the survey shows that there is
ns'k K esa feêh ds crZuksa dh LFkkuh; ekax esa dkQh deh vkbZ gS] ftlls vast potential for developing wind as alternative source of
dqEgkjksa ds chp #fp de gks xbZ gSA energy.
(a) Neither I nor II can be concluded
iou ÅtkZ dk ,d v{k; lzksr gS vkSj ,d ,;jks tujsVj bls fctyh
u rks I vkSj u gh II fu"d"kZ fudkyk tk ldrk gSA esa ifjoÆrr dj ldrk gSA gkykafd bl {ks= esa cgqr dqN ugÈ fd;k
(b) Only II can be concluded x;k gS] losZ{k.k ls irk pyrk gS fd ÅtkZ ds oSdfYid lzkrs ds :i esa
dsoy II fu"d"kZ fudkyk tk ldrk gSA iou ds fodkl dh O;kid laHkkouk gSA
(c) Only I can be concluded Conclusions / fu"d"kZ %
dsoy Ia fu"d"kZ fudkyk tk ldrk gSA I. Energy by wind is comparatively newly emerging field.
(d) Both I and II can be concluded gok }kjk ÅtkZ rqyukRed :i ls u;k mHkjrk gqvk {ks= gSA
I vkSj II nksuksa fu"d"kZ fudkys tk ldrs gSaA II. The energy crisis can be dealt by exploring more in the
Sol. (c) field of aero-generation.
Conclusion I is correct because in statement and conclusion ,;jks&tsujs'ku ds {ks= esa vkSj [kkst dj ÅtkZ ladV ls fuiVk tk
are related to each other. They both are saying about the ldrk gSA
declining of employment. (a) if only conclusion I follows
In conclusion II saying about demand of pottery, but statement ;fn dsoy fu"d"kZ I vuqlj.k djrk gS
is not talking about demand so conclusion II is wrong. (b) if only conclusion II follows
Only I can be concluded ;fn dsoy fu"d"kZ II vuqlj.k djrk gS
Correct answer is option (c) (c) if either I and II follows
Case 2 : Understanding the connection between statement and ;fn ;k rks I vkSj II vuqlj.k djrk gS
conclusion (d) if neither I nor II follows and
dFku vkSj fu"d"kZ ds chp lacaèk dks le>uk ;fn u rks I vkSj u gh II vuqlj.k djrk gS vkSj

322
Best App for Govt. Jobs : Neonclasses (Download Now)

(e) if both I and II follow ;fn I vkSj II nksuksa vuqlj.k djrs gSa
;fn I vkSj II nksuksa vuqlj.k djrs gSa Sol. (a)
Sol. (e) According to the statement, the political prisoners can be
The phrase not much has been done in this field indicates that divided into two groups those who were released and those
wind energy is a comparatively newly emerging field. So, I who were put in jail for political dharnas. However, no person
follows. The expression there is vast potential for developing involved in murder was released. This means that no political
wind as alternative source of energy proves II to be true. prisoner had committed murder. So, I follows. Clearly, II is not
Case 4 : Irrelevant conclusion directly related to the statement and does not follow.
Case 6 : Defenitely False
vçklafxd fu"d"kZ
4. Statement / dFku % fuf'pr :i ls >wBk
To cultivate interest in reading, the schools has made it 6. Passage
compulsory from June this year for each student to read In the present scenario, the idea of entrepreneurship is
two books per week and submit a weekly report on the getting wide spread popularity. This does not mean that
books. many people are getting attracted to start their own
i<+us esa #fp iSnk djus ds fy,] Ldwyksa us bl lky twu ls çR;sd business but it is due to the fact that the job market is
Nk= dks çfr lIrkg nks fdrkcsa i<+uk vkSj fdrkcksa ij ,d lkIrkfgd getting narrow. The number of unemployed engineering
fjiksVZ tek djuk vfuok;Z dj fn;k gSA graduates is getting multiplied every year. Now is the time
Conclusions / fu"d"kZ % to create a startup and to do something for you. But this
I. Interest in reading can be created by force. requires a clear visionary and a lot of patience followed by
i<+us esa #fp tcjnLrh iSnk dh tk ldrh gSA hard work. Prime minister’s Make in India and Made in
II. Some students will eventually develop interest in reading. India campaign are also adding fuel to the whole process.
dqN Nk= varr% i<+us esa #fp fodflr djsaxsA If such initiatives will get right direction and acclaim, then
(a) if only conclusion I follows soon India can eradicate the black patch of unemployment
;fn dsoy fu"d"kZ I vuqlj.k djrk gS from its head.
(b) if only conclusion II follows
orZeku ifj–'; esa] m|ferk ds fopkj dks O;kid çlkj yksdfç;rk
;fn dsoy fu"d"kZ II vuqlj.k djrk gS fey jgh gSA bldk eryc ;g ugha gS fd cgqr ls yksx viuk [kqn
(c) if either I and II follows
dk O;olk; 'kq: djus ds fy, vkdf"kZr gks jgs gSa] ysfdu ;g bl
;fn ;k rks I vkSj II vuqlj.k djrk gS rF; ds dkj.k gS fd ukSdjh cktkj ladh.kZ gks jgk gSA gj lky
(d) if neither I nor II follows and csjkstxkj bathfu;fjax Lukrdksa dh la[;k dbZ xquk c<+ jgh gSA vc
;fn u rks I vkSj u gh II vuqlj.k djrk gS vkSj ,d LVkVZ vi cukus vkSj vkids fy, dqN djus dk le; gSA ysfdu
(e) if both I and II follow blds fy, ,d Li"V nwjn'khZ vkSj dM+h esgur ds ckn cgqr /kS;Z dh
;fn I vkSj II nksuksa vuqlj.k djrs gSa vko';drk gksrh gSA ç/kku ea=h ds esd bu bafM;k vkSj esM bu bafM;k
Sol. (b) vfHk;ku Hkh iwjh çfØ;k ds fy, bZa/ku dk leFkZu dj jgs gSaA vxj bl
Clearly, the new scheme intends to develop interest in reading rjg dh igy dks lgh fn'kk feysxh vkSj ç'kalk feysxh] rks tYn gh
by incorporating the habit in their routine. So, only II follows Hkkjr vius flj ls csjkstxkjh ds dkys iSp dks feVk ldrk gSA
while I does not. Conclusion/ fu"d"kZ −
Case 5 : Concluding by analyzing the statement The present situation shows the number of people getting
dFku dk fo'ys"k.k djds fu"d"kZ fudkyuk jobs is more than that of previous years.
5. Statement / dFku % orZeku fLFkfr ls irk pyrk gS fd ukSdjh ikus okys yksxksa dh la[;k
All those political prisoners were released on bail who fiNys o"kksaZ dh rqyuk esa vf/kd gSA
had gone to jail for reasons other than political dharnas. We are required to mark the answer in the following ways
Bail was not granted to persons involved in murders. –
mu lHkh jktuhfrd cafn;ksa dks tekur ij fjgk dj fn;k x;k tks gesa fuEufyf[kr rjhdksa ls mÙkj dks fpfUgr djuk gksxk &
jktuhfrd èkjuksa ds vykok vU; dkj.kksa ls tsy x, FksA gR;k esa I. Mark (a) if the conclusion holds the truth definitely.
'kkfey yksxksa dks tekur ugÈ feyhA ;fn fu"d"kZ fuf'pr :i ls lR; dks /kkj.k djrk gSA
Conclusions / fu"d"kZ % II. Mark (b) if the conclusion holds the truth probably but not
I. No political prisoner had committed murder. definitely.
fdlh jktuhfrd dSnh us gR;k ugÈ dh FkhA ;fn fu"d"kZ lR; dks j[krk gS ysfdu fuf'pr :i ls ughaA
II. Some politicians were not arrested. III. Mark (c) If conclusion cannot be judged for true or false
dqN jktusrkvksa dks fxj¶rkj ugÈ fd;k x;k FkkA due to insufficiency of adequate data.
(a) if only conclusion I follows ;fn fu"d"kZ i;kZIr MsVk dh vi;kZIrrk ds dkj.k lgh ;k xyr ds
;fn dsoy fu"d"kZ I vuqlj.k djrk gS fy, ugha vkadk tk ldrk gSA
(b) if only conclusion II follows IV. Mark (d) if the conclusion holds the falsehood probably
;fn dsoy fu"d"kZ II vuqlj.k djrk g but not definitely.
(c) if either I and II follows ;fn fu"d"kZ >wB dks idM+rk gS] ysfdu fuf'pr :i ls ughaA
;fn ;k rks I vkSj II vuqlj.k djrk gS V. Mark (e) if the conclusion holds the falsehood definitely.
(d) if neither I nor II follows and ;fn fu"d"kZ fuf'pr :i ls >wB dks /kkj.k djrk gSA
;fn u rks I vkSj u gh II vuqlj.k djrk gS vkSj Sol. (e)
(e) if both I and II follow

323
Download Free PDFs & e-Books from Neon Classes App

The passage is clearly talking about the problems of true or false. This is because it needs some extra statement to
unemployment. This clearly indicates that the number of clarify.
jobless people has increased than before. So, we can conclude Case 8 : Defenitely True
that the given statement is definitely false.
Case 7 : Insufficient data fuf'pr :i ls lR;
vi;kZIr MsVk 8. Passage
In the present scenario, the idea of entrepreneurship is
7. Passage getting wide spread popularity. This does not mean that
In the present scenario, the idea of entrepreneurship is many people are getting attracted to start their own
getting wide spread popularity. This does not mean that business but it is due to the fact that the job market is
many people are getting attracted to start their own getting narrow. The number of unemployed engineering
business but it is due to the fact that the job market is graduates is getting multiplied every year. Now is the time
getting narrow. The number of unemployed engineering to create a start up and to do something for you. But this
graduates is getting multiplied every year. Now is the time requires a clear visionary and a lot of patience followed by
to create a startup and to do something for you. But this hard work. Prime minister’s Make in India and Made in
requires a clear visionary and a lot of patience followed by India campaign are also aiding fuel to the whole process. If
hard work. Prime minister’s Make in India and Made in such initiatives will get right direction and acclaim, then
India campaign are also aiding fuel to the whole process. If soon India can eradicate the black patch of unemployment
such initiatives will get right direction and acclaim, then from its head.
soon India can eradicate the black patch of unemployment orZeku ifj–'; esa] m|ferk ds fopkj dks O;kid çlkj yksdfç;rk
from its head. fey jgh gSA bldk eryc ;g ugha gS fd cgqr ls yksx viuk [kqn
orZeku ifj–'; esa] m|ferk ds fopkj dks O;kid çlkj yksdfç;rk dk O;olk; 'kq: djus ds fy, vkdf"kZr gks jgs gSa] ysfdu ;g bl
fey jgh gSA bldk eryc ;g ugha gS fd cgqr ls yksx viuk [kqn rF; ds dkj.k gS fd ukSdjh cktkj ladh.kZ gks jgk gSA gj lky
dk O;olk; 'kq: djus ds fy, vkdf"kZr gks jgs gSa] ysfdu ;g bl csjkstxkj bathfu;fjax Lukrdksa dh la[;k dbZ xquk c<+ jgh gSA vc
rF; ds dkj.k gS fd ukSdjh cktkj ladh.kZ gks jgk gSA gj lky ,d LVkVZ vi cukus vkSj vkids fy, dqN djus dk le; gSA ysfdu
csjkstxkj bathfu;fjax Lukrdksa dh la[;k dbZ xquk c<+ jgh gSA vc blds fy, ,d Li"V nwjn'khZ vkSj dM+h esgur ds ckn cgqr /kS;Z dh
,d LVkVZ vi cukus vkSj vkids fy, dqN djus dk le; gSA ysfdu vko';drk gksrh gSA ç/kku ea=h ds esd bu bafM;k vkSj esM bu bafM;k
blds fy, ,d Li"V nwjn'khZ vkSj dM+h esgur ds ckn cgqr /kS;Z dh vfHk;ku Hkh iwjh çfØ;k ds fy, bZa/ku dk leFkZu dj jgs gSaA vxj bl
vko';drk gksrh gSA ç/kku ea=h ds esd bu bafM;k vkSj esM bu bafM;k rjg dh igy dks lgh fn'kk feysxh vkSj ç'kalk feysxh] rks tYn gh
vfHk;ku Hkh iwjh çfØ;k ds fy, bZa/ku dk leFkZu dj jgs gSaA vxj bl Hkkjr vius flj ls csjkstxkjh ds dkys iSp dks feVk ldrk gSA
rjg dh igy dks lgh fn'kk feysxh vkSj ç'kalk feysxh] rks tYn gh Conclusion/ fu"d"kZ −
Hkkjr vius flj ls csjkstxkjh ds dkys iSp dks feVk ldrk gSA Entrepreneurship can be seen as an ultimate solution of
Conclusion/ fu"d"kZ − this problem.
Engineering graduates are only the worst sufferers of this m|ferk dks bl leL;k ds vafre lek/kku ds :i esa ns[kk tk ldrk
situation. gSA
bathfu;fjax Lukrd dsoy bl fLFkfr ds lcls [kjkc ihfM+r gSaA A candidate is required to mark the answer in the
A candidate is required to mark the answer in the following ways –
following ways – ,d mEehnokj dks fuEufyf[kr rjhdksa ls mÙkj dks fpfàr djuk
,d mEehnokj dks fuEufyf[kr rjhdksa ls mÙkj dks fpfàr djuk vko';d gS &
vko';d gS & I. Mark (a) if the conclusion holds the truth definitely.
I. Mark (a) if the conclusion holds the truth definitely. ;fn fu"d"kZ fuf'pr :i ls lR; dks /kkj.k djrk gSA
;fn fu"d"kZ fuf'pr :i ls lR; dks /kkj.k djrk gSA II. Mark (b) if the conclusion holds the truth probably but not
II. Mark (b) if the conclusion holds the truth probably but not definitely.
definitely. ;fn fu"d"kZ lR; dks j[krk gS ysfdu fuf'pr :i ls ughaA
;fn fu"d"kZ lR; dks j[krk gS ysfdu fuf'pr :i ls ughaA III. Mark (c) If conclusion cannot be judged for true or false
III. Mark (c) If conclusion cannot be judged for true or false due to insufficiency of adequate data.
due to insufficiency of adequate data. ;fn fu"d"kZ i;kZIr MsVk dh vi;kZIrrk ds dkj.k lgh ;k xyr ds
;fn fu"d"kZ i;kZIr MsVk dh vi;kZIrrk ds dkj.k lgh ;k xyr ds fy, ugha vkadk tk ldrk gSA
fy, ugha vkadk tk ldrk gSA IV. Mark (d) if the conclusion holds the falsehood probably
IV. Mark (d) if the conclusion holds the falsehood probably but not definitely.
but not definitely. ;fn fu"d"kZ >wB dks idM+rk gS] ysfdu fuf'pr :i ls ughaA
;fn fu"d"kZ >wB dks idM+rk gS] ysfdu fuf'pr :i ls ughaA V. Mark (e) if the conclusion holds the falsehood definitely.
V. Mark (e) if the conclusion holds the falsehood definitely. ;fn fu"d"kZ fuf'pr :i ls >wB dks /kkj.k djrk gSA
;fn fu"d"kZ fuf'pr :i ls >wB dks /kkj.k djrk gSA Sol. (a)
Sol. (c) Yes; because as the job market is getting narrow and the
It is given in the statement that the number of unemployed number of unemployed people is increasing, entrepreneurship
engineers is getting multiplied. But the passage has not told can solve both these problems by creating enough job market.
anything specific that whether only the engineers are the People can generate job and other people can get jobs easily.
worst hit of the situation or people with some other profession Case 9 : Not definitely true
also facing heat. So, this indicates that the statement may be

324
Best App for Govt. Jobs : Neonclasses (Download Now)

10. Passage
fuf'pr :i ls lR; ugÈ gS Asia has become the growth center of the world economy
9. Passage in recent years . Within the region, India and South Korea
In the present scenario, the idea of entrepreneurship is are the third and fourth largest economies after China and
getting wide spread popularity. This does not mean that Japan. Though the Asian growth stories mainly revolve
many people are getting attracted to start their own around India and China. South Korea has remained a key
business but it is due to the fact that the job market is player for these countries as one of their major trading
getting narrow. The number of unemployed engineering and investment partners. South Korea adopted outward
graduates are getting multiplied every year. Now is the oriented economic policies with the beginning of its first
time to create a start up and to do something for you. But five year economic development plan in 1962. which
this requires a clear visionary and a lot of patience resulted in high growth and the integration of the Korean
followed by hard work. Prime minister’s Make in India economy with the rest of the world. Subsequently, high
and Made in India campaign are also aiding fuel to the and consistent economic growth made South Korea one of
whole process. If such initiatives will get right direction the high-income economies in Asia. Korea is still growing
and acclaim, then soon India can eradicate the black patch at a faster rate compared to other developed economies.
of unemployment from its head. India on the other hand, adopted an import substitution
orZeku ifj–'; esa] m|ferk ds fopkj dks O;kid çlkj yksdfç;rk policy since its independence until the early 90s. Since,
fey jgh gSA bldk eryc ;g ugha gS fd cgqr ls yksx viuk [kqn then India has introduced wide-ranging economic policy
dk O;olk; 'kq: djus ds fy, vkdf"kZr gks jgs gSa] ysfdu ;g bl reforms and is moving toward market- driven economy.
rF; ds dkj.k gS fd ukSdjh cktkj ladh.kZ gks jgk gSA gj lky This has resulted in consistent high economic growth over
csjkstxkj bathfu;fjax Lukrdksa dh la[;k dbZ xquk c<+ jgh gSA vc the last one- and –a half decade.
,d LVkVZ vi cukus vkSj vkids fy, dqN djus dk le; gSA ysfdu ,f'k;k gky ds o"kksZa esa vFkZO;oLFkk dk fodkl dsæa cu x;k gSA bl
blds fy, ,d Li"V nwjn'khZ vkSj dM+h esgur ds ckn cgqr /kS;Z dh {ks= ds Hkhrj] Hkkjr vkSj nf{k.k dksfj;k] phu vkSj tkiku ds ckn
vko';drk gksrh gSA ç/kku ea=h ds esd bu bafM;k vkSj esM bu bafM;k rhljh vkSj pkSFkh lcls cM+h vFkZO;oLFkk gSaA gkykafd ,f'k;kbZ fodkl
vfHk;ku Hkh iwjh çfØ;k ds fy, bZa/ku dk leFkZu dj jgs gSaA vxj bl dh dgkfu;ka eq[; :i ls Hkkjr vkSj phu ds pkjksa vksj ?kwerh gSaA
rjg dh igy dks lgh fn'kk feysxh vkSj ç'kalk feysxh] rks tYn gh nf{k.k dksfj;k vius çeq[k O;kikfjd vkSj fuos'k Hkkxhnkjksa esa ls ,d ds
Hkkjr vius flj ls csjkstxkjh ds dkys iSp dks feVk ldrk gSA :i esa bu ns'kksa ds fy, ,d çeq[k f[kykM+h cuk gqvk gSA nf{k.k
Conclusion/ fu"d"kZ − dksfj;k us 1962 esa viuh igyh iapo"kÊ; vkfFkZd fodkl ;kstuk dh
Make in India campaign will definitely influence all people 'kq#vkr ds lkFk ckgj dh vksj mUeq[k vkfFkZd uhfr;ksa dks viuk;kA
to start entrepreneurship. blds ifj.kkeLo:i mPp fodkl gqvk vkSj 'ks"k nqfu;k ds lkFk
m|ferk 'kq: djus ds fy, esd bu bafM;k vfHk;ku fuf'pr :i ls dksfj;kbZ vFkZO;oLFkk dk ,dhdj.k gqvkA blds ckn] mPp vkSj
lHkh yksxksa dks çHkkfor djsxkA yxkrkj vkfFkZd fodkl us nf{k.k dksfj;k dks ,f'k;k esa mPp vk;
A candidate is required to mark the answer in the okyh vFkZO;oLFkkvksa esa ls ,d cuk fn;kA vU; fodflr vFkZO;oLFkkvksa
following ways – dh rqyuk esa dksfj;k vHkh Hkh rst nj ls c<+ jgk gSA nwljh vksj] Hkkjr
,d mEehnokj dks fuEufyf[kr rjhdksa ls mÙkj dks fpfàr djuk us 90 ds n'kd dh 'kq#vkr rd viuh Lora=rk ds ckn ls vk;kr
vko';d gS & çfrLFkkiu uhfr dks viuk;kA rc ls] Hkkjr us O;kid vkfFkZd uhfr
I. Mark (a) if the conclusion holds the truth definitely. lqèkkjksa dh 'kq#vkr dh gS vkSj cktkj lapkfyr vFkZO;oLFkk dh vksj c<+
;fn fu"d"kZ fuf'pr :i ls lR; dks /kkj.k djrk gSA jgk gSA blls fiNys ,d& Ms<+ n'kd esa yxkrkj mPp vkfFkZd fodkl
II. Mark (b) if the conclusion holds the truth probably but not gqvk gSA
definitely. Conclusion/ fu"d"kZ −
;fn fu"d"kZ lR; dks j[krk gS ysfdu fuf'pr :i ls ughaA Only Korean economy is considered as robust by the
III. Mark (c) If conclusion cannot be judged for true or false international community.
due to insufficiency of adequate data. dsoy dksfj;kbZ vFkZO;oLFkk dks varjkZ"Vªh; leqnk; }kjk etcwr ekuk
;fn fu"d"kZ i;kZIr MsVk dh vi;kZIrrk ds dkj.k lgh ;k xyr ds tkrk gSA
fy, ugha vkadk tk ldrk gSA I. Mark (a) if the conclusion holds the truth definitely.
IV. Mark (d) if the conclusion holds the falsehood probably ;fn fu"d"kZ fuf'pr :i ls lR; dks /kkj.k djrk gSA
but not definitely. II. Mark (b) if the conclusion holds the truth probably but not
;fn fu"d"kZ >wB dks idM+rk gS] ysfdu fuf'pr :i ls ughaA definitely.
V. Mark (e) if the conclusion holds the falsehood definitely. ;fn fu"d"kZ lR; dks j[krk gS ysfdu fuf'pr :i ls ughaA
;fn fu"d"kZ fuf'pr :i ls >wB dks /kkj.k djrk gSA III. Mark (c) If conclusion cannot be judged for true or false
Sol. (b) due to insufficiency of adequate data.
Make in India is a campaign that aims at producing more ;fn fu"d"kZ i;kZIr MsVk dh vi;kZIrrk ds dkj.k lgh ;k xyr ds
entrepreneurs according to the passage but it’s success solely fy, ugha vkadk tk ldrk gSA
depends upon the way government will promote and market IV. Mark (d) if the conclusion holds the falsehood probably
this. If the exact massage can be successfully derived to each but not definitely.
corner of the country then it may help. So, it can be probably ;fn fu"d"kZ >wB dks idM+rk gS] ysfdu fuf'pr :i ls ughaA
true but not definitely true. V. Mark (e) if the conclusion holds the falsehood definitely.
Case 10 : Probably false ;fn fu"d"kZ fuf'pr :i ls >wB dks /kkj.k djrk gSA
'kk;n xyr Sol. (d)

325
Download Free PDFs & e-Books from Neon Classes App

This inference is probably false because economies of China, ;fn fdlh rdZ esa dksbZ ckr ekuh tk, rks og rdZ iw.kZr;k% xyr gks
India and Japan are also booming. tk;sxkA
;g vuqeku 'kk;n xyr gS D;ksfa d phu] Hkkjr vkSj tkiku dh Statement/dFku :- Will BJP government come to Bihar?
vFkZO;oLFkk,a Hkh QyQwy jgh gSaA D;k fcgkj esa fctsih dh ljdkj vk;sxhA
STATEMENT & ARGUMENTS Argument/rdZ :-
(i) Yes, Modiji gave a very good speech.
Definition : -
gk¡ eksnhth us cgqr vPNk Hkk"k.k fn;k Fkk
The argument given in favor of the occurrence or absence of
(ii) Yes, because the BJP has done well in the last five years
an event is called argument
gk¡] D;ksfa d fiNys 5 lkyksa esa B.J.P. us Bhd&Bkd dke fd;k gSA
ifjHkk"kk %&
(IV) Questioner/iz’uokpd :- If an argument is said to be
fdlh ?kVuk ds gksus ;k uk gksus ds i{k esa tks nfyy nh tkrh gS mls
questioner, then that argument is completely wrong.
rdZ dgrs gSA
;fn fdlh rdZ esa iz’uokpd dh ckr dgh tk;s rks og rdZ iw.kZr;k%
In these questions it cannot be seen whether the party is right
xyr rdZ gks tk;sxkA
or the opposition is right. Rather, the arguments on both sides
Statement/dFku :- What should India to be declared a Hindu
are examined to be solid or not.
nation ?
bu iz’uksa esa ;g ugh ns[kk tkrk fd i{k lgh ;k foi{k lgh gSA cfYd
D;k Hkkjr dks fgUnq jk"Vª ns’k ?kksf"kr dj nsuk pkfg,A
ftl i{k dh nfyy etcwr gksxh og rdZ lR; ekuk tk;sxkA
Argument/rdZ :-
(1) Weak argument/detksj rdZ →
(i) No, would this be a prudent measure?
(I) Follow/vuqlj.k djuk
ugha] D;k ;g foosdiw.kZ mik; gksxk\
(II) Only Solution/,dek= jkLrk
(ii) No, what will happen to our secularism policy?
(III) Imagination/vuqeku ;k dYiuk djuk
ugha] gekjh /keZfujis{krk fd uhfr dk D;k gksxk\
(IV) Question /iz’uokpd
(2) Strong Argument/etcwr rdZ :-
(I) Follow/vuqlj.k djuk%& If an argument is said to follow the
(I) Prove oneself/Lo;a dks fl) djuk
other so that argument is a weak argument.
(II) The Argument should be clear./rdZ Li"V gksuk pkfg,A
;fn fdlh rdZ eSa nwljs ds vuqlj.k dh ckr dgh tk;s rks og rdZ
(III) Argument should show experience/rdZ esa vuqHko fn[kuk
detksj rdZ gksxkA
pkfg,A
Statement/dFku :- Should the prime minister wear expensive
(IV) Argument must contain logic/rdZ esa ykWftd gksuk pkfg,A
coats?
(V) The argument should be related to the subject/rdZ fo"k; ls
D;k iz/kkuea=h dks egaxs dksV iguus pkfg,A
lEcfU/kr gksuk pkfg,A
Argument/rdZ :-
(I) Prove oneself/Lo;a dks fl) djuk :- If any argument proves
(i) Yes, because Nehruji also does this.
itself to be perfect, then that argument will be considered
gkW] D;ksfa d usg: th Hkh ,slk fd;k djrs FksA
correct.
(ii) Yes, the prime minister should do it, because the prime
;fn dksbZ rdZ vius & vki dks iw.kZr% fl) djrk gks rks og rdZ
minister of france would also do it.
gk¡ iz/kkuea=h dks ,slk djuk pkfg,] D;ksfa d Ýkal ds iz/kkuea=h Hkh ,slk lgh ekuk tk;sxk
Statement/dFku : should education be made compulsory for
fd;k djrs FksA
children up to 14 years of age?
Statement/dFku% → What should India liberalize its economy?
D;k 14 o"kZ rd ds cPpksa ds fy, f’k{kk vfuok;Z dj nsuh pkfg,A
D;k Hkkjr dks viuh vFkZO;oLFkk dk mnkjhdj.k dj nsuk pkfg,A
Argument/rdZ :
Argument/rdZ :-
(i) Yes, because this will improve the standard of living and
(i) Yes, because it is also done in western countries.
educated person will be aware of their health hygiene.
gk¡] D;ksfa d if’peh ns’kksa esa Hkh ,slk fd;k tkrk gSA
gk¡] D;ksfa d blls thou Lrj esa lq/kkj gksxk rFkk f’kf{kr O;fDr vius
(ii) Yes, because it is also done in china.
LokLF;] LoPNrk ds izfr tkx:d gksxkA
gkW phu esa Hkh ,slk fd;k tkrk gSA
(ii) Education depends on the valuable things for the
(II) Only solution/,dek= jkLrk :- If it is said in an argument that
development of the nation and the education of the children.
this will be the only way, so that argument would be wrong
,d jk"Vª ds fodkl ds fy, f’k{kk cgweYw ; pht gS rFkk mlds fodkl
perfection.
dh uho cPpksa dh f’k{kk ij fuHkZj djrh gSA
;fn fdlh rdZ esa ;g dgk tk;s dh ;gh ,dek= jkLrk gksxk rks og
Statement/dFku : - should final exams be made mandatory in
rdZ vr% xYkr gks tk;sxkA
India?
Statement/dFku: - Should school education in India be free.
D;k Hkkjr esa vafre ifj{kkvksa dks vfuok;Z dj nsuk pkfg,A
D;k Hkkjr esa Ldwyh f’k{kk dks fu% 'kqYd dj nsuk pkfg,A
Argument/rdZ :-
Argument/rdZ :-
(i) Yes, it is a fruitless process.
(i) Yes, this is the only way to improve literacy rate.
gk¡] ;g ,d fu"Qy izfØ;k gSA
gk¡ lk{kjrk nj esa lq/kkj ykus dk ;gh ,d ek= jkLrk gSA
(ii) we do not this then A person’s mental level cannot be
(ii) No, this will increase the burden on the exchequer.
marked.
ugha] blls jktdks"k ij cks> c<+ tk;sxk
ge ,slk ugh djrs gS rks ,d O;fDr ds ekufld Lrj dk vkadyu
(III) Imagination Assumption/ vuqeku ;k dYiuk djuk :- If
ugha fd;k tk ldrk gSA
anything is to be believed in an argument, so that argument
(II) The Argument should be clear/rdZ Li"V gksuk pkfg, : - If any
would be wrong perfection.
argument related to the subject is correctly or with details,
then that argument will be treated as correct.

326
Best App for Govt. Jobs : Neonclasses (Download Now)

;fn fdlh rdZ esa fo"k; ls lEcfU/kr fdlh ckr dks lgh rjhds ls ;k (c) if either argument I or argument II is strong/;fn rdZ I ;k
fooj.k ds lkFk j[kk x;k gks rks rdZ dks lgh rdZ ekuk tk;sxkA rdZ II etcwr gSA
Statement/dFku :- should banks be interviewed for selection ? (d) if neither argument I nor argument II is strong/;fn u rks
D;k cSadksa esa p;u gsrq lk{kkRdkj gksuk pkfg,\ rdZ I vkSj u gh rdZ II etcwr gSA
Argument/rdZ : - (e) if both arguments I and II are strong./;fn nksuksa rdZ I vkSj II
(i) Yes, Because doing so runs the recommendation. etcwr gSAA
gk¡] D;ksfa d ,slk djus ls flQkfj’k pyrh gSA 1. Statement/dFku
(ii) Yes, from this, the mental level of that person can be Should there be only a uniform rate of income tax irrespective
ascertained. of the level of income?
gk¡] blls ml O;fDr ds ekufld Lrj dk irk yxk;k tk ldrk gSA D;k vk; ds Lrj ds ckotwn vk;dj dh ,d leku nj gksuh pkfg,\
(III) Argument should show experience rdZ esa vuqHko fn[kuk Arguments/rdZ
pkfg, :- If the argument given does not match the experience of I. Yes, this will substantially reduce the work of the officials of
daily life then it is a weak argument. the income tax department.
;fn fn;k x;k rdZ nsfud thou ds vuqHko ls esy ugha [kkrk gS rks gka] blls vk;dj foHkkx ds vfèkdkfj;ksa ds dke esa dkQh deh
og detksj rdZ gSA vk,xhA
Statement/dFku :- Should BODMAS rule be followed to solve II. No, this will reduce government tax collection to a large
the simplification questions? extent.
D;k] ljyhdj.k ds Questions dks gy djus gsrq BODMAS ds fu;eksa ugha] blls ljdkjh dj laxzg dkQh gn rd de gks tk,xkA
dk ikyu djuk pkfg,A Sol. (b)
Argument/rdZ : - Argument I is weak because reduction of work load of IT
(i) No, Because it leads to wrong Answer officials is not too desirable a motive. Argument II is strong as
ugha D;ksfa d ;g xyr mÙkj dh vksj ys tkrk gSA reduced tax collection will have a bad impact on state
(ii) Most of the mathmatics work is done by the BODMAS as it activities.
leads to the correct answer. rdZ I detksj gS D;ksfa d vkbZVh vfèkdkfj;ksa dk dke dk cks> de
xf.kr ds vf/kdÙkj dke BODMAS ds fu;e ls fd, tkrs gS D;ksfa d djuk Hkh ,d edln ugha gSA rdZ II etcwr gSA D;ksfa d dj laxzg
;g lgh mÙkj dh vksj ys tkrk gSA de gksus ls jkT; dh xfrfofèk;ksa ij cqjk vlj iM+sxkA
(IV) Logical : - Here the argument is logical, not philosophical. 2. Statement/dFku
rdZ rkfdZd gksrk gSA nk’kZfud ugha gksrk gSA Keeping in consideration the longevity of life in India, should
Statement/dFku :- should the exam be finished? the age-limit for retirement in the government jobs be
D;k ijh{kkvksa dks lekIr dj nsuk pkfg,A increased?
Argument/rdZ :- Hkkjr esa thou dh yach mez dks è;ku esa j[krs gq,] D;k ljdkjh
(i) No because expiration of every existing item is unfair. ukSdfj;ksa esa lsokfuo`fÙk dh vk;q&lhek c<+kbZ tkuh pkfg,\
ugha] D;ksfa d gj ekStwnk oLrq dh lekfIr vuqfpr gSA Arguments/rdZ
(ii) Yes, because change is law of nature. I. Yes, other countries have decided so long before.
gk¡] D;ksfa d ifjorZu gh izd`fr dk fu;e gSA gka] vU; ns'kksa us cgqr igys gh QSlyk dj fy;k gSA
(V) Related to subject :- Argument is related to the subject. II. Yes, it is the actual demand of lakh of employees.
rdZ fo"k; ls lEcfU/kr gksrk gSA gka] ;g yk[kksa deZpkfj;ksa dh okLrfod ekax gSA
Statement/dFku :- Computer education should be made Sol. (b)
compulsory in school.? Argument I is weak as it is bassed on example. Argument II is
D;k, dEI;wVj f’k{kk dks fo|ky; esa vfuok;Z djuk pkfg,A strong as it is the demand of lakhs of employees, it is justified.
Argument/rdZ :- rdZ I detksj gS D;ksafd ;g mnkgj.k ij vkèkkfjr gSA rdZ II etcwr
(i) Yes, because it will increase the number of teachers in gSA D;ksfa d ;g yk[kksa deZpkfj;ksa dh ekax gS] ;g mfpr gSA
schools. 3. Statement/dFku
gk¡ D;ksfa d blls fo|ky; esa f’k{kdksa dh la[;k c<+ tk,xhA Should new universities be established in India?
(ii) In view of today’s era, it is very important to teach children D;k Hkkjr esa u, foÜofo|ky;ksa dh LFkkiuk gksuh pkfg,\
computer education Arguments/rdZ
vkt ds ;qx dks ns[krs gq, cPpksa dks dEI;qVj fd f’k{kk nsuk cgqr gh I. No, we have not yet achieved the target of literacy.
t:jh gSA ugha] geus vHkh rd lk{kjrk dk y{; gkfly ugha fd;k gSA
Direction (1-5): Each question below is followed by two II. No, we have to face the problems of highly educated but
arguments numbered I and II. You have to decide which of the unemployed youths.
argument is a ‘strong’ argument and which is a ‘weak’ ugha] gesa mPp f'kf{kr ysfdu csjkstxkj ;qokvksa dh leL;kvksa dk
argument. lkeuk djuk gksxkA
uhps fn, x, çR;sd ç'u ds ckn nks rdZ I vkSj II fn, x, gSA vkidks Sol. (d)
;g r; djuk gksxk fd dkSu lk rdZ ,d etcwr *rdZ gS vkSj dkSu Both are weak. There is no relation between the establishment
lkA detksj* rdZ gSA of universities and the achievement of the literacy target. The
Given answer second argument appears illogical that due to establishment of
(a) if only argument I is strong/;fn dsoy rdZ I etcwr gSA more universities we shall have highly educated and
(b) if only argument II is strong/;fn dsoy rdZ II etcwr gSA unemployed youths. Hence, both the arguments are weak.

327
Download Free PDFs & e-Books from Neon Classes App

nksuksa detksj gSA foÜofo|ky;ksa dh LFkkiuk vkSj lk{kjrk y{; dh III. No, the films portrary the society and hence such scenes
miyfCèk ds chp dksbZ lacèa k ugha gSA nwljk rdZ vrkfdZd yxrk gS fd should be an integral part of the movie, if the storyline
vfèkd foÜofo|ky;ksa dh LFkkiuk ds dkj.k gekjs ikl mPp f'kf{kr demands so.
vkSj csjkstxkj ;qok gksaxsA blfy,] nksuksa rdZ detksj gSA ugha] fQYesa lekt dks fpf=r djrh gSa vkSj blfy, bl rjg ds –
4. Statement/dFku ';ksa dks fQYe dk ,d vfHkUu fgLlk gksuk pkfg,] vxj dgkuh ,slk
Should non-vegetarian food be totally banned in our Country? ekaxrh gSA
D;k gekjs ns'k esa ekalkgkjh Hkkstu ij iwjh rjg ls çfrcaèk yxk nsuk (a) None is strong/dksbZ Hkh etcwr ugha gS
pkfg,\ (b) I and II are strong/I vkSj II etcwr gSA
Arguments/rdZ (c) II and III are strong /II vkSj III etcwr gSA
I. Yes, it is expensive and therefore, it is beyond the means of (d) I and III are strong/I vkSj III etcwr gSA
most people in our country. (e) All are strong/lHkh etcwr gSA
gka] ;g egaxk gS vkSj blfy,] ;g gekjs ns'k esa vfèkdka'k yksxksa ds Sol. (d)
ekè;e ls ijs gSA Argument I is strong because such a reduction in trend will be
II. No, nothing should be banned in a democratic country like a desirable consequence. Argument II is weak as it silent as to
ours. what effect the ban will have on the creative pursuits. III is
ugha] gekjs tSls yksdrkaf=d ns'k esa dqN Hkh çfrcafèkr ugha gksuk strong as a ban will take away from the power of the portrayal.
pkfg,A rdZ I etcwr gSA D;ksfa d bl rjg dh ço`fÙk esa deh ,d okaNuh;
Sol. (b) ifj.kke gksxkA rdZ II detksj gS D;ksfa d ;g ekSu gS fd çfrca/k dk
Restriction on the diet of people will deny them their basic s kA III etcwr gSA D;ksfa d ,d
jpukRed xfrfof/k;ksa ij D;k çHkko iM+x
human right. So, only Argument II holds strong. çfrca/k fp=.k dh 'kfä ls nwj ys tk,xkA
yksxksa ds vkgkj ij çfrcaèk mUgsa muds cqfu;knh ekuo vfèkdkj ls 7. Statement/dFku
oafpr djsxkA blfy,] dsoy rdZ II etcwr gSA Should the Government order closure of all educational
5. Statement/dFku institutions for a month to avoid fast spreading of the
Should there be a ban on product advertisements? contagious viral infection?
D;k mRikn foKkiuksa ij çfrcaèk gksuk pkfg,\ D;k laØked ok;jy laØe.k ds rsth ls çlkj ls cpus ds fy,
Arguments/rdZ ljdkj dks lHkh 'kS{kf.kd laLFkkuksa dks ,d eghus ds fy, can djus
I. No, it is an age of advertising unless your advertisement is dk vkns'k nsuk pkfg,\
better than that of your other competitors; the products will Arguments/rdZ
not be sold. I. No, the closure of educational institutions alone is not the
ugha] ;g foKkiu dk nkSj gS tc rd fd vkidk foKkiu vkids vU; solution for curbing the spread of the viral infection.
çfrLifèkZ;ksa dh rqyuk esa csgrj u gks] mRikn ugh fcdsxkA ugha] vdsys f'k{k.k laLFkkuksa dk can gksuk ok;jy laØe.k ds çlkj dks
II. Yes, the money spent on advertising is very huge and it jksdus dk mik; ugha gSA
inflates the cost of the products. II. No, students will visit crowded place like malls, markets,
gka] foKkiu ij [kpZ gksus okyk iSlk cgqr cM+k gS vkSj ;g mRiknksa dh play grounds etc in more numbers and spread the disease, as
ykxr dks c<+krk gSA they will have a lot of spare time at their disposal.
Sol. (e) ugha] Nk= HkhM+&HkkM+ okyh txg tSls e‚y] ckt+kj] [ksy ds eSnku
It is a known fact that unless you create awareness through bR;kfn esa vfèkd la[;k esa tk,¡xs vkSj chekjh QSyk,¡ xs] D;ksfa d muds
advertisement, about your product, you lag behind from your ikl muds fuiVku esa cgqr de le; gksxkA
competitors. Also heavy cost on advertisements adds to your III. Yes, young persons are more prone to get affected by the
product. Hence, both the arguments are strong. viral infection and hence, they should remain indoors.
;g ,d Kkr rF; gS fd tc rd vki vius mRikn ds ckjs esa foKkiu gka] ;qok O;fä;ksa dks ok;jy laØe.k ls çHkkfor gksus dk [krjk gksrk
ds ekè;e ls tkx:drk iSnk ugha djrs gSa] rc rd vki vius gS vkSj blfy,] mUgsa ?kj ds vanj jguk pkfg,A
çfrLifèkZ;ksa ls fiNM+ tkrs gSA foKkiuksa ij Hkh Hkkjh ykxr vkids (a) None is strong/dksbZ Hkh etcwr ugha gS
mRikn esa tqM+ tkrh gSA blfy,] nksuksa rdZ etcwr gSA (b) Only I is strong/dsoy I etcwr gSA
6. Statement/dFku (c) Only III is strong /dsoy III etcwr gSA
Should smoking cigarettes and drinking alcohol by the actors (d) I and II are strong/I vkSj II etcwr gSA
be completely banned in the movies in India? (e) All are strong/lHkh etcwr gSA
D;k Hkkjr esa fQYeksa esa flxjsV ihuk vkSj vfHkusrkvksa }kjk 'kjkc ihuk Sol. (c)
iwjh rjg ls çfrcaf/kr gksuk pkfg,\ Argument I is weak as it merely tries to evade the issue.
Arguments/rdZ Argument II may turn out to be true but it is bassed on a
I. Yes, this will significantly reduce the trend on smoking negative mind set, may be it’s mere of an assumption. Hence, II
cigarettes and drinking alcohol among the youth in India. is weak. Argument III gets into the reason and is therefore
gka] blls Hkkjr esa ;qokvksa ds chp flxjsV ihus vkSj 'kjkc ihus ds strong.
pyu esa dkQh deh vk,xhA rdZ I detksj gS D;ksfa d ;g dsoy eqís dks feVkus dh dksf'k'k djrk gSA
II. No, there should be no such ban on the creative pursuits of rdZ II lgh gks ldrk gS] ysfdu ;g ,d udkjkRed fnekx lsV ij
the filmmaker. vk/kkfjr gS] ;g ,d /kkj.kk dk ek= gks ldrk gSA blfy,] II detksj
ugha] fQYe fuekZrk dh jpukRed xfrfofèk;ksa ij bl rjg dk çfrcaèk gSA rdZ III dkj.k esa gks tkrk gS vkSj blfy, etcwr gksrk gSA
ugha gksuk pkfg,A 8. Statement/dFku

328
Best App for Govt. Jobs : Neonclasses (Download Now)

Should there be a complete ban on setting up thermal power arguments numbered I, II and III. You have to decide which of
plants in India? the arguments is ‘strong’ and which is ‘weak’.
D;k Hkkjr esa FkeZy ikoj IykaV yxkus ij iw.kZ çfrcaèk gksuk pkfg,\ egRoiw.kZ ç'uksa ds ckjs esa fu.kZ; ysus esa] ,d etcwr rdZ vkSj ,d
Arguments/rdZ detksj rdZ ds chp varj djuk okaNuh; gSA ,d etcwr rdZ nksuksa
I. Yes, this is the only way to control further addition to egRoiw.kZ vkSj lh/ks loky ls lacaf/kr gksuk pkfg,A ,d detksj rdZ
environmental pollution. lh/ks loky ls lacaf/kr ugha gks ldrk gS vkSj ;g ekewyh egRo dk gks
gka] ;g i;kZoj.k çnw"k.k ds vfrfjä fu;a=.k dk ,dek= rjhdk gSA ldrk gS ;k loky ds Li"V igyw ls lacaf/kr gks ldrk gSA uhps fn,
II. No, there is a huge shortage of electricity in most parts of x, çR;sd ç'u ds ckn I, II vkSj III rhu rdZ fn, x, gSA vkidks
the country and hence, generation of electricity needs to be ;g r; djuk gksxk fd dkSu lk rdZ etcwr gSA vkSj dkSu lk
augmented. detksj gSA
ugha] ns'k ds vfèkdka'k fgLlksa esa fctyh dh Hkkjh deh gS vkSj blfy,] 10. Statement/dFku
fctyh mRiknu dks c<+kus dh vko';drk gSA Should the Government impose restrictions on access to
III. No, many developed countries continue to set up thermal sensitive information to the journalists to avoid the media
power plants in their countries. hype?
ugha] dbZ fodflr ns'k vius ns'kksa esa FkeZy ikoj IykaV yxkuk tkjh ljdkj dks ehfM;k çpkj ls cpus ds fy, i=dkjksa dks laosnu'khy
j[krs gSA tkudkjh rd igqp
a ij çfrca/k yxkuk pkfg,\
(a) None is strong/dksbZ Hkh etcwr ugha gSA Arguments/rdZ
(b) Only I is strong/dsoy I etcwr gSA I. Yes, the media creates hype and publishes distorted
(c) Only II is strong /dsoy II etcwr gSA information at times.
(d) Only III is strong /dsoy III etcwr gSA gka] ehfM;k çpkj cukrk gS vkSj dbZ ckj fo—r tkudkjh çdkf'kr
(e) Either I Or II is strong/;k rks I ;k II etcwr gSA djrk gSA
Sol. (c) II. No, journalists should have an access to all the information
Argument I is weak because of the use of only Argument II is as media is the best source to expose the malfunction in the
strong as the country’s power need cannot be ignored. society.
Argument III is weak because it is the argument based on ugha] i=dkjksa ds ikl lHkh lwpukvksa dh igqap gksuh pkfg, D;ksfa d
example. lekt esa [kjkch dks mtkxj djus ds fy, ehfM;k lcls vPNk lzksr
rdZ I detksj 'dsoy' 'kCn ds iz;ksx ds dkj.k rdZ II dk etcwr gSA gSA
D;ksfa d ns'k dh 'kfä dh vko';drk dks utjvankt ugha fd;k tk III. Yes, at times it leads to harassment of those who are
ldrk gSA rdZ III detksj gS D;ksafd ;g mnkgj.k ds vk/kkj ij rdZ affected and alleged to be involved in the crisis.
gSA gka] dbZ ckj ;g mu yksxksa dk mRihM+u djrk gS tks çHkkfor gksrs gSa
9. Statement/dFku vkSj ladV esa 'kkfey gksus dk vkjksi yxkrs gSA
Should English be the medium of instruction for higher (a) All I, II and III are strong/lHkh I, II vkSj III etcwr gSAA
education in India? (b) I and II are strong/I vkSj II etcwr gSAA
D;k Hkkjr esa mPp f'k{kk ds fy, vaxzsth f'k{kk dk ekè;e gksuk (c) II and III are strong/II vkSj III etcwr gSAA
pkfg,\ (d) Only II is strong/dsoy II etcwr gSA
Arguments/rdZ (e) None is strong/dksbZ Hkh etcwr ugha gSA
I. Yes, even in advanced countries like UK and USA, the Sol. (a)
medium of instruction is English for higher education. Argument I is strong because at times media can be
gka] ;wds vkSj ;w,l, tSls mUur ns'kksa esa Hkh] mPp f'k{kk ds fy, f'k{kk responsible for creating hype of a distorted information. Also,
dk ekè;e vaxzsth gSA journalists play an important role in exposing the
II. Yes, English is much widely spoken language in the world malfunctioning of the society, so Argument II is also strong.
today. Argument III is strong because it some times leads to
gka] vkt nqfu;k esa vaxzsth cgqr vfèkd cksyh tkus okyh Hkk"kk gSA harassment of affected person. So, all the three statement are
Sol. (b) strong.
Anything successful in other countries may not succeed in rdZ I etcwr gSA D;ksfa d dbZ ckj ehfM;k fo—r lwpuk ds çpkj ds
India. However, since English is much widely spoken language fy, ftEesnkj gks ldrk gSA lkFk gh] i=dkj lekt dh [kjkch dks
in the world today and hence, should be adopted, is a strong mtkxj djus esa egRoiw.kZ Hkwfedk fuHkkrk gS] blfy, rdZ II Hkh etcwr
idea. Hence, argument II is the strong argument. gSA rdZ III etcwr gSA D;ksfa d ;g dqN ckj çHkkfor O;fä dk mRihM+u
dksbZ pht vxj fons'kkh ns'kks esa lQy gS] bldk eryc ;g ugha gS djrk gSA blfy,] rhuksa dFku etcwr gSA
fd Hkkjr esa Hkh gksxkA gkyk¡fd] vkt nqfu;k esa vaxzsth cgqr vf/kd 11. Statement/dFku
cksyh tkus okyh Hkk"kk gS vkSj blfy,] bls viuk;k tkuk pkfg,] ,d Should the Government introduce a system of obtaining bond
etcwr fopkj gSA blfy,] rdZ II etcwr rdZ gSA from students for working in India before sanctioning
Directions (10-15) In making decisions about important education loanfor higher studies
questions, it is desirable to distinguish between a ‘strong’ D;k ljdkj dks f'k{kk _.k ds fy, mPp v/;;u dks eatwjh nsus ls
argument and a ‘weak’ argument. A ‘strong’ argument must be igys Hkkjr esa dke djus ds fy, Nk=ksa ls izfrKk&i= çkIr djus dh
both important and directly related to the question. A ‘weak’ ç.kkyh 'kq: djuh pkfg,
argument may not be directly related to the question and may Arguments/rdZ
be of minor importance or may be related to the trival aspect I. No, this is not a workable solution and will obstruct the
of the question. Each question below is followed by three development of young taken in the country.

329
Download Free PDFs & e-Books from Neon Classes App

ugha] ;g ,d O;kogkfjd lek/kku ugha gS vkSj ns'k esa mBk, x, gk¡] blls Nk=ksa dks ,d o"kZ ds varjky ds fcuk viuh f’k{kk iwjh djus
;qokvksa ds fodkl esa ck/kk cusxkA esa enn feysxhA
II. Yes, this is the only way to ensure use of the talent of our II. Yes, it is a futuristic strategy to help students and motivate
country for the development of the country and not only an them to higher education.
individual. gk¡] ;g Nk=ksa dh lgk;rk djus vkSj mUgsa mPp f’k{kk ds fy, izsfjr
gka] ;g ns'k ds fodkl ds fy, gekjs ns'k dh çfrHkk dk mi;ksx djus dh Hkfo";ksUeq[kh j.kuhfr gSA
lqfuf'pr djus dk ,dek= rjhdk gS] u fd dsoy ,d O;fä ds fy, III. Not so, such students do not like to continue their next
fodklA studies without passing all the subjects in the web.
III. No, this step will be too harsh. ugha] ,sls Nk= HSC esa lHkh fo"k;ksa dks ikl fd, fcuk vxyh i<+kbZ
ugha] ;g dne cgqr dBksj gksxkA tkjh djuk ilan ugha djrs gSaA
(a) Only I is strong/ dsoy I etcwr gSA (a) Only I is solid@dsoy I Bksl gSA
(b) Only II is strong/ dsoy II etcwr gSA (b) Only II is solid@dsoy II Bksl gSA
(c) Only I and II are strong/ dsoy I vkSj II etcwr gSA (c) Only III is solid@dsoy III Bksl gSA
(d) None is strong/ dksbZ Hkh etcwr ugha gSA (d) Either II or III and I is solid/;k rks II ;k III vkSj I Bksl gSA
(e) None of these/ buesa ls dksbZ ugha (e) None of these@buesa ls dksbZ ugha
Sol. (a) Sol. (a)
Only Argument I holds strong because this is definitely not the Only logic seems solid. This will save students' time.
solution. Arguments II and III are weak arguments. dsoy rdZ I Bksl izrhr gksrk gSA blls Nk=ksa ds le; dh cpr gksxhA
dsoy rdZ I etcwr j[krk gSA D;ksfa d ;g fuf'pr :i ls lek/kku ugha 14. Statement/dFku
gSA rdZ II vkSj III detksj rdZ gSA Should there be a common syllabus for all subjects in graduate
13. Statement/dFku courses in all the universities across the country?
Should there be a complete ban on Indian professionals D;k ns'k Hkj ds lHkh fo'ofo|ky;ksa esa Lukrd ikBîØeksa esa lHkh
seeking jobs elsewhere after getting their education in India? fo"k;ksa ds fy, ,d leku ikBîØe gksuk pkfg,\
D;k Hkkjr esa viuh f'k{kk çkIr djus ds ckn dgha vkSj ukSdjh ikus Arguments/rdZ
okys Hkkjrh; is'ksojksa ij iw.kZ çfrca/k yxuk pkfg,\ I. Yes, this is the only way to spring in uniformity in the
Arguments/rdZ education system in the country.
I. Yes, this is the only way to sustain present rate of gk¡] ;g ns'k esa f'k{kk ç.kkyh esa ,d:irk ds fy, ,dek= rjhdk gSA
technological development in India. II. Yes, it will help standardize the quality of graduation
gk¡] ;g Hkkjr esa rduhdh fodkl dh orZeku nj dks cuk, j[kus dk certificates being given by different universities in the country.
,dek= rjhdk gSA gka] ;g ns'k ds fofHkUu fo'ofo|ky;ksa }kjk fn, tk jgs Lukrd
II. No, the Indians settled abroad send huge amount of foreign çek.ki=ksa dh xq.koÙkk dks ekudh—r djus esa enn djsxkA
exchange and this constitutes a significant part of foreign III. No, each university should have the autonomy to decide its
exchange reserve. syllabus based on the specific requirement of the university.
ugha] fons'kksa esa cls Hkkjrh; cM+h ek=k esa fons'kh eqæk Hkstrs gSa ugha] çR;sd fo'ofo|ky; dks fo'ofo|ky; dh fof'k"V vko';drk ds
vkSj ;g ,d egRoiw.kZ fgLlk gS] fons'kh eqæk HkaMkj dk A vk/kkj ij viuk ikBîØe r; djus dh Lok;Ùkrk gksuh pkfg,A
III. No, the practical knowledge gained by Indians by working (a) None is strong/dksbZ Hkh etcwr ugha gSA
in other countries help India to develop its economy. (b) Only I is strong dsoy I etcwr gSA
ugha] vU; ns'kksa esa dke djds Hkkjrh;ksa }kjk çkIr O;kogkfjd Kku (c) Only II is strong/ dsoy II etcwr gSA
Hkkjr dks viuh vFkZO;oLFkk dks fodflr djus esa enn djrk gSA (d) I and II are strong/ I vkSj II etcwr gSA
(a) None is strong/ dksbZ Hkh etcwr ugha gSA (e) II and III are strong/II vkSj III etcwr gSA
(b) All are strong/ lHkh etcwr gSAa Sol. (e)
(c) I and II are strong/ I vkSj II etcwr gSA In Argument I the use of key word only makes it weak. If there
(d) Only III is strong/ dsoy III etcwr gSA is common syllabus for graduation across the universities.
(e) II and III are strong/ II vkSj III etcwr gSA Then this will standardize the quality of graduation
Sol. (a) certificates, which is desirable, hence Argument II is strong
None of the given arguments is strong. Argument III is also strong as specific requirement and
fn, x, rdksaZ esa ls dksbZ Hkh etcwr ugha gSA autonomy of the university cannot be overlooked.
14. Statement/dFku dsoy 'kCn dk iz;ksx rdZ I dks detksj cukrk gSA ;fn fo'ofo|ky;ksa
Students who have failed in one or two subjects in the Bb, esa Lukrd ds fy, lkekU; ikBîØe gSA fQj ;g Lukrd çek.ki=ksa
should they all be allowed to enroll in degree courses and dh xq.koÙkk dks ekudh—r djsxk] tks okaNuh; gS] blfy, rdZ II
continue on to the next course provided they successfully pass etcwr gSA rdZ III Hkh fof'k"V vko';drk ds :i esa etcwr gSA vkSj
the supplementary examination? fo'ofo|ky; dh Lok;Ùkrk dh vuns[kh ugha dh tk ldrh gSA
tks Nk= HSC esa ,d ;k nks fo"k;ksa esa Qsy gq, gSa] D;k bu lHkh dks 15. Statement/dFku
fMxzh dkslZ esa nkf[kyk ysus nsuk pkfg, vkSj vxyh i<+kbZ tkjh j[kus Should admission to all professional courses be made on the
nsuk pkfg, c’krsZ os lIyhesVa jh ijh{kk dks lQyrkiwoZd ikl dj ysa\ basis of past academic performance rather than through
Arguments/rdZ entrance tests?
I. Yes, it will help the students to complete their education lHkh O;kolkf;d ikBîØeksa esa ços'k ds fy, ços'k ijh{kk ds ctk;
without a gap of one year. fiNys 'kS{kf.kd çn'kZu ds vk/kkj ij ços'k fn;k tkuk pkfg,\
Arguments/rdZ

330
Best App for Govt. Jobs : Neonclasses (Download Now)

I. Yes, it will be beneficial for those candidates who are unable gka] blls bu laxBuksa dh n{krk ds Lrj dks c<+kus esa enn feysxh vkSj
to bear the expenses of entrance tests. ;s ykHknk;d çfr"Bku cu tk,axsA
gk¡] ;g mu mEehnokjksa ds fy, Qk;nsean gksxk tks ços'k ijh{kk dk Sol. (b)
[kpZ ogu djus esa vleFkZ gSA Only argument II seems to be suitable.
II. Yes, many deserving candidates securing high marks in dsoy rdZ II mi;qä çrhr gksrk gSA
their qualifying academic examinations do not perform well on 17. Statements/dFku :
such entrance tests. Should there be compulsory medical examination of both the
gka] vius ;ksX; 'kS{kf.kd ijh{kkvksa esa mPp vad gkfly djus okys dbZ man and the woman before they marry each other?
;ksX; mEehnokj ,sls ços'k ijh{kkvksa esa vPNk çn'kZu ugha djrs gSA D;k ,d nwljs ls 'kknh djus ls igys iq#"k vkSj efgyk nksuksa dh
III. No, the standard of examinations and assessment vfuok;Z fpfdRlk ijh{kk gksuh pkfg,\
conducted by different boards and universities are not Arguments/rdZ :
comparable and hence there is a need to conduct entrance I. No, this is an intrusion to the privacy of an individual and
tests to calibrate them on a common yardstick. hence cannot be tolerated.
ugha] fofHkUu cksMksaZ vkSj fo'ofo|ky;ksa }kjk vk;ksftr ijh{kkvksa vkSj ugha] ;g fdlh O;fä dh xksiuh;rk ds fy, ,d ?kqliSB gS vkSj
ewY;kadu ds ekud rqyuh; ugha gSa vkSj blfy, mUgsa ,d lkekU; blfy, bls cnkZ'r ugha fd;k tk ldrk gSA
ekin.M ij tkapus ds fy, ços'k ijh{kk vk;ksftr djus dh II. Yes, this will substantially reduce the risk of giving birth to
vko';drk gSA children with serious ailments.
(a) I and II are strong/ I vkSj II etcwr gSA gka] blls xaHkhj chekfj;ksa okys cPpksa dks tUe nsus dk tksf[ke dkQh
(b) II and III are strong/ II vkSj III etcwr gSA gn rd de gks tk,xkA
(c) I and III are strong/ I vkSj III etcwr gSA Sol. (b)
(d) III is strong/ III etcwr gSA Only argument II is strong.
(e) All are strong/ lHkh etcwr gSA dsoy rdZ II etcwr gSA
Sol. (d) Instructions (20-21): Each question below is followed by two
Only Argument III is strong because the standard of arguments numbered I and II. You have to decide which of the
examinations and assessment conducted by different boards arguments is a ‘Strong’ argument and which is a ‘Weak’
and universities are not comparable and entrance test help argument.
calibrate them on a common yardstick. funs'Z k% uhps fn, x, çR;sd ç'u ds ckn I vkSj II nks rdZ fn, x, gSA
dsoy rdZ III etcwr gSA D;ksafd fofHkUu cksMksZa vkSj fo'ofo|ky;ksa }kjk vkidks ;g r; djuk gksxk fd dkSu lk rdZ etcwr rdZ gS vkSj dkSu
vk;ksftr ijh{kkvksa vkSj ewY;kadu ds ekud rqyuh; ugha gSa vkSj ços'k lk rdZ 'detksj' gSA
ijh{kk mUgsa ,d lkekU; ekin.M ij tkapus esa enn djrh gSA Given answer
Direction (Q16-17): Each question below is followed by two (a) if only argument I is ‘strong’/;fn dsoy rdZ I *etcwr gSA*
arguments numbered I and II. You have to decide which of the (b) if only argument II is ‘strong’/;fn dsoy rdZ II *etcwr gSA*
argument is a ‘strong’ argument and which is a ‘weak’ (c) if either I or II is ‘strong’/;fn ;k rks I ;k II etcwr gSA
argument. (d) if neither I nor II is ‘strong’/;fn u rks I vkSj u gh II etcwr gSA
uhps fn, x, çR;sd ç'u ds ckn nks rdZ I vkSj II fn, x, gSA vkidks (e) if both I and II are ‘strong’/;fn I vkSj II nksuks½a etcwr *gSa
;g r; djuk gksxk fd dkSu lk rdZ ,d etcwr *rdZ gS vkSj dkSu 20. Statements/dFku :
lkA detksj* rdZ gSA Should India engage into a dialogue with neighboring
Given answer countries to stop cross border tension?
(a) if only argument I is strong/;fn dsoy rdZ I etcwr gSA D;k Hkkjr dks lhek ikj ruko dks jksdus ds fy, iM+kslh ns'kksa ds lkFk
(b) if only argument II is strong/;fn dsoy rdZ II etcwr gSA ckrphr esa 'kkfey gksuk pkfg,\
(c) if either argument I or argument II is strong/;fn rdZ I ;k Arguments/rdZ :
rdZ II etcwr gSA I. Yes, this is the only way to reduce the cross border terrorism
(d) if neither argument I nor argument II is strong/;fn u rks and stop loss of innocent lives.
rdZ I vkSj u gh rdZ II etcwr gSA gk¡] ;g lhek ikj vkradokn dks de djus vkSj funksZ"k yksxksa ds
(e) if both arguments I and II are strong./;fn nksuksa rdZ I vkSj II uqdlku dks jksdus dk ,dek= rjhdk gSA
etcwr gSAA II. No, neighbouring countries cannot be relied upon in such
16. Statements/dFku : matters, they may still engage in subversive activities.
Should all the nonperforming employees in the public sector ugha] iM+kslh ns'kksa dks ,sls ekeyksa esa Hkjkslk ugha fd;k tk ldrk gS] os
be compulsorily retrenched from service? vHkh Hkh fo/oald xfrfof/k;ksa esa layXu gks ldrs gSA
D;k lkoZtfud {ks= ds lHkh xSj&dkedkth deZpkfj;ksa dks vfuok;Z Sol. (a)
:i ls lsok ls gVk fn;k tkuk pkfg,\ Only argument I holds strong. In the present context the
Arguments/rdZ : solution of any major problem lies in the proper dialogue
I. No, this will give an unjust handle to the management and between the concerned parties. Argument II is based on
they may use it indiscriminately. individual’s opinion and hence it cannot be strong.
ugha] ;g çca/ku dks ,d vuqfpr izca/k dk rjhdk nsxk vkSj os bldk dsoy rdZ I etcwr j[krk gSA orZeku lanHkZ esa fdlh Hkh cM+h leL;k
vuqfpr mi;ksx dj ldrs gSA dk lek/kku lacaf/kr i{kksa ds chp mfpr ckrphr esa fufgr gSA rdZ II
II. Yes, this will help increase the level of efficiency of these O;fä dh jk; ij vk/kkfjr gS vkSj blfy, ;g etcwr ugha gks ldrk
organizations and these will become profitable gSA
establishments. 21. Statements/dFku :

331
Download Free PDFs & e-Books from Neon Classes App

Should the railways in India be privatized in a phased manner ughA igys ge vius gh yksxksa dh leL;kvksa dks gy djsa tSls xjhch]
like other public sector enterprises? dqiks"k.kA
D;k Hkkjr esa jsyos dk vU; lkoZtfud {ks= ds m|eksa dh rjg Sol. (a)
pj.kc) rjhds ls futhdj.k fd;k tkuk pkfg,\ A peace-loving nation like India can well join an international
Arguments/rdZ : forum which seeks to bring different nations on friendly terms
I. Yes, this is the only way to bring in competitiveness and with each other. So, argument I holds strong. Argument II
provide better service to the public. highlights a different aspect. The internal problems of a nation
gk¡] ;g çfrLi/kkZ esa ykus vkSj turk dks csgrj lsok çnku djus dk should not debar it from strengthening international ties. So,
,dek= rjhdk gSA argument II is vague.
II. No, this will pose a threat to national security of our country Hkkjr tSlk 'kkafrfç; jk"Vª ,d varjjk"Vªh; eap ls tqM+ ldrk gS tks
as multinationals will enter into the fray. fofHkUu ns'kksa dks ,d&nwljs ds lkFk fe=rkiw.kZ 'krksaZ ij ykus dk ç;kl
ugha] ;g gekjs ns'k dh jk"Vªh; lqj{kk ds fy, [krjk iSnk djsxk D;ksfa d djrk gSA blfy,] rdZ I etcwr gSA rdZ II ,d vyx igyw ij çdk'k
cgqjk"Vªh; daifu;ka eSnku esa mrjsaxhA Mkyrk gSA fdlh jk"Vª dh vkarfjd leL;kvksa dks varjkZ"Vªh; laca/kksa dks
Sol. (d) etcwr djus ls ugha gVkuk pkfg,A rks] rdZ II detksj gSA
The use of the word “only” in the argument I makes it invalid. 24. Statement/dFku :
The policy of open market advocates privatization. Each Should the vehicles older than 15 years be rejected in metros
country is opening its market for foreign investors. How, in India?
privatization will pose a threat to national security? D;k Hkkjr esa egkuxjksa esa 15 lky ls vf/kd iqjkus okguksa dks vLohdkj
rdZ I esa 'dsoy' 'kCn dk mi;ksx bls vekU; cukrk gSA [kqys cktkj dj fn;k tkuk pkfg,\
dh uhfr futhdj.k dh odkyr djrh gSA çR;sd ns'k fons'kh fuos'kdksa Arguments/rdZ:
ds fy, viuk cktkj [kksy jgk gSA dSls] futhdj.k jk"Vªh; lqj{kk ds I. Yes. This is a significant stop be lower down the pollution
fy, [krjk gksxk\ level in metros.
22. Statement/dFku : gk¡A ;g egkuxjksa esa çnw"k.k ds Lrj dks de djus okyk egRoiw.kZ
Should the educated unemployed youth be paid iM+ko gSA
‘unemployment allowance’ by the Government? II. No. it will be very difficult for vehicle owners to shift to
D;k ljdkj }kjk f'kf{kr csjkstxkj ;qokvksa dks 'csjkstxkjh HkÙkk* fn;k other parts in country because they will not get suitable job for
tkuk pkfg,\ their very existence.
Arguments/rdZ: ugha] okgu ekfydksa ds fy, ns'k ds vU; fgLlksa esa f'k¶V gksuk cgqr
I. Yes. It will provide them some monetary help to either seek eqf'dy gksxk D;ksfa d mUgsa vius vfLrRo ds fy, mi;qä ukSdjh ugha
employment or to kickstart some ‘self-employment’ venture. feysxhA
gk¡A ;g mUgsa jkstxkj dh ryk'k esa ;k dqN Lo&jkstxkj miØe dks Sol. (a)
vkxs djus ds fy, dqN ekSfæd lgk;rk çnku djsxkA Clearly, 15 year old vehicles are not Euro-compliant and hence
II. No. it will dampen their urge to do something to earn their cause much more pollution than the recent ones. So, argument
livelihood and thus promote idleness among the unemployed I holds. Argument II is vague since owners of these vehicles
youth. need not shift themselves. They might sell off their vehicles
ugha] ;g mudh vkthfodk dekus ds fy, dqN djus ds fy, muds and buy new ones – a small price which every citizen can
vkxzg dks de dj nsxk vkSj bl rjg csjkstxkj ;qokvksa ds chp afford for a healthy environment.
vkyL; dks c<+kok nsxkA tkfgj gS] 15 lky iqjkus okgu ;wjks&vkKkdkjh ugha gSa vkSj blfy,
Sol. (e) gky ds lk/kuksa dh rqyuk esa cgqr vf/kd çnw"k.k dk dkj.k gSA
Young people, who do not get employment due to the large blfy,] rdZ I etcwr gSA rdZ II vLi"V gS D;ksfa d bu okguksa ds
number of applicants in all fields, must surely be given ekfydksa dks [kqn dks LFkkukarfjr djus dh vko';drk ugha gSA os vius
allowance so that they can support themselves. So, argument I okguksa dks csp ldrs gSa vkSj u, [kjhn ldrs gSa & ,d NksVh lh
is valid. However, such allowances would mar the spirit to dher tks çR;sd ukxfjd LoLFk i;kZoj.k ds fy, [kpZ dj ldrk gSA
work, in them and make them idle. So, argument II also holds. 25. Statement/dFku :
;qok] ftUgsa lHkh {ks=ksa esa vkosndksa dh cM+h la[;k ds dkj.k jkstxkj Should persons convicted of criminal offences in the past be
ugha feyrk gS] mUgsa fuf'pr :i ls HkÙkk fn;k tkuk pkfg, rkfd os allowed to contest elections in India?
viuk leFkZu dj ldsA blfy,] rdZ I ekU; gSA gkyk¡fd] ,sls HkÙks D;k vrhr esa vkijkf/kd vijk/kksa ds nks"kh O;fä;ksa dks Hkkjr esa pquko
dke djus dh Hkkouk dks de djsaxsa ] muesa vkSj mUgsa fuf"Ø; dj yM+us dh vuqefr nh tkuh pkfg,\
a sA rks] rdZ II Hkh etcwr gSA
nsx Arguments/rdZ:
23. Statement/dFku : I. No. Such persons cannot serve the cause of the people and
Should India become a permanent member of UN’s Security country.
Council? ughA ,sls O;fä yksxksa vkSj ns'k ds dkj.k dh lsok ugha dj ldrsA
D;k Hkkjr dks UN dh lqj{kk ifj"kn dk LFkk;h lnL; cuuk pkfg,\ II. Yes. It is democracy – let people decide whom to vote.
Arguments/rdZ: gk¡A ;g yksdra= gS & yksxksa dks r; djuk pkfg, fd fdls oksV nsuk
I. Yes. India has emerged as a country which loves peace and gSA
amity. Sol. (a)
gk¡A Hkkjr ,d ,sls ns'k ds :i esa mHkjk gS tks 'kkafr vkSj lkSgknZ dks Clearly, persons with criminal background cannot stand to
ilan djrk gSA serve as the representatives of the common people. So, they
II. No. Let us first solve problems of our own people like should not be allowed to contest elections. Thus, only
poverty, malnutrition. argument I holds, while II does not.

332
Best App for Govt. Jobs : Neonclasses (Download Now)

tkfgj gS] vkijkf/kd i`"BHkwfe okys O;fä vke yksxksa ds çfrfuf/k;ksa ds fu.kZ; ysuk gksrk gS dFku ,oa fu"d"kZ esa fdlh Hkh izdkj fd
:i esa lsok djus ds fy, [kM+s ugha gks ldrsA blfy,] mUgsa pquko dYiuk ;k /kkj.kk djuk xyr ekuk tk,xk
yM+us dh vuqefr ugha nh tkuh pkfg,A bl çdkj] dsoy rdZ I etcwr Statement: Establishes a direct relationship between the
gS] tcfd II ugha gSA speaker and the listener.
dFku → dgus okys rFkk lquus okys ds chp lh/kk lEca/k lFkkfir
STATEMENT & CONCLUSION djrk gSA
Definition → In the statement and conclusion, assuming all the → There are some rules of weak and strong
facts of the statement to be 100% true, even if they are false, conclusions. Based on which the truth and untrue conclusion
the decision has to be made in the conclusions related is to be found.
to it, it is wrong to make any assumption or assumption in the → detksj rFkk etcwr fu"d"kZ ds dqN fu;e gSA ftlds
statement and conclusion. vk/kkj ij lR; rFkk vlR; fu"d"kZ dk irk yxkuk gSA
ifjHkk"kk → dFku ,oa fu"d"kZ esa dFku dh lkjh ckrksa dks 100% lR;
ekurs gq, pkgs os vlR; gh D;ksa u gks fu"d"kksZa esa mlls lEcfU/kr
Weak conclusion Strong detksj fu"d"kZ in the wrong / dFku conclusion,
conclusion statement the words
detksj ekuk tk,xkA
of
ls HkVdk gqvk
related to it
etcwr IV. The conclusion
is untrue if conclusion
fu"d"kZ xyr
will be
fu" given a will be
ekuk tk;sxk
considered
fu" Inference or considered correct. /
d"kZ assumptions. true/ fu"d"kZ lafo/kku
d"kZ fu"d"kZ esa vuqeku dk dFku esa la’kks/ku dh
I. If the statement I. If the ;k dYiuk nh lh/kk lEc/k ckrs dFku esa
is positive, its statement xbZ gks rks og lR; ekuk fd xbZ gks rks
conclusion contains vlR; gksrh gSA tk;sxk fu"d"kZ esa
will never be advice, V. If the time III. In the blls tqM+h
negative. results, period (past, conclusion ckrs lgh
;fn dFku publicity future) is said of ekuh tk;sxh
ldkjkRed gks advertiseme in the government V. Any event has
rks mldk nts, conclusion order, rule two aspects,
fu"d"kZ dHkh Hkh redressal, then it will be will be one of
udkjkRed ugha information considered as considered which is in
gksxk etc. and in weak straightfor the
the conclusion. ward statement
II. The opposite
conclusion ;fn fu"d"kZ esa le; because and the
conclusion of
it is stated dkfyu government other in the
the statement
that it ¼Hkwrdky] order is the conclusion.
will be
shows the Hkfo"dky½ ckrs responsibili Will be
considered
root cause dgh tk;s rks ty of people considered
incorrect /
of the og detksj to follow right
dFku dk
statement. fu"d"kZ ekuk rules. fdlh Hkh ?kVuk
foykse fu"d"kZ
;fn dFku esa tk;sxk ljdkjh vkns’k] ds nks igyw
xyr ekuk
lykg] VI. If the data in
fu;e dk gksrs gS
tk,xk
ifj.kke] the fu"d"kZ esa ftlesa ls
III. If only, sole, all,
izpkj conclusion is lh/kk lEc/k ,d dFku esa
always, every,
highest, foKkiu] insufficient, a lR; ekuk gksrk gS vkSj
highest, etc. fuokj.k] weak tk;sxk nwljk fu"d"kZ
come to the lwpuk vkfn conclusion D;ksfa d esa gS] tks
conclusion fn xbZ gS will be ljdkjh lgh ekuk
then that will vkSj fu"d"kZ considered. vkns’k] fu;e tk;sxk
be considered esa mlls fu"d"kZ esa vkdM+s dh ikyuk VI. Permission
as weak tqM+h ckrs vi;kZIr gksus djuk yksxks on its own
conclusion. dgh xbZ gks ij detksj dk ftEesnkjh can make
;fn fu"d"kZ esa tks dFku ds fu"d"kZ ekuk gS the
dsoy] ,dek=] ewy dkj.k tk;sxk IV. The talk of conclusion
lHkh] ges’kk dks fn[kkrk VII. The constitution stronger.
]izR;sd] gS rks lR; conclusion amendment [kqn dk vuqeku
mPpre] ekuk missing from has been fu"d"kZ dks
loksPZ pre tk;sxkA the statement done in the etcwr cuk
vkfn 'kCn vk II. Direct will be statement ldrk gSA
tk, rks og connection considered and in the VII. Follows the

333
Download Free PDFs & e-Books from Neon Classes App web. : www.neonclasses.com
conclusion ;fn u rks I vkSj u gh II vuqlj.k djrk gS ?kVrs lalkèkuksa ds lkFk vkcknh esa o`f) vkus
based on (e) if both I and II follow okys fnuksa esa dbZ fodkl'khy ns'kksa dk ifj–
the ;fn I vkSj II nksuksa vuqlj.k djrs gSa '; cuus tk jgh gSA
statement Sol. (e) Conclusions:
dFku ds vk/kkj Clearly, the statement stresses on the I. Population of developing countries
ij fu"d"kZ fact that heeding to national needs is will not continue to increase in future.
vuqlj.k much more important than space Hkfo"; esa fodkl'khy ns'kksa dh tula[;k ugha
djrk gks research programmes, which stray the c<+sxhA
VII. The concerned authorities from the former. II. It will be very difficult for the
universal So, both I and II follow. government of developing countries to
truth given Li"V :i ls] ;g dFku bl rF; ij cy provide its people decent quality of life.
in the nsrk gS fd jk"Vªh; vko';drkvksa ds fy, fodkl'khy ns'kksa dh ljdkj ds fy, vius
conclusion gsfxax varfj{k vuqlaèkku dk;ZØeksa dh rqyuk yksxksa dks thou dh lH; xq.koÙkk çnku
will be esa cgqr vfèkd egRoiw.kZ gS] tks iwoZ ls lacafèkr djuk cgqr eqf'dy gksxkA
considered vfèkdkfj;ksa dks HkVdkrk gSA blfy,] I vkSj Given answer
true/ II nksuksa vuqlj.k djrs gSaA (a) if only conclusion I follows
lkoZHkkSfed 2. Statement: ;fn dsoy fu"d"kZ I vuqlj.k djrk gS
lR; fu"d"kZ Quality has a price tag. India is (b) if only conclusion II follows
esa fn;k x;k allocating lots of funds to education. ;fn dsoy fu"d"kZ II vuqlj.k djrk gS
gksrks lR; xq.koÙkk dk ,d ewY; VSx gSA Hkkjr f'k{kk ds (c) if either I or II follows
ekuk tk;sxk fy, cgqr lkjs èku vkoafVr dj jgk gSA ;fn ;k rks I ;k II vuqlj.k djrk gS
IX. The Conclusions: (d) if neither I nor II follows
conclusion I. Quality of education in India would ;fn u rks I vkSj u gh II vuqlj.k djrk gS
will be improve soon. (e) if both I and II follow
considered Hkkjr esa f'k{kk dh xq.koÙkk esa tYn gh lqèkkj ;fn I vkSj II nksuksa vuqlj.k djrs gSa
to be true if gksxkA Sol. (b)
it is related II. Funding alone can enhance quality of The fact given in I is quite contrary to
to the education. the given statement. So, I does not
subject vdsys vuqnku f'k{kk dh xq.koÙkk dks c<+k follow. II mentions the direct
fu"d"kZ fo"k; ls ldrk gSA implications of the state discussed in the
lacaf/kr gksus Given answer statement. Thus, II follows.
ij lR; (a) if only conclusion I follows I esa fn;k x;k rF; fn, x, dFku ds
ekuk tk;sxk ;fn dsoy fu"d"kZ I vuqlj.k djrk gS fcYdqy foijhr gSA blfy,] I bldk ikyu
(b) if only conclusion II follows ugha djrk gSA fu"d"kZ II dFku esa ppkZ dh
;fn dsoy fu"d"kZ II vuqlj.k djrk gS xbZ jkT; ds çR;{k fufgrkFkksZa dk mYys[k
1. Statement: (c) if either I or II follows djrk gSA bl çdkj] II vuqlj.k djrk gSA
The Minister questioned the utility of ;fn ;k rks I ;k II vuqlj.k djrk gS 4. Statement:
the space research programme and (d) if neither I nor II follows Leaders, who raise much hue and cry
suggested its replacement by other ;fn u rks I vkSj u gh II vuqlj.k djrk gS about the use of Hindi, generally send
areas of felt national needs. (e) if both I and II follow their children to English medium
ea=h us varfj{k vuqlaèkku dk;ZØe dh ;fn I vkSj II nksuksa vuqlj.k djrs gSa schools.
mi;ksfxrk ij loky mBk;k vkSj eglwl fd, Sol. (a) usrk] tks fganh ds mi;ksx ds ckjs esa cgqr
x, jk"Vªh; t:jrksa ds vU; {ks=ksa }kjk blds According to the statement, funding is vfèkd 'kksjxqy djrs gSa] vke rkSj ij vius
çfrLFkkiu dk lq>ko fn;kA necessary to improve quality and India cPpksa dks vaxzsth ekè;e ds Ldwyksa esa Hkstrs gSaA
Conclusions: is allocating funds to education. This Conclusions:
I. Exploring the space does not means that quality of education will I. India lacks good Hindi medium
contribute to critical national needs. improve in India. So, I follows. But schools.
varfj{k dh [kkst egRoiw.kZ jk"Vªh; funding alone is sufficient to enhance Hkkjr esa vPNs fganh ehfM;e Ldwyksa dk vHkko
vko';drkvksa esa ;ksxnku ugha djrh gSA quality, is not true. So, II does not follow. gSA
II. Research should be oriented to dFku ds vuqlkj] xq.koÙkk esa lqèkkj ds fy, II. There is a world of difference
national needs. foÙk iks"k.k vko';d gS vkSj Hkkjr f'k{kk ds between preaching and practicing.
vuqlaèkku dks jk"Vªh; vko';drkvksa ds fy, fy, èku vkoafVr dj jgk gSA bldk eryc mins'k vkSj vH;kl ds chp cgqr vf/kd
mUeq[k gksuk pkfg,A gS fd Hkkjr esa f'k{kk dh xq.koÙkk esa lqèkkj varj gSA
Given answer gksxkA rks] I vuqlj.k djrk gSA ysfdu Given answer
(a) if only conclusion I follows xq.koÙkk c<+kus ds fy, vdsys èku i;kZIr gS] (a) if only conclusion I follows
;fn dsoy fu"d"kZ I vuqlj.k djrk gS lp ugha gSA rks] II vuqlj.k ugha djrk gSA ;fn dsoy fu"d"kZ I vuqlj.k djrk gS
(b) if only conclusion II follows 3. Statement: (b) if only conclusion II follows
;fn dsoy fu"d"kZ II vuqlj.k djrk gS Population increase coupled with ;fn dsoy fu"d"kZ II vuqlj.k djrk gS
(c) if either I or II follows depleting resources is going to be the (c) if either I or II follows
;fn ;k rks I ;k II vuqlj.k djrk gS scenario of many developing countries ;fn ;k rks I ;k II vuqlj.k djrk gS
(d) if neither I nor II follows in day to come. (d) if neither I nor II follows

334
Best App for Govt. Jobs : Neonclasses (Download Now) Neon Publications
;fn u rks I vkSj u gh II vuqlj.k djrk gS I. In future, we should provide good
(e) if both I and II follow teachers and equipment to these Given answer
;fn I vkSj II nksuksa vuqlj.k djrs gSa schools. (a) if only conclusion I follows
Sol. (c) Hkfo"; esa] gesa bu Ldwyksa dks vPNs f'k{kd ;fn dsoy fu"d"kZ I vuqlj.k djrk gS
Clearly, either I or II could be the reason vkSj midj.k miyCèk djkus pkfg,A (b) if only conclusion II follows
for the situation expressed in the II. We need not open any more schools ;fn dsoy fu"d"kZ II vuqlj.k djrk gS
statement. in the future. (c) if either I or II follows
Li"V :i ls] ;k rks I ;k II dFku esa O;ä gesa Hkfo"; esa vkSj vfèkd Ldwy [kksyus dh ;fn ;k rks I ;k II vuqlj.k djrk gS
fLFkfr dk dkj.k gks ldrk gSA vko';drk ugha gSA (d) if neither I nor II follows
5. Statement: Given answer ;fn u rks I vkSj u gh II vuqlj.k djrk gS
Our securities investment carry market (a) if only conclusion I follows (e) if both I and II follow
risk. Consult your investment advisor or ;fn dsoy fu"d"kZ I vuqlj.k djrk gS ;fn I vkSj II nksuksa vuqlj.k djrs gSa
agent before investing. (b) if only conclusion II follows Sol. (d)
gekjh çfrHkwfr;ksa dk fuos'k cktkj tksf[ke ;fn dsoy fu"d"kZ II vuqlj.k djrk gS Nothing about the details of the
mBkrk gSA fuos'k djus ls igys vius fuos'k (c) if either I or II follows employees’ income or the cause of their
lykgdkj ;k ,tsVa ls lykg ysAa ;fn ;k rks I ;k II vuqlj.k djrk gS refusal to declare their income and
Conclusions: (d) if neither I nor II follows assets, can be deducted from the given
I. One should not invest in securities. ;fn u rks I vkSj u gh II vuqlj.k djrk gS statement. So, neither I nor II follows.
fdlh dks çfrHkwfr;ksa esa fuos'k ugha djuk (e) if both I and II follow deZpkfj;ksa dh vk; ds fooj.k ;k mudh vk;
pkfg,A ;fn I vkSj II nksuksa vuqlj.k djrs gSa vkSj laifÙk dh ?kks"k.kk ls budkj djus ds
II. The investment advisor calculates the Sol. (a) dkj.k ds ckjs esa dqN Hkh ugha fn;k tk ldrk
market risk with certainty. Clearly, the statement stresses the need gS] tks fn, x, fooj.k ls dkVs tk ldrs gSaA
fuos'k lykgdkj fuf'prrk ds lkFk cktkj to provide good teachers and equipment blfy,] u rks I vkSj u gh II vuqlj.k djrk
ds tksf[ke dh x.kuk djrk gSA to schools. So, I follows. However, the gSA
Given answer fact that education system in India is 8. Statement/dFku
(a) if only conclusion I follows progressing with regard to schools does The Chief Minister emphasized the point
;fn dsoy fu"d"kZ I vuqlj.k djrk gS not imply that no more schools should that the Government will try its best for
(b) if only conclusion II follows be opened. So, II does not follow. the development of farmers and rural
;fn dsoy fu"d"kZ II vuqlj.k djrk gS Li"V :i ls] ;g dFku Ldwyksa esa vPNs poor.
(c) if either I or II follows f'k{kd vkSj midj.k çnku djus dh eq[;ea=h us bl ckr ij tksj fn;k fd
;fn ;k rks I ;k II vuqlj.k djrk gS vko';drk ij cy nsrk gSA rks] I vuqlj.k ljdkj fdlkuksa vkSj xzkeh.k xjhcksa ds fodkl
(d) if neither I nor II follows djrk gSA gkyk¡fd] ;g rF; fd Hkkjr esa ds fy, iwjh dksf'k'k djsxhA
;fn u rks I vkSj u gh II vuqlj.k djrk gS f'k{kk ç.kkyh Ldwyksa ds lacèa k esa çxfr dj Conclusions/fu"d"kZ
(e) if both I and II follow jgh gS] bldk vFkZ ;g ugha gS fd vkSj vfèkd I. The former government had not tried
;fn I vkSj II nksuksa vuqlj.k djrs gSa Ldwy ugha [kksys tkus pkfg,A rks] II vuqlj.k seriously for the development of these
Sol. (b) ugha djrk gSA people.
Investment in securities involves risk. 7. Statement: iwoZ ljdkj us bu yksxksa ds fodkl ds fy,
This does not mean that one should not The Government run company had xaHkhjrk ls ç;kl ugha fd;k FkkA
invest in securities. So, I does not follow. asked its employee to declare their II. This Government will not try
Since the statement advise one to income and assets but it has been seriously for the development of urban
consult investment advisor before strongly resisted by employees’ union poor.
investing, so II follows. and no employee is going to declare his ;g ljdkj 'kgjh xjhcksa ds fodkl ds fy,
çfrHkwfr;ksa esa fuos'k esa tksf[ke 'kkfey gSA income. xaHkhjrk ls ç;kl ugha djsxhA
bldk eryc ;g ugha gS fd fdlh dks ljdkj pykus okyh daiuh us vius deZpkjh Given answer
çfrHkwfr;ksa esa fuos'k ugha djuk pkfg,A dks viuh vk; vkSj laifÙk dh ?kks"k.kk djus (a) if only conclusion I follows
blfy,] I bldk ikyu ugha djrk gSaA pwafd ds fy, dgk Fkk] ysfdu deZpkfj;ksa ds la?k ;fn dsoy fu"d"kZ I vuqlj.k djrk gS
dFku fuos'k ls igys fuos'k lykgdkj ls }kjk bldk dM+k fojksèk fd;k x;k gS vkSj (b) if only conclusion II follows
ijke'kZ djus dh lykg nsrk gS] blfy, II dksbZ Hkh deZpkjh viuh vk; ?kksf"kr ugha djus ;fn dsoy fu"d"kZ II vuqlj.k djrk gS
vuqlj.k djrk gSA okyk gSA (c) if either I or II follows
6. Statement: Conclusions: ;fn ;k rks I ;k II vuqlj.k djrk gS
Although the education system has I. The employees of this company do not (d) if neither I nor II follows
progressed from the point of view of the seem to have any additional undisclosed ;fn u rks I vkSj u gh II vuqlj.k djrk gS
number of schools, most of them are ill- income besides their salary. (e) if both I and II follow
equipped and have not achieved bl daiuh ds deZpkfj;ksa dks vius osru ds ;fn I vkSj II nksuksa vuqlj.k djrs gSa
excellence in imparting education. vykok dksbZ vfrfjä v?kksf"kr vk; ugha Sol. (d)
;|fi f'k{kk ç.kkyh Ldwyksa dh la[;k ds – yxrh gSA From the commitment of the present
f"Vdks.k ls vkxs c<+h gS] ysfdu muesa ls II. The employees’ union wants all Government, it cannot be concluded that
vfèkdka'k iw.kZr;k lqlfTtr ugha gSa vkSj senior officers to declare their income former Government did not do anything
f'k{kk çnku djus esa mR—"Vrk gkfly ugha first. serious about the development nor does
dj ik, gSaA deZpkjh la?k pkgrk gS fd lHkh ofj"B it mean that present Government will
Conclusions: vfèkdkjh igys viuh vk; ?kksf"kr djsaA not do anything for the development of

335
Download Free PDFs & e-Books from Neon Classes App web. : www.neonclasses.com
other sectors, hence none of the lHkh panu ds o`{k iq"iu voLFkk esa gksrs gSaA ,d oxZ esa] rhu&pkSFkkbZ yM+ds QqVc‚y
conclusion follows. (c) At least one-half of the Ashoka trees [ksyrs gSa] ,d vkèkk fØdsV [ksyrs gSa]
orZeku ljdkj dh çfrc)rk ls] ;g fu"d"kZ are old. ,d&pkSFkkbZ tks fØdsV [ksyrs gSa] os QqVc‚y
ugha fudkyk tk ldrk gS fd iwoZ ljdkj us v'kksd ds isM+ksa esa ls de ls de ,d vkèkk ugha [ksyrs gSaA
fodkl ds ckjs esa dqN Hkh xaHkhj ugha fd;k iqjkuk gSA Conclusions : /fu"d"kZ
Fkk vkSj u gh bldk eryc gS fd orZeku (d) One-half of the sandal trees are at (a) Two-third of the boys play only
ljdkj vU; {ks=ksa ds fodkl ds fy, dqN Hkh the flowering stage. football.
ugha djsxh] blfy, dksbZ Hkh fu"d"kZ bl panu ds isM+ksa dk vkèkk fgLlk iq"iu voLFkk nks&frgkbZ yM+ds dsoy QqVc‚y [ksyrs gSaA
çdkj ugha gSA esa gksrk gSA (b) One-fourth of the boys play neither
9. Statement/dFku (e) None of these cricket nor football.
It is almost impossible to survive and buesa ls dksbZ ugha ,d&pkSFkkbZ yM+ds u rks fØdsV [ksyrs gSa
prosper in this world without sacrificing Given answer vkSj u gh QqVc‚yA
Ethics and Morality. (a) if only conclusion I follows (c) One-third of the boys play neither
vkpkj & fopkj vkSj uSfrdrk dk R;kx fd, ;fn dsoy fu"d"kZ I vuqlj.k djrk gS cricket nor football.
fcuk bl nqfu;k esa cus jgus vkSj le`) gksuk (b) if only conclusion II follows ,d frgkbZ yM+ds u rks fØdsV [ksyrs gSa vkSj
yxHkx vlaHko gSA ;fn dsoy fu"d"kZ II vuqlj.k djrk gS u gh QqVc‚yA
Conclusions/fu"d"kZ (c) if either I or II follows (d) One-eight of the boys play neither
I. World appreciates some Concepts ;fn ;k rks I ;k II vuqlj.k djrk gS cricket nor football.
but may not uphold it. (d) if neither I nor II follows ,d&vkB yM+ds u rks fØdsV [ksyrs gSa vkSj
nqfu;k dqN voèkkj.kkvksa dh ljkguk djrh gS ;fn u rks I vkSj u gh II vuqlj.k djrk gS u gh QqVc‚yA
ysfdu bls cjdjkj ugha j[k ldrhA (e) if both I and II follow (e) Two-fifth of the boys play only
II. Concept of Ethics and ;fn I vkSj II nksuksa vuqlj.k djrs gSa football.
Morality are not practicle in life. Sol. (e) nks&ik¡posa yM+ds dsoy QqVc‚y [ksyrs gSaA
vkpkj & fopkj vkSj uSfrdrk dh voèkkj.kk 11. Statement : /dFku Given answer
thou esa O;kogkfjd ugha gSA Many business offices are located in (a) if only conclusion I follows
Given answer buildings having two to eight floors. If a ;fn dsoy fu"d"kZ I vuqlj.k djrk gS
(a) if only conclusion I follows building has more than three floors, it (b) if only conclusion II follows
;fn dsoy fu"d"kZ I vuqlj.k djrk gS has a lift. ;fn dsoy fu"d"kZ II vuqlj.k djrk gS
(b) if only conclusion II follows dbZ O;kolkf;d dk;kZy; nks ls vkB eaftyksa (c) if either I or II follows
;fn dsoy fu"d"kZ II vuqlj.k djrk gS okys Hkouksa esa fLFkr gSaA ;fn fdlh bekjr esa ;fn ;k rks I ;k II vuqlj.k djrk gS
(c) if either I or II follows rhu ls vfèkd eaftysa gSa] rks mlesa ,d fy¶V (d) if neither I nor II follows
;fn ;k rks I ;k II vuqlj.k djrk gS gSA ;fn u rks I vkSj u gh II vuqlj.k djrk gS
(d) if neither I nor II follows Conclusions : /fu"d"kZ (e) if both I and II follow
;fn u rks I vkSj u gh II vuqlj.k djrk gS (a) All floors may be reached by lifts. ;fn I vkSj II nksuksa vuqlj.k djrs gSa
(e) if both I and II follow lHkh eaftyksa ij fy¶Vksa }kjk igqapk tk Sol. (d)
;fn I vkSj II nksuksa vuqlj.k djrs gSa ldrk gSA 13. Statement/dFku
Sol. (e) (b) Only floors above the third floor The nation ‘X’ faced the increased
Conclusions I and II convey almost the have lifts. international opposition due to its
same meaning that principles related to dsoy rhljh eafty ls Åij dh eaftyksa esa decision of performing eight nuclear
Ethics and Morality seem to be good but fy¶V gSA explosions.
are not practicable in real life. Hence, (c) Seventh floors has lift. jk"Vª ‘X’ dks vkB ijek.kq foLQksV djus ds
both the conclusions follow. lkroha eafty esa fy¶V gSaA vius fu.kZ; ds dkj.k c<+s gq, varjkZ"Vªh;
fu"d"kZ I vkSj II yxHkx ,d gh vFkZ nsrs gSa (d) Second floors do not have lifts. fojksèk dk lkeuk djuk iM+kA
fd vkpkj& fopkj vkSj uSfrdrk ls lacafèkr nwljh eaftyksa esa fy¶V ugha gSA Conclusions/fu"d"kZ
fl)kar vPNs yxrs gSa ysfdu okLrfod thou Given answer I. The citizens of the nation have
esa O;kogkfjd ugha gSaA blfy,] nksuksa fu"d"kZ (a) if only conclusion I follows favoured others. They may become
dk ikyu djrs gSaA ;fn dsoy fu"d"kZ I vuqlj.k djrk gS powerful.
10. Statement : /dFku (b) if only conclusion II follows jk"Vª ds ukxfjd nwljksa ds i{kèkj gSa vkSj os
A forest has as many sandal trees as it ;fn dsoy fu"d"kZ II vuqlj.k djrk gS 'kfä'kkyh cu ldrs gSaA
has Ashoka trees. Three-fourth of the (c) if either I or II follows II. Some powerful nations do not want
trees are old ones and half of the trees ;fn ;k rks I ;k II vuqlj.k djrk gS that others may become powerful.
are at the flowering stage. (d) if neither I nor II follows dqN 'kfä'kkyh jk"Vª ugha pkgrs gSa fd nwljs
,d taxy esa mrus gh panu ds isM+ gSa ftrus ;fn u rks I vkSj u gh II vuqlj.k djrk gS 'kfä'kkyh cu ldsaA
esa v'kksd ds isM+ gSaA rhu&pkSFkkbZ isM+ iqjkus gSa (e) if both I and II follow Given answer
vkSj vkèks isM+ iq"iu voLFkk esa gSaA ;fn I vkSj II nksuksa vuqlj.k djrs gSa (a) if only conclusion I follows
Conclusions : /fu"d"kZ Sol. (c) ;fn dsoy fu"d"kZ I vuqlj.k djrk gS
(a) All Ashoka trees are at the flowering 12. Statement : /dFku (b) if only conclusion II follows
stage. In a class, three-fourth of the boys play ;fn dsoy fu"d"kZ II vuqlj.k djrk gS
lHkh v'kksd ds isM+ iq"iu voLFkk esa gSaA football, one-half play cricket, one- (c) if either I or II follows
(b) All sandal trees are at the flowering fourth of those who play cricket do not ;fn ;k rks I ;k II vuqlj.k djrk gS
stage. play football. (d) if neither I nor II follows

336
Best App for Govt. Jobs : Neonclasses (Download Now) Neon Publications
;fn u rks I vkSj u gh II vuqlj.k djrk gS mUgksua s orZeku çf'k{k.k dk;ZØe dks vU; ;fn dsoy fu"d"kZ I vuqlj.k djrk gS
(e) if both I and II follow rjhdksa ls cnyus dh vko';drk ij tksj (b) if only conclusion II follows
;fn I vkSj II nksuksa vuqlj.k djrs gSa fn;k tks çcaèkdksa dh okLrfod ;ksX;rk dks ;fn dsoy fu"d"kZ II vuqlj.k djrk gS
Sol. (b) lkeus yk,xkA (c) if either I or II follows
Nuclear explosion is not the result of the Conclusions/fu"d"kZ ;fn ;k rks I ;k II vuqlj.k djrk gS
favour by the citizens. Hence, I. It is important to bring out the real (d) if neither I nor II follows
Conclusion I does not follow. However, merit of the managers. ;fn u rks I vkSj u gh II vuqlj.k djrk gS
increase in the international opposition çcaèkdksa dh okLrfod ;ksX;rk dks ckgj ykuk (e) if both I and II follow
is due to the fact that some powerful egRoiw.kZ gSA ;fn I vkSj II nksuksa vuqlj.k djrs gSa
nations don’t want that others may II. The present training programme does Sol. (e)
become powerful. not bring out the real merit of the Both the conclusions follow. The waver
ijek.kq foLQksV ukxfjdksa ds i{k dk ifj.kke managers. is incorporated because some of the
ugha gSA blfy,] fu"d"kZ I vuqlj.k ugha orZeku çf'k{k.k dk;ZØe çcaèkdksa dh students for MA will not have previous
djrk gSA gkyk¡fd] varjkZ"Vªh; fojksèk esa o`f) okLrfod ;ksX;rk dks lkeus ugha ykrk gSA experience and some of the students
bl rF; ds dkj.k gS fd dqN 'kfä'kkyh Given answer will have previous experience of social
jk"Vª ugha pkgrs gSa fd vU; jk"Vª 'kfä'kkyh (a) if only conclusion I follows work.
cu ldsAa ;fn dsoy fu"d"kZ I vuqlj.k djrk gS nksuksa fu"d"kZ dk ikyu djrs gSaA fiNys
14. Statement/dFku (b) if only conclusion II follows vuqHko dks ekQ djus 'kkfey fd;k x;k gS
The multinational fast food chains have ;fn dsoy fu"d"kZ II vuqlj.k djrk gS D;ksfa d MA ds fy, dqN Nk=ksa dks fiNys
started their operation in India after (c) if either I or II follows vuqHko ugha gksxk vkSj dqN Nk=ksa dks
facing many problems but association of ;fn ;k rks I ;k II vuqlj.k djrk gS lkekftd dk;Z dk fiNyk vuqHko gksxkA
farmers are ready for competition. (d) if neither I nor II follows 17. Statement/dFku
cgqjk"Vªh; QkLV QwM psu us dbZ leL;kvksa ;fn u rks I vkSj u gh II vuqlj.k djrk gS Animals live on oxygen.
dk lkeuk djus ds ckn Hkkjr esa viuk (e) if both I and II follow i'kq v‚Dlhtu ij jgrs gSaA
v‚ijs'ku 'kq: fd;k gS ysfdu fdlkuksa dk ;fn I vkSj II nksuksa vuqlj.k djrs gSa Conclusions/fu"d"kZ
la?k çfrLièkkZ ds fy, rS;kj gSA Sol. (e) I. Plants do not live on oxygen
Conclusions/fu"d"kZ Both the conclusions follow from the ikSèks v‚Dlhtu ij ugha jgrs gSa
I. Association of farmers are not statement. Conclusion I is the direct II. Anything that needs oxygen is bound
supporting modernization. result of the statement. Conclusion II to be animal.
fdlkuksa dk la?k vkèkqfudhdj.k dk leFkZu follows because in the given statement, ftl fdlh Hkh pht dks v‚Dlhtu dh
ugha dj jgk gSA there is a need to replace the present t:jr gksrh gS] og i'kq gksus dks ck/; gSA
II. Association of Farmers are not ready system with a new one. Given answer
for competition with the multinational dFku ls nksuksa fu"d"kZ fudyrs gSaA fu"d"kZ I (a) if only conclusion I follows
companies. dFku dk çR;{k ifj.kke gSA fu"d"kZ II bl ;fn dsoy fu"d"kZ I vuqlj.k djrk gS
fdlku la?k cgqjk"Vªh; daifu;ksa ds lkFk çdkj gS D;ksfa d fn, x, dFku esa] orZeku (b) if only conclusion II follows
çfrLièkkZ ds fy, rS;kj ugha gSaA ç.kkyh dks ,d ubZ iz.kkyh ds lkFk cnyus ;fn dsoy fu"d"kZ II vuqlj.k djrk gS
Given answer dh vko';drk gSA (c) if either I or II follows
(a) if only conclusion I follows 16. Statement/dFku ;fn ;k rks I ;k II vuqlj.k djrk gS
;fn dsoy fu"d"kZ I vuqlj.k djrk gS In case of the outstanding candidates, (d) if neither I nor II follows
(b) if only conclusion II follows the condition of previous experience of ;fn u rks I vkSj u gh II vuqlj.k djrk gS
;fn dsoy fu"d"kZ II vuqlj.k djrk gS social work may be waived by the (e) if both I and II follow
(c) if either I or II follows admission Committee for MA (social ;fn I vkSj II nksuksa vuqlj.k djrs gSa
;fn ;k rks I ;k II vuqlj.k djrk gS work). Sol. (d)
(d) if neither I nor II follows cgqr vPNs mEehnokj gksus dh fLFkfr esa] ,e, None of the conclusions follows.
;fn u rks I vkSj u gh II vuqlj.k djrk gS ¼lkekftd dk;Z½ ds fy, ços'k lfefr }kjk ‘Animals live on oxygen’ does not imply
(e) if both I and II follow lkekftd dk;Z ds fiNys vuqHko dh 'krZ dks that other things do not need oxygen.
;fn I vkSj II nksuksa vuqlj.k djrs gSa ekQ fd;k tk ldrk gSA fu"d"kZ dk dksbZ Hkh vuqlj.k ugha djrk gSA
Sol. (d) Conclusions/fu"d"kZ ^i'kq v‚Dlhtu ij jgrs gSa dk vFkZ ;g ugha
Conclusion I is totally unrelated to the I. Some of the students for MA (social gS fd vU; phtksa dks v‚Dlhtu dh
statement and Conclusion II is contrary work) will have previous experience of vko';drk ugha gSA
to the statement. Hence, none of the social work. 18. Statement/dFku
conclusions follows. ,e, ¼lkekftd dk;Z½ ds fy, dqN Nk=ksa dks Religions provide the means for
fu"d"kZ I dFku ds fy, iwjh rjg ls lkekftd dk;Z dk fiNyk vuqHko gksxkA attaining eternal peace. People should
vlacfa èkr gS vkSj fu"d"kZ II dFku ds foijhr II. Some of the students for MA (social follow these means.
gSA blfy,] dksbZ Hkh fu"d"kZ bl çdkj ugha work) will not have previous experience èkeZ 'kkÜor 'kkafr çkIr djus ds fy, lkèku
gSA of social work. çnku djrs gSaA yksxksa dks bu lkèkuksa dk
15. Statement/dFku ,e, ¼lkekftd dk;Z½ ds fy, dqN Nk=ksa dks ikyu djuk pkfg,A
He emphasized the need to replace the lkekftd dk;Z dk fiNyk vuqHko ugha Conclusions/fu"d"kZ
present training programme by other gksxkA I. Religions ensure prosperous life.
methods which will bring out the real Given answer èkeZ le`) thou lqfuf'pr djrs gSaA
merit of the managers. (a) if only conclusion I follows

337
Download Free PDFs & e-Books from Neon Classes App web. : www.neonclasses.com
II. Religions help people to eradicate fctyh dh njsa lLrh gks xbZ gSaA I. Only farmers are attempting suicides
poverty. Given answer due to poverty and debt.
èkeZ yksxksa dks xjhch mUewyu esa enn djrk (a) if only conclusion I follows xjhch vkSj dtZ ds dkj.k dsoy fdlku
gSA ;fn dsoy fu"d"kZ I vuqlj.k djrk gS vkRegR;k dk ç;kl dj jgs gSaA
Given answer (b) if only conclusion II follows II. No serious effort has been made till
(a) if only conclusion I follows ;fn dsoy fu"d"kZ II vuqlj.k djrk gS now for the upliftment of any section of
;fn dsoy fu"d"kZ I vuqlj.k djrk gS (c) if either I or II follows society.
(b) if only conclusion II follows ;fn ;k rks I ;k II vuqlj.k djrk gS lekt ds fdlh Hkh oxZ ds mRFkku ds fy,
;fn dsoy fu"d"kZ II vuqlj.k djrk gS (d) if neither I nor II follows vc rd dksbZ xaHkhj ç;kl ugha fd;k x;k
(c) if either I or II follows ;fn u rks I vkSj u gh II vuqlj.k djrk gS gSA
;fn ;k rks I ;k II vuqlj.k djrk gS (e) if both I and II follow Given answer
(d) if neither I nor II follows ;fn I vkSj II nksuksa vuqlj.k djrs gSa (a) if only conclusion I follows
;fn u rks I vkSj u gh II vuqlj.k djrk gS Sol. (d) ;fn dsoy fu"d"kZ I vuqlj.k djrk gS
(e) if both I and II follow Neither of the conclusions follows from (b) if only conclusion II follows
;fn I vkSj II nksuksa vuqlj.k djrs gSa the given statement. ;fn dsoy fu"d"kZ II vuqlj.k djrk gS
Sol. (d) fn, x, dFku esa ls dksbZ Hkh fu"d"kZ (c) if either I or II follows
None of the conclusions is correct. vuqlj.k ugha djrk gSA ;fn ;k rks I ;k II vuqlj.k djrk gS
dksbZ Hkh fu"d"kZ lgh ugha gSA 21. Statement/dFku (d) if neither I nor II follows
19. Statement/dFku Most Indians are aware that they have a ;fn u rks I vkSj u gh II vuqlj.k djrk gS
According to the education minister, the great heritage, but few would include (e) if both I and II follow
students will have the option of skipping science in it. ;fn I vkSj II nksuksa vuqlj.k djrs gSa
the board examination for class X from vfèkdka'k Hkkjrh; bl ckr ls voxr gSa fd Sol. (d)
this academic year to ease some muds ikl ,d egku fojklr gS] ysfdu dqN Both the conclusions are not related to
pressure on them. gh blesa foKku dks 'kkfey djsaxsA the statement. Hence, neither I nor II
f'k{kk ea=h ds vuqlkj] Nk=ksa ds ikl mu ij Conclusions/fu"d"kZ follows.
dqN ncko de djus ds fy, bl 'kS{kf.kd I. Many Indians consider science to have nksuksa fu"d"kZ dFku ls lacfa èkr ugha gSaA
o"kZ ls nloha d{kk ds fy, cksMZ ijh{kk NksM+us made Indian heritage great. blfy,] u rks I vkSj u gh II vuqlj.k djrk
dk fodYi gksxkA dbZ Hkkjrh; ekurs gS fd foKku us Hkkjrh; gSA
Conclusions/fu"d"kZ fojklr dks egku cuk;k gSA 23. Statement/dFku
I. Class X board examination was II. Many Indians are not aware that India Workers feel highly motivated when
compulsory till the last academic year. has a great scientific heritage. they get sense of involvement by
fiNys 'kS{kf.kd o"kZ rd d{kk nloha dh cksMZ cgqr ls Hkkjrh; bl ckr ls voxr ugha gSa participating in the management of
ijh{kk vfuok;Z FkhA fd Hkkjr ds ikl ,d egku oSKkfud companies.
II. Students were traumatized by the fojklr gSA daiuh izca/ku esa Hkkx ysus o Hkkxhnkjh dh
pressure created by these board exams. Given answer Hkkouk ls d;ZdrkZ vR;f/kd izsfjr eglwl
bu cksMZ ijh{kkvksa }kjk cuk, x, ncko ls (a) if only conclusion I follows djrs gSA
Nk= =Lr FksA ;fn dsoy fu"d"kZ I vuqlj.k djrk gS Conclusions/fu"d"kZ
Given answer (b) if only conclusion II follows I. Workers should be motivated to
(a) if only conclusion I follows ;fn dsoy fu"d"kZ II vuqlj.k djrk gS produce more.
;fn dsoy fu"d"kZ I vuqlj.k djrk gS (c) if either I or II follows Jfedksa dks vfèkd mRiknu djus ds fy,
(b) if only conclusion II follows ;fn ;k rks I ;k II vuqlj.k djrk gS çsfjr fd;k tkuk pkfg,A
;fn dsoy fu"d"kZ II vuqlj.k djrk gS (d) if neither I nor II follows II. Workers should be allowed to
(c) if either I or II follows ;fn u rks I vkSj u gh II vuqlj.k djrk gS participate in the management of
;fn ;k rks I ;k II vuqlj.k djrk gS (e) if both I and II follow companies.
(d) if neither I nor II follows ;fn I vkSj II nksuksa vuqlj.k djrs gSa Jfedksa dks vfèkd mRiknu djus ds fy,
;fn u rks I vkSj u gh II vuqlj.k djrk gS Sol. (b) dk;ZdrZkvksa dks daiuh izc/ku esa fglk ysus
(e) if both I and II follow From the given statement, only dh vuqefr nh tkuh pkfg,A
;fn I vkSj II nksuksa vuqlj.k djrs gSa Conclusion II follows. Given answer
Sol. (a) fn, x, dFku ls] dsoy fu"d"kZ II vuqlj.k (a) if only conclusion I follows
Only Conclusion II follows. djrk gSA ;fn dsoy fu"d"kZ I vuqlj.k djrk gS
dsoy fu"d"kZ II vuqlj.k djrk gSA 22. Statement/dFku (b) if only conclusion II follows
20. Statement/dFku Reacting to the news of suicides by the ;fn dsoy fu"d"kZ II vuqlj.k djrk gS
Power consumption in every family has farmers, the Prime Minister stated that (c) if either I or II follows
been doubled during the last 5 yr. his government will make every ;fn ;k rks I ;k II vuqlj.k djrk gS
gj ifjokj esa fctyh dh [kir fiNys 5 possible effort for the upliftment of poor (d) if neither I nor II follows
lky ds nkSjku nksxquh gks xbZ gSA farmers and farmhands. ;fn u rks I vkSj u gh II vuqlj.k djrk gS
Conclusions/fu"d"kZ fdlkuksa }kjk vkRegR;k dh [kcjksa ij (e) if both I and II follow
I. There is a lot of development in the çfrfØ;k nsrs gq,] çèkku ea=h us dgk fd ;fn I vkSj II nksuksa vuqlj.k djrs gSa
society. mudh ljdkj xjhc fdlkuksa vkSj fdlkuksa ds Sol. (b)
lekt esa cgqr fodkl gqvk gSA mRFkku ds fy, gj laHko ç;kl djsxhA Only Conclusion II follows. As the
II. Power rates have become cheaper. Conclusions/fu"d"kZ workers feel motivated when they get

338
Best App for Govt. Jobs : Neonclasses (Download Now) Neon Publications
sense of involvement by participating in ;fn u rks I vkSj u gh II vuqlj.k djrk gS (d) If neither I nor II is implicit/;fn u rks
the management of companies, so they (e) if both I and II follow I vkSj u gh II fufgr gS
should be allowed to participate. ;fn I vkSj II nksuksa vuqlj.k djrs gSa (e) If both I and II are implicit/;fn I vkSj
dsoy fu"d"kZ II vuqlj.k djrk gSA tSlk fd Sol. (e) II nksuksa fufgr gSa
Jfedksa dks rc çsj.kk feyrh gS] tc mUgsa Both the conclusions follow. Since the Sol. (e)
daifu;ksa ds çcaèku esa Hkkx ysus dh Hkkouk option is available from next year, it Both First and second follow.
feyrh gS] blfy, mUgsa Hkkx ysus dh vuqefr implies that this option was not I vkSj II nksuks vuqlj.k djrs gSA
nh tkuh pkfg,A available earlier. Secondly, students who 2. If it does not rain throughout this
24. Statement/dFku are weak in Mathematics and Science month, most farmers would be in
Power consumption in every family has will be benefited. trouble this year.
been doubled during the last five years. nksuksa fu"d"kZ dk ikyu djrs gSaA pwafd fodYi vxj bl iwjs eghus ckfj'k ugha gksrh gS] rks
gj ifjokj esa fctyh dh [kir fiNys ikap vxys o"kZ ls miyCèk gS] bldk eryc gS fd bl lky T;knkrj fdlku eqf'dy esa iM+
o"kksaZ ds nkSjku nksxquh gks xbZ gSA ;g fodYi igys miyCèk ugha FkkA nwljk] tk,axsA
Conclusions/fu"d"kZ xf.kr vkSj foKku esa detksj Nk= ykHkkfUor I. Timely rain is essential for farming.
I. There is a lot of development in the gksx
a sA [ksrh ds fy, le; ij ckfj'k t:jh gSA
society.
lekt esa cgqr fodkl gqvk gSA
Statement and II. Most of the farmers are generally
dependent on rains.
II. Power rates have become cheaper. Assumption T;knkrj fdlku vkerkSj ij ckfj'k ij
fctyh dh njsa lLrh gks xbZ gSaA Definition → If before saying anything fuHkZj gSaA
Given answer someone keeps that thing in mind. That (a) If only Assumption I is implicit/;fn
(a) if only conclusion I follows idea is called assumption it means dsoy vuqeku I fufgr gS
;fn dsoy fu"d"kZ I vuqlj.k djrk gS assumption is a hidden fact on the basis (b) If only Assumption II is implicit/;fn
(b) if only conclusion II follows of which something is said or spoken. dsoy vuqeku II fufgr gS
;fn dsoy fu"d"kZ II vuqlj.k djrk gS ifjHkk"kk → vxj dksbZ O;fDr dqN dgus ls (c) If either I or II is implicit/;fn ;k rks I
(c) if either I or II follows igys ml ckr dks è;ku esa j[krk gS rks ml ;k II fufgr gS
;fn ;k rks I ;k II vuqlj.k djrk gS fopkj dks iwoZ èkkj.kk dgk tkrk gS bldk (d) If neither I nor II is implicit/;fn u rks
(d) if neither I nor II follows I vkSj u gh II fufgr gS
eryc gS fd èkkj.kk ,d fNik gqvk rF; gS
;fn u rks I vkSj u gh II vuqlj.k djrk gS (e) If both I and II are implicit/;fn I vkSj
ftlds vkèkkj ij dqN dgk ;k cksyk tkrk
(e) if both I and II follow II nksuksa fufgr gSa
gSA
;fn I vkSj II nksuksa vuqlj.k djrs gSa Sol. (e)
Directions (1–2) In each of the
Sol. (d) Since the statement speaks of the
questions below, is given a statement
Neither of the conclusions follows from essentiality and requirement of rain for
followed by two assumptions numbered
the given statement. farmers, hence both the assumptions
I and II. An assumption is something
fn, x, dFku esa ls dksbZ Hkh fu"d"kZ are implicit in the statement.
supposed or taken for granted. You have
vuqlj.k ugha djrk gSA to consider the statement and the pwafd dFku fdlkuksa ds fy, ckfj'k dh
25. Statement/dFku assumptions and decide which of the vfuok;Zrk vkSj vko';drk dh ckr djrk gS]
From the next academic year, students assumptions (s) is/are implicit in the blfy, dFku esa nksuksa èkkj.kk,a fufgr gSaA
will have the option of dropping statement. Directions : In the questions has a
Mathematics and Science for their uhps fn, x, çR;sd ç'u esa] ,d dFku fn;k statement followed by two assumptions
school leaving certificate examination. x;k gS] ftlds ckn I vkSj II dh nks èkkj.kk,¡ numbered I and II. An assumption is
vxys 'kS{kf.kd o"kZ ls] Nk=ksa ds ikl vius gSa rFkk fcuk izek.k ds lgh eku ysuk gSA something supposed or taken for
Ldwy NksM+us ds çek.k i= ijh{kk ds fy, vkidks dFku vkSj ekU;rkvksa ij fopkj granted. Consider the statement and the
xf.kr vkSj foKku NksM+us dk fodYi gksxkA djuk gS vkSj fu.kZ; ysuk gS fd dFku esa following assumptions.
Conclusions/fu"d"kZ funs’Z k : fuEu ç'u esa ,d dFku gS ftlds
dkSu lh èkkj.kk,¡ fufgr gSaA
I. Students, who are weak in Science and ckn I vkSj II dh nks èkkj.kk,¡ gSaA ,d èkkj.kk
1. Please do not lean out of the running
Mathematics, will be benefitted. dqN ekuk tkrk gS ;k fcuk izek.k ds lgh
bus, a notice in a tourist bus.
tks Nk= foKku vkSj xf.kr esa detksj gSa] os i;ZVd cl esa ,d lwpukA —i;k pyrh cl eku ysuk gSA dFku vkSj fuEufyf[kr
ykHkkfUor gksx
a sA ls ckgj u fudysa] ekU;rkvksa ij fopkj djsAa
II. Earlier students did not have the 3. It is felt that when the airline is facing
I. Leaning out of running bus is
choice of continuing their education stiff competition coupled with a
dangerous.
without leaving these subjects. precarious financial position, the top
pyrh cl ls ckgj fudyuk [krjukd gSA
igys Nk=ksa ds ikl bu fo"k;ksa dks fy, fcuk level posts should be kept open for
II. The passengers are likely to pay
viuh f'k{kk tkjh j[kus dk fodYi ugha FkkA attention to this notice. outside professionals rather than
Given answer ;k=h lEHkor;k bl uksfVl ij è;ku nsxAsa internal candidates.
(a) if only conclusion I follows ;g eglwl fd;k tkrk gS fd tc ,;jykbu
(a) If only Assumption I is implicit/;fn
;fn dsoy fu"d"kZ I vuqlj.k djrk gS dks vfuf'pr foÙkh; fLFkfr ds lkFk dM+h
dsoy vuqeku I fufgr gS
(b) if only conclusion II follows çfrLièkkZ dk lkeuk djuk iM+ jgk gS] rks
(b) If only Assumption II is implicit/;fn
;fn dsoy fu"d"kZ II vuqlj.k djrk gS vkarfjd mEehnokjksa ds ctk; 'kh"kZ Lrj ds
dsoy vuqeku II fufgr gS
(c) if either I or II follows inksa dks ckgjh is'ksojksa ds fy, [kqyk j[kk
(c) If either I or II is implicit/;fn ;k rks I
;fn ;k rks I ;k II vuqlj.k djrk gS tkuk pkfg,A
;k II fufgr gS
(d) if neither I nor II follows

339
Download Free PDFs & e-Books from Neon Classes App web. : www.neonclasses.com
I. Internal candidates aspire only for (a) If only Assumption I is implicit/;fn Directions (6–8) In each of the
getting promotions, without much dsoy vuqeku I fufgr gS question below, is given a statement
contribution. (b)If only Assumption II is implicit/;fn followed by two assumptions numbered
vkarfjd mEehnokj dsoy cgqr vfèkd dsoy vuqeku II fufgr gS I and II. An assumption is something
;ksxnku ds fcuk] inksUufr ikus dh vkdka{kk (c) If either I or II is implicit/;fn ;k rks I supposed or taken for granted. You have
j[krs gSaA ;k II fufgr gS to consider the statement and the
II. Experienced professionals are more (d) If neither I nor II is implicit/;fn u rks assumptions and decide which of the
likely to handle the problems of the assumptions (s) is/are implicit in the
I vkSj u gh II fufgr gS
airline. statement.
(e) If both I and II are implicit/;fn I vkSj
vuqHkoh is'ksojksa dks ,;jykbu dh leL;kvksa uhps fn, x, çR;sd ç'u esa] ,d dFku fn;k
II nksuksa fufgr gSa
dks laHkkyus dh vfèkd t:jr gSA x;k gS] ftlds ckn I vkSj II dh nks èkkj.kk,¡
Sol. (d)
(a) If only Assumption I is implicit/;fn gSaA vkidks dFku vkSj ekU;rkvksa ij fopkj
Assumption I is invalid as the
dsoy vuqeku I fufgr gSA assumption is that the students have
djuk gS vkSj fu.kZ; ysuk gS fd dFku esa
(b) If only Assumption II is implicit access to the internet. But it is not dkSu lh èkkj.kk,¡ fufgr gSaA
;fn dsoy vuqeku II fufgr gS necessary that they have this access at 6. “To keep myself up-to-date, I always
(c) If either I or II is implicit/;fn ;k rks I home. Further, past practice may not listen to prime time news on television”,
;k II fufgr gS have been borne in mind while A candidate tells the interview board.
(d) If neither I nor II is implicit/;fn u rks switching over to internet only display. "[kqn dks vi&Vw&MsV j[kus ds fy,] eSa ges'kk
I vkSj u gh II fufgr gS Hence, Assumption II is also invalid. Vsyhfotu ij çkbe Vkbe lekpkj lqurk gwa"]
(e) If both I and II are implicit/;fn I vkSj ekU;rk I fufgr ugh gS] D;ksfa d ;g èkkj.kk gS ,d mEehnokj lk{kkRdkj cksMZ dks crkrk gSA
II nksuksa fufgr gSa fd Nk=ksa dh baVjusV rd igqap gSA ysfdu I. The candidate does not read
Sol. (b) ;g t:jh ugha gS fd ?kj esa mudh igqpa gksA newspaper.
Experienced professionals are more blds vykok] fiNys vH;kl dks dsoy mEehnokj v[kckj ugha i<+rk gSA
likely to handle the professional hurdles. baVjusV ij cnyko djrs le; è;ku esa ugha II. Recent news are broadcast only on
vuqHkoh is'ksojksa dks is'ksoj ckèkkvksa dks j[kk x;k gSA blfy,] eku II Hkh vekU; gSA television.
laHkkyus dh vfèkd laHkkouk gSA 5. Give adequate job related training to the
gky dh [kcjsa dsoy Vsyhfotu ij çlkfjr
Directions (4–5) In each question employees before assigning them full- dh tkrh gSaA
below is given a statement followed by fledged work. (a) If only Assumption I is implicit/;fn
two assumptions numbered I and II. An deZpkfj;ksa dks iw.kZ dk;Z lkSaius ls igys mUgsa dsoy vuqeku I fufgr gS
assumption is something supposed or nh xbZ ukSdjh ls lacafèkr çf'k{k.k nsAa (b) If only Assumption II is implicit/;fn
taken for granted. You have to consider I. Training helps in boosting the dsoy vuqekuII fufgr gS
the statement and the following performance of employees. (c) If either Assumption I or II is
assumptions and decide which of the çf'k{k.k deZpkfj;ksa ds çn'kZu dks c<+kus esa implicit/
assumptions (s) is/are implicit in the enn djrk gSA ;fn ;k rks vuqeku I ;k II fufgr gS
statement. II. Employees have no skill sets before (d) If neither Assumption I nor II is
uhps fn, x, çR;sd ç'u esa ,d dFku fn;k training is provided to them. implicit
x;k gS ftlds ckn I vkSj II dh nks èkkj.kk,¡ çf'k{k.k çnku djus ls igys deZpkfj;ksa ds ;fn u rks vuqeku I vkSj u gh II fufgr gS
gSaA ;nh èkkj.kk esa dqN ekuk tkrk gS tks ikl dksbZ dkS'ky ugha gSA (e) If both Assumption I and II are
dFku esa fn;k x;k gS vkidks dFku vkSj (a) If only Assumption I is implicit/;fn implicit
fuEufyf[kr ekU;rkvksa ij fopkj djuk gksxk dsoy vuqeku I fufgr gS ;fn I vkSj II nksuks fufgr gSa
vkSj ;g r; djuk gksxk fd dFku esa dkSu (b)If only Assumption II is implicit/;fn Sol. (d)
lh èkkj.kk,¡ fufgr gSaA Assumption I is not implicit as the
dsoy vuqeku II fufgr gS
4. A leading university has begun a candidates listening to news on the
(c) If either I or II is implicit/;fn ;k rks I
practice of displaying results only on the television does not mean that he does
;k II fufgr gS
Internet rather than on the main notice not read newspaper. Assumption II is
(d) If neither I nor II is implicit/;fn u rks
boards. also invalid because of the word ‘only’.
I vkSj u gh II fufgr gS The word ‘only’ disconnect Assumption
,d çeq[k foÜofo|ky; us eq[; lwpuk iêksa (e) If both I and II are implicit/;fn I vkSj
ds ctk; dsoy baVjusV ij ifj.kke çnf'kZr II from he given statement.
II nksuksa fufgr gSa vuqeku gS fd I varfuZfgr ugha gSD;ksfa d
djuk 'kq: fd;k gSA
Sol. (a) Vsyhfotu ij lekpkj lquus okys mEehnokjksa
I. All the students enrolled with the No doubts, Assumption I is implicit in
university have access to Internet at dk eryc ;g ugha gS fd og v[kckj ugha
the need for training but what happens i<+rs gSaA ^dsoy^ 'kCn ds dkj.k eku II Hkh
home.
in case II of it? It takes things to an
foÜofo|ky; esa ukekafdr lHkh Nk=ksa ds ikl vekU; gSA 'kCn ^dsoy^ mUgksua s fn, x, dFku
extreme with the phrase “no skill sets”.
?kj ij baVjusV rd igqap gSA ls eku fy;k gSA
Hence Assumption II is an invalid
II. Most of the students referred to the 7. Government should deploy army to
assumption.
result displayed on both the internet as rehabilitate the people displaced due to
dksbZ lansg ugha gS] ekU;rk I çf'k{k.k dh
well as the notice boards earlier. the earthquake.
vko';drk esa fufgr gS ysfdu ekU;rk II
vfèkdka'k Nk=ksa us baVjusV ij vkSj lkFk gh Hkwdai ds dkj.k foLFkkfir gq, yksxksa ds
ekeys esa D;k gksrk gS ;g dksbZ ^dkS'ky ugh^ iquokZl ds fy, ljdkj dks lsuk rSukr djuh
uksfVl cksMksaZ ij çnf'kZr ifj.kke dks lanfHkZr okD;k’ka bls xyr cukrk gSA blfy, vuqeku
fd;kA pkfg,A
II ,d vekU; èkkj.kk gSA

340
Best App for Govt. Jobs : Neonclasses (Download Now) Neon Publications
I. Army can be used for purposes other ;fn u rks vuqeku I vkSj u gh II fufgr gS (e) If both Assumption I and II are
than war also. (e) If both Assumption I and II are implicit
lsuk dk bLrseky ;q) ds vykok vU; mís';ksa implicit ;fn eku fy;k tk, rks I vkSj II fufgr gSa
ds fy, Hkh fd;k tk ldrk gSA ;fn I vkSj II nksuks fufgr gSa Sol. (a)
II. Only army can rehabilitate the Sol. (e) Boards have been put with the
displaced victims of earthquake. It is clear that appointment of faculty assumption that it will get desired
dsoy lsuk gh Hkwdai ls foLFkkfir ihfM+rksa dk required funds, that is why it has been response from the motorists. Hence,
iquokZl dj ldrh gSA stated that no budgetary provisions will Assumption I is implicit. Assumption II
(a) If only Assumption I is implicit/;fn be made. Secondly, budget has been is not implicit because it neglates the
dsoy vuqeku I fufgr gS restricted to be invested in other areas. desired response from the boards put
(b) If only Assumption II is implicit/;fn Hence, both the assumption are implicit. up.
dsoy vuqekuII fufgr gS ;g Li"V gS fd ladk; ds fy, vko';d èku cksMksaZ dks bl èkkj.kk ds lkFk j[kk x;k gS fd
(c) If either Assumption I or II is dh t:jr gS] blhfy, ;g dgk x;k gS fd bls eksVj pkydksa ls okafNr çfrfØ;k
implicit/ dksbZ ctVh; çkoèkku ugha fd, tk,axsA nwljs] feysxhA blfy,] vuqeku gS fd I fufgr gSA
;fn ;k rks vuqeku I ;k II fufgr gS ctV dks vU; {ks=ksa esa fuos'k djus ds fy, vuqeku II fufgr ugha gS D;ksfa d ;g yxk,
(d) If neither Assumption I nor II is çfrcafèkr fd;k x;k gSA blfy,] nksuksa èkkj.kk x, cksMksaZ ls okafNr çfrfØ;k dks udkjrk gSA
implicit fufgr gSaA 10. The municipal corporation advised all
;fn u rks vuqeku I vkSj u gh II fufgr gS Directions (9–11) In each question the people living in the shanties along
(e) If both Assumption I and II are below, is given a statement is given the beaches to move to higher places
implicit followed by two assumptions numbered during monsoon.
;fn I vkSj II nksuks fufgr gSa I and II. An assumption is something uxj fuxe us leqæ rVksa ds fdukjs >ksifM+;ks
Sol. (a) supposed or taken for granted. You have esa cls ?kkVksa esa jgus okys lHkh yksxksa dks
It is given in the statement that the to consider the statement and the ekulwu ds nkSjku Åaps LFkkuksa ij tkus dh
Government should deploy army to following assumptions and decide which lykg nhA
rehabilitate people which means that of assumption (s) is/are implicit in the I. Many people living in the shanties may
army can be used for purposes other statement. leave the city and relocate themselves
than war. Hence, Assumption I implicit. uhps fn, x, çR;sd ç'u esa] ,d dFku fn;k elsewhere in the state.
The word ‘only’ makes the assumption II x;k gS] ftlds ckn I vkSj II dh nks èkkj.kk,¡ >ksia fM+;ksa esa jgus okys dbZ yksx 'kgj NksM+
baseless. nh xbZ gSaA ,d èkkj.kk dqN ekuk tkrk gS ;k ldrs gSa vkSj jkT; esa dgha vkSj [kqn dks
;g dFku esa fn;k x;k gS fd ljdkj dks ,sls nh xbZ gSA vkidks dFku vkSj fuEufyf[kr LFkkukarfjr dj ldrs gSaA
yksxksa ds iquokZl ds fy, rSukr fd;k tkuk ekU;rkvksa ij fopkj djuk gksxk vkSj ;g II. Majority of the people living in the
pkfg,] ftldk vFkZ gS fd lsuk dk bLrseky r; djuk gksxk fd dFku esa dkSu lh èkkj.kk shanties along the beach may try to
;q) ds vykok vU; mís';ksa ds fy, fd;k tk gSA relocate to higher places during
ldrk gSA blfy,] vuqeku I fufgr gSA 'kCn 9. The highway police authority put up monsoon.
^dsoy^ èkkj.kk II dks vkèkkjghu cukrk gSA large boards at regular intervals leqæ rV ds fdukjs dh ?kkfV;ksa esa jgus okys
8. No budgetary provision for the purpose indicating the speed limit and dangers of vfèkdka'k yksx ekulwu ds nkSjku mPp LFkkuksa
of appointing additional faculty would over speeding on the highway. ij LFkkukarfjr gksus dk ç;kl dj ldrs gSaA
be made in the context of Institute’s jktekxZ iqfyl çkfèkdj.k fu;fer varjky ij (a) If only Assumption I is implicit/;fn
changed financial priorities. cM+s cksMZ yxkrk gS tks jktekxZ ij xfr dh dsoy vuqeku I fufgr gS
laLFkku dh cnyh gqbZ foÙkh; çkFkfedrkvksa ds lhek vkSj [krjksa dks n'kkZrk gSA (b)If only Assumption II is implicit/;fn
lanHkZ esa vfrfjä ladk; fu;qä djus ds I. Most of the motorists may drive their dsoy vuqeku II fufgr gS
mís'; ls dksbZ ctVh; çkoèkku ugha fd;k vehicles within the speed limit on the (c) If either Assumption I or II is implicit
tk,xkA highways. ;fn ;k rks vuqeku I ;k II fufgr gS
I. Appointment of faculty requires funds. vfèkdka'k eksVj pkyd vius okguksa dks (d) If neither Assumption I nor II is
ladk; dh fu;qfä ds fy, èku dh jktekxksZa ij xfr lhek ds Hkhrj pyk ldrs implicit
vko';drk gksrh gSA gSaA ;fn u rks vuqeku I vkSj u gh II fufgr gS
II. There are areas other than II. Motorists generally ignore such (e) If both Assumption I and II are
appointment of faculty which require cautions and over speed on the highway. implicit
more financial attention. eksVj pkyd vkerkSj ij bl rjg dh ;fn eku fy;k tk, rks I vkSj II fufgr gSa
ladk; dh fu;qfä ds vykok vU; {ks= gSa lkoèkkfu;ksa dh vuns[kh djrs gSa vkSj jktekxZ Sol. (b)
ftUgsa vfèkd foÙkh; è;ku nsus dh vko';drk ij xfr c<+krs gSaA The corporation has not advised the
gSA (a) If only Assumption I is implicit/;fn people to leave the city. Hence, I is not
(a) If only Assumption I is implicit/;fn dsoy vuqeku I fufgr gS implicit. II is implicit as any advice is
dsoy vuqeku I fufgr gS (b)If only Assumption II is implicit/;fn given with the assumption that it will be
(b) If only Assumption II is implicit/;fn dsoy vuqeku II fufgr gS listened to and then listeners will do
dsoy vuqekuII fufgr gS (c) If either Assumption I or II is implicit what the advice suggests.
(c) If either Assumption I or II is ;fn ;k rks vuqeku I ;k II fufgr gS fuxe us yksxksa dks 'kgj NksM+us dh lykg
implicit/ (d) If neither Assumption I nor II is ugha nh gSA blfy,] I fufgr ugha gSA II
;fn ;k rks vuqeku I ;k II fufgr gS implicit fufgr gS D;ksfa d fdlh Hkh lykg dks bl
(d) If neither Assumption I nor II is ;fn u rks vuqeku I vkSj u gh II fufgr gS èkkj.kk ds lkFk fn;k tkrk gS fd bls lquk
implicit

341
Download Free PDFs & e-Books from Neon Classes App web. : www.neonclasses.com
tk,xk vkSj fQj Jksrk ogh djsaxs tks lykg çnku djsxk] mu yksxksa dks NksM+dj ftUgsa II. The present office staff is inefficient.
nsrs gSaA ;k=k HkÙkk çnku fd;k x;k gSA orZeku dk;kZy; deZpkjh v{ke gSA
11. Government has decided to relocate all I. Most of the employees will travel by (a) if only Assumption I is implicit
the factories from the city with the office transport. vxj dsoy vuqeku I fufgr gS
immediate effect to reduce pollution. vfèkdka'k deZpkjh dk;kZy; ifjogu }kjk (b) if only Assumption II is implicit
ljdkj us çnw"k.k dks de djus ds fy, 'kgj ;k=k djsaxsA vxj dsoy vuqeku II fufgr gS
ds lHkh dkj[kkuksa dks rRdky çHkko ls II. Those who are provided with (c) if either Assumption I or II is implicit
LFkkukarfjr djus dk fu.kZ; fy;k gSA travelling allowance will not read such ;fn ;k rks vuqeku I ;k II fufgr gS
I. Pollution in the city is being caused notice. (d) if neither Assumption I nor II is
only because of the factories existing ftUgsa ;k=k HkÙkk çnku fd;k tkrk gS] os bl implicit
there. rjg ds uksfVl dks ugha i<+saxsA ;fn u rks vuqeku I vkSj u gh II fufgr gS
'kgj esa çnw"k.k dsoy ogka ekStwn dkj[kkuksa (a) if only assumption I is implicit (e) if both Assumptions I and II are
dh otg ls gks jgk gSA ;fn dsoy èkkj.kk I fufgr gS implicit
II. People may be able to manage (b) if only assumption II is implicit ;fn nksuksa vuqeku I vkSj II fufgr gSa
travelling daily to the relocated ;fn dsoy èkkj.kk II fufgr gS Sol. (a)
factories. (c) if neither Assumption I nor II implicit Here, the manager stresses the need to
yksx LFkkukarfjr dkj[kkuksa esa nSfud ;k=k ;fn u rks vuqeku I vkSj u gh II fufgr gS appoint more subordinate officers. It
djus esa l{ke gks ldrs gSaA (d) if both assumption I and II are does not mean that present staff is not
(a) If only Assumption I is implicit/;fn implicit efficient. In fact, he wants to say that
dsoy vuqeku I fufgr gS ;fn nksuksa èkkj.kk I vkSj II fufgr gSa work in much more than the working
(b)If only Assumption II is implicit/;fn Sol. (a) capacity of present staff strength. Hence,
dsoy vuqeku II fufgr gS I is implicit as any notification is given Assumption II is invalid. But Assumption
(c) If either Assumption I or II is implicit with the assumption that targeted I is implicit because without assuming
;fn ;k rks vuqeku I ;k II fufgr gS people will respond according to the the availability of subordinate officers,
want of such the notification. II is not the manager would not have said so.
(d) If neither Assumption I nor II is
implicit implicit as notification has been given ;gka] çcaèkd vfèkd vèkhuLFk vfèkdkfj;ksa dks
;fn u rks vuqeku I vkSj u gh II fufgr gS for all employees and hence all are fu;qä djus dh vko';drk ij cy nsrk gSA
(e) If both Assumption I and II are expected to read it. bldk eryc ;g ugh gS fd orZeku deZpkjh
implicit ekU;rk I fufgr ugh gS] fdlh Hkh vfèklwpuk dq'ky ugha gSA okLro es]a og dguk pkgrk gS
;fn eku fy;k tk, rks I vkSj II fufgr gSa dks bl èkkj.kk ds lkFk fn;k x;k gS fd r; fd orZeku deZpkfj;ksa dh dk;Z {kerk dh
Sol. (c) fd;s x;s yksx bl rjg dh vfèklwpuk ds rqyuk esa cgqr vfèkd gSA blfy,] eku fy;k
Assumption I is invalid because of the vuqlkj tokc nsaxsA II fufgr ugha gS D;ksfa d x;k gS fd f}rh; vekU; gSA ysfdu ;g ekuk
world ‘only’. Assumption II is implicit as lHkh deZpkfj;ksa ds fy, vfèklwpuk nh xbZ gS tkrk gS fd I vèkhu gS D;ksfa d vèkhuLFk
without considering this factor the vkSj blfy, lHkh ls bls i<+us dh mEehn dh vfèkdkfj;ksa dh miyCèkrk ds fcuk] çcaèkd us
relocation of factories would not make tkrh gSA ,slk ugha dgk gksxkA
sense. Directions (13–14): In each question 14. Statement/dFku
ekU;rk I dsoy 'kCn ds iz;ksx dkj.k vekU; below is given a Statement followed by Even after retirement from international
gSA ekU;rk II fufgr gS D;ksfa d bl rF; ij two assumptions numbered I and II. An cricket, the ‘Rawalpindi express’ is in
fopkj fd, fcuk dkj[kkuksa ds LFkkukarj.k dk assumption is something supposed or the controversy again. This time he
dksbZ eryc ugha gksxkA taken for granted. You have to consider targeted Sachin Tendulkar through his
Directions (12): Each of these the Statement and the following book ‘Controversially your’, a news
questions, given below consists of a assumption and decide which of the about Shoaib Akhtar.
statement followed by two assumptions assumption(s) is/are implicit in the varjkZ"Vªh; fØdsV ls lsokfuo`fÙk ds ckn Hkh]
numbered as I and II. Decide which of Statement. 'jkoyfiaMh ,Dlçsl' fQj ls fooknksa esa gSA
the assumption (s) is/ are implicit from uhps fn, x, çR;sd ç'u esa ,d dFku fn;k bl ckj mUgksua s 'kks,c v[rj ds ckjs esa ,d
the statement and then decide which of x;k gS ftlds ckn I vkSj II dh nks èkkj.kk,¡ lekpkj] lfpu rsna qydj ij viuh iqLrd
the answer is correct. gSaA èkkj.kk dks dqN ekuk tkrk gS tks dFku ‘Controversially your’ ds ekè;e ls fu'kkuk
uhps fn, x, bu ç'uksa esa ls çR;sd esa I vkSj ds vk/kkj ij gksrh gSA vkidks dFku vkSj lkèkkA
II ds :i esa fxus x, nks vuqekuksa ds ckn fuEufyf[kr èkkj.kk ij fopkj djuk gS vkSj Assumptions/ekU;rk,a
,d dFku 'kkfey gSA ;g r; djsa fd dkSu ;g fu.kZ; djuk gS fd dFku esa dkSu lh I. Shoaib Akhtar does not play test
lh èkkj.kk dFku ls fudyh xbZ gS vkSj fQj èkkj.kk fufgr gSA cricket now.
r; djsa fd mÙkj esa ls dkSu lk lgh gSA 13. Statement/dFku 'kks,c v[rj vc VsLV fØdsV ugha [ksyrs gSaA
12. All the employees are notified that the “We must appoint more subordinate II. One can target anybody through one’s
organization will provide transport officers on our office staff”, the manager write up.
facilities at half the cost from the nearby said to the chairman. O;fä fdlh ij Hkh fy[kus ds ekè;e ls
railway station to the office except for "gesa vius dk;kZy; ds deZpkfj;ksa ij vfèkd fu'kkuk cuk ldrk gSA
those who have been provided with vèkhuLFk vfèkdkfj;ksa dh fu;qfä djuh (a) if only Assumption I is implicit
travelling allowance. pkfg,"] çcaèkd us vè;{k ls dgkA vxj dsoy vuqeku I fufgr gS
lHkh deZpkfj;ksa dks lwfpr fd;k tkrk gS fd Assumptions/ekU;rk,a (b) if only Assumption II is implicit
laxBu vkl&ikl ds jsyos LVs'ku ls I. Subordinate officers are available. vxj dsoy vuqeku II fufgr gS
dk;kZy; rd vkèkh ykxr ij ifjogu lqfoèkk vèkhuLFk vfèkdkjh miyCèk gSaA (c) if either Assumption I or II is implicit

342
Best App for Govt. Jobs : Neonclasses (Download Now) Neon Publications
;fn ;k rks vuqeku I ;k II fufgr gS Hkkjr esa czkaMksa dk çcaèku Bhd ls ugha fd;k fLFkfr rukoiw.kZ gS] bldk eryc gS fd dqN
(d) if neither Assumption I nor II is tkrk gSA xaHkhj ?kVuk gqbZ gSA blfy,] vuqeku gS fd
implicit III. The Media is always very positive I fufgr gSA nwljk] ;g fn;k tkrk gS fd
;fn u rks vuqeku I vkSj u gh II fufgr gS towards Brands fLFkfr vHkh Hkh rukoiw.kZ cuh gqbZ gS vkSj
(e) if both Assumptions I and II are ehfM;k ges'kk czkaM~l ds çfr cgqr yksxksa ls ?kj ds vanj jgus dk vuqjksèk fd;k
implicit ldkjkRed gS x;k gSA blfy,] vuqeku II fufgr gSA
;fn nksuksa vuqeku I vkSj II fufgr gSa (a) Only I/dsoy I èkkj.kk III xyr gS vkSj fn, x, dFku ls
Sol. (e) (b) Both I and II/I vkSj II nksuksa bldk dksbZ lacaèk ugha gSA blfy, III ,d
‘Retirement from international cricket’ (c) Both I and III/I vkSj III nksuksa vekU; èkkj.kk gSA
means Shoaib does not play test cricket (d) I, II and III/I, II vkSj III Directions (18): In the questions
now as test cricket is a part of Sol. (a) below, is given a statement followed by
international cricket. Hence, Only Assumption I is implicit three assumptions numbered I, II and
Assumption I is implicit. Assumption II dsoy vuqeku I fufgr gS III. An assumption is something
is also implicit because it is clearly said Directions (17): In question below, is supposed or taken for granted, you have
that Shoaib targeted Tendukar through given a Statement followed by three to consider the statement and the
his book and a book is a write up. assumptions numbered I, II and III. An assumptions and decide which of the
‘Retirement from international cricket’ assumption is something supposed or assumption(s) is/are implicit in the
dk eryc 'kks,c vc VsLV fØdsV ugha taken for granted, you have to consider statement.
[ksyrk gS D;ksfa d VsLV fØdsV varjjk"Vªh; the Statement and the assumptions and uhps fn, x, ç'u es]a I] II vkSj III fxus x,
fØdsV dk ,d fgLlk gSA blfy,] vuqeku decide which of the assumption(s) rhu vuqekuksa ds ckn ,d dFku fn;k x;k
gS fd I fufgr gSA vuqeku II Hkh fufgr gS is/are implicit in the Statement. gSA ,d èkkj.kk dqN ekuk tkrk gS ;k nh xbZ
D;ksfa d ;g Li"V :i ls dgk tkrk gS fd fn, x, çR;sd ç'u esa] I] II vkSj III fxus gS] vkidks dFku vkSj ekU;rkvksa ij fopkj
'kks,c us viuh iqLrd ds ekè;e ls rsnqydj x, rhu vuqekuksa ds ckn ,d dFku fn;k djuk gksxk vkSj ;g r; djuk gksxk fd
dks fu’kkuk fd;k vkSj ,d iqLrd ,d ys[ku x;k gSA ,d èkkj.kk dqN ekuk tkrk gS ;k dFku esa dkSu lh èkkj.kk fufgr gS @ gSaA
gSA nh xbZ gS] vkidks dFku vkSj ekU;rkvksa ij 18. Statement/dFku
Directions (16): In the following fopkj djuk gksxk vkSj ;g r; djuk gksxk “If you have obtained 75% or more
question, a Statement is given followed fd dFku esa dkSu lh èkkj.kk fufgr gSaA marks in X standard, your admission to
by three Assumption I, II and III. In the 17. Statement/dFku our coaching class for XII standard is
context of the questions an assumption The situation of this area still continues guaranteed”, an advertisement.
is something to be taken for granted. to be tense and out of control. People ";fn vkius X ekud esa 75% ;k vfèkd vad
You have to consider the Statement and are requested to be in their homes only. çkIr fd, gSa] rks vkidks ckjgoha d{kk ds fy,
the assumption and decide which of the bl {ks= dh fLFkfr vHkh Hkh rukoiw.kZ vkSj gekjs dksfpax Dykl esa ços'k dh xkjaVh gS"
assumptions is implicit in the Statement, fu;a=.k ls ckgj gSA yksxksa ls vuqjksèk gS fd os ,d foKkiuA
then decide which one of the answer is vius ?kjksa esa gh jgsAa Assumptions/ekU;rk,a
correct.
Assumptions/ekU;rk,a I. Bright students generally do not opt
fuEufyf[kr ç'u esa] ,d dFku ds ckn rhu
I. There had been some serious for attending coaching classes.
vuqeku I] II vkSj III fn, x, gSaA lokyksa ds
incidents. vkerkSj ij mTtoy fo|kFkhZ dksfpax Dykl
lanHkZ esa ,d èkkj.kk ds fy, dqN fy;k tkuk dqN xaHkhj ?kVuk,a gqbZ FkhaA esa Hkkx ysus dk fodYi ugha pqurs gSaA
gSA vkidks dFku vkSj èkkj.kk ij fopkj djuk II. People will not go to the office. II. The coaching class has adequate
gksxk vkSj ;g r; djuk gksxk fd dFku esa yksx n¶rj ugha tk,axsA capacity to accommodate all such
dkSu lh ekU;rk,¡ fufgr gSa] fQj fu.kZ; ysa III. Normality will its be restored students.
fd buesa ls dkSu lk mÙkj lgh gSA shortly. dksfpax Dykl ds ikl ,sls lHkh Nk=ksa dks
16. Statement/dFku 'kh?kz gh lkekU;rk cgky dj nh tk,xhA lek;ksftr djus dh i;kZIr {kerk gSA
In a recently held all-India conference, (a) All are implicit/lHkh fufgr gSa III. The advertisement will be ignored.
the session on Brand Management in (b) None is implicit/dksbZ Hkh fufgr ugha gS foKkiu dks utjvankt fd;k tk,xkA
India surprisingly attracted a large (c) Only I is implicit/dsoy I fufgr gS (a) Only II is implicit/dsoy II fufgr gS
number of participants and also (d) I and III are implicit/I vkSj III fufgr (b) Only III is implicit/ dsoy III
received excellent media coverage in the
gSa fufgr gS
leading newspapers.
(e) I and II are implicit/I vkSj II fufgr gSa (c) None is implicit/dksbZ Hkh fufgr ugha gS
gky gh esa vk;ksftr vf[ky Hkkjrh; lEesyu
Sol. (e) (d) Only I is implicit/dsoy I fufgr gS
esa] Hkkjr esa czkaM çcaèku ij l= us
The situation is tense, it means that (e) All are implicit/lHkh fufgr gSa
vk'p;Ztud :i ls cM+h la[;k esa çfrHkkfx;ksa
some serious incident has occurred. Sol. (a)
dks vkdf"kZr fd;k vkSj çeq[k lekpkj i=ksa esa Hence, Assumption I is implicit. Assumption I is invalid as it goes against
mR—"V ehfM;k dojst Hkh çkIr fd;kA Secondly, it is given that situation still the given statement. Further, without
Assumptions/ekU;rk,a continues to be tense and people have assuming II, such advertisement makes
I. Nobody expected such an encouraging been requested to be inside home. no sense. Hence, Assumption II is
response to Brand Management. Hence, Assumption II is implicit. implicit. Assumption III again is an
fdlh dks Hkh czkaM çcaèku ds çfr ,slh Assumption III is rubbish and has no invalid assumption because any
mRlkgtud çfrfØ;k dh mEehn ugha FkhA relation with the given Statement. advertisement is given with the
II. Brands are not managed properly in Hence, III is an invalid assumption. assumption of getting desired response
India. and not for being ignored.

343
Download Free PDFs & e-Books from Neon Classes App web. : www.neonclasses.com
eku fy;k x;k gS fd ;g fn, x, dFku ds Assumption I is valid. II is not implicit as gSaA ,d èkkj.kk dqN ekuk tkrk gS ;k nh xbZ
foijhr gS] blfy, IvekU; gS A blds vykok] such an assumption makes the given gSA vkidks dFku vkSj fuEufyf[kr ekU;rkvksa
f}rh; ekus fcuk] bl rjg ds foKkiu dk Statement invalid because of its ij fopkj djuk gksxk vkSj ;g r; djuk
dksbZ eryc ugha gSA blfy,] vuqeku II negativity. gksxk fd dFku esa dkSu lh èkkj.kk fufgr gSA
fufgr gSA èkkj.kk III fQj ls ,d vekU; tc dksbZ vkns'k ikfjr fd;k tkrk gS] rks ;g 21. Statement/dFku
èkkj.kk gS D;ksafd fdlh Hkh foKkiu dks okafNr ekuk tkrk gS fd yksx bldk vuqikyu The railway authority has rescheduled
çfrfØ;k çkIr djus dh èkkj.kk ds lkFk fn;k a sA blfy,] ekU;rk I oSèk gS rFkk II
djsx the departure time of many long-
tkrk gS uk dh vuns[kk fd, tkus ds fy,A fufgr ugha gS D;ksafd bl rjg dh èkkj.kk nh distance trains and put up the revised
Directions (19–20) : In each question xbZ dFku dks mldh udkjkRedrk ds dkj.k timing on its website.
below, is given a Statement followed by vekU; cuk nsrh gSA jsyos çkfèkdj.k us dbZ yach nwjh dh Vªsuksa ds
two assumptions numbered I and II. An 20. Statement/dFku çLFkku le; esa la'kksèku fd;k gS vkSj viuh
assumption is something supposed or The Government decided to levy a toll osclkbV ij la'kksfèkr le; fn;k gSA
taken for granted. You have to consider tax of Rs. 100 for every vehicle using the Assumptions/ekU;rk,a
the Statement and the following superhighway connecting the two big I. The passengers may note the change
assumptions and decide which of the cities of the state. in departure times from the website.
assumption(s) is/are implicit in the ljdkj us Vksy VSDl olwyus dk QSlyk ;k=h osclkbV ls çLFkku ds le; esa
Statement. fd;kA jkT; ds nks cM+s 'kgjksa dks tksM+us okys ifjorZu dks uksV dj ldrs gSaA
uhps fn, x, çR;sd ç'u esa] I vkSj II fxus lqij gkbZos dk mi;ksx djus okys çR;sd II. The passengers may be able to notice
x, nks vuqekuksa ds ckn ,d dFku fn;k x;k okgu ds fy, 100 :- gSA the change Time and board their
gSA ,d èkkj.kk esa dqN ekuk tkrk gS ;k tks Assumptions/ekU;rk,a respective trains before departure.
dFku esa fn xbZ gSA vkidks dFku vkSj I. Majority of the vehicles travelling ;k=h ifjorZu le; dks uksfVl dj ldrs gSa
fuEufyf[kr ekU;rkvksa ij fopkj djuk gksxk between these two cities may not use vkSj çLFkku ls igys viuh lacafèkr Vªsuksa esa
vkSj ;g r; djuk gksxk fd dFku esa dkSu the superhighway. lokj gks ldrs gSaA
lh èkkj.kk fufgr gSA bu nksuksa 'kgjksa ds chp ;k=k djus okys (a) if only Assumption I is implicit
19. Statement/dFku vfèkdka'k okgu lqijgkbos dk mi;ksx ugha vxj dsoy vuqeku I fufgr gS
The state administrations banned dj ldrs gSaA (b) if only Assumption II is implicit
gathering of more than fifty people at II. The Government may not be able to vxj dsoy vuqeku II fufgr gS
any place during the visit of foreign recover the cost incurred for (c) if either Assumption I or Assumption
dignitaries to the city. constructing the superhighway from the II is implicit
jkT; ds ç'kklu us 'kgj ds fons'kh x.kekU; toll tax collection. vxj ;k rks vuqeku I ;k vuqeku II fufgr
yksxksa dh ;k=k ds nkSjku fdlh Hkh LFkku ij ljdkj Vksy VSDl laxzg ls lqij gkbZos ds gS
ipkl ls vfèkd yksxksa ds bdëk gksus ij fuekZ.k ds fy, ykxr olwy ugha dj ldrh (d) if neither Assumption I nor
çfrcaèk yxk fn;kA gSA Assumption II is implicit
Assumptions/ekU;rk,a (a) if only Assumption I is implicit ;fn u rks vuqeku I vkSj u gh vuqeku II
I. People may avoid gathering at any vxj dsoy vuqeku I fufgr gS fufgr gS
place in the city during the period of (b) if only Assumption II is implicit (e) if both Assumptions I and II are
visit of foreign dignitaries. vxj dsoy vuqeku II fufgr gS implicit
fons'kh x.kekU; O;fä;ksa dh ;k=k dh vofèk (c) if either Assumption I or II is implicit ;fn nksuksa vuqeku I vkSj II fufgr gSa
ds nkSjku yksx 'kgj esa fdlh Hkh LFkku ij ;fn ;k rks vuqeku I ;k II fufgr gS Sol. (e)
bdëk gksus ls cp ldrs gSaA (d) if neither Assumption I nor II is Assumption I and II must be valid
II. Many people may ignore the implicit otherwise the given Statement makes no
prohibitory orders and gather to get a ;fn u rks vuqeku I vkSj u gh II fufgr gS sense.
glimpse of the dignitaries. (e) if both Assumptions I and II are ekU;rk I vkSj II ekU; gksuk pkfg, vU;Fkk
cgqr ls yksx fu"ksèkkRed vkns'kksa dh vuns[kh implicit fn, x, dFku dk dksbZ eryc ugha gSA
dj ldrs gSa vkSj x.kekU; yksxksa dh ,d ;fn nksuksa vuqeku I vkSj II fufgr gSa 22. Statement/dFku
>yd ikus ds fy, bdëk gks ldrs gSaA Sol. (d) The civic administrative has asked the
(a) if only Assumption I is implicit Assumption I is irrelevant while II is residents of the dilapidated buildings to
vxj dsoy vuqeku I fufgr gS contrary to the assumption being made. move out as these buildings will be
(b) if only Assumption II is implicit eku fy;k tk, fd I vçklafxd gS] tcfd II demolished within the next thirty days.
vxj dsoy vuqeku II fufgr gS vuqeku ds foijhr gSA ukxfjd ç'kklu us ttZj bekjrksa ds
(c) if either Assumption I or II is implicit Directions (21–22) In each question fuokfl;ksa dks ckgj tkus ds fy, dgk gS
;fn ;k rks vuqeku I ;k II fufgr gS below is given a Statement followed by D;ksfa d vxys rhl fnuksa ds Hkhrj bu bekjrksa
(d) if neither Assumption I nor II is two assumptions numbered I and II. An dks èoLr dj fn;k tk,xkA
implicit assumption is something supposed or Assumptions/ekU;rk,a
;fn u rks vuqeku I vkSj u gh II fufgr gS taken for granted. You have to consider I. The civic administration may be able
(e) if both Assumptions I and II are the Statement and the following to demolish these buildings as per
implicit assumptions and decide which of the schedule.
;fn nksuksa vuqeku I vkSj II fufgr gSa assumption(s) is/are implicit in the ukxfjd ç'kklu bu Hkouksa dks vuqlwph ds
Sol. (a) Statement. vuqlkj èoLr djus esa l{ke gks ldrk gSA
When an order is passed, it is assumed uhps fn, x, çR;sd ç'u esa ,d dFku fn;k II. The residents of these buildings may
that people will comply with it. Hence, x;k gS ftlds ckn I vkSj II dh nks èkkj.kk,¡ vacate and stay elsewhere.

344
Best App for Govt. Jobs : Neonclasses (Download Now) Neon Publications
bu bekjrksa ds fuoklh [kkyh dj ldrs gSa vxj ;k rks vuqeku I ;k vuqeku II fufgr vxj ;k rks vuqeku I ;k vuqeku II fufgr
vkSj vU; txgksa ij jg ldrs gSaA gS gS
(a) if only Assumption I is implicit (d) if neither Assumption I nor (d) if neither Assumption I nor
vxj dsoy vuqeku I fufgr gS Assumption II is implicit Assumption II is implicit
(b) if only Assumption II is implicit ;fn u rks vuqeku I vkSj u gh vuqeku II ;fn u rks vuqeku I vkSj u gh vuqeku II
vxj dsoy vuqeku II fufgr gS fufgr gS fufgr gS
(c) if either Assumption I or Assumption (e) if both Assumptions I and II are (e) if both Assumptions I and II are
II is implicit implicit implicit
vxj ;k rks vuqeku I ;k vuqeku II fufgr ;fn nksuksa vuqeku I vkSj II fufgr gSa ;fn nksuksa vuqeku I vkSj II fufgr gSa
gS Sol. (e) Sol. (b)
(d) if neither Assumption I nor If someone make a promise, he assume In his opinion, the politician basically
Assumption II is implicit that he would be able to fulfil it. Hence, intends to point towards moral
;fn u rks vuqeku I vkSj u gh vuqeku II Assumption I is valid. Again, since the principles of politics. Hence, Assumption
fufgr gS business ventures are planned to be II is implicit. Politician may or may not
(e) if both Assumptions I and II are setup in rural areas, Assumption II must be close to the Prime Minister. So,
implicit be implicit. Assumption I is not implicit.
;fn nksuksa vuqeku I vkSj II fufgr gSa vxj dksbZ oknk djrk gS] rks og eku ysrk gS mudh jk; esa] jktuhfrK ewy :i ls
Sol. (e) fd og mls iwjk djus esa l{ke gksxkA jktuhfr ds uSfrd fl)karksa dh vksj ladrs
Both are positive Statements and are in blfy,] eku ysa fd I oSèk gSA fQj ls] pwafd djuk pkgrs gSaA blfy,] vuqeku II fufgr
accordance with the given Statement. O;kolkf;d m|e xzkeh.k {ks=ksa esa lsVvi fd, gSA jktuhfrK çèkkuea=h ds djhch gks ldrs
nksuksa ldkjkRed dFku gSa vkSj fn, x, dFku tkus dh ;kstuk gS] blfy, vuqeku II dks gSa ;k ugha HkhA blfy,] eku ysa fd I fufgr
ds vuqlkj gSaA fufgr gSA ugha gS A
Directions (23) : In question below is Directions (24): In each question Directions (25) : In question below is
given a Statement followed by two below is given a Statement followed by given a Statement followed by two
assumptions numbered I and II. An two assumptions numbered I and II. An assumptions numbered I and II. An
assumption is something supposed or assumption is something supposed or assumption is something supposed or
taken for granted. You have to consider taken for granted. You have to consider taken for granted. You have to consider
the Statement and the following the Statement and the following the Statement and the following
assumptions and decide which of the assumptions and decide which of the assumptions and decide which of the
assumption(s) is/are implicit in the assumption(s) is/are implicit in the assumption(s) is /are implicit in the
Statement. Statement. Statement.
uhps fn, x, ç'u esa ,d dFku fn;k x;k uhps fn, x, çR;sd ç'u esa ,d dFku fn;k uhps fn, x, ç'u esa ,d dFku fn;k x;k
gS ftlds ckn I vkSj II dh nks èkkj.kk,¡ gSaA x;k gS ftlds ckn I vkSj II dh nks èkkj.kk,¡ gS ftlds ckn I vkSj II dh nks èkkj.kk,¡ gSaA
,d èkkj.kk dqN ekuk tkrk gS ;k nh xbZ gSA gSaA ,d èkkj.kk dqN ekuk tkrk gS ;k nh xbZ ,d èkkj.kk dqN ekuk tkrk gS ;k nh xbZ gSA
vkidks dFku vkSj fuEufyf[kr ekU;rkvksa ij gSA vkidks dFku vkSj fuEufyf[kr ekU;rkvksa vkidks dFku vkSj fuEufyf[kr ekU;rkvksa ij
fopkj djuk gksxk vkSj ;g r; djuk gksxk ij fopkj djuk gksxk vkSj ;g r; djuk fopkj djuk gksxk vkSj ;g r; djuk gksxk
fd dFku esa dkSu lh èkkj.kk fufgr gS @ gSaA gksxk fd dFku esa dkSu lh èkkj.kk fufgr gS fd dFku esa dkSu lh èkkj.kk fufgr gS @ gSaA
23. Statement/dFku @ gSaA 25. Statement/dFku
The Government has recently 24. Statement/dFku The Government has made an appeal to
announced an incentive package for “Considering that his ministry contains all the citizens to honestly pay income
setting up new business ventures in the many hawala-tainted ministers, so the tax and file returns reflecting the true
rural areas and promised uninterrupted Prime Minister has a moral obligation to income level to help the Government to
power supply to all the units. resign”, a politician. carry out development activities.
ljdkj us gky gh esa xzkeh.k {ks=ksa esa u, ";g ekurs gq, fd muds ea=ky; esa dbZ ljdkj us lHkh ukxfjdksa ls bZekunkjh ls
O;kikj miØe LFkkfir djus ds fy, çksRlkgu gokyk&nkxh ea=h gSa] rks çèkku ea=h dk vk;dj dk Hkqxrku djus vkSj ljdkj dks
iSdst dh ?kks"k.kk dh gS vkSj lHkh bdkb;ksa dks bLrhQk nsus dk uSfrd nkf;Ro gS"] ,d fodkl xfrfofèk;ksa dks iwjk djus esa enn
fujarj fctyh vkiwfrZ dk oknk fd;k gSA jktuhfrKA djus ds fy, okLrfod vk; Lrj dks n'kkZrs
Assumptions/ekU;rk,a Assumptions/ekU;rk,a gq, fjVuZ nkf[ky djus dh vihy dh gSA
I. The Government may be able to I. The politician is not close to the Prime Assumptions/ekU;rk,a
supply adequate power to all such units. Minister. I. People may now start paying more
ljdkj ,slh lHkh bdkb;ksa dks i;kZIr fctyh jktusrk çèkkuea=h ds djhch ugha gSaA taxes in response to the appeal.
dh vkiwfrZ djus esa l{ke gks ldrh gSA II. The politician would like to inculcate yksx vc vihy ds tokc esa vfèkd dj nsuk
II. People living in the rural areas may moral principles in politics. 'kq: dj ldrs gSaA
welcome the Government decision. jktusrk jktuhfr esa uSfrd fl)karksa dks II. The total income tax collection may
xzkeh.k {ks=ksa esa jgus okys yksx ljdkj ds fodflr djuk pkgsx
a sA considerably increase in the near future.
QSlys dk Lokxr dj ldrs gSaA (a) if only Assumption I is implicit fudV Hkfo"; esa dqy vk;dj laxzg esa dkQh
(a) if only Assumption I is implicit vxj dsoy vuqeku I fufgr gS o`f) gks ldrh gSA
vxj dsoy vuqeku I fufgr gS (b) if only Assumption II is implicit (a) if only Assumption I is implicit
(b) if only Assumption II is implicit vxj dsoy vuqeku II fufgr gS vxj dsoy vuqeku I fufgr gS
vxj dsoy vuqeku II fufgr gS (c) if either Assumption I or II is implicit (b) if only Assumption II is implicit
(c) if either Assumption I or II is implicit vxj dsoy vuqeku II fufgr gS

345
Download Free PDFs & e-Books from Neon Classes App web. : www.neonclasses.com
(c) if either Assumption I or II is implicit dk;Zokgh ¼mik;½ & fdlh leL;k ds ¼XIII½There should be a logical positive
vxj ;k rks vuqeku I ;k vuqeku II fufgr lek/kku gsrq mBk;k x;k dne dk;Zokgh solution in the proceedings.
gS dgykrk gSA dk;Zokgh esa ,d rkfdZd ldkjkRed lek/kku
(d)if neither Assumption I nor Key points of action gksuk pkfg;sA
Assumption II is implicit dk;Zokgh ds egRoiw.kZ fcUnq → ¼XIV½Key words - Only, always, never, if all
;fn u rks vuqeku I vkSj u gh vuqeku II ¼I½ If the problem is simple, then the the words come, then that action can be
fufgr gS solution should also be simple. wrong.
(e)if both Assumptions I and II are vxj leL;k lk/kkj.k gks rks mik; Hkh Key Words – dsoy] ges’kk] dHkh ugha] lHkh
implicit lk/kkj.k gh gksuk pkfg,A vkfn 'kCn vk;s rks rks og dk;Zokgh xyr gks
;fn nksuksa vuqeku I vkSj II fufgr gSa ¼II½ If the problem is difficult / complicated, ldrh gSA
Sol. (e) then the remedy or action should also ¼XV½ If the problem is international, then
Assumptions I and II both are implicit be difficult. action should be based on the common
because both are imminent positive leL;k dfBu@tfVy gks rks mik; ;k ideology.
outcomes assumed. dk;Zokgh Hkh dfBu gksuh pkfg,A ;fn leL;k vUrZjk"Vªh; gks rks dk;Zokgh
ekU;rkvksa I vkSj II nksuksa fufgr gSa D;ksfa d ¼III½ Action given should be based on lkekU; fopkj/kkjk ds vk/kkj ij gksuh
nksuksa vklUu ldkjkRed ifj.kke gSaA experience. pkfg,A
STATEMENT & COURSE nh xbZ leL;k dh dk;Zokgh vuqHko ds ¼XVI½ If there are two cases at the same time
vk/kkj ij gksuh pkfg, in the proceedings, it will go to the case
OF ACTION ¼IV½ Action taken on the basis of the right of either or.
advice will be considered strong. ;fn dk;Zokgh esa ,d gh le; esa nks action
Statement – Courses of Action lgh lykg ds vk/kkj ij dh xbZ dk;Zokgh gks rks og ;k rks ;k okys dsl esa pyh
dFku & dk;Zokgh etcwr ekuh tk;sxhA tk,xhA
The problems that occur based on the ¼V½ The action should be related to the Direction (1 - 25):
given statement. Exiting from that subject. (a) If only I follows /;fn dsoy Ia vuqlj.k
problem is called action, that is, finding dk;Zokgh fo"k; ls lEcfU/kr gksA djrk gSA
the best solution to that problem is ¼VI½ Action should diagnose one problem (b) If only II follows /;fn dsoy II
called action. and not cause another. vuqlj.k djrk gSA
fn;s x;s dFku ds vk/kkj ij tks leL;k,sa dk;Zokgh ls ,d leL;k dk funku gksuk (c) If either I or II follows/;fn ;k rks I ;k
gksrh gSA ml leL;k ls ckgj fudyus dks gh pkfg, uk fd mlls nwljh leL;k mRiUu II vuqlj.k djrk gSA
dk;Zokgh dgk tkrk gS vFkkZr~ ml leL;k dk gksuh pkfg,A (d) If neither I nor II follows /;fn u rks I
lcls vPNk mik; [kkstuk gh dk;Zokgh ¼VII½ Action should be logical and not vkSj u gh II vuqlj.k djrk gSA
dgykrk gSA philosophical. (e) If both I and II follow /;fn I vkSj II
The given problem should be taken only dk;Zokgh rkfdZd gksuh pkfg, uk fd nksuksa vuqlj.k djrs gSa A
on the basis of its real life and its nk’kZfud gksuh pkfg, 1. Statement :/ dFku
solution. ¼VIII½ If there appears to be universal truth Almost 90% of the fights of one of the
nh xbZ leL;k dks vius okLrfod thou ds in action, then it will be considered as private airline company were cancelled
vk/kkj ij eku dj gh mldk mik; fudkyuk strong action. for the fourth consecutive day as the
pkfg;sA ;fn dk;Zokgh esa lkoZHkkSfed lR; izrhr gksrk pilots refused to join their duties in
In this type of questions, two or three gks rks og etcwr dk;Zokgh ekuh tk;sxhA protest against sacking of two of their
actions are given in the form of problem, ¼IX½ If there is talk of national interest in the colleagues by the airline management.
day-to-day occurrence, policy, proceedings, then it will be considered
omnipresent perception and choosing truth. ,d futh ,;jykbu daiuh dh yxHkx 90%
the best course of action. ;fn dk;Zokgh esa jk"Vªghr dh ckr dh tk, mM+kus yxkrkj pkSFks fnu jí dj nh xbZ
bl izdkj ds iz’uksa esa leL;k] fnu&izfrfnu rks og lR; ekuh tk;sxhA D;ksfa d ik;yVksa us ,;jykbu çcaèku }kjk
dh ?kVuk] uhfr] loZO;kih /kkj.kk ds :Ik esa ¼X½ Thoughts of themselves are not kept in muds nks lg;ksfx;ksa dks c[kkZLr djus ds
nks ;k rhu dk;Zokfg;ka nh tkrh gS muesa the proceedings but should be fojksèk esa vius dk;Z esa 'kkfey gksus ls budkj
loZJs"B dk;Zokgh dk p;u djuk gksrk gSA confirmed on social grounds. dj fn;k FkkA
Statement (Problem) = Problem can be dk;Zokgh essa [kqn ds fopkj ugha j[ks tkrs gS Courses of Action/ dk;Zokgh
of many types like → epidemic, cfYd lkekftd vk/kkj ij iq"Vh gksuh I. The management of the airline
stalemate, dowry system, flood, pkfg;sA company should be ordered by the
earthquake, world wide, economic ¼XI½ Must have personal experience in Government to immediately reinstate
crisis, financial crisis, unemployment action the sacked pilots to end the crisis.
bus or train accident communal riot, dk;Zokgh esa O;fDrxr vuqHko gksuk pkfg;s ladV dks lekIr djus ds fy, ,;jykbu
terrorist incident etc. daiuh ds çcaèku dks ljdkj dks c[kkZLr
¼XII½ In the proceedings, there is either a
dFku ¼leL;k½ ¾ leL;k dbZ izdkj dh gks ik;yVksa dks rqjar cgky djus dk vkns'k nsuk
complete diagnosis of the problem or it
ldrh gS tSls → egkekjh] xfrjks/k] ngst is said to reduce the problem. pkfg,A
izFkk] ck<+] HkwdEi] fo’o O;kih] vkfFkZd ladV] dk;Zokgh esa ;k rks ml leL;k dk iwjk II. The Government Should immediately
foŸkh; ladV] csjkstxkjh cl ;k Vªsu nq?kZVuk funku gks ;k ml leL;k dks de djus dh take steps to end the impasse between
lkEiznkf;d naxk] vkradh ?kVuk vkfnA ckr dgh xbZ gksA the management and the pilots to help
Action (Remedy) - Action taken to solve the helpless passengers.
a problem is called Action.

346
Best App for Govt. Jobs : Neonclasses (Download Now) Neon Publications
ljdkj dks vlgk; ;kf=;ksa dh lgk;rk ds Hkkjr dks Hkh rqjar bl çdkj dk lEesyu dk;Zokgh I vuqlj.k djrk gSA D;ksfa d ,sls
fy, çcaèku vkSj ik;yVksa ds chp xfrjksèk dks vk;ksftr djuk pkfg,A le; ij pquko djkuk fujFkZd gksxkA II
lekIr djus ds fy, rqjar dne mBkus Sol. (a) dk;Zokgh Hkkjh {kfr dks ns[krs gq, vuqlj.k
pkfg,A Implementation of such action plans as djrk gSA
Sol. (b) farmed by the conference is a right 6. Statement :/dFku
If the Government goes for I, it would course of action as it will give an Although the Indian economy is still
impinge on the autonomy of the private immediate and effective impact on the heavily dependent on agriculture, its
sector. standard of education for the children. share in global agricultural trade is less
;fn ljdkj dk;Zokgh ds vuqlkj dh tkrh gS] lEesyu }kjk dh xbZ dk;Zokgh ykxw djuk than the share of agricultural exports to
rks ;g futh {ks= dh Lok;Ùkrk ij çHkko ,d lgh dne gS D;ksfa d ;g cPpksa ds f'k{kk total exports.
MkysxkA ds ekud ij rRdky vkSj dk;Zdkjh çHkko ;|fi Hkkjrh; vFkZO;oLFkk vHkh Hkh —f"k ij
2. Statement :/ dFku MkysxkA cgqr vfèkd fuHkZj gS] oSf’od —f"k O;kikj esa
A major part of the local market in the 4 Statement :/ dFku bldh fgLlsnkjh dqy fu;kZr ds fy,] —f"k
city was gutted due to a short circuit A number of school children in the local fu;kZr dh fgLlsnkjh ls de gSA
causing extensive damage to goods and schools have fallen ill after the Courses of Action / dk;Zokgh
property. consumption of their subsidised tiffin I. Efforts should be made to increase our
'kgj esa LFkkuh; cktkj dk ,d cM+k fgLlk provided by the school authority. agricultural production.
'k‚VZ lfdZV ds dkj.k fujk'k gks x;k Fkk] LFkkuh; Ldwyksa ds dbZ Ldwyh cPps Ldwy gekjs —f"k mRiknu dks c<+kus ds ç;kl fd,
ftlls eky vkSj laifÙk dks O;kid uqdlku vFk‚fjVh }kjk çnku dh xbZ lfClMh okys tkus pkfg,A
gqvkA fVfQ+u dks [kkus ds ckn chekj iM+ x, gSA II. The exports of non- agricultural
Courses of Action / dk;Zokgh Courses of Action / dk;Zokgh commodities should be reduced.
I. The Government should issue strict I. The tiffin facility of all schools should xSj&—f"k oLrqvksa ds fu;kZr dks de fd;k
guidelines for all establishments be discontinued with immediate effect. tkuk pkfg,A
regarding installation and maintenance lHkh Ldwyksa dh fVfQu lqfoèkk rRdky çHkko Sol. (a)
of electrical fittings. ls can dj nh tkuh pkfg,A Only by increasing our agricultural
ljdkj dks fo|qr fQfVax dh LFkkiuk vkSj II. The Government should implement a production, we can have a better
j[kj[kko ds ckjs esa lHkh çfr"Bkuksa ds fy, system to certify the quality of tiffin position in international agricultural
l[r fn'kkfunZs'k tkjh djuk pkfg,A provided by the School. trade. Reduction in non-agricultural
II. The government should relocate all ljdkj dks Ldwy }kjk çnku fd, x, fVfQu commodities will further worsen our
the markets to the outskirts of the city. dh xq.koÙkk dks çekf.kr djus ds fy, ,d position. So, only I is right course of
ljdkj dks 'kgj ds ckgjh bykdksa ds lHkh ç.kkyh ykxw djuh pkfg,A action.
cktkjksa dks LFkkukarfjr djuk pkfg,A Sol. (b) dsoy gekjs —f"k mRiknu esa o`f) djds] ge
Sol. (a) I is invalid because of being an extreme varjjk"Vªh; —f"k O;kikj esa csgrj fLFkfr esa gks
I follows as a measure of caution. But II action. II is proper corrective measure. ldrs gSA xSj&—f"k oLrqvksa esa deh ls gekjh
won’t solve the problem and poor eSa ,d pje dk;Zokgh gksus ds dkj.k vekU; fLFkfr vkSj [kjkc gks tk,xhA blfy,] dsoy
electrical fittings would wreak havoc gSA II mfpr lqèkkjkRed mik; gSA dk;Zokgh I gh dk;Zokgh dk lgh rjhdk gSA
where ever the market be. 5. Statement :/dFku 7. Statement :/dFku
dk;Zokgh I lkoèkkuh ds ,d mik; ds :i esa Heavy rains hit the state during October, The rate of inflation has reached its
vuqlj.k djrk gSA ysfdu II leL;k dk just before the state Assembly elections highest in last twenty years and there is
lekèkku ugha djsxk vkSj [kjkc fctyh dh and caused heavy damage to standing no sign of it softening in the coming
fQfVax dgj cjik,xh tgka dHkh Hkh cktkj crops in most parts of the state. months.
gksxkA jkT; foèkkulHkk pquko ls Bhd igys vDVwcj fiNys chl o"kksZa esa eqækLQhfr dh nj vius
3. Statement :/dFku ds nkSjku jkT; esa Hkkjh ckfj'k gqbZ] vkSj jkT; mPpre Lrj ij igqap xbZ gS vkSj vkus okys
The world conference of ‘Education for ds vfèkdka'k fgLlksa esa [kM+h Qlyksa dks Hkkjh eghuksa esa blds uje gksus dk dksbZ ladsr ugha
All' took place in 1990. Members who uqdlku igqapkA gSA
attended conference endorsed the Courses of Action / dk;Zokgh Courses of Action / dk;Zokgh
frame-work of action for meeting the I. Elections should be postponed to give I. The Government should initiate steps
basic learning needs of all children. candidates the opportunity to campaign. like reducing Government taxes on
"Education for All" dk fo'o lEesyu 1990 mEehnokjksa dks çpkj djus dk volj nsus ds essential commodities with immediately
esa gqvkA lEesyu esa 'kkfey gksus okys lnL;ksa fy, pquko LFkfxr dj fn;k tkuk pkfg,A effect.
us lHkh cPpksa dh cqfu;knh lh[kus dh t:jrksa II. The Government should announce a ljdkj dks rRdky çHkko ls vko';d oLrqvksa
dks iwjk djus ds fy, dk;Zokgh ds <kaps dk relief package for those who are ij ljdkjh djksa dks de djus tSls dne
leFkZu fd;kA affected. 'kq: djus pkfg,A
Courses of Action / dk;Zokgh ljdkj dks çHkkfor gksus okyksa ds fy, jkgr II. Farmers Should be asked by the
I. India should suitably implement the iSdst dh ?kks"k.kk djuh pkfg,A Government to sell their products at
action points of this conference. Sol. (e) lower prices.
Hkkjr dks bl lEesyu ds dk;Zokgh fcanqvksa dks I follows because handing the elections fdlkuksa ls dgk tk, fd os vius mRiknksa dks
mi;qä :i ls ykxw djuk pkfg,A at this juncture would render the de dherksa ij cspsA
II. India should also immediately exercise meaningless. II follows for Sol. (d)
organise this type of conference. ‘heavy damage. Going for I and II will create other
problems.

347
Download Free PDFs & e-Books from Neon Classes App web. : www.neonclasses.com
I vkSj II nksuks vU; izdkj dh leL;k,a iSnk egkekjh dh tk¡p ds fy, ,gfr;krh mik; be deployed but only during the
djsx
a sA gj pkj lky ds ckn fd, tkus pkfg,A sensitive hours.
8. Statement :/ dFku II. People should be advised to drink dk;Zokgh dk ,d mfpr dk;Zokgh bu Dycksa
The state Government has decided to boiled water during the monsoon dks Lo;a O;ogkj djus ds fy, uksfVl nsuk
declare ‘Kala Azar’ as a notifiable season. gksxkA ;gka rd fd iqfyl dfeZ;ksa dks Hkh
disease under the Epidemic Act. Under yksxksa dks ekulwu ds ekSle esa mcyk gqvk rSukr fd;k tk ldrk gS ysfdu laosnu'khy
the Epidemic Act, 1897, family members ikuh ihus dh lykg nh tkuh pkfg,A ?kaVksa ds nkSjkuA
or neighbours of the patient are liable to (e) Both II and III follow / vkSj III nksuksa 11. Statement :/ dFku
be punished in case they did not inform vuqlj.k djrs gSa A A major railway accident involving a
the State Authorities. Sol. (b) mail train was averted due to the
jkT; ljdkj us egkekjh vfèkfu;e ds rgr It is not necessary that such an epidemic presence of mind of one signal man at a
Þdkyk vtkjß dks ,d mYys[kuh; chekjh occurs after four years. Hence, Course of wayside cabin.
?kksf"kr djus dk fu.kZ; fy;k gSA egkekjh action I is not the right course of action. ,d flXuy eSu dh cqf) rRijrk ls ,d esy
vfèkfu;e] 1897 ds rgr] ejht ds ifjokj Secondly, prevention during monsoon VªSu }kjk gks ldus okyh ,d cM+h nq?kZZVuk
ds lnL; ;k iM+ksfl;ksa dks] vfèkdkfj;ksa dks season is the right step to face the Vy x;hA
lwfpr ugha djus dh fLFkfr esa nafMr gksus ds problem of conjunctivitis. So course of Courses of Action / dk;Zokgh
fy, mÙkjnk;h gSA action II is the right course of action. I. The railway track for atleast 50 km
Courses of Action / dk;Zokgh ;g vko';d ugha gS fd bl rjg dh should be cleared off any traffic ahead of
I. Efforts should be made to efficiently egkekjh pkj lky ckn gksrh gSA blfy,] all the mail trains.
implement the Act. dk;Zokgh I dk;Zokgh dk lgh dk;Zokgh ugha de ls de 50 fdeh ds fy, jsyos VªSd dks
vfèkfu;e dks dq'kyrk ls ykxw djus ds gSA nwljs] ekulwu ds ekSle esa jksdFkke vk¡[k lHkh esy Vªsuksa ls vkxs fdlh Hkh VªSfQd dks
ç;kl fd, tkus pkfg,A vkus dh leL;k dk lkeuk djus ds fy, gVk nsuk pkfg,A
II. The cases of punishment should be lgh dne gSA blfy, dk;Zokgh dk rjhdk II II. The railways signalling systems
propagated through mass, so, that more dk;Zokgh dk lgh rjhdk gSA should immediately be made automatic.
people become aware of the stern 10. Statement :/ dFku jsyos flXuy ç.kkyh dks rqjar Lopkfyr
action. The members belonging to two local cuk;k tkuk pkfg,A
ltk ds ekeyksa dks cM+s iSekus ij çpkfjr clubs occasionally fight with each other III. The signal man should be rewarded
fd;k tkuk pkfg,] rkfd vfèkd ls vfèkd on the main road and block traffic so as to encourage others.
yksx dM+h dk;Zokgh ds ckjs esa tkx:d gks movement. flXuy eSu dks iqjL—r fd;k tkuk pkfg,
ldsA nks LFkkuh; Dycksa ds lnL; dHkh&dHkh eq[; rkfd nwljksa dks çksRlkfgr fd;k tk ldsA
Sol. (e) lM+d ij yM+rs gS vkSj ;krk;kr dks can (a) All follow / lHkh vuqlj.k djrs gSa A
It is given in the Statement that ‘Kala djok nsrs gSA (b) None follow / dksbZ Hkh vuqlj.k ugha
Azar’ has been declared as notifiable Courses of Action / dk;Zokgh djrk gSA
disease. Since, it is a good effort by the I. The local police station should (c) Both I and II follow / I vkSj II nksuksa
Government, hence effort should be immediately deploy police personnel vuqlj.k djrs gSa A
made to implement the Act. Moreover, a round the clock on the main road. (d) Both II and III follow / II vkSj III nksuksa
deterrent in the form of punishment will LFkkuh; iqfyl LVs'ku dks eq[; lM+d ij vuqlj.k djrs gSa A
ensure effective implementation of the pkSchlksa ?kaVs iqfyl dfeZ;ksa dks rSukr djuk (e) None of the above / mijksä esa ls dksbZ
Act. pkfg,A ugha
;g dFku esa fn;k x;k gS Þdkyk vtkj* dks II. Those involved in fighting should be Sol. (a)
mYys[kuh; chekjh ?kksf"kr fd;k x;k gSA identified and put behind bars. Courses of Action I and II will directly
pwafd] ;g ljdkj }kjk ,d vPNk ç;kl gS] yM+kbZ esa 'kkfey yksxksa dh igpku dh tkuh improve the working condition of
blfy, vfèkfu;e dks ykxw djus dk ç;kl pkfg, vkSj mUgsa lyk[kksa ds ihNs Mkyk tkuk railways. Course of action III through,
fd;k tkuk pkfg,A blds vykok] ltk ds pkfg,A not directly related with improving
:i esa ,d fuokjd vfèkfu;e çHkkoh III. The local administration should send railway traffic conditions but will
dk;kZUo;u dks lqfuf'pr djsxkA notice to the management of the two encourage other signal men to be more
9. Statement :/ dFku clubs with immediate effect. watchful on their duties, which in turn,
Every year, at the beginning or at the LFkkuh; ç'kklu dks rRdky çHkko ls nksuksa will reduce the probability of accidents.
end of the monsoon, we have some Dycksa ds çcaèku dks uksfVl Hkstuk pkfg,A dk;Zokgh I vkSj II jsyos dh dk;Z'khy fLFkfr
cases of conjuctivitis, but this year, it (a) Both I and II follow / I vkSj II nksuksa a sA dk;Zokgh III ds ekè;e
esa lhèks lqèkkj djsx
seems to be a major epidemic, vuqlj.k djrs gSa A ls] lhèks jsyos ;krk;kr dh fLFkfr esa lqèkkj ls
witnessed after nearly four years. (b) Both I and III follow / I vkSj III nksuksa lacafèkr ugha gS] ysfdu vU; flXuy iq#"kksa dks
gj lky] ekulwu dh 'kq#vkr esa ;k var es]a vuqlj.k djrs gSa A vius drZO;ksa ij vfèkd pkSdl jgus ds fy,
gesa datfDVokbfVl ds dqN ekeys gksrs gSa] (c) Only III follow / dsoy III vuqlj.k çksRlkfgr djsxk] tks nq?kZVukvksa dh laHkkouk
ysfdu bl lky] ;g yxHkx pkj o"kksaZ ds ckn djrk gSA dks de djsxkA
ns[kk x;k ,d çeq[k egkekjh gSA (d) All follow / lHkh vuqlj.k djrs gSA 12. Statement :/ dFku
Courses of Action / dk;Zokgh Sol. (c) Any further increase in pollution level in
I. Precautionary measures should be A proper course of action would be the city, by way of industrial effluent
taken after every four years to check the serving notices to these clubs to behave and automobile exhaustion, would pose
epidemic. themselves. Even police personnel may a severe threat to the inhabitants.

348
Best App for Govt. Jobs : Neonclasses (Download Now) Neon Publications
'kgj esa çnw"k.k ds Lrj esa fdlh Hkh rjg dh (c) Both II and III follow / II vkSj III nksuksa 15. Statement :/ dFku
o`f)] vkS|ksfxd vif'k"V vkSj v‚Vkseksckby ds vuqlj.k djrs gSa A A large number of students who have
dkj.k FkdkoV fuokfl;ksa ds fy, ,d xaHkhj (d) All follow / lHkh vuqlj.k djrs gSa A passed their XII std. terminal
[krjk gksxkA Sol. (c) examination in the country could not get
Courses of Action / dk;Zokgh It is given in the Statement that admission to colleges as the number of
I. All the factories in the city should be chloroquine has because ineffective for seats available are grossly inadequate.
immediately closed down. malaria patients. Therefore, researchers cM+h la[;k esa ,sls Nk= ftUgksua s viuh ckjgoha
'kgj dh lHkh QSfDVª;ksa dks rRdky can fd;k should develop a new medicine for the dh ijh{kk ikl dh gSA ns'k esa VfeZuy ijh{kk
tkuk pkfg,A patients and all the patients suffering dks d‚ystksa esa ços'k ugha fey ldrk Fkk
II. The automobiles should not be from malaria should be checked for D;ksfa d miyCèk lhVksa dh la[;k ldy
allowed to run near roads for more than identification of the causal mosquito. vi;kZIr gSA
four hours a day. ;g dFku esa fn;k x;k gS fd DyksjksDohu Courses of Action / dk;Zokgh
v‚Vkseksckby dks fnu esa pkj ?kaVs ls vfèkd eysfj;k ds jksfx;ksa ds fy, vçHkkoh gSA I. The evaluations system of XII Std.
lM+d ij [ksyus dh vuqefr ugha nh tkuh blfy,] 'kksèkdrkZvksa dks jksfx;ksa ds fy, ,d terminal examination should be made
pkfg,A ubZ nok fodflr djuh pkfg, vkSj eysfj;k more tough so that fewer students pass
III. The Government should restrict the ls ihfM+r lHkh jksfx;ksa dks dkVus okys ePNj the examination.
issue of fresh licenses to factories and dh igpku ds fy, tk¡p djuh pkfg,A ckjgoha d{kk dk ewY;kadu ç.kkyh] VfeZuy
automobiles. 14. Statement :/ dFku ijh{kk dks vkSj vfèkd dfBu cuk;k tkuk
ljdkj dks dkj[kkuksa vkSj v‚Vkseksckby dks There is a considerable increase in the pkfg, rkfd de Nk= ijh{kk ikl dj ldsA
u, ykblsal ds eqís dks çfrcafèkr djuk number of persons affected by water – II. The Government should encourage
pkfg,A borne diseases during monsoon period. the private sector to open new colleges
(a) Only III follows/ dsoy III vuqlj.k ekulwu dh vofèk ds nkSjku ty tfur jksxksa by providing them land at cheaper rate.
djrk gSA ls çHkkfor O;fä;ksa dh la[;k esa dkQh o`f) ljdkj dks futh {ks= dks lLrs nj ij tehu
(b) All follow / lHkh vuqlj.k djrs gSa A gqbZ gSA miyCèk djkdj u, d‚yst [kksyus ds fy,
(c) Only II follows / dsoy II vuqlj.k Courses of Action / dk;Zokgh çksRlkfgr djuk pkfg,A
djrk gSA I. The question should be raised in the III. The rich people should be asked to
(d) None follows / dksbZ Hkh vuqlj.k ugha Legislative Assembly. send their wards to foreign countries for
djrk gSA foèkku lHkk esa ç'u mBk;k tkuk pkfg,A higher students enabling the needy
II. The Government should disseminate students to get admission in college
Sol. (a)
Only course of action III is the right adequate information regarding the within the country.
course of action because other courses pure drinking water to people. vehj yksxksa dks ns'k ds Hkhrj d‚yst esa ços'k
are not feasible. yksxksa dks 'kq) is;ty ds ckjs esa ljdkj dks ikus ds fy, t:jrean Nk=ksa dks l{ke djus
dk;Zokgh III ,d lgh dk;Zokgh gS] D;ksfa d i;kZIr tkudkjh dk çlkj djuk pkfg,A okys mPp Nk=ksa ds fy, vius okMksaZ dks
vU; dk;Zokfg;ka laHko ugha gSA III. All the hospitals in the city should be fons'kksa esa Hkstus ds fy, dgk tkuk pkfg,A
13. Statement :/ dFku equipped properly for the treatment of (a) Only I follows / dsoy I vuqlj.k djrk
Some strains of mosquito have become patents during monsoon period. gSA
resistant to chloroquine, the widely ekulwu vofèk ds nkSjku isVasV ds mipkj ds (b) Only II follows / dsoy II vuqlj.k
used medicine for malaria patients. fy, 'kgj ds lHkh vLirkyksa dks Bhd ls djrk gSA
eysfj;k ds jksfx;ksa ds fy, O;kid :i ls lqlfTtr fd;k tkuk pkfg,A (c) Only I and II follows/ dsoy I vkSj II
bLrseky dh tkus okyh nokbZ DyksjksfDou (a) All follow / lHkh dk vuqlj.k djrh gSA bl çdkj gSa
ePNj blds fy, çfrjksèkh cu x, gSA (b) Only I and II follow / dsoy I vkSj II (d) Only II and III follow / dsoy II vkSj
Courses of Action / dk;Zokgh vuqlj.k djrs gSa A III vuqlj.k djrs gSa A
I. Selling of chloroquine should be (c) Only II and III follow / dsoy II vkSj (e) None of these/ buesa ls dksbZ ugha
stopped. III vuqlj.k djrs gSa A Sol. (b)
DyksjksDohu dh fcØh dks jksdk tkuk pkfg,A (d) Only I and III follow / dsoy I vkSj III Clearly, reducing the number of
II. Researches should develop a new vuqlj.k djrs gSa A aspirants for admission to colleges or
medicine for patients affected by such (e) None follows / dksbZ Hkh vuqlj.k ugh sending the students of well-to-do
mosquitos. djrs gSA families to foreign countries for higher
'kksèkdrkZ dks ,sls ePNjksa ls çHkkfor jksfx;ksa ds Sol. (c) studies, is no proper solution, So, both I
fy, ,d ubZ nok fodflr djuh pkfg,A Any aspect of health has two factors to and III do not follow. The right solution
III. All the patients suffering from tackle with – prevention and treatment. is to increase the number of colleges so
malaria should be checked for prevention includes creating awareness as to accommodate the increasing
identification of causal mosquito. among people, and treatment includes number of admission – seekers. So, only
eysfj;k ls ihfM+r lHkh jksfx;ksa dks dkj.k providing adequate medical facilities. II follows.
ePNj dh igpku ds fy, tk¡p dh tkuh So, both II and III follow. Li"V :i ls] egkfo|ky;ksa esa ços'k ds fy,
pkfg,A LokLF; ds fdlh Hkh igyw ls fuiVus ds nks mEehnokjksa dh la[;k de djuk ;k mPp
(a) None follows / dksbZ ugha vuqlj.k djrs dkjd gSa & jksdFkke vkSj mipkjA jksdFkke esa vè;;u ds fy, vPNh rjg ls djus okys
gSa A yksxksa esa tkx:drk iSnk djuk 'kkfey gS] ifjokjksa ds Nk=ksa dks fons'k Hkstuk] dksbZ
(b) Both I and II follow / I vkSj II nksuksa vkSj mipkj esa i;kZIr fpfdRlk lqfoèkk,a çnku mfpr lekèkku ugha gS] blfy,] I vkSj III
vuqlj.k djrs gSa A djuk 'kkfey gSA rks] II vkSj III nksuksa nksuksa dk ikyu ugha djrs gSA lgh lekèkku
vuqlj.k djrs gSa A ;g gS fd ços'k dh c<+rh la[;k dks
349
Download Free PDFs & e-Books from Neon Classes App web. : www.neonclasses.com
lek;ksftr djus ds fy, d‚ystksa dh la[;k yksxksa }kjkA blds vykok] ijh{kkFkÊ tks mÙkj lkoZtfud {ks= ds cSadksa }kjk tkap ds fy,
c<+kbZ tk,A rks] dsoy II vuqlj.k djrk gSA iqfLrdkvksa ds ewY;kadu ds vius drZO; ls fu;ked çkfèkdj.k ds le{k ekeyk mBk;k
16. Statement :/ dFku cprs gSa] mUgsa nafMr fd;k tkuk pkfg,A tkuk pkfg, D;ksfa d os bl rjg dh dVkSrh
It has been reported by one of the TV blfy,] I vkSj II nksuksa vuqlj.k djrs gSa A] dk ikyu ugha dj ldrs gSA
channels that the answer papers of tcfd III ugha djrk gSA II. Public sector banks must adopt such
Board examination of one State are 17. Statement :/ dFku policy to remain in competition.
evaluated by students studying in the Poverty is increasing because the lkoZtfud {ks= ds cSadksa dks çfrLièkkZ esa cus
same standard with help of model people, who are deciding how to tackle jgus ds fy, ,slh uhfr viukuh pkfg,A
answers as instructed by the examiners. it, know absolutely nothing about the III. The public sector banks should
Vhoh pSuyksa esa ls ,d }kjk ;g crk;k x;k gS poor. advertise their special feature
fd ,d jkT; dh cksMZ ijh{kk ds mÙkj xjhch c<+ jgh gS D;ksfa d yksx] tks bldk repeatedly so that they do not lose their
iqfLrdkvksa dk ewY;kadu mlh ekud ds lkFk lkeuk djus dk QSlyk dj jgs gSa] xjhcksa ds future customers.
vè;;u djus okys Nk=ksa }kjk fd;k tkrk gS] ckjs esa fcYdqy dqN ugha tkurs gSA lkoZtfud {ks= ds cSadksa dks vius fo'ks"k
tSlk fd ijh{kkfFkZ;ksa }kjk funZs'k ds vuqlkj Courses of Action / dk;Zokgh lqfoèkk dk ckj&ckj foKkiu nsuk pkfg, rkfd
fd;k tkrk gSA I. The decision makers should go to the os vius Hkfo"; ds xzkgdksa dks u [kks,A
Courses of Action / dk;Zokgh grass root levels. (a) All follow / lHkh vuqlj.k djrs gSa A
I. All such examiners should be fu.kZ; fuekZrkvksa dks tehuh Lrj ij tkuk (b) Only I and II follow / dsoy I vkSj II
immediately suspended from their pkfg,A vuqlj.k djrs gSa A
official positions. II. The decision makers should come (c) Only I and III follow / dsoy I vkSj III
,sls lHkh ijh{kkfFkZ;ksa dks muds vkfèkdkfjd from the poorer sections of the society. vuqlj.k djrs gSa A
inksa ls rqjar fuyafcr dj fn;k tkuk pkfg,A fu.kZ; fuekZrkvksa dks lekt ds xjhc oxksZa ls (d) Only either II or III follows/ dsoy II
II. All such papers evaluated by the vkuk pkfg,A ;k III vuqlj.k djrk gSA
students should be immediately III. A new set of decision makers should (e) None of these / buesa ls dksbZ ugha
confiscated and got evaluated by replace the existing one. Sol. (d)
qualified teachers. fu.kZ; cukus okyks ds ,d u, lsV dks ekStwnk Clearly, the private sector banks have
Nk=ksa }kjk ewY;kadu fd, x, ,sls lHkh i=ksa fu.kkZ;dksa dks cnyuk pkfg,A done so to attract more and more
dks rqjar tCr dj fy;k tkuk pkfg, vkSj (a) Only I follows / dsoy I vuqlj.k djrk customers of public sector banks. Thus,
;ksX; f'k{kdksa }kjk ewY;kadu fd;k tkuk gSA public sector banks should either reduce
pkfg,A (b) Only II follows / dsoy II vuqlj.k the rate of interest to match with that of
III. The Board should explore djrk gSA private sector banks or highlight those
possibilities even though they are (c) Only either I or III follows/ dsoy I ;k features which make them stand apart
remote, of getting the answer papers of III vuqlj.k djrk gSA from private sector banks. So, either II
this examination evaluated by (d) All follow / lHkh vuqlj.k djrs gSa A or III follows.
computerised machines. Li"V :i ls] futh {ks= ds cSadksa us lkoZtfud
(e) None of these / buesa ls dksbZ ugha
daI;wVjhd`r e'khuksa }kjk ewY;kadu fd, x, {ks= ds cSadksa ds vfèkd ls vfèkd xzkgdksa dks
Sol. (a)
bl ijh{kk dh mÙkjiqfLrdk çkIr djus ds The Statement indirectly asserts that the vkdf"kZr djus ds fy, ,slk fd;k gSA bl
ckotwn] cksMZ dks nwjLFk gksrs gq, Hkh decision makers can work effectively to çdkj] lkoZtfud {ks= ds cSadksa dks ;k rks
laHkkoukvksa dk irk yxkuk pkfg,A eliminate poverty only if they get to futh {ks= ds cSadksa ds lkFk esy [kkus ds fy,
(a) Only I follows / dsoy Ia vuqlj.k djrk know the basic problem afflicting the C;kt dh nj dks de djuk pkfg, ;k mu
gSA poor people through interaction with fo'ks"krkvksa dks mtkxj djuk pkfg, tks mUgsa
(b) Only II follows / dsoy II vuqlj.k them. So, only I follows. futh cSadksa ls vyx djrh gSA rks] II ;k III
djrk gSA dFku vçR;{k :i ls ;g nkok djrk gS fd vuqlj.k djrk gSA
(c) Only III follows / dsoy III vuqlj.k fu.kZ;drkZ xjhch dks [kRe djus ds fy, 19. Statement :/ dFku
djrk gSA çHkkoh :i ls dke dj ldrs gSa] tc mUgsa It is reported that during the last
(d) Only I and II follow / dsoy I vkSj II xjhc yksxksa dks muds lkFk ckrphr ds ekè;e fortnight there has been three eases of
vuqlj.k djrs gSa A ls ihfM+r djus okyh cqfu;knh leL;k dk near miss situation involving two air
(e) All follow / lHkh vuqlj.k gSA irk pysA blfy,] dsoy I vuqlj.k djrk crafts over the busy city airport. In all
Sol. (d) gSA these cases both the aircrafts came
Students who work hard all the year 18. Statement :/ dFku perilously close to each other and could
round to perform well at the Board Many private sector banks have reduced avoid collision as the pilots acted just in
examination deserve to be evaluated invest rate on housing loans in time.
correctly by experts, and not comparison to public sector banks. ;g crk;k x;k gS fd fiNys ,d i[kokM+s ds
mechanically or by inexperienced dbZ futh {ks= ds cSadksa us lkoZtfud {ks= ds nkSjku 'kgj ds O;Lr gokbZvìs ij nks ok;q
people. Besides, examiners who shirk cSadksa dh rqyuk esa vkokl _.k ij fuos'k nj f'kYiksa ds lkFk fudVorÊ fLFkfr esa rhu
their duty of evaluating answer papers, dks de fd;k gSA lqxerk vkbZ gSA bu lHkh ekeyksa esa nksuksa
ought to be punished. So, both I and II Courses of Action / dk;Zokgh foeku ,d&nwljs ds lehi vk x, vkSj
follow, while III does not. Vdjko ls cps D;ksfa d ik;yVksa us le; jgrs
I. The case should be raised before the
tks Nk= cksMZ ijh{kk esa vPNk çn'kZu djus ds regulatory authority for investigation by gh dk;Zokgh dj nh FkhA
fy, lky Hkj dM+h esgur djrs gSa] mudk the public sector banks as they cannot Courses of Action / dk;Zokgh
ewY;kadu fo'ks"kKksa }kjk lgh rjhds ls fd;k follow such reduction.
tkuk pkfg,] u fd ;a=or~ ;k vuqHkoghu
350
Best App for Govt. Jobs : Neonclasses (Download Now) Neon Publications
I. The pilots of all the six air crafts the slums of western part of the town to LFkkuh; ç'kklu dks uqdlku dh lhek dk
involved in these incidents should other areas. vkdyu djus vkSj mipkjkRed mik;ksa dk
immediately be de-rostered. uxjikfydk çkfèkdj.k dks 'kgj ds if'peh lq>ko nsus ds fy, vfèkdkfj;ksa dh ,d Vhe
bu ?kVukvksa esa 'kkfey lHkh Ng gokbZ f'kYiksa fgLls dh >qfXx;ksa esa jgus okys lHkh yksxksa dks Hkstuh pkfg,A
ds ik;yVksa dks rqjar gVkdj nwljh txg vU; {ks=ksa esa LFkkukarfjr djus ds fy, dne Sol. (c)
LFkkuakrfjr fd;k tkuk pkfg,A mBkuk pkfg,A Either action I or II follows because if
II. Some flights should be diverted to (a) Only I follows/ dsoy Ia vuqlj.k djrk fisherman did not construct their huts
other airports for the next few months gSA along the coast line then the loss due to
to decongest the sky over the city (b) Only II follows / dsoy II vuqlj.k huge tidal waves may be less. But in
airport. djrk gSA present situation, the local
dqN gokbZvìksa dks vxys dqN eghuksa ds fy, (c) Only I and II follow / dsoy I vkSj II administration should send a team of
'kgj ds gokbZvìs ij vkdk'k dks NksVk djus vuqlj.k djrs gSa A officials to assess the extent of damage
ds fy, vU; gokbZ vìksa ij Hkstk tkuk (d) All I, II and III follow/ lHkh I] II vkSj and suggest remedical measures.
pkfg,A III vuqlj.k djrs gSa A
;k rks dk;Zokgh I ;k II vuqlj.k djrh gS
III. Air traffic controllers of the city (e) None of these/ buesa ls dksbZ ugha
D;ksfa d ;fn eNqvkjs rV js[kk ds lkFk viuh
airport should be sent for refresher Sol. (d)
>ksifM+;ksa dk fuekZ.k ugha djrs gSa rks fo'kky
courses in batches to prepare All the three course of action are Tokj dh ygjksa ds dkj.k uqdlku de gks
themselves to handle such pressure suitable for pursuing. ldrk gSA ysfdu orZeku fLFkfr esa] LFkkuh;
situation. rhuksa dk;Zokgh mi;qä gSA ç'kklu dks uqdlku dh lhek dk vkdyu
'kgj ds gokbZ vìs ds ok;q ;krk;kr fu;a=dksa 21. Statement :/ dFku djus vkSj mipkjkRed mik;ksa dk lq>ko nsus
dks ,sls ncko dh fLFkfr ls fuiVus ds fy, The rate of inflation has reached its ds fy, vfèkdkfj;ksa dh ,d Vhe Hkstuh
[kqn dks rS;kj djus ds fy, cSpksa esa fjÝs'kj highest in last twenty years and there is pkfg,A
dk;Zokghksa ds fy, Hkstk tkuk pkfg,A
(a) Only I follows / dsoy I vuqlj.k djrs
no sign of it softening in the coming
months.
Statement and
gSa A
(b) Only II follows / dsoy II vuqlj.k
eqækLQhfr dh nj fiNys chl o"kksZa esa vius Inference
mPpre Lrj ij igqap xbZ gS vkSj vkus okys 1. Direction : A statement is given
djrs gSa A eghuksa esa blds uje gksus dk dksbZ ladsr ugha followed by two inferences I and II.
(c) Only III follows / dsoy III vuqlj.k gSA You have to consider the statement to
djrs gSa A Courses of Action / dk;Zokgh be true even if it seems to be at
(d) Only II and III follow/ dsoy II vkSj I. Government should initiate steps like variance with commonly known facts.
III vuqlj.k djrs gSa A reducing Government taxes on essential You have to decide which of the given
(e) None of these / buesa ls dksbZ ugha commodities with immediate effect. inferences, If any, follow from the
Sol. (b) ljdkj dks vko';d oLrqvksa ij ljdkjh djksa given statement.
To avoid collisions, the flights should be dks rRdky çHkko ls de djus tSls dne funZs'k: ,d dFku ds ckn nks vuqeku I vkSj II
diverted to other airports mBkus pkfg,A fn, x, gSaA vkidks dFku dks lR; ekuuk gS
Vdjko ls cpus ds fy,] gokbZ tgktks dks II. Farmers should be asked by the Hkys gh og loZKkr rF;ksa ls fHkUu çrhr gksrk
vU; gokbZ vìksa ij Hkst fn;k tkuk pkfg, Government to sell their produce at gksA vkidks r; djuk gS fd fn, x, dFkuksa
20. Statement :/ dFku lower prices. esa ls dkSu lk vuqeku] ;fn dksbZ gks] fn, x,
Many people living in the slums of fdlkuksa ls dgk tk, fd os viuh mit dks dFku dk vuqlj.k djrk gSA
western part of the town are diagnosed de dherksa ij cspsA Statement /dFku%
to be suffering from malaria. Sol. (e) National Expressway – A is the widest
'kgj ds if'peh fgLls dh >qfXx;ksa esa jgus Clearly both the Courses of Action are expressway and is used by few
okys dbZ yksx eysfj;k ls ihfM+r gSA appropriate for pursuing. people. When people find an empty
Courses of Action / dk;Zokgh Li"V :i ls nksuksa dk;Zokgh mi;qä gSA and wide road, people drive at high
I. The municipal corporation should 22. Statement :/dFku speed, which leads to accidents.
immediately make necessary A huge tidal wave swept away many us'kuy ,Dlçslos & A lcls pkSM+k ,Dlçslos
arrangements to spray mosquito fishing boats and hutments of the gS vkSj bldk bLrseky de yksx djrs gSaA
repellent in the affected area. fishermen living along the coast line. tc yksx [kkyh vkSj pkSM+h lM+d ns[krs gSa rks
uxj fuxe dks çHkkfor {ks= esa ePNjksa ls ,d fo'kky Tokj dh ygj dbZ eNyh idM+us yksx rst xfr ls okgu pykrs gSa] ftlls
cpkus okyh Øhe ds fNM+dko dh rRdky okyh ukoksa vkSj rV js[kk ds fdukjs jgus okys nq?kZVuk,a gksrh gSaA
O;oLFkk djuh pkfg,A eNqvkjksa dh >ksifM+;ksa esa cg xbZA Inference/vuqeku%
II. The municipal authority should Courses of Action / dk;Zokgh I. All accidents happens due to over
immediately make necessary I. The fishermen should henceforth be speeding.
arrangements to provide quick medical restrained from constructing their huts lHkh nq?kZVuk,a vfèkd xfr ds dkj.k gksrh gSaA
help to affected people. along the cost line. II. High speed increases the risk of
çHkkfor yksxksa dks Rofjr fpfdRlk lgk;rk eNqvkjksa dks leqnz lhek ds lkFk viuh >ksiM+h accident.
çnku djus ds fy, uxjikfydk çkfèkdj.k cukus ls jksdk tkuk pkfg,A rst xfr ls nq?kZVuk dk [krjk c<+ tkrk gSA
dks rqjar vko';d O;oLFkk djuh pkfg,A II. The local administration should send (a) If only I follows
III. The municipal authority should take a team of officials to assess the extent of ;fn dsoy I vuqlj.k djrk gSA
steps to shift all the people staying in damage and suggest remedial measures. (b) If both I and II follow

351
Download Free PDFs & e-Books from Neon Classes App web. : www.neonclasses.com
;fn I vkSj II nksuksa vuqlj.k djrs gSa good effect on water and human health. funs’Z k % ,d dFku ds ckn nks vuqeku I vkSj
(c) If only II follows Hence Inference I and II follow. II fn, x, gSaA ;g ekurs gq, fd fn;k x;k
;fn dsoy II vuqlj.k djrk gS 3. Directions : A statement is given dFku lR; gS] Hkys gh ;g lkekU; :Ik ls
(d) If neither I nor II follows followed by two inferences I and II. Kkr rF;ksa ls fHkUu izrhr gksrk gks] r; djsa
;fn u rks I vkSj u gh II vuqlj.k djrk gS You have to consider the statement to fd fn, x, vuqekuksa esa ls dkSu&lk@ls
Sol. (c) be true even if it seems to be at vuqeku dFku dk rkfdZd :Ik ls vuqlj.k
Inference I is not follows because in variance with commonly known facts. djrk gS@djrs gSaA
statement not talking about that all You have to decide which of the given Statement /dFku%
accident happens only due to over inferences, if any, follow from the
In a football match between Team -X
speeding. given statement.
and Team -Y, the total number of
Inference II is says, high speed increases funZs'k: ,d dFku ds ckn nks vuqeku I vkSj II
goals scored by Team -X was 5, out of
the risk of accident and it clearly related fn, x, gSaA vkidks dFku dks lR; ekuuk gS which 3 goals were scored by the left
to statement so it follows. Hkys gh og loZKkr rF;ksa ls fHkUu çrhr gksrk footed player.
2. Directions : A statement is given gksA vkidks r; djuk gS fd fn, x, dFkuksa Vhe – X vkSj Vhe – Y ds chp ,d QqVckWy
followed by two inferences I and II. esa ls dkSu lk vuqeku] ;fn dksbZ gks] fn, x, eSp esa] Vhe – X }kjk fd, x, xksyksa dh dqy
You have to consider the statement to dFku dk vuqlj.k djrk gSA la[;k 5 Fkh] ftuesa ls 3 xksy ck,¡ iSj okys
be true even if it seems to be at Statement /dFku% f[kykM+h }kjk fd, x, FksA
variance with commonly known facts. Many students are addicted to mobile Inference/vuqeku%
You have to decide which of the given games and this leads to poor I. The left footed player was adept at
inferences, if any, follow from the academic performance. scoring goals.
given statement. dbZ Nk= eksckby xsEl ds vknh gSa vkSj blls
funZs'k: ,d dFku ds ckn nks vuqeku I vkSj II ck,¡ iSj okyk f[kykM+h xksy djus esa ekfgj
vdknfed çn'kZu [kjkc gksrk gSA FkkA
fn, x, gSaA vkidks dFku dks lR; ekuuk gS Inference/vuqeku% II. 60% of the players in Team Y are
Hkys gh og loZKkr rF;ksa ls fHkUu çrhr gksrk I. Many students are not paying left footed.
gksA vkidks r; djuk gS fd fn, x, dFkuksa attention to studies due to mobile Vhe – Y ds 60% f[kykM+h ck,¡ iSj okys gSaA
esa ls dkSu lk vuqeku] ;fn dksbZ gks] fn, x, games. (a) Both I and II follow.
dFku dk vuqlj.k djrk gSA dbZ Nk= eksckby xse ds dkj.k i<+kbZ ij I vkSj II, nksuksa vuqlj.k djrs gSaA
Statement /dFku% è;ku ugha ns jgs gSaA (b) Only II follows.
Extensive use of pesticides in II. It is only because of mobile games dsoy II vuqlj.k djrk gSA
agricultural production contaminates that students fail in the examination. (c) Neither I nor II follows.
soil and water, remains in the crops eksckby xse dh otg ls gh Nk= ijh{kk esa u rks I vkSj u gh II vuqlj.k djrk gSA
and finally enter food chain, thereby Qsy gks tkrs gSaA (d) Only I follows.
posing threat to the human beings. (a) If only II follows dsoy I vuqlj.k djrk gSA
—f"k mRiknu esa dhVuk'kdksa dk O;kid ;fn dsoy II vuqlj.k djrk gS Sol. (c)
mi;ksx feêh vkSj ikuh dks nwf"kr djrk gS] (b) If neither I nor II follows Here the total number of goals scored by
;g Qlyksa esa cuk jgrk gS vkSj varr% [kk| ;fn u rks I vkSj u gh II vuqlj.k djrk gS team X and team Y is not the same. 5
J`[a kyk esa ços'k dj tkrk gS] ftlls euq"; ds (c) If both I and II follows goals are scored by only team Y. Out of
fy, [krjk iSnk gks tkrk gSA ;fn I vkSj II nksuksa vuqlj.k djrs gSa which 3 goals have been scored by the
Inference/vuqeku% (d) If only I follows left footed player.
I. Excessive use of pesticides in ;fn dsoy I vuqlj.k djrk gS I. When a left-footed player is good at
agricultural production is not good Sol. (d) scoring goals, he has scored only 3 out
for people. In statement: saying about students are of 5 goals.
—f"k mRiknu esa dhVuk'kdksa dk vR;fèkd addicted to mobile games and this is the II. Team Y has 60% left-footed players.
mi;ksx yksxksa ds fy, vPNk ugha gSA reason of their poor academic The above both inference does not
II. Excessive use of pesticides in performance. explain the statement.
agricultural production can have a In Inference I clearly seen that mobile 5. Direction: In each of the following
bad effect on water. games are the bad reason of not paying question, some statements are given.
—f"k mRiknu esa dhVuk'kdksa ds vR;fèkd attention to studies. So, Inference I is They may be inferred from the main
mi;ksx ls ikuh ij cqjk çHkko iM+ ldrk gSA correct. statement or not. Read both the
(a) If only II follows In Inference II saying about fail in the statements carefully and mark:
;fn dsoy II vuqlj.k djrk gS examination but in statement not saying funs'Z k% fuEufyf[kr çR;sd ç'u esa dqN dFku
(b) If neither I nor II follows about fail so Inference II not follows. fn, x, gSaA eq[; dFku ls mudk vuqeku
;fn u rks I vkSj u gh II vuqlj.k djrk gS 4. Directions : A statement is given yxk;k tk ldrk gS ;k ugÈA nksuksa dFkuksa
(c) If only I follows followed by two inferences I and II. dks è;kuiwoZd if<+, vkSj fpfUgr dhft,%
vxj dsoy I vuqlj.k djrk gS You have to consider the statement to Statement/dFku%
(d) If both I and II follows be true even if it seems to be at
There is a general protest from the
;fn I vkSj II nksuksa vuqlj.k djrs gSa variance with commonly known facts.
inhabitants of a certain area that the
Sol. (d) You have to decide which of the given
report submitted by a on riots in that
Excessive use of pesticides in inferences, if any, follow from the
area has been fabricated, based on
agricultural production does not have a given statement.
prejudices and has present
commission whole event.

352
Best App for Govt. Jobs : Neonclasses (Download Now) Neon Publications
dFku% ,d fuf'pr {ks= ds fuokfl;ksa dk ,d esgqy pkSsdlh dks idM+s tkus ls cpus ds (c)Only III follows
lkekU; fojksèk gS fd ml {ks= esa naxksa ij ,d fy, fdlh nwljs ns'k esa 'kj.k ysuh pkfg,A dsoy III vuqlj.k djrk gS
çLrqr dh xà fjiksVZ iwokZxzgksa ds vkèkkj ij III. The Indian High Commissioner (d)Only IV follows
x<+h xà gS vkSj vk;ksx us iwjh ?kVuk dks should himself go and arrest Mehul dsoy IV vuqlj.k djrk gS
çLrqr fd;k gSA Choksi. (e)None follows
Inference/vuqeku% Hkkjrh; mPpk;qä dks Lo;a tkdj esgqy dksà vuqlj.k ugÈ djrk
I. Another inquiry commission should pkSsdlh dks fxj¶rkj djuk pkfg,A Sol. (a)
be sent for proper investigations. (a) If only I follows Obviously, Inference I follows. If most of
mfpr tkap ds fy, ,d vkSj tkap vk;ksx vxj dsoy I vuqlj.k djrk gS the dresses in that shop are expensive,
Hkstk tkuk pkfg,A (b) If only II follows then some (part of the most) dresses in
II. The members of commission ;fn dsoy II vuqlj.k djrk gS that shop must be expensive. Hence, the
should be severely punished. (c) If both I and II follow correct option is A.
vk;ksx ds lnL;ksa dks dM+h ls dM+h ltk ;fn I vkSj II nksuksa vuqlj.k djrs gSa 8. In the following question, a statement
feyuh pkfg,A (d) If both II and III follow is given followed by four Inferences.
III. The next commission should ;fn II vkSj III nksuksa vuqlj.k djrs gSa You have to consider the statement to
consist of inhabitants of that area (e) If none of them follow be true even if it seems to be at
only. vxj muesa ls dksà Hkh ikyu ugÈ djrk gS variance from commonly known
vxys vk;ksx esa ml {ks= ds fuokfl;ksa dk Sol. (e) facts. You have to decide which of the
Explanation: given Inferences can definitely be
gh lekos'k gksuk pkfg,A
None of the courses of action follow. All drawn from the given statement.
(a) If only I follows
of them contain a lot of extraneous Indicate your answer.
vxj dsoy I vuqlj.k djrk gS
information and are impractical fuEufyf[kr ç'u esa ,d dFku ds ckn pkj
(b) If only II follows
solutions. vuqeku fn, x, gSaA vkidks dFku dks lR;
;fn dsoy II vuqlj.k djrk gS
7. In the following question, a statement ekuuk gS Hkys gh og loZKkr rF;ksa ls fHkUu
(c) If both I and II follows
;fn I vkSj II nksuksa vuqlj.k djrs gSa is given followed by four Inferences. çrhr gksrk gksA vkidks r; djuk gS fd fn,
(d) If both II and III follows You have to consider the statement to x, dFkuksa esa ls dkSu lk vuqeku fuf'pr
;fn II vkSj III nksuksa vuqlj.k djrs gSa be true even if it seems to be at :i ls fudkyk tk ldrk gSA viuk mÙkj
(e) If none of them follows variance from commonly known bafxr djsAa
;fn muesa ls dksà Hkh vuqlj.k ugÈ djrk gS facts. You have to decide which of the Statement/dFku%
given Inferences can definitely be A large number of language
Sol. (e)
drawn from the given statement. specialists write novels.
Explanation:
Indicate your answer. cM+h la[;k esa Hkk"kk fo'ks"kK miU;kl fy[krs
I does not follow because it may cause a
chain reaction. II is negative inference
fuEufyf[kr ç'u esa ,d dFku ds ckn pkj gSaA
because without the proper vuqeku fn, x, gSaA vkidks dFku dks lR; Inferences/vuqeku%
investigation the punishment will not be ekuuk gS Hkys gh og loZKkr rF;ksa ls fHkUu I. Some language specialists do not
the correct action, hence II does not çrhr gksrk gksA vkidks r; djuk gS fd fn, write novels.
follow. III also leads to biased report. x, dFkuksa esa ls dkSu lk vuqeku fuf'pr dqN Hkk"kk fo'ks"kK miU;kl ugÈ fy[krs gSaA
6. Statement/dFku% :i ls fudkyk tk ldrk gSA viuk mÙkj II. A novelist need not be a language
The Indian High Commission in bafxr djsAa specialist.
Antigua and Barbuda had asked the Statement/dFku% ,d miU;kldkj dks Hkk"kk fo'ks"kK gksus dh
authorities there to stop billionaire Most dresses in that shop are vko';drk ugÈ gSA
diamond trader Mehul Choksi from expensive. III. Language speciality is highly
travelling around the world, sources ml nqdku esa T;knkrj diM+s egaxs gSaA helpful to write novels.
said on Monday. Inferences/vuqeku% miU;kl fy[kus esa Hkk"kk dh fo'ks"krk
,aVhxqvk vkSj ckjcqMk esa Hkkjrh; mPpk;ksx us I. Some dresses in that shop are vR;fèkd lgk;d gksrh gSA
ogka ds vfèkdkfj;ksa ls vjcifr ghjk O;kikjh expensive. IV. All novelists are language
esgqy pkSsdlh dks nqfu;k Hkj esa ;k=k djus ls ml nqdku esa dqN diM+s egaxs gSaA specialists.
jksdus ds fy, dgk Fkk] lw=ksa us lkseokj dks II. There are cheap dresses also in lHkh miU;kldkj Hkk"kk fo'ks"kK gSaA
dgkA that shop. (a)Only I is appropriate
Inference/vuqeku% ml nqdku esa lLrs diM+s Hkh feyrs gSaA dsoy I gh mi;qä gS
I. All billionaire diamond traders III. Handloom dresses in that shop (b)Only II is appropriate
should immediately stop travelling to are cheap. dsoy II mi;qä gS
Antigua and Barbuda for the time ml nqdku esa gSaMywe ds diM+s lLrs feyrs (c)Only III is appropriate
being. gSaA dsoy III mi;qä gS
lHkh vjcifr ghjk O;kikfj;ksa dks fQygky IV. There are no cheap dresses (d)Only IV is appropriate
,aVhxqvk vkSj ckjcqMk dh ;k=k rqjar can dj available in that shop. dsoy IV mi;qä gS
nsuh pkfg, ml nqdku esa lLrs diM+s ugÈ feyrsA (e)All are appropriate/lHkh mi;qä gSa
II. Mehul Choksi should seek asylum (a) Only I follows Sol. (c)
in some other country to avoid being dsoy I vuqlj.k djrk gS The third inference is the most
caught. (b)Only II follows appropriate.
dsoy II vuqlj.k djrk gS

353
Download Free PDFs & e-Books from Neon Classes App web. : www.neonclasses.com
In order to write some pieces of commonly known facts. You are to vkuan d‚yksuh esa fctyh vkiwÆr dy 12 cts
literature, it is necessary to have decide which of the given ds ckn rhu ?kaVs ds fy, dV tk,xh D;ksfa d
command over language. Hence, the conclusions/inferences can definitely ejEer dk dke py jgk gSAÞ
correct option is C. be drawn from the given statement. Inferences/ vuqeku%
9. In the following question, two Indicate your answer. I. Residents of Anand colony may use
statements are given followed by four ,d dFku ds ckn vuqeku@vuqeku fn, x, their electrical appliances before 12
Inferences. You have to consider the gSaA vkidks dFku dks lR; ekuuk gS Hkys gh o"clock tomorrow.
statements to be true even if they og loZKkr rF;ksa ls fHkUu çrhr gksrk gksA vkuan d‚yksuh ds fuoklh dy 12 cts ls
seems to be at variance from vkidks r; djuk gS fd fn, x, dFku esa ls igys vius fctyh ds midj.kksa dk mi;ksx
commonly known facts. You have to dkSu lk vuqeku@vuqeku fuf'pr :i ls dj ldrs gSaA
decide which of the given Inferences fudkyk tk ldrk gSA viuk mÙkj bafxr II. Residents of Anand colony need
can definitely be drawn from the djsAa training for using electricity
given statement. Indicate your Statements/dFku% economically.
answer. vkuan d‚yksuh ds fuokfl;ksa dks fctyh dk
I. All the watches sold in that shop are
fuEufyf[kr ç'u es]a nks dFkuksa ds ckn pkj made of white metal. vkÆFkd :i ls mi;ksx djus ds fy,
vuqeku fn, x, gSaA vkidks dFkuksa dks lR; ml nqdku esa csph tkus okyh lHkh ?kfM+;k¡ çf'k{k.k dh vko';drk gSA
ekuuk gS Hkys gh os loZKkr rF;ksa ls fHkUu lQsn èkkrq ls cuh gSaA
çrhr gksrs gksAa vkidks r; djuk gS fd fn, II. Some of Rico watches are sold in (a)Only inference I follows
x, dFkuksa esa ls dkSu lk vuqeku fuf'pr that shop. dsoy vuqeku I vuqlj.k djrk gS
:i ls fudkyk tk ldrk gSA viuk mÙkj ml nqdku esa jhdks dh dqN ?kfM+;k¡ fcdrh (b)Only inference II follows
bafxr djsAa gSaA dsoy vuqeku II vuqlj.k djrk gS
Statements/ dFku% Inferences/vuqeku% (c)Both the inferences follow
(1) India is becoming industrialised. 1.All watches of white metals are nksuksa vuqeku vuqlj.k djrs gSa
Hkkjr vkS|ksxh—r gksrk tk jgk gSA manufactured by Rico. (d)Either a or b
(2) Pollution is a problem associated lQ+ns èkkrqvksa dh lHkh ?kfM+;k¡ fjdks }kjk ;k rks A ;k B
with industrialisation. fuÆer dh tkrh gSaA (d)None of the inferences follows
çnw"k.k vkS|ksxhdj.k ls tqM+h ,d leL;k gSA 2. Some of Rico watches are of white dksà Hkh vuqeku vuqlj.k ugÈ djrk gS
Inferences/ vuqeku% metal. Sol. (a)
I. All industrial centers are polluted. jhdks dh dqN ?kfM+;k¡ lQsn èkkrq dh gksrh The given statement is a notice for the
lHkh vkS|ksfxd dsæa çnwf"kr gSaA gSaA residents of Anand Colony to inform
II. India is polluted. 3. None of Rico watches are of white them about the electricity cut so that
Hkkjr çnwf"kr gSA metal. they use their electric appliances before
III. Polluted nations are jhdks dh dksà Hkh ?kM+h lQsn èkkrq dh ugÈ 12 PM clock. The statement has no
industrialised. gSA reference to whether or not the
çnwf"kr jk"Vª vkS|ksxh—r gSaA 4. Some of Rico watches of white residents are using electricity
IV. India may become polluted. metal are sold in that shop. economically.
Hkkjr çnwf"kr gks ldrk gS ml nqdku esa lQsn èkkrq dh dqN jhdks 12. Statement is given followed by few
(a)All are appropriate/lHkh mi;qä gSa ?kfM+;k¡ fcdrh gSaA inferences as given in the options.
(b)None is appropriate/ (a)1 and 2 inferences only You have to consider the statement to
dksà mfpr ugÈ gS dsoy 1 vkSj 2 vuqeku be true even if it seems to be at
(c)Only IV is appropriate (b)2 and 4 inferences only variance from commonly known
dsoy IV mi;qä gS dsoy 2 vkSj 4 vuqeku facts. You have to decide which of the
(d)Only II is appropriate given inferences, if any, follow from
(C)1 and 3 inferences only
dsoy II mi;qä gS the given statement.
dsoy 1 vkSj 3 vuqeku
(e)Only III is appropriate dFku ds ckn dqN vuqeku fn, x, gSa tSlk
(d)1 and 4 inferences only
dsoy III mi;qä gS dsoy 1 vkSj 4 vuqeku fd fodYiksa esa fn;k x;k gSA vkidks dFku
Sol. (c) (e)All the given inferences
dks lR; ekuuk gS Hkys gh og loZKkr rF;ksa
Only Inference IV is appropriate. The fn, x, lHkh vuqeku ls fHkUu çrhr gksrk gksA vkidks r; djuk gS
use of term 'all' in the Inference I makes Sol. (b) fd fn, x, dFkuksa esa ls dkSu lk vuqeku]
it invalid. From a general statement, we Since, All the watches sold in that shop ;fn dksà gks] fn, x, dFku dk vuqlj.k
cannot derive definite Conclusion. There are made of white metal and Some of djrk gSA
are many factors responsible for Rico watches are sold in that shop, Some Statement/ dFku% "If a person is
pollution. Therefore, we cannot say that of the Rico watches must be made of rich, he has a lot of influence."
polluted nations are industrialised. As white metal. dFku% Þ;fn dksà O;fä vehj gS] rks mldk
pollution is a problem associated with Hence, 2 and 4 inferences only follow cgqr çHkko gSAÞ
industrialisation, India may become the given statements. What inference can you draw from
polluted. Hence, the correct option is (c). 11. Statement/ dFku% the above statement?
10. A statement is given followed by "Electric supply in Anand Colony will mijksä dFku ls vki D;k vuqeku fudky
conclusions/inferences. You have to be cut tomorrow after 12 o"clock for ldrs gSa\
consider the statement to be true three hours because repairing work Inferences/ vuqeku%
even if it seems to be at variance from will be carried out."•

354
Best App for Govt. Jobs : Neonclasses (Download Now) Neon Publications
I.Kamala is rich, so she has a lot of (c) True/ lgh the statement that it happens generally
influence. (d) Can't say/ ugha dg ldrs but not always.
deyk vehj gSa] blfy, mudk dkQh ncnck Sol. (d) Hence, probably true.
gSA There are many other ingredients that 5. Taj is in Agra. Agra is in India.
II.Poor people cannot have influence. can cause the white colour and rice Therefore, the Taj is in India.
xjhc yksxksa dk çHkko ugÈ gks ldrkA contains various kinds of nutrients and rkt vkxjk esa gSA vkxjk Hkkjr esa gSA
III.Ram has a lot of influence, so he is minerals . So, we cannot say that white blfy,] rkt Hkkjr esa gSA
rich. colour is due to the calcium. (a) True/ lgh
jke dk cgqr çHkko gS] blfy, og èkuh gSaA (b) Probably false/ laHkor% xyr gS
IV.Govind is not rich, so he does not 2. 1. Water boils at 100 degree C (c) False/ xyr
have a lot of influence. ikuh 100 fMxzh lsfYl;l ij mcyrk gSA (d) Can't say/ ugha dg ldrs
xksfoUn èkuoku ugÈ gS] blfy, mldk çHkko 2. Water freezes at 0 degree C Sol. (a)
vfèkd ugÈ gSA ikuh 0 fMxzh lsfYl;l ij te tkrk gSA According to question, Taj is in Agra and
V.None of these/buesa ls dksà ugÈ Conclusion: At low pressure, water Agra is in India. So it is absolutely true
Sol. (a) boils at lower temperature that the Taj is in India.
Inference 1: Kamala is rich, so she has a The conclusion drawn is . 6. Is Cube a polygon?
lot of influence. This option is similar to fu"d"kZ:- de ncko ij] ikuh de rkieku ij D;wc ,d cgqHkqt gS\
the given statement, thus this option has mcyrk gS fu"d"kZ fudkyk x;k gS (a) TRUE/lgh (b) FALSE/xyr
to be true. (a) True/ lgh (b) false/ xyr gS Sol. (b)
vuqeku 1% deyk èkuh gS] blfy, mldk (c) Either probably true or probably FALSE
cgqr çHkko gSA ;g fodYi fn, x, dFku ds false False. Cube is not a Polygon.
leku gS] bl çdkj ;g fodYi lR; gksuk ;k rks 'kk;n lp gS ;k 'kk;n xyr gS Because polygon is a two dimensional
pkfg,A (d) Irrelavent/ vfu;fer figure and can be drawn on a flat surface
Inference 2: Poor people cannot have Sol. (d) Given statements that water boils at whereas Cube is three dimensional
influence. This may be true but there is 100 degree and freeze at 0 degree figure and cannot be drawn on a flat
no given statements establishing this doesn't comment anything on pressure surface.
fact definitely thus this option is wrong. dependence of boiling and freezing 7. 21st letter of the alphabet is a
vuqeku 2% xjhc yksxksa dk çHkko ugÈ gks points. Hence, the given conclusion is Consonant.
ldrkA ;g lp gks ldrk gS ysfdu fuf'pr not satisfied by any of them. It is o.kZekyk dk 21 oka v{kj ,d O;atu gSA
:i ls bl rF; dh iqf"V djus okyk dksà irrelavent. (a) TRUE/lgh (b) FALSE/xyr
Hkh dFku ugÈ gS blfy, ;g fodYi xyr 3. 1. Oxygen is a gas./v‚Dlhtu ,d xSl gSA Sol. (b)
gSA 2. This cylinder contains gas./bl FALSE
Inference 3: Ram has a lot of influence, flysM
a j esa xSl gSA We know there are 26 alphabets in
so he is rich. This may be true but there Conclusion: This cylinder contains English. The 21st letter in the alphabet
is no given statements establishing this Oxygen. The conclusion drawn is. is 'U'.
fact definitely thus this option is wrong. fu"d"kZ: bl flysaMj esa v‚Dlhtu gksrk gS We have 5 vowels and 21 consonants.
vuqeku 3% jke dk çHkko cgqr gS] blfy, fu"d"kZ fudkyk x;k gSA Vowels :: A, E, I, O, U.
og èkuh gSA ;g lp gks ldrk gS ysfdu (a) True/ lp U is not consonant and is a vowel.
fuf'pr :i ls bl rF; dh iqf"V djus (b) Probably true/'kk;n lp gS Hence, the given statement that 21st
(c) False/ xyr letter in the alphabet i.e, U is consonant
okyk dksà Hkh dFku ugÈ gS blfy, ;g
is a FALSE.
fodYi xyr gSA (d) Irrelavent/ vfu;fer
8. Which sentence is written correctly?
Inference 4: Govind is not rich, so he Sol. (b) Probably true
dkSu lk okD; lgh fy[kk x;k gS\
does not have a lot of influence. This It is not certain that cylinder contained
(a) They never watch no movie during
may be true but there is no given gas is oxygen , it might be nitrogen
the working days, but in holidays they
statements establishing this fact cylinder or may be some other gas .
watch one movie.
definitely thus this option is wrong. 4. People who are bald are generally of
the intellectual type. Arun is bald.
os dke ds fnuksa esa dHkh dksbZ fQYe ugha
vuqeku 4% xksfoUn èkuoku ugÈ gS] blfy,
Therefore Arun is an intellectual. ns[krs gSa] ysfdu Nqfê;ksa esa os ,d fQYe ns[krs
mldk vfèkd çHkko ugÈ gSA ;g lp gks
xats yksx vkerkSj ij ckSf)d çdkj ds gksrs gSaA
ldrk gS ysfdu fuf'pr :i ls bl rF;
gSaA v#.k xatk gSA blfy, v#.k ,d (b) They never watch none movie
dh iqf"V djus okyk dksà Hkh dFku ugÈ gS
cqf)thoh gSA during the working days, but in holidays
blfy, ;g fodYi xyr gSA
they watch one movie.
(a) False/ xyr
Truth of statement (b) Probably true/'kk;n lp gS
os dke ds fnuksa esa dHkh dksbZ fQYe ugha
1. The presence of calcium in milk ns[krs gSa] ysfdu Nqfê;ksa esa os ,d fQYe ns[krs
(c) True/ lgh
makes it white. Rice, too, is white. gSaA
(d) Can't say/ ugha dg ldrs
Therefore rice also contains calcium (c) They don't never watch any movie
Sol. (b)
nw/k esa dSfY'k;e dh ekStwnxh bls lQsn during the working days, but in holidays
It is given in the statement that bald
cukrh gSA pkoy] Hkh] lQsn gSA blfy, they watch one movie.
people are generally intellectual and if
pkoy esa Hkh dSfY'k;e gksrk gSA os dke ds fnuksa esa dHkh dksbZ fQYe ugha
Arun is bald, we can probably say that
(a) False/ xyr he is an intellectual but we cannot say
ns[krs gSa] ysfdu Nqfê;ksa esa os ,d fQYe ns[krs
(b) Probably false/laHkor% xyr gS that it is 100% true because as given in
gSaA

355
Download Free PDFs & e-Books from Neon Classes App web. : www.neonclasses.com
(d) They watch no movie during the dqN rF;ghu yx ldrs gSaA bl vlko/kkuh 12. Which sentence is Punctuated
working days, but in holidays they dks vuns[kk djsa vkSj rkfdZd lq/kkjksa dks ns[ksAa correctly?
watch one movie. og dFku pqusa tks xyr ;k lafnX/k gks (a) The conference was only supposed
os dke ds fnuksa esa dksbZ fQYe ugha ns[krs gSa] (a) Ramesh is tall. Ramesh is a boy. to be an hour; however, it ran for nearly
ysfdu Nqfê;ksa esa os ,d fQYe ns[krs gSaA Therefore, boys are tall. two and half hours.
Sol. (d) jes'k yack gSA jes'k ,d yM+dk gSA blfy, (b) The conference was only supposed
They watch no movie during the yM+ds yacs gksrs gSaA to be an hour, however; it ran for nearly
working days, but in holidays they (b) All who can fly are animals. Some two and half hours.
watch one movie. birds can fly. Therefore, some birds are (c) The conference was only supposed
They watch no movie during the animals. to be an hour. However, it ran for nearly
working days, but in holidays they lHkh tks mM+ ldrs gSa os tkuoj gSaA dqN i{kh two and half hours.
watch one movie. is correctly written mM+ ldrs gSaA blfy,] dqN i{kh tkuoj gSaA (d) The conference was only supposed
among the given options. (c) Men live in houses. Houses grow on to be an hour; however it ran for nearly
9. Each of the following questions has trees. Trees float in water. Therefore, two and half hours.
four statements. Three are logically men float in water. Sol. (a)
correct. Some of which may look iq#"k ?kjksa esa jgrs gSaA isM+ksa ij ?kj c<+rs gSA The conference was only supposed to be
factually absurd. Ignore this an hour; however, it ran for nearly two
isM+ ikuh esa rSjrs gS] blfy,] iq#"k ikuh esa
absurdity and look to the logical and half hours.
rSjrs gSaA
corrections. Choose the statement The correct punctuated sentence is A
(d) All living things are mobile. Some
which is wrong or doubtful: The conference was only supposed to be
non-living things are mobile. Therefore,
fuEufyf[kr ç'uksa esa ls çR;sd esa dFku fn, an hour; however, it ran for nearly two
some mobile are living and some non-
x, gSaA rhu rkfdZd :i ls lgh gSaA ftuesa and half hours.
living things.
ls dqN rF;ghu yx ldrs gSaA bl 13. Which of the following statement(s)
lHkh thfor phtsa eksckby gSaA dqN futhZo
vlko/kkuh dks vuns[kk djsa vkSj rkfdZd is/are TRUE?
phtsa eksckby gSaA blfy,] dqN eksckby fuEufyf[kr esa ls dkSu lk dFku / dFku
lq/kkjksa dks ns[ksAa og dFku pqusa tks xyr ;k thfor gSa vkSj dqN xSj&thfor phtsa gSaA
lafnX/k gks TRUE gS / gS\a
Sol. (a)
(a) Birds fly in the air. Trees are birds. I. √11 + √7 < √10 + √8.
Ramesh is tall. Ramesh is a boy.
Therefore, trees fly in the air. II. √17 + √11 > √15 + √13
Therefore, boys are tall.
i{kh gok esa mM+rs gSaA isM+ i{kh gSaA blfy,] (a) Only I/ dsoy I
There is not any relation between boys
gok esa isM+ mM+rs gSaA (b) Only II/ dsoy II
and tall. So, we cannot say anything.
(b) Some boys steal. All who steal are (c) Both I and II/ I vkSj II nksuksa
11. Which of the following statement(s)
naughty. All naughty are honest. (d) Neither I nor II/ u rks I vkSj u gh II
is/are TRUE?
Therefore, some boys are honest fuEufyf[kr esa ls dkSu lk dFku lgh gSa\ Sol. (a) Clearly seen that Only I is true.
dqN yM+ds pksjh djrs gSaA pksjh djus okys I. The total number of positive factors 14. Which of the following options
lHkh 'kjkjrh gksrs gSaA lHkh 'kjkjrh bZekunkj of 72 is 12. correctly describes the given
gSaA blfy,] dqN yM+ds bZekunkj gksrs gSa 72 ds ldkjkRed dkjdksa dh dqy la[;k 12 statement?
(c) All girls like dance. Some girls are gSA fuEufyf[kr esa ls dkSu lk fodYi fn, x,
Indian. All Indians are artists. Therefore, II. The sum of first 20 odd numbers is
dFku dk lgh o.kZu djrk gSA
some artists like dance. 400. Statement:
lHkh yM+fd;ksa dks Mkal ilan gksrk gSA dqN igys 20 fo"ke la[;kvksa dk ;ksx 400 gSA There are four rooks in a game of
yM+fd;ka Hkkjrh; gSaA lHkh Hkkjrh; dykdkj chess.
III. Largest two digit prime number is 97.
gSaA blfy,] dqN dykdkjksa dks u`R; ilan gSA lcls cM+h nks vadksa dh çeq[k la[;k 97 gSA 'krjat ds [ksy esa pkj gkFkh gksrs gSaA
(d) All liars are not thieves. All thieves (a) Never/ dHkh ugha
(a) Only I and II/ dsoy I vkSj II
are criminals. Therefore, all liars are (b) Often/ vDlj
(b) Only II and III/ dsoy II vkSj III
criminals (c) Always/ ges'kk
(c) Only I and III/ dsoy I vkSj III
lHkh >wBs pksj ugha gSaA lHkh pksj vijk/kh gSaA (d) Sometimes/ dHkh&dHkh
(d) All are true./ lHkh lR; gSaA
blfy, lHkh >wBs vijk/kh gSa Sol. (c) Chess is a two-player board game
Sol. (d)
Sol. (d) and each player has two rooks. Hence,
Statement I:
All liars are not thieves. All thieves are there are always four rooks in the game
Positive factors of 72: 1, 2, 3, 4, 6, 8, 9,
criminals. Therefore, all liars are of chess.
12, 18, 24, 36, 72
criminals Total factors = 12
Since all liars are not thieves so all liars 15. If we are going early in the morning
So, Statement I is true.
cannot be criminals. towards the south the sun will be
Statement II:
10. Each of the following questions has visible at our left:
We know that sum of first n odd
four statements. Three are logically numbers = n2
;fn ge lqcg tYnh nf{k.k dh vksj tk jgs
correct. Some of which may look Now, sum of first 20 odd numbers = 202 =
gSa rks lw;Z gekjh ckbZa vksj fn[kkbZ nsxk :
factually absurd. Ignore this 400 (a) Always/ ges'kk (b) Never/ dHkh ugha
absurdity and look to the logical So, Statement II is also true. (c) Often/ vDlj (d) Sometimes/
corrections. Choose the statement Statement III: dHkh&dHkh
which is wrong or doubtful:- Largest two digit prime number is 97. Sol. (a)
fuEufyf[kr ç'uksa esa ls çR;sd esa pkj dFku So, Statement III is also true. Early in the morning the sun is in the
gSaA rhu rkfdZd :i ls lgh gSaA ftuesa ls direction of east. If we are going
356
Best App for Govt. Jobs : Neonclasses (Download Now) Neon Publications
towards the south, our face will be in the Hence, option (c) will be correct. fn, x, la;kstu dk ty esa fufeZr lgh
direction of South and our left hand will 4. Choose the alternative which is izfrfcac p;fur djsaA
be in the direction of east. Hence if we closely resembles the water-image of
go early in the morning towards the the given combination.
south the sun will always be visible at og fodYi pqusa tks fn, x, la;kstu dh
our left. ty&Nfo ds leku gSA
16. My ten years old niece is taller than
my twelve years old son:
esjs ckjg lky ds csVs dh rqyuk esa esjh nl
lky dh Hkrhth yach gS:
(a) 1 (b) 2 (c) 3 (d) 4
(a) Always/ ges'kk (b) Never/ dHkh ugha
Sol. (b) (a)
(c) Often/ vDlj (d) Sometimes/
Water image of ‘a’and ‘4’ is incorrect in
dHkh&dHkh options (a) & (c) , (d).
Sol. (d) Hence, option (b) will be correct.
Sometime younger children become 5. Select the correct water image of the
gain better height than elder children. given combination of letters. (b)
Water image v{kjksa ds fn, x, la;kstu dh lgh ikuh dh
Nfo dk p;u djsAa
1. Choose the alternative which is DFNZSR
(c)
closely resembles the water-image of (a)
the given combination. (b) (c) (d)
og fodYi pqusa tks fn, x, la;kstu dh (c)
ty&Nfo ds leku gSA (d)
(d) (c) (d)
Sol. (d) Sol. (b)
Water image of ‘D’and ‘R’ is incorrect in 9. Choose the correct water image
(a) 1 (b) 2 (c) 3 (d) 4 options (a) & (c) . of the question figure, fr om the
Sol. (d) Sequence of the word for water image is given answer figures.
Water image of ‘L’ and ‘U’ is incorrect in wrong in option (b). fn, x, la;kstu dk ty esa fufeZr lgh
options (b) & (c). Hence, option (d) will be correct. izfrfcac p;fur djsaA
Sequence of the word for water image is 6. Select the correct water image of the
wrong in option (a). given combination of number.
Hence, option (d) will be correct. la[;k ds fn, x, la;kstu dh lgh ikuh dh
Nfo dk p;u djsAa
2. Choose the alternative which is 8965423
closely resembles the water-image of (a)
the given combination. (b)
og fodYi pqusa tks fn, x, la;kstu dh (a) (b) (c) (d)
(c) Sol. (a)
ty&Nfo ds leku gSA
(d) 10. Choose the correct water i mage
of the question figure, fr om the
Sol. (c)
given answer figures.
Water image of ‘6’, ‘2’ and ‘9’ is incorrect
in options (a), (b) & (d) .
fn, x, la;kstu dk ty esa fufeZr lgh
(a) 1 (b) 2 (c) 3 (d) 4 Hence, option (c) will be correct. izfrfcac p;fur djsaA
Sol. (c) 7. Select the correct water image of the
Water image of ‘9’, ‘R’ and ‘S’ is incorrect given combination.
in options (a) , (b) & (d). fn, x, la;kstu dk ty esa fufeZr lgh
Hence, option (c) will be correct. izfrfcac p;fur djsaA
3. Choose the alternative which is
closely resembles the water-image of
the given combination. (a)
og fodYi pqusa tks fn, x, la;kstu dh (b) (a) (b) (c) (d)
ty&Nfo ds leku gSA Sol. (c)
(c)
11. Which of the answer figure is the
(d) water image of given figure?
Sol. (d) dkSu lh mÙkj vkd`fr iz’u vkd`fr dk ty
Water image of ‘5’, ‘1’ and ‘9’ is incorrect izfrfcEc gksxh\
in options (a), (b) & (c) . Question figure/iz’u vkd`fr
(a) 1 (b) 2 (c) 3 (d) 4
Hence, option (d) will be correct.
Sol. (c)
8. Select the correct water image of the
Water image of ‘1’and ‘3’ is incorrect in
given combination.
options (a) , (b) & (d).

357
Download Free PDFs & e-Books from Neon Classes App web. : www.neonclasses.com
Sol. (c)
14. Which of the answer figure is the
water image of given figure?
dkSu lh mÙkj vkd`fr iz’u vkd`fr dk ty
izfrfcEc gksxh\

Question figure/iz’u vkd`fr


Answer figure/ mÙkj vkd`fr M N
Answer figure/ mÙkj vkd`fr

(a) (b) (c) (d)


(a) (b) (c) (d)
Sol. (b) Answer figure/ mÙkj vkd`fr
12. Which of the answer figure is the Sol. (c)
water image of given figure? 17. Which of the answer figure is the
dkSu lh mÙkj vkd`fr iz’u vkd`fr dk ty water image of given figure?
izfrfcEc gksxh\ dkSu lh mÙkj vkd`fr iz’u vkd`fr dk ty
Question figure/iz’u vkd`fr izfrfcEc gksxh\
Question figure/iz’u vkd`fr

Sol. (a)
15. Which of the answer figure is the
water image of given figure?
dkSu lh mÙkj vkd`fr iz’u vkd`fr dk ty
izfrfcEc gksxh\
Question figure/iz’u vkd`fr
Answer figure/ mÙkj vkd`fr
Answer figure/ mÙkj vkd`fr

Answer figure/ mÙkj vkd`fr

(a) (b) (c) (d) PQR


PQR
PQR
RQP
Sol. (c)
18. Choose the correct water image of the
given figure (X) from amongst the
Sol. (d) four alternatives.
13. Which of the answer figure is the pkj fodYiksa esa ls nh xbZ vk—fr (X) dh
water image of given figure? (a) (b) (c) (d) lgh ikuh dh Nfo pqusAa
dkSu lh mÙkj vkd`fr iz’u vkd`fr dk ty
izfrfcEc gksxh\ Sol. (a)
16. Which of the answer figure is the
Question figure/iz’u vkd`fr mirror image of given figure? which
is based on MN.
(a) 1 (b) 2 (c) 3 (d) 4
dkSu lh mÙkj vkd`fr iz’u vkd`fr dk niZ.k
Sol. (b)
izfrfcEc gS] tcfd ni.kZ MN dh fLFkfr esa 19. Choose the correct water image of the
j[kk tkrk gSA given figure (X) from amongst the
Question figure/iz’u vkd`fr four alternatives.
pkj fodYiksa esa ls nh xbZ vk—fr (X) dh
Answer figure/ mÙkj vkd`fr lgh ikuh dh Nfo pqusAa

358
Best App for Govt. Jobs : Neonclasses (Download Now) Neon Publications
24. Which of the answer figure is the
water image of given figure?
dkSu lh mÙkj vkd`fr iz’u vkd`fr dk ty
izfrfcEc gksxh\

(a) (b) (c) (d)


(a) 1 (b) 2 (c) 3 (d) 4
Sol. (b) Sol. (b)
20. Choose the correct water image of the 28. Which of the answer figure is the
(X)
given figure (X) from amongst the water image of given figure?
four alternatives. dkSu lh mÙkj vkd`fr iz’u vkd`fr dk ty
pkj fodYiksa esa ls nh xbZ vk—fr (X) dh izfrfcEc gksxh\
lgh ikuh dh Nfo pqusAa

(a) (b) (c) (d)

Sol. (b) (X)


(a) 1 (b) 2 (c) 3 (d) 4 25. Which of the answer figure is the
Sol. (b) water image of given figure?
21. Choose the correct water image of the dkSu lh mÙkj vkd`fr iz’u vkd`fr dk ty
given figure (X) from amongst the izfrfcEc gksxh\
four alternatives.
pkj fodYiksa esa ls nh xbZ vk—fr (X) dh (a) (b) (c) (d)
lgh ikuh dh Nfo pqusAa
Sol. (c)
(X) 29. Which of the answer figure is the
water image of given figure?
dkSu lh mÙkj vkd`fr iz’u vkd`fr dk ty
(a) 1 (b) 2 (c) 3 (d) 4 izfrfcEc gksxh\
Sol. (b)
22. Choose the correct water image of the
given figure (X) from amongst the (a) (b) (c) (d)
four alternatives.
pkj fodYiksa esa ls fn, x, vkadM+s (X) dh Sol. (d) (X)
lgh ty izfrfcEc pqusAa 26. Which of the answer figure is the
water image of given figure?
dkSu lh mÙkj vkd`fr iz’u vkd`fr dk ty
izfrfcEc gksxh\

(a) (b) (c) (d)

Sol. (a)
(a) 1 (b) 2 (c) 3 (d) 4
(X) 30. Which of the answer figure is the
Sol. (a)
water image of given figure?
23. Which of the answer figure is the
dkSu lh mÙkj vkd`fr iz’u vkd`fr dk ty
water image of given figure?
dkSu lh mÙkj vkd`fr iz’u vkd`fr dk ty izfrfcEc gksxh\
izfrfcEc gksxh\

(a) (b) (c) (d)

Sol. (d) (X)


27. Which of the answer figure is the
(X)
water image of given figure?
dkSu lh mÙkj vkd`fr iz’u vkd`fr dk ty
izfrfcEc gksxh\

(a) (b) (c) (d)


(a) (b) (c) (d)
Sol. (c)
Sol. (a) (X) Word Analogy
359
Download Free PDFs & e-Books from Neon Classes App web. : www.neonclasses.com

1.Country & Capitals / (c) Assam : Jorhat


vle % tksjgV
ml fodYi dk p;u djsa ftlesa 'kCnksa ds
chp ogh lacaèk gSa tks fn, x, 'kCnksa ds ;qXe
ns’k ,oa jkt/kkuh (d) Meghalaya : Shillong
es?kky; % f’kykax
ds chp gSA
Tissues : Histology
1. Select the option that is related to the Sol. (d) Ård % fgLVksykWth
third word in the same way as the Ranchi is the capital of Jharkhand. (a) Insects : Malacology
second word is related to the first Likewise Shillong is the capital of dhV % eSykdksykWth
word. Meghalaya. (b) Amphibians : Herpetology
ml fodYi dk p;u djsa tks rhljs 'kCn ls
mlh çdkj lacafèkr gS tSls nwljk 'kCn igys 4.State and Court/ jkT; ,oa mHk;pj % gjisVksykWth
(c) Mountains : Onomatology
'kCn ls lacfa èkr gSA
Canada : Ottawa :: Cuba : ? U;k;ky; ioZr % vksukseSVksykWth
(d) Shells : Ornithology
dukMk % vksVkok %% D;wck % \ 4. Select the option that is related to the 'kSy % vksfuZFkksykWth
(a) Bern/cuZ (b) Madrid/eSfMªM third term in the same way as the Sol. (b)
(c) Oslo/vksLyks (d) Havana/gokuk second term is related to the first Histology is the branch of biology which
Sol. (d) term. studies the tissues.
Ottawa is the capital of Canada. ml fodYi dk p;u djsa tks rhljs in ls Similarly, Herpetology is the branch of
Similarly, Havana is the capital of Cuba. mlh çdkj lacafèkr gS tSls nwljk in igys zoology dealing with reptiles and
2.Country & Currency/ ns’k ,oa in ls lacafèkr gSA amphibians.

eqnzk
Uttar Pradesh : Allahabad High Court
: : Bihar : ? 7.Quantity and Unit ek=k ,oa
/
mÙkj izns’k % bykgkckn mPp U;k;ky; %%
2. Select the option that is related to fcgkj % \ bdkbZ
the third word in the same way as the (a) Nalanda High Court/ukyank mPp 7. Select the word-pair in which the two
second word is related to the first U;k;ky; words are related in the same way as
word. (The words must be (b) Patna High Court/iVuk mPp U;k;ky; are the two words in the following
considered as meaningful English (c) Gaya High Court/x;k mPp U;k;ky; word-pair.
words and must not be related to (d) Darbhanga High Court/njHkaxk mPp ml 'kCn&;qXe dk p;u djsa ftlesa nks 'kCn
each other based on the number of mlh çdkj lacfa /kr gSa tSls fuEufyf[kr
U;k;ky;
letters/number of 'kCn&;qXe esa nks 'kCn gSaA
Sol. (b)
consonants/vowels in the word). Resistance : Ohm
Allahabad high court is the high court of
ml fodYi dk p;u djsa tks rhljs 'kCn ls çfrjks/k % vksã
the state of uttae Pradesh.
mlh çdkj lacaf/kr gS tSls nwljk 'kCn igys (a) Time : Clock / le; % ?kM+h
Similarly, Patna high court is the high
'kCn ls lacaf/kr gSA ¼'kCnksa dks lkFkZd vaxzsth court of the state of Bihar. (b) Temperature : Thermometer
'kCn ekuk tkuk pkfg, vkSj 'kCn esa v{kjksa dh rkieku % FkekZehVj
la[;k/O;atuks/a Lojksa dh la[;k ds vk/kkj ij 5.River & City / ufn;ka ,oa 'kgj (c) Pressure : Barometer
,d nwljs ls lacaf/kr ugha gksuk pkfg,A½ 5. Select the option that is related to the ncko % cSjksehVj
Russia : Ruble :: Myanmar : ? third word in the same way as the (d) Length : Meter/ yackbZ % ehVj
:l : :cy :: E;kaekj : second word is related to the first Sol. (d)
(a) Dirham/fnjge (b) Baht/ckr word. Ohm is the unit of resistance.
(c) Kyat/D;kr (d) Peseta/islV
s k ml fodYi dk p;u djsa tks rhljs 'kCn ls Similarly, Meter is the unit of length.
Sol. (c)
Ruble is the currency of Russia.
mlh çdkj lacafèkr gS tSls nwljk 'kCn igys
'kCn ls lacfa èkr gSA 8.Alloy / feJ /kkrq
Likewise, Kyat is the currency of Varanasi : Ganga : : Mathura : ? 8. Select the word-pair in which the two
Myanmar. Okkjk.klh % xaxk % % eFkqjk % \ words are related in the same way as
3.State and Capital/ jkT; ,oa (a) Godavari/xksnkojh
(b) Ganga/xaxk
are the two words in the given word-
pair.

jkt/kkuh (c) Yamuna/;equk


(d) Narmada/ueZnk
ml 'kCn&;qXe dk p;u djsa ftlesa nks 'kCn
mlh çdkj lacafèkr gSa tSls fn, x,
3. Select the option in which the words Sol. (c) 'kCn&;qXe esa nks 'kCn gSaA
share the same relationship as that Varanasi is situated on the banks of the Steel : Iron
shared by the given pair of words. Ganga river. bLikr % yksgk
ml fodYi dk p;u djsa ftlesa 'kCnksa ds Similarly, Mathura is situated on the (a) Chromium : Silver
chp ogh lacaèk gSa tks fn, x, 'kCnksa ds ;qXe banks of the Yamuna river. Øksfe;e % pkanh
ds chp gSA (b) Brass : Copper
Jharkhand : Ranchi 6.Branch of Study/ v/;;u dh ihry % rkack
>kj[kaM % jkaph (c) Bronze : Iron
(a) Mizoram : Kohima 'kk[kk dkaL; % yksgk
fetksje % dksfgek 6. Select the option in which the words (d) Nickel : Tin
(b) Sikkim : Itanagar share the same relationship as that fudsy % fVu
flfDde % bZVkuxj shared by the given pair of words. Sol. (b)

360
Best App for Govt. Jobs : Neonclasses (Download Now) Neon Publications
Steel is an alloy of lron and Carbon. 'kCn ls lacfa /kr gS vkSj pkSFkk 'kCn rhljs 'kCn 14. Select the option in which the words
Similarly, Brass is an alloy of Copper and ls lacaf/kr gSA share the same relationship as that
Zinc. Asthma : Lungs :: Arthritis : Bones :: shared by the given pair of words.
ml fodYi dk p;u djsa ftlesa 'kCnksa ds
9.Vitamins / foVkfeUl Conjunctivitis : ?
nek : QsQMs+ :: xfB;k : gfÏ;k¡ :: chp ogh lacaèk gSa tks fn, x, 'kCnksa ds ;qXe
9. Select the option that is related to the us=’ys"eyk’kks/k : ? ds chp gSA
third word in the same way as the (a) Heart/ân; (b) Skin/Ropk Swan : Cygnet/gal % gal’kkod
second word is related to the first (c) Eyes/uS= (d) Brain/efLr"d (a) Turkey : Poult/is: i{kh % pwtk
word. Sol. (c) (b) Whale : Fry/Ogsy % eNYkh dk >qaM
ml fodYi dk p;u djsa tks rhljs 'kCn ls Asthma is a lung disease And Arthritis is (c) Deer : Joey/fgj.k % daxk:’kkod
mlh izdkj lacaf/kr gS tSls nwljk 'kCn igys a bone related disease. Similarly, (d) Fish : Cub/eNyh % i'kq’kkod
'kCn ls lacfa /kr gSA Conjunctivitis is a eye diseases. Sol. (a)
Night Blindness : Vitamin A :: Beri A baby swan is called a cygnet.
Beri : ?
jrkSa/kh : foVkfeu , : csjh&csjh : ? 12.Symbolic Relationship/ izrhd Similarly, a baby turkey is called poults.

(a) Vitamin B/ foVkfeu ch


(b) Vitamin A/ foVkfeu , laca/k 15.animal sounds/ tkuojksa dh
(c) Vitamin C/ foVkfeu lh 12. Select the word pair in which the two
(d) Vitamin D/ foVkfeu Mh words are related in the same way as vkokt
Sol. (a) the word pair given below. 15. Select the option in which the words
As Night blindness occurs due to ml 'kCn ;qXe dk p;u dhft, ftlesa nksuksa share the same relationship as that
deficiency of Vitamin A. 'kCn mlh çdkj lacafèkr gSa tSls uhps fn, x, shared by the given pair of words.
Likewise, beri beri occurs due to 'kCn ;qXe esa gSaA ml fodYi dk p;u djsa ftlesa 'kCnksa ds
deficiency of Vitamin B. Feet/iSj : Socks/eksts+ chp ogh lacaèk gSa tks fn, x, 'kCnksa ds ;qXe
10.Instrument & Measurement/ ;a= (a) Finger/maxyh : Nail/uk[kwu
(b) Head/flj : Turban/ixM+h
ds chp gSA
Horses : Neigh/?kksM+k % fgufgukuk

,oa ekiu (c) Legs/iSj : Vest/cfu;ku


(d) Abdomen/isV : Pants/iSaV
(a) Parrots : Bray /rksrk % pwj djuk
(b) Pigs : Buzz/lwvj % HkuHkukuk
10. Select the option in which the words Socks are worn on feet. (c) Hares : Boom/[kjxks’k % 'kksj epkuk
share the same relationship as that Similarly, turban is worn on head. (d) Ducks : Quack/cr[k % DoSd
shared by the given pah of words.
ml fodYi dk p;u djsa ftlesa 'kCn ogh [ksy
13.Games & Place of Playing/
Sol. (d)
The sound made by Horse is Neigh.
lacaèk lk>k djrs gSa tks fn, x, 'kCnksa ds pah
}kjk lk>k fd, x, gSaA ,oa [ksy lacaf/kr LFkku Similarly, The sound made by Ducks is
Quack.
Clock : Time 13. Select the option that is related to the
?kM+h % le; fifth word in the same way as the 16.List of Animals and their
(a) Balance : Scale
rjktw % Ldsy
second word is related to the first
word and the fourth word is related tkuojksa
Homes/ vkSj
(b) Taseometer : Wind to the third word.
VSlksehVj % gok ml fodYi dk p;u djsa tks ikaposa 'kCn ls muds ?kjksa dh lwph
(c) Anemometer : Strains mlh çdkj lacafèkr gS tSls nwljk 'kCn igys 16. Select the word-pair in which the two
,uheksehVj % LVªsu 'kCn ls lacafèkr gS vkSj pkSFkk 'kCn rhljs 'kCn words are related in the same way as
(d) Ammeter : Current ls lacafèkr gSA are the two words in the given pair.
vehVj % djaV Pitch : Cricket :: Court : Badminton :: ml 'kCn&tksM+h dk p;u djsa ftlesa nks 'kCn
Sol. (d) Ring : ? mlh rjg lacaf/kr gSa tSls fn, x, tksM+s esa nks
Clock is a mechanical or electrical device fip : fØdsV :: dksVZ : cSMfeaVu :: fjax : \ 'kCn gSaA
for measuring time. (a) Football/QqVcky (b) Rugby/jXch Lion : Forest/ flag : ou
Similarly, Ammeter is an instrument (c) Hockey/gkWdh (d) Boxing /ckWfDalx (a) Bee : Honey/ e/kqeD[kh : 'kgn
used to measure current in a circuit.
Sol. (d) (b) Shark : Ocean/'kkdZ : egklkxj
The game Cricket is played on pitch and (c) Flower : Honey/ Qwy : 'kgn
11.Disease and Organ / chekjh ,oa the game Badminton is played on Court.
Similarly, Boxing is played on ring.
(d) Sorrow : Love/ nq[k : çse
Sol. (b)
izHkkfor vax 14.Animals & Animal Baby As Lion is lives in forest. Likewise, Shark

11. Select the option that is related to the tkuoj


Names/ ,oa is lives in Ocean.
17.Animals and their Movements
fifth word in the same way as the
second word is related to the first tkuojksa ds cPpksa ds i'kq vkSj mudh
list/
word and the fourth word is related
to the third word.
ml fodYi dk p;u djsa tks ikapos 'kCn ls
uke xfrfof/k;ksa dh lwph
mlh izdkj lacaf/kr gS tSls nwljk 'kCn igys

361
Download Free PDFs & e-Books from Neon Classes App web. : www.neonclasses.com
17. Select the option in which the words 20. Select the word-pair that best ml fodYi dk p;u djs]a ftlesa 'kCnksa dk
share the same relationship as that represents a similar relationship to vkil esa ogh laca/k gS] tks uhps fn, x,
shared by the given pair of words. the one expressed in the pair of 'kCn&;qXe ds 'kCnksa ds chp gSA
ml fodYi dk p;u djsa ftlesa 'kCnksa ds words given below. (The words must Player : Team/f[kykM+h % Vhe
chp ogh lacaèk gSa tks fn, x, 'kCnksa ds ;qXe be considered as meaningful English (a) Flower : Bouquet/Qwy % xqynLrk
ds chp gSA words and must not be related to (b) Ant : Bee/phaVh % e/kqeD[kh
Gallop : Horse/ ljiV : ?kksM+k each other based on the number of (c) Student : Teacher/Nk= % f’k{kd
(a) Scamper : Mouse/ HkxnM+ : pwgk¡ letters/number of (d) Purpose : Crowd/mÌs’; % HkhM+
(b) Elephant : Amble/ gkFkh : Vgyuk consonants/vowels in the word)
Sol. (a)
(c) Bird : Kangaroo/ i{kh : daxk: ml 'kCn&tksM+h dk p;u djsa tks uhps fn,
Group of Players is called a Team.
(d) Run : Lion/ nkSM+ : 'ksj x, 'kCnksa dh tksM+h esa O;ä fd, x, leku Similarly, a group of flowers is called a
Sol. (a) laca/k dk lcls vPNk çfrfuf/kRo djrk gSA Bouquet.
Gallop is the fastest pace of a horse. ¼'kCnksa dks lkFkZd vaxzsth 'kCn ekuk tkuk
pkfg, vkSj 'kCn esa v{kjksa dh
Similarly, Scamper is to run quickly in a 22. Based on gender / प
hurry, which mouse do generally. la[;k/O;atuksa/Lojksa dh la[;k ds vk/kkj ij
23. Select the option that is related to the
18. Instruments and Indian Musicians ,d nwljs ls lacaf/kr ugha gksuk pkfg,½
third number in the same way as the
who play them Lend : Borrow/ m/kkj nsuk : m/kkj ysuk
second number is related to the first
(a) Happy: Joyous/ [kq'k % gf"kZr
fLVªax ok|;a= vkSj mUgsa (b) Stern : Strict/ dBksj % l[r
number.
ml fodYi dk p;u djsa tks rhljh la[;k
(c) Exciting : Boring/ jksekapd : mckÅ
ctkus okys Hkkjrh; (d) Sharp : Edged/ uqdhyk : fdukjk
ls mlh çdkj lacfa èkr gS tSls nwljh la[;k
igyh la[;k ls lacafèkr gSA
Sol. (c)
laxhrdkj Borrow is the antonym of lend.
Chicken : Rooster : : Duck : ?
eqxhZ % eqxkZ %% cÙk[k % \
18. Select the option in which the words Similarly, boring in the antonym of
share the same relationship as that exciting . (a) Bumble-bee/Hkojk
shared by the given pair of words.
ml fodYi dk p;u djsa ftlesa 'kCnksa ds 20. Individual, thing and class/ O;fä (b) Monk/ eksd
(c) Deer/fgj.k
a
chp ogh lacaèk gSa tks fn, x, 'kCnksa ds ;qXe
ds chp gSA ;k oLrq vkSj oxZ (d) Male duck/uj cÙk[k
Sol. (d)
Zakir Hussain : Tabla 21. Select the option that is related to the Chicken is the female version of the
Tkfdj gqlSu % rcyk fifth word in the same way as the rooster.
(a) Hariprasad Chaurasiya : Flute second word is related to the first Similarly, Duck is the female version of
gfjizlkn pkSjfl;k % ckalqjh word and the fourth word is related male duck.
(b) Bismillah Khan : Sarod to the third word.
ml fodYi dk p;u djsa tks ikaposa 'kCn ls 23. Person and their grave/ औ
fcfLeYykg [kku % ljksn
(c) Ravi Shankar : Santoor mlh çdkj lacafèkr gS tSls nwljk 'kCn igys
jfo 'kadj % larwj 'kCn ls lacafèkr gS vkSj pkSFkk 'kCn rhljs 'kCn 24. Select the option in which the words
(d) Amjad Ali Khan : Shehnai ls lacafèkr gSA share the same relationship as that
vetn vyh [kku % 'kgukbZ Snake : Reptile :: Frog : Amphibian :: shared by the given pair of words.
Sol. (a) Orchid : ? ml fodYi dk p;u djsa ftlesa 'kCnksa ds
Zakir Hussain is one of the greatest table Lk¡ki % ljhl`i %% esa<+d % mHk;pj %% vkfdZM % chp ogh lacaèk gSa tks fn, x, 'kCnksa ds ;qXe
player. \ ds chp gSA
Similarly, Hariparasad Chaurasiya is one (a) Petals/ia[kqfM+;ka (b) Stem/ruk Raj Ghat : Mahatma Gandhi
of the greatest flute player. (c) Animal/tkuoj (d) Flower/Qwy jkt ?kkV % egkRek xka/kh
19. English words (Antonyms/Synonym) Sol. (d) (a) Shanti Vana : Rajiv Gandhi
19. Select the word—pair in which the Snake is come under category of 'kkafr ou % jktho xka/kh
two words are related in the same Rerptile. (b) Vijay Ghat : Lal Bahadur Shastri
way as are the two words in the Frog is come under category of fot; ?kkV % yky cgknqj 'kkL=h
following word-pair. Amphibian. (c) Shakti Sthal : Sanjay Gandhi
'kCn&;qXe dk p;u djsa ftlesa nks 'kCn mlh Similarly, Orchid is come under category 'kfDr LFky % lat; xka/kh
rjg lacafèkr gSa tSls fuEufyf[kr 'kCn&;qXe of Flower. (d) Veer Bhoomi : Indira Gandhi
esa nks 'kCn gSaA ohj Hkwfe % bafnjk xka/kh
Smile : Laugh/eqLdqjkgV % galuk
(a) Run : Stand/nkSM+uk % [kM+k gksuk 21. Individual and group/ O;fä Sol. (b)
Raj Ghat is a memorial dedicated to
Mahatma Gandhi Ji.
(b) Sad : Happy/nq[kh % [kq’k
(c) Blend : Mixture/feyuk % feJ.k vkSj lewg Similarly, Vijay Ghat is the memorial of
(d) Speak : Shout/cksyuk % fpYykuk 22. Select the option in which the words India’s second Prime Minister Lal
Sol. (d) share the same relationship as that Bahadur Shastri.
Laugh is a louder version of smile. shared by the given pair of words. 24. Part and whole relationship/
Similarly, Shout is a bouder version of 25. Select the option that is related to the
speak. third word in the same way as the

362
Best App for Govt. Jobs : Neonclasses (Download Now) Neon Publications
second word is related to the first (c) Farmer : Field/ fdlku : {ks= Sol. (d)
one. (d) Film : Producer/ fQYe : fuekZrk Diagnosis is the process of identification
ml fodYi dk p;u djsa tks rhljs 'kCn ls Sol. (d) of the nature of an illness by the Doctor
mlh çdkj lacafèkr gS tSls nwljk 'kCn igys Chef is a professional cook. and painting is the process which is
'kCn ls lacfa èkr gSA Similarly, Producer manages the making done by painter.
Island : Sea ∷ Oasis : ?/ }hi : lkxj : of a film. Similarly, Judgement is a decision or
u[kfyLrku : \ 28. Worker and tool opinion about someone or something by
29. Select the word-pair in which the two the Judge.
(a) Mountain/ igkM
words are related in the same way as 31. Things and their keeping place
(b) Desert/ jsfxLrku
are the two words in the given word- 32. Select the option that is related to the
(c) Trees/ isM
pair. third term in the same way as the
(d) Air/ ok;q
ml 'kCn&;qXe dk p;u djsa ftlesa nks 'kCn second term is related to the first
Sol. (b) term.
mlh çdkj lacafèkr gSa tSls fn, x,
An Island is a piece of land which is ml fodYi dk p;u djsa tks rhljs in ls
surrounded by water. And, Sea is an
'kCn&;qXe esa nks 'kCn gSaA
Surgeon : Scalpel mlh çdkj lacaf/kr gS tSls nwljk in igys
expanse of water that coves most of the
’kY;&fpfdRld % 'kY;&fpfdRld dh Nqjh in ls lacaf/kr gSA
earth’s surface and surrounds its land
(a) Tailor : Shirt/nthZ % deht Ship : Harbour : : Aeroplane : ?
masses.
(b) Doctor : Hospital/fpfdRld % vLirky Tkgkt % canjxkg %% gokbZ tgkt % \
Similarly, Oasis is a fertile spot in a
(c) Navy : Lieutenant/usoh % ysf¶VusV a (a) Runway/juos
desert, where water is found.
(d) Sculptor : Chisel/ewfrZdkj % Nsuh (b) Hangar/gSaxj
25. Product and raw material
26. Select the option that is related to the Sol. (d) (c) Control tower/daVªksy Vkoj
third term in the same way as the A scalpel is an instrument used by a (d) Destination/xUrO;
second term is related to the first ‘Surgeon’ for surgery. Sol. (b)
term. Similarly, A ‘chisel’ is a tool used for Harbour is a place on the coast where
ml fodYi dk p;u dhft,] ftldk rhljs cutting a hard material such as wood, ships may moor in shelter.
in ls ogh laca/k gS] tks nwljs in dk igys stone, or metal used by Sculptors. Similarly, Hangar is a large building with
in ls gSA 29. Worker and working place an extensive floor area, typically for
Wall % Brick %% Clothes % \ 30. Select the word-pair in which the two housing aircraft or Aeroplane.
words are related in the same way as 32. Simple name on the basis
nhokj % bZaV %% diM+k % \
are the two words in the following 33. Select the option that is related to the
(a) Mill/fey (b) Design/fMtkbZu
word-pair third term in the same way as the
(c) Skirt/LdVZ (d) yarn/lwr
ml 'kCn&;qXe dk p;u djsa ftlesa nks 'kCn second term is related to the first
Sol. (d)
mlh çdkj lacafèkr gSa tSls fuEufyf[kr term.
Wall is made of Bricks.
'kCn&;qXe esa nks 'kCn gSaA ml fodYi dk p;u djsa tks rhljs in ls
Similarly, Fabric is made of Yarn.
Doctor : Hospital/MkWDVj % vLirky mlh çdkj lacafèkr gS tSls nwljk in igys
26. Tool and action
(a) Teacher : School/f’k{kd % Ldwy in ls lacafèkr gSA
27. Select the word—pair in which the
(b)Governor:President/jkT;iky% jk"Vªifr Apple : Fruit :: Spinach : ?
two words are related in the same
way as are the two words in the (c)Supervisor: Officer/i;Zo{s kd % vf/kdkjh lsc % Qy %% ikyd % \
(d) Museum : Visitor/laxzgky; % vkxarqd (a) Food/Hkkstu
following word-pair.
'kCn&;qXe dk p;u djsa ftlesa nks 'kCn mlh Sol. (a) (b) Flower/Qwy
rjg lacafèkr gSa tSls fuEufyf[kr 'kCn&;qXe Doctor works in Hospital. (c) Root/tM+
esa nks 'kCn gSaA Similarly, Teacher works in School. (d) Vegetable/ouLifr
Needle : Sew/lqbZ % flyuk Sol. (d)
(a) Pen : Pencil/dye % isfa ly 30. Worker and action Apple is a kind of fruit.
31. Select the option that is related to the Similarly, Spinach is a kind of a Leafy
(b) Chisel : Carve/Nsuh % uDdk’kh
fifth word in the same way as the vegetable.
(c) Axe : Handle/dqYgkM+h % gRFkk
second word is related to the first MIX PRACTICE QUESTIONS
(d) Gun : Bullet/cUnwd % xksyh
word and the fourth word is related 1. Angioplasty : Heart : Dialysis : ?
Sol. (b)
to the third word. ,aft;ksIykLVh % gn; % Mk;fyfll % \
Needle is used for Sewing.
ml fodYi dk p;u djsa tks ikaposa 'kCn ls (a) Kidney (b) Lungs
Similarly, Chisel is used for carving
mlh çdkj lacafèkr gS tSls nwljk 'kCn igys (c) Brain (d) Stomach
meterials.
'kCn ls lacafèkr gS vkSj pkSFkk 'kCn rhljs 'kCn Sol. (a)
27. Worker and product
ls lacafèkr gSA Angioplasty is used to treat heart
28. Select the option in which the words
Doctor : Diagnosis :: Painter : disease. Similarly, dialysis is done while
share the same relationship as that
Painting :: Judge : ? treating the kidney.
shared by the given pair of words.
fpfdRld % funku %% fp=dkj % fp=dkjh %% ân; jksx ds bykt ds fy, ,aft;ksIykLVh dk
ml fodYi dk p;u djsa ftlesa 'kCnksa ds
U;k;k/kh’k % \ mi;ksx fd;k tkrk gSA blh rjg fdMuh dk
chp ogh lacaèk gSa tks fn, x, 'kCnksa ds ;qXe
(a) Lawyer/odhy bykt djrs le; Mk;fyfll fd;k tkrk gSA
ds chp gSA
(b) Punishment/ltk 2. Chief Justice : President
Food/[kkuk : Chef/ ckopÊ
(c) Court/U;k;ky; eq[; U;k;k/kh’k % jk"Vªifr
(a) Painter : Painting/ isVa j : isfa Vax
(d) Judgement/QSlyk (a) Coporater : Mayor/ik"kZn % es;j
(b) Hospital : Doctor/ vLirky : M‚DVj

363
Download Free PDFs & e-Books from Neon Classes App web. : www.neonclasses.com
(b) Chief Minister : Cabinet Minister/ used in astronomy to define planetary (c) Analgesic/ihM+kgj
eq[;ea=h % dSfcusV ea=h motion. (d) Hospital/vLirky
(c) Chief of Army Staff : Prime Minister/ ikbFkkxksjl ize; s dk iz;ksx T;kfefr esa fd;k Sol. (c)
Fky lsuk/;{k % iz/kku ea=hs tkrk gSA dsIyj ds fu;e dk mi;ksx [kxksy ‘Tea’ is a type of ‘Beverage’ Similarly
(d) Municipal Commissioner : State foKku esa xzgksa dh xfr dks ifjHkkf"kr djus ds ‘Analgesic’ is a type of ‘Medicine’
Government/uxj vk;qDr % jkT; ljdkj fy, fd;k tkrk gSA pk; ,d izdkj dh is; gS blh izdkj ihM+kgkj
Sol. (d) 7. Sahara : Africa ,d izdkj dh nok gSA
The Chief Justice is appointed by the lgkjk % vÝhdk 12. Hive : Bee/NÙkk % e/kqeD[kh
president similarly, the Municipal (a) Thar : Australia/Fkkj % vkWLVªsfy;k (a) Burrow : Hare/fcy % [kjxks’k
Commissioner is appointed by the state (b) Sandy : America/lSaMh % vesfjdk (b) Stable : cow/vLrcy % xk;
government. (c) Gobi : Asia/xksch % ,f’k;k (c) Sty : Dog/’kwdj’kkyk % dqÙkk
eq[; U;k;k/kh’k dh fu;qfDr jk"Vªifr }kjk dh (d) Gibson : Europe/fxClu % ;wjksi (d) Eyrie : Bear/cktuhM+ % Hkkyw
tkrh gS] mlh rjg uxj vk;qDr dh fu;qfDr Sol. (c) Sol. (a)
jkT; ljdkj }kjk dh tkrh gSA Sahara is a desert in Africa. Similarly, Animal & its living place Hive = a dome-
3. Malaria : Parasite : Stomach ulcers : ? Gobi is a desert in Asia. shaped or box like structure in which
eysfj;k % ijthoh % % isV ds Nkys % \ lgkjk vQzhdk dk ,d e:LFky gS blh izdkj bees are kept. Burrow = a hole or tunnel
(a) Protozoa/izksVkstksvk xksch ,f’k;k dk ,d e:LFky gSA dug by a small animal, especially a
(b) Parasites/ijthoh 8. Criticism : Condemnation : : Solidarity :? rabbit or Hare, as a dwelling.
(c) Insects/dhM+s vkykspuk % fuank % % ,dtqVrk % \ tkuoj vkSj mldk jgus dk LFkku&NÙkk 
(d) Bacteria/cSDVhfj;k (a) Heinous/t?kU; ,d xqcan ds vkdkj dk ;k ckWDl tSlk <kapk
Sol. (d) (b) Prosperity/le`f} ftlesa e/kqeD[kh j[kh tkrh gSA fcy & ,d
Malaria is caused by (plasmodium) (c) Guilty/nks"kh NksVk tkuoj] fo’ks"k :i ls ,d [kjxks’k ;k
parasites. Similarly, stomach ulcers are (d) Unity/,drk [kjxks’k }kjk ,d vkok’k ds :i esa [kksnk
caused by (helicobacter pylori) bacteria. Sol. (d) x;k Nsn ;k lqjaxA
eysfj;k ¼IykTesfM;e½ ijthoh ds dkj.k gksrk ‘Condemnation’ is the synonym to 13. Jackal : Howl : : Rain
gSA blh rjg] isV ds vYlj ‘Criticism’ & ‘Unity’ is the synonym to fl;kj % xjtuk % % ckfj’k % \
¼gsfydkscDS vjikbyksjh½ cSDVhfj;k ds dkj.k ‘Solidarity’. (a) Husle/Å/ke
gksrs gSA fuank vkykspuk dk i;kZ; gS vkSj ,drk (b) Thunder/xMxMkgV
4. Human – beings : Mammals : : Snake : ,dtqVrk dk i;kZ; gSA (c) Drops/cwn
a as
ãweu fcaXl % LFku/kkjh % % lkai 9. Exam : Examiner . (d) Patter/iViVkgV
(a) Amphibian/mHk;dj (a) Election : observer/pquko % i;Zos{kd Sol. (d)
(b) Reptile/ljhl`i (b) Bank : Loan/cSad % +_.k The noise made by ‘Jackals’ is known as
(c) Mammals/Lru/kkjh (c) Inspection : Acceptance/tkap % ‘Howl’. Similarly, the noise made by
(d) Non-chordates/xSj&dkWMV sZ ~l Lohdk;Zrk ‘Rain’ is called ‘Patter’
Sol. (b) (d) Film : Comedian/fQYe % dkWefs M;u fl;kj }kjk fd, x, 'kksj dks *xjtuk* ds
Human beings are a kind of mammal. Sol. (a) uke ls tkuk tkrk gSA blh izdkj o"kkZ }kjk
Similarly, snakes are a kind of reptile. Profession & his duty. Invigilator’s duty fd, x, 'kksj dks iViVkgV dgk tkrk gSA
euq"; ,d izdkj dk Lrkuik;h gS blh rjg is to monitor during the exam. Similarly, 14. Blunder : Error
lkai Hkh ,d rjg ds ljhl`Ik gksrs gSA ‘Election’ is monitored by an ‘Observer’ cM+h Hkwy % v’kqf)
5. World Environment Day ¼fo’o i;kZoj.k is’kk vkSj mldk drZO;A ijh:dk dk drZO; (a) Euphoria : Happiness/mRlkg % [kq’kh
fnol½ : 5 June : : ? : 20 August ijh{kk ds nkSjku fuxjkuh djukA blh rjg] (b) Speak : Hear/cksyuk % lquuk
(a) Renewable energy Day/v{k; ÅtkZ pquko dh fuxjkuh i;Zos{kd }kjk dh tkrh (c) War : Peace/;q) % 'kkafr
(b) International Women’s Day/ gSA (d) Anger : Rage/Øks/k % jks"k
varjkZ"Vªh; efgyk fnol 10. Faculty : Teachers/fo’ks"kK % f’k{kd Sol. (a)
(c) World AIDS Day/fo’o ,M~l fnol (a) Ants : Flocks/phafV;ka % >q.M Blunder : The highest intensity of error
(d) World Water Day/fo’o ty fnol (b) Fleet : Trucks/csM+k % Vªd Similarly, Euphoria : a feeling or state of
Sol. (a) (c) Galaxy : Apartments/xSysDlh % extreme excitement and happiness.
World Envionment day is celebrated on vikVZeaVs cMh Hkwy % =qfV dh mPpre rhozrk blh
5th June. Similarly, renewable energy (d) Colony : Wolves/dkWyksuh % HksfM+;s izdkj] mRlkg vR;f/kd mÙkstuk vkSj [kq’kh
day is celebrated on 20th August. Sol. (b) dh Hkkouk ;k fLFkfrA
fo’o i;kZoj.k fnol 5 twu dks euk;k tkrk Group of Teachers is known as ‘Faculty’ 15. Den : Lion : : Stable : ?
gS blh rjg 20 vxLr dks v{k; ÅtkZ fnol Similarly, Group of Trucks is known as ekan % 'ksj % % vLrcy % \
euk;k tkrk gSA ‘Fleet’ (a) Car/dkj (b) Leopard/rsn a qvk
6. Pythagoras : Geometry : Kepler f’k{kdksa ds lewg dks ladk; ds :i esa tkuk (c) Bird/i{kh (d) Horse/?kksM+k
ikbFkkxksjl % T;ksesVªh % dsIysj % \ tkrk gS blh izdkj Vªdksa ds lewg dks csM+s ds Sol. (d)
(a) Electronics (b) Geography :i esa tkuk tkrk gSA Lion lives in the Den and the Horse lives
(c) Astronomy (d) Optics 11. Beverages : Tea : : Medicine : ? in the Stable.
Sol. (c) is; inkFkZ % pk; % % vkS"kf/k % \ 'ksj ekan esa jgrk gS vkSj ?kksM+k vLrcy esa
Pyhagora’s theorem is used in (a) Disease/jksx jgrk gSA
geometery. Similarly, Kepler’s law is (b) Doctor/fpfdRld 16. Colour : Red : : Profession : ?

364
Best App for Govt. Jobs : Neonclasses (Download Now) Neon Publications
jax % yky % % is’kk % \ ftl izdkj ^mM+us* dk laca/k ^i{kh* ls gS mlh ftl izdkj Ropk foKku dk lac/a k Ropk ls
(a) Lawyer/odhy izdkj ^ljiV nkSM+* dk laca/k _____ ls gSA gksrk gS mlh izdkj gM~Mh dk laca/k gM~Mh ls
(b) Court/vnkyr (a) Tiger/ck?k (b) Lion/’ksj gksrk gSA
(c) School/fo|ky; (c) Horse/?kksM+k (d) Elephant/’kgjh 26. Frown : Displeasure
(d) Black/dkyk Sol. (c) rsoj % vizlUrk
Sol. (a) As, ‘Fly’ is related to ‘Bird’ Similarly, (a) Grief : Sorrow
Red is the name of a colour and Lawyer ‘Gallop’ is related to ‘Horse’ nq[k % 'kksd djuk
is the name of a profession. ftl izdkj *mM+uk* dk laca/k i{kh ls gS] mlh (b) Madnes : Illness
yky ,d jax dk uke gS vkSj odhy is’ks dk izdkj ljiV nkSM+ dk lac/a k ?kksM+s ls gSA ikxyiu] jksx chekjh
uke gSA 22. Meat : Vegetarian : : Alcohol : ? (c) Laugh : Comedy
17. Cardiologist : Heart : : Chiropractor : ? ekal % 'kkdkgkjh % % 'kjkc % \ [kq’k] gkL;
àn; jksx fo’ks"kK % àn; % % dkbjksizSDVj % \ (a) Teetotaller/e|R;kxh (d) Smile : Ecstasy
(a) Skin/Ropk (b) Foot/iSj (b) Smoker//kweziku djus okyk g¡lh ije vkuan
(c) Chest/lhuk (d) Joints/tksM+ (c) Water/ikuh Sol. (d)
Sol. (d)] (d) Dietician/vkgkj fo’ks"kK As, Frown is related to Displeasue
Cardiologist is the doctor of Heart and Sol. (a) Similarly , Smile is related to Ecstasy.
Chiropractor is the doctor of Joints. As per question Meat is related to
ân; jksx fo’ks"kK ân; ds fpfdRld gS vkSj Vegetarian. Like a vegetarian avoids 27. ‘Division’ is related to ‘Unity’ in the same
dk;jksiSzDVj tksM+ksa ds fpfdRld gSA Meat. In the same way Teetotaller is way as ‘_____’ is related to ‘improvement’
18. Chickenpox : Virus : : Ringworm : ? related to Alcohol. Teetotaller who ftl izdkj ^VwV* dk laca/k ^,drk* ls gS] mlh
pspd % fo"kk.kq % % nkn % \ avoids Alcohol. rjg ^lq/kkj* dk laca/k ______ ls gSA
(a) Protozoan/izksVkstksvk iz’u ds vuqlkj ekal 'kkdkgkjh ls lacaf/kr gSA (a) Rough/[kqjnqjk
(b) Bacterium/thok.kq tSls 'kkdkgkjh ekal ls ijgst djrk gSA mlh (b) Crash/{kfr
(c) Nematodes/fuesVksM rjg Teetotaller 'kjkc ls lacaf/kr gSA (c) Inconvenience/vlqfo/kk
(d) Fungus/dod Teetotaller tks 'kjkc ls ijgst djrk gSA (d) Stitch/flykbZ
Sol. (d) 23. ‘China’ is related to ‘Yuan’ in the same Sol. As, ’Division’ is related to ‘Unity’
As Virus causes Chickenpox. Similarly, way as ‘Japan’ is related to ‘_____’. Similarly, ‘Crash’ is related to
Fungus causes Ringworm. ftl rjg ^phu dk laca/k* ^^;wvku** ls gS] ‘Improvement’
pwafd ok;jl pspd dk dkj.k curk gSA blh mlh rjg ^tkiku* dk laca/k _____ ls gSA ftl izdkj foHkktu dk laca/k ,drk ls gS]
rjg] dod nkn dk dkj.k curk gSA (a) Rand/jS.M (b) Subhi/lq’kh mlh izdkj nq?kZVuk dk laca/k lq/kkj ls gSA
19. Star : Galaxy : : Flower : ? (c) Lira/yhjk (d) Yen/;su 28. Caress : Affection
rkjk % vkdk’kxaxk % % Qwy % \ Sol. (d) lgykuk % izse
(a) Jasmine/pesyh As, ‘Yuan’ is China’s currency Similarly, (a) Crime : Confession/vijk/k %
(b) Blossom/f[kyuk ‘Yen’ is Japan’s currency. LohdkjksfDr
(c) Bouquet/xqynLrk tSls ;qvku phu dh eqnzk gS] oSls gh ;su (b) Interview : Selection/lk{kkRdkj %
(d) Petal/iÙkh tkiku dh eqnzk gSA p;u
Sol. (c) 24. Potato : Starch : : Stevia : ? (c) Anger : Emotion/Øks/k % Hkkouk
In this question, Star is related to the vkyw % LVkpZ % % LVhfo;k % \ (d) Kick : Hostility/ykr ekjuk % nq’euh
galaxy. Collection of stars is the galaxy. (a) Butter/eD[ku (b) Milk/nw/k Sol. (d)
In the same way a group of flowrs is (c) Salt/ued (d) Sugar/phuh As, Caress related to Affection Similarly,
Bouquet. Kick related to Hostility
bl iz’u esa rkjk vkdk’kxaxk ls lacaf/kr gSA Sol. (c) tSls] nqykj Lusg ls lacaf/kr gS] oSls gh ykr
rkjksa dk laxzg vkdk’kxaxk gSA mlh izdkj As, ‘Potato’ is related to ‘Starch’ then, 'k=qrk ls lacaf/kr gSA
Qwyksa dk ,d lewg xqynLrk gSA ‘Stevia’ is related to ‘Sugar’ 29. Resistance : Ohm
20. ‘Judgment’ is related to ‘Deliver’ in the tSls vkyq dk laca/k LVkpZ ls gS] oSls gh izfrjks/k % vksea
same way as ‘Rule’ is related to ‘____’? LVhfo;k dk laca/k phuh ls gSA (a) Time : Clock/le; % ?kM+h
ftl izdkj ^QSlyk* dk laca/k ^lqukuk* ls 25. Dermatology : Skin (b) Temperature :
gSmlh izdkj ^fu;e* dk lac/a k ^_____* ls gSA RopkfoKku % Ropk Thermometer/rkieku % FkekZehVj
(a) Implement/vuqikyu djuk (a) Cardiology : Angiography/àn;jksx (c) Pressure : Barometer/nkc % nkcekih
(b) Practive/dk;kZfUor djuk foKku % ,aft;ksxzkQh (d) Length : Metre/yackbZ % ehVj
(c) Follow/ekuuk (b) Paediatrics : Lungs/ckyfpfdRlk % Sol.(d)
(d) Audience : Speech/Jksrk % Hkk"k.k QsQM+s Resistance is measured in Ohm,
Sol. (a) (c) Orthopaedics : Bones/vkWFkksi
Z SfMDl % similarly Length measured in meter.
As, ‘Judgement’ is related to ‘Deliver’ gM~fM;k¡ izfrjks/k vkse esa ekik tkrk gS] blh izdkj
Similarly, Rule is related to ‘Implement’ (d) Medicine : Treatment/nokbZ % mipkj yEckbZ dks ehVj esa ekik tkrk gSA
tSls *QSlyk* ,d laca/k *lqukuk* ls gS] mlh Sol. (c) 30. Brass : Copper
izdkj *fu;e* dk lac/a k *vuqikyuk* djus ls As, Dermatology is related to Skin ihry % rkack
gSA Similarly, Orthopaedics is related to (a) Chromium : Silver/Øksfe;e % pk¡nh
21. ‘Fly’ is related to ‘Bird’ in the same way Bones. (b) Bronze : Gold/dkaL; % lksuk
as ‘Gallop’ is related to ‘_____’ (c) Nickle : Iron/fudsy % yksgk
(d) Solder : Tin/Vkadk : fVu
365
Download Free PDFs & e-Books from Neon Classes App web. : www.neonclasses.com
Sol. (d) ftl izdkj *flusek* dk laca/k *n’kZd* ls gS] (b) Stag : Doe/fgj.k % fgj.kh
As, ‘Brass’ is related to ‘Copper’ mh izdkj *ppZ* dk laca/k *lHkk* ls gSA (c) Cold : Horse/cNM+k % ?kksM+k
Similarly, ‘Solder’ is related to ‘Tin’. 35. Punjab : Bhangra : : Gujarat : ? (d) Bee : Drone/eknk e/kqeD[kh % uj
ftl izdkj *ihry* dk laca/k *rkack* ls gS (a) Bihu/fcgw (b) Garba/xjck e/kqeD[kh
mlh izdkj lksYMj dk laca/k *fVu* ls gSA (c) Ghumar/?kwej (d) Kathak/dFkd Sol. (d)
31. Barter : Commodities Sol. (b) As a fox is related to vixen Similarly, Bee
oLrq & fofue; % oLrq,a As Bhangra is fok dance in Punjab. is related to Drone.
(a) Treat : Disease Similarly, Garba is folk dance in Gujarat. ftl izdkj uj ykseM+h dk laca/k eknk ykseM+h
(b) Debate : Discussion ftl izdkj HkkaxMk iatkc dk yksdu`R; gS]] ls gS] mlh izdkj eknk e/kqeD[kh dk laca/k uj
(c) Movement : Traffic mlh izdkj xjck xqtjkr dkA e/kqeD[kh ls gSA
(d) Correspond : Letters 36. Players : Team : : ? : ? 41. Book : Thesaurus
Sol. (d) f[kykM+h % Vhe % % \ % \ iqLrd % Kkudks’k
As, Barter’ is related to ‘Commodities’ (a) Car : Group/dkj % lewg (a) Tree : Forest/o`{k % ou
same As, ‘Correspond’ is related to (b) Ship : Fleet/tgkt % csM+k (b) Reptile : Python/ljhl`Ik % vtxj
‘Letters’. (c) Airplane : Flight/foeku % mM+ku (c) Furniture : Wood/QuhZpj % ydM+h
ftl izdkj *oLrq&fofue;* dk laca/k *oLrqvks*a (d) Pen : Heap/dye % <+sjk (d) Tennis : Ball/Vsful % ckWy
ls gS] mlh izdkj *i=kpkj* dk laca/k *i=ks*a ls Sol. (b) Sol. (b)
gSA If players are related to team, therefor As, Book is related to Thesaurs, Reptile
32. ‘Geology’ is related to ‘Rocks’ in the ship is related to Fleet. is related to Python
same way as ‘Cytology’ is related to ;fn f[kykM+h dk lac/a k Vhe ls gS] blh izdkj tSlk fd iqLrd Kkudks’k ls lacaf/kr gS
‘________’. tgkt dk laca/k csM+k ls gSA ljhl`i] vtxj ls lacaf/kr gSA
ftl izdkj ^HkwxHkZ’kkL=* dk laca/k ^pV~Vkuks*a 37. Light : Lumen : : ? : ? 42. Small : Dimension
ls mlh izdkj ^dksf’kdk foKku* dk laca/k izdk’k % yqesu % % \ % \ NksVk % foLrkj
^&&* ls gSA (a) Temperature : Candela/rkieku % (a) Cold : Winter/BaM % lnhZ
(a) Organism/tho dsM
a syk (b) Heat : Temperature/xehZ % rkieku
(b) Cells/dksf’kdk,a (b) Density : Kilogram/?kuRo % fdyksxzke (c) Triangle : Area/f=Hkqtkdkj % {ks=Qy
(c) Plants/ikS/ks (c) Pressure : Pascal/nkc % ikLdy (d) Heavy : Weight/Hkkjh % otu
(d) Life/thou (d) Force : Meter/cy % ehVj Sol. (d)
Sol. (b) Sol. (c) The given word when come together
As, ‘Geology’ is related to ‘Rocks’ same As, Lumen is a unit of light similarly, makes a particular meaning similarly in
As, ‘Cytology’ is related to ‘Cells’. pascal is a unit of pressure. option (d) both words make a particular
ftl izdkj *HkwxHkZ’kkL=* dk laca/k *pV~Vkuksa ls ftl izdkj izdk’k dh bdkbZ yqeus gksrk gS] meaning when come together.
gS] mlh izdkj *dksf’kd foKku* dk laca/k mlh izdkj nkc dh bdkbZ ikLdy gksrk gSA fn;s x;s 'kCn tc ,d lkFk vkrs gS rks
*dksf’kdkvks(a ls gSA 38. Person : Biography : : ? : fodYi ¼Mh½ esa leku :i ls ,d fo’ks"k vFkZ
33. ‘Study’ is related to ‘Knowledge’ in the O;fDr % thouh % % \ % \ cukrk gS nksuksa 'kCn ,d lkFk vkus ij ,d
same way that ‘Work’ is related to: (a) World : Encyclopedia/fo’o % fo’ks"k vFkZ cukrs gSA
ftl izdkj ^v/;;u* dk laca/k ^Kku* ls gS fo’odks’k 43. Businessman : Profit : : ? : ?
mlh izdkj ^dk;Z* dk laca/k ^----------* ls gSA (b) Nation : History/jk"Vª % bfrgkl O;olk;h % ykHk % % \ % \
(a) Salary/osru (a) Doctor : Patient/fpfdRld % ejht
(c) Place : Economy/LFkku % vFkZO;oLFkk
(b) Employment/csjkstxkj (b) Worker : Wages/deZpkjh % osru
(d) Country : Contitution/ns’k % lafo/kku
(c) Experience/vuqHko (c) Police : Criminal/iqfyl % vijk/kh
Sol. (b)
(d) Training/izf’k{k.k If person is related to Biography, (d) Court : Judge/U;k;ky; % U;k;k/kh’k
Sol. (c) therefore Nation is related to History. Sol. (b)
As, ‘Study’ is related to ‘Knowledge’ in ;fn O;fDr dk lac/a k thouh ls gS] rks blh As Businessman is related to Profit.
the same way ‘Work’ is related to izdkj] jk"Vª dk laca/k bfrgkl ls gSA Similarly, Worker is related to Wages
‘Experience’. ftl izdkj O;olk;h dk laca/k ykHk ls gS
39. Ramanujan : Mathematician : : Sushruta
ftl izdkj *v/;;u* dk laca/k Kku ls gS] :? mlh izdkj deZpkjh dk laca/k osru ls gSA
mlh izdkj *dk;Z* dk laca/k *vquHko* ls gSA jkekuqt % xf.krK % % lqJqr % \ 44. Binocular : View : : ? : ?
34. ‘Cinema’ is related to ‘Audience’ in the (a) Scientist/oSKkfud nwjchu % n’kZu % % \ % \
same way as ‘Church’ is related to ‘_____’.
(b) Architect/okLrqdkj (a) Shovel : Scoop/dkjNk % fudkyuk
ftl izdkj ^flusek* dk laca/k ^n’kZdx.k* ls
(c) Physician/fpfdRld (b) Spanner : Guard/ukius okyk % j{kd
gS mlh izdkj ^ppZ* dk laca/k ^------------* ls gSA
(d) Astronomer/[kxksy’kkL=h (c) Tongs : Pore/fpeVk % fNnz
(a) Congregation/lekxe
Sol. (d) (d) Chisel : Grind/Nsuh % ihluk
(b) Meditation//;ku
If Ramanujan is a Mathematician, Sol. (a)
(c) Concentration/,dkxzrk Therefore Sushruta is a Physican. As Binocular is used to see distant
(d) Prayer/izkFkZuk ;fn jkekuqt dk laca/k xf.krK ls gS] rks blh objects, similarly, Shovel is used to
Sol. (a) izdkj] lqJqr dk laca/k fpfdRld ls gSA Scoop.
As, ‘Cinema’ is related to ‘Audience’ 40. Fox : Vixen : : ? : ? ftl izdkj nwjchu ls ns[kk tkrk gS] mlh
Similarly, ‘Chruch’ is related to uj ykseM+h % eknk ykseM+h % % \ % \ izdkj djNk [kksnus o ckgj fudkyus ds dke
‘Congregation’.
(a) Butterfly : Caterpillar/frYyh % bYyh vkrk gSA

366
Best App for Govt. Jobs : Neonclasses (Download Now) Neon Publications
45. Book : Pages : : Ladder : ? 50. Car : Steering Wheel : : Motorcycle : ? vk’kk dk foijhr fujk’kk mlh izdkj ikxy
iqLrd % i`"B % % lh<+h % \ dkj % fLV;fjax Oghy % % eksVjlkbfdy % \ dk foijhr fopkj’khy gksrk gSA
(a) Steps/dne (b) Walking/pyuk (a) Tyre/Vk;j 56. Screw : Screw driver : : Nail : ?
(c) Parts/Hkkx (d) Levels/Lrj (b) Headlight/gsMykbV isp
a % isp
a dl % % dhy % \
Sol. (a) (c) Engine/batu (a) Nail driver/dhy dl
As book is a collection of Pages, (d) Handle bar/gSaMy ckj (b) Polish/ikWyh’k
Similarly, Ladder has Steps. Sol. (d) (c) Steel/LVhy
ftl izdkj ,d iqLrd esa i`"B gksrs gSa] mlh 51. River : Stream : : Ocean : ? (d) Hammer/gFkkSM+k
izdkj lhf<+;ksa ij ik¡o j[kus ds fy, Steps unh % /kkjk % % egklkxj % \ Sol. (d)
gksrs gSA (a) Current/izokg (b) Pond/rkykc As screw driver is used to tight the
46. Phycology : Algae : : ? : ? (c) Dam/cka/k (d) Sea/leqnz screw, similarly, the hammer is used to
'kSoky foKku % 'kSoky % % \ % \ Sol. (a) fix nail.
(a) Ectomonoly : Amphibians As, River is related to Stream, Similarly, ftl izdkj isap dks dlus ds fy, isapdl dk
lzko izdj.k foKku % mHk;pj Ocean is related to ‘Current’. izz;ksax gksrk gS] mlh izdkj dhy dks Bksdus ds
(b) Selenography : Secretions ftl izdkj unh dk laca/k /kkjk ls gS mlh fy, gFkkSMk+s dk iz;ksx fd;k tkrk gSA
panzHkwxksy foKku % {kj.k&fØ;k izdkj egklkxj dk laca/k izokg ls gSA 57. Cotton : White : : Coal : ?
(c) Heametology : Blood/ :f/kj foKku/ 52. Sheep : Lamb : : Cow : ? dikl % lQsn % dks;yk % \
jDr HksM+ % eseuk % % xk; % \ (a) Fire/vkx (b) Mine/[knku
(d) Entomology : Soil (a) Kitten/fcYyh dk cPpk (c) Wood/ydM+h (d) Black/dkyk
dhV foKku % feV~Vh (b) Cub/’kkod Sol. (d)
Sol. (c) (c) Calf/cNM+k The colour of cotton is white. Similarly,
The study of Algae is called Phycology, (d) Caterpillar/bYyh the colour of coal is black.
similarly, the study of blood is called Sol. (c) dikl dk jax lQsn gksrk gS blh izdkj
Haematology. As, the young one of Sheep is Lamb, dks;ys dk jax dkyk gksrk gSA
ftl izdkj 'kSoky foKku dk laca/k 'kSoky ls Similarly, the young one of Cow is Calf. 58. Goitre : Iodine : : Anaemia : ?
gS mlh izdkj :f/kj foKku dk laca/k jDr ls tSls HksM+ ds cPps dks eseuk dgrs gS] mlh ?ks?a kk % vk;ksMhu % % ,uhfe;k % \
gSA izdkj xk; ds cPps dks cNM+k dgrs gSA (a) Vitamin D/foVkfeu Mh
47. Leaves : Rustle : : ? : ? 53. Frog : Amphibian : : Lizard : ? (b)Iron/ykSg
ifÙk;k¡ % ljljkgV % % \ % \ esaVd % mHk;pj % % fNidyh % \ (c) Vitamin E/foVkfeu bZ
(a) Rain : Jingle/o"kkZ % >u>ukgV (a) Rodent/d`rad (d) Calcium/dSfY’k;e
(b) Tyres : Sizzle/VkW;j % NuNukgV (b) Mammal/Lruik;h Sol. (b)
(c) Clouds : Thunder/ckny % xMxMkgV (c) Reptile/ljhl`Ik As, Lack of Iodine is a cause of Goitre,
(d) Snow : Drizzle/cQZ % cwna k ckanh (d) Insect/dhV Similarly, Lack of Iron is cause of
Sol. (c) Sol. (c) Anaemia.
As the sound of Leaves is called Rustle, As frog is from Amphibian species, ?ks?kk jksx vk;ksMhu dh deh ls gksrk gSA blh
Similarly, the sound of cloud is called Similarly, Lizard is from Reptile species. izdkj ,uhfe;k jksx ykSgs dh deh ls gksrk gSA
Thunder. ftl izdkj es<a +d mHk;pj iztkfr dk gS mlh 59. Calendar : Date : : Index : ?
ftl izdkj ifÙk;ksa esa ljljkgV dh vkokt izdkj fNidyh ljhl`Ik iztkfr dh gksrh gSA iapkax % frfFk % % lwph % \
vkrh gSa] mlh izdkj ckny esa xM+xM+kgV dh 54. Father : Parent : : Sister : ? (a) Name of Author/ys[kd dk uke
vkokt vkrh gSA firk % ekrk&firk % % cgu % \ (b) Glossary/’kCnkoyh
48. Mason : Plumb line : : ? : ? (a) Other/vU; (c) Contents/fo"ke&oLrq
jkt feL=h % lkgqy lw= % % \ % \ (b) Brother/HkkbZ (d) Summary/lkjka’k
(a) Surgeon : Scalpet/’kY; fpfdRld % (c) Daughter/csVh Sol. (c)
{kqfjdk (d) Sibling/lgksnj As, Calendar is related to Date, similarly,
(b) Sculptor : Spade/ewfrZdkj % dqnky Sol. (d) Index is related to Contents
(c) Blacksmith : Forcep/yksgkj % fpeVk As, father is related to Parent, Similarly, tSls iapkax frfFk ls lacaf/kr gS mlh izdkj
(d) Gardener : Saw/ckxcku % vkjk Sister is related to Sibling. lwph fo"ke&lwph ls lacaf/kr gSA
Sol. (a) ftl izdkj firk] ekrk&firk ls lacaf/kr gS] 60. Ampere : Electric current : : fathom : ?
As Mason uses plumb line in his mlh izdkj] cgu] lgksnj ls lacaf/kr gSA ,Eih;j % fo|qr izokg % % QSFke % \
profession, Similarly, Surgeon uses 55. Hope : Despair : : Lunatic : ? (a) Depth of water/ty dh xgjkbZ
scalpel. vk’kk % fujk’kk % % ikxy % \ (b) Frequency/vko`fr
ftl izdkj jktfeL=h dk lac/a k lkgqy lw= ls (a) Honest/bZekunkj (c) Sound level//ofu Lrj
gS] mlh izdkj 'kY; fpfdRld dk laca/k (b) Hardworking/ifjJeh (d) Work or energy/dk;Z ;k ÅtkZ
{kqfjd ls gSA (c) Sane/fopkj’khy Sol. As, Ampere is a unit which is used to
49. Influenza : Virus : : Ringworm : ? (d) Stylish/lthyk measure he Electric current, Similarly,
ba¶yq,atk % fo"kk.kq % % nkn % \ Sol. (c) Fathom is a unit which is used to
(a) Bacteria/cSDVhfj;k As Despair is an antonym of Hope, measure to Depth of water.
(b) Fungi/dod Similarly, the antonym of Lunatic is ftl izdkj ,Eih;j ls fo|qr izokg dks ekik
(c) Parasite/ijthoh Sane. tkrk gS] mlh izdkj QSFke ls ty dh xgjkbZ
(d) Protonzoa/izksVkstksvk dh tk¡p dh tkrh gSA
Sol. (b) Ringwarm is caused by fungus.
367
Download Free PDFs & e-Books from Neon Classes App web. : www.neonclasses.com
61. Canada : Ottawa : : ? : ? HksM+ ds ekal dks eVu dgk tkrk gS blh Hkkjr % vke % % U;wthySaM % % \
dukMk % vksVkok % % \ % \ izdkj fgj.k ds ekal dks e`xekal dgk tkrk (a) Apples/lsc
(a) Egypt : Cairo/feL= % dkfgjk gSA (b) Kiwi/fdoh
(b) Norway : Havana/ukWosZ % gokuk 66. Macabre : Lovely : : Baneful : ? (c) Grapes/vaxwj
(c) France : Rome/Ýkal % jkse foHkRl % I;kjk % % fouk’kdkjh % \ (d) Bananas/dsyk
(d) Kenya : Teheran/dsU;k % rsgjku (a) Innocuous/vgkfudkj.k Sol. (b)
Sol. (a) (b) Churlish/vHknz India is famous for mango, Similarly
As, Ottawa is Canada’s capital similarly, (c) Filter/Nkuuk kiwis are found in New Zealand.
Cairo is Egypt’s capital. (d) Ugly/xank Hkkjr vke ds fy, izfl} gS blh rjg dhoh
vksVkok dukMk dh jkt/kkuh gSA mlh izdkj Sol. (a) U;wthySaM esa ik, tkrs gSA
dkfgjk feL= dh jkt/kkuh gSA Macabre is just opposite to Lovely. 71. International Literacy Day September 8 :
62. Bark : Dog : : ? : Sparrow Similarly, Baneful is just opposite of : International Woman’s Day : ?
HkkSaduk % dqÙkk % % \ % fpfM+;k innocuous. varjkZ"Vªh; lk{kjrk fnol % 8 flrEcj % %
(a) Grunt/?kqj?kqjkuk ,d nqljs ds foykse gSA varjkZ"Vªh; efgyk fnol % % \
(b) Snort/QquQqukuk 67. Demographer : People : Philatelist : ? (a) March 8 (b) June 26
(c) Howl/fpYykuk tulkaf[;dh fo’ks"kK % tu % fVdV (c) April 22 (d) November 4
(d) Chirp/pw¡ pw¡ djuk laxzgdrkZ % \ Sol. (a)
Sol. (d) (a) Fossils/thok’e International Literacy Day is celebrated
As dog is used to Bark, similarly, (b) Stamps/eqgj on September 8 every year. Similarly,
Sparrow is used to Chirp. (c) Photography/fp=dkjh International Women’s Day is
tSls dqÙkk HkkSadrk gS mlh izdkj fpfM+;k pw¡& (d) Music/laxhr celebrated on March 8.
pw¡ djrh gSA Sol. (b) varjk"Vªh; lk{kjrk fnol gj lky 8 flracj
63. Mitochondria : Energy : : DNA : ? Demographer studies the changing dks euk;k tkrk gS blh rjg 8 ekpZ dks
ekbVksdkWfMª;k % ÅtkZ % % Mh-,u-, % \ numbers of births, deaths, housing, civic varjk"Vªh; efgyk fnol euk;k tkrk gSA
(a) Inheritance/ amenities etc in a community over a 72. Qualm : Nausea : : Burn : ?
(b) Reproduction/iztuu period of time. In other words, lansg % ?k`.kk % % tyuk % \
(c) Locomotion/xfr’khyrk demographer collects various data (a) Fresh/rktk
(d) Immunity/xfrj{kk about people. Similarly Philatelist is a (b) Sear/>qyluk
Sol. (a) person who collects or knows a lot (c) Sensible/le>nkj
Mitochondria is referred to as the about postage stamps. (d) Wet/xhyk
powerhouse of the cell in which tulkaf[;dh; ukxfjd dh tUe] e`R;q] vkokl Sol. (b)
respiration of the cell and energy vkSj mudh cnyrh la[;k dk v/;;u djrk Qualm cause nausea, Similarly, burn
production occur. DNA is unit of gS mlh izdkj fVdV laxzgdrkZ fVdVksa ds ckjs causes sear.
inheritance. esa tkudkjh j[krk gSA lansg ?k`.kk dk dkj.k curk blh rjg tyus
ekbVksdkWfUMª;k dks dksf’kdk dk ÅtkZ?kj dgk 68. Germany : Mark : : Morocco : ? ls fl;j gksrk gSA
tkrk gS vkSj Mh-,u-, oa’kkuqdze dh bdkbZa gSA teZuh % ekdZ % % eksjDdks % \ 73. Polio : Virus : : Anthrax : ?
64. Chisel : Sculptor : : Harrow : ? (a) Dollar/MkWyj iksfy;ks % fo"kk.kq % % vaFkzsDl % \
Nsuh ls dkVuk % ewfrZdkj % % gSjks % \ (b) Lira/yhjk (a) Fungus/QaQwnh
(a) Gardener/ckxoku (c) Dirham/fnjge (b) Bacteria/thok.kq
(b) Mason/feL=h (d) Taka/Vdk (c) Virus/fo"kk.kq
(c) Blacksmith/yksgkj Sol. (c) (d) Insect/dhV
(d) farmer/fdlku Here country-currency relationship has Sol. (b)
Sol. (a) been shown. Deutsche Mark is the The causative organism of Polio is virus.
Here Tool-Worker relationship has been currency of Germany. Dirham is the Similarly, the causative organism of
shown. Chisel is the tool of Sculptor. currency of Morocco. anthrax is bacteria.
Similarly, Harrow is the tool of ;gka ns’k eqnzk laca/k fn[kk;k x;k gS M;w’k iksfy;ks dk dkjd tho fo"kk.kq gS blh izdkj
Gardener. ekdZ teZuh dh eqnzk gS fnjge eksjDdks dh ,aFksDz l dk dkjd tho thok.kq gSaA
;gka Vwy&cdZj lac/a k fn[kk;k x;k gS Nsuh eqnzk gSA 74. Bihu : Assam : : Onam : ?
eqfrZdkj dk midj.k gS blh izdkj gSjks ekyh 69. String : Violin : : ? : Piano fcgw % vle % % vksue % ?
dk midj.k gSA rkj % ok;yu % % \ % fi;kuks (a) Karnataka/dukZVd
65. Sheep : Mutton : : Deer : ? (a) Music/laxhr (b) Tune/lqj (b) Kerala/dsjyk
HksM+ % eVu % % fgj.k % \ (c) Chord/rkj (d) Key/dqath (c) Jammu&Kashmir/tEew d’ehj
(a) meat/ehV (b) flesh/ekal Sol. (d) (d) Rajasthan/jktLFkku
(c) venison/fgj.k dk ekal Violin is a musical instrument with Sol. (b)
(d) veal/cNM+s dk ekal strings. Similarly, Piano is a large Bihu is celebrated in Assam. Similarly,
Sol. (c) musical instrument played by pressing Onam is celebrated in Kerala.
The flesh of sheep is called mutton. the black or white keys of a keyboard. chgw vle es euk;k tkrk gS blh rjg
Similarly, the flesh of deer is called ok;fyu ,d ok| ;a= gS ftlesa rkj gksrs gS dNqvk ,d ljhl`i gSA
venison. fi;kuks esa dkyk ;k lQsn dhcksMZ gksrk gS 75. Yard : Inch : : Quart : ?
ftldks nckdj ctk;k tkrk gSA ;kMZ % bap % % DokVZ % \
70. India : Mango : : New Zealand : ? (a) Gallon/xSyu (b) Ounce/vksl
a
368
Best App for Govt. Jobs : Neonclasses (Download Now) Neon Publications
(c) Milk/nw/k (d) Liquid/rjy Confirmed and inveterate are related ftl izdkj ^fØ;k* dk laca/k ^izfrfØ;k* ls gS
Sol. (b) terms, Similarly, Financial and Bankrupt mlh izdkj ^mdlkuk* _____ ls lacaf/kr gSA
Inch is smaller unit of length than the are related terms. (a) Reception/Lokxr
yard. Similarly, Ounce is smaller unit of fuf’pr vkSj iqjkuk ,d nwljs ls lacaf/kr gS (b) Vision/nwjn`f"V
volume than the quart. blh rjg foÙkh; vkSj fnokfy;k ,d nwljs ls (c) Response/vuqfØ;k
76. Optimist : Cheerful : : Pessimist : ? lacaf/kr gSA (d) Feedback/izfriqf"V
vk’kkoknh % g"kZoknh % % fujk’kkoknh % \ 80. Astronomy : Stars : : Geology : ? Sol. (c)
(a) Gloomy/nq%[kh [kxksy foKku % rkjk % % Hkw&foKku % \ As, ‘Action’ is related to ‘Reaction’
(b) Mean/rqPN (c) Petty/vYi (a) Geometry/js[kk xf.kr Similarly, ‘Stimulus’ is related to
(d) Helpful/lgk;d (b) Science/foKku ‘Response’
Sol. (a) (c) Earth/i`Foh ftl izdkj fdz;k dk lac/a k izfrfdz;k ls gS
Optimist is a person who is always (d) Sky/vkleku mlh izdkj mÙkstuk dk laca/k vuqfØ;k ls gSA
hopeful and expects the best in all things Sol. (c) fuf’p
and hence remains cheerful. Pessimist is The scientific study and exploration of
WORD BASE
a person who expects the worst to
happen. Therefore such person remains
Stars and other heavenly bodies is CLASSIFICATION
known as Astronomy. Similarly, the
gloomy. 1. Country & Capitals / ns’k ,oa jkt/kkuh
scientific study of the Earth and its
vk’kkoknh og O;fDr gS tks ges’kk vk’kkfUor 1. The way 'Australia' is related to
interior is called Geology.
jgrk gS vkSj lHkh phtksa esas loZJs"B dh 'Canberra', in the same way 'France' is
rkjksa dk oSKkfud v/;;u [kxksy foKkku esa
mEehn djrk gS vkSj blfy, galeq[k jgrk gS related to '__________'.
djrs gS blh izdkj i`Foh rFkk blds vkarfjd
isflfeLV og O;fDr gS tks lcls [kjkc gksus ftl çdkj ^v‚LVªsfy;k* ^dSucjk* ls lacfaèkr
Hkkx dk v/;;u HkwxHkZ’kkL= esa djrs gSA
dh mEehn djrk gSA blfy, ,slk O;fDr gS] mlh çdkj ^Ýkal* ^&&&&&&&&&&* ls
81. Triangle : Hexagon : ?
mnkl jgrk gSA lacfaèkr gSA
f=Hkqt % "kV~Hkqt % \
(a) Rome/jkse (b) Jakarta/tdkrkZ
77. Ammeter : Current : : ? : ? (a) Cone : Sphere/’kadq % xksyk)Z
vehVj % fonq;r izokg % % \ % \ (c) Moscow/ekLdks (d) Paris/isfjl
(b) Rectangle : Octagon/vk;r % v"BHkqt
(a) Scale : Speed/Ldsy % xfr Sol. (d)
(c) Pentagon : Heptagon/iapHkqt % lIrHkqt
(b) Seismograph : Density/HkwdEiys[kh % Canberra is the capital of Australia.
(d) Angle : Quadrilateral/dks.k % prqHkqZt
Similarly, Paris is the capital of France.
?kuRo
2. Country & Currency/ ns’k ,oa eqnzk
(c) Speedometer : Density/xfrekfi % ?kuRo Sol. (b)
2. ‘China' is related to ‘Yuan' in the same
(d) Anemometer: Speed of Wind/ ,uheksehVj Triangle consists of three straight lines.
way as ‘Japan' is related to ‘………..’.
% ok;q dh xfr Hexagon consists of six lines. Similarly,
ftl çdkj 'phu' dk lacaèk ';qvku' ls gS
Sol. (d) rectangle consists of four lines and that
mlh çdkj 'tkiku' dk lacaèk '------------' ls gSA
Ammeter is an instrument which is used of Octagon consists of eighth lines.
(a) Lira/yhjk (b) Rand/jaM+
to measure the flow of current. f=Hkqt esa rhu lh/kh js[kk,a gksrh gS "kV~Hkqt
(c) Sushi/lq'kh (d) Yen/;su
Similarly, Anemometer is an instrument, Ng iafDr;ksa ls feydj curk gS blh rjg
used to measure the speed of wind. Sol. (d)
vk;r esa pkj] js[kk,a gksrh gS vkSj v"Vdks.k esa
,ehVj ,d midj.k gS ftldk mi;ksx fo|qr Yuan is the currency of China.
vkB js[kk,aA
Similarly, Yen is the currency of Japan.
/kkjk dks ekius ds fy, fd;k tkrk gS blh 82. Coal : Black diamond : : Petroleum : ?
3.Country & Continent/ ns’k ,oa egk}hi
izdkj ,uheksehVj ,d midj.k gS ftldk dks;yk % dkyk ghjk % isVªksy % \
3. ‘Belgium’ is related to ‘Europe’ in the
mi;ksx gok dh xfr ekius ds fy, fd;k (a) Solid gold/Bksl lksuk
same way as ‘Japan’ is related to ‘.....’.
tkrk gSA (b) Black gold/dkyk lksuk
'csfYt;e' dk lacaèk ';wjksi' ls mlh çdkj gS
78. Pressure : Barometer : : ? Odometer (c) Liquid gold/rjy lksuk
tSls 'tkiku' dk lacaèk '-----' ls gSA
nkc % ok;qnkcekih % % \ % ehyekih (d) White gold/lQsn lksuk
(a) Africa/vÝhdk
(a) Humidity/vknzZrk Sol. (c) (b) Australia/vkWLVªsfy;k
(b) Distance/nwjh Coal is referred to as Black Diamond,
(c) Asia/,f’k;k
(c)Thickness/eksVkbZ Similarly Peroleum is called Liquid Gold.
(d) Europe/;wjksi
(d) Wind/ok;q 83. Video : Cassette : : Computer : ?
Sol. (c)
Sol. (b) fofM;ks % dSlsV % % dEI;wVj % \
Belgium is in the Europe continent.
Barometer is used to measure the (a) Files/Qkby (b)Floppy/¶ykih
Similarly, Japan is in Asia.
pressure. Similarly, Odometer is used to (c) Bits/fcV (d) Audit/vkWfMV
4.State and Capital/ jkT; ,oa jkt/kkuh
measure the distance. Sol. (b)
4. ‘Bhubaneswar’ is related to ‘Odisha’ in
cSjksehVj dk iz;ksx nkc ekius ds fy, fd;k Video is recorded in cassette. Similarly,
the same way as ‘Aizawl’ is related to:
tkrk gS blh izdkj nwjh ekius ds fy, Floppy is storage device used in
ftl çdkj 'HkqousÜoj' dk lacaèk 'mM+hlk' ls
vksMksehVj dk iz;ksx fd;k tkrk gSA computer.
gS] mlh çdkj 'vkbtksy' dk lacèa k fdlls gS\
79. Confirmed : Inveterate : : Financial : ? ohfM;ks dks dSlsV esa fjdkWMZ dj ysrs gS bl
(a) Mizoram/fetksje
fuf’pr % iqjkuk % % vkfFkZd % \ izdkj ¶ykWih Hkh dEI;wVj dk LVksjst fMokbl
(b) Meghalaya/es?kky;
(a) Callow/vuqHkoghu gSA
(c) Manipur/ef.kiqj
(b) Incredible/vrqY; 84. ‘Action’ is related to ‘Reaction’ in the
(d) Tripura/f=iqjk
(c) Bankrupt/fnokfy;k same way as ‘Stimulus’ is related to
‘_______’ Sol. (a)
(d) Knot/xkaB cka/kuk
Sol. (c)
369
Download Free PDFs & e-Books from Neon Classes App web. : www.neonclasses.com
Bhubneswar city is the capital of Odisha 9.Instrument & Measurement/ ;a= ,oa 13.List of Animals and their Homes/
state. ekiu tkuojksa vkSj muds ?kjksa dh lwph
Similarly, Aizawl city is the capital of 9. 'Ammeter’ is related to ‘Current’ in the 13. ‘Bee’ is related to ‘Hive’ in the same way
Mizoram state. same way as ‘Anemometer’ is related to as ‘Horse’ is related to:
5.River & City / ufn;ka ,oa 'kgj ‘________’. 'eèkqeD[kh' dk lacaèk 'NÙks' ls mlh çdkj gS
5. ‘Lucknow' is related to 'Gomti' in the ',ehVj' 'djaV' ls mlh rjg lacafèkr gS tSls tSls '?kksM+k' fdlls lacafèkr gS
same way as 'Varanasi' is related to ',uheksehVj' ‘----------’ ls lacafèkr gSA (a) Cottage/>ksi a M+h
‘________’. (a) Temperature/ rkieku (b) Stable/vLrcy
ftl çdkj 'y[kuÅ' dk lacaèk 'xkserh' ls gS (b) Potential Difference/ laHkkfor varj (c) Den/ekn
mlh çdkj 'okjk.klh' dk lacaèk ‘----------’ ls (c) Water/ikuh (d) House/?kj
gSA (d) Wind Speed and Direction/ gok dh Sol. (b)
(a) Yamuna/;equk (b) Saryu/lj;w xfr vkSj fn'kk A Hive is a home for Bees.
(c) Ganga/xaxk (d) Sol. (d) Similarly, Stable is home for Horse.
Godavari/xksnkojh Ammeter is an instrument used to 14. Animals and their Movements list/i'kq
Sol. (c) measure electric current. vkSj mudh xfrfof/k;ksa dh lwph
Lucknow is situated on bank of river Similarly, Anemometer is an instrument 14. ‘Fly’ is related to 'Bird' in the same way
Gomti. used to measure wind speed and as 'Gallop' is related to '
Similarly, Varanasi is situated on bank of direction. ftl çdkj 'mM+uk' dk lacaèk 'i{kh' ls gS]
river Ganga. 10. Disease and Organ / chekjh ,oa izHkkfor mlh çdkj 'ljiV nkSM+uk' dk lacaèk ……..' ls
6.Planets in Order From the Sun/lw;Z ls vax gSA
Øe esa xzg 10. ‘Dumbness’ is related to ‘Speech’ in the (a) Tiger/ck?k (b) Lion/'ksj
6. 'Tuesday' is related to 'Mars' in the same same way as ‘_________’ is related to (c) Horse/?kksM+k (d) Elephant/gkFkh
way as 'Thursday' is related to '......' 'Memory'. Sol. (c)
ftl çdkj 'eaxyokj' dk lacaèk 'eaxy' ls gS ftl izdkj xwxakiu^ ^cksyus^ ls lcaf/kr gS Flying is a form of movement of birds.
mlh çdkj 'xq#okj' dk lacaèk '------' ls gSA mlh izdkj _________ ^Le`fr^ ls lacfa /kr gSA Similarly, galloping is a form of
(a) Venus/’kqØ (a) Amnesia/Le`fryksi movement of horses.
(b) Mercury/cq/k (b) Recognition/vfHkKku 15. English words (Antonyms/Synonym)
(c) Saturn/’kfu xzg (c) Thoughtlessness/fopkjghu 15. ‘Valuable’ is related to ‘Precious’ in the
(d) Jupiter/c`gLifr (d) Illusion/Hkze same way as ‘Dry" is related.
Sol. (d) Sol. (a) 'ewY;oku' dk laca/k 'cgqeYw ;' gS rks blh çdkj
Tuesday is named after Tyr’s day, the Dumbness is not communicating ':[kk' dk lacaèk fdlls gSA
Norse god of war associated with the vocally. (a) Firm/Bksl (b) Arid/’kq"d
planet Mars. Similarly, Amnesia is referred when a (c) Wet/xhyk (d) Rough/dBksj
Similarly, Thursday is named after person losses memory. Sol. (b)
Thor’s day, the god associated with the 11. Games & Place of Playing/[ksy ,oa [ksy Synonym of valuable is precious in the
planet Jupiter. lacaf/kr LFkku same way synonym of dry is Arid.
7.Branch of Study/ v/;;u dh 'kk[kk 11. ‘Skating’ is related to ‘Rink’ in the same 16. ‘Action’ is related to ‘Reaction’ in the
7. 'Cardiologist' is related to 'Heart' in the way as ‘Tennis’ is related to: same way as ‘Stimulus’ is related to
same way as 'Neurologist'is related to ' 'LdsfVax' dk lacaèk 'fjad' ls mlh çdkj gS ‘________’.
ftl çdkj 'ân; jksx fo'ks"kK' dk lacaèk tSls 'Vsful' fdlls lacafèkr gS ‘fØ;k’ dk laca/k ‘izfrfØ;k’ ls mlh çdkj gS
'ân;' ls gS] mlh çdkj 'raf=dk foKkuh' dk (a) Racket/jSdsV (b) Court/dksVZ tSls ‘izsj.kk’ dk laca/k ‘________’ ls gSA
lacaèk fdl ls gSA (c) Pitch/fip (d) Arena/v[kkM+k (a) Reception/vfHkuanu
(a) Ears/dku (b) Teeth/nkar Sol. (b) (b) Vision/n`f"V
(c) Brain/efLr"d (d) Lungs/QsQM+s Rink is the surface used for Skating. (c) Response/vuqfØ;k
Sol. (c) Similarly, Court is the surface used for (d) Feedback/izfriqf"V
Cardiologist deals with the disorders of playing Tennis. Sol. (c)
the heart. 12. Animals & Animal Baby Names/tkuoj An action leads to a reaction.
Similarly, Neurologist deals with the ,oa tkuojksa ds cPpksa ds uke Similarly, a stimulus leads to a response.
discorders of the brain. 12. 'Deer' is related to 'Antelope', 'Chicken' 16. Individual, thing and class/ O;fä ;k
8.Quantity and Unit /ek=k ,oa bdkbZ is related to '________'. oLrq vkSj oxZ
8. ‘Mass’ is related to ‘Kilogram’ in the fgjuh* dk tks lacaèk ^e`x'kkod* ls gS] ^eqxh*Z 17. ‘Earthworm’ is related to ‘Non-
same way as ‘Force’ is related to: dk ogh lacaèk ^&&&&&&&&* ls gSA chordates’ in the same way as ‘Fish’ is
ftl çdkj 'ekl' dk lacaèk 'fdyksxzke' ls gS] (a) Cygnet/jktgal 'kkod related to ‘______’.
mlh çdkj 'cy' dk lacaèk fdlls gS\ (b) Lion cub/flag’kkod ^dsp
a q,^ dk ^vd’ks#dh^ ls ogh laca/k gS] tks
(a) Newton/U;wVu (c) Lamb/eseuk ^eNyh^ dk ‘______’ ls gSA
(b) Pascal/ikLdy (d) Chicks/pwts (a) Chordates/d’ks#dh
(c) Joule/twy Sol. (d) (b) Non-chordates/ vd’ks#dh
(d) Second/lsdM a Antelope is young one of deer. (c) Reptile/ljhl`i
Sol. (a) Similarly, Chicks is young one of (d) Mammal/Lruik;h izk.kh
The SI unit of mass is Kilogram. Chicken. Sol. (a)
Similarly, The SI unit of force is Newton.

370
Best App for Govt. Jobs : Neonclasses (Download Now) Neon Publications
Earthworm is animal without notochord Sol. (a) (b)Depression /fMizs’ku
comes under Non-chordates species. The job of the tailar is to sew the dress. (c)Malaria /eysfj;k
Similarly, Fish is animal with backbone Similarly, the job of a cobbler is to make (d)Dengue /Msx a w
comes under chordats species. shoes. Sol. (b)
17. Based on gender /fyax ds vuqlkj 22. Based on Computer Typhoid, Malaria and dengue are a kind
18. ‘Stag’ is related to ‘Doe' in the same way 23. 'Mouse’ is related to ‘Computer’ in the of stomach infection while Depression is
as ‘Bull' is related to same way as ‘Flash’ is related to a state of mind.
'fgj.k','fgj.kh' ls mlh çdkj lacafèkr gS tSls ‘________’. 6. (a) Africa /vQzhdk
'cSy' fdlls lacafèkr gS\ 'ekml' dk lacaèk 'daI;wVj' ls mlh çdkj gS (b) Europe /;wjksi
(a) Cow/xk; (b) Mare/?kksM+h tSls '¶yS'k' dk lacaèk '&&&&&&&&' ls gSA (c) North America /mÙkjh vesfjdk
(c) Vixen/cSy (d) Ox/ykseM+h (a) Camera/dSejk (b) Sparkle/ped (d) South Africa /nf{k.kh vQzhdk
Sol. (a) (c) Card/dkMZ (d) Printer/fizaVj Sol. (d)
The feminine gender of Stag is Doe. Sol. (a) Africa, Europe and North America are
Similarly, the feminine gender of Bull is Mouse is a device used for ponting and continents while South Africa is a
Cow. operating computer. country.
18. Part and whole relationship/ Similarly, Flash is a device used for 7. (a) Larva/ykokZ
19. ‘Bird‘ is related to ‘Wings’ in the same profucting brief burst of light in camera. (b) Foal /?kksM+s dk cPpk
way as 'Fish’ is related to ‘ ...... ‘. 23. Simple name on the basis (c) Nymph /dkWdjksp dk cPpk
'i{kh' dk lacèa k 'ia[k' ls mlh çdkj gS tSls 24. ‘Month’ is related to ‘March’ in the same (d) Stag /ckjgflaxk
'eNyh' dk lacaèk '------' ls gSA way as ‘Season’ is related to ‘ ..........
Sol. (d)
(a) Belly/isV (b) Scales/’kYd ftl çdkj 'eghuk' dk lacaèk 'ekpZ' ls gS Larva, foal and nymph are all infants of
(c) Gills/xyQM+k (d) Fins/ehuia[k mlh çdkj ls 'ekSle' dk lacaèk ------------ ls gSA animals while Stag is a male deer.
Sol. (d) (a) Year/o"kZ (b) Festival/R;kSgkj 8. (a) Ladakh/yn~nk[k
Bird needs wings for movement from (c) Spring/olar (d) Week/lIrkg (b) Shimla/f’keyk
one place to another. Sol. (c) (c) Dehradun/nsgjknwu
Similarly, fish needs fins for its March is a Month. (d) Gangtok/xSaxVksd
movement. Similarly, Spring is a Season.
Sol. (a)
19. Tool and action MIX PRACTICE QUESTIONS
Except Ladakh, all other options
20. ‘Oven’ is related to ‘Baking’ in the 1. (a) Kelvin /dsfYou
represent capital of different states.
similar way as ‘knife’ is related to (b) Celsius/lsfYl;l 9. (a) Diabetes/e/kqeg s
‘________’. (c) Pascal/ikLdy (b) Cancer/dSalj
'vksou' 'csfdax' ls mlh rjg lacafèkr gS tSls (d) Fahrenheit/QkjsugkbV (c) Asthma /vLFkek
'pkdw' dk lacaèk '&&&&&&&&' ls gSA Sol. (c) Kelvin, Celsius and Fahrenheit are (d) Bone Marrow/es:naM
(a) Grating (b) Painting units of temperature measurement.
Sol. (d)
(c) Chopping (d) Making While pascal is the unit to measure
Bone marrow is the spongy tissue inside
Sol. (c) pressure.
some of your bones. The rest of the
Cake is baked in an oven. 2. (a) Decibel meter/MsflcyehVj
three options are the name of diseases.
Similarly, things is chopping by knife. (b) Udometer/mMksehVj 10. (a) Litre/yhVj
20. Worker and working place (c) Ombrometer/vkWEcksehVj (b) Distance/nwjh
21. ‘Scientist’ is related to ‘Laboratory’ in (d) Pluviometer/IyqfovksehVj (c) Minute /feuV
the same way as ‘Teacher’ is related to Sol. (a) Except for the decibel meter, all
‘______’. (d) Kilogram/fdyksxzke
other instruments are used to measure
'oSKkfud' dk lacaèk 'ç;ksx'kkyk' ls mlh Sol. (b)
precipitation. Decibel meter is used to
çdkj gS tSls 'f'k{kd' dk lacaèk Except ‘Distance’ all other options are
measure noise.
‘S.I. units’.
'&&&&&&&&' ls gSA 3. (a) Biomass/ck;ksekl
11. (a) Trumpet/rqjgh
(a) Job/ ukSdjh (b) Coal /dks;yk
(b) Xylophone/flykQu
(b) Research/ vuqlaèkku (c) Natural Gas/izd`frd xSl
(c) Saxophone/lsDlksQksu
(c) Students/ Nk= (d) Nuclear Fuel /ijek.kq xSl
(d) Clarinet/’kgukbZ
(d) School/ fo|ky; Sol. (d)
Sol. (b)
Sol. (d) Biomass, coal & natural, all these are
The xylophone is a musical instrument
Scientist works in a laboratory. directly obtained from nature except
in the percussion family that consists of
Similarly, Teacher works in a School. nuclear fuel.
wooden bars struck by mallets. All
21. Worker and action 4. (a) Den/ekan
others are Wind instruments.
22. ‘Tailor’ is related to ‘Dress’ in the same (b) Burrow /ekan 12. (a) Pancreas/vXuk’k;
way as ‘Cobbler’ is related to ‘________’. (c) Stable /fLFkj (b) Liver/;d`r
'ntÊ' 'iks'kkd' ls mlh rjg lacfa èkr gS tSls (d)Barrack /lsukokl (c) Renin/jsfuu
'eksph' ‘----------’ ls lacafèkr gSA Sol. (d) (d) Kidney/xqnkZ
(a) Shoes/twrs Barrack is a place where soldiers live.
Sol. (c)
(b) Cycle/lkbZfdy However, den, burrow and stable are
All other options except renin are
(c) Furniture/QuhZpj the residing place of an animal.
organs, Whereas, Renin is an enzyme
(d) Building/bekjr 5. (a) Typhoid/Vk;QkbM
secreted by the kidney & a part of the

371
Download Free PDFs & e-Books from Neon Classes App web. : www.neonclasses.com
physiological system that regulates Duster, Chalk & blackboard are different Except option (c) all pairs belong to the
blood pressure. types of stationery items. same category. While in option C, frog is
13. (a) Meanness/nfjnzrk 21. (a) Thyroxine/FkkbZjksfDlu amphibian and Crocodile belongs to the
(b) Prosperity/le`f} (b) Insulin/balqfyu reptile family.
(c) Lack /vHkko (c) Adrenalin/,Mªsufs yu 29. (a) Intermission/e/;karj
(d) Pauperism/daxkyh (d) Hormone/gkWeksu Z (b) Conclusion/lekiu
Sol. (b) Sol. (d) (c) Valediction/fonkbZ
All the other options mean negative Thyroxine, Insulin and Adrenaline are (d) Termination/lekfIr
habit, behavior or situation in terms of different types of hormones. Sol. (a)
money and amenities. Whereas, 22. (a) Political Science/jktuhfrd foKku Intermission means Pause/Break
Prosperity means ‘successful in material (b) Biology/tho foKku Conclusion means End/Finish
terms, flourishing financially’. (c) History/bfrgkl Valediction means Farewell/ goodbye
14. (a) Truth/lp (d) Sociology/lekt’kkL= Termination means End/Terminate
(b) Non-Violence/vfgalk Sol. (b) Except option (a) all have the same
(c) Hatred/?k`.kk Except Biology, all the others belong to meaning.
(d) Peace/’kkafr the study of social science. 30. (a) Jealous/bZ";kZ
Sol. (c) 23. (a) Cells/dksf’kdk,a (b) Weep/jksuk
Only ‘Hatred’ is a negative word (action) (b) Atoms/ijek.kq (c) Sad/mnkl
while, truth, peace and nonviolence are (c) Tissues/mÙkd (d) Angry/Øksf/kr
positive words. (d) Organs/vax Sol. (b)
15. (a) Gladsome/lq[k Sol. (b) All the options except B are feelings
(b) Morose/mnkl Except Atoms, the other three belong to whereas Weep is an action.
(c) Joyous/vkuafnr our body. 31. (a) Guava/ve:n
(d) Cheerful/galeq[k 24. (a) Pedology/Hkwfe foKku (b) Pomegranate/vukj
Sol. (b) (b) Seismology/Hkwdai foKku (c) Cucumber/[khjk
Except Morose, all the other words are (c) Terminology/’kCnkoyh (d) Papaya/iihrk
synonyms. (d) Cardiology/gn~;jksx foKku Sol. (c)
16. (a) Authentic/fo’oluh; Sol. (c) Except option (c) all other are similar to
(b) Genuine/okLrfod Except Terminology, the other three are each other.
(c) Spurious/tkyh the study of different types of science 32. (a) Humorous/gkL;iw.kZ
(d) Bonafide/izkef.kd related to the body. (b) Comical/etkfd;k
Sol. (c) 25. (a) Germany/teZuh (c) Hilarious/izlUufpr
Except Spurious, all the other words are (b) Singapore/flaxkiqj (d) Gagster/el[kjk
synonyms. (c) Austria/vkfLVª;k Sol. (d)
17. (a) Stable/?kqM+lky (d) Italy/bVyh Except option (d) all other are similar to
(b) Court/dksVZ Sol. (b) each other.
(c) Den/xqQk Except Singapore, all the others belong 33. (a) Successor/mÙkjkf/kdkjh
(d) Igloo/fgedqVh to Europe. (b) Persistence/n`<+rk
Sol. (b) 26. (a) Dogri : Jammu and Kashmir/Mksxjh% (c) Patience//khjt
Except, Court, all the others are places to tEew vkSj d’ehj (d) Stability/fLFkjrk
reside. (b) Tamil: Kerala/rfey% dsjyk Sol. (a)
18. (a) Steam/Hkki (c) Bori: Arunachal Pradesh/cksjh% Except option (a) all other are similar to
(b) Grill/fxzy v:.kkpy izns’k each other.
(c) Fire/vkx (d) Konkani: Goa/dksda .kh % xksok 34. (a) Thorium/Fkksfj;e
(d) Roast/Hkwuuk Sol. (b) (b) Uranium/;wjfs u;e
Sol. (c) Except Tamil : Kerala, all others are (c) Radium/jsfM;e
Fire is used for Grill, Steam and Roast. states and their languages. Malayalam is (d) Sodium/lksfM;e
So, Fire is different from others. commonly spoken in Kerala. Sol. (d)
19. (a) Car/dkj 27. (a) Warrior: Sword/ ;ks}k %ryokj Except option (d) all other are similar to
(b) Jeep/thi (b) Mason: Knife/etnwj% pkdw each other.
(c) Truck/Vªd (c) Author: Pen/ys[kd% isu 35. (a) Groundnut/ewx a Qyh
(d) Ship/leqnzh tgkt (d) Carpenter: Saw/c<+bZ % vkjh (b) Mustard/ljlksa
Sol. (d) Sol. (b) (c) Cumin/thjk
Ship runs/ moves in water, others on Warrior uses sword, Author uses Pen, (d) Fennel/lksQa
the roads. Carpenter uses Saw But Mason does not Sol. (a)
20. (a) Blackboard/CySdcksMZ use Knife as a professional tool. Except option (a) all other are similar to
(b) Stationery/ys[ku lkexzh 28. (a) Parrot/rksrk & Eagle/ x:M each other.
(c) Duster /MLVj (b) Bear/Hkkyw & Deer/ fgju 36. (a) Hexagon/"kV~dks.k
(d) Chalk /pkWd (c) Frog/esa<+d & Crocodile/ exjePN (b) Ellipse/nh?kZo`r
Sol. (b) (d) Snake/lkai & Lizard/ fNidyh (c) Parallelogram/lekaukrj prqHkqZt
Sol. (c) (d) Rhombus/leprqHkZqt

372
Best App for Govt. Jobs : Neonclasses (Download Now) Neon Publications
Sol. (b) In other pairs the first word is the cause fju dks NksM+dj] nwljs lHkh ugkus ¼’kkSpky;½
Except, ellipse, all other are dynamic to happen the event given in second esa iz;ksx gksus okys lkcqu tcfd fju diM+s
shape. word. /kksus esa iz;ksx gksrk gSA
37. (a) Camouflage/Nykoj.k vU; ;qXeksa esa igyk 'kCn nwljs 'kCn esa gksus 50. (a) SMS/,l-,e-,l
(b) Mask/lekiu okyh ?kVuk dk dkj.k gSA (b) Speed post/LihM iksLV
(c) Conceal/fNikuk 44. (a) Head : Brain/flj % efLr"d (c) Letter/i=
(d) Horde/>q.M+ (b) Hand : Wrist/gkFk % dykbZ (d) Money order /euh vkWMZj
Sol. (c) (c) Arm : Shoulder/Hkqtk % da/kk Sol. (a)
Except Horde, all others are synonyms (d) Feet : Ankle/iSj % V[kuk SMS (Short Messaging Service) is sent
of each other. Sol. (a) through a mobile phone, while all others
>qM
a dks NksM+dj vU; lHkh ,d nwljs ds The pair Head: Brain is different from are sent through post office.
lekukFkhZ 'kCn gSaA others. SMS eksckby ls tcfd nwljs lHkh Mkd[kkuk
38. (a) Krishna/d`".kk ;qXe flj% efLr"d vU; ls fHkUu gSA ls Hksts tkrs gSA
(b) Mahanadi/egkunh 45. (a) Newspaper/lekpkj&i= 51. (a) Perimeter/ifjeki
(c) Tapti/rkfIr (b) Journal/nSfud if=dk (b) Area/{ks=
(d) Godavari/xksnkojh (c) Novel/miU;kl (c) Density/?kuRo
Sol. (c) (d) Magazine/if=dk (d) Distance /nwjh
Tapti river flows westward and other Sol. (c) Sol. (c)
three rivers are eastward rivers. Novel is different from others. Density is different from the other three
rkIrh unh if’pe dh vksj cgrh gS vkSj vU; miU;kl vU; ls fHkUu gSA words.
rhu ufn;k¡ iwoZ dh vksj cgusa okyh ufn;k¡ gSA 46. (a) M.L.A./,e-,y-, *?kuRo* nwljs lHkh ls vyx gSA
39. (a) Ludo/ywMks (b) President/jk"Vªifr 52. (a) Berne/cuZ
(b) Chess/'krjat (c) Prime Minister /iz/kkuea=h (b) Geneva/ftusok
(c) Polo/iksyks (d) Minister/ea=h (c) Zurich/T;wfjd
(d) Carrom/dSje Sol. (b) (d) Bristol /fczLVy
Sol. (c) President is the executive head of the Sol. (d)Except Bristol, all others are cities of
Except Polo all given options are indoor nation. President is not elected directly Switzerland. Berne is the capital of
sports. by the people in India. Switzerland.
iksyks ds vykok lHkh indoor games gSaA jk"Vªifr jk"Vª dk 'kh"kZ vf/kdkjh gksrk gSA fczLVy dks NksM+dj] nwljs lHkh fLoV~tjySaM
40. (a) Almond/cknke jk"Vªifr izR;{k :i ls ns’k dh turk }kjk ds 'kgj gSA cuZ] fLoV~tjySaM dh jkt/kkuh gSA
(b) Dates/[ktwj ugha pquk tkrk gSA 53. (a) Obstetrician/izlwfr foKku
(c) Walnut/v[kjksV 47. (a) Water: Tap/ikuh% uy (b) Podiatrist/iksfM;kfVªLV
(d) Cashew/dktw (b) Oxygen: Life/vkWDlhtu% thou (c) Pulmonologist/’kjhj jpuk foKku
Sol. (b) (c) Power: Machine/{kerk % e’khu (d) Prosthetist /izksLFksfVLV
Except Dates others are dry fruits. (d) Oil : Lamp/rsy % ySEi Sol. (d)
[ktwj ds vykok vU; lHkh lw[ks esos gSaA Sol. (a) Except prosthetist, all others are related
41. (a) Red/yky Except in Water: Tap, in all others the to real organs of mankind.
(b) White/lQsn first is necessary for the second to be izksLFksfVLV dks NksM+dj] nwljs lHkh ekuo ds
(c) Blue/uhyk active. okLrfod vax gSaA
(d) Yellow/ihyk ¼ikuh % uy½ dks NksM+dj] ckdh lHkh esa igyk 54. (a) Content/lkexzh
Sol. (b) nwljs dks pykus ds fy;s vko’;d gSA (b) Bibliography/xzaFk lwph
Except White, all are rainbow colors. 48. (a) Sparrow/xkSjs;k (c) Press/izsl
lQsn dks NksM+dj] ckdh lHkh banz/kuq"kh; jax (b) Kingfisher/eNyh idM+us okyk i{kh (d) Preface/izLrkouk
gSA (c) Crane/Øsu Sol. (c)
42. (a) Lemon/uhacw (d) Humming bird /gfEeax cMZ Press is different from the other three
(b) Carrot/xktj Sol. (a) words. Content, Preface and
(c) Beetroot/pqdanj Except sparrow, all others have long and Bibliography are parts of a book.
(d) Turnip/’kyte sharp back. *nkc* nwljs 'kCnksa ls vyx gSA fo"k; lwph
Sol. (a) xkSjs;k dks NksM+dj nwljs lHkh yEch vkSj izLrkouk] xzaFklwph ,d iqLrd ds Hkkx gSA
Lemon is different from others. uqdhyh pksap okys gSA 55. (a) Tetanus/fVVusl
uhcw dk Qy isM+ ij gksrk gS] ckdh lHkh dk 49. (a) Cinthol/flUFkksy (b) Pneumonia/fueksfu;k
feV~Vh ds vUnjA (b) Rin/fju (c) tuberculosis/V;wcjdqyksfll
43. (a) Clouds : Rain/ckny % o"kkZ (c) Hamam /geke (d) Hepatitis /gsiV s kbfVl
(b) Injury : Pain/?kko % nnZ (d) Pears /ih;lZ Sol. (d)
(c) Boiling : Evaporation/mcyuk % Sol. (b) Except hepatitis, all other diseases are
ok"ihdj.k Except Rin, all others are bathing (toilet) caused by bacteria. Hepatitis is caused
(d) Freezing : Cold /teuk % BaM+k soaps. Rin is a detergent bar used for by virus.
Sol. (d) cleaning cloths. gsisVkbfVl dks NksM+dj] nwljs lHkh jksx
thok.kq ls iuirs gSA gSisVkbfVl ok;jl ls
QSyrk gSA

373
Download Free PDFs & e-Books from Neon Classes App web. : www.neonclasses.com
56. (a) Tulip/V~;wfyi (b) Meat/ekal tM+ ikS/ks tehu ds vanj dk Hkkx gSA ruk]
(b) Lotus/dey (c) Meet/feyuk iÙkh rFkk Qwy] ikS/ksa ds ,d Hkkx gSA
(c) Marigold/xsna s dk Qwy (d) Beat /iNkM+uk 68. (a) Ladder/lh<+h
(d) Rose /xqykc Sol. (c) (b) Staircase/LVs;jdsl
Sol. (b) In the word MEET, the second and the (c) Bridge/lsrq
Lotus is different from all other flowers third letters are the same. (d) Escalator /Lokpkfyr lh<+h
as it is grown in muddy water. ‘MEET’ 'kCn esa nwljk vkSj rhljk v{kj Sol. (c)
dey] nwljs Qwyksa ls vyx gSA ;g dhpM+ esa leku gSA Bridge is different from the other three.
c<+rk gSA 63. (a) Teaching/f’k{k.k Except Bridge, all others are used for
57. (a) Hill Myna/fgy eSuk (b) Counselling/dkmalfyax vertical movement.
(b) House Sparrow/xkSjS;k (c) Instruction/vuqns’k lsrq vU; rhuksa ls fHkUu gSA lsrq dks NksM+dj
(c) Emerald Dove/iUuk dcwrj (d) Guidance /ekxZn’kZu lHkh m/okZ/kj esa gksrh gSA
(d) Imperial Eagle /’kkgh fx} Sol. (a) 69. (a) Perception/vuqHkwfr
Sol. (b) Teaching is different from the other (b) Discernment/izHksn
House sparrow is a common type of three words. (c) Penetration/Hksnu
bird. *f’k{k.k* nwljs 'kCnksa ls vyx gSA (d) Insinuation /vk{ksi
xkSjs;k ,d lkekU; i{kh gSA 64. (a) Japanese/tkikuh Sol. (d)
58. (a) Polaris/iksykfjl (b) Swiss/fLol Insinuation is different from the other
(b) Nike/ukbd (c) French/QzSap words.
(c) Crux/ØD’k (d) German /teZu Insinuate (Verb) means “to suggest
(d) Phoenix /QhfuDl Sol. (a) something bad about somebody/
Sol. (c) Japanese are Asians while all others are something in an unpleasant and indirect
Except Crux, all others are renowned Europeans. way.”
companies. tkikuh] ,f’k;u vkSj ckdh lHkh ;wjksfi;u gSAa Insinuation (Noun) means “the action of
Crux dks NksM+dj nwljh lHkh izfl} dEifu;ka 65. (a) Pound (Sterling)/ikamM insinuating something.”
gSA (b) Deutsche Mark/M~;w’k ekdZ vk{ksi nwljks 'kCnksa ls fHkUu gS] vk{ksi fdz;k
59. (a) Compress/nkc (c) Yen/;su dk vFkZ fdlh ds ckjs esa@;k fdlh pht ds
(b) Condense/la?kfur (d) Franc /QzSad ckjs esa lq>ko dk ladsr nsuk] vfiz; vkSj nwljs
(c) Concise/laf{kIr Sol. (c) rjg ls vk{ksi ¼laKk½ dk vFkZ gSA fdlh pht
(d) Convert /cnyuk Yen is the currency of Japan, an Asian fdz;k dh vkSj ladsr djukA
Sol. (d) country. Pound (Sterling), Deutsche 70. (a) Swimming/rSjuk
Except convert, all other words simply Mark and Franc are currencies of (b) Sailing/ty;k=k djuk
squeezing or reducing. European countries. (c) Diving/xksrk yxkuk
Convert dks NksM+dj] nwljs lHkh 'kCn ;su tkiku dh eqnzk gSA tks ,d ,f’k;u ns’k (d) Driving /okgu pykuk
fupksM+uk ;k de djuk crkrs gSA gSA iksaM ¼LVsjfyax½] MkW;psekdZ vkSj QzSad Sol. (d)
60. (a) Time : Seconds/le; % lsd.M ;wjksfi;u ns’kksa dh eqnzk;sa gSaA Except (Driving), all other activities are
(b) Pressure : Barometer/ncko%cSjksehVj 66. (a) Game - Coach/[ksy & dksp performed in water.
(c) Length : Metre/yackbZ % ehVj (b) Drama - Director/Mªkek & Mk;jsDVj okgu pykuk NksM+dj] vU; lHkh dk;Z ikuh esa
(d) Volume : Litre/vk;ru % yhVj (c) Counselling - Counsellor/lykg & fd;s tkrs gSaA
Sol. (b) lykgdkj 71. (a) Foreword/izkDdFku
Except in the pair Pressure Barometer, (d) Student- Teacher /fo|kFkhZ & v/;kid (b) Text/fo"k;&okD;
in all other pairs the second is the unit Sol. (d) (c) Index/lwpdkad
of the first Barometer is a scientific Except the pair Student Teacher, in all (d) Preface /iwoy
Z s[k
instrument used for measuring other pairs the activity and releated Sol. (c)
atmospheric pressure. terms are given. The work of coach is to Index is different from the others
ok;qnkc ekih dks NksM+dj nwljs lHkh tksM+ks esa impart training in games. Director because it is the list of subject- matter
nwljk igys dh bZdkbZ gSA cSjksehVj ,d directs drama. Counseller provides and references.
oSKkfud ;a= gSA tks ok;q ds nkc dks ekius esa counseling. lwpdkad nwljks ls vyx gS D;ksafd ;g
iz;ksx fd;k tkrk gSA fo|kFkhZ&f’k{kd ds lewg dks NksM+dj vU; fo"k;&lwph vkSj lanHkksZ dh lwph gSA
61. (a) Lieutenant General/ysf¶VusV a tujy lHkh fdz;k;sa muds dk;Z ls lacaf/kr gS] [ksy 72. (a) Wave/rajx
(b) Major/estj dks fl[kkus ds fy;s dksp] Mªkek esa funs’Z kd] (b) Current//kkjk
(c) Colonel/duZy vkSj lykgdkj lykg nsrk gSA (c) Tide/Tokj
(d) Commodore /deksMksj 67. (a) Stem/ruk (d) Storm /vk¡/kh
Sol. (d) (b) Leaf/iÙkh Sol. (c)
Lieutenant General, Major and Colonel (c) Flower/Qwy Except Tide, all other terms are related
are Commissioned ranks of the Indian (d) Root /tM+ to both air and water. But tide is a
Army. Sol. (d) regular rise and fall in the level of sea,
lsukifr] estj vkSj duZy lHkh Hkkjrh; QkSt Root is underground part of a plant. caused by the attraction of the moon
ds in gSA Stem leaf and flower are visible parts of and sun.
62. (a) Heat/xehZ a plant. Tokj dks NksM+dj] lHkh dk laca/k gok vkSj
ikuh ls gS] ysfdu Tokj ij leqnz dh lrg
374
Best App for Govt. Jobs : Neonclasses (Download Now) Neon Publications
ij fu;fer mrkj&p<+ko lw;Z vkSj pUnzek ds xqykch dks NksM+dj] nwljs lHkh banz/kuq"k ds jax
vkd"kZ.k ls gksrk gSA gSA
73. (a) Room/dejk 79. (a) Yellow Orange/ihyk&ukjaxh
(b) Chamber/dpgjh ;k d{k (b) Red Yellow/yky&ihyk
(c) Veranda/cjkenk (c) Red Orange/yky&ukjaxh
(d) Cabin /dsfcu (d) Yellow Green /ihyk&gjk
Sol. (c) Sol. (a)
Except Veranda, all other are Red, Green and Blue are called primary
surrounded by four walls. colours or basic colours.
cjkenk dks NksM+dj] ckdh lHkh pkj nhokjksa ls yky] gjk vkSj uhyk izkFkfed jax gSA
f?kjs gSaA yky $ gjk  ihyk jax
74. (a) Mouth Organ/ekamFk vkWxZu Red + Green  Yellow colour
(b) Electric Guitar/fo|qr fxVkj Except in option (a), in all others there is
(c) Keyboard/dh&cksMZ one primary colour.
(d) Sonata /lksukVk fodYi (a) dks NksM+dj] nwljs lHkh izkFkfed
Sol. (d) jax gSA
Except Sonata, all others are musical 80. (a) Hive /e/kqeD[khdk NÙkk
instruments. (b) Heifer/cNh;k
lksukVk dks NksM+dj ckdh lHkh laxhr dh (c) Byre/xkS’kkyk
oLrq;sa gSaA (d) Eyrie /Å¡pk ?kkslyk
75. (a) Wolf/HksfM+;k Sol. (b)
(b) Cat/fcYyh Except heifer, all others are places of
(c) Dog/dqÙkk living.
(d) Fox /ykseM+h 81. (a) Foal/?kksM+s dk cPpk
Sol. (b) (b) Howl/ph[k
Except Cat, all others belong to dog (c) Caw/ dkao & dkao
family. The members of cat family are (d) Chirp /dyjo
lions, tiger and leopard etc. Sol. (a)
fcYyh dks NksM+dj vU; lHkh dqÙks ds ifjokj Except foal, all other options are sounds
ls gSA fcYyh ds ifjokj ls laca/k j[kus okys by different animals.
tkuoj] 'ksj] ck?k rFkk rsna qvk vkfnA 82. (a) Tamil Nadu : South/rfeyukMq % nf{k.k
76. (a) Garden- Gardener/cxhpk&ekyh (b) Punjab : North /iatkc % mÙkj
(b) Song - Singer/xkuk&xk;d (c) Assam : North East/vle % mÙkj iwoZ
(c) Art - Artist/dyk&dykdkj (d) Kerala : North West /dsjyk % mÙkj
(d) Dance- Dancer / ukp&ukpus okyk@ if’pe
okyh Sol. (d)
Sol. (a) Except Kerala : North West, all other
One who works in garden is called states and their direction is right.
gardener. All others are Art forms. 83. (a) Intact : Unflawed/v{kr % vuQy
cxhps esa dk;Z djus okys dks ekyh dgrs gS] (b) Bright : Luminous /pedhyk %
vU; lHkh dyk ds {ks= ls gSaA pednkj
77. (a) Wool/Åu (c) Pure : Perfect/’kq} % fcYdqy lgh
(b) Feather/ij ¼ia[k½ (d) Entire : Partial /lai.w kZ % vkaf’kd
(c) Hair/cky Sol. (d)
(d) Grass /?kkl The given pair of words are synonyms of
Sol. (d) each other. However, in option (d), the
Except grass, all others a can be words are antonyms of each other.
obtained from animals and birds. Grass 84. (a) Assemble/bdV~Bk djuk
is a vegetation. (b) Construct/fuekZ.k
?kkl dks NksM+dj] vU; lHkh dks Ik’kq vkSj (c) Disband /m[kkM+uk
if{k;ksa ls izkIr dj ldrs gSA ?kkl ouLifr (d) Create/l`tu djuk
esa vkrh gSA Sol. (c)
78. (a) Violet/cSaxuh Except option (c), all three refers to
(b) Pink/xqykch joining things while disband refers to
(c) Blue/uhyk breaking up.
(d) Orange /ukjaxh
Sol. (b)
Except Pink, all other colors are parts of
a rainbow.

375

You might also like